Вы находитесь на странице: 1из 595

Upper Limb

1. A 43 year old man is stabbed outside a nightclub. He suffers a transection of his median nerve just as it leaves the
brachial plexus. Which of the following features is least likely to ensue?
A. Ulnar deviation of the wrist
B. Complete loss of wrist flexion
C. Loss of pronation
D. Loss of flexion at the thumb joint
E. Inability to oppose the thumb
Answer: B

Loss of the median nerve will result in loss of function of the flexor muscles. However, flexor carpi ulnaris will still function and
produce ulnar deviation and some residual wrist flexion. High median nerve lesions result in complete loss of flexion at the thumb
joint.

Median nerve

The median nerve is formed by the union of a lateral and medial root respectively from the lateral (C5,6,7) and medial (C8 and
T1) cords of the brachial plexus; the medial root passes anterior to the third part of the axillary artery. The nerve descends lateral
to the brachial artery, crosses to its medial side (usually passing anterior to the artery). It passes deep to the bicipital aponeurosis
and the median cubital vein at the elbow.
It passes between the two heads of the pronator teres muscle, and runs on the deep surface of flexor digitorum superficialis (within
its fascial sheath).
Near the wrist it becomes superficial between the tendons of flexor digitorum superficialis and flexor carpi radialis, deep to
palmaris longus tendon. It passes deep to the flexor retinaculum to enter the palm, but lies anterior to the long flexor tendons
within the carpal tunnel.

Branches
Region Branch
Upper arm No branches, although the nerve commonly communicates with the musculocutaneous nerve
Forearm Pronator teres
Flexor carpi radialis
Palmaris longus
Flexor digitorum superficialis
Flexor pollicis longus
Flexor digitorum profundus (only the radial half)
Distal Palmar cutaneous branch
forearm
Hand Motor supply (LOAF)
(Motor)
 Lateral 2 lumbricals
 Opponens pollicis
 Abductor pollicis brevis
 Flexor pollicis brevis

Hand  Over thumb and lateral 2 ½ fingers


(Sensory)  On the palmar aspect this projects proximally, on the dorsal aspect only the distal regions are innervated
with the radial nerve providing the more proximal cutaneous innervation.

Patterns of damage
Damage at wrist

 e.g. carpal tunnel syndrome


 paralysis and wasting of thenar eminence muscles and opponens pollicis (ape hand deformity)
 sensory loss to palmar aspect of lateral (radial) 2 ½ fingers

Damage at elbow, as above plus:

 unable to pronate forearm


1
 weak wrist flexion
 ulnar deviation of wrist

Anterior interosseous nerve (branch of median nerve)

 leaves just below the elbow


 results in loss of pronation of forearm and weakness of long flexors of thumb and index finger

2. A 24 year old man falls and sustains a fracture through his scaphoid bone. From which of the following areas does the
scaphoid derive the majority of its blood supply?
A. From its proximal medial border
B. From its proximal lateral border
C. From its proximal posterior surface
D. From the proximal end
E. From the distal end

Answer: E
The blood supply to the scaphoid enters from a small non articular surface near its distal end. Transverse fractures through the
scaphoid therefore carry a risk of non union.

Scaphoid bone

The scaphoid has a concave articular surface for the head of the capitate and at the edge of this is a crescentic surface for the
corresponding area on the lunate.
Proximally, it has a wide convex articular surface with the radius. It has a distally sited tubercle that can be palpated. The
remaining articular surface is to the lateral side of the tubercle. It faces laterally and is associated with the trapezium and trapezoid
bones.
The narrow strip between the radial and trapezial surfaces and the tubercle gives rise to the radial collateral carpal ligament. The
tubercle receives part of the flexor retinaculum. This area is the only part of the scaphoid that is available for the entry of blood
vessels. It is commonly fractured and avascular necrosis may result.

3. A 56 year old man requires long term parenteral nutrition and the decision is made to insert a PICC line for long term
venous access. This is inserted into the basilic vein at the region of the elbow. As the catheter is advanced, into which
venous structure is the tip of the catheter most likely to pass from the basilic vein?
A. Subclavian vein
B. Axillary vein
C. Posterior circumflex humeral vein
D. Cephalic vein
E. Superior vena cava
Answer: B
The basilic vein drains into the axillary vein and although PICC lines may end up in a variety of fascinating locations the axillary
vein is usually the commonest site following from the basilic. The posterior circumflex humeral vein is encountered prior to the
axillary vein. However, a PICC line is unlikely to enter this structure because of its angle of entry into the basilic vein.

Basilic vein

The basilic and cephalic veins both provide the main pathways of venous drainage for the arm and hand. It is continuous with the
palmar venous arch distally and the axillary vein proximally.

Path

 Originates on the medial side of the dorsal venous network of the hand, and passes up the forearm and arm.
 Most of its course is superficial.
 Near the region anterior to the cubital fossa the vein joins the cephalic vein.
 Midway up the humerus the basilic vein passes deep under the muscles.
 At the lower border of the teres major muscle, the anterior and posterior circumflex humeral veins feed into it.
 Joins the brachial veins to form the axillary vein.

2
Theme: Nerve injury

A. Ulnar nerve
B. Musculocutaneous nerve
C. Radial nerve
D. Median nerve
E. Axillary nerve
F. Intercostobrachial nerve

What is the most likely nerve injury for the scenario given? Each option may be used once, more than once or not at all.

4. A 23 year old man is involved in a fight outside a nightclub and sustains a laceration to his right arm. On examination
he has lost extension of the fingers in his right hand.

Answer: Radial nerve

The radial nerve supplies the extensor muscle group.

5. A 40 year old lady trips and falls through a glass door and sustains a severe laceration to her left arm. Amongst her
injuries it is noticed that she has lost the ability to adduct the fingers of her left hand.

Answer: Ulnar nerve

The interossei are supplied by the ulnar nerve.

6. A 28 year old rugby player injures his right humerus and on examination is noted to have a minor sensory deficit
overlying the point of deltoid insertion into the humerus.

Answer: Axillary nerve

This patch of skin is supplied by the axillary nerve

Brachial plexus
Origin Anterior rami of C5 to T1
Sections of the plexus  Roots, trunks, divisions, cords, branches
 Mnemonic:Real Teenagers Drink Cold Beer

Roots  Located in the posterior triangle


 Pass between scalenus anterior and medius

Trunks  Located posterior to middle third of clavicle


 Upper and middle trunks related superiorly to the subclavian artery
 Lower trunk passes over 1st rib posterior to the subclavian artery

Divisions Apex of axilla


Cords Related to axillary artery

7. A 21 year old man is stabbed in the antecubital fossa. A decision is made to surgically explore the wound. At operation
the surgeon dissects down onto the brachial artery. A nerve is identified medially, which nerve is it likely to be?
A. Radial
B. Recurrent branch of median
C. Anterior interosseous
D. Ulnar
3
E. Median
Answer: E

8. A man sustains a laceration between the base of the little finger and wrist. Several weeks after the injury there is loss
of thumb adduction power. Which nerve is most likely to have been injured?
A. Superficial ulnar nerve
B. Deep ulnar nerve
C. Median nerve
D. Radial nerve
E. Recurrent branch of median nerve
Answer: B

Ulnar nerve injury at wrist

Branches of the ulnar nerve in the wrist and hand


At the wrist the ulnar nerve divides into superficial and deep branches. The superficial branch lies deep to the palmaris brevis. It
divides into two; to produce digital nerves, which innervate the skin of the medial third of the palm and the palmar surface of one
and a half fingers.
The deep branch arises from the nerve on the flexor retinaculum lateral to the pisiform bone. It passes posteriorly between the
abductor and short flexor of the little finger supplying them, and supplying and piercing the opponens digiti minimi near its origin
from the flexor retinaculum, turns laterally over the distal surface of the Hook of the Hamate bone. It eventually passes between
the two heads of adductor pollicis with the deep palmar arch and ends in the first dorsal interosseous muscle. In the palm the deep
branch also innervates the lumbricals and interosseous muscles.

9. A 25 year old man is stabbed in the upper arm. The brachial artery is lacerated at the level of the proximal humerus,
and is being repaired. A nerve lying immediately lateral to the brachial artery is also lacerated. Which of the following
is the nerve most likely to be?
A. Ulnar nerve
B. Median nerve
C. Radial nerve
D. Intercostobrachial nerve
E. Axillary nerve
Answer: B
The brachial artery begins at the lower border of teres major and terminates in the cubital fossa by branching into the radial and
ulnar arteries. In the upper arm the median nerve lies closest to it in the lateral position. In the cubital fossa it lies medial to it.

Brachial artery

The brachial artery begins at the lower border of teres major as a continuation of the axillary artery. It terminates in the cubital
fossa at the level of the neck of the radius by dividing into the radial and ulnar arteries.
Relations
Posterior relations include the long head of triceps with the radial nerve and profunda vessels intervening. Anteriorly it is
overlapped by the medial border of biceps.
It is crossed by the median nerve in the middle of the arm.
In the cubital fossa it is separated from the median cubital vein by the bicipital aponeurosis.
The basilic vein is in contact at the most proximal aspect of the cubital fossa and lies medially.

10. What is the course of the median nerve relative to the brachial artery in the upper arm?
A. Medial to anterior to lateral
B. Lateral to posterior to medial
C. Medial to posterior to lateral
D. Medial to anterior to medial
E. Lateral to anterior to medial
Answer: E
Relations of median nerve to the brachial artery:
Lateral -> Anterior -> Medial
4
The median nerve descends lateral to the brachial artery, it usually passes anterior to the artery to lie on its medial side. It passes
deep to the bicipital aponeurosis and the median cubital vein at the elbow. It enters the forearm between the two heads of the
pronator teres muscle.

A 22 year old falls over and lands on a shard of glass. It penetrates the palmar aspect of his hand, immediately lateral to
the pisiform bone. Which of the following structures is most likely to be injured?

A. Palmar cutaneous branch of the median nerve


B. Lateral tendons of flexor digitorum superficialis
C. Ulnar artery
D. Flexor carpi radialis tendons
E. Lateral tendons of flexor digitorum profundus
Answer: C
The ulnar nerve and artery are at most immediate risk in this injury.

Hand

Anatomy of the hand


Bones  8 Carpal bones
 5 Metacarpals
 14 phalanges

Intrinsic Muscles 7 Interossei - Supplied by ulnar nerve

 3 palmar-adduct fingers
 4 dorsal- abduct fingers

Intrinsic muscles Lumbricals

 Flex MCPJ and extend the IPJ.


 Origin deep flexor tendon and insertion dorsal extensor hood mechanism.
 Innervation: 1st and 2nd- median nerve, 3rd and 4th- deep branch of the ulnar nerve.

Thenar eminence  Abductor pollicis brevis


 Opponens pollicis
 Flexor pollicis brevis

Hypothenar eminence  Opponens digiti minimi


 Flexor digiti minimi brevis
 Abductor digiti minimi

11. A motorcyclist is involved in a road traffic accident. He suffers a complex humeral shaft fracture which is plated. Post
operatively he complains of an inability to extend his fingers. Which of the following structures is most likely to have
been injured?
A. Ulnar nerve
B. Radial nerve
C. Median nerve
D. Axillary nerve
E. None of the above
Answer: B

5
Mnemonic for radial nerve muscles: BEST: B rachioradialis; E xtensors; S upinator; T riceps. The radial nerve is responsible for
innervation of the extensor compartment of the forearm.
Radial nerve

Continuation of posterior cord of the brachial plexus (root values C5 to T1)

Path

 In the axilla: lies posterior to the axillary artery on subscapularis, latissimus dorsi and teres major.
 Enters the arm between the brachial artery and the long head of triceps (medial to humerus).
 Spirals around the posterior surface of the humerus in the groove for the radial nerve.
 At the distal third of the lateral border of the humerus it then pierces the intermuscular septum and descends in front of
the lateral epicondyle.
 At the lateral epicondyle it lies deeply between brachialis and brachioradialis where it then divides into a superficial and
deep terminal branch.
 Deep branch crosses the supinator to become the posterior interosseous nerve.

Regions innervated

Motor (main nerve)  Triceps


 Anconeus
 Brachioradialis
 Extensor carpi radialis

Motor (posterior  Extensor carpi ulnaris


interosseous branch  Extensor digitorum
 Extensor indicis
 Extensor digiti minimi
 Extensor pollicis longus and brevis
 Abductor pollicis longus

Sensory The area of skin supplying the proximal phalanges on the dorsal aspect of the hand is supplied by the
radial nerve (this does not apply to the little finger and part of the ring finger)

Muscular innervation and effect of denervation


Anatomical Muscle affected Effect of paralysis
location
Shoulder Long head of triceps Minor effects on shoulder stability in abduction
Arm Triceps Loss of elbow extension
Forearm Supinator Weakening of supination of prone hand and elbow flexion in mid
Brachioradialis prone position
Extensor carpi radialis longus and
brevis

12. A 48 year old lady is undergoing an axillary node clearance for breast cancer. Which of the structures listed below are
most likely to be encountered during the axillary dissection?
A. Cords of the brachial plexus
B. Thoracodorsal trunk
C. Internal mammary artery
D. Thoracoacromial artery
E. None of the above
Answer: B
Beware of damaging the thoracodorsal trunk if a latissimus dorsi flap reconstruction is planned.

The thoracodorsal trunk runs through the nodes in the axilla. If injured it may compromise the function and blood supply to
latissimus dorsi, which is significant if it is to be used as a flap for a reconstructive procedure.

Axilla

Boundaries of the axilla


6
Medially Chest wall and Serratus anterior
Laterally Humeral head
Floor Subscapularis
Anterior aspect Lateral border of Pectoralis major
Fascia Clavipectoral fascia

Content:
Long thoracic nerve (of Bell) Derived from C5-C7 and passes behind the brachial plexus to enter the axilla. It lies on the
medial chest wall and supplies serratus anterior. Its location puts it at risk during axillary surgery
and damage will lead to winging of the scapula.
Thoracodorsal nerve and Innervate and vascularise latissimus dorsi.
thoracodorsal trunk
Axillary vein Lies at the apex of the axilla, it is the continuation of the basilic vein. Becomes the subclavian
vein at the outer border of the first rib.
Intercostobrachial nerves Traverse the axillary lymph nodes and are often divided during axillary surgery. They provide
cutaneous sensation to the axillary skin.
Lymph nodes The axilla is the main site of lymphatic drainage for the breast.

13. 53 year old lady is recovering following a difficult mastectomy and axillary nodal clearance for carcinoma of the
breast. She complains of shoulder pain and on examination has obvious winging of the scapula. Loss of innervation to
which of the following is the most likely underlying cause?
A. Latissimus dorsi
B. Serratus anterior
C. Pectoralis minor
D. Pectoralis major
E. Rhomboids
Answer: B

Winging of the scapula is most commonly the result of long thoracic nerve injury or dysfunction. Iatrogenic damage during the
course of the difficult axillary dissection is the most likely cause in this scenario. Damage to the rhomboids may produce winging
of the scapula but would be rare in the scenario given.

Long thoracic nerve

 Derived from ventral rami of C5, C6, and C7 (close to their emergence from intervertebral foramina)
 It runs downward and passes either anterior or posterior to the middle scalene muscle
 It reaches upper tip of serratus anterior muscle and descends on outer surface of this muscle, giving branches into it
 Winging of Scapula occurs in long thoracic nerve injury (most common) or from spinal accessory nerve injury (which
denervates the trapezius) or a dorsal scapular nerve injury

14. A 23 year old man falls and slips at a nightclub. A shard of glass penetrates the skin at the level of the medial
epicondyle, which of the following sequelae is least likely to occur?
A. Atrophy of the first dorsal interosseous muscle
B. Difficulty in abduction of the the 2nd, 3rd, 4th and 5th fingers
C. Claw like appearance of the hand
D. Loss of sensation on the anterior aspect of the 5th finger
E. Partial denervation of flexor digitorum profundus
Answer: C
Injury to the ulnar nerve in the mid to distal forearm will typically produce a claw hand. This consists of flexion of the 4th and 5th
interphalangeal joints and extension of the metacarpophalangeal joints. The effects are potentiated when flexor digitorum
profundus is not affected, and the clawing is more pronounced.More proximally sited ulnar nerve lesions produce a milder clinical
picture owing to the simultaneous paralysis of flexor digitorum profundus (ulnar half). This is the 'ulnar paradox', due to the more
proximal level of transection the hand will typically not have a claw like appearance that may be seen following a more distal
injury. The first dorsal interosseous muscle will be affected as it is supplied by the ulnar nerve.

Ulnar nerve

Origin:C8, T1
Supplies (no muscles in the upper arm)
 Flexor carpi ulnaris
 Flexor digitorum profundus
7
 Flexor digiti minimi
 Abductor digiti minimi
 Opponens digiti minimi
 Adductor pollicis
 Interossei muscle
 Third and fourth lumbricals
 Palmaris brevis

Path
 Posteromedial aspect of ulna to flexor compartment of forearm, then along the ulnar. Passes beneath the flexor carpi
ulnaris muscle, then superficially through the flexor retinaculum into the palm of the hand.

Branch Supplies
Articular branch Flexor carpi ulnaris
Medial half of the flexor digitorum profundus
Palmar cutaneous branch (Arises near the middle of the Skin on the medial part of the palm
forearm)
Dorsal cutaneous branch Dorsal surface of the medial part of the hand
Superficial branch Cutaneous fibres to the anterior surfaces of the medial one and one-
half digits
Deep branch Hypothenar muscles
All the interosseous muscles
Third and fourth lumbricals
Adductor pollicis
Medial head of the flexor pollicis brevis

Effects of injury
Damage at the wrist  Wasting and paralysis of intrinsic hand muscles (claw hand)
 Wasting and paralysis of hypothenar muscles
 Loss of sensation medial 1 and half fingers

Damage at the elbow  Radial deviation of the wrist


 Clawing less in 3rd and 4th digits

15. A 43 year old lady is due to undergo an axillary node clearance as part of treatment for carcinoma of the breast.
Which of the following fascial layers will be divided during the surgical approach to the axilla?
A. Sibsons fascia
B. Pre tracheal fascia
C. Waldayers fascia
D. Clavipectoral fascia
E. None of the above
Answer: D
The clavipectoral fascia is situated under the clavicular portion of pectoralis major. It protects both the axillary vessels and nodes.
During an axillary node clearance for breast cancer the clavipectoral fascia is incised and this allows access to the nodal stations.
The nodal stations are; level 1 nodes inferior to pectoralis minor, level 2 lie behind it and level 3 above it. During a Patey
Mastectomy surgeons divide pectoralis minor to gain access to level 3 nodes. The use of sentinel node biopsy (and stronger
assistants!) have made this procedure far less common.

16. A 23 year old climber falls and fractures his humerus. The surgeons decide upon a posterior approach to the middle
third of the bone. Which of the following nerves is at greatest risk in this approach?
A. Ulnar
B. Antebrachial
C. Musculocutaneous
D. Radial
E. Intercostobrachial
Answer: D
8
The radial nerve wraps around the humerus and may be injured during a posterior approach. An IM nail may be preferred as it
avoids the complex dissection needed for direct bone exposure.

Theme: Nerve injury

A. Median nerve
B. Ulnar nerve
C. Radial nerve
D. Posterior interosseous nerve
E. Anterior interosseous nerve
F. Musculocutaneous nerve
G. Axillary nerve
H. Brachial Trunks C5-6
I. Brachial trunks C6-7
J. Brachial Trunks C8-T1

Please select the most likely lesion site for each scenario. Each option may be used once, more than once or not at all.

17. A 42 year old teacher is admitted with a fall. An x-ray confirms a fracture of the surgical neck of the humerus. Which
nerve is at risk?
Answer: Axillary nerve

The Axillary nerve winds around the bone at the neck of the humerus. The axillary nerve is also at risk during shoulder
dislocation.
18. A 32 year old window cleaner is admitted after falling off the roof. He reports that he had slipped off the top of the
roof and was able to cling onto the gutter for a few seconds. The patient has Horner's syndrome.

Answer: Brachial
Trunks C8-T1

The patient has a Klumpke's paralysis involving brachial trunks C8-T1. Classically there is weakness of the hand intrinsic
muscles. Involvement of T1 may cause a Horner's syndrome. It occurs as a result of traction injuries or during delivery.
19. A 32 year old rugby player is hit hard on the shoulder during a rough tackle. Clinically his arm is hanging loose on the
side. It is pronated and medially rotated.

Answer: Brachial
Trunks C5-6
The patient has an Erb's palsy involving brachial trunks C5-6.

20. Which of the following nerves is responsible for innervation of the triceps muscle?
A. Radial
B. Ulnar
C. Axillary
D. Median
E. None of the above
Answer: A
To remember nerve roots and their relexes:
1-2 Ankle (S1-S2)
3-4 Knee (L3-L4)
5-6 Biceps (C5-C6)
7-8 Triceps (C7-C8)

The radial nerve innervates all three heads of triceps, with a separate branch to each head.

Triceps

Origin  Long head- infraglenoid tubercle of the scapula.


 Lateral head- dorsal surface of the humerus, lateral and proximal to the groove of the radial nerve
 Medial head- posterior surface of the humerus on the inferomedial side of the radial groove and both of
the intermuscular septae

Insertion  Olecranon process of the ulna. Here the olecranon bursa is between the triceps tendon and olecranon.
9
 Some fibres insert to the deep fascia of the forearm, posterior capsule of the elbow (preventing the
capsule from being trapped between olecranon and olecranon fossa during extension)

Innervation Radial nerve


Blood Profunda brachii artery
supply
Action Elbow extension. The long head can adduct the humerus and and extend it from a flexed position
Relations The radial nerve and profunda brachii vessels lie between the lateral and medial heads

21. Which of the following muscles inserts onto the lesser tuberostiy of the the humerus?
A. Subscapularis
B. Deltoid
C. Supraspinatus
D. Teres minor
E. Infraspinatus

Answer: A

With the exception of subscapularis which inserts into the lesser tuberosity, the muscles of the rotator cuff insert into the greater
tuberosity.

Shoulder joint
 Shallow synovial ball and socket type of joint.
 It is an inherently unstable joint, but is capable to a wide range of movement.
 Stability is provided by muscles of the rotator cuff that pass from the scapula to insert in the greater tuberosity (all except
sub scapularis-lesser tuberosity).

Glenoid labrum
 Fibrocartilaginous rim attached to the free edge of the glenoid cavity
 Tendon of the long head of biceps arises from within the joint from the supraglenoid tubercle, and is fused at this point to
the labrum.
 The long head of triceps attaches to the infraglenoid tubercle

Fibrous capsule
 Attaches to the scapula external to the glenoid labrum and to the labrum itself (postero-superiorly)
 Attaches to the humerus at the level of the anatomical neck superiorly and the surgical neck inferiorly
 Anteriorly the capsule is in contact with the tendon of subscapularis, superiorly with the supraspinatus tendon, and
posteriorly with the tendons of infraspinatus and teres minor. All these blend with the capsule towards their insertion.
 Two defects in the fibrous capsule; superiorly for the tendon of biceps. Anteriorly there is a defect beneath the
subscapularis tendon.
 The inferior extension of the capsule is closely related to the axillary nerve at the surgical neck and this nerve is at risk in
anteroinferior dislocations. It also means that proximally sited osteomyelitis may progress to septic arthritis.

Movements and muscles


Flexion Anterior part of deltoid
Pectoralis major
Biceps
Coracobrachialis
Extension Posterior deltoid
Teres major
Latissimus dorsi
Adduction Pectoralis major
Latissimus dorsi
Teres major
Coracobrachialis
Abduction Mid deltoid
Supraspinatus
Medial rotation Subscapularis
Anterior deltoid
Teres major
10
Latissimus dorsi
Lateral rotation Posterior deltoid
Infraspinatus
Teres minor

Important anatomical relations


Anteriorly Brachial plexus
Axillary artery and vein
Posterior Suprascapular nerve
Suprascapular vessels
Inferior Axillary nerve
Circumflex humeral vessels

22. Which of the following nerves is not contained within the posterior triangle of the neck?
A. Accessory nerve
B. Phrenic nerve
C. Greater auricular nerve
D. Ansa cervicalis
E. Lesser occiptal nerve
Answer: D

Ansa cervicalis is a content of the anterior triangle of the neck.

Posterior triangle of the neck

Boundaries
Apex Sternocleidomastoid and the Trapezius muscles at the Occipital bone
Anterior Posterior border of the Sternocleidomastoid
Posterior Anterior border of the Trapezius
Base Middle third of the clavicle

Contents
Nerves  Accessory nerve
 Phrenic nerve
 Three trunks of the brachial plexus
 Branches of the cervical plexus: Supraclavicular nerve, transverse cervical nerve, great auricular nerve,
lesser occipital nerve

Vessels  External jugular vein


 Subclavian artery

Muscles  Inferior belly of omohyoid


 Scalene

Lymph  Supraclavicular
nodes  Occipital

23. A 73 year old lady suffers a fracture at the surgical neck of the humerus. The decision is made to operate. There are
difficulties in reducing the fracture and a vessel lying posterior to the surgical neck is injured. Which of the following
is this vessel most likely to be?
A. Axillary artery
B. Brachial artery
C. Thoracoacromial artery
D. Transverse scapular artery
E. Posterior circumflex humeral artery
Answer: E

The circumflex humeral arteries lie at the surgical neck and is this scenario the posterior circumflex is likely to be injured. The
11
thoracoacromial and transverse scapular arteries lie more superomedially. The posterior circumflex humeral artery is a branch of
the axillary artery.

24. Which of the structures listed below lies posterior to the carotid sheath at the level of the 6th cervical vertebra?
A. Hypoglossal nerve
B. Vagus nerve
C. Cervical sympathetic chain
D. Ansa cervicalis
E. Glossopharyngeal nerve
Answer: C

The carotid sheath is crossed anteriorly by the hypoglossal nerves and the ansa cervicalis. The vagus lies within it. The cervical
sympathetic chain lies posteriorly between the sheath and the prevertebral fascia.

Common carotid artery

The right common carotid artery arises at the bifurcation of the brachiocephalic trunk, the left common carotid arises from the
arch of the aorta. Both terminate at the level of the upper border of the thyroid cartilage (the lower border of the third cervical
vertebra) by dividing into the internal and external carotid arteries.

Left common carotid artery


This vessel arises immediately to the left and slightly behind the origin of the brachiocephalic trunk. Its thoracic portion is 2.5- 3.5
cm in length and runs superolaterally to the sternoclavicular joint.

In the thorax
The vessel is in contact, from below upwards, with the trachea, left recurrent laryngeal nerve, left margin of the oesophagus.
Anteriorly the left brachiocephalic vein runs across the artery, and the cardiac branches from the left vagus descend in front of it.
These structures together with the thymus and the anterior margins of the left lung and pleura separate the artery from the
manubrium.

In the neck
The artery runs superiorly deep to sternocleidomastoid and then enters the anterior triangle. At this point it lies within the carotid
sheath with the vagus nerve and the internal jugular vein. Posteriorly the sympathetic trunk lies between the vessel and the
prevertebral fascia. At the level of C7 the vertebral artery and thoracic duct lie behind it. The anterior tubercle of C6 transverse
process is prominent and the artery can be compressed against this structure (it corresponds to the level of the cricoid).
Anteriorly at C6 the omohyoid muscle passes superficial to the artery.
Within the carotid sheath the jugular vein lies lateral to the artery.

Right common carotid artery


The right common carotid arises from the brachiocephalic artery. The right common carotid artery corresponds with the cervical
portion of the left common carotid, except that there is no thoracic duct on the right. The oesophagus is less closely related to the
right carotid than the left.

Summary points about the carotid anatomy

Path
Passes behind the sternoclavicular joint (12% patients above this level) to the upper border of the thyroid cartilage, to divide into
the external (ECA) and internal carotid arteries (ICA).

Relations

 Level of 6th cervical vertebra crossed by omohyoid


 Then passes deep to the thyrohyoid, sternohyoid, sternomastoid muscles.
 Passes behind the carotid tubercle (transverse process 6th cervical vertebra)-NB compression here stops haemorrhage.
 The inferior thyroid artery passes posterior to the common carotid artery.
 Then : Left common carotid artery crossed by thoracic duct, Right common carotid artery crossed by recurrent laryngeal
nerve

12
25. A 45 year old man presents with a lipoma located posterior to the posterior border of the sternocleidomastoid muscle,
approximately 4cm superior to the middle third of the clavicle. During surgical excision of the lesion troublesome
bleeding is encountered. Which of the following is the most likely source?
A. Internal jugular vein
B. External jugular vein
C. Common carotid artery
D. Vertebral artery
E. Second part of the subclavian artery
Answer: B

The external jugular vein runs obliquely in the superficial fascia of the posterior triangle. It drains into the subclavian vein. During
surgical exploration of this area the external jugular vein may be injured and troublesome bleeding may result. The internal
jugular vein and carotid arteries are located in the anterior triangle. The third, and not the second, part of the subclavian artery is
also a content of the posterior triangle

26. Which of the following upper limb muscles is not innervated by the radial nerve?
A. Extensor carpi ulnaris
B. Abductor digit minimi
C. Anconeus
D. Supinator
E. Brachioradialis
Answer: B

Abductor digiti minimi is innervated by the ulnar nerve

27. Which of the following forms the floor of the anatomical snuffbox?
A. Radial artery
B. Cephalic vein
C. Extensor pollicis brevis
D. Scaphoid bone
E. Cutaneous branch of the radial nerve
Answer: D

The scaphoid bone forms the floor of the anatomical snuffbox. The cutaneous branch of the radial nerve is much more
superficially and proximally located.

Anatomical snuffbox

Posterior border Tendon of extensor pollicis longus


Anterior border Tendons of extensor pollicis brevis and abductor pollicis longus
Proximal border Styloid process of the radius
Distal border Apex of snuffbox triangle
Floor Trapezium and scaphoid
Content Radial artery

28. A 32 year old lady complains of carpal tunnel syndrome. The carpal tunnel is explored surgically. Which of the
following structures will lie in closest proximity to the hamate bone within the carpal tunnel?
A. The tendon of abductor pollicis longus
B. The tendons of flexor digitorum profundus
C. The tendons of flexor carpi radialis longus
D. Median nerve
E. Radial artery
Answer: B

The carpal tunnel contains nine flexor tendons: Flexor digitorum profundus, Flexor digitorum superficialis, Flexor pollicis longus
The tendon of flexor digitorum profundus lies deepest in the tunnel and will thus lie nearest to the hamate bone.

Carpal bones
No tendons attach to: Scaphoid, lunate, triquetrum (stabilised by ligaments)

13
29. A 45 year man presents with hand weakness. He is given a piece of paper to hold between his thumb and index finger.
When the paper is pulled, the patient has difficulty maintaining a grip. Grip pressure is maintained by flexing the
thumb at the interphalangeal joint. What is the most likely nerve lesion?
A. Posterior interosseous nerve
B. Deep branch of ulnar nerve
C. Anterior interosseous nerve
D. Superficial branch of the ulnar nerve
E. Radial nerve
Answer: B
This is a description of Froment's sign, which tests for ulnar nerve palsy. It mainly tests for the function of adductor pollicis. This
is supplied by the deep branch of the ulnar nerve. Remember the anterior interosseous branch, which innervates the flexor pollicis
longus (hence causing flexion of the thumb IP joint), branches off more proximally to the wrist.

30. A 10 year old by falls out of a tree has suffers a supracondylar fracture. He complains of a painful elbow and forearm.
There is an obvious loss of pincer movement involving the thumb and index finger with minimal loss of sensation. The
most likely nerve injury is to the:

A. Ulnar nerve
B. Radial nerve
C. Anterior interosseous nerve
D. Axillary nerve damage
E. Median nerve damage above the elbow
Answer: C

The anterior interosseous nerve is a motor branch of the median nerve just below the elbow. When damaged it classically causes:
Pain in the forearm, Loss of pincer movement of the thumb and index finger (innervates the long flexor muscles of flexor pollicis
longus & flexor digitorum profundus of the index and middle finger), Minimal loss of sensation due to lack of a cutaneous branch
A 32 year old attends neurology clinic complaining of tingling in his hand. He has radial deviation of his wrist and there is
mild clawing of his fingers, with the 3rd and 4th digits being relatively spared. What is the most likely lesion?

A. Ulnar nerve damage at the wrist


B. Ulnar nerve damage at the elbow
C. Radial nerve damage at the elbow
D. Median nerve damage at the wrist
E. Median nerve damage at the elbow
Answer: B

At the elbow the ulnar nerve lesion affects the flexor carpi ulnaris and flexor digitorum profundus.

31. A 23 year old man is involved in a fight and is stabbed in his upper arm. The ulnar nerve is transected. Which of the
following muscles will not demonstrate compromised function as a result?
A. Flexor carpi ulnaris
B. Medial half of flexor digitorum profundus
C. Palmaris brevis
D. Hypothenar muscles
E. Pronator teres
Answer: E

M edial lumbricals, A dductor pollicis, F lexor digitorum profundus/Flexor digiti minimi, I nterossei, A bductor digiti minimi and
opponens
Innervates all intrinsic muscles of the hand (EXCEPT 2: thenar muscles & first two lumbricals - supplied by median nerve)
Pronator teres is innervated by the median nerve. Palmaris brevis is innervated by the ulnar nerve

32. Which of the structures listed below overlies the cephalic vein?
A. Extensor retinaculum
B. Bicipital aponeurosis
C. Biceps muscle
D. Antebrachial fascia
E. None of the above
14
Answer: E
The cephalic vein is superficially located in the upper limb and overlies most the fascial planes. It pierces the coracoid membrane
(continuation of the clavipectoral fascia) to terminate in the axillary vein. It lies anterolaterally to biceps.

Cephalic vein

Path

 Dorsal venous arch drains laterally into the cephalic vein


 Crosses the anatomical snuffbox and travels laterally up the arm
 At the antecubital fossa connected to the basilic vein by the median cubital vein
 Pierces deep fascia of deltopectoral groove to join axillary vein

33. A 22 year old man is involved in a fight. He sustains a laceration to the posterior aspect of his wrist. In the emergency
department the wound is explored and the laceration is found to be transversely orientated and overlies the region of
the extensor retinaculum, which is intact. Which of the following structures is least likely to be injured in this
scenario?

A. Dorsal cutaneous branch of the ulnar nerve


B. Tendon of extensor indicis
C. Basilic vein
D. Superficial branch of the radial nerve
E. Cephalic vein
Answer: B

The extensor retinaculum attaches to the radius proximal to the styloid, thereafter it runs obliquely and distally to wind around the
ulnar styloid (but does not attach to it). The extensor tendons lie deep to the extensor retinaculum and would therefore be less
susceptible to injury than the superficial structures.

Extensor retinaculum

The extensor rentinaculum is a thickening of the deep fascia that stretches across the back of the wrist and holds the long extensor
tendons in position.
Its attachments are:

 The pisiform and hook of hamate medially


 The end of the radius laterally

Structures related to the extensor retinaculum

Structures superficial to the retinaculum  Basilic vein


 Dorsal cutaneous branch of the ulnar nerve
 Cephalic vein
 Superficial branch of the radial nerve

Structures passing deep to the extensor  Extensor carpi ulnaris tendon


retinaculum  Extensor digiti minimi tendon
 Extensor digitorum and extensor indicis tendon
 Extensor pollicis longus tendon
 Extensor carpi radialis longus tendon
 Abductor pollicis longus and extensor pollicis brevis tendons

Beneath the extensor retinaculum fibrous septa form six compartments that contain the extensor muscle tendons. Each
compartment has its own synovial sheath.
The radial artery
The radial artery passes between the lateral collateral ligament of the wrist joint and the tendons of the abductor pollicis longus
and extensor pollicis brevis.

15
34. A man has an incision sited than runs 8cm from the deltopectoral groove to the midline. Which of the following is not
at risk of injury?
A. Cephalic vein
B. Shoulder joint capsule
C. Axillary artery
D. Pectoralis major
E. Trunk of the brachial plexus
Answer: B
This region will typically lie medial to the joint capsule. The diagram below illustrates the plane that this would transect and as it
can be appreciated the other structures are all at risk of injury.

Pectoralis major muscle

Origin From the medial two thirds of the clavicle, manubrium and sternocostal angle
Insertion Crest of the greater tubercle of the humerus
Nerve supply Lateral pectoral nerve
Actions Adductor and medial rotator of the humerus

Theme: Nerve Injury

A. Median nerve
B. Ulnar nerve
C. Radial nerve
D. Musculocutaneous nerve
E. Axillary nerve
F. Anterior interosseous nerve
G. Posterior interosseous nerve

For each scenario please select the most likely underlying nerve injury. Each option may be used once, more than once or not at
all.

35. A 19 year old student is admitted to A&E after falling off a wall. He is unable to flex his index finger. An x-ray
confirms a supracondylar fracture.

Answer : Median nerve


This median nerve is at risk during a supracondylar fracture.
36. A well toned weight lifter attends clinic reporting weakness of his left arm. There is weakness of flexion and supination
of the forearm.
Answer: Musculocutaneous nerve
Mucocutaneous nerve compression due to entrapment of the nerve between biceps and brachialis. Elbow flexion and
supination of the arm are affected. This is a rare isolated injury.
37. An 18 year old girl sustains an Holstein-Lewis fracture. Which nerve is at risk?

Answer: Radial nerve


Proximal lesions affect the triceps. Also paralysis of wrist extensors and forearm supinators occur. Reduced sensation of
dorsoradial aspect of hand and dorsal 31/2 fingers. Holstein-Lewis fractures are fractures of the distal humerus with radial
nerve entrapment.

38. A 35 year old farm labourer is injures the posterior aspect of his hand with a mechanical scythe. He severs some of his
extensor tendons in this injury. How many tunnels lie in the extensor retinaculum that transmit the tendons of the
extensor muscles?
A. One
B. Three
C. Four
D. Five
E. Six
Answer: E
There are six tunnels, each lined by its own synovial sheath.

The radial artery


The radial artery passes between the lateral collateral ligament of the wrist joint and the tendons of the abductor pollicis longus

16
and extensor pollicis brevis.

39. A 23 year old man is injured during a game of rugby. He suffers a fracture of the distal third of his clavicle, it is a
compound fracture and there is evidence of arterial haemorrhage. Which of the following vessels is most likely to be
encountered first during subsequent surgical exploration?
A. Posterior circumflex humeral artery
B. Axillary artery
C. Thoracoacromial artery
D. Sub scapular artery
E. Lateral thoracic artery
Answer: C
The thoracoacromial artery arises from the second part of the axillary artery. It is a short, wide trunk, which pierces the
clavipectoral fascia, and ends, deep to pectoralis major by dividing into four branches.

Thoracoacromial artery
The thoracoacromial artery (acromiothoracic artery; thoracic axis) is a short trunk, which arises from the forepart of the axillary
artery, its origin being generally overlapped by the upper edge of the Pectoralis minor. Projecting forward to the upper border of
the Pectoralis minor, it pierces the coracoclavicular fascia and divides into four branches: pectoral, acromial, clavicular, and
deltoid.

Branch Description

Pectoral Descends between the two Pectoral muscles, and is distributed to them and to the breast, anastomosing with the
branch intercostal branches of the internal thoracic artery and with the lateral thoracic.

Acromial Runs laterally over the coracoid process and under the Deltoid, to which it gives branches; it then pierces that
branch muscle and ends on the acromion in an arterial network formed by branches from the suprascapular,
thoracoacromial, and posterior humeral circumflex arteries.
Clavicular Runs upwards and medially to the sternoclavicular joint, supplying this articulation, and the Subclavius
branch

Deltoid Arising with the acromial, it crosses over the Pectoralis minor and passes in the same groove as the cephalic vein,
branch between the Pectoralis major and Deltoid, and gives branches to both muscles.

40. 68 year old man falls onto an outstretched hand. Following the accident he is examined in the emergency department.
On palpating his anatomical snuffbox there is tenderness noted in the base. What is the most likely injury in this
scenario?
A. Rupture of the tendon of flexor pollicis
B. Scaphoid fracture
C. Distal radius fracture
D. Rupture of flexor carpi ulnaris tendon
E. None of the above
Answer: B

A fall onto an outstretched hand is a common mechanism of injury for a scaphoid fracture. This should be suspected clinically if
there is tenderness in the base of the anatomical snuffbox. A tendon rupture would not result in bony tenderness.

41. Which of the following structures passes through the quadrangular space near the humeral head?
A. Axillary artery
B. Radial nerve
C. Axillary nerve
D. Median nerve
E. Transverse scapular artery
Answer: C

The quadrangular space is bordered by the humerus laterally, subscapularis superiorly, teres major inferiorly and the long head of
triceps medially. It lies lateral to the triangular space. It transmits the axillary nerve and posterior circumflex humeral artery.

17
42. Which of the following structures separates the ulnar artery from the median nerve?
A. Brachioradialis
B. Pronator teres
C. Tendon of biceps brachii
D. Flexor carpi ulnaris
E. Brachialis
Answer: B

It lies deep to pronator teres and this separates it from the median nerve.

43. A 32 year old motorcyclist is involved in a road traffic accident. His humerus is fractured and severely displaced. At
the time of surgical repair the surgeon notes that the radial nerve has been injured. Which of the following muscles is
least likely to be affected by an injury at this site?
A. Extensor carpi radialis brevis
B. Brachioradialis
C. Abductor pollicis longus
D. Extensor pollicis brevis
E. None of the above
Answer: E
The radial nerve supplies the extensor muscles, abductor pollicis longus and extensor pollicis brevis (the latter two being
innervated by the posterior interosseous branch of the radial nerve).

44. Which muscle is responsible for causing flexion of the interphalangeal joint of the thumb?
A. Flexor pollicis longus
B. Flexor pollicis brevis
C. Flexor digitorum superficialis
D. Flexor digitorum profundus
E. Adductor pollicis
Answer: A

There are 8 muscles:


1. Two flexors (flexor pollicis brevis and flexor pollicis longus)
2. Two extensors (extensor pollicis brevis and longus)
3. Two abductors (abductor pollicis brevis and longus)
4. One adductor (adductor pollicis)
5. One muscle that opposes the thumb by rotating the CMC joint (opponens pollicis).

Flexor and extensor longus insert on the distal phalanx moving both the MCP and IP joints.

45. An 18 year old man is stabbed in the axilla during a fight. His axillary artery is lacerated and repaired. However, the
surgeon neglects to repair an associated injury to the upper trunk of the brachial plexus. Which of the following
muscles is least likely to demonstrate impaired function as a result?
A. Palmar interossei
B. Infraspinatus
C. Brachialis
D. Supinator brevis
E. None of the above
Answer: A

The palmar interossei are supplied by the ulnar nerve. Which lies inferiorly and is therefore less likely to be injured.

46. A 23 year old man is involved in a fight, during the dispute he sustains a laceration to the posterior aspect of his right
arm, approximately 2cm proximal to the olecranon process. On assessment in the emergency department he is unable
to extend his elbow joint. Which of the following tendons is most likely to have been cut?
A. Triceps
B. Pronator teres
C. Brachioradialis
D. Brachialis
E. Biceps
Answer: A

The triceps muscle extends the elbow joint. The other muscles listed all produce flexion of the elbow joint.

18
47. Which of the following muscles does not attach to the radius?
A. Pronator quadratus
B. Biceps
C. Brachioradialis
D. Supinator
E. Brachialis
Answer: E
The brachialis muscle inserts into the ulna. The other muscles are all inserted onto the radius.

Radius
Bone of the forearm extending from the lateral side of the elbow to the thumb side of the wrist

Upper end

 Articular cartilage- covers medial > lateral side


 Articulates with radial notch of the ulna by the annular ligament
 Muscle attachment- biceps brachii at the tuberosity

Shaft

Muscle attachment-
Upper third of the body Supinator, Flexor digitorum superficialis, Flexor pollicis longus
Middle third of the body Pronator teres
Lower quarter of the body Pronator quadratus , tendon of supinator longus
Lower end

 Quadrilateral
 Anterior surface- capsule of wrist joint
 Medial surface- head of ulna
 Lateral surface- ends in the styloid process

Posterior surface: 3 grooves containing:

1. Tendons of extensor carpi radialis longus and brevis


2. Tendon of extensor pollicis longus
3. Tendon of extensor indicis

48. Which of the following is not an intrinsic muscle of the hand?


A. Opponens pollicis
B. Palmaris longus
C. Flexor pollicis brevis
D. Flexor digiti minimi brevis
E. Opponens digiti minimi
Answer: B

Mnemonic for intrinsic hand muscles


'A OF A OF A'

A dductor pollicis (thenar muscles)


O pponens digiti minimi
F lexor digiti minimi brevis A bductor pollicis brevis
A bductor digiti minimi (hypothenar muscles) O pponens pollicis
F lexor pollicis brevis

Palmaris longus originates in the forearm.

19
49. A 28 year old man lacerates the posterolateral aspect of his wrist with a knife in an attempted suicide. On arrival in
the emergency department the wound is inspected and found to be located over the lateral aspect of the extensor
retinaculum (which is intact). Which of the following structures is at greatest risk of injury?
A. Superficial branch of the radial nerve
B. Radial artery
C. Dorsal branch of the ulnar nerve
D. Tendon of extensor carpi radialis brevis
E. Tendon of extensor digiti minimi
Answer: A

The superficial branch of the radial nerve passes superior to the extensor retinaculum in the position of this laceration and is at
greatest risk of injury. The dorsal branch of the ulnar nerve and artery also pass superior to the extensor retinaculum n but are
located medially.

50. Transection of the radial nerve at the level of the axilla will result in all of the following except:
A. Loss of elbow extension.
B. Loss of extension of the interphalangeal joints.
C. Loss of metacarpophalangeal extension.
D. Loss of triceps reflex.
E. Loss of sensation overlying the first dorsal interosseous.
Answer: B
51. Which of the following muscls cause shoulder abduction
A. Teres major
B. Pectoralis major
C. Coracobrachialis
D. Supraspinatus
E. Latissimus dorsi
Answer: D

Supraspinatus is an abductor of the shoulder.

52. Which of the following structures is not closely related to the brachial artery?
A. Ulnar nerve
B. Median nerve
C. Cephalic vein
D. Long head of triceps
E. Median cubital vein
Answer: C
The cephalic vein lies superficially and on the contralateral side of the arm to the brachial artery. The relation of the ulnar nerves
and others are demonstrated in the image below:

53. The following statements relating to the musculocutaneous nerve are true except?
A. It arises from the lateral cord of the brachial plexus
B. It provides cutaneous innervation to the lateral side of the forearm
C. If damaged then extension of the elbow joint will be impaired
D. It supplies the biceps muscle
E. It runs beneath biceps
Answer: C

It supplies biceps, brachialis and coracobrachialis so if damaged then elbow flexion will be impaired.

Musculocutaneous nerve
 Branch of lateral cord of brachial plexus

Path

 It penetrates the Coracobrachialis muscle


 Passes obliquely between the Biceps brachii and the Brachialis to the lateral side of the arm
 Above the elbow it pierces the deep fascia lateral to the tendon of the Biceps brachii
 Continues into the forearm as the lateral cutaneous nerve of the forearm

20
Innervates

 Coracobrachialis
 Biceps brachii
 Brachialis

54. Which ligament keeps the head of the radius connected to the radial notch of the ulna?
A. Annular (orbicular) ligament
B. Quadrate ligament
C. Radial collateral ligament of the elbow
D. Ulnar collateral ligament
E. Radial collateral ligament
Answer: A

The annular ligament connects the radial head to the radial notch of the ulna. This is illustrated below:

55. A 38 year old man presents to the clinic with shoulder weakness. On examination he has an inability to initiate
shoulder abduction. Which of the nerves listed below is least likely to be functioning normally?
A. Suprascapular nerve
B. Medial pectoral nerve
C. Axillary nerve
D. Median nerve
E. Radial nerve
Answer: A
Suprascapular nerve
The suprascapular nerve arises from the upper trunk of the brachial plexus. It lies superior to the trunks of the brachial plexus and
passes inferolaterally parallel to them. It passes through the scapular notch, deep to trapezius. It innervates both supraspinatus and
infraspinatus and initiates abduction of the shoulder. If damaged, patients may be able to abduct the shoulder by leaning over the
affected side and deltoid can then continue to abduct the shoulder.
56. Which of the following vessels provides the greatest contribution to the arterial supply of the breast?
A. External mammary artery
B. Thoracoacromial artery
C. Internal mammary artery
D. Lateral thoracic artery
E. Subclavian artery

Answer: C

60% of the arterial supply to the breast is derived from the internal mammary artery. The external mammary and lateral thoracic
arteries also make a significant (but lesser) contribution. This is of importance clinically in performing reduction mammoplasty
procedures.

Breast

The breast itself lies on a layer of pectoral fascia and the following muscles:
1. Pectoralis major
2. Serratus anterior
3. External oblique

Breast anatomy
Nerve supply Branches of intercostal nerves from T4-T6.
Arterial supply  Internal mammary (thoracic) artery
 External mammary artery (laterally)
 Anterior intercostal arteries
 Thoraco-acromial artery

Venous drainage Superficial venous plexus to sub clavian, axillary and intercostal veins.

21
Lymphatic drainage  70% Axillary nodes
 Internal mammary chain
 Other lymphatic sites such as deep cervical and supraclavicular fossa (later in disease)

57. A baby is found to have a Klumpke's palsy post delivery. Which of the following is most likely to be present?
A. Loss of flexors of the wrist
B. Weak elbow flexion
C. Pronation of the forearm
D. Adducted shoulder
E. Shoulder medially rotated
Answer: A

Features of Klumpkes Paralysis: Claw hand (MCP joints extended and IP joints flexed), Loss of sensation over medial aspect of
forearm and hand, Horner's syndrome, Loss of flexors of the wrist

A C8, T1 root lesion is called Klumpke's paralysis and is caused by delivery with the arm extended.

58. With respect to the basilic vein, which statement is false?


A. Its deep anatomical location makes it unsuitable for use as an arteriovenous access site in fistula surgery
B. It originates from the dorsal venous network on the hand
C. It travels up the medial aspect of the forearm
D. Halfway between the shoulder and the elbow it lies deep to muscle
E. It joins the brachial vein to form the axillary vein
Answer: A

It is used in arteriovenous fistula surgery during a procedure known as a basilic vein transposition.

A 78 year old man is lifting a heavy object when a feels a pain in his forearm and is unable to continue. He has a swelling
over his upper forearm. An MRI scan shows a small cuff of tendon still attached to the radial tuberosity consistent with a
recent tear. Which of the following muscles has been injured?

A. Pronator teres
B. Supinator
C. Aconeus
D. Brachioradialis
E. Biceps brachii
Answer: E
Biceps inserts into the radial tuberosity. Distal injuries of this muscle are rare but are reported and are clinically more important
than more proximal ruptures.

59. Which of the following is a branch of the third part of the axillary artery?
A. Superior thoracic
B. Lateral thoracic
C. Dorsal scapular
D. Thoracoacromial
E. Posterior circumflex humeral
Answer: E
The other branches include :Subscapular, Anterior circumflex humeral
Axilla
Boundaries of the axilla
Medially Chest wall and Serratus anterior
Laterally Humeral head
Floor Subscapularis
Anterior aspect Lateral border of Pectoralis major
Fascia Clavipectoral fascia

Content:
Long thoracic nerve (of Bell) Derived from C5-C7 and passes behind the brachial plexus to enter the axilla. It lies on the
medial chest wall and supplies serratus anterior. Its location puts it at risk during axillary surgery
22
and damage will lead to winging of the scapula.
Thoracodorsal nerve and Innervate and vascularise latissimus dorsi.
thoracodorsal trunk
Axillary vein Lies at the apex of the axilla, it is the continuation of the basilic vein. Becomes the subclavian
vein at the outer border of the first rib.
Intercostobrachial nerves Traverse the axillary lymph nodes and are often divided during axillary surgery. They provide
cutaneous sensation to the axillary skin.
Lymph nodes The axilla is the main site of lymphatic drainage for the breast.

Theme: Nerve lesions

A. Intercostobrachial
B. Median
C. Axillary
D. Radial
E. Ulnar
F. Musculocutaneous
G. Brachial plexus upper cord
H. Brachial plexus lower cord

Please select the most likely nerve injury for the scenarios given. Each option may be used once, more than once or not at all.

60. A 23 year old rugby player sustains a Smiths Fracture. On examination opposition of the thumb is markedly
weakened.

Answer: Median
This high velocity injury can often produce significant angulation and displacement. Both of these may impair the function of
the median nerve with loss of function of the muscles of the thenar eminence
61. A 45 year old lady recovering from a mastectomy and axillary node clearance notices that sensation in her armpit is
impaired.

Answer: Intercostobrachial
The intercostobrachial nerves are frequently injured during axillary dissection. These nerves traverse the axilla and supply
cutaneous sensation.
62. An 8 year old boy falls onto an outstretched hand and sustains a supracondylar fracture. In addition to a weak radial
pulse the child is noted to have loss of pronation of the affected hand.

Answer: Median
This is a common injury in children. In this case the angulation and displacement have resulted in median nerve injury.

63. Froment's test which muscle function is tested?


A. Flexor pollicis longus
B. Adductor pollicis longus
C. Abductor pollicis brevis
D. Adductor pollicis
E. Opponens pollicis
Answer: D

Nerve signs
Froment's sign

 Assess for ulnar nerve palsy


 Adductor pollicis muscle function tested
 Hold a piece of paper between their thumb and index finger. The object is then pulled away. If ulnar nerve palsy, unable
to hold the paper and will flex the flexor pollicis longus to compensate (flexion of thumb at interphalangeal joint).

 More sensitive than Tinel's sign Phalen's test


 Hold wrist in maximum flexion and the test is
positive if there is numbness in the median nerve  Assess carpal tunnel syndrome
distribution.
23
 Tap the median nerve at the wrist and the test is Tinel's sign
positive if there is tingling/electric-like sensations
over the distribution of the median nerve.  Assess for carpal tunnel syndrome
Theme: Cutaneous innervation

A. Ulnar nerve
B. Fifth cervical spinal segment
C. Radial nerve
D. Musculocutaneous nerve
E. Median nerve
F. None of these

Please select the source of innervation for the region described. Each option may be used once, more than once or not at all.

64. The skin on the palmar aspect of the thumb


Answer: Median nerve
The median nerve supplies cutaneous sensation to this region.

65. The nail bed of the index finger


Answer: Median nerve
66. The skin overlying the medial aspect of the palm
Answer: Ulnar nerve

67. From which of the following foramina does the opthalmic branch of the trigeminal nerve exit the skull?
A. Foramen ovale
B. Foramen rotundum
C. Foramen spinosum
D. Superior orbital fissure
E. Foramen magnum
Answer: D
Mnemonic:Standing Room Only -Exit of branches of trigeminal nerve from the skull
V1 -Superior orbital fissure
V2 -foramen Rotundum
V3 -foramen Ovale

The opthalmic branch of the trigeminal nerve exits the skull through the superior orbital fissure.

Trigeminal nerve
The trigeminal nerve is the main sensory nerve of the head. In addition to its major sensory role, it also innervates the muscles of
mastication.

Distribution of the trigeminal nerve


Sensory  Scalp
 Face
 Oral cavity (and teeth)
 Nose and sinuses
 Dura mater

Motor  Muscles of mastication


 Mylohyoid
 Anterior belly of digastric
 Tensor tympani
 Tensor palati

Autonomic connections (ganglia)  Ciliary


 Sphenopalatine
 Otic
 Submandibular

24
Path
 Originates at the pons
 Sensory root forms the large, crescentic trigeminal ganglion within Meckel's cave, and contains the cell bodies of
incoming sensory nerve fibres. Here the 3 branches exit.
 The motor root cell bodies are in the pons and the motor fibres are distributed via the mandibular nerve. The motor root
is not part of the trigeminal ganglion.

Branches of the trigeminal nerve


Ophthalmic nerve Sensory only
Maxillary nerve Sensory only
Mandibular nerve Sensory and motor

Sensory
Ophthalmic Exits skull via the superior orbital fissure
Sensation of: scalp and forehead, the upper eyelid, the conjunctiva and cornea of the eye, the nose (including
the tip of the nose, except alae nasi), the nasal mucosa, the frontal sinuses, and parts of the meninges (the dura
and blood vessels).
Maxillary Exit skull via the foramen rotundum
nerve Sensation: lower eyelid and cheek, the nares and upper lip, the upper teeth and gums, the nasal mucosa, the
palate and roof of the pharynx, the maxillary, ethmoid and sphenoid sinuses, and parts of the meninges.
Mandibular Exit skull via the foramen ovale
nerve Sensation: lower lip, the lower teeth and gums, the chin and jaw (except the angle of the jaw), parts of the
external ear, and parts of the meninges.

Motor
Distributed via the mandibular nerve.
The following muscles of mastication are innervated:
 Masseter
 Tensor veli palatini  Temporalis
 Mylohyoid  Medial pterygoid
 Anterior belly of digastric  Lateral pterygoid
 Tensor tympani
Other muscles innervated include:
68. 42 year old lady has had an axillary node clearance for breast malignancy. Post operatively she reports weakness of
the shoulder. She is unable to push herself forwards from a wall with the right arm and the scapula is pushed out
medially from the chest wall. What is the most likely nerve injury?
A. C5, C6
B. C8, T1
C. Axillary nerve
D. Long thoracic nerve
E. Spinal accessory nerve

Answer: D
The patient has a winged scapula caused by damage to the long thoracic nerve (C5,6,7) during surgery. The long thoracic nerve
innervates serratus anterior. Serratus anterior causes pushing out of the scapula during a punch.

NB winging of the scapular laterally may indicate trapezius muscle weakness. Innervated by the spinal accessory nerve.

69. A 36 year old male is admitted for elective surgery for a lymph node biopsy in the supraclavicular region. Post
operatively the patient has difficulty shrugging his left shoulder. What nerve has been damaged?
A. Phrenic nerve
B. Axillary nerve
C. C5, C6 lesion
D. C8, T1 lesion
E. Accessory nerve

Answer: E
The accessory nerve lies in the posterior triangle and may be injured in this region. Apart from problems with shrugging the
shoulder, he may also have difficulty lifting his arm above his head.
25
70. Which of the following muscles is supplied by the musculocutaneous nerve?
A. Brachialis
B. Latissimus dorsi
C. Flexor carpi ulnaris
D. Teres minor
E. Triceps
Answer: A
Mnemonic Muscles innervated by the musculocutaneous nerve BBC: Biceps brachii, Brachialis, Coracobrachialis

71. A 17 year old male presents to the clinic. He complains of difficulty using his left hand. It has been a persistent
problem since he sustained a distal humerus fracture as a child. On examination there is diminished sensation
overlying the hypothenar eminence and medial one and half fingers. What is the most likely nerve lesion?
A. Anterior interosseous nerve
B. Posterior interosseous nerve
C. Ulnar nerve
D. Median nerve
E. Radial nerve
Answer: C

This sensory deficit pattern is most consistent with ulnar nerve injury.

72. A 72 year old male with end stage critical ischaemia is undergoing an axillo-femoral bypass. What structure is not
closely related to the axillary artery?
A. Posterior cord of the brachial plexus
B. Scalenus anterior muscle
C. Pectoralis minor muscle
D. Axillary vein
E. Lateral cord of the brachial plexus
Answer: B

The axillary artery is the continuation of the subclavian artery. It is surrounded by the cords of the brachial plexus (from
whichthey are named). The axillary vein runs alongside the axillary artery throughout its length.

73. Which of the following carpal bones is a sesamoid bone in the tendon of flexor carpi ulnaris?
A. Triquetrum
B. Lunate
C. Pisiform
D. Scaphoid
E. Capitate

Answer: C

This small bone has a single articular facet. It projects from the triquetral bone at the ulnar aspect of the wrist where most regard it
as a sesamoid bine lying within the tendon of flexor carpi ulnaris.

74. A 70 year old man falls and fractures his scaphoid bone. The fracture is displaced and the decision is made to insert a
screw to fix the fracture. Which of the following structures lies directly medial to the scaphoid?
A. Lunate
B. Pisiform
C. Trapezoid
D. Trapezium
E. None of the above

Answer: A
The lunate lies medially in the anatomical plane. Fractures of the scaphoid that are associated with high velocity injuries may
cause associated lunate dislocation.

Scaphoid bone
The scaphoid has a concave articular surface for the head of the capitate and at the edge of this is a crescentic surface for the
corresponding area on the lunate.
Proximally, it has a wide convex articular surface with the radius. It has a distally sited tubercle that can be palpated. The
remaining articular surface is to the lateral side of the tubercle. It faces laterally and is associated with the trapezium and trapezoid
bones.
The narrow strip between the radial and trapezial surfaces and the tubercle gives rise to the radial collateral carpal ligament. The
26
tubercle receives part of the flexor retinaculum. This area is the only part of the scaphoid that is available for the entry of blood
vessels. It is commonly fractured and avascular necrosis may result.

75. A 73 year old lady is hit by a car. She suffers a complex fracture of the distal aspect of her humerus with associated
injury to the radial nerve. Which of the following movements will be most impaired as a result?
A. Elbow extension
B. Elbow flexion
C. Shoulder abduction
D. Wrist extension
E. None of the above

Answer: D

The triceps will not be affected so elbow extension will be preserved. Loss of wrist extension will be the most obvious effect.

76. An 18 year old man develops a severe spreading sepsis of the hand. The palm is explored surgically and the flexor
digiti minimi brevis muscle is mobilised to facilitate drainage of the infection. Which of the following structures is not
closely related to this muscle?
A. The hook of hamate
B. Median nerve
C. Superficial palmar arterial arch
D. Digital nerves arising from the ulnar nerve
E. None of the above

Answer: B

The flexor digiti minimi brevis originates from the Hamate, on its undersurface lie the ulnar contribution to the superficial palmar
arterial arch and digital nerves derived from the ulnar nerve. The median nerve overlies the flexor tendons.

77. A 22 year old man develops an infection in the pulp of his little finger. What is the most proximal site to which this
infection may migrate?
A. The metacarpophalangeal joint
B. The distal interphalangeal joint
C. The proximal interphalangeal joint
D. Proximal to the flexor retinaculum
E. Immediately distal to the carpal tunnel

Ansewr: D

The 5th tendon sheath extends from the little finger to the proximal aspect of the carpal tunnel. This carries a significant risk of
allowing infections to migrate proximally.

78. Which of the following muscles is not innervated by the deep branch of the ulnar nerve?
A. Adductor pollicis
B. Hypothenar muscles
C. All the interosseous muscles
D. Opponens pollicis
E. Third and fourth lumbricals
Answer: D

Which of the following structures lie between the lateral and medial heads of the triceps muscle?

A. Radial nerve
B. Median nerve
C. Ulnar nerve
D. Axillary nerve
E. Medial cutaneous nerve of the forearm
Answer: A

27
The radial nerve runs in its groove on between the two heads. The ulnar nerve lies anterior to the medial head. The axillary nerve
passes through the quadrangular space. This lies superior to lateral head of the triceps muscle and thus the lateral border of the

79. Into which of the following structures does the superior part of the fibrous capsule of the shoulder joint insert?
A. The surgical neck of the humerus
B. The body of the humerus
C. The bicipital groove
D. Immediately distal to the greater tuberosity
E. The anatomical neck of the humerus

Answer: E

The shoulder joint is a shallow joint, hence its great mobility. However, this comes at the expense of stability. The fibrous capsule
attaches to the anatomical neck superiorly and the surgical neck inferiorly

80. Damage to the posterior cord of the brachial plexus will not result in any of the following except:
A. Klumpkes palsy
B. Anaesthesia overlying the lateral aspect of the forearm
C. A warm sweaty hand on the affected side
D. Loss of flexion of the arm
E. Anaesthesia overlying the posterior surface of the arm

Answer: E

The radial nerve gives cutaneous branches which supply the forearm posteriorly and the arm laterally. Division of the posterior
cord will impair the upper level of cutaneous sensation. However, the lateral cutaneous nerve of the forearm arises from the
musculocutaneous nerve and would be unaffected. Loss of sympathetic function would not result in a sweaty hand. Klumpkes
palsy occurs when the lower roots are C8-T1 are damaged.

81. A woman develops winging of the scapula following a Patey mastectomy. What is the most likely cause?
A. Division of pectoralis minor to access level 3 axillary nodes
B. Damage to the brachial plexus during axillary dissection
C. Damage to the long thoracic nerve during axillary dissection
D. Division of the thoracodorsal trunk during axillary dissection
E. Damage to the thoracodorsal trunk during axillary dissection

Answer: C

The serratus anterior muscle is supplied by the long thoracic nerve which runs along the surface of serratus anterior and is liable to
injury during nodal dissection. Although pectoralis minor is divided during a Patey mastectomy (now seldom performed) it is rare
for this alone to produce winging of the scapula.

Which of the following is not closely related to the capitate bone?

A. Lunate bone
B. Scaphoid bone
C. Ulnar nerve
D. Hamate bone
E. Trapezoid bone
Answer: C
The ulnar nerve and artery lie adjacent to the pisiform bone. The capitate bone articulates with the lunate, scaphoid, hamate and
trapezoid bones, which are therefore closely related to it.

Capitate bone
This is the largest of the carpal bones. It is centrally placed with a rounded head set into the cavities of the lunate and scaphoid
bones. Flatter articular surfaces are present for the hamate medially and the trapezoid laterally. Distally the bone articulates
predominantly with the middle metacarpal.

82. An injury to the spinal accessory nerve will affect which of the following movements?
A. Lateral rotation of the arm
28
B. Adduction of the arm at the glenohumeral joint
C. Protraction of the scapula
D. Upward rotation of the scapula
E. Depression of the scapula
Answer: D
The spinal accessory nerve innervates trapezius. The entire muscle will retract the scapula. However, its upper and lower fibres act
together to upwardly rotate it.

83. A 23 year old man falls over whilst intoxicated and a shard of glass transects his median nerve at the proximal border
of the flexor retinaculum. His tendons escape injury. Which of the following features will not be present?
A. Weakness of thumb abduction
B. Loss of sensation on the dorsal aspect of the thenar eminence
C. Loss of power of opponens pollicis
D. Adduction and lateral rotation of the thumb at rest
E. Loss of power of abductor pollicis brevis

Answer: B
The median nerve may be injured proximal to the flexor retinaculum. This will result in loss of flexor pollicis brevis, opponens
pollicis and the first and second lumbricals. When the patient is asked to close the hand slowly there is a lag of the index and
middle fingers reflecting the impaired lumbrical muscle function. The sensory changes are minor and do not extend to the dorsal
aspect of the thenar eminence.
Abductor pollicis longus will contribute to thumb abduction (and is innervated by the posterior interosseous nerve) and therefore
abduction will be weaker than prior to the injury.

84. A 23 year old man falls and injures his hand. There are concerns that he may have a scaphoid fracture as there is
tenderness in his anatomical snuffbox on clinical examination. Which of the following forms the posterior border of
this structure?
A. Basilic vein
B. Radial artery
C. Extensor pollicis brevis
D. Abductor pollicis longus
E. Extensor pollicis longus

Answer: E

Its boundaries are extensor pollicis longus, medially (posterior border) and laterally (anterior border) by the tendons of abductor
pollicis longus and extensor pollicis brevis.

85. A 28 year old man is stabbed outside a nightclub in the upper arm. The median nerve is transected. Which of the
following muscles will demonstrate impaired function as a result?
A. Palmaris brevis
B. Second and third interossei
C. Adductor pollicis
D. Abductor pollicis longus
E. Abductor pollicis brevis

Answer: E

Palmaris brevis - Ulnar nerve


Palmar interossei- Ulnar nerve
Adductor pollicis - Ulnar nerve
Abductor pollicis longus - Posterior interosseous nerve
Abductor pollicis brevis - Median nerve

The median nerve innervates all the short muscles of the thumb except the adductor and the deep head of the short flexor.
Palmaris and the interossei are innervated by the ulnar nerve.

29
Which of the following is not a branch of the posterior cord of the brachial plexus?

A. Thoracodorsal nerve
B. Axillary nerve
C. Radial nerve
D. Lower subscapular nerve
E. Musculocutaneous nerve
Answer: E
Mnemonic branches off the posterior cord: S ubscapular (upper and lower), T horacodorsal, A xillary, R adial

The musculocutaneous nerve is a branch off the lateral cord.

86. A 56 year old machinist has his arm entrapped in a steel grinder and is brought to the emergency department. On
examination, he is unable to extend his metacarpophalangeal joints and abduct his shoulder. He has weakness of his
elbow and wrist. What has been injured?
A. Ulnar nerve
B. Axillary nerve
C. Medial cord of brachial plexus
D. Lateral cord of brachial plexus
E. Posterior cord of brachial plexus
Answer: E
The posterior cord gives rise to:
 Radial nerve ((innervates the triceps, brachioradialis, wrist extensors, and finger extensors)
 Axillary nerve (innervates deltoid and teres minor)
 Upper subscapular nerve (innervates subscapularis)
 Lower subscapular nerve (innervates teres major and subscapularis)
 Thoracodorsal nerve (innervates latissimus dorsi)

This is a description of a posterior cord lesion. Remember that the posterior cord gives rise to the axillary and radial nerve.

Cords of the brachial plexus

The brachial plexus cords are described according to their relationship with the axillary artery. The cords pass over the 1st rib near
to the dome of the lung and pass beneath the clavicle immediately posterior to the subclavian artery.

Lateral cord: Anterior divisions of the upper and middle trunks form the lateral cord. Origin of the lateral pectoral nerve (C5, C6,
C7)

Medial cord: Anterior division of the lower trunk forms the medial cord. Origin of the medial pectoral nerve (C8, T1), the medial
brachial cutaneous nerve (T1), and the medial antebrachial cutaneous nerve (C8, T1)

Posterior cord. Formed by the posterior divisions of the 3 trunks (C5-T1). Origin of the upper and lower subscapular nerves (C7,
C8 and C5, C6, respectively) and the thoracodorsal nerve to the latissimus dorsi (also known as the middle subscapular nerve, C6,
C7, C8), axillary and radial nerve

87. A motor cyclist is involved in a road traffic accident causing severe right shoulder injuries. He is found to have an
adducted, medially rotated shoulder. The elbow is fully extended and the forearm pronated. Which is the most likely
diagnosis?
A. C8, T1 root lesion
B. C5, C6 root lesion
C. Radial nerve lesion
D. Ulnar nerve lesion
E. Axillary nerve lesion

Answer: B

Erbs Palsy C5, C6 lesion


The features include:Waiter's tip position

 Loss of shoulder abduction (deltoid and supraspinatus paralysis)


 Loss of external rotation of the shoulder (paralysis of infraspinatus and teres major)
 Loss of elbow flexion (paralysis of biceps, brachialis and brachioradialis)
 Loss of forearm supination (paralysis of Biceps)
30
The motorcyclist has had an Erb's palsy (C5, C6 root lesion). This is commonly known to be associated with birth injury when a
baby has a shoulder dystocia.

88. A 23 year old man has a cannula inserted into his cephalic vein. Through which structure does the cephalic vein pass?
A. Interosseous membrane
B. Triceps
C. Pectoralis major
D. Clavipectoral fascia
E. Tendon of biceps
Answer: D
The cephalic vein is a favored vessel for arteriovenous fistula formation and should be preserved in patients with end stage renal
failure. The cephalic vein penetrates the calvipectoral fascia (but not the pectoralis major) prior to terminating in the axillary vein.

89. Which of the following is not a muscle of the rotator cuff?


A. Subscapularis
B. Teres minor
C. Supraspinatus
D. Infraspinatus
E. Deltoid
Answer: E
Deltoid may abduct the shoulder and is not a rotator cuff muscle.

Muscles of the rotator cuff

Muscle Innervation
Supraspinatus muscle Suprascapular nerve
Infraspinatus muscle Suprascapular nerve
Teres minor muscle Axillary nerve
Subscapularis muscle Superior and inferior subscapular nerves

90. A 32 year old man is stabbed in the neck and the inferior trunk of his brachial plexus is injured. Which of the
modalities listed below is least likely to be affected?
A. Initiating abduction of the shoulder
B. Abduction of the fingers
C. Flexion of the little finger
D. Sensation on the palmar aspect of the little finger
E. Gripping a screwdriver

Answer: A

Inferior trunk of brachial plexus. C8 and T1 rootsContributes to ulnar nerve and part of median nerve. The inferior trunk of the
brachial plexus is rarely injured. Nerve roots C8 and T1 are the main contributors to this trunk. Therefore an injury to this site will
most consistently affect the ulnar nerve. The inferior trunk also contributes to the median nerve by way of the posterior division
and therefore some impairment of grip is almost inevitable.

91. As it exits the axilla the radial nerve lies on which of the following muscles?
A. Supraspinatus
B. Infraspinatus
C. Teres major
D. Deltoid
E. Pectoralis major
Answer: C

The radial nerve passes through the triangular space to leave the axilla. The superior border of this is bounded by the teres major
muscle to which the radial nerve is closely related.

31
92. A 62 year old man presents with arm weakness. On examination he has a weakness of elbow extension and loss of
sensation on the dorsal aspect of the first digit. What is the site of the most likely underlying defect?
A. Axillary nerve
B. Median nerve
C. Ulnar nerve
D. Radial nerve
E. Musculocutaneous nerve
Answer:D

The long head of the triceps muscle may be innervated by the axillary nerve and therefore complete loss of triceps muscles
function may not be present even with proximally sited nerve lesions.

From which of the following structures does the long head of the triceps muscle arise?

A. Coracoid process
B. Acromion
C. Infraglenoid tubercle
D. Coraco-acromial ligament
E. Coraco-humeral ligament
Answer: C

The long head arises from the infraglenoid tubercle. The fleshy lateral and medial heads are attached to the posterior aspect of the

93. A 58 year old lady presents with a mass in the upper outer quadrant of the right breast. Which of the following
statements relating to the breast is untrue?
A. The internal mammary artery provides the majority of its arterial supply
B. Nipple retraction may occur as a result of tumour infiltration of the clavipectoral fascia
C. The internal mammary artery is a branch of the subclavian artery
D. Up to 70% of lymphatic drainage is to the ipsilateral axillary nodes
E. None of the above
Answer: B

Nipple retraction is a feature of breast malignancy. However, it is typically caused by tumour infiltration of Coopers Ligaments
that run through the breast and surround the lobules. The clavipectoral fascia encases the axillary contents. The lymphatic
drainage of the breast is to the axilla and also to the internal mammary chain. The breast is well vascularised and the internal
mammary artery is a branch of the subclavian artery.

Theme: Nerve injury

A. Median nerve
B. Ulnar nerve
C. Radial nerve
D. Anterior interosseous nerve
E. Posterior interosseous nerve
F. Axillary nerve
G. Musculocutaneous nerve

Please select the nerve at risk of injury in each scenario. Each option may be used once, more than once or not at all.

94. A 43 year old typist presents with pain at the dorsal aspect of the upper part of her forearm. She also complains of
weakness when extending her fingers. On examination triceps and supinator are both functioning normally. There is
weakness of most of the extensor muscles. However, there is no sensory deficit.

Answer: Posterior interosseous nerve

The radial nerve may become entrapped in the "arcade of Frohse" which is a superficial part of the supinator muscle which
overlies the posterior interosseous nerve. This nerve is entirely muscular and articular in its distribution. It passes postero-
inferiorly and gives branches to extensor carpi radialis brevis and supinator. It enters supinator and curves around the lateral
and posterior surfaces of the radius. On emerging from the supinator the posterior interosseous nerve lies between the
superficial extensor muscles and the lowermost fibres of supinator. It then gives branches to the extensors.

95. A 28 year teacher reports difficulty with writing. There is no sensory loss. She is known to have an aberrant Gantzer
muscle.
32
Answer: Anterior interosseous nerve

Anterior interosseous lesions occur due to fracture, or rarely due to compression. The Gantzer muscle is an aberrant accessory
of the flexor pollicis longus and is a risk factor for anterior interosseous nerve compression. Remember loss of pincer grip and
normal sensation indicates an interosseous nerve lesion.

96. A 35 year tennis player attends reporting tingling down his arm. He says that his 'funny bone' was hit very hard by a
tennis ball. There is weakness of abduction and adduction of his extended fingers.

Answer: Ulnar nerve

The ulnar nerve arises from the medial cord of the brachial plexus (C8, T1 and contribution from C7). The nerve descends
between the axillary artery and vein, posterior to the cutaneous nerve of the forearm and then lies anterior to triceps on the
medial side of the brachial artery. In the distal half of the arm it passes through the medial intermuscular septum, and
continues between this structure and the medial head of triceps to enter the forearm between the medial epicondyle of the
humerus and the olecranon. It may be injured at this site in this scenario.

97. A 53 year old lady presents with pain and discomfort in her hand. She works as a typist and notices that the pain is
worst when she is working. She also suffers symptoms at night. Her little finger is less affected by the pain. Which of
the nerves listed below is most likely to be affected?
A. Radial
B. Median
C. Ulnar
D. Anterior interosseous nerve
E. Posterior interosseous nerve
Answer: B

The most likely diagnosis here is carpal tunnel syndrome, the median nerve is compressed in the wrist and symptoms usually
affect the fingers and wrist either at night or when the hand is being used (e.g. as a typist).

98. A 24 year female is admitted to A&E with tingling of her hand after a fall. She is found to have a fracture of the
medial epicondyle. What is the most likely nerve lesion?
A. Ulnar nerve
B. Radial nerve
C. Median nerve
D. Axillary nerve
E. Cutaneous nerve
Answer: A

The radial nerve is located near the lateral epicondyle.

A 43 year old lady is undergoing an axillary node clearance for breast cancer. The nodal disease is bulky. During
clearance of the level 3 nodes there is suddenly brisk haemorrhage. The most likely vessel responsible is:

A. Thoracoacromial artery
B. Cephalic vein
C. Thoracodorsal trunk
D. Internal mammary artery
E. Posterior circumflex humeral artery

Answer: A

The thoracoacromial artery pierces the pectoralis major and gives off branches within this space. The level 3 axillary nodes lie
between pectoralis major and minor.Although the thoracodorsal trunk may be injured during an axillary dissection it does not lie
within the level 3 nodes.

99. A 73 year old lady with long standing atrial fibrillation develops a cold and pulseless white arm. A brachial embolus is
suspected and a brachial embolectomy is performed. Which of the following structures is at greatest risk of injury
during this procedure?
A. Radial nerve
B. Cephalic vein

33
C. Ulnar nerve
D. Median nerve
E. None of the above
Answer: D
The median nerve lies close to the brachial artery in the antecubital fossa. This is the usual site of surgical access to the brachial
artery for an embolectomy procedure. The median nerve may be damaged during clumsy application of vascular clamps to the
artery.

100.Which of the following fingers is not a point of attachment for the palmar interossei?
A. Middle finger
B. Little finger
C. Ring finger
D. Index finger
E. None of the above

Answer: A

101.A 6 year old sustains a supracondylar fracture of the distal humerus. There are concerns that the radial nerve may
have been injured. What is the relationship of the radial nerve to the humerus at this point?
A. Anterolateral
B. Anteromedial
C. Posterolateral
D. Posteromedial
E. Immediately anterior
Answer: A

The radial nerve lies anterolateral to the humerus in the supracondylar area.

102.The following are true of the ulnar nerve except:


A. It innervates the palmar interossei
B. Derived from the medial cord of the brachial plexus
C. Supplies the muscles of the thenar eminence
D. Supplies the medial half of flexor digitorum profundus
E. Passes superficial to the flexor retinaculum
Answer: C

These are supplied by the median nerve and atrophy of these is a feature of carpal tunnel syndrome

103.Which muscle is responsible for causing flexion of the distal interphalangeal joint of the ring finger?
A. Flexor digitorum superficialis
B. Lumbricals
C. Palmar interossei
D. Flexor digitorum profundus
E. Flexor digiti minimi brevis
Answer: D

Flexor digitorum superficialis and flexor digitorum profundus are responsible for causing flexion. The superficialis tendons insert
on the bases of the middle phalanges; the profundus tendons insert on the bases of the distal phalanges. Both tendons flex the
wrist, MCP and PIP joints; however, only the profundus tendons flex the DIP joints.

104.Which of the following muscles lies medial to the long thoracic nerve?
A. Serratus anterior
B. Latissimus dorsi
C. Pectoralis major
D. Pectoralis minor
E. None of the above

34
Lower Limb
1. A 25 year old man is stabbed in the groin and the area, which lies within the femoral triangle is explored. Which
structure forms the lateral wall of the femoral triangle?
A.Adductor longus
B. Pectineus
C. Adductor magnus
D.Sartorius
E. Conjoint tendon
Answer: D
The sartorius forms the lateral wall of the femoral triangle (see below).

Femoral triangle anatomy


Boundaries

Superiorly Inguinal ligament


Laterally Sartorius
Medially Adductor longus
Floor pectineus and adductor longus muscles medially and iliopsoas muscle laterally.
Roof  Fascia lata and Superficial fascia
 Superficial inguinal lymph nodes (palpable below the inguinal ligament)
 Great saphenous vein

Contents

 Femoral vein (medial to lateral)


 Femoral artery-pulse palpated at the mid inguinal point
 Femoral nerve
 Deep and superficial inguinal lymph nodes
 Lateral cutaneous nerve
 Great saphenous vein
 Femoral branch of the genitofemoral nerve

2. Which of the following is not contained within the deep posterior compartment of the lower leg?
A. Tibialis posterior muscle
B. Posterior tibial artery
C. Tibial nerve
D. Sural nerve
E. Flexor hallucis longus

Answer: D

The deep posterior compartment lies anterior to soleus. The sural nerve is superficially sited and therefore not contained within it.

Lower limb- Muscular compartments

Anterior compartment

Muscle Nerve Action

Tibialis anterior Deep peroneal nerve Dorsiflexes ankle joint, inverts foot
Extends lateral four toes, dorsiflexes
Extensor digitorum longus Deep peroneal nerve
ankle joint
Peroneus tertius Deep peroneal nerve Dorsiflexes ankle, everts foot

Extensor hallucis longus Deep peroneal nerve Dorsiflexes ankle joint, extends big toe

35
Posterior and peroneal compartments

Muscle Nerve Action


Everts foot, assists in plantar
Peroneus longus Superficial peroneal nerve
flexion
Peroneal compartment
Peroneus brevis Superficial peroneal nerve Plantar flexes the ankle joint
Plantar flexes the foot, may
Gastrocnemius Tibial nerve
Superficial posterior also flex the knee
compartment
Soleus Tibial nerve Plantar flexor

Flexor digitorum longus Tibial Flexes the lateral four toes

Deep posterior compartment Flexor hallucis longus Tibial Flexes the great toe

Tibialis posterior Tibial Plantar flexor, inverts the foot

3. A 20 year old lady presents with pain on the medial aspect of her thigh. Investigations show a large ovarian cyst.
Compression of which of the nerves listed below is the most likely underlying cause?
A. Sciatic
B. Genitofemoral
C. Obturator
D. Ilioinguinal
E. Femoral cutaneous

Answer: C

The cutaneous branch of the obturator nerve is frequently absent. However, the obturator nerve is a recognised contributor to
innervation of the medial thigh and large pelvic tumours may compress this nerve with resultant pain radiating distally.

Obturator nerve

The obturator nerve arises from L2, L3 and L4 by branches from the ventral divisions of each of these nerve roots. L3 forms the
main contribution and the second lumbar branch is occasionally absent. These branches unite in the substance of psoas major,
descending vertically in its posterior part to emerge from its medial border at the lateral margin of the sacrum. It then crosses the
sacroiliac joint to enter the lesser pelvis, it descends on obturator internus to enter the obturator groove. In the lesser pelvis the
nerve lies lateral to the internal iliac vessels and ureter, and is joined by the obturator vessels lateral to the ovary or ductus
deferens.

Supplies

 Medial compartment of thigh


 Muscles supplied: external obturator, adductor longus, adductor brevis, adductor magnus (not the lower part-sciatic
nerve), gracilis
 The cutaneous branch is often absent. When present, it passes between gracilis and adductor longus near the middle part
of the thigh, and supplies the skin and fascia of the distal two thirds of the medial aspect.

Obturator canal

 Connects the pelvis and thigh: contains the obturator artery, vein, nerve which divides into anterior and posterior
branches.

36
4. A 22 year old man suffers a compound fracture of the tibia. During attempted surgical repair the deep peroneal nerve
is divided. Which of the following muscles will not be affected as a result?
A. Tibialis anterior
B. Peroneus longus
C. Extensor hallucis longus
D. Extensor digitorum longus
E. Peroneus tertius
Answer: B
Peroneus longus is innervated by the superficial peroneal nerve (L4, L5, S1).

Deep peroneal nerve

Origin From the common peroneal nerve, at the lateral aspect of the fibula, deep to peroneus longus
Nerve root values L4, L5, S1, S2
Course and relation  Pierces the anterior intermuscular septum to enter the anterior compartment of the lower leg
 Passes anteriorly down to the ankle joint, midway between the two malleoli

Terminates In the dorsum of the foot


Muscles innervated  Tibialis anterior
 Extensor hallucis longus
 Extensor digitorum longus
 Peroneus tertius
 Extensor digitorum brevis

Cutaneous Web space of the first and second toes


innervation
Actions  Dorsiflexion of ankle joint
 Extension of all toes (extensor hallucis longus and extensor digitorum longus)
 Eversion of the foot

After its bifurcation past the ankle joint, the lateral branch of the deep peroneal nerve innervates the extensor digitorum brevis and
the extensor hallucis brevis
The medial branch supplies the web space between the first and second digits.

5. Which of the following forms the medial wall of the femoral canal?
A. Pectineal ligament
B. Adductor longus
C. Sartorius
D. Lacunar ligament
E. Inguinal ligament
Answer: D
Femoral canal

The femoral canal lies at the medial aspect of the femoral sheath. The femoral sheath is a fascial tunnel containing both the
femoral artery laterally and femoral vein medially. The canal lies medial to the vein.

Borders of the femoral canal


Laterally Femoral vein
Medially Lacunar ligament
Anteriorly Inguinal ligament
Posteriorly Pectineal ligament

Contents

 Lymphatic vessels
 Cloquet's lymph node

Physiological significance
Allows the femoral vein to expand to allow for increased venous return to the lower limbs.
37
Pathological significance
As a potential space, it is the site of femoral hernias. The relatively tight neck places these at high risk of strangulation.

6. You decide to take an arterial blood gas from the femoral artery. Where should the needle be inserted to gain the
sample?
A. Mid point of the inguinal ligament
B. Mid inguinal point
C. 2cm inferomedially to the pubic tubercle
D. 2cm superomedially to the pubic tubercle
E. 3cm inferolaterally to the deep inguinal ring

Answer: B

The mid inguinal point in the surface marking for the femoral artery.

7. A 34 year old man is shot in the postero- inferior aspect of his thigh. Which of the following lies at the most lateral
aspect of the popliteal fossa?
A. Popliteal artery
B. Popliteal vein
C. Common peroneal nerve
D. Tibial nerve
E. Small saphenous vein

Answer: C

The contents of the popliteal fossa are (from medial to lateral): Popliteal artery; Popliteal vein; Tibial nerve
Common peroneal nerve
The sural nerve is a branch of the tibial nerve and usually arises at the inferior aspect of the popliteal fossa. However, its anatomy
is variable.

Popliteal fossa
Boundaries of the popliteal fossa
Laterally Biceps femoris above, lateral head of gastrocnemius and plantaris below
Medially Semimembranosus and semitendinosus above, medial head of gastrocnemius below
Floor Popliteal surface of the femur, posterior ligament of knee joint and popliteus muscle
Roof Superficial and deep fascia

Contents

 Popliteal artery and vein


 Small saphenous vein
 Common peroneal nerve
 Tibial nerve
 Posterior cutaneous nerve of the thigh
 Genicular branch of the obturator nerve
 Lymph nodes

8. A 76 year old man complains of symptoms of claudication. The decision is made to measure his ankle brachial
pressure index. The signal from the dorsalis pedis artery is auscultated with a hand held doppler device. This vessel is
the continuation of which of the following?
A. Posterior tibial artery
B. Anterior tibial artery
C. Peroneal artery
D. Popliteal artery
E. None of the above

Answer: B

The dorsalis pedis is a continuation of the anterior tibial artery.

38
Foot- anatomy

Arches of the foot


The foot is conventionally considered to have two arches.

 The longitudinal arch is higher on the medial than on the lateral side. The posterior part of the calcaneum forms a
posterior pillar to support the arch. The lateral part of this structure passes via the cuboid bone and the lateral two
metatarsal bones. The medial part of this structure is more important. The head of the talus marks the summit of this arch,
located between the sustentaculum tali and the navicular bone. The anterior pillar of the medial arch is composed of the
navicular bone, the three cuneiforms and the medial three metatarsal bones.
 The transverse arch is situated on the anterior part of the tarsus and the posterior part of the metatarsus. The cuneiforms
and metatarsal bases narrow inferiorly, which contributes to the shape of the arch.

Intertarsal joints
Sub talar joint Formed by the cylindrical facet on the lower surface of the body of the talus and the posterior facet on
the upper surface of the calcaneus. The facet on the talus is concave anteroposteriorly, the other is
convex. The synovial cavity of this joint does not communicate with any other joint.
Talocalcaneonavicular The anterior part of the socket is formed by the concave articular surface of the navicular bone,
joint posteriorly by the upper surface of the sustentaculum tali. The talus sits within this socket
Calcaneocuboid joint Highest point in the lateral part of the longitudinal arch. The lower aspect of this joint is reinforced by
the long plantar and plantar calcaneocuboid ligaments.
Transverse tarsal joint The talocalcaneonavicular joint and the calcaneocuboid joint extend accross the tarsus in an irregular
transverse plane, between the talus and calcaneus behind and the navicular and cuboid bones in front.
This plane is termed the transverse tarsal joint.
Cuneonavicular joint Formed between the convex anterior surface of the navicular bone and the concave surface of the the
posterior ends of the three cuneiforms.
Intercuneiform joints Between the three cuneiform bones.
Cuneocuboid joint Between the circular facets on the lateral cuneiform bone and the cuboid. This joint contributes to the
tarsal part of the transverse arch.
A detailed knowledge of the joints is not required for MRCS Part A. However, the contribution they play to the overall structure of
the foot should be appreciated

Muscles of the foot


Muscle Origin Insertion Nerve Action
supply
Abductor Medial side of the calcaneus, flexor Medial side of the Medial Abducts the great toe
hallucis retinaculum, plantar aponeurosis base of the proximal plantar
phalanx nerve
Flexor Medial process of the calcaneus, plantar Via 4 tendons into the Medial Flexes all the joints of the
digitorum eponeurosis. middle phalanges of plantar lateral 4 toes except for the
brevis the lateral 4 toes. nerve interphalangeal joint.
Abductor From the tubercle of the calcaneus and from Together with flexor Lateral Abducts the little toe at the
digit the plantar aponeurosis digit minimi brevis plantar metatarsophalangeal joint
minimi into the lateral side of nerve
the base of the
proximal phalanx of
the little toe
Flexor From the medial side of the plantar surface Into the proximal Medial Flexes the metatarsophalangeal
hallucis of the cuboid bone, from the adjacent part of phalanx of the great plantar joint of the great toe.
brevis the lateral cuneiform bone and from the toe, the tendon nerve
tendon of tibialis posterior. contains a sesamoid
bone
Adductor Arises from two heads. The oblique head Lateral side of the Lateral Adducts the great toe towards
hallucis arises from the sheath of the peroneus base of the proximal plantar the second toe. Helps maintain
longus tendon, and from the plantar surfaces phalanx of the great nerve the transverse arch of the foot.
of the bases of the 2nd, 3rd and 4th toe.
metatarsal bones. The transverse head arises
from the plantar surface of the lateral 4
metatarsophalangeal joints and from the
deep transverse metatarsal ligament.
Extensor On the dorsal surface of the foot from the Via four thin tendons Deep Extend the
digitorum upper surface of the calcaneus and its which run forward peroneal metatarsophalangeal joint of
39
brevis associated fascia and medially to be the medial four toes. It is
inserted into the unable to extend the
medial four toes. The interphalangeal joint without
lateral three tendons the assistance of the lumbrical
join with hoods of muscles.
extensor digitorum
longus.

Detailed knowledge of the foot muscles are not needed for the MRCS part A

Nerves in the foot

Lateral plantar nerve


Passes anterolaterally towards the base of the 5th metatarsal between flexor digitorum brevis and flexor accessorius. On the
medial aspect of the lateral plantar artery. At the base of the 5th metatarsal it splits into superficial and deep branches.

Medial plantar nerve


Passes forwards with the medial plantar artery under the cover of the flexor retinaculum to the interval between abductor hallucis
and flexor digitorum brevis on the sole of the foot.

Plantar arteries
Arise under the cover of the flexor retinaculum, midway between the tip of the medial malleolus and the most prominent part of
the medial side of the heel.

 Medial plantar artery. Passes forwards medial to medial plantar nerve in the space between abductor hallucis and flexor
digitorum brevis.Ends by uniting with a branch of the 1st plantar metatarsal artery.
 Lateral plantar artery. Runs obliquely across the sole of the foot. It lies lateral to the lateral plantar nerve. At the base of
the 5th metatarsal bone it arches medially across the foot on the metatarsals

Dorsalis pedis artery


This vessel is a direct continuation of the anterior tibial artery. It commences on the front of the ankle joint and runs to the
proximal end of the first metatarsal space. Here is gives off the arcuate artery and continues forwards as the first dorsal metatarsal
artery. It is accompanied by two veins throughout its length. It is crossed by the extensor hallucis brevis

9. A 67 year old man is due to undergo a revisional total hip replacement using a posterior approach. After dividing
gluteus maximus in the line of its fibres there is brisk arterial bleeding. Which of the following vessels is likely to be
responsible?
A. Profunda femoris artery
B. External iliac artery
C. Internal iliac artery
D. Obturator artery
E. Inferior gluteal artery

Answer: E

The inferior gluteal artery runs on the deep surface of the gluteus maximus muscle. It is a branch of the internal iliac artery. It is
commonly divided during the posterior approach to the hip joint.

Hip joint

 Head of femur articulates with acetabulum of the pelvis


 Both covered by articular hyaline cartilage
 The acetabulum forms at the union of the ilium, pubis, and ischium
 The triradiate cartilage (Y-shaped growth plate) separates the pelvic bones
 The acetabulum holds the femoral head by the acetabular labrum
 Normal angle between femoral head and femoral shaft is 130o

Ligaments

40
 Transverse ligament: joints anterior and posterior ends of the articular cartilage
 Head of femur ligament (ligamentum teres): acetabular notch to the fovea. Contains arterial supply to head of femur in
children.

Extracapsular ligaments

 Iliofemoral ligament: inverted Y shape. Anterior iliac spine to the trochanteric line
 Pubofemoral ligament: acetabulum to lesser trochanter
 Ischiofemoral ligament: posterior support. Ischium to greater trochanter.

Blood supply
Medial circumflex femoral and lateral circumflex femoral arteries (Branches of profunda femoris)

2 anastomoses: Cruciate and the trochanteric anastomoses (provides most of the blood to the head of the femur) Hence the need
for hemiarthroplasty when there is a displaced femoral head fracture. These anastomoses exist between the femoral artery or
profunda femoris and the gluteal vessels.

10. Which of the following nerves passes through the greater and lesser sciatic foramina?
A. Pudendal nerve
B. Sciatic nerve
C. Superior gluteal nerve
D. Inferior gluteal nerve
E. Posterior cutaneous nerve of the thigh
Answer: A
Structures passing through the lesser and greater sciatic foramina (medial to lateral): PIN

 Pudendal nerve
 Internal pudendal artery
 Nerve to obturator internus

The pudendal nerve originates from the ventral rami of the second, third, and fourth sacral nerves (S2, S3, S4). It passes between
the piriformis and coccygeus muscles and exits the pelvis through the the greater sciatic foramen. It crosses the spine of the
ischium and reenters the pelvis through the lesser sciatic foramen. It passes through the pudendal canal. The pudendal nerve gives
off the inferior rectal nerves. It terminates into 2 branches: perineal nerve, and the dorsal nerve of the penis or the dorsal nerve of
the clitoris.

Greater sciatic foramen

Contents
Nerves  Sciatic Nerve
 Superior and Inferior Gluteal Nerves
 Internal Pudendal Nerve
 Posterior Femoral Cutaneous Nerve
 Nerve to Quadratus Femoris
 Nerve to Obturator internus

Vessels  Superior Gluteal Artery and vein


 Inferior Gluteal Artery and vein
 Internal Pudendal Artery and vein

Piriformis
The piriformis is a landmark for identifying structures passing out of the sciatic notch

 Above piriformis: Superior gluteal vessels


 Below piriformis: Inferior gluteal vessels, sciatic nerve (10% pass through it, <1% above it) and posterior cutaneous
nerve of the thigh

41
Greater sciatic foramen boundaries
Anterolaterally Greater sciatic notch of the ilium
Posteromedially Sacrotuberous ligament
Inferior Sacrospinous ligament and the ischial spine
Superior Anterior sacroiliac ligament

Contents of the lesser sciatic foramen

 Tendon of the obturator internus


 Pudendal nerve
 Internal pudendal artery and vein
 Nerve to the obturator internus

11. A 65 year old man with long standing atrial fibrillation develops an embolus to the lower leg. The decision is made to
perform an embolectomy, utilising a trans popliteal approach. After incising the deep fascia, which of the following
structures will the surgeons encounter first on exploring the central region of the popliteal fossa?
A. Popliteal vein
B. Common peroneal nerve
C. Popliteal artery
D. Tibial nerve
E. None of the above

Answer: D
The tibial nerve lies superior to the vessels in the inferior aspect of the popliteal fossa. In the upper part of the fossa the tibial
nerve lies lateral to the vessels, it then passes superficial to them to lie medially. The popliteal artery is the deepest structure in the
popliteal fossa.

12. A 43 year old lady presents with varicose veins and undergoes a saphenofemoral disconnection, long saphenous vein
stripping to the ankle and isolated hook phlebectomies. Post operatively she notices an area of numbness superior to
her ankle. What is the most likely cause for this?
A. Sural nerve injury
B. Femoral nerve injury
C. Saphenous nerve injury
D. Common peroneal nerve injury
E. Superficial peroneal nerve injury

Answer: C

The sural nerve is related to the short saphenous vein. The saphenous nerve is related to the long saphenous vein below the knee
and for this reason full length stripping of the vein is no longer advocated.

Saphenous vein

Long saphenous vein

This vein may be harvested for triple or quadruple bypass surgery

 Originates at the 1st digit where the dorsal vein merges with the dorsal venous arch of the foot
 Passes anterior to the medial malleolus and runs up the medial side of the leg
 At the knee, it runs over the posterior border of the medial epicondyle of the femur bone
 Then passes laterally to lie on the anterior surface of the thigh before entering an opening in the fascia lata called the
saphenous opening
 It joins with the femoral vein in the region of the femoral triangle at the saphenofemoral junction

Tributaries

 Medial marginal, superficial epigastric, superficial iliac circumflex and superficial external pudendal veins

42
Short saphenous vein

 Originates at the 5th digit where the dorsal vein merges with the dorsal venous arch of the foot, which attaches to the
great saphenous vein.
 It passes around the lateral aspect of the foot (inferior and posterior to the lateral malleolus) and runs along the posterior
aspect of the leg (with the sural nerve)
 Passes between the heads of the gastrocnemius muscle, and drains into the popliteal vein, approximately at or above the
level of the knee joint.

13. A 34 year old man undergoes excision of a sarcoma from the right buttock. During the procedure the sciatic nerve is
sacrificed. Which of the following will not occur as a result of this process?
A. Loss of extension at the knee joint
B. Foot drop
C. Inability to extend extensor hallucis longus
D. Loss of sensation to the posterior aspect of the thigh
E. Loss of sensation to the posterior aspect of the lower leg

Answer: A

Extension of the knee joint is caused by the obturator and femoral nerves.

Sciatic nerve

Origin Spinal nerves L4 - S3


Articular Branches Hip joint
Muscular branches in upper leg  Semitendinosus
 Semimembranosus
 Biceps femoris
 Part of adductor magnus

Cutaneous sensation  Posterior aspect of thigh


 Gluteal region
 Entire lower leg (except the medial aspect)

Terminates At the upper part of the popliteal fossa by dividing into the tibial and peroneal nerves

 The nerve to the short head of the biceps femoris comes from the common peroneal part of the sciatic and the other
muscular branches arise from the tibial portion.
 The tibial nerve goes on to innervate all muscles of the foot except the extensor digitorum brevis (which is innervated by
the common peroneal nerve).

14. A 24 year old lady is stabbed in the buttock. Following the injury the wound is sutured in the emergency department.
Eight weeks later she attends the clinic, as she walks into the clinic room she has a waddling gait and difficulty with
thigh abduction. On examination she has buttock muscle wasting. Which nerve has been injured?
A. Superior gluteal nerve
B. Obturator nerve
C. Sciatic nerve
D. Femoral nerve
E. Inferior gluteal nerve
Answer: A

Damage to the superior gluteal nerve will result in a Trendelenberg gait.

Trendelenberg test
Injury or division of the superior gluteal nerve results in a motor deficit that consists of weakened abduction of the thigh by
gluteus medius, a disabling gluteus medius limp and a compensatory list of the body weakened gluteal side. The compensation
results in a gravitational shift so that the body is supported on the unaffected limb.
When a person is asked to stand on one leg the gluteus medius usually contracts as soon as the contralateral leg leaves the floor,
preventing the pelvis from dipping towards the unsupported side. When a person with paralysis of the superior gluteal nerve is
43
asked to stand on one leg, the pelvis on the unsupported side descends, indicating that the gluteus medius on the affected side is
weak or non functional ( a positive Trendelenberg test).

15. A 73 year old lady presents with symptoms of faecal incontinence. On examination she has weak anal sphincter
muscles. What are the main nerve root values of the nerves supplying the external anal sphincter?
A. S2,3
B. L5, S1
C. S4,5
D. S5
E. S2,3,4
Answer: E
The external anal sphincter is innervated by the inferior rectal branch of the pudendal nerve, this has root values of S2, 3 and the
perineal branch of S4.

Anal sphincter

 Internal anal sphincter composed of smooth muscle continuous with the circular muscle of the rectum. It surrounds the
upper two- thirds of the anal canal and is supplied by sympathetic nerves.
 External anal sphincter is composed of striated muscle which surrounds the internal sphincter but extends more distally.
 The nerve supply of the external anal sphincter is from the inferior rectal branch of the pudendal nerve (S2 and S3) and
the perineal branch of the S4 nerve roots.

16. A 72 year old man has a fall. He is found to have a fractured neck of femur and goes on to have a left hip
hemiarthroplasty. Two months post operatively he is found to have an odd gait. When standing on his left leg his
pelvis dips on the right side. There is no foot drop. What is the cause?
A. Sciatic nerve damage
B. L5 radiculopathy
C. Inferior gluteal nerve damage
D. Previous poliomyelitis
E. Superior gluteal nerve damage

Answer: E

This patient has a trendelenburg gait caused by damage to the superior gluteal nerve causing weakness of the abductor muscles.
Classically a patient is asked to stand on one leg and the pelvis dips on the opposite side. The absence of a foot drop excludes the
possibility of polio or L5 radiculopathy.

Gluteal region

Gluteal muscles

 Gluteus maximus: inserts to gluteal tuberosity of the femur and iliotibial tract
 Gluteus medius: attach to lateral greater trochanter
 Gluteus minimis: attach to anterior greater trochanter
 All extend and abduct the hip

Deep lateral hip rotators

 Piriformis
 Gemelli
 Obturator internus
 Quadratus femoris

Nerves
Superior gluteal nerve (L5, S1)  Gluteus medius
 Gluteus minimis
 Tensor fascia lata

Inferior gluteal nerve Gluteus maximus


44
Damage to the superior gluteal nerve will result in the patient developing a Trendelenberg gait. Affected patients are unable to
abduct the thigh at the hip joint. During the stance phase, the weakened abductor muscles allow the pelvis to tilt down on the
opposite side. To compensate, the trunk lurches to the weakened side to attempt to maintain a level pelvis throughout the gait
cycle. The pelvis sags on the opposite side of the lesioned superior gluteal nerve.

17. Which of the following structures lies posterior to the femoral nerve in the femoral triangle?
A. Adductor longus
B. Pectineus
C. Psoas major
D. Iliacus
E. None of the above
Answer: D

The iliacus lies posterior to the femoral nerve in the femoral triangle. The femoral sheath lies anterior to the iliacus and pectineus
muscles.

Femoral nerve

Root values L2, 3, 4


Innervates  Pectineus
 Sartorius
 Quadriceps femoris
 Vastus lateralis/medialis/intermedius

Branches  Medial cutaneous nerve of thigh


 Saphenous nerve
 Intermediate cutaneous nerve of thigh

Path
Penetrates psoas major and exits the pelvis by passing under the inguinal ligament to enter the femoral triangle, lateral to the
femoral artery and vein.

Mnemonic for femoral nerve supply

(don't) M I S V Q Scan for PE


M edial cutaneous nerve of the thigh
I ntermediate cutaneous nerve of the thigh
S aphenous nerve
V astus
Q uadriceps femoris
S artorius
PE ectineus

18. Which of the following ligaments contains the artery supplying the head of femur in children?
A.Transverse ligament
B. Ligamentum teres
C. Iliofemoral ligament
D.Ischiofemoral ligament
E. Pubofemoral ligament

Answer: B

19. A 68 year old man with critical limb ischaemia is undergoing a femoro-distal bypass graft. During mobilisation of the
proximal part of the posterior tibial artery which of the following is at greatest risk of injury?
A. Tibial nerve
B. Sciatic nerve
C. Saphenous nerve
D. Common peroneal nerve
E. Medial superior genicular artery
45
Answer: A

The tibial nerve is closely related to the posterior tibial artery. The tibial nerve crosses the vessel posteriorly approximately 2.5cm
distal to its origin. At its origin the nerve lies medial and then lateral after it crosses the vessel as described.

Posterior tibial artery

 Larger terminal branch of the popliteal artery


 Terminates by dividing into the medial and lateral plantar arteries
 Accompanied by two veins throughout its length
 Position of the artery corresponds to a line drawn from the lower angle of the popliteal fossa, at the level of the neck of
the fibula, to a point midway between the medial malleolus and the most prominent part of the heel

Relations of the posterior tibial artery


Proximal to distal
Anteriorly Tibialis posterior
Flexor digitorum longus
Posterior surface of tibia and ankle joint
Posterior Tibial nerve 2.5 cm distal to its origin
Fascia overlying the deep muscular layer
Proximal part covered by gastrocnemius and soleus
Distal part covered by skin and fascia

20. Which of the following structures lies deepest in the popliteal fossa?
A. Popliteal artery
B. Popliteal vein
C. Tibial nerve
D. Common peroneal nerve
E. Popliteal lymph nodes

Answer: A
From superficial to deep:
The common peroneal nerve exits the popliteal fossa along the medial border of the biceps tendon. Then the tibial nerve lies
lateral to the popliteal vessels to pass posteriorly and then medially to them. The popliteal vein lies superficial to the popliteal
artery, which is the deepest structure in the fossa.

21. An intravenous drug user develops a false aneurysm and requires emergency surgery. The procedure is difficult and
the femoral nerve is inadvertently transected. Which of the following muscles is least likely to be affected as a result?
A. Sartorius
B. Vastus medialis
C. Pectineus
D. Quadriceps femoris
E. Adductor magnus
Answer: E

Adductor magnus is innervated by the obturator and sciatic nerve. The pectineus muscle is sometimes supplied by the obturator
nerve but this is variable. Since the question states least likely, the correct answer is adductor magnus

22. Which of the following structures does not pass posteriorly to the medial malleolus?
A. Posterior tibial artery
B. Tibial nerve
C. Tibialis anterior tendon
D. Tendon of flexor digitorum longus
E. Tendon of flexor hallucis longus
Answer: C

Mnemonic for structures posterior to the medial malleolus: Tom Dick And Nervous Harry: T ibialis posterior tendon; flexor
Digitorum longus; A rtery; N erve; H allucis longus

46
Medial malleolus

The following structures pass posterior to medial malleolus (in order):

 Tibialis posterior tendon


 Flexor digitorum longus tendon
 Posterior tibial artery
 Tibial nerve
 Tendon of flexor hallucis longus

23. A 44 year old man has a malignant melanoma and is undergoing a block dissection of the groin. The femoral triangle
is being explored for intra operative bleeding. Which of the following forms the medial border of the femoral triangle?
A. Femoral artery
B. Biceps femoris
C. Adductor longus
D. Sartorius
E. Adductor magnus
Answer: C

Vastus medialis forms the lateral border of the adductor canal. The sartorius muscles forms the roof of the adductor canal.
Adductor longus forms the medial boundary of the femoral triangle (see below).

24. The foramen marking the termination of the adductor canal is located in which of the following?
A.Adductor longus
B. Adductor magnus
C. Adductor brevis
D.Sartorius
E. Semimembranosus
Answer: B

The foramen marking the distal limit of the adductor canal is contained within adductor magnus. The vessel passes through this
region to enter the popliteal fossa.

Adductor canal

 Also called Hunter's or subsartorial canal

 Immediately distal to the apex of the femoral triangle, lying in the middle third of the thigh. Canal terminates at the
adductor hiatus.

Contents

 Saphenous nerve
 Superficial femoral artery
 Superficial femoral vein

Borders
Laterally Vastus medialis muscle
Posteriorly Adductor longus, adductor magnus
Roof Sartorius

25. A 24 year old motor cyclist is involved in a road traffic accident. He suffers a tibial fracture which is treated with an
intra medullary nail. Post operatively he develops a compartment syndrome. Surgical decompression of the anterior
compartment will relieve pressure on all of the following muscles except?
A. Peroneus brevis
B. Peroneus tertius
C. Extensor digitorum longus
D. Tibialis anterior
E. None of the above
47
Answer: A
The anterior compartment contains: Tibialis anterior, Extensor digitorum longus, Peroneus tertius, Extensor hallucis longus,
Anterior tibial artery, All the muscles are innervated by the deep peroneal nerve.

26. A 42 year old lady is reviewed in the outpatient clinic following a routine surgical procedure. She complains of
diminished sensation at the lateral aspect of her foot. Which of the following nerves is likely to be affected?
A. Sural
B. Superficial peroneal
C. Deep peroneal
D. Medial plantar
E. Lateral Plantar

Answer: A

The sural nerve supplies the lateral aspect of the foot. It runs alongside the short saphenous vein and may be injured in short
saphenous vein surgery.

Foot- Cutaneous sensation

Region Nerve
Lateral plantar Sural
Dorsum (not 1st web space) Superficial peroneal
1st Web space Deep peroneal
Extremities of toes Medial and lateral plantar nerves
Proximal plantar Tibial
Medial plantar Medial plantar nerve
Lateral plantar Lateral plantar nerve

27. A sprinter attends A&E with severe leg pain. He had forgotten to warm up and ran a 100m sprint race. Towards the
end of the race he experienced pain in the posterior aspect of his thigh. The pain worsens, localising to the lateral
aspect of the knee. The sprinter is unable to flex the knee. What structure has been injured?
A.Anterior cruciate ligament
B. Posterior cruciate ligament
C. Semimembranosus tendon
D.Semiteninosus tendon
E. Biceps femoris tendon

Answer: E
The biceps femoris is commonly injured in sports that require explosive bending of the knee as seen in sprinting, especially if the
athlete has not warmed up first. Avulsion most commonly occurs where the long head attaches to the ischial tuberosity. Injuries to
biceps femoris are more common than to the other hamstrings.

Biceps femoris

The biceps femoris is one of the hamstring group of muscles located in the posterior upper thigh. It has two heads.

Long head

Origin Ischial tuberosity


Insertion Fibular head
Action Knee flexion, lateral rotation tibia, extension hip
Innervation Tibial nerve (L5, S1, S2)
Arterial supply Profunda femoris artery, inferior gluteal artery, and the superior muscular branches of popliteal artery

Short head

Origin Lateral lip of linea aspera, lateral supracondylar ridge of femur


Insertion Fibular head
Action Knee flexion, lateral rotation tibia
Innervation Common peroneal nerve (L5, S1, S2)
Arterial supply Profunda femoris artery, inferior gluteal artery, and the superior muscular branches of popliteal artery

48
28. The sciatic nerve lies deep to the following structures except:
A. Gluteus maximus
B. The femoral cutaneous nerve
C. Long head of biceps femoris
D. Gluteus medius
E. Branch of the inferior gluteal artery

Answer: D

The gluteus medius does not extend around to the sciatic nerve.

Sciatic nerve

Origin Spinal nerves L4 - S3


Articular Branches Hip joint
Muscular branches in upper leg  Semitendinosus
 Semimembranosus
 Biceps femoris
 Part of adductor magnus

Cutaneous sensation  Posterior aspect of thigh


 Gluteal region
 Entire lower leg (except the medial aspect)

Terminates At the upper part of the popliteal fossa by dividing into the tibial and peroneal nerves

 The nerve to the short head of the biceps femoris comes from the common peroneal part of the sciatic and the other
muscular branches arise from the tibial portion.
 The tibial nerve goes on to innervate all muscles of the foot except the extensor digitorum brevis (which is innervated by
the common peroneal nerve).

29. A 72 year old lady is suspected of having a femoral hernia. At which of the following sites is it most likely to be
identifiable clinically?
A.Mid inguinal point
B. Above and medial to the pubic tubercle
C. Below and lateral to the pubic tubercle
D.Mid point of the inguinal ligament
E. 3 cm superomedially to the superficial inguinal ring

Answer: C

Femoral hernias exit the femoral canal below and lateral to the pubic tubercle. Femoral hernia occur mainly in women due to their
difference in pelvic anatomy. They are at high risk of strangulation and therefore should be repaired.

30. Which of the following represents the root values of the sciatic nerve?
A. L4 to S3
B. L1 to L4
C. L3 to S1
D. S1 to S4
E. L5 to S1
Answer: A

31. The common peroneal nerve, or its branches, supply the following muscles except:
A. Peroneus longus
B. Tibialis anterior
C. Extensor hallucis longus
D. Flexor digitorum brevis
E. Extensor digitorum longus
Answer: D
49
Flexor digitorum is supplied by the tibial nerve.

Common peroneal nerve

Derived from the dorsal divisions of the sacral plexus (L4, L5, S1 and S2).

This nerve supplies the skin and fascia of the anterolateral surface of the leg and the dorsum of the foot. It also innervates the
muscles of the anterior and peroneal compartments of the leg, extensor digitorum brevis as well as the knee, ankle and foot joints.

It is laterally placed within the sciatic nerve. From the bifurcation of the sciatic nerve it passes inferolaterally in the lateral and
proximal part of the popliteal fossa, under the cover of biceps femoris and its tendon. To reach the posterior aspect of the fibular
head. It ends by dividing into the deep and superficial peroneal nerves at the point where it winds around the lateral surface of the
neck of the fibula in the body of peroneus longus, approximately 2cm distal to the apex of the head of the fibula. It is palpable
posterior to the head of the fibula.

Branches
In the thigh Nerve to the short head of biceps
Articular branch (knee)
In the popliteal fossa Lateral cutaneous nerve of the calf
Neck of fibula Superficial and deep peroneal nerves

32. An 83 year old lady presents with a femoral hernia and undergoes a femoral hernia repair. Which of the following
forms the posterior wall of the femoral canal?
A.Pectineal ligament
B. Lacunar ligament
C. Inguinal ligament
D.Adductor longus
E. Sartorius

Answer: A

33. Which of the following structures does not pass behind the lateral malleolus?
A. Peroneus brevis tendon
B. Sural nerve
C. Short saphenous vein
D. Peroneus longus tendon
E. Tibialis anterior tendon
Answer: E
Tibialis anterior tendon passes at the medial malleolus.

Structures posterior to the lateral malleolus and superficial to superior peroneal retinaculum: sural nerve and short
saphenous vein
Structures posterior to the lateral malleolus and deep to superior peroneal retinaculum:peroneus longus tendon and
peroneus brevis tendon

The calcaneofibular ligament is attached at the lateral malleolus

34. Theme: Nerve lesions

A. Iliohypogastric nerve
B. Ilioinguinal nerve
C. Lateral cutaneous nerve of the thigh
D. Femoral nerve
E. Saphenous nerve
F. Genitofemoral nerve

Please select the most likely nerve implicated in the situation described. Each option may be used once, more than once or not at
all.

50
A 42 year old woman complains of a burning pain of her anterior thigh which worsens on walking. There is a positive tinel
sign over the inguinal ligament.
Answer: Lateral cutaneous nerve of the thigh

The lateral cutaneous nerve supplies sensation to the anterior and lateral aspect of the thigh. Entrapment is commonly due to intra
and extra pelvic causes. Treatment involves local anaesthetic injections.

A 29 year old woman has had a Pfannenstiel incision. She has pain over the inguinal ligament which radiates to the lower
abdomen. There is tenderness when the inguinal canal is compressed.

Answer: Ilioinguinal nerve

A 22 year man is shot in the groin. On examination he has weak hip flexion, weak knee extension, and impaired
quadriceps tendon reflex, as well as sensory deficit in the anteromedial aspect of the thigh.

Answer: Femoral nerve

This is a classical description of a femoral nerve injury.

Nerve lesions during surgery


A variety of different procedures carry the risk of iatrogenic nerve injury. These are important not only from the patients
perspective but also from a medicolegal standpoint.

The following operations and their associated nerve lesions are listed here:

 Posterior triangle lymph node biopsy and accessory nerve lesion.


 Lloyd Davies stirrups and common peroneal nerve.
 Thyroidectomy and laryngeal nerve.
 Anterior resection of rectum and hypogastric autonomic nerves.
 Axillary node clearance; long thoracic nerve, thoracodorsal nerve and intercostobrachial nerve.
 Inguinal hernia surgery and ilioinguinal nerve.
 Varicose vein surgery- sural and saphenous nerves.
 Posterior approach to the hip and sciatic nerve.
 Carotid endarterectomy and hypoglossal nerve.

There are many more, with sound anatomical understanding of the commonly performed procedures the incidence of nerve lesions
can be minimised. They commonly occur when surgeons operate in an unfamiliar tissue plane or by blind placement of
haemostats (not recommended).

35. A 23 year old man is stabbed in the groin, several structures are injured and the adductor longus muscle has been
lacerated. Which of the following nerves is responsible for the innervation of adductor longus?
A.Femoral nerve
B. Obturator nerve
C. Sciatic nerve
D.Common peroneal nerve
E. Ilioinguinal nerve
Answer: B

The adductors are innervated by the obturator nerve

Adductor longus
Origin Anterior body of pubis
Insertion Middle third of linea aspera
Action Adducts and flexes the thigh, medially rotate the hip
Innervation Anterior division of obturator nerve (L2, L3, L4)

36. Which of the following muscles does not recieve any innervation from the sciatic nerve?
A. Semimembranosus
B. Quadriceps femoris
C. Biceps femoris
D. Semitendinosus
51
E. Adductor magnus
Answer: B

The sciatic nerve is traditionally viewed as being a nerve of the posterior compartment. It is known to contribute to the innervation
of adductor magnus (although the main innervation to this muscle is from the obturator nerve). The quadriceps femoris is nearly
always innervated by the femoral nerve.

37. The following are true of the femoral nerve except:


A. It is derived from L2, L3 and L4 nerve roots
B. It supplies sartorius
C. It supplies quadriceps femoris
D. It gives cutaneous innervations via the saphenous nerve
E. It supplies adductor longus
Answer: E

Adductor longus is supplied by the obturator nerve.

38. Which of the following is true in relation to the sartorius muscle?


A. Innervated by the deep branch of the femoral nerve
B. Inserts at the fibula
C. It is the shortest muscle in the body
D. Forms the Pes anserinus with Gracilis and semitendinous muscle
E. Causes extension of the knee
Answer: D

It is innervated by the superficial branch of the femoral nerve. It is a component of the pes anserinus.

Sartorius

 Longest strap muscle in the body


 Most superficial muscle in the anterior compartment of the thigh

Origin Anterior superior iliac spine


Insertion Medial surface of the of the body of the tibia (upper part). It inserts anterior to gracilis and semitendinosus
Nerve Supply Femoral nerve (L2,3)
Action  Flexor of the hip and knee, slight abducts the thigh and rotates it laterally
 It assists with medial rotation of the tibia on the femur. For example it would play a pivotal role in
placing the right heel onto the left knee ( and vice versa)

Important The middle third of this muscle, and its strong underlying fascia forms the roof of the adductor canal , in which
relations lie the femoral vessels, the saphenous nerve and the nerve to vastus medialis.

39. Theme: Nerve lesions

A. Sciatic nerve
B. Peroneal nerve
C. Tibial Nerve
D. Obturator nerve
E. Ilioinguinal nerve
F. Femoral nerve
G. None of the above

Please select the most likely nerve injury for the scenario given. Each option may be used once, more than once or not at all

A 56 year old man undergoes a low anterior resection with legs in the Lloyd-Davies position. Post operatively he complains
of foot drop.
Answer: Peroneal nerve
Positioning legs in Lloyd- Davies stirrups can carry the risk of peroneal nerve neuropraxia if not done carefully.

52
A 23 year old man complains of severe groin pain several weeks after a difficult inguinal hernia repair.
Answer: Ilioinguinal nerve
The ilioinguinal nerve may have been entrapped in the mesh causing a neuroma.
A 72 year old man develops a foot drop after a revision total hip replacement.
Answer: Sciatic nerve
This may be done by a number of approaches, in this scenario a posterior approach is the most likely culprit.

40. Which muscle is supplied by the superficial peroneal nerve?


A. Peroneus tertius
B. Sartorius
C. Adductor magnus
D. Peroneus brevis
E. Gracilis
Answer: D
Superficial peroneal nerve

Supplies

 Lateral compartment of leg: peroneus longus, peroneus brevis (action: eversion and plantar flexion)
 Sensation over dorsum of the foot (except the first web space, which is innervated by the deep peroneal nerve)

Path

 Passes between peroneus longus and peroneus brevis along the length of the proximal one third of the fibula
 10-12 cm above the tip of the lateral malleolus, the superficial peroneal nerve pierces the fascia
 6-7 cm distal to the fibula, the superficial peroneal nerve bifurcates into intermediate and medial dorsal cutaneous nerves

41. Which of the following structures separates the posterior cruciate ligament from the popliteal artery?
A.Oblique popliteal ligament
B. Transverse ligament
C. Popliteus tendon
D.Biceps femoris
E. Semitendinosus
Answer: A

The posterior cruciate ligament is separated from the popliteal vessels at its origin by the oblique popliteal ligament. The
transverse ligament is located anteriorly.

Knee joint

The knee joint is a synovial joint, the largest and most complicated. It consists of two condylar joints between the femure and tibia
and a sellar joint between the patella and the femur. The tibiofemoral articular surfaces are incongruent, however, this is improved
by the presence of the menisci. The degree of congruence is related to the anatomical position of the knee joint and is greatest in
full extension.

Knee joint compartments

Tibiofemoral  Comprise of the patella/femur joint, lateral and medial compartments (between femur condyles and
tibia)
 Synovial membrane and cruciate ligaments partially separate the medial and lateral compartments

Patellofemoral  Ligamentum patellae


 Actions: provides joint stability in full extension

53
Fibrous capsule
The capsule of the knee joint is a complex, composite structure with contributions from adjacent tendons.

Anterior The capsule does not pass proximal to the patella. It blends with the tendinous expansions of vastus medialis
fibres and lateralis
Posterior These fibres are vertical and run from the posterior surface of the femoral condyles to the posterior aspect of the
fibres tibial condyle
Medial fibres Attach to the femoral and tibial condyles beyond their articular margins, blending with the tibial collateral
ligament
Lateral fibres Attach to the femur superior to popliteus, pass over its tendon to head of fibula and tibial condyle

Bursae

Anterior  Subcutaneous prepatellar bursa; between patella and skin


 Deep infrapatellar bursa; between tibia and patellar ligament
 Subcutaneous infrapatellar bursa; between distal tibial tuberosity and skin

Laterally  Bursa between lateral head of gastrocnemius and joint capsule


 Bursa between fibular collateral ligament and tendon of biceps femoris
 Bursa between fibular collateral ligament and tendon of popliteus

Medially  Bursa between medial head of gastrocnemius and the fibrous capsule
 Bursa between tibial collateral ligament and tendons of sartorius, gracilis and semitendinosus
 Bursa between the tendon of semimembranosus and medial tibial condyle and medial head of gastrocnemius

Posterior Highly variable and inconsistent

Ligaments

 Medial collateral ligament: Medial epicondyle femur to medial tibial condyle: valgus stability
 Lateral collateral ligament: Lateral epicondyle femur to fibula head: varus stability
 Anterior cruciate ligament: Anterior tibia to lateral intercondylar notch femur: prevents tibia sliding anteriorly
 Posterior cruciate ligament: Posterior tibia to medial intercondylar notch femur: prevents tibia sliding posteriorly
 Patellar ligament: Central band of the tendon of quadriceps femoris, extends from patella to tibial tuberosity

Menisci
Medial and lateral menisci compensate for the incongruence of the femoral and tibial condyles.
Composed of fibrous tissue.
Medial meniscus is attached to the tibial collateral ligament.
Lateral meniscus is attached to the loose fibres at the lateral edge of the joint and is separate from the fibular collateral ligament.
The lateral meniscus is crossed by the popliteus tendon.

Nerve supply
The knee joint is supplied by the femoral, tibial and common peroneal divisions of the sciatic and by a branch from the obturator
nerve. Hip pathology pain may be referred to the knee.

Blood supply
Genicular branches of the femoral artery, popliteal and anterior tibial arteries all supply the knee joint.

42. A 25 year old man undergoes an excision of a pelvic chondrosarcoma, during the operation the obturator nerve is
sacrificed. Which of the following muscles is least likely to be affected as a result?
A. Adductor longus
B. Pectineus
C. Adductor magnus
D. Sartorius
E. Gracilis
Answer: D

Sartorius is supplied by the femoral nerve. In approximately 20% of the population, pectineus is supplied by the accessory
obturator nerve.

54
43. Which nerve supplies the 1st web space of the foot?
A. Popliteal nerve
B. Superficial peroneal nerve
C. Deep peroneal nerve
D. Tibial nerve
E. Saphenous nerve
Answer: C
The first web space is innervated by the deep peroneal nerve.
44. Which of the following structures does not pass behind the piriformis muscle in the greater sciatic foramen?
A.Sciatic nerve
B. Posterior cutaneous nerve of the thigh
C. Inferior gluteal artery
D.Obturator nerve
E. None of the above
Answer: D
The obturator nerve does not pass through the greater sciatic foramen.

45. A 78 year old lady falls over in her nursing home and sustains a displaced intracapsular fracture of the femoral neck.
A decision is made to perform a hemi arthroplasty through a lateral approach. Which of the following vessels will be
divided to facilitate access?
A.Saphenous vein
B. Superior gluteal artery
C. Superficial circumflex iliac artery
D.Profunda femoris artery
E. Transverse branch of the lateral circumflex artery
Answer: E
During the Hardinge style lateral approach the transverse branch of the lateral circumflex artery is divided to gain access.

46. A 72 year old lady with osteoporosis falls and sustains an intracapsular femoral neck fracture. The fracture is
completely displaced. Which of the following vessels is the main contributor to the arterial supply of the femoral head?
A.Deep external pudendal artery
B. Superficial femoral artery
C. External iliac artery
D.Circumflex femoral arteries
E. Superficial external pudendal artery
Answer: D

The vessels which form the anastomoses around the femoral head are derived from the medial and lateral circumflex femoral
arteries. These are usually derived from the profunda femoris artery.

47. The following statements relating to the ankle joint are true except?
A. Three groups of ligaments provide mechanical stability
B. The sural nerve lies medial to the Achilles tendon at its point of insertion
C. Eversion of the foot occurs at the sub talar joint
D. The flexor hallucis longus tendon is the most posterior structure at the medial malleolus
E. The saphenous nerve crosses the ankle joint.

Answer: B

The sural nerve lies behind the distal fibula. Inversion and eversion are sub talar movements. The structures passing behind the
medial malleolus from anterior to posterior include: tibialis posterior, flexor digitorum longus, posterior tibia vein, posterior tibial
artery, nerve, flexor hallucis longus.

Ankle joint: The ankle joint is a synovial joint composed of the tibia and fibula superiorly and the talus inferiorly.

Ligaments of the ankle joint

 Deltoid ligament (medially)

55
 Lateral collateral ligament
 Talofibular ligaments (both anteriorly and posteriorly)

The calcaneofibular ligament is separate from the fibrous capsule of the joint. The two talofibular ligaments are fused with it.

The components of the syndesmosis are

 Antero-inferior talofibular ligament


 Postero-inferior talofibular ligament
 Inferior transverse talofibular ligament
 Interosseous ligament

Movements at the ankle joint

 Plantar flexion (55 degrees)


 Dorsiflexion (35 degrees)
 Inversion and eversion movements occur at the level of the sub talar joint

Nerve supply: Branches of deep peroneal and tibial nerves.

48. A 19 year old man is playing rugby when he suddenly notices a severe pain at the posterolateral aspect of his right
thigh. Which of the following muscle groups is most likely to have been injured?
A.Semimembranosus
B. Semitendinosus
C. Long head of biceps femoris
D.Gastrocnemius
E. Soleus
Answer: C

The biceps femoris is the laterally located hamstring muscle. The semitendinosus and semimembranosus are located medially.
Rupture of gastrocnemius and soleus may occur but is less common.

49. A 22 year old man is involved in a fight and is stabbed in the posterior aspect of his right leg. The knife passes into the
popliteal fossa. He sustains an injury to his tibial nerve. Which of the following muscles is least likely to be
compromised as a result?
A. Tibialis posterior
B. Flexor hallucis longus
C. Flexor digitorum brevis
D. Soleus
E. Peroneus tertius
Answer: E

Peroneus tertius is innervated by the deep peroneal nerve.

Tibial nerve
Begins at the upper border of the popliteal fossa and is a branch of the sciatic nerve. Root values: L4, L5, S1, S2, S3

Muscles innervated: Popliteus, Gastrocnemius, Soleus, Plantaris, Tibialis posterior, Flexor hallucis longus and Flexor digitorum
brevis. Terminates by dividing into the medial and lateral plantar nerves.

50. At which of the following anatomical locations does the common peroneal nerve bifurcate into the superficial and deep
peroneal nerves?
A. Immediately anterior to the linea aspera
B. At the lateral aspect of the neck of the fibula
C. Within the substance of tibialis anterior muscle
D. At the inferomedial aspect of the popliteal fossa
56
E. Under the medial head of gastrocnemius

Answer: B

The common peroneal nerve bifurcates at the neck of the fibula (where it is most likely to be injured).

51. A 48 year old motor cyclist sustains a complex lower limb fracture in a motor accident. For a time the popliteal artery
is occluded and eventually repaired. Subsequently he develops a compartment syndrome and the anterior and
superficial posterior compartments of the lower leg are decompressed. Unfortunately, the operating surgeon neglects
to decompress the deep posterior compartment. Which of the following muscles is least likely to be affected as a
result?
A. Flexor digitorum longus
B. Plantaris
C. Tibialis posterior
D. Flexor hallucis longus
E. None of the above
Answer: B
The plantaris muscle lies within the superficial posterior compartment of the lower leg.

52. A 23 year old lady is undergoing a trendelenberg procedure for varicose veins. During the dissection of the
saphenofemoral junction, which of the following is most liable to injury?
A. Superficial circumflex iliac artery
B. Superficial circumflex iliac vein
C. Femoral artery
D. Femoral nerve
E. Deep external pudendal artery

Answer: E
The deep external pudendal artery runs under the long saphenous vein close to its origin and may be injured. It is at greatest risk
of injury during the flush ligation of the saphenofemoral junction. Provided an injury is identified and vessel ligated, injury is
seldom associated with any serious adverse sequelae.

53. A 52 year female post hysterectomy attends clinic. She reports pain and reduced sensation over the medial aspect of
her thigh. Clinically thigh adduction is weak. What is the most likely nerve injury?
A.Obturator nerve
B. Sciatic nerve
C. Femoral nerve
D.L3 cord compression
E. Deep peroneal nerve
Answer: A

The obturator nerve supplies sensation to the medial aspect of the thigh and causes adduction and internal rotation of the thigh.
Injury occurs during pelvic or abdominal surgery.
L3 cord compression is unlikely.

54. With regard to the sciatic nerve which statement is false?


A. It is derived from L4 to S3
B. It contains nerve roots from the posterior division of the lumbosacral plexus only
C. It divides to give the tibial nerve and common peroneal nerve
D. It provides cutaneous sensation to the posterior aspect of the thigh
E. It provides cutaneous sensation to the entire lower leg with the exception of its medial aspect

Answer: B

It is derived from both anterior and posterior divisions of the lumbosacral plexus. The sciatic nerve is the longest and widest nerve
in the human body. It is particularly susceptible to trauma in the posterior approach to the hip.

55. Which of the following is a content of the adductor canal?


A.Saphenous nerve
B. Sural nerve
C. Femoral nerve
D.Profunda branch of the femoral artery
57
E. Saphenous vein
Answer: A

It contains the saphenous nerve and the superficial branch of the femoral artery.

56. A 56 year old lady with metastatic breast cancer develops an oestolytic deposit in the proximal femur. One morning
whilst getting out of bed she notices severe groin pain. X-rays show that the lesser trochanter has been avulsed. Which
muscle is the most likely culprit?
A. Vastus lateralis
B. Psoas major
C. Piriformis
D. Gluteus maximus
E. Gluteus medius

Answer: B

The psoas major inserts into the lesser trochanter and contracts when raising the trunk from the supine position. When oestolytic
lesions are present in the femur the lesser trochanter may be avulsed.

Psoas Muscle

Origin
The deep part originates from the transverse processes of the five lumbar vertebrae, the superficial part originates from T12 and
the first 4 lumbar vertebrae.
Insertion
Lesser trochanter of the femur.
Innervation
Anterior rami of L1 to L3.
Action
Flexion and external rotation of the hip. Bilateral contraction can raise the trunk from the supine position.

57. A 34 year old man is injured by farm machinery and sustains a laceration at the superolateral aspect of the popliteal
fossa. The medial aspect of biceps femoris is lacerated. Which of the following underlying structures is at greatest risk
of injury?
A. Gracilis
B. Sural nerve
C. Nerve to semimembranosus
D. Popliteal artery
E. Common peroneal nerve

Answer: E

The common peroneal nerve lies under the medial aspect of biceps femoris and is therefore at greatest risk of injury. The tibial
nerve may also be damaged in such an injury (but is not listed here). The sural nerve branches off more inferiorly.

58. A laceration to the upper lateral margin of the popliteal fossa may injure which of the following nerves?
A.Common peroneal nerve
B. Sural nerve
C. Sciatic nerve
D.Saphenous nerve
E. Tibial nerve
Answer: A

The sural nerve exits at the lower latero-medial aspect of the fossa and is more at risk in short saphenous vein surgery. The tibial
nerve lies more medially and is even less likely to be injured in this location.

59. An elderly lady falls and lands on her hip. On examination her hip is tender to palpation and x-rays are taken. There
are concerns that she may have an intertrochanteric fracture. What is the normal angle between the femoral neck and
the femoral shaft?
58
A.90o
B. 105o
C. 80o
D.130o
E. 180o
Answer: D

The normal angle between the femoral head and shaft is 130o. Changes to this angle may occur as a result of disease or pathology
and should be investigated.

60. An 18 year old athlete attends orthopaedic clinic reporting pain and swelling over the medial aspect of the knee joint.
The pain occurs when climbing the stairs, but is not present when walking on flat ground. Clinically there is pain over
the medial, proximal tibia and the McMurray test is negative. What is the most likely cause of this patient's
symptoms?
A. Anterior cruciate ligament tear
B. Prepatellar bursitis
C. Medial meniscus injury
D. Pes Anserinus Bursitis
E. Fracture of tibia
Answer: D
Pes anserinus: GOOSE'S FOOT: Combination of sartorius, gracilis and semitendinous tendons inserting into the anteromedial
proximal tibia. Pes Anserinus Bursitis is common in sportsmen due to overuse injuries. The main sign is of pain in the medial
proximal tibia. As the McMurray test is negative, medial meniscal injury is excluded.

61. Which of the following nerves innervates the long head of the biceps femoris muscle?
A.Inferior gluteal nerve
B. Tibial nerve
C. Superior gluteal nerve
D.Common peroneal nerve
E. Obturator nerve
Answer: B

The short head of biceps femoris, which may occasionally be absent, is innervated by the common peroneal component of the
sciatic nerve. The long head is innervated by the tibial nerve.

62. Which of the following bones is related to the cuboid at its distal articular surface?
A.All metatarsals
B. 5th metatarsal
C. Calcaneum
D.Medial cuneiform
E. 3rd metatarsal
Answer: B

The cuboid is located at the lateral aspect of the foot between the calcaneus posteriorly and the 4th and 5th metatarsals distally.

63. A 40-year-old man presents with pain in his lower back and 'sciatica' for the past three days. He describes bending
down to pick up a washing machine when he felt 'something go'. He now has severe pain radiating from his back down
the right leg. On examination he describes paraesthesia over the anterior aspect of the right knee and the medial
aspect of his calf. Power is intact and the right knee reflex is diminished. The femoral stretch test is positive on the
right side. Which nerve or nerve root is most likely to be affected?
A. Common peroneal nerve
B. Lateral cutaneous nerve of the thigh
C. L5
D. L3
E. L4

Answer: E

59
Prolapsed disc
A prolapsed lumbar disc usually produces clear dermatomal leg pain associated with neurological deficits.
Features

 Leg pain usually worse than back


 Pain often worse when sitting

The table below demonstrates the expected features according to the level of compression:

L3 nerve root compression Sensory loss over anterior thigh


Weak quadriceps
Reduced knee reflex
Positive femoral stretch test
L4 nerve root compression Sensory loss anterior aspect of knee
Weak quadriceps
Reduced knee reflex
Positive femoral stretch test
L5 nerve root compression Sensory loss dorsum of foot
Weakness in foot and big toe dorsiflexion
Reflexes intact
Positive sciatic nerve stretch test
S1 nerve root compression Sensory loss posterolateral aspect of leg and lateral aspect of foot
Weakness in plantar flexion of foot
Reduced ankle reflex
Positive sciatic nerve stretch test

Management

 Similar to that of other musculoskeletal lower back pain: analgesia, physiotherapy, exercises
 Persistent symptoms, muscular weakness, bladder or bowel dysfunction are indications for urgent MRI scanning to
delineate the disease extent to allow surgical planning
 Plain spinal x-rays have no useful role in establishing the extent of disk disease

64. Onto which of the following structures does the anterior cruciate ligament insert?
A.Posterolateral aspect of the lateral femoral condyle
B. Posteromedial aspect of the lateral femoral condyle
C. Posterolateral aspect of the medial femoral condyle
D.Posteromedial aspect of the medial femoral condyle
E. None of the above

Answer: B

The anterior cruciate ligament is attached to the anterior intercondylar area of the tibia. Is then passes posterolaterally to insert into
the posteromedial aspect of the lateral femoral condyle.

65. A 40 year old lady presents with varicose veins, these are found to originate from the short saphenous vein. As the vein
is mobilised close to its origin which of the following structures is at greatest risk of injury?
A.Sciatic nerve
B. Sural nerve
C. Common peroneal nerve
D.Tibial nerve
E. Popliteal artery

Answer; B

The sural nerve is closely related and damage to this structure is a major cause of litigation. The other structures may all be injured
but the risks are lower.

60
66. A 72 year old man with non reconstructible arterial disease is undergoing an above knee amputation. The posterior
compartment muscles are divided. Which of the following muscles does not lie in the posterior compartment of the
thigh?
A. Biceps femoris
B. Quadriceps femoris
C. Semitendinosus
D. Semimembranosus
E. None of the above

Answer: B

The quadriceps femoris lies in the anterior compartment.

Compartments of the thigh

Formed by 3 septae passing from the femur to the fascia lata.


Compartment Nerve Muscles Blood supply
 Iliacus
 Tensor fasciae latae
Anterior compartment Femoral  Sartorius Femoral artery
 Quadriceps femoris

 Adductor longus/magnus/brevis
 Gracilis
Medial compartment Obturator Profunda femoris artery and obturator artery
 Obturator externus

 Semimembranosus
 Semitendinosus
Posterior compartment Sciatic Branches of Profunda femoris artery
 Biceps femoris

Which of the following structures is not closely related to the posterior tibial artery?

F. Soleus posteriorly
G. Tibial nerve medially
H. Deep peroneal nerve laterally
I. Flexor hallucis longus postero-inferiorly
J. Popliteus

Answer: C

The deep peroneal nerve lies in the anterior compartment. The tibial nerve lies medially. At its termination it lies deep to the
flexor retinaculum.

67. A 30 year old man presents with back pain and the surgeon tests the ankle reflex. Which of the following nerve roots
are tested in this manoeuvre?
A. S3 and S4
B. L4 and L5
C. L3 and L4
D. S1 and S2
E. S4 only

Answer: D

Ankle reflex

61
The ankle reflex is elicited by tapping the Achilles tendon with a tendon hammer. It tests the S1 and S2 nerve roots. It is typically
delayed in L5 and S1 disk prolapses.

68. Which of the following structures is not closely related to the piriformis muscle?
A. Superior gluteal nerve
B. Sciatic nerve
C. Inferior gluteal artery
D. Inferior gluteal nerve
E. Medial femoral circumflex artery
Answer: E
Nerve supply of lateral hip rotators

Piriformis: ventral rami S1, S2


Obturator internus: nerve to obturator internus
Superior gemellus: nerve to obturator internus
Inferior gemellus: nerve to quadratus femoris
Quadrator femoris: nerve to quadrator femoris

The piriformis muscle is an important anatomical landmark in the gluteal region. The following structures are closely related:

 Sciatic nerve
 Inferior gluteal artery and nerve
 Superior gluteal artery and nerve

The medial femoral circumflex artery runs deep to quadratus femoris.

Gluteal muscles

 Gluteus maximus: inserts to gluteal tuberosity of the femur and iliotibial tract
 Gluteus medius: attach to lateral greater trochanter
 Gluteus minimis: attach to anterior greater trochanter
 All extend and abduct the hip

Deep lateral hip rotators

 Piriformis
 Gemelli
 Obturator internus
 Quadratus femoris

Nerves
Superior gluteal nerve (L5, S1)  Gluteus medius
 Gluteus minimis
 Tensor fascia lata

Inferior gluteal nerve Gluteus maximus


Damage to the superior gluteal nerve will result in the patient developing a Trendelenberg gait. Affected patients are unable to
abduct the thigh at the hip joint. During the stance phase, the weakened abductor muscles allow the pelvis to tilt down on the
opposite side. To compensate, the trunk lurches to the weakened side to attempt to maintain a level pelvis throughout the gait
cycle. The pelvis sags on the opposite side of the lesioned superior gluteal nerve.

69. A 77 year old man with symptoms of intermittent claudication is due to have his ankle brachial pressure indices
measured. The vessel is impalpable. Which of the following tendinous structures lies medial to it, that may facilitate its
identification?
A.Extensor digitorum longus tendon
B. Peroneus tertius tendon
C. Extensor hallucis longus tendon
D.Extensor digitorum brevis tendon
62
E. Flexor digitorum longus tendon
Answer: C

The extensor hallucis longus tendon lies medial to the dorsalis pedis artery.

70. The integrity of which muscle is assessed by the Trendelenberg test?


A. Sartorius
B. Quadratus femoris
C. Semimembranosus
D. Gluteus medius
E. Piriformis

Answer: D

71. A 78 year old man presents with symptoms consistent with intermittent claudication. To assess the severity of his
disease you decide to measure his ankle brachial pressure index. To do this you will identify the dorsalis pedis artery.
Which of the following statements relating to this vessel is false?
A.It originates from the peroneal artery
B. It is crossed by the tendon of extensor hallucis brevis
C. Two veins are usually closely related to it
D.It passes under the inferior extensor retinaculum
E. The tendon of extensor hallucis longus lies medial to it.

Answer: A

The dorsalis pedis artery is a direct continuation of the anterior tibial artery.

72. Which of the following statements relating to the knee joint is false?
A.It is the largest synovial joint in the body
B. When the knee is fully extended all ligaments of the knee joint are taut
C. Rupture of the anterior cruciate ligament may result in haemarthrosis
D.The posterior aspect of the patella is extrasynovial
E. The joint is innervated by the femoral, sciatic and obturator nerves
Answer: D

The posterior aspect is intrasynovial and the knee itself comprises the largest synovial joint in the body. It may swell considerably
following trauma such as ACL injury. Which may be extremely painful owing to rich innervation from femoral, sciatic and ( a
smaller) contribution from the obturator nerve. During full extension all ligaments are taut and the knee is locked.

73. Which of the following does not exit the pelvis through the greater sciatic foramen?
A. Superior gluteal artery
B. Internal pudendal vessels
C. Sciatic nerve
D. Obturator nerve
E. Inferior gluteal nerve

Answer: D

The obturator nerve exits through the obturator foramen.

74. A 78 year old man is undergoing a femoro-popliteal bypass graft. The operation is not progressing well and the
surgeon is complaining of poor access. Retraction of which of the following structures will improve access to the
femoral artery in the groin?
A.Quadriceps
B. Adductor longus
C. Adductor magnus
D.Pectineus
E. Sartorius
Answer: E

63
At the lower border of the femoral triangle the femoral artery passes under the sartorius muscle. This can be retracted to improve
access.

75. A builder falls off a ladder whilst laying roof tiles. He sustains a burst fracture of L3. The MRI scan shows complete
cord transection at this level as a result of the injury. Which clinical sign will not be present?
A. Flaccid paralysis of the legs
B. Extensor plantar response
C. Sensory loss in the legs
D. Incontinence
E. Loss of patellar tendon reflex
Answer: B

In lower motor neuron lesions everything is reduced

The main purpose of this question is to differentiate the features of an UMN lesion and a LMN lesion. The features of a LMN
lesion include:

 Flaccid paralysis of muscles supplied


 Atrophy of muscles supplied.
 Loss of reflexes of muscles supplied.
 Muscles fasciculation

For lesions below L1 LMN signs will occur. Hence in an L3 lesion, there will be loss of the patella reflex but there will be no
extensor plantar reflex.

Spinal cord

 Located in a canal within the vertebral column that affords it structural support.
 Rostrally is continues to the medulla oblongata of the brain and caudally it tapers at a level corresponding to the L1-2
interspace (in the adult), a central structure, the filum terminale anchors the cord to the first coccygeal vertebra.
 The spinal cord is characterised by cervico-lumbar enlargements and these, broadly speaking, are the sites which
correspond to the brachial and lumbar plexuses respectively.

There are some key points to note when considering the surgical anatomy of the spinal cord:

 During foetal growth the spinal cord becomes shorter than the spinal canal, hence the adult site of cord termination at the
L1-2 level.
 Due to growth of the vertebral column the spine segmental levels may not always correspond to bony landmarks as they
do in the cervical spine.
 The spinal cord is incompletely divided into two symmetrical halves by a dorsal median sulcus and ventral median
fissure. Grey matter surrounds a central canal that is continuous rostrally with the ventricular system of the CNS.
 The grey matter is sub divided cytoarchitecturally into Rexeds laminae.
 Afferent fibres entering through the dorsal roots usually terminate near their point of entry but may travel for varying
distances in Lissauers tract. In this way they may establish synaptic connections over several levels
 At the tip of the dorsal horn are afferents associated with nociceptive stimuli. The ventral horn contains neurones that
innervate skeletal muscle.

The key point to remember when revising CNS anatomy is to keep a clinical perspective in mind. So it is worth classifying
the ways in which the spinal cord may become injured. These include:

 Trauma either direct or as a result of disc protrusion


 Neoplasia either by direct invasion (rare) or as a result of pathological vertebral fracture
 Inflammatory diseases such as Rheumatoid disease, or OA (formation of osteophytes compressing nerve roots etc.
 Vascular either as a result of stroke (rare in cord) or as complication of aortic dissection
 Infection historically diseases such as TB, epidural abscesses.

The anatomy of the cord will, to an extent dictate the clinical presentation. Some points/ conditions to remember:

64
 Brown- Sequard syndrome-Hemisection of the cord producing ipsilateral loss of proprioception and upper motor neurone
signs, plus contralateral loss of pain and temperature sensation. The explanation of this is that the fibres decussate at
different levels.
 Lesions below L1 will tend to present with lower motor neurone signs

76. A 66 year old man with peripheral vascular disease is undergoing a below knee amputation. In which of the lower leg
compartments does peroneus brevis lie?
A. Lateral compartment
B. Anterior compartment
C. Superficial posterior compartment
D. Deep posterior compartment
E. None of the above

Answer: A

The interosseous membrane separates the anterior and posterior compartments. The deep and superficial compartments are
separated by the deep transverse fascia. The peroneus brevis is part of the lateral compartment.

77. A 70 year old man is due to undergo an arterial bypass procedure for claudication and foot ulceration. The anterior
tibial artery will form the site of the distal arterial anastomosis. Which of the following structures is not closely related
to it?
A. Interosseous membrane
B. Deep peroneal nerve
C. Tibialis posterior
D. Extensor hallucis longus
E. Dorsalis pedis artery
Answer: C
As an artery of the anterior compartment, the anterior tibial artery is closely related to tibialis anterior.

Anterior tibial artery

 Begins opposite the distal border of popliteus


 Terminates in front of the ankle, continuing as the dorsalis pedis artery
 As it descends it lies on the interosseous membrane, distal part of the tibia and front of the ankle joint
 Passes between the tendons of flexor digitorum and flexor hallucis longus distally
 It is related to the deep peroneal nerve, it lies anterior to the middle third of the vessel and lateral to it in the lower third

78. Which of the following muscles does not cause lateral rotation of the hip?
A. Obturator internus
B. Quadratus femoris
C. Gemellus inferior
D. Piriformis
E. Pectineus
Answer: E

Mnemonic lateral hip rotators: P-GO-GO-Q (top to bottom): Piriformis, Gemellus superior, Obturator internus, Gemellus inferior,
Obturator externus, Quadratus femoris. Pectineus adducts and medially rotates the femur.

79. Which of the following structures does not pass anterior to the lateral malleolus?
A. Anterior tibial artery
B. Extensor digitorum longus
C. Tibialis anterior
D. Peroneus brevis
E. Peroneus tertius
Answer: D
Peroneus brevis passes posterior to the lateral malleolus.

80. Which of the following statements relating to sartorius is untrue?


A. It is supplied by the femoral nerve
B. It forms the lateral boundary of the femoral triangle
C. The middle third forms the roof of the adductor canal
65
D. It is a flexor of the hip and knee
E. It inserts into the medial femoral condyle

Ansewr: E

It inserts into the medial aspect of the upper part of the tibia.

81. Which of the following structures are not closely related to the adductor longus muscle?
A. Long saphenous vein
B. Tendon of iliacus
C. The profunda branch of the femoral artery
D. Pectineus muscle
E. Femoral nerve
Answer: B

Femoral triangle:Adductor longus medially; Inguinal ligament superiorly; Sartorius muscle laterallyAdductor longus forms the
medial border of the femoral triangle. It is closely related to the long saphenous vein which overlies it and the profunda branch of
the femoral artery. The femoral nerve is related to it inferiorly. However, the tendon of iliacus inserts proximally and is not in
contact with adductor longus.

82. Which of the following muscles is not within the posterior compartment of the lower leg?
A. Peroneus brevis
B. Flexor digitalis longus
C. Soleus
D. Popliteus
E. Flexor hallucis longus
Answer: A
Peroneus brevis lies in the lateral compartment.

83. Which of the following actions will be impaired?


A.Extension of the great toe
B. Adduction of the thigh
C. Flexion of the knee joint
D.Extension of the knee joint
E. Eversion of the foot
Answer: D

The femoral nerve supplies the quadriceps muscle which is responsible for extension at the knee joint.

84. Which of the following structures are at risk of direct injury following a fracture dislocation of the femoral condyles?
A.Popliteal artery
B. Sciatic nerve
C. Plantaris muscle
D.Tibial artery
E. Tibial nerve
Answer: A

The heads of gastrocnemius will contract to pull the fracture segment posteriorly. The popliteal artery lies against the bone and
may be damaged or compressed.

66
Abdomen
85. A 56 year old man is undergoing a radical nephrectomy via a posterior approach. Which of the following
structures is most likely to be encountered during the operative approach?

A. 8th rib
B. 10th rib

C. 6th rib
D. 12th rib

E. 9th rib

Answer: D

The 11th and 12th ribs lie posterior to the kidneys and may be encountered during a posterior approach. A
pneumothorax is a recognised complication of this type of surgery.

Renal anatomy

Each kidney is about 11cm long, 5cm wide and 3cm thick. They are located in a deep gutter alongside the projecting
verterbral bodies, on the anterior surface of psoas major. In most cases the left kidney lies approximately 1.5cm higher
than the right. The upper pole of both kidneys approximates with the 11th rib (beware pneumothorax during
nephrectomy). On the left hand side the hilum is located at the L1 vertebral level and the right kidney at level L1-2.
The lower border of the kidneys is usually alongside L3.

The table below shows the anatomical relations of the kidneys:

Relations
Relations Right Kidney Left Kidney
Posterior Quadratus lumborum, diaphragm, psoas major, Quadratus lumborum, diaphragm, psoas major,
transversus abdominis transversus abdominis
Anterior Hepatic flexure of colon Stomach, Pancreatic tail
Superior Liver, adrenal gland Spleen, adrenal gland

Fascial covering
Each kidney and suprarenal gland is enclosed within a common and layer of investing fascia that is derived from the
transversalis fascia into anterior and posterior layers (Gerotas fascia).

Renal structure
Kidneys are surrounded by an outer cortex and an inner medulla which usually contains between 6 and 10 pyramidal
structures. The papilla marks the innermost apex of these. They terminate at the renal pelvis, into the ureter.
Lying in a hollow within the kidney is the renal sinus. This contains:
1. Branches of the renal artery
2. Tributaries of the renal vein
3. Major and minor calyces's
4. Fat

Structures at the renal hilum


The renal vein lies most anteriorly, then renal artery (it is an end artery) and the ureter lies most posterior.

67
86. A 44 year old lady is undergoing an abdominal hysterectomy and the ureter is identified during the ligation
of the uterine artery. At which site does it insert into the bladder?

A. Posterior
B. Apex

C. Anterior
D. Base

E. Superior aspect of the lateral side

Answer: D

The ureters enter the bladder at the upper lateral aspect of the base of the bladder. They are about 5cm apart from each
other in the empty bladder. Internally this aspect is contained within the bladder trigone.

Ureter
 25-35 cm long
 Muscular tube lined by transitional epithelium
 Surrounded by thick muscular coat. Becomes 3 muscular layers as it crosses the bony pelvis
 Retroperitoneal structure overlying transverse processes L2-L5
 Lies anterior to bifurcation of iliac vessels
 Blood supply is segmental; renal artery, aortic branches, gonadal branches, common iliac and internal
iliac
 Lies beneath the uterine artery

87. A 6 month old child is brought to the surgical clinic because of non descended testes. What is the main
structure that determines the descent path of the testicle?
A. Processus vaginalis
B. Cremaster
C. Mesorchium
D.Inguinal canal
E. Gubernaculum

Answer: E
The gubernaculum is a ridge of mesenchymal tissue that connects the testis to the inferior aspect of the scrotum. Early
in embryonic development the gubernaculum is long and the testis are located on the posterior abdominal wall. During
foetal growth the body grows relative to the gubernaculum, with resultant descent of the testis.

Testicular embryology
Until the end of foetal life the testicles are located within the abdominal cavity. They are initially located on the
posterior abdominal wall on a level with the upper lumbar vertebrae (L2). Attached to the inferior aspect of the testis
is the gubernaculum testis which extends caudally to the inguinal region, through the canal and down to the superficial
skin. Both the testis and the gubernaculum are extra-peritoneal.
As the foetus grows the gubernaculum becomes progressively shorter. It carries the peritoneum of the anterior
abdominal wall (the processus vaginalis). As the processus vaginalis descends the testis is guided by the
gubernaculum down the posterior abdominal wall and the back of the processus vaginalis into the scrotum. By the
third month of foetal life the testes are located in the iliac fossae, by the seventh they lie at the level of the deep
inguinal ring. The processus vaginalis usually closes after birth, but may persist and be the site of indirect hernias. Part
closure may result in development of cysts on the cord.

68
88. A 28 year old man requires a urethral catheter to be inserted prior to undergoing a splenectomy. Where is
the first site of resistance to be encountered on inserting the catheter?
a. Bulbar urethra
b. Membranous urethra
c. Internal sphincter
d. Prostatic urethra
e. Bladder neck

Answer: B

The membranous urethra is the least distensible portion of the urethra. This is due to the fact that it is surrounded by
the external sphincter.

Urethral anatomy
Female urethra
The female urethra is shorter and more acutely angulated than the male urethra. It is an extra-peritoneal structure and
embedded in the endopelvic fascia. The neck of the bladder is subjected to transmitted intra-abdominal pressure and
therefore deficiency in this area may result in stress urinary incontinence. Between the layers of the urogenital
diaphragm the female urethra is surrounded by the external urethral sphincter, this is innervated by the pudendal
nerve. It ultimately lies anterior to the vaginal orifice.

Male urethra
In males the urethra is much longer and is divided into four parts.

Pre-prostatic Extremely short and lies between the bladder and prostate gland.It has a stellate lumen and is
urethra between 1 and 1.5cm long.Innervated by sympathetic noradrenergic fibres, as this region is
composed of striated muscles bundles they may contract and prevent retrograde ejaculation.
Prostatic urethra This segment is wider than the membranous urethra and contains several openings for the
transmission of semen (at the midpoint of the urethral crest).
Membranous Narrowest part of the urethra and surrounded by external sphincter. It traverses the perineal
urethra membrane 2.5cm postero-inferior to the symphysis pubis.
Penile urethra Travels through the corpus songiosum on the underside of the penis. It is the longest urethral
segment.It is dilated at its origin as the infrabulbar fossa and again in the gland penis as the
navicular fossa. The bulbo-urethral glands open into the spongiose section of the urethra 2.5cm
below the perineal membrane.
The urothelium is transitional in nature near to the bladder and becomes squamous more distally.

89. A 23 year old man undergoes an orchidectomy. The right testicular vein is ligated; into which structure
does it drain?
A. Right renal vein
B. Inferior vena cava
C. Common iliac vein
D. Internal iliac vein
E. External iliac vein

Answer: B

The testicular venous drainage begins in the septa and these veins together with those of the tunica vasculosa converge
on the posterior border of the testis as the pampiniform plexus. The pampiniform plexus drains to the testicular vein.
The left testicular vein drains into the left renal vein. The right testicular vein drains into the inferior vena cava.
69
Scrotal and testicular anatomy

Spermatic cord
Formed by the vas deferens and is covered by the following structures:
Layer Origin
Internal spermatic fascia Transversalis fascia
Cremasteric fascia From the fascial coverings of internal oblique
External spermatic fascia External oblique aponeurosis

Contents of the cord


Vas deferens Transmits sperm and accessory gland secretions
Testicular artery Branch of abdominal aorta supplies testis and epididymis
Artery of vas deferens Arises from inferior vesical artery
Cremasteric artery Arises from inferior epigastic artery
Pampiniform plexus Venous plexus, drains into right or left testicular vein
Sympathetic nerve fibres Lie on arteries, the parasympathetic fibres lie on the vas
Genital branch of the genitofemoral nerve Supplies cremaster
Lymphatic vessels Drain to lumbar and para-aortic nodes

Scrotum

 Composed of skin and closely attached dartos fascia.


 Arterial supply from the anterior and posterior scrotal arteries
 Lymphatic drainage to the inguinal lymph nodes
 Parietal layer of the tunica vaginalis is the innermost layer

Testes

 The testes are surrounded by the tunica vaginalis (closed peritoneal sac). The parietal layer of the tunica
vaginalis adjacent to the internal spermatic fascia.
 The testicular arteries arise from the aorta immediately inferiorly to the renal arteries.
 The pampiniform plexus drains into the testicular veins, the left drains into the left renal vein and the right
into the inferior vena cava.
 Lymphatic drainage is to the para-aortic nodes.

90. A 21 year old man has an inguinal hernia and is undergoing a surgical repair. As the surgeons approach
the inguinal canal they expose the superficial inguinal ring. Which of the following forms the lateral edge of
this structure?
A.Inferior epigastric artery
B. Conjoint tendon
C. Rectus abdominis muscle
D.External oblique aponeurosis
E. Transversalis fascia

Answer: D

The external oblique aponeurosis forms the anterior wall of the inguinal canal and also the lateral edge of the
superficial inguinal ring. The rectus abdominis lies posteromedially and the transversalis posterior to this.

Inguinal canal

Location

70
 Above the inguinal ligament
 The inguinal canal is 4cm long

Boundaries of the inguinal canal


Floor  External oblique aponeurosis
 Inguinal ligament
 Lacunar ligament

Roof  Internal oblique


 Transversus abdominis

Anterior wall External oblique aponeurosis


Posterior wall  Transversalis fascia
 Conjoint tendon

Laterally  Internal ring


 Fibres of internal oblique

Medially  External ring


 Conjoint tendon

Contents
Males Spermatic cord and ilioinguinal nerve As it passes through the canal the spermatic cord has 3
coverings:

 External spermatic fascia from external oblique


aponeurosis
 Cremasteric fascia
 Internal spermatic fascia

Females Round ligament of uterus and


ilioinguinal nerve

Related anatomy of the inguinal region: the boundaries of Hesselbachs triangle

91. A 67 year old man is undergoing a transurethral resection of a bladder tumour using diathermy. Suddenly
during the procedure the patients leg begins to twitch. Stimulation of which of the following nerves is the
most likely cause?
A.Femoral
B. Pudendal
C. Sciatic
D.Obturator
E. Gluteal

Answer: D

The obturator nerve is most closely related to the bladder.

Obturator nerve

The obturator nerve arises from L2, L3 and L4 by branches from the ventral divisions of each of these nerve roots. L3
forms the main contribution and the second lumbar branch is occasionally absent. These branches unite in the
71
substance of psoas major, descending vertically in its posterior part to emerge from its medial border at the lateral
margin of the sacrum. It then crosses the sacroiliac joint to enter the lesser pelvis, it descends on obturator internus to
enter the obturator groove. In the lesser pelvis the nerve lies lateral to the internal iliac vessels and ureter, and is joined
by the obturator vessels lateral to the ovary or ductus deferens.

Supplies

 Medial compartment of thigh


 Muscles supplied: external obturator, adductor longus, adductor brevis, adductor magnus (not the lower part-
sciatic nerve), gracilis
 The cutaneous branch is often absent. When present, it passes between gracilis and adductor longus near the
middle part of the thigh, and supplies the skin and fascia of the distal two thirds of the medial aspect.

Obturator canal

 Connects the pelvis and thigh: contains the obturator artery, vein, nerve which divides into anterior and
posterior branches.

92. A 45 year old lady is undergoing a Whipples procedure for carcinoma of the pancreatic head. The bile duct
is transected. Which of the following vessels is mainly responsible for the blood supply to the bile duct?
A.Cystic artery
B. Hepatic artery
C. Portal vein
D.Left gastric artery
E. None of the above

Answer: B

The bile duct has an axial blood supply which is derived from the hepatic artery and from retroduodenal branches of
the gastroduodenal artery. Unlike the liver there is no contribution by the portal vein to the blood supply of the bile
duct. Damage to the hepatic artery during a difficult cholecystectomy is a recognized cause of bile duct strictures.

Gallbladder
 Fibromuscular sac with capacity of 50ml
 Columnar epithelium

Relations of the gallbladder


Anterior Liver
Posterior  Covered by peritoneum
 Transverse colon
 1st part of the duodenum

Laterally Right lobe of liver


Medially Quadrate lobe of liver

Arterial supply: Cystic artery (branch of Right hepatic artery)

Venous drainage: Cystic vein

Nerve supply: Sympathetic- mid thoracic spinal cord, Parasympathetic- anterior vagal trunk

72
Common bile duct

Origin: confluence of cystic and common hepatic ducts

Relations:

At origin  Medially - Hepatic artery


 Posteriorly- Portal vein

Distally  Duodenum - anteriorly


 Pancreas - medially and laterally
 Right renal vein – posteriorly

Arterial supply Branches of hepatic artery and retroduodenal branches of gastroduodenal artery

Calot's triangle

Medially Common hepatic duct


Inferiorly Cystic duct
Superiorly Inferior edge of liver
Contents Cystic artery

93. 7 year old boy presents with right iliac fossa pain and there is a clinical suspicion that appendicitis is
present. From which of the following embryological structures is the appendix derived?
A. Vitello-intestinal duct
B. Uranchus
C. Foregut
D. Hindgut
E. Midgut

Answer: E
The appendix is derived from the midgut

It is derived from the midgut which is why early appendicitis may present with periumbilical pain.

Appendix

 Location: Base of caecum.


 Up to 10cm long.
 Mainly lymphoid tissue (Hence mesenteric adenitis may mimic appendicitis).
 Caecal taenia coli converge at base of appendix and form a longitudinal muscle cover over the appendix. This
convergence should facilitate its identification at surgery if it is retrocaecal and difficult to find (which it can
be when people start doing appendicectomies!)
 Arterial supply: Appendicular artery (branch of the ileocolic).
 It is intra peritoneal.

McBurney's point: 1/3 of the way along a line drawn from the Anterior Superior Iliac Spine to the Umbilicus
6 Positions:

 Retrocaecal 74%
 Pelvic 21%
 Postileal
 Subcaecal
 Paracaecal
 Preileal
73
94. A 34 year old man presents to the surgical clinic 8 months following a laparotomy for a ruptured spleen. He
complains of a nodule in the centre of his laparotomy wound. This is explored surgically and a stitch
granuloma is found and excised. From which of the following cell types do granulomata arise?
A. Polymorpho nucleocytes
B. Plasma cells
C. Reed- Sternberg cells
D. Platelets
E. Macrophages

Answer: E
Granulomas are organised collections of macrophages
Macrophages give origin to granulomas.

Chronic inflammation

Overview
Chronic inflammation may occur secondary to acute inflammation. In most cases chronic inflammation occurs as a
primary process. These may be broadly viewed as being one of three main processes:

 Persisting infection with certain organisms such as Mycobacterium tuberculosis which results in delayed type
hypersensitivity reactions and inflammation.
 Prolonged exposure to non-biodegradable substances such as silica or suture materials which may induce an
inflammatory response.
 Autoimmune conditions involving antibodies formed against host antigens.

Acute vs. Chronic inflammation


Acute inflammation Chronic inflammation
Changes to existing vascular structure and increased permeability of Angiogenesis predominates
endothelial cells
Infiltration of neutrophils Macrophages, plasma cells and lymphocytes
predominate
Process may resolve with: Healing by fibrosis is the main result

 Suppuration
 Complete resolution
 Abscess formation
 Progression to chronic inflammation
 Healing by fibrosis

Granulomatous inflammation
A granuloma consists of a microscopic aggregation of macrophages (with epithelial type arrangement =epitheliod).
Large giant cells may be found at the periphery of granulomas.

Mediators
Growth factors released by activated macrophages include agents such as interferon and fibroblast growth factor (plus
many more). Some of these such as interferons may have systemic features resulting in systemic symptoms and signs,
which may be present in individuals with long standing chronic inflammation.

The finding of granulomas is pathognomonic of chronic inflammation,

74
95. A 53 year old man is undergoing a radical gastrectomy for carcinoma of the stomach. Which of the
following structures will need to be divided to gain access to the coeliac axis?
A.Lesser omentum
B. Greater omentum
C. Falciform ligament
D.Median arcuate ligament
E. Gastrosplenic ligament

Answer: A

The lesser omentum will need to be divided. During a radical gastrectomy this forms one of the nodal stations that will
need to be taken.

Coeliac axis

The coeliac axis has three main branches.

 Left gastric
 Hepatic: branches-Right Gastric, Gastroduodenal, Right Gastroepiploic, Superior Pancreaticoduodenal,
Cystic.
 Splenic: branches- Pancreatic, Short Gastric, Left Gastroepiploic

It occasionally gives off one of the inferior phrenic arteries.

Relations
Anteriorly Lesser omentum
Right Right coeliac ganglion and caudate process of liver
Left Left coeliac ganglion and gastric cardia
Inferiorly Upper border of pancreas and renal vein

96. A 17 year old lady presents with right iliac fossa pain and diagnosed as having acute appendicitis. You take
her to theatre to perform a laparoscopic appendicectomy. During the procedure the scrub nurse distracts
you and you inadvertently avulse the appendicular artery. The ensuing haemorrhage is likely to be supplied
directly from which of the following vessels?
A.Inferior mesenteric artery
B. Superior mesenteric artery
C. Ileo-colic artery
D.Internal iliac artery
E. None of the above

Answer: C

The appendicular artery is a branch of the ileocolic artery.

97. A 63 year old man who smokes heavily presents with dyspepsia. He is tested and found to be positive for
helicobacter pylori infection. One evening he has an episode of haematemesis and collapses. What is the
most likely vessel to be responsible?
A. Portal vein
B. Short gastric arteries
C. Superior mesenteric artery
75
D. Gastroduodenal artery
E. None of the above

Answer: D

He is most likely to have a posteriorly sited duodenal ulcer. These can invade the gastroduodenal artery and present
with major bleeding. Although gastric ulcers may invade vessels they do not tend to produce major bleeding of this
nature.

Gastroduodenal artery
Supplies
Pylorus, proximal part of the duodenum, and indirectly to the pancreatic head (via the anterior and posterior superior
pancreaticoduodenal arteries)

Path
Most commonly arises from the common hepatic artery of the coeliac trunk
Terminates by bifurcating into the right gastroepiploic artery and the superior pancreaticoduodenal artery

98. Which of the following nerves is responsible for the cremasteric reflex?
A.Lateral femoral cutaneous nerve
B. Femoral nerve
C. Obturator nerve
D.Genitofemoral nerve
E. None of the above

Answer: D

The motor and sensory fibres of the genitofemoral nerve are tested in the cremasteric reflex. A small contribution is
also played by the ilioinguinal nerve and thus the reflex may be lost following an inguinal hernia repair.

Genitofemoral nerve

Supplies
- Small area of the upper medial thigh

Path
- Arises from the first and second lumbar nerves
- Passes obliquely through Psoas major, and emerges from its medial border opposite the fibrocartilage between the
third and fourth lumbar vertebrae.
- It then descends on the surface of Psoas major, under cover of the peritoneum
- Divides into genital and femoral branches.
- The genital branch passes through the inguinal canal, within the spermatic cord, to supply the skin overlying the skin
and fascia of the scrotum. The femoral branch enters the thigh posterior to the inguinal ligament, lateral to the femoral
artery. It supplies an area of skin and fascia over the femoral triangle.

 It may be injured during abdominal or pelvic surgery, or during inguinal hernia repairs.

99. A 73 year old lady is admitted with brisk rectal bleeding. Despite attempts at resuscitation the bleeding
proceeds to cause haemodynamic compromise. An upper GI endoscopy is normal. A mesenteric angiogram

76
is performed and a contrast blush is seen in the region of the sigmoid colon. The radiologist decides to
embolise the vessel supplying this area. At what spinal level does it leave the aorta?
A.L2
B. L1
C. L4
D.L3
E. T10

Answer: D
The inferior mesenteric artery leaves the aorta at L3. It supplies the left colon and sigmoid. It's proximal continuation
to communicate with the middle colic artery is via the marginal artery.

Levels

Transpyloric plane
Level of the body of L1

 Pylorus stomach
 Left kidney hilum (L1- left one!)
 Right hilum of the kidney (1.5cm lower than the left)
 Fundus of the gallbladder
 Neck of pancreas
 Duodenojejunal flexure
 Superior mesenteric artery
 Portal vein
 Left and right colic flexure
 Root of the transverse mesocolon
 2nd part of the duodenum
 Upper part of conus medullaris
 Spleen

Can be identified by asking the supine patient to sit up without using their arms. The plane is located where the lateral
border of the rectus muscle crosses the costal margin.

Anatomical planes
Subcostal plane Lowest margin of 10th costal cartilage
Intercristal plane Level of body L4 (highest point of iliac crest)
Intertubercular plane Level of body L5

Common level landmarks


Inferior mesenteric artery L3
Bifurcation of aorta into common iliac arteries L4
Formation of IVC L5 (union of common iliac veins)
Diaphragm apertures  Vena cava T8
 Oesophagus T10
 Aortic hiatus T12

100. 43 year old man is undergoing a right hemicolectomy and the ileo-colic artery is ligated. From which of
the following vessels is is derived?
A.Inferior mesenteric artery
B. Superior mesenteric artery
77
C. Coeliac axis
D.Aorta
E. None of the above

Answer: B

The ileocolic artery is a branch of the SMA and supplies the right colon and terminal ileum. The transverse colon is
supplied by the middle colic artery. As veins accompany arteries in the mesentery and are lined by lymphatics, high
ligation is the norm in cancer resections. The ileo-colic artery branches off the SMA near the duodenum.

Colon anatomy

The colon is about 1.5m long although this can vary considerably.
Components:

 Ascending colon
 Transverse colon
 Descending colon
 Sigmoid colon

Arterial supply
Superior mesenteric artery and inferior mesenteric artery: linked by the marginal artery.
Ascending colon: ileocolic and right colic arteries
Transverse colon: middle colic artery
Descending and sigmoid colon: left colic artery

Venous drainage
From regional veins (that accompany arteries) to superior and inferior mesenteric vein

Lymphatic drainage
Initially along nodal chains that accompany supplying arteries, then para-aortic nodes.

Embryology
Midgut- Second part of duodenum to 2/3 transverse colon
Hindgut- Distal 1/3 transverse colon to anus

Peritoneal location
The right and left colon are part intraperitoneal and part extraperitoneal. The sigmoid and transverse colon are
generally wholly intraperitoneal. This has implications for the sequelae of perforations, which will tend to result in
generalised peritonitis in the wholly intra peritoneal segments.

101. A 53 year old man is undergoing a distal pancreatectomy for trauma. Which of the following vessels is
responsible for the arterial supply to the tail of the pancreas?
A.Splenic artery
B. Pancreaticoduodenal artery
C. Gastric artery
D.Hepatic artery
E. Superior mesenteric artery

Answer: A

78
Pancreatic head is supplied by the pancreaticoduodenal artery. Pancreatic tail is supplied by branches of the splenic
artery
There is an arterial "watershed" in the supply between the head and tail of the pancreas. The head is supplied by the
pancreaticoduodenal artery and the tail is supplied by branches of the splenic artery.

Pancreas

The pancreas is a retroperitoneal organ and lies posterior to the stomach. It may be accessed surgically by dividing the
peritoneal reflection that connects the greater omentum to the transverse colon. The pancreatic head sits in the
curvature of the duodenum. It's tail lies close to the hilum of the spleen, a site of potential injury during splenectomy.

Relations
Posterior to the pancreas
Pancreatic head Inferior vena cava
Common bile duct
Right and left renal veins
Superior mesenteric vein and artery
Pancreatic neck Superior mesenteric vein, portal vein
Pancreatic body- Left renal vein
Crus of diaphragm
Psoas muscle
Adrenal gland
Kidney
Aorta
Pancreatic tail Left kidney

Anterior to the pancreas


Pancreatic head 1st part of the duodenum
Pylorus
Gastroduodenal artery
SMA and SMV(uncinate process)
Pancreatic body Stomach
Duodenojejunal flexure
Pancreatic tail Splenic hilum

Superior to the pancreas


Coeliac trunk and its branches common hepatic artery and splenic artery

Grooves of the head of the pancreas


2nd and 3rd part of the duodenum

Arterial supply

 Head: pancreaticoduodenal artery


 Rest: splenic artery

Venous drainage

 Head: superior mesenteric vein


 Body and tail: splenic vein

Ampulla of Vater
79
 Merge of pancreatic duct and common bile duct
 Is an important landmark, halfway along the second part of the duodenum, that marks the anatomical
transition from foregut to midgut (also the site of transition between regions supplied by coeliac trunk and
SMA)>

102. Surgical occlusion of which of these structures, will result in the greatest reduction in hepatic blood
flow?
A.Portal vein
B. Common hepatic artery
C. Right hepatic artery
D.Coeliac axis
E. Left hepatic artery

Answer: A

The portal vein transports 70% of the blood supply to the liver, while the hepatic artery provides 30%. The portal vein
contains the products of digestion. The arterial and venous blood is dispersed by sinusoids to the central veins of the
liver lobules; these drain into the hepatic veins and then into the IVC. The caudate lobe drains directly into the IVC
rather than into other hepatic veins.

Liver

Structure of the liver


Right lobe  Supplied by right hepatic artery
 Contains Couinard segments V to VIII (-/+Sg I)

Left lobe  Supplied by the left hepatic artery


 Contains Couinard segments II to IV (+/- Sg1)

Quadrate lobe  Part of the right lobe anatomically, functionally is part of the left
 Couinard segment IV
 Porta hepatis lies behind
 On the right lies the gallbladder fossa
 On the left lies the fossa for the umbilical vein

Caudate lobe  Supplied by both right and left hepatic arteries


 Couinard segment I
 Lies behind the plane of the porta hepatis
 Anterior and lateral to the inferior vena cava
 Bile from the caudate lobe drains into both right and left hepatic ducts

Detailed knowledge of Couinard segments is not required for MRCS Part A

 Between the liver lobules are portal canals which contain the portal triad: Hepatic Artery, Portal Vein,
tributary of Bile Duct.

Relations of the liver

Anterior Postero inferiorly


Diaphragm Oesophagus
Xiphoid process Stomach
Duodenum
Hepatic flexure of colon
80
Right kidney
Gallbladder
Inferior vena cava

Porta hepatis
Location Postero inferior surface, it joins nearly at right angles with the left sagittal fossa, and separates the caudate
lobe behind from the quadrate lobe in front
Transmits  Common hepatic duct
 Hepatic artery
 Portal vein
 Sympathetic and parasympathetic nerve fibres
 Lymphatic drainage of the liver (and nodes)

Ligaments
Falciform ligament  2 layer fold peritoneum from the umbilicus to anterior liver surface
 Contains ligamentum teres (remnant umbilical vein)
 On superior liver surface it splits into the coronary and left triangular ligaments

Ligamentum teres Joins the left branch of the portal vein in the porta hepatis
Ligamentum venosum Remnant of ductus venosus

Arterial supply: Hepatic artery

Venous: Hepatic veins and Portal vein

Nervous supply: Sympathetic and parasympathetic trunks of coeliac plexus

103. A 23 year old man presents with appendicitis. A decision is made to perform an appendicectomy. The
operation commences with a 5cm incision centered on McBurneys point. Which of the following structures
will be encountered first during the dissection?
A. External oblique aponeurosis
B. Internal oblique muscle
C. Transversalis fascia
D. Rectus sheath
E.Peritoneum

Answer: A

The external oblique will be encountered first in this location. The rectus sheath lies more medially.
The external oblique muscle is the most superficial of the abdominal wall muscles. It originates from the 5th to 12th
ribs and passes inferomedially to insert into the linea alba, pubic tubercle and anterior half of the iliac crest. It is
innervated by the thoracoabdominal nerves (T7-T11) and sub costal nerves.

Abdominal wall

The 2 main muscles of the abdominal wall are the rectus abdominis (anterior) and the quadratus lumborum (posterior).
The remaining abdominal wall consists of 3 muscular layers. Each muscle passes from the lateral aspect of the
quadratus lumborum posteriorly to the lateral margin of the rectus sheath anteriorly. Each layer is muscular
posterolaterally and aponeurotic anteriorly.

81
Muscles of abdominal wall
External  Lies most superficially
oblique  Originates from 5th to 12th ribs
 Inserts into the anterior half of the outer aspect of the iliac crest, linea alba and pubic
tubercle
 More medially and superiorly to the arcuate line, the aponeurotic layer overlaps the rectus
abdominis muscle
 The lower border forms the inguinal ligament
 The triangular expansion of the medial end of the inguinal ligament is the lacunar ligament.

Internal oblique  Arises from the thoracolumbar fascia, the anterior 2/3 of the iliac crest and the lateral 2/3 of
the inguinal ligament
 The muscle sweeps upwards to insert into the cartilages of the lower 3 ribs
 The lower fibres form an aponeurosis that runs from the tenth costal cartilage to the body
of the pubis
 At its lowermost aspect it joins the fibres of the aponeurosis of transversus abdominis to
form the conjoint tendon.

Transversus  Innermost muscle


abdominis  Arises from the inner aspect of the costal cartilages of the lower 6 ribs , from the anterior
2/3 of the iliac crest and lateral 1/3 of the inguinal ligament
 Its fibres run horizontally around the abdominal wall ending in an aponeurosis. The upper
part runs posterior to the rectus abdominis. Lower down the fibres run anteriorly only.
 The rectus abdominis lies medially running from the pubic crest and symphysis to insert
into the xiphoid process and 5th, 6th and 7th costal cartilages. The muscles lies in a
aponeurosis as described above.
 Nerve supply: anterior primary rami of T7-12

Surgical notes
During abdominal surgery it is usually necessary to divide either the muscles or their aponeuroses. During a midline
laparotomy it is desirable to divide the aponeurosis. This will leave the rectus sheath intact above the arcuate line and
the muscles intact below it. Straying off the midline will often lead to damage to the rectus muscles, particularly below
the arcuate line where they may often be in close proximity to each other.

104. A 23 year old man is undergoing an inguinal hernia repair. The surgeons mobilise the spermatic cord
and place it in a hernia ring. A small slender nerve is identified superior to the cord. Which of the following
nerves is it most likely to be?

A. Iliohypogastric nerve
B. Pudendal nerve
C. Femoral branch of the genitofemoral nerve

D. Ilioinguinal nerve

E. Obturator nerve

Answer: D

The ilioinguinal nerve passes through the inguinal canal and is the nerve most commonly identified during hernia
surgery. The genitofemoral nerve splits into two branches, the genital branch passes through the inguinal canal within
the cord structures. The femoral branch of the genitofemoral nerve enters the thigh posterior to the inguinal ligament,
lateral to the femoral artery. The iliohypogastric nerve pierces the external oblique aponeurosis above the superficial
inguinal ring.

82
Ilioinguinal nerve

Arises from the first lumbar ventral ramus with the iliohypogastric nerve. It passes inferolaterally through the
substance of psoas major and over the anterior surface of quaratus lumborum. It pierces the internal oblique muscle
and passes deep to the aponeurosis of the external oblique muscle. It enters the inguinal canal and then passes through
the superficial inguinal ring to reach the skin.

Branches

 To supply those muscles of the abdominal wall through which it passes.


 Skin and fascia over the pubic symphysis, superomedial part of the femoral triangle, surface of the scrotum,
root and dorsum of penis or labum majus in females.

105. A 45 year old man is undergoing a low anterior resection for a carcinoma of the rectum. Which of the
following fascial structures will need to be divided to mobilise the mesorectum from the sacrum and
coccyx?
A. Denonvilliers fascia
B. Colles fascia
C. Sibsons fascia
D. Waldeyers fascia
E. None of the above

Answer: D
Fascial layers surrounding the rectum:

 Anteriorly lies the fascia of Denonvilliers


 Posteriorly lies Waldeyers fascia

Waldeyers fascia separates the mesorectum from the sacrum and will need to be divided.

Rectum

The rectum is approximately 12 cm long. It is a capacitance organ. It has both intra and extraperitoneal components.
The transition between the sigmoid colon is marked by the disappearance of the tenia coli.The extra peritoneal rectum
is surrounded by mesorectal fat that also contains lymph nodes. This mesorectal fatty layer is removed surgically
during rectal cancer surgery (Total Mesorectal Excision). The fascial layers that surround the rectum are important
clinical landmarks, anteriorly lies the fascia of Denonvilliers. Posteriorly lies Waldeyers fascia.

Extra peritoneal rectum

 Posterior upper third


 Posterior and lateral middle third
 Whole lower third

Relations
Anteriorly (Males) Rectovesical pouch
Bladder
Prostate
Seminal vesicles
Anteriorly (Females) Recto-uterine pouch (Douglas)
Cervix
Vaginal wall
Posteriorly Sacrum
83
Coccyx
Middle sacral artery
Laterally Levator ani
Coccygeus

Arterial supply
Superior rectal artery

Venous drainage
Superior rectal vein

Lymphatic drainage

 Mesorectal lymph nodes (superior to dentate line)


 Internal iliac and then para-aortic nodes
 Inguinal nodes (inferior to dentate line)

106. A 73 year old lady is admitted with acute mesenteric ischaemia. A CT angiogram is performed and a
stenotic lesion is noted at the origin of the superior mesenteric artery. At which of the following levels does
this branch from the aorta?
A. L1
B. L2
C. L3
D. L4
E.L5
Answer: A

The SMA leaves the aorta at L1. It passes under the neck of the pancreas prior to giving its first branch the inferior
pancreatico-duodenal artery.

 Branches off aorta at L1


 Supplies small bowel from duodenum (distal to ampulla of vater) through to mid transverse colon
 Takes more oblique angle from aorta and thus more likely to recieve emboli than coeliac axis

Relations of superior mesenteric artery


Superiorly Neck of pancreas
Postero-inferiorly Third part of duodenum
Uncinate process
Posteriorly Left renal vein
Right Superior mesenteric vein

Branches of the superior mesenteric artery

 Inferior pancreatico-duodenal artery


 Jejunal and ileal arcades
 Ileo-colic artery
 Right colic artery
 Middle colic artery

84
107. 72 year old man is undergoing an open abdominal aortic aneurysm repair. The aneurysm is located in a
juxtarenal location and surgical access to the neck of aneurysm is difficult. Which of the following
structures may be divided to improve access?
A.Cisterna chili
B.Transverse colon
C.Left renal vein
D.Superior mesenteric artery
E. Coeliac axis

Answer: C

The left renal vein will be stretched over the neck of the anuerysm in this location and is not infrequently divided. This
adds to the nephrotoxic insult of juxtarenal aortic surgery as a supra renal clamp is also often applied. Deliberate
division of the Cisterna Chyli will not improve access and will result in a chyle leak. Division of the transverse colon
will not help at all and would result in a high risk of graft infection. Division of the SMA is pointless for a juxtarenal
procedure.

Abdominal aorta

Abdominal aortic topography

Origin T12
Termination L4
Posterior relations L1-L4 Vertebral bodies
Anterior relations Lesser omentum
Liver
Left renal vein
Inferior mesenteric vein
Third part of duodenum
Pancreas
Parietal peritoneum
Peritoneal cavity
Right lateral relations Right crus of the diaphragm
Cisterna chyli
Azygos vein
IVC (becomes posterior distally)
Left lateral relations 4th part of duodenum
Duodenal-jejunal flexure
Left sympathetic trunk

108. Your consultant decides to perform an open inguinal hernia repair under local anaesthesia. Which of
the following dermatomal levels will require blockade?
A. T10
B. T12
C. T11
D. S1
E. S2

Answer: B

109. A 53 year old man presents with an inguinal hernia. Which of the following surface landmarks may be
used to identify the location of the deep inguinal ring?
A. Mid point of the inguinal ligament
B. The mid inguinal point
C. The pubic tubercle
D. The medial edge of external oblique
85
E. 2cm supero medially to the femoral artery
Answer: A

Surface marking of deep inguinal ring = midpoint of inguinal ligament

The surface markings of the deep inguinal ring are a commonly examined topic and should be memorised. The surface
marking is the midpoint of the inguinal ligament. The mid inguinal point is the surface marking for the femoral artery.
The pubic tubercle marks the site of the superficial inguinal ring.

110. At which level is the hilum of the left kidney located?


A. L1
B. L2
C. T12
D. T11
E. L3

Answer: A
Remember L1 ('left one') is the level of the hilum of the left kidney. This is commonly tested in the mrcs exam.

111. You are assisting in an open right adrenalectomy for a large adrenal adenoma. The consultant is
distracted and you helpfully pull the adrenal into the wound to improve the view. Unfortunately this is
followed by brisk bleeding. The vessel responsible for this is most likely to be:
A. Portal vein
B. Phrenic vein
C. Right renal vein
D. Superior mesenteric vein
E. Inferior vena cava

Answer: E
It drains directly via a very short vessel. If the sutures are not carefully tied then it may be avulsed off the IVC. An
injury best managed using a Satinsky clamp and a 6/0 prolene suture.

Adrenal gland anatomy

Anatomy

Location Superomedially to the upper pole of each kidney


Relationships of the right Diaphragm-Posteriorly, Kidney-Inferiorly, Vena Cava-Medially, Hepato-renal pouch
adrenal and bare area of the liver-Anteriorly
Relationships of the left Crus of the diaphragm-Postero- medially, Pancreas and splenic vessels-Inferiorly,
adrenal Lesser sac and stomach-Anteriorly
Superior adrenal arteries- from inferior phrenic artery, Middle adrenal arteries - from
Arterial supply aorta, Inferior adrenal arteries -from renal arteries
Venous drainage of the Via one central vein directly into the IVC
right adrenal
Venous drainage of the left Via one central vein into the left renal vein
adrenal

112. An enthusiastic surgical registrar undertakes his first solo splenectomy. The operation is far more
difficult than anticipated and the registrar leaves a tube drain to the splenic bed at the end of the
procedure. Over the following 24 hours approximately 500ml of clear fluid has entered the drain.
Biochemical testing of the fluid is most likely to reveal:
A.Elevated creatinine
B. Elevated triglycerides
86
C. Elevated glucagon
D.Elevated amylase
E. None of the above

Answer: D

During splenectomy the tail of the pancreas may be damaged. The pancreatic duct will then drain into the splenic bed,
amylase is the most likely biochemical finding. Glucagon is not secreted into the pancreatic duct.

Splenic anatomy

The spleen is the largest lymphoid organ in the body. It is an intraperitoneal organ, the peritoneal attachments
condense at the hilum where the vessels enter the spleen. Its blood supply is from the splenic artery (derived from the
coeliac axis) and the splenic vein (which is joined by the IMV and unites with the SMV).

 Embryology: derived from mesenchymal tissue


 Shape: clenched fist
 Position: below 9th-12th ribs
 Weight: 75-150g

Relations

 Superiorly- diaphragm
 Anteriorly- gastric impression
 Posteriorly- kidney
 Inferiorly- colon
 Hilum: tail of pancreas and splenic vessels
 Forms apex of lesser sac (containing short gastric vessels)

113. A 56 year old lady is referred to the colorectal clinic with symptoms of pruritus ani. On examination a
polypoidal mass is identified inferior to the dentate line. A biopsy confirms squamous cell carcinoma. To
which of the following lymph node groups will the lesion potentially metastasise?
A. Internal iliac
B. External iliac
C. Mesorectal
D. Inguinal
E.None of the above

Answer: D

Lesions distal to the dentate line drain to the inguinal nodes. Occasionally this will result in the need for a block
dissection of the groin.

114. A 72 year old man develops a hydrocele which is being surgically managed. As part of the procedure
the surgeons divide the tunica vaginalis. From which of the following is this structure derived?
A.Peritoneum
B. External oblique aponeurosis
C. Internal oblique aponeurosis
D.Transversalis fascia
87
E. Rectus sheath

Answer: A

The tunica vaginalis is derived from peritoneum, it secretes the fluid that fills the hydrocele cavity.

115. A 43 year old lady is donating her left kidney to her sister and the surgeons are harvesting the left
kidney. Which of the following structures will lie most anteriorly at the hilum of the left kidney?
A.Left renal artery
B. Left renal vein
C. Left ureter
D.Left ovarian vein
E. Left ovarian artery

Answer: B

The renal veins lie most anteriorly, then artery and ureter lies posteriorly.

Renal arteries
 The right renal artery is longer than the left renal artery
 The renal vein/artery/pelvis enter the kidney at the hilum

Relations

 Right: Anterior- IVC, right renal vein, the head of the pancreas, and the descending part of the duodenum.
 Left:Anterior- left renal vein, the tail of the pancreas.

Branches

 The renal arteries are direct branches off the aorta (upper border of L2)
 In 30% there may be accessory arteries (mainly left side). Instead of entering the kidney at the hilum, they
usually pierce the upper or lower part of the organ.
 Before reaching the hilum of the kidney, each artery divides into four or five segmental branches (renal vein
anterior and ureter posterior); which then divide within the sinus into lobar arteries supplying each pyramid
and cortex.
 Each vessel gives off some small inferior suprarenal branches to the suprarenal gland, the ureter, and the
surrounding cellular tissue and muscles.

116. A 56 year old lady is due to undergo a left hemicolectomy for carcinoma of the splenic flexure. The
surgeons decide to perform a high ligation of the inferior mesenteric vein. Into which of the following does
this structure usually drain?
A. Portal vein
B. Inferior vena cava
C. Left renal vein
D. Left iliac vein
E. Splenic vein
Answer: E
Beware of ureteric injury in colonic surgery.

The IMV drains into the splenic vein.

Left colon
88
Position

 As the left colon passes inferiorly its posterior aspect becomes extraperitoneal, and the ureter and gonadal
vessels are close posterior relations that may become involved in disease processes
 At a level of L3-4 (variable) the left colon becomes the sigmoid colon and wholly intraperitoneal once again
 The sigmoid colon is a highly mobile structure and may even lie of the right side of the abdomen
 It passes towards the midline, the taenia blend and this marks the transition between sigmoid colon and upper
rectum.

Blood supply

 Inferior mesenteric artery


 However, the marginal artery (from the right colon) contributes and this contribution becomes clinically
signifi cant when the IMA is divided surgically (e.g. During AAA repair)

117. Which of the following statements relating to quadratus lumborum is false?


A.Causes flexion of the thoracic spine
B.Causes the rib cage to be pulled down
C.Innervated by anterior primary rami of T12 and L1-3
D.Attached to the iliac crest
E. Inserts into the 12th rib
Answer: A

The rectus abdominis causes flexion of the thoracic spine.

Attached to the medial iliac crest and iliolumbar ligament.


Inserts: 12th rib
Action: pulls the rib cage inferiorly. Lateral flexion.
Nerve supply: anterior primary rami of T12 and L1-3

118. A 67 year old man is undergoing an angiogram for gastro intestinal bleeding. The radiologist advances
the catheter into the coeliac axis. At what spinal level does this vessel typically arise from the aorta?
A.T10
B. L3
C. L4
D.T12
E. None of the above
Answer: D

The coeliac axis lies at T12, it takes an almost horizontal angle off the aorta. It has three major branches.

Abdominal aortic branches

Branches Level Paired Type


Inferior phrenic T12 (Upper border) Yes Parietal
Coeliac T12 No Visceral
Superior mesenteric L1 No Visceral
Middle suprarenal L1 Yes Visceral
Renal L1-L2 Yes Visceral
Gonadal L2 Yes Visceral
Lumbar L1-L4 Yes Parietal
Inferior mesenteric L3 No Visceral
Median sacral L4 No Parietal
89
Common iliac L4 Yes Terminal

119. During a radical gastrectomy for carcinoma of the stomach the surgeons remove the omentum. What is
the main source of its blood supply?
A.Ileocolic artery
B. Superior mesenteric artery
C. Gastroepiploic artery
D.Middle colic artery
E. Inferior mesenteric artery
Answer: C

The vessels supplying the omentum are the omental branches of the right and left gastro-epiploic arteries. The colonic
vessels are not responsible for the arterial supply to the omentum. The left gastro-epiploic artery is a branch of the
splenic artery and the right gastro-epiploic artery is a terminal branch of the gastroduodenal artery.

Omentum

 The omentum is divided into two parts which invest the stomach. Giving rise to the greater and lesser
omentum. The greater omentum is attached to the inferolateral border of the stomach and houses the gastro-
epiploic arteries.
 It is of variable size but is less well developed in children. This is important as the omentum confers
protection against visceral perforation (e.g. Appendicitis).
 Inferiorly between the omentum and transverse colon is one potential entry point into the lesser sac.
 Several malignant processes may involve the omentum of which ovarian cancer is the most notable.

120. A 45 year old man has a long fermoral line inserted to provide CVP measurements. The catheter passes
from the common iliac vein into the inferior vena cava. At which of the following vertebral levels will this
occur?
A.L5
B.L4
C.S1
D.L3
E. L2
Answer: A

The common iliac veins fuse with the IVC at L5.

Inferior vena cava

Origin

 L5

Path

 Left and right common iliac veins merge to form the IVC.
 Passes right of midline
 Paired segmental lumbar veins drain into the IVC throughout its length
 The right gonadal vein empties directly into the cava and the left gonadal vein generally empties into the left
renal vein.
 The next major veins are the renal veins and the hepatic veins
 Pierces the central tendon of diaphragm at T8

90
 Right atrium

Relations

Anteriorly Small bowel, first and third part of duodenum, head of pancreas, liver and bile duct, right common iliac
artery, right gonadal artery
Posteriorly Right renal artery, right psoas, right sympathetic chain, coeliac ganglion

Levels

Level Vein
T8 Hepatic vein, inferior phrenic vein, pierces diaphragm
L1 Suprarenal veins, renal vein
L2 Gonadal vein
L1-5 Lumbar veins
L5 Common iliac vein, formation of IVC

121. What is the nerve root value of the external urethral sphincter?
A.S4
B. S1, S2, S3
C. S2, S3, S4
D.L3, L4, L5
E. L5, S1, S2
Answer: C

The external urethral sphincter is innervated by branches of the pudendal nerve, therefore the root values are S2, S3,
S4.

122. A 32 year old man is undergoing a splenectomy. Division of which of the following will be necessary
during the procedure?
A. Left crus of diaphragm
B. Short gastric vessels
C. Gerotas fascia
D. Splenic flexure of colon
E. Marginal artery

Answer: B

During a splenectomy the short gastric vessels which lie within the gastrosplenic ligament will need to be divided. The
splenic flexure of the colon may need to be mobilised. However, it will almost never need to be divided, as this is
watershed area that would necessitate a formal colonic resection in the event of division.

123. Two teenagers are playing with an airgun when one accidentally shoots his friend in the abdomen. He is
brought to the emergency department. On examination there is a bullet entry point immediately to the
right of the rectus sheath at the level of the 1st lumbar vertebra. Which of the following structures is most
likely to be injured by the bullet?
A.Head of pancreas
B. Right ureter
C. Right adrenal gland
D.Fundus of the gallbladder
E. Gastric antrum

91
Answer: D
The fundus of the gallbladder lies at this level and is the most superficially located structure.

124. Which of the following anatomical planes separates the prostate from the rectum?
A.Sibsons fascia
B. Denonvilliers fascia
C. Levator ani muscle
D.Waldeyers fascia
E. None of the above
Answer: B

The Denonvilliers fascia separates the rectum from the prostate. Waldeyers fascia separates the rectum from the
sacrum

Prostate gland

The prostate gland is approximately the shape and size of a walnut and is located inferior to the bladder. It is separated
from the rectum by Denonvilliers fascia and its blood supply is derived from the internal iliac vessels. The internal
sphincter lies at the apex of the gland and may be damaged during prostatic surgery, affected individuals may
complain of retrograde ejaculation.

Summary of prostate gland


Arterial supply Inferior vesical artery (from internal iliac)
Venous drainage Prostatic venous plexus (to paravertebral veins)
Lymphatic Internal iliac nodes
drainage
Innervation Inferior hypogastric plexus
Dimensions  Transverse diameter (4cm)
 AP diameter (2cm)
 Height (3cm)

Lobes  Posterior lobe: posterior to urethra


 Median lobe: posterior to urethra, in between ejaculatory ducts
 Lateral lobes x 2
 Isthmus

Zones  Peripheral zone: subcapsular portion of posterior prostate. Most prostate cancers are
here
 Central zone
 Transition zone
 Stroma

Relations
Anterior Pubic symphysis
Prostatic venous plexus
Posterior Denonvilliers fascia
Rectum
Ejaculatory ducts
Lateral Venous plexus (lies on prostate)
Levator ani (immediately below the puboprostatic ligaments)

92
125. A 56 year old lady is undergoing an adrenalectomy for Conns syndrome. During the operation the
surgeon damages the middle adrenal artery and haemorrhage ensues. From which of the following
structures does this vessel originate?
A.Aorta
B. Renal artery
C. Splenic artery
D.Coeliac axis
E. Superior mesenteric artery
Answer: A

The middle adrenal artery is usually a branch of the aorta, the lower adrenal artery typically arises from the renal
vessels.

126. A 24 year old man falls and lands astride a manhole cover. He suffers from a injury to the anterior
bulbar urethra. Where will the extravasated urine tend to collect?
A.Lesser pelvis
B. Connective tissue of the scrotum
C. Deep perineal space
D.Ischiorectal fossa
E. Posterior abdominal wall

Answer: B

This portion of the urethra is contained between the perineal membrane an the membranous layer of the superficial
fascia. As these are densely adherent to the ischiopubic rami, extravasated urine cannot pass posteriorly because the 2
layers are continuous around the superficial transverse perineal muscles.

Lower genitourinary tract trauma

 Most bladder injuries occur due to blunt trauma


 85% associated with pelvic fractures
 Easily overlooked during assessment in trauma
 Up to 10% of male pelvic fractures are associated with urethral or bladder injuries

Types of injury

Urethral injury  Mainly in males


 Blood at the meatus (50% cases)
 There are 2 types:

i.Bulbar rupture
- most common
- straddle type injury e.g. bicycles
- triad signs: urinary retention, perineal haematoma, blood at the meatus
ii. Membranous rupture
- can be extra or intraperitoneal
- commonly due to pelvic fracture
- Penile or perineal oedema/ hematoma
- PR: prostate displaced upwards (beware co-existing retroperitoneal haematomas
as they may make examination difficult)

- Investigation: ascending urethrogram


- Management: suprapubic catheter (surgical placement, not percutaneously)

93
External genitalia injuries (i.e.,  Secondary to injuries caused by penetration, blunt trauma, continence- or
the penis and the scrotum) sexual pleasure-enhancing devices, and mutilation

Bladder injury  rupture is intra or extraperitoneal


 presents with haematuria or suprapubic pain
 history of pelvic fracture and inability to void: always suspect bladder or
urethral injury
 inability to retrieve all fluid used to irrigate the bladder through a Foley
catheter indicates bladder injury
 investigation- IVU or cystogram
 management: laparotomy if intraperitoneal, conservative if
extraperitoneal

127. A 73 year old man presents with symptoms of mesenteric ischaemia. As part of his diagnostic work up a
diagnostic angiogram is performed .The radiologist is attempting to cannulate the coeliac axis from the
aorta. At which of the following vertebral levels does this is usually originate?
A. T10
B. L2
C. L3
D. T8
E. T12
Answer: E

Coeliac trunk branches:

Left Hand Side (LHS): Left gastric, Hepatic, Splenic

The coeliac axis branches off the aorta at T12.

Coeliac axis

Relations

Anteriorly Lesser omentum


Right Right coeliac ganglion and caudate process of liver
Left Left coeliac ganglion and gastric cardia
Inferiorly Upper border of pancreas and renal vein

128. During a liver resection a surgeon performs a pringles manoeuvre to control bleeding. Which of the
following structures will lie posterior to the epiploic foramen at this level?
A. Hepatic artery
B. Cystic duct
C. Greater omentum
D. Superior mesenteric artery
E. Inferior vena cava
Answer: E
Bleeding from liver trauma or a difficult cholecystectomy can be controlled with a vascular clamp applied at the
epiploic foramen.

The epiploic foramen has the following boundaries:


Anteriorly: (in the free edge of the lesser omentum): Bile duct to the right, portal vein behind and hepatic artery to the
left.

94
Posteriorly: Inferior vena cava
Inferiorly:1st part of the duodenum
Superiorly: Caudate process of the liver

129. Which of the following is not considered a major branch of the descending thoracic aorta?
A.Bronchial artery
B. Mediastinal artery
C. Inferior thyroid artery
D.Posterior intercostal artery
E. Oesophageal artery

Answer: C

The inferior thyroid artery is usually derived from the thyrocervical trunk, a branch of the subclavian artery.

Thoracic aorta

Origin T4
Terminates T12
Relations  Anteriorly (from top to bottom)-root of the left lung, the pericardium, the oesophagus, and the
diaphragm
 Posteriorly-vertebral column, azygos vein
 Right- hemiazygos veins, thoracic duct
 Left- left pleura and lung

Branches  Lateral segmental branches: Posterior intercostal arteries


 Lateral visceral: Bronchial arteries supply bronchial walls and lung excluding the alveoli
 Midline branches: Oesophageal arteries

130. During a right hemicolectomy the caecum is mobilised. As the bowel is retracted medially a vessel is
injured, posterior to the colon. Which of the following is the most likely vessel?
A.Right colic artery
B. Inferior vena cava
C. Aorta
D.External iliac artery
E. Gonadal vessels

Answer: E

The gonadal vessels and ureter are important posterior relations that are at risk during a right hemicolectomy.

Caecum

Location  Proximal right colon below the ileocaecal valve


 Intraperitoneal

Posterior relations  Psoas


 Iliacus
 Femoral nerve
 Genitofemoral nerve
 Gonadal vessels

Anterior relations Greater omentum


Arterial supply Ileocolic artery
Lymphatic drainage Mesenteric nodes accompany the venous drainage
95
The caecum is the most distensible part of the colon and in complete large bowel obstruction with a competent
ileocaecal valve the most likely site of eventual perforation.

131. A 66 year old man is undergoing a left nephro-ureterectomy. The surgeons remove the ureter, which of
the following is responsible for the blood supply to the proximal ureter?
A.Branches of the renal artery
B. External iliac artery
C. Internal iliac artery
D.Direct branches from the aorta
E. Common iliac artery
Answer: A

The proximal ureter is supplied by branches from the renal artery.

132. Which of the following is not a content of the porta hepatis?


A. Portal vein
B. Hepatic artery
C. Cystic duct
D. Hepatic lymph nodes
E. None of the above

Answer: C
The cystic duct lies outside the porta hepatis and is an important landmark in laparoscopic cholecystectomy. The
structures in the porta hepatis are:

 Portal vein
 Hepatic artery
 Common hepatic duct

These structures divide immediately after or within the porta hepatis to supply the functional left and right lobes of the
liver.
The porta hepatis is also surrounded by lymph nodes, that may enlarge to produce obstructive jaundice and
parasympathetic nervous fibres that travel along vessels to enter the liver.

133. A man is stabbed in the abdomen during a fight. He is brought to the emergency department. On
examination there is a laceration in the anterior abdominal wall immediately lateral to the left rectus
abdominis muscle on a level with the upper border of the first lumbar vertebra. Which of the following
structures is most likely to have been injured?

A. Head of the pancreas


B. Gastric antrum

C. Spleen

D. Right lobe of the liver


E. Superior mesenteric artery

Answer: C

The spleen is the most likely target in this instance.

134. A surgeon is due to perform a laparotomy for perforated duodenal ulcer. An upper midline incision is
to be performed. Which of the following structures is the incision most likely to divide?
A.Rectus abdominis muscle
96
B.External oblique muscle
C.Linea alba
D.Internal oblique muscle
E. None of the above

Answer: C

Upper midline abdominal incisions will involve the division of the linea alba. Division of muscles will not usually
improve access in this approach and they would not be routinely encountered during this incision.

Abdominal incisions

Theme in January 2012 exam

Midline incision  Commonest approach to the abdomen


 Structures divided: linea alba, transversalis fascia, extraperitoneal fat, peritoneum (avoid
falciform ligament above the umbilicus)
 Bladder can be accessed via an extraperitoneal approach through the space of Retzius

Paramedian  Parallel to the midline (about 3-4cm)


incision  Structures divided/retracted: anterior rectus sheath, rectus (retracted), posterior rectus
sheath, transversalis fascia, extraperitoneal fat, peritoneum
 Incision is closed in layers

Battle  Similar location to paramedian but rectus displaced medially (and thus denervated)
 Now seldom used

Kocher's Incision under right subcostal margin e.g. Cholecystectomy (open)


Lanz Incision in right iliac fossa e.g. Appendicectomy
Gridiron Oblique incision centered over McBurneys point- usually appendicectomy (less cosmetically
acceptable than Lanz
Gable Rooftop incision
Pfannenstiel's Transverse supra pubic, primarily used to access pelvic organs
McEvedy's Groin incision e.g. Emergency repair strangulated femoral hernia
Rutherford Extraperitoneal approach to left or right lower quadrants. Gives excellent access to iliac vessels
Morrison and is the approach of choice for first time renal transplantation.

135. A 59 year old man is undergoing an extended right hemicolectomy for a carcinoma of the hepatic
flexure of the colon. The surgeons divide the middle colonic vein close to its origin. Into which of the
following structures does this vessel primarily drain?
A.Superior mesenteric vein
B. Portal vein
C. Inferior mesenteric vein
D.Inferior vena cava
E. Ileocolic vein

Answer: A

97
The middle colonic vein drains into the SMV, if avulsed during mobilisation then dramatic haemorrhage can occur
and be difficult to control.

Transverse colon

 The right colon undergoes a sharp turn at the level of the hepatic flexure to become the transverse colon.
 At this point it also becomes intraperitoneal.
 It is connected to the inferior border of the pancreas by the transverse mesocolon.
 The greater omentum is attached to the superior aspect of the transverse colon from which it can easily be
separated. The mesentery contains the middle colic artery and vein. The greater omentum remains attached to
the transverse colon up to the splenic flexure. At this point the colon undergoes another sharp turn.

Relations
Superior Liver and gall-bladder, the greater curvature of the stomach, and the lower end of the spleen
Inferior Small intestine
Anterior Greater omentum
Posterior From right to left with the descending portion of the duodenum, the head of the pancreas, convolutions of
the jejunum and ileum, spleen

136. A 23 year old man is stabbed in the chest approximately 10cm below the right nipple. In the emergency
department a abdominal ultrasound scan shows a large amount of intraperitoneal blood. Which of the
following statements relating to the likely site of injury is untrue?
A.Part of its posterior surface is devoid of peritoneum.
B. The quadrate lobe is contained within the functional right lobe.
C. Its nerve supply is from the coeliac plexus.
D.The hepatic flexure of the colon lies posterio-inferiorly.
E. The right kidney is closely related posteriorly.

Answer: B

The right lobe of the liver is the most likely site of injury. Therefore the answer is B as the quadrate lobe is
functionally part of the left lobe of the liver. The liver is largely covered in peritoneum. Posteriorly there is an area
devoid of peritoneum (the bare area of the liver). The right lobe of the liver has the largest bare area (ans is larger
thant the left lobe).

137. Which of the following nerves passes through the greater sciatic foramen and innervates the perineum?
A. Pudendal
B. Sciatic
C. Superior gluteal
D. Inferior gluteal
E. Posterior cutaneous nerve of the thigh
Answer: A
3 divisions of the pudendal nerve:

 Rectal nerve
 Perineal nerve
 Dorsal nerve of penis/ clitoris

All these pass through the greater sciatic foramen.

The pudendal nerve innervates the perineum. It passes between piriformis and coccygeus medial to the sciatic nerve.

98
Gluteal region

Gluteal muscles

 Gluteus maximus: inserts to gluteal tuberosity of the femur and iliotibial tract
 Gluteus medius: attach to lateral greater trochanter
 Gluteus minimis: attach to anterior greater trochanter
 All extend and abduct the hip

Deep lateral hip rotators

 Piriformis
 Gemelli
 Obturator internus
 Quadratus femoris

Nerves
Superior gluteal nerve (L5, S1)  Gluteus medius
 Gluteus minimis
 Tensor fascia lata

Inferior gluteal nerve Gluteus maximus


Damage to the superior gluteal nerve will result in the patient developing a Trendelenberg gait. Affected patients are
unable to abduct the thigh at the hip joint. During the stance phase, the weakened abductor muscles allow the pelvis to
tilt down on the opposite side. To compensate, the trunk lurches to the weakened side to attempt to maintain a level
pelvis throughout the gait cycle. The pelvis sags on the opposite side of the lesioned superior gluteal nerve.

138. A 53 year old man is undergoing a left hemicolectomy for carcinoma of the descending colon. From
which embryological structure is this region of the gastrointestinal tract derived?

A. Vitellino-intestinal duct

B. Hind gut

C. Mid gut

D. Fore gut

E. Woolffian duct

Answer: B

The left colon is embryologically part of the hind gut. Which accounts for its separate blood supply via the IMA.

139. You excitedly embark on your first laparoscopic cholecystectomy and during the operation the
anatomy of Calots triangle is more hostile than anticipated. Whilst trying to apply a haemostatic clip you
avulse the cystic artery. This is followed by brisk haemorrhage. From which source is this most likely to
originate ?
A.Right hepatic artery
B. Portal vein
C. Gastroduodenal artery
D.Liver bed
99
E. Common hepatic artery

Answer: A

The cystic artery is a branch of the right hepatic artery. There are recognised variations in the anatomy of the blood
supply to the gallbladder. However, the commonest situation is for the cystic artery to branch from the right hepatic
artery.

140. A 43 year old man suffers a pelvic fracture which is complicated by an injury to the junction of the
membranous urethra to the bulbar urethra. In which of the following directions is the extravasated urine
most likely to pass?
A.Posteriorly into extra peritoneal tissues
B. Laterally into the buttocks
C. Into the abdomen
D.Anteriorly into the connective tissues surrounding the scrotum
E. None of the above

Answer: D

The superficial perineal pouch is a compartment bounded superficially by the superficial perineal fascia, deep by the
perineal membrane (inferior fascia of the urogenital diaphragm), and laterally by the ischiopubic ramus. It contains the
crura of the penis or clitoris, muscles, viscera, blood vessels, nerves, the proximal part of the spongy urethra in males,
and the greater vestibular glands in females.
When urethral rupture occurs as in this case the urine will tend to pass anteriorly because the fascial condensations
will prevent lateral and posterior passage of the urine.

Urogenital triangle

The urogenital triangle is formed by the:

 Ischiopubic inferior rami


 Ischial tuberosities

A fascial sheet is attached to the sides, forming the inferior fascia of the urogenital diaphragm.

It transmits the urethra in males and both the urethra and vagina in females. The membranous urethra lies deep this
structure and is surrounded by the external urethral sphincter.

Superficial to the urogenital diaphragm lies the superficial perineal pouch. In males this contains:

 Bulb of penis
 Crura of the penis
 Superficial transverse perineal muscle
 Posterior scrotal arteries
 Posterior scrotal nerves

In females the internal pudendal artery branches to become the posterior labial arteries in the superficial perineal
pouch.

141. Which of the following statements relating to the gastroduodenal artery is untrue?
100
A.It runs posterior to the 1st part of the duodenum
B. It originates from the common hepatic artery
C. The bile duct is a close relation
D.The portal vein is closely related anteriorly
E. It terminates as the gastro-epiploic and superior pancreaticoduodenal artery

Answer: D

The portal vein is located posteriorly and then separated from the artery by the pancreas. The anatomy of this artery is
important as it is a site of bleeding in posteriorly sited duodenal ulcers. At laparotomy for bleeding from this vessel,
the relation of the bile duct should be remembered less it be caught inadvertently in a stitch.

Gastroduodenal artery

Supplies
Pylorus, proximal part of the duodenum, and indirectly to the pancreatic head (via the anterior and posterior superior
pancreaticoduodenal arteries)

Path
Most commonly arises from the common hepatic artery of the coeliac trunk
Terminates by bifurcating into the right gastroepiploic artery and the superior pancreaticoduodenal artery

142. Through which of the following foramina does the genital branch of the genitofemoral nerve exit the
abdominal cavity?
A. Superficial inguinal ring
B. Sciatic notch
C. Obturator foramen
D. Femoral canal
E. Deep inguinal ring

Answer: E

The genitofemoral nerve divides into two branches as it approaches the inguinal ligament. The genital branch passes
anterior to the external iliac artery through the deep inguinal ring into the inguinal canal. It communicates with the
ilioinguinal nerve in the inguinal canal (though this is seldom of clinical significance).

143. A 63 year old lady is diagnosed as having an endometrial carcinoma arising from the uterine body. To
which nodal region will the tumour initially metastasise?
A.Para aortic nodes
B. Iliac lymph nodes
C. Inguinal nodes
D.Pres sacral nodes
E. Mesorectal lymph nodes

Answer: B

Tumours of the uterine body will tend to spread to the iliac nodes initially. When the tumour is expanding to cross
different nodal margins this is of considerable clinical significance if nodal clearance is performed during a Wertheims
type hysterectomy.

Lymphatic drainage of the uterus and cervix

101
 The uterine fundus has a lymphatic drainage that runs with the ovarian vessels and may thus drain to the para-
aortic nodes. Some drainage may also pass along the round ligament to the inguinal nodes.
 The body of the uterus drains through lymphatics contained within the broad ligament to the iliac lymph
nodes.
 The cervix drains into three potential nodal stations; laterally through the broad ligament to the external iliac
nodes, along the lymphatics of the uterosacral fold to the presacral nodes and posterolaterally along
lymphatics lying alongside the uterine vessels to the internal iliac nodes.

144. Which of the following structures is not located in the superficial perineal space in females?
A.Posterior labial arteries
B. Pudendal nerve
C. Superficial transverse perineal muscle
D.Greater vestibular glands
E. None of the above

Answer: B

The pudendal nerve is located in the deep perineal space and then branches to innervate more superficial structures.

145. Which of the following is not a branch of the hepatic artery?


A.Pancreatic artery
B.Cystic artery
C.Right gastric artery
D.Superior Pancreaticoduodenal artery
E. Right Gastroepiploic artery

Answer: A

The pancreatic artery is a branch of the splenic artery.

146. A 56 year old man is undergoing a nephrectomy. The surgeons divide the renal artery. At what level do
these usually branch off the abdominal aorta?
A.T9
B. L2
C. L3
D.T10
E. L4

Answer: B

The renal arteries usually branch off the aorta on a level with L2.

147. A 22 year old man presents with appendicitis. At operation the appendix is retrocaecal and difficult to
access. Division of which of the following anatomical structures should be undertaken?
A.Ileocolic artery
B. Mesentery of the caecum
C. Gonadal vessels
D.Lateral peritoneal attachments of the caecum
E. Right colic artery

Answer: D

The commonest appendiceal location is retrocaecal. Those struggling to find it at operation should trace the tenia to
the caecal pole where the appendix is located. If it cannot be mobilised easily then division of the lateral caecal
peritoneal attachments (as for a right hemicolectomy) will allow caecal mobilisation and facilitate the procedure.

102
148. A 56 year old man is left impotent following an abdomino-perineal excision of the colon and rectum.
What is the most likely explanation?
A.Psychosexual issues related to an end colostomy
B. Damage to the sacral venous plexus during total mesorectal excision
C. Damage to the left ureter during sigmoid mobilisation
D.Damage to the hypogastric plexus during mobilisation of the inferior mesenteric artery
E. Damage to the internal iliac artery during total mesorectal excision

Answer: D

Autonomic nerve injury is the most common cause.

Nerve lesions during surgery

A variety of different procedures carry the risk of iatrogenic nerve injury. These are important not only from the
patients perspective but also from a medicolegal standpoint.

The following operations and their associated nerve lesions are listed here:

 Posterior triangle lymph node biopsy and accessory nerve lesion.


 Lloyd Davies stirrups and common peroneal nerve.
 Thyroidectomy and laryngeal nerve.
 Anterior resection of rectum and hypogastric autonomic nerves.
 Axillary node clearance; long thoracic nerve, thoracodorsal nerve and intercostobrachial nerve.
 Inguinal hernia surgery and ilioinguinal nerve.
 Varicose vein surgery- sural and saphenous nerves.
 Posterior approach to the hip and sciatic nerve.
 Carotid endarterectomy and hypoglossal nerve.

There are many more, with sound anatomical understanding of the commonly performed procedures the incidence of
nerve lesions can be minimised. They commonly occur when surgeons operate in an unfamiliar tissue plane or by
blind placement of haemostats (not recommended).

149. A 73 year old man is due to undergo a radical prostatectomy for carcinoma of the prostate gland. To
which of the following lymph nodes will the tumour drain primarily?
A.Para aortic
B.Internal iliac
C.Superficial inguinal
D.Meso rectal
E. None of the above
Answer: B

The prostate lymphatic drainage is primarily to the internal iliac nodes and also the sacral nodes. Although internal
iliac is the first site.

150. A 28 year old man has sustained a non salvageable testicular injury to his left testicle. The surgeon
decides to perform an orchidectomy and divides the left testicular artery. From which of the following does
this vessel originate?
A.Abdominal aorta
B. Internal iliac artery
C. Inferior epigastric artery
D.Inferior vesical artery
E. External iliac artery

103
Answer: A

The testicular artery is a branch of the abdominal aorta.

151. A 44 year old man is stabbed in the back and the left kidney is injured. A haematoma forms, which of
the following fascial structures will contain the haematoma?
A.Waldeyers fascia
B. Sibsons fascia
C. Bucks fascia
D.Gerotas fascia
E. Denonvilliers fascia
Answer: D

152. Which of the following structures is not directly related to the right adrenal gland?
A. Diaphragm posteriorly
B. Bare area of the liver anteriorly
C. Right renal vein
D. Inferior vena cava
E. Hepato-renal pouch
Answer: C

The right renal vein is very short and lies more inferiorly.

153. Mobilisation of the left lobe of the liver will facilitate surgical access to which of the following?
A.Abdominal oesophagus
B. Duodenum
C. Right colic flexure
D.Right kidney
E. Pylorus of stomach

Answer: A

The fundus of the stomach is a posterior relation. The pylorus lies more inferolaterally. During a total gastrectomy
division of the ligaments holding the left lobe of the liver will facilitate access to the proximal stomach and abdominal
oesophagus. This manoeuvre is seldom beneficial during a distal gastrectomy.

154. A 32 year old man presents with an inguinal hernia and undergoes an open surgical repair. The
surgeons decide to place a mesh on the posterior wall of the inguinal canal to complete the repair, which of
the following structures will lie posterior to the mesh?
A. Transversalis fascia
B. External oblique
C. Rectus abdominis
D. Obturator nerve
E. None of the above
Answer: A

Inguinal canal walls: 'MALT: 2M, 2A, 2L, 2T':

Starting from superior, moving around in order to posterior:


Superior wall (roof): 2 Muscles:Internal oblique, transversus abdominis
Anterior wall: 2 Aponeuroses: Aponeurosis of external oblique, Aponeurosis of internal oblique
Lower wall (floor): 2 Ligaments: Inguinal Ligament, Lacunar Ligament Posterior wall: 2Ts: Transversalis fascia,
Conjoint Tendon
104
This is actually quite a straightforward question. It is simply asking for the structure that forms the posterior wall of
the inguinal canal. This is composed of the transversalis fascia, the conjoint tendon and more laterally the deep
inguinal ring.

155. A 22 year old man is involved in a fight outside a nightclub. He is stabbed in the back, on the left side,
approximately 3cm below the 12th rib in the mid scapular line. The structure most likely to be injured first
as a result is the:
A.Spleen
B. Left kidney
C. Left adrenal gland
D.Left ureter
E. None of the above

Answer: B

The left kidney lies in this location and is the most likely structure to be injured. The Spleen lies more superiorly, and
the left adrenal and ureter are unlikely to be injured in isolation.

156. A 23 year old man is undergoing an hernia repair and the mesh is to be sutured to the inguinal
ligament. From which of the following does the inguinal ligament arise?
A. Transversus abdominis fascia
B. Internal oblique
C. Rectus sheath
D. Rectus abdominis muscle
E. External oblique aponeurosis

Answer: E

The inguinal ligament is formed by the external oblique aponeurosis. It runs from the pubic tubercle to the anterior
superior iliac spine.

Abdominal wall

The 2 main muscles of the abdominal wall are the rectus abdominis (anterior) and the quadratus lumborum (posterior).
The remaining abdominal wall consists of 3 muscular layers. Each muscle passes from the lateral aspect of the
quadratus lumborum posteriorly to the lateral margin of the rectus sheath anteriorly. Each layer is muscular
posterolaterally and aponeurotic anteriorly.

157. A 56 year old man is undergoing a high anterior resection. Which of the following structures is at
greatest risk of injury in this procedure?
A.Superior mesenteric artery
B. Left ureter
C. External iliac vein
D.External iliac artery
E. Inferior vena cava

Answer: B

A careless surgeon may damage all of these structures. However, the structure at greatest risk and most frequently
encountered is the left ureter.

105
158. A 42 year old lady undergoes a difficult cholecystectomy and significant bleeding is occurring. The
surgeons place a vascular clamp transversely across the anterior border of the epiploic foramen. Which of
the following structures will be occluded in this manoeuvre?

A. Cystic artery
B. Cystic duct

C. Left gastric artery

D. Portal vein
E. None of the above
Answer: D

During liver surgery bleeding may be controlled using a Pringles manoeuvre, this involves placing a vascular clamp
across the anterior aspect of the epiploic foramen. The portal vein, hepatic artery and common bile duct are occluded.

159. A 56 year old lady undergoes a Hartmans style resection of the sigmoid colon, with ligation of the
vessels close to the colon. Which of the following vessels will be responsible to supplying the rectal stump
directly?
A.Superior mesenteric artery
B. Middle colic artery
C. Superior rectal artery
D.Inferior mesenteric artery
E. External iliac artery

Answer: C

This question is addressing the blood supply to the rectum. Which is supplied by the superior rectal artery. High
ligation of the IMA may compromise this structure. However, the question states that during the Hartmans procedure
the vessels were ligated close to the bowel. Implying that the superior rectal was preserved.

160. On inspecting the caecum, which of the following structures is most likely to be identified at the point at
which all the tenia coli converge?
A.Gonadal vessels
B. Appendix base
C. Appendix tip
D.Ileocaecal valve
E. Ileocolic artery

Answer: B

The tenia coli converge at the base of the appendix.

161. Which of the following structures lies most posteriorly at the porta hepatis?
A. Cystic artery
B. Common hepatic artery
C. Left hepatic artery
D. Portal vein
E. Common bile duct

Answer: D

106
The portal vein is the most posterior structure at the porta hepatis.The common bile duct is a continuation of the
common hepatic duct and is formed by the union of the common hepatic duct and the cystic duct.

162. A 76 year old man is undergoing an abdominal aortic aneurysm repair. The surgeons occlude the aorta
with two clamps, the inferior clamp being placed at the point of aortic bifurcation. Which of the following
vertebral bodies will lie posterior to the clamp at this level?
A. L1
B. T10
C. L4
D. L5
E. L2

Answer: C

The aorta bifurcates at L4. An important landmark that is tested frequently.

163. Which of the following statements relating to the greater omentum is false?
A.It is less well developed in children under 5.
B. It has no relationship to the lesser sac.
C. It contains the gastroepiploic arteries.
D.Has an attachment to the transverse colon.
E. It may be a site of metastatic disease in ovarian cancer.

Answer: B

It is connected with the lesser sac and the transverse colon. This plane is entered when performing a colonic resection.
It is a common site of metastasis in many visceral malignancies.

164. A 48 year old man with newly diagnosed hypertension is found to have a phaeochromocytoma of the
left adrenal gland and is due to undergo a laparoscopic left adrenalectomy. Which of the following
structures is not directly related to the left adrenal gland?
A.Crus of the diaphragm
B. Lesser curvature of the stomach
C. Splenic hilum
D.Pancreas
E. Splenic artery
Answer: C

The splenic hilum lies more laterally and is therefore not a direct relation of the left adrenal gland.

Adrenal gland anatomy


Anatomy

Location Superomedially to the upper pole of each kidney


Relationships of the right Diaphragm-Posteriorly, Kidney-Inferiorly, Vena Cava-Medially, Hepato-renal pouch
adrenal and bare area of the liver-Anteriorly
Relationships of the left Crus of the diaphragm-Postero- medially, Pancreas and splenic vessels-Inferiorly,
adrenal Lesser sac and stomach-Anteriorly
Superior adrenal arteries- from inferior phrenic artery, Middle adrenal arteries - from
Arterial supply aorta, Inferior adrenal arteries -from renal arteries
Venous drainage of the Via one central vein directly into the IVC
right adrenal
Venous drainage of the left Via one central vein into the left renal vein
adrenal

107
165. An 18 year old boy is undergoing an appendicectomy for appendicitis. At which of the following
locations is the appendix most likely to be found?
A.Pre ileal
B. Pelvic
C. Retrocaecal
D.Post ileal
E. None of the above

Answer: C

Most appendixes lie in the retrocaecal position. If a retrocaecal appendix is difficult to remove then mobilisation of the
right colon significantly improves access.

166. A 56 year old man is undergoing a pancreatectomy for carcinoma. During resection of the gland which
of the following structures will the surgeon not encounter posterior to the pancreas itself?
A.Left crus of the diaphragm
B. Superior mesenteric vein
C. Common bile duct
D.Portal vein
E. Gastroduodenal artery

Answer: E

The gastroduodenal artery lies anterior to the pancreas.

167. A 55 year old man is admitted with a brisk haematemesis. He is taken to the endoscopy department and
an upper GI endoscopy is performed by the gastroenterologist. He identifies an ulcer on the posterior
duodenal wall and spends an eternity trying to control the bleeding with all the latest haemostatic
techniques. He eventually asks the surgeons for help. A laparotomy and anterior duodenotomy are
performed, as the surgeon opens the duodenum a vessel is spurting blood into the duodenal lumen. From
which of the following does this vessel arise?
A.Left gastric artery
B. Common hepatic artery
C. Right hepatic artery
D.Superior mesenteric artery
E. Splenic artery

Answer: B

The vessel will be the gastroduodenal artery, this arises from the common hepatic artery.

168. Which of the following is not a content of the rectus sheath?


A.Pyramidalis
B.Superior epigastric artery
C.Inferior epigastric vein
D.Internal iliac artery
E. Rectus abdominis

Answer: D

108
The rectus sheath also contains:
superior epigastric vein
inferior epigastric artery

169. Which of the following vessels does not drain directly into the inferior vena cava?
A.Superior mesenteric vein
B.Right common iliac
C.Right hepatic vein
D.Left hepatic vein
E. Right testicular vein

Answer: A

The superior mesenteric vein drains into the portal vein. The right and left hepatic veins drain into it directly, this can
account for major bleeding in more extensive liver shearing type injuries.

170. A 17 year old male has a suspected testicular torsion and the scrotum is to be explored surgically. The
surgeon incises the skin and then the dartos muscle. What is the next tissue layer that will be encountered
during the dissection?
A.Visceral layer of the tunica vaginalis
B. Cremasteric fascia
C. Parietal layer of the tunica vaginalis
D.External spermatic fascia
E. Internal spermatic fascia
Answer: D

The layers that will be encountered are (in order):


1. Skin; 2. Dartos fascia and muscle; 3. External spermatic fascia; 4. Cremasteric muscle and fascia; 5. Internal
spermatic fascia; 6. Parietal layer of the tunica vaginalis

The layers of the spermatic cord and scrotum are a popular topic in the MRCS exam.

171. A 19 year old man undergoes an open inguinal hernia repair. The cord is mobilised and the deep
inguinal ring identified. Which of the following structures forms its lateral wall?
A.External oblique aponeurosis
B.Transversalis fascia
C.Conjoint tendon
D.Inferior epigastric artery
E. Inferior epigastric vein
Answer: B

The transversalis fascia forms the superolateral edge of the deep inguinal ring. The epigastric vessels form its
inferomedial wall.

172. A 34 year old lady presents with symptoms of faecal incontinence. Ten years previously she gave birth
to a child by normal vaginal delivery. Injury to which of the following nerves is most likely to account for
this process?
A.Genitofemoral
B. Ilioinguinal
C. Pudendal
D.Hypogastric autonomic nerve
E. Obturator

109
Answer: C

Damage to the pudendal nerve is classically associated with faecal incontinence and it is for this reason that sacral
neuromodulation is a popular treatment for the condition. Injury to the hypogastric autonomic nerves is an aetiological
factor in the development of constipation.

Pudendal nerve

The pudendal nerve arises from nerve roots S2, S3 and S4 and exits the pelvis through the greater sciatic foramen. It
re-enters the pelvis through the lesser sciatic foramen. It travels inferior to give innervation to the anal sphincters and
external urethral sphincter. It also provides cutaneous innervation to the region of perineum surrounding the anus and
posterior vulva.

Traction and compression of the pudendal nerve by the foetus in late pregnancy may result in late onset pudendal
neuropathy which may be part of the process involved in the development of faecal incontinence.

173. A 56 year old man undergoes an abdomino-perineal excision of the rectum. He is assessed in the
outpatient clinic post operatively. His wounds are well healed. However, he complains of impotence. Which
of the following best explains this problem?
A. Sciatic nerve injury
B. Damage to the internal iliac artery
C. Damage to the hypogastric nerve plexus
D. Damage to the vas
E. Damage to the genitofemoral nerve

Answer: C

Autonomic supply to the penis is via the hypogastric plexus of nerves. These may be damaged during mobilisation of
the proximal rectum from the sacrum and result in impotence post operatively. The addition of radiotherapy greatly
increases the risks of impotence following surgery.

Penile erection

Physiology of erection
Autonomic  Sympathetic nerves originate from T11-L2 and parasympathetic nerves from S2-4 join to form
pelvic plexus.
 Parasympathetic discharge causes erection, sympathetic discharge causes ejaculation and
detumescence.

Somatic Supplied by dorsal penile and pudendal nerves. Efferent signals are relayed from Onufs nucleus (S2-4)
nerves to innervate ischiocavernosus and bulbocavernosus muscles.

Autonomic discharge to the penis will trigger the veno-occlusive mechanism which triggers the flow of arterial blood
into the penile sinusoidal spaces. As the inflow increases the increased volume in this space will secondarily lead to
compression of the subtunical venous plexus with reduced venous return. During the detumesence phase the arteriolar
constriction will reduce arterial inflow and thereby allow venous return to normalise.

Priapism
Prolonged unwanted erection, in the absence of sexual desire, lasting more than 4 hours.

Classification of priaprism

110
Low flow priaprism Due to veno-occlusion (high intracavernosal pressures).

 Most common type


 Often painful
 Often low cavernosal flow
 If present for >4 hours requires emergency treatment

High flow priaprism Due to unregulated arterial blood flow.

 Usually presents as semi rigid painless erection

Recurrent priaprism Typically seen in sickle cell disease, most commonly of high flow type.

Causes

 Intracavernosal drug therapies (e.g. for erectile dysfunction>


 Blood disorders such as leukaemia and sickle cell disease
 Neurogenic disorders such as spinal cord transection
 Trauma to penis resulting in arterio-venous malformations

Tests

 Exclude sickle cell/ leukaemia


 Consider blood sampling from cavernosa to determine whether high or low flow (low flow is often hypoxic)

Management

 Ice packs/ cold showers


 If due to low flow then blood may be aspirated from copora or try intracavernosal alpha adrenergic agonists.
 Delayed therapy of low flow priaprism may result in erectile dysfunction.

174. Which of the following is not a branch of the descending abdominal aorta?
A. Inferior mesenteric artery
B. Inferior phrenic artery
C. Superior mesenteric artery
D. Superior phrenic artery
E. Renal artery
Answer: D
Mnemonic for the Descending abdominal aorta branches from diaphragm to iliacs:

'Prostitutes Cause Sagging Swollen Red Testicles [in men] Living In Sin':

Phrenic [inferior]
Celiac
Superior mesenteric
Suprarenal [middle]
Renal
Testicular ['in men' only]
Lumbars
Inferior mesenteric
Sacral

111

The superior phrenic artery branches from the aorta in the thorax.
 Abdominal aortic branches

Branches Level Paired Type


Inferior phrenic T12 (Upper border) Yes Parietal
Coeliac T12 No Visceral
Superior mesenteric L1 No Visceral
Middle suprarenal L1 Yes Visceral
Renal L1-L2 Yes Visceral
Gonadal L2 Yes Visceral
Lumbar L1-L4 Yes Parietal
Inferior mesenteric L3 No Visceral
Median sacral L4 No Parietal
Common iliac L4 Yes Terminal

175. A 23 year old man is admitted with a suspected ureteric colic. A KUB style x-ray is obtained. In which
of the following locations is the stone most likely to be visualised?
A.The tips of the transverse processes between L2 and L5
B. The tips of transverse processes between T10-L1
C. At the crest of the ilium
D.Over the S3 foramina
E. Over the sacrococcygeal joint

Answer: A

The ureter lies anterior to L2 to L5 and stones may be visualised at these points, they may also be identified over the
sacro-iliac joints.

176. A 55 year old man is due to undergo a radical prostatectomy for carcinoma of the prostate gland.
Which of the following vessels directly supplies the prostate?
A.External iliac artery
B.Common iliac artery
C.Internal iliac artery
D.Inferior vesical artery
E. None of the above
Answer: D

The arterial supply to the prostate gland is from the inferior vesical artery, it is a branch of the prostatovesical artery.
The prostatovesical artery usually arises from the internal pudendal and inferior gluteal arterial branches of the
internal iliac artery.

177. From which of the following embryological structures is the ureter derived?
A. Uranchus
B. Wolffian duct
C. Vitello-intestinal duct
D. Mesonephric duct
E. Cloaca

Answer: D

The ureter develops from the mesonephric duct.

112
178. A 56 year old man is having a long venous line inserted via the femoral vein into the right atrium for
CVP measurements. The catheter is advanced through the IVC. At which of the following levels does this
vessel enter the thorax?
A.L2
B.T10
C.L1
D.T8
E. T6
Answer: D

The IVC passes through the diaphragm at T8.

179. A 62 year old man is undergoing a left hemicolectomy for carcinoma of the descending colon. The
registrar commences mobilisation of the left colon by pulling downwards and medially. Blood soon appears
in the left paracolic gutter. The most likely source of bleeding is the:
A.Marginal artery
B. Left testicular artery
C. Spleen
D.Left renal vein
E. None of the above
Answer: C

The spleen is commonly torn by traction injuries in colonic surgery. The other structures are associated with bleeding
during colonic surgery but would not manifest themselves as blood in the paracolic gutter prior to incision of the
paracolonic peritoneal edge.

180. A 53 year old man with a chronically infected right kidney is due to undergo a nephrectomy. Which of
the following structures would be encountered first during a posterior approach to the hilum of the right
kidney?
A.Right renal artery
B. Ureter
C. Right renal vein
D.Inferior vena cava
E. Right testicular vein
Answer: B

The ureter is the most posterior structure at the hilum of the right kidney and would therefore be encountered first
during a posterior approach.

Which of the following regions of the male urethra is entirely surrounded by Bucks fascia?

A. Preprostatic part
B. Prostatic part
C. Membranous part
D. Spongiose part
E. None of the above

Answer: D
Bucks fascia is a layer of deep fascia that covers the penis it is continuous with the external spermatic fascia and the
penile suspensory ligament. The membranous part of the urethra may partially pass through Bucks fascia as it passes
into the penis. However, the spongiose part of the urethra is contained wholly within Bucks fascia.

113
181. 48 year old lady is undergoing a left sided adrenalectomy for an adrenal adenoma. The superior
adrenal artery is injured and starts to bleed, from which of the following does this vessel arise?
A.Left renal artery
B. Inferior phrenic artery
C. Aorta
D.Splenic
E. None of the above
Answer: B

182. Theme: Abdominal pain


 Appendicitis
 Threatened miscarriage
 Ectopic pregnancy
 Irritable bowel syndrome
 Mittelschmerz
 Pelvic inflammatory disease
 Adnexial torsion
 Endometriosis
 Degenerating fibroid

Please select the most likely cause of abdominal pain for the clinical scenario given. Each option may be used once,
more than once or not at all.

A. An 18 year-old girl presents to the Emergency Department with sudden onset sharp, tearing pelvic pain
associated with a small amount of vaginal bleeding. She also complains of shoulder tip pain. On
examination she is hypotensive, tachycardic and has marked cervical excitation.

Answer: Ectopic pregnancy. The history of tearing pain and haemodynamic compromise in a women of child
bearing years should prompt a diagnosis of ectopic pregnancy.

B. A 25 year-old lady presents to her GP complaining of a two day history of right upper quadrant pain,
fever and a white vaginal discharge. She has seen the GP twice in 12 weeks complaining of pelvic pain
and dyspareunia.

Answer:pelvic inflammatory disease. The most likely diagnosis is pelvic inflammatory disease. Right upper
quadrant pain occurs as part of the Fitz Hugh Curtis syndrome in which peri hepatic inflammation occurs.

C. A 16 year old female presents to the emergency department with a 12 hour history of pelvic discomfort.
She is otherwise well and her last normal menstrual period was 2 weeks ago. On examination she has a
soft abdomen with some mild supra pubic discomfort.

Answer: Mittelschmerz. Mid cycle pain is very common and is due to the small amount of fluid released
during ovulation. Inflammatory markers are usually normal and the pain typically subsides over the next 24-
48 hours.

Gynaecological causes of abdominal pain


A number of women will present with abdominal pain and subsequently be diagnosed with a gynaecological disorder.
In addition to routine diagnostic work up of abdominal pain, all female patients should also undergo a bimanual
vaginal examination, urine pregnancy test and consideration given to abdominal and pelvic ultrasound scanning.
When diagnostic doubt persists a laparoscopy provides a reliable method of assessing suspected tubulo-ovarian

114
pathology.

Differential diagnoses of abdominal pain in females


Diagnosis Features Investigation Treatment
Mittelschmerz Usually mid cycle pain. Full blood count- Conservative
Often sharp onset. usually normal
Little systemic disturbance. Ultrasound- may
May have recurrent episodes. show small quantity
Usually settles over 24-48 hours. of free fluid
Endometriosis 25% asymptomatic, in a further Ultrasound- may Usually managed medically, complex
25% associated with other pelvic show free fluid disease will often require surgery and
organ pathology. Laparoscopy will some patients will even require formal
Remaining 50% may have usually show lesions colonic and rectal resections if these
menstrual irregularity, infertility, areas are involved
pain and deep dyspareurina.
Complex disease may result in
pelvic adhesional formation with
episodes of intermittent small
bowel obstruction.
Intra-abdominal bleeding may
produce localised peritoneal
inflammation.
Recurrent episodes are common.
Ovarian torsion Usually sudden onset of deep Ultrasound may show Laparoscopy
seated colicky abdominal pain. free fluid
Associated with vomiting and Laparoscopy is
distress. usually both
Vaginal examination may reveal diagnostic and
adnexial tenderness. therapeutic
Ectopic Symptoms of pregnancy without Ultrasound showing Laparoscopy or laparotomy is
gestation evidence of intra uterine no intra uterine haemodynamically unstable. A
gestation. pregnancy and beta salphingectomy is usually performed.
Present as an emergency with HCG that is elevated
evidence of rupture or impending May show intra
rupture. abdominal free fluid
Open tubular ruptures may have
sudden onset of abdominal pain
and circulatory collapse, in other
the symptoms may be more
prolonged and less marked.
Small amount of vaginal
discharge is common.
There is usually adnexial
tenderness.
Pelvic Bilateral lower abdominal pain Full blood count- Usually medical management
inflammatory associated with vaginal Leucocytosis
disease discharge. Pregnancy test
Dysuria may also be present. negative (Although
Peri-hepatic inflammation infection and
secondary to Chlamydia (Fitz pregnancy may co-
Hugh Curtis Syndrome) may exist)
produce right upper quadrant Amylase - usually
discomfort. normal or slightly
Fever >38o raised
High vaginal and
urethral swabs

115
183. An 80 year old lady with a caecal carcinoma is undergoing a right hemicolectomy performed through a
transverse incision. The procedure is difficult and the incision is extended medially by dividing the rectus
sheath. Brisk arterial haemorrhage ensues. From which of the following does the damaged vessel originate?
A.Internal iliac artery
B.External iliac artery
C.Superior vesical artery
D.Inferior vesical artery
E. None of the above
Answer: B

The vessel damaged is the epigastric artery. This originates from the external iliac artery (see below).

Epigastric artery
The inferior epigastric artery arises from the external iliac artery immediately above the inguinal ligament. It then
passes along the medial margin of the deep inguinal ring. From here it continues superiorly to lie behind the rectus
abdominis muscle.

184. A 73 year old man has a large abdominal aortic aneurysm. During a laparotomy for planned surgical
repair the surgeons find the aneurysm is far more proximally located and lies near the origin of the SMA.
During the dissection a vessel lying transversely across the aorta is injured. What is this vessel most likely
to be?
A.Left renal vein
B.Right renal vein
C.Inferior mesenteric artery
D.Ileocolic artery
E. Middle colic artery

Answer: A

The left renal vein runs across the surface of the aorta and may require deliberate ligation during juxtarenal aneurysm
repair.

Abdominal aorta
Origin T12
Termination L4
Posterior relations L1-L4 Vertebral bodies
Anterior relations Lesser omentum
Liver
Left renal vein
Inferior mesenteric vein
Third part of duodenum
Pancreas
Parietal peritoneum
Peritoneal cavity
Right lateral relations Right crus of the diaphragm
Cisterna chyli
Azygos vein
IVC (becomes posterior distally)
Left lateral relations 4th part of duodenum
Duodenal-jejunal flexure
Left sympathetic trunk

116
185. A 18 year old man presents with an indirect inguinal hernia and undergoes surgery. The deep inguinal
ring is exposed and held with a retractor at its medial aspect. Which structure is most likely to lie under the
retractor?
A. Ureter
B. Inferior epigastric artery
C. Internal iliac vein
D. Femoral artery
E. Lateral border of rectus abdominis
Answer: B

Boundaries of the deep inguinal ring:

 Superolaterally - transversalis fascia


 Inferomedially - inferior epigastric artery

The deep inguinal ring is closely related to the inferior epigastric artery. The inferior epigastric artery forms part of the
structure referred to as Hesselbach's triangle.

186. In a patient with a carcinoma of the distal sigmoid colon, what is the most likely source of its blood
supply?
A.Ileocolic artery
B. External iliac artery
C. Internal iliac artery
D.Superior mesenteric artery
E. Inferior mesenteric artery

Answer: E

During a high anterior resection of such tumours, the inferior mesenteric artery is ligated. Note that the branches
(mainly middle rectal branch) of the internal iliac artery are important in maintaining vascularity of the rectal stump
and hence the integrity of the anastomoses.

187. A patient is due to undergo a right hemicolectomy for a carcinoma of the caecum. Which of the
following vessels will require high ligation to provide optimal oncological control?
A.Middle colic artery
B. Inferior mesenteric artery
C. Superior mesenteric artery
D.Ileo-colic artery
E. None of the above

Answer: D

The ileo - colic artery supplies the caecum and would require high ligation during a right hemicolectomy. The middle
colic artery should generally be preserved when resecting a caecal lesion.
This question is essentially asking you to name the vessel supplying the caecum. The SMA does not directly supply
the caecum, it is the ileocolic artery which does this.

Caecum

Location  Proximal right colon below the ileocaecal valve


 Intraperitoneal

Posterior relations  Psoas


117
 Iliacus
 Femoral nerve
 Genitofemoral nerve
 Gonadal vessels

Anterior relations Greater omentum


Arterial supply Ileocolic artery
Lymphatic drainage Mesenteric nodes accompany the venous drainage

The caecum is the most distensible part of the colon and in complete large bowel obstruction with a competent
ileocaecal valve the most likely site of eventual perforation.

188. A 72 year old man is undergoing a repair of an abdominal aortic aneurysm. The aorta is cross clamped
both proximally and distally. The proximal clamp is applied immediately inferior to the renal arteries. Both
common iliac arteries are clamped distally. A longitudinal aortotomy is performed. After evacuating the
contents of the aneurysm sac a significant amount of ongoing bleeding is encountered. This is most likely to
originate from:
A.The coeliac axis
B. Testicular artery
C. Splenic artery
D.Superior mesenteric artery
E. Lumbar arteries

Answer: E

The lumbar arteries are posteriorly sited and are a common cause of back bleeding during aortic surgery. The other
vessels cited all exit the aorta in the regions that have been cross clamped.

Abdominal aortic branches

Branches Level Paired Type


Inferior phrenic T12 (Upper border) Yes Parietal
Coeliac T12 No Visceral
Superior mesenteric L1 No Visceral
Middle suprarenal L1 Yes Visceral
Renal L1-L2 Yes Visceral
Gonadal L2 Yes Visceral
Lumbar L1-L4 Yes Parietal
Inferior mesenteric L3 No Visceral
Median sacral L4 No Parietal
Common iliac L4 Yes Terminal

The superficial inguinal ring is traversed by which of the following nerves?

A. Subcostal
B. Iliohypogastric
C. Ilioinguinal
D. Obturator
E. Pudendal

Answer: C

118
Ilioinguinal nerve entrapment may be a cause of neuropathic pain following inguinal hernia surgery.
The ilioinguinal nerve passes through the superfical inguinal ring and is routinely encountered when exploring the
inguinal canal during hernia surgery. The iliohypogastric nerve pierces the aponeurosis of the external oblique muscle
superior to the superficial inguinal ring.

Ilioinguinal nerve

Arises from the first lumbar ventral ramus with the iliohypogastric nerve. It passes inferolaterally through the
substance of psoas major and over the anterior surface of quaratus lumborum. It pierces the internal oblique muscle
and passes deep to the aponeurosis of the external oblique muscle. It enters the inguinal canal and then passes through
the superficial inguinal ring to reach the skin.

Branches

 To supply those muscles of the abdominal wall through which it passes.


 Skin and fascia over the pubic symphysis, superomedial part of the femoral triangle, surface of the scrotum,
root and dorsum of penis or labum majus in females.

189. A 63 year old man undergoes a radical cystectomy for carcinoma of the bladder. During the procedure
there is considerable venous bleeding. What is the primary site of venous drainage of the urinary bladder?
A.Vesicoprostatic venous plexus
B. Internal iliac vein
C. External iliac vein
D.Gonadal vein
E. Common iliac vein

Answer: A

The urinary bladder has a rich venous plexus surrounding it, this drains subsequently into the internal iliac vein. The
vesicoprostatic plexus may be a site of considerable venous bleeding during cystectomy.

Bladder
The empty bladder is contained within the pelvic cavity. It is usually a three sided pyramid. The apex of the bladder
points forwards towards the symphysis pubis and the base lies immediately anterior to the rectum or vagina.
Continuous with the apex is the medial umbilical ligament, during development this was the site of the uranchus.
The inferior aspect of the bladder is retroperitoneal and the superior aspect covered by peritoneum. As the bladder
distends it will tend to separate the peritoneum from the fascia of tansversalis. For this reason a bladder that is
distended due to acute urinary retention may be approached with a suprapubic catheter that avoids entry into the
peritoneal cavity.
The trigone is the least mobile part of the bladder and forms the site of the ureteric orifices and internal urethral
orifice. In the empty bladder the ureteric orifices are approximately 2-3cm apart, this distance may increase to 5cm in
the distended bladder.

Arterial supply
The superior and inferior vesical arteries provide the main blood supply to the bladder. These are branches of the
internal iliac artery.

Venous drainage
In males the bladder is drained by the vesicoprostatic venous plexus. In females the bladder is drained by the
vesicouterine venous plexus. In both sexes this venous plexus will ultimately drain to the internal iliac veins.

Lymphatic drainage
119
Lymphatic drainage is predominantly to the external iliac nodes, internal iliac and obturator nodes also form sites of
bladder lymphatic drainage.

Innervation
Parasympathetic nerve fibres innervate the bladder from the pelvic splanchnic nerves. Sympathetic nerve fibres are
derived from L1 and L2 via the hypogastric nerve plexuses. The parasympathetic nerve fibres will typically cause
detrusor muscle contraction and result in voiding. The muscle of the trigone is innervated by the sympathetic nervous
system. The external urethral sphincter is under concious control. During bladder filling the rate of firing of nerve
impulses to the detrusor muscle is low and receptive relaxation occurs. At higher volumes and increased intra vesical
pressures the rate of neuronal firing will increase and eventually voiding will occur.

A 60 year old female is undergoing a Whipples procedure for adenocarcinoma of the pancreas. As the surgeons
begin to mobilise the pancreatic head they identify a large vessel passing inferiorly over the anterior aspect of
the pancreatic head. What is it likely to be?

A. Superior mesenteric artery


B. Coeliac axis
C. Inferior mesenteric artery
D. Aorta
E. Left gastric artery

Answer: A

The superior mesenteric artery arises from the aorta and passes anterior to the lower part of the pancreas. Invasion of
this structure is a relative contra indication to resectional surgery.

190. An 18 year old man is undergoing an orchidectomy via a scrotal approach. The surgeons mobilise the
spermatic cord. From which of the following is the outermost layer of this structure derived?
A.Internal oblique aponeurosis
B. External oblique aponeurosis
C. Transversalis fascia
D.Rectus sheath
E. Campers fascia

Answer: B

The outermost covering of the spermatic cord is derived from the external oblique aponeurosis.This layer is added as
the cord passes through the superficial inguinal ring.

191. A 53 year old male presents with a carcinoma of the transverse colon. Which of the following structures
should be ligated close to their origin to maximise clearance of the tumour?
A.Superior mesenteric artery
B. Inferior mesenteric artery
C. Middle colic artery
D.Ileo-colic artery
E. Superior rectal artery

Answer: C

The middle colic artery supplies the transverse colon and requires high ligation during cancer resections. It is a branch
of the superior mesenteric artery.

192. Which of the following structures does not lie posterior to the right kidney?

120
A.Psoas major
B. Transversus abdominis
C. Quadratus lumborum
D.Medial artcuate ligament
E. 10th rib

Answer: E

The 10th rib lies more superior. The 12th rib is a closer relation posteriorly.

193. A 73 year old lady presents with a femoral hernia. Which of the following structures forms the lateral
wall of the femoral canal?
A.Pubic tubercle
B.Femoral vein
C.Femoral artery
D.Conjoint tendon
E. Femoral nerve

Answer: B

The canal exists to allow for the physiological expansion of the femoral vein, which lies lateral to it.

Femoral canal

The femoral canal lies at the medial aspect of the femoral sheath. The femoral sheath is a fascial tunnel containing
both the femoral artery laterally and femoral vein medially. The canal lies medial to the vein.

Borders of the femoral canal

Laterally Femoral vein


Medially Lacunar ligament
Anteriorly Inguinal ligament
Posteriorly Pectineal ligament
Contents

 Lymphatic vessels
 Cloquet's lymph node
Physiological significance
Allows the femoral vein to expand to allow for increased venous return to the lower limbs.

Pathological significance
As a potential space, it is the site of femoral hernias. The relatively tight neck places these at high risk of
strangulation.

194. How many unpaired branches leave the abdominal aorta to supply the abdominal viscera?
A.One
B. Two
C. Three
D.Four
E. Five

Answer: C

121
There are three unpaired branches to the abdominal viscera. These include the coeliac axis, the SMA and IMA.
Branches to the adrenals, renal arteries and gonadal vessels are paired. The fourth unpaired branch of the abdominal
aorta, the median sacral artery, does not directly supply the abdominal viscera.

195. A 78 year old man develops a carcinoma of the scrotum. To which of the following lymph node groups
may the tumour initially metastasise?
A.Para aortic
B. Obturator
C. Inguinal
D.Meso rectal
E. None of the above

Answer: C

The scrotum is drained by the inguinal nodes.

196. An 22 year old soldier is shot in the abdomen and amongst his various injuries is a major disruption to
the abdominal aorta. There is torrential haemorrhage and the surgeons decide to control the aorta by
placement of a vascular clamp immediately inferior to the diaphragm. Which of the following vessels may
be injured in this maneouvre?
A.Inferior phrenic arteries
B. Superior phrenic arteries
C. Splenic artery
D.Renal arteries
E. Superior mesenteric artery
Answer: A

As the first branches of the abdominal aorta the inferior phrenic arteries are at greatest risk. The superior phrenic
arteries lie in the thorax. The potential space at the level of the diaphragmatic hiatus is a potentially useful site for
aortic occlusion. However, leaving the clamp applied for more than about 10 -15 minutes usually leads to poor
outcomes.

197. Which of the following statements relating to the gallbladder is untrue?


A.The fundus is usually intra peritoneal
B. Arterial supply is from the cystic artery
C. The cystic artery is usually located in Calots triangle
D.Calots triangle may rarely contain an aberrant hepatic artery
E. Cholecystokinin causes relaxation of the gallbladder

Answer: E

CCK causes gallbladder contraction.

198. A 25 year old man is being catheterised, prior to a surgical procedure. As the catheter enters the
prostatic urethra which of the following changes will occur?
A. Resistance will increase significantly
B. Resistance will increase slightly
C. It will lie horizontally
D. Resistance will decrease
E. It will deviate laterally
Answer: D

The prostatic urethra is much wider than the membranous urethra and therefore resistance will decrease. The prostatic
urethra is inclined vertically.

122
199. Which of the following nerves is the primary source of innervation to the anterior scrotal skin?
A.Genital branch of the genitofemoral nerve
B. Pudendal nerve
C. Ilioinguinal nerve
D.Femoral branch of the genitofemoral nerve
E. Obturator nerve

Answer: C

The pudendal nerve may innervate the posterior skin of the scrotum. The anterior innervation of the scrotum is
primarily provided by the ilioinguinal nerve. The genital branch of the genitofemoral nerve provides a smaller
contribution.

Scrotal sensation
The scrotum is innervated by the ilioinguinal nerve and the pudendal nerve. The ilioinguinal nerve arises from L1 and
pierces the internal oblique muscle. It eventually passes through the superficial inguinal ring to innervate the anterior
skin of the scrotum.The pudendal nerve is the principal nerve of the perineum. It arises in the pelvis from 3 nerve
roots. It passes through both greater and lesser sciatic foramina to enter the perineal region. The perineal branches pass
anteromedially and divide into posterior scrotal branches. The posterior scrotal branches pass superficially to supply
the skin and fascia of the perineum. It cross communicates with the inferior rectal nerve.

200. The transversalis fascia contributes to which of the following?


A. Pectineal ligament
B. Deep inguinal ring
C. Cremaster muscle and fascia
D. Inguinal ligament
E. External spermatic fascia
Answer: B

The internal spermatic fascia (derived from transversalis fascia) invests:


Ducuts deferens
Testicular vessels
The principal outpouching of the transversalis fascia is the internal spermatic fascia. The mouth of the outpouching is
the deep inguinal ring.

201. A 73 year old lady is admitted with right iliac fossa pain. A plain abdominal x-ray is taken and the
caecal diameter measured. Which of the following caecal diameters are pathological?
A.4cm
B. 5cm
C. 6cm
D.7cm
E. 10cm

Answer: E

8 cm is still within normal limits. However, caecal diameters of 9 and 10 are pathological and should prompt further
investigation.

Right colon

Ileocaecal valve

123
 Entry point of the terminal ileum to the caecum
 An important colonoscopic landmark
 The ileocaecal valve is not always competent and this may allow partial decompression of an obstructed colon
Appendix

 At the base of the caecum the taenia coalesce to mark the base of the appendix
 This is a reliable way of locating the appendix surgically and is a constant landmark
 The appendix has a small mesentery (the mesoappendix) and in this runs the appendiceal artery, a branch of
the ileocolic artery.

The posterior aspect of the right colon is extra peritoneal and the anterior aspect intraperitoneal.

Relations

 Posterior :Iliacus, Iliolumbar ligament, Quadratus lumborum, Transverse abdominis, Diaphragm at the tip of
the last rib; Lateral cutaneous, ilioinguinal, and iliohypogastric nerves; the iliac branches of the iliolumbar
vessels, the fourth lumbar artery, gonadal vessels, ureter and the right kidney.

 Superior: Right kidney which is embedded in the perinephric fat

 Medial: Mesentery which contains the ileocolic artery that supplies the right colon and terminal ileum. A
further branch , the right colic artery, also contributes to supply the hepatic flexure and proximal transverse
colon. Medially these pass through the mesentery to join the SMA. This occurs near to the head of the
pancreas and care has to be taken when ligating the ileocolic artery near to its origin in cancer cases for fear of
impinging on the SMA.

- Anterior: Coils of small intestine, the right edge of the greater omentum, and the anterior abdominal wall.

Nerve supply: Parasympathetic fibres of the vagus nerve (CN X)

Arterial supply: Ileocolic artery and right colic artery, both branches of the SMA. While the ileocolic artery is almost
always present, the right colic can be absent in 5-15% of individuals.

202. Which of the following options in relation to the liver is true?


A. Ligamentum venosum is an anterior relation of the liver
B. The portal triad comprises the hepatic artery, hepatic vein and tributary of the bile duct
C. The liver is completely covered by peritoneum
D. There are no nerves within the porta hepatis
E. The caudate lobe is superior to the porta hepatis
Answer: E
'VC goes with VC'

The ligamentun Venosum and Caudate is on same side as Vena Cava [posterior].
Ligamentum venosum is posterior to the liver. The portal triad contains the portal vein rather than the hepatic vein.
There is the 'bare area of the liver' created by a void due to the coronary ligament layers being widely separated. There
are sympathetic and parasympathetic nerves in the porta hepatis.

203. The following statements regarding the rectus abdominis muscle are true except:
A. It runs from the symphysis pubis to the xiphoid process
B. Its nerve supply is from the ventral rami of the lower 6 thoracic nerves
C. It has collateral supply from both superior and inferior epigastric vessels
124
D. It lies in a muscular aponeurosis throughout its length
E. It has a number of tendinous intersections that penetrate through the anterior layer of the muscle

Answer: D

The aponeurosis is deficient below the arcuate line.


Rectus abdominis: Arises from 5th, 6th, 7th costal cartilages.
Inserts into the pubis.
The muscle lies in the rectal sheath, which also contains the superior & inferior epigastric artery and vein.
Action: flexion of thoracic and lumbar spine.
Nerve supply: anterior primary rami of T7-12

204. A 42 year old male sustains a back injury resulting in the compression of the conus medullaris. Which
of the dematomes below is most likely to be affected by this process?
A.S1
B.L1
C.S3
D.L3
E. L5
Answer: C
The perineum is innervated by S3 and S4, S2 runs down the posterior aspect of the leg and would also be affected.

205. During liver mobilisation for a cadaveric liver transplant the hepatic ligaments will require
mobilisation. Which of the following statements relating to these structures is untrue?
A.Lesser omentum arises from the porta hepatis and passes the lesser curvature of the stomach
B. The falciform ligament divides into the left triangular ligament and coronary ligament
C. The liver has an area devoid of peritoneum
D.The coronary ligament is attached to the liver
E. The right triangular ligament is an early branch of the left triangular ligament

Answer: E

The right triangular ligament is a continuation of the coronary ligament.

Physiology
1. There is decreased secretion of which one of the following hormones in response to major surgery:
A. Insulin
B. Cortisol
C. Renin
D. Anti diuretic hormone
E. Prolactin
Answer: A
Endocrine parameters reduced in stress response: Insulin, Testosterone, Oestrogen
Insulin is often released in decreased quantities following surgery.

Stress response: Endocrine and metabolic changes: Surgery precipitates hormonal and metabolic changes causing the stress
response. Stress response is associated with: substrate mobilization, muscle protein loss, sodium and water retention, suppression
of anabolic hormone secretion, activation of the sympathetic nervous system, immunological and haematological changes. The
hypothalamic-pituitary axis and the sympathetic nervous systems are activated and there is a failure of the normal feedback
mechanisms of control of hormone secretion.

A summary of the hormonal changes associated with the stress response: Increased: Growth hormone, Cortisol, Cortisol,
Renin, Adrenocorticotrophic hormone (ACTH), Aldosterone, Prolactinm, Antidiuretic hormone, Glucagon. Decreased: Insulinm
Testosterone, Oestrogen. No Change: Thyroid stimulating hormone, Luteinizing hormone, Follicle stimulating hormone

Sympathetic nervous system: Stimulates catecholamine release. Causes tachycardia and hypertension
125
Pituitary gland: ACTH and growth hormone (GH) is stimulated by hypothalamic releasing factors, corticotrophin releasing
factor (CRF) and somatotrophin (or growth hormone releasing factor). Perioperative increased prolactin secretion occurs by
release of inhibitory control. Secretion of thyroid stimulating hormone (TSH), luteinizing hormone (LH) and follicle stimulating
hormone (FSH) does not change significantly. ACTH stimulates cortisol production within a few minutes of the start of surgery.
More ACTH is produced than needed to produce a maximum adrenocortical response.

Cortisol: Significant increases within 4-6h of surgery (>1000 nmol litre-1). The usual negative feedback mechanism fails and
concentrations of ACTH and cortisol remain persistently increased. The magnitude and duration of the increase correlate with the
severity of stress and the response is not abolished by the administration of corticosteroids. The metabolic effects of cortisol are
enhanced: Skeletal muscle protein breakdown to provide gluconeogenic precursors and amino acids for protein. synthesis in the
liver. Stimulation of lipolysis'. Anti-insulin effect'. Mineralocorticoid effects. Anti-inflammatory effects

Growth hormone: Increased secretion after surgery has a minor role. Most important for preventing muscle protein breakdown
and promote tissue repair by insulin growth factors

Alpha Endorphin: Increased

Antidiuretic hormone: An important vasopressor and enhances haemostasis. Renin is released causing the conversion of
angiotensin I to angiotensin II, which causes the secretion of aldosterone from the adrenal cortex. This increases sodium
reabsorption at the distal convoluted tubule

Insulin: Release inhibited by stress. Occurs via the inhibition of the alpha cells in the pancreas by the α2-adrenergic inhibitory
effects of catecholamines. Insulin resistance by target cells occurs later. The perioperative period is characterized by a state of
functional insulin deficiency

Thyroxine (T4) and tri-iodothyronine (T3): Circulating concentrations are inversely correlated with sympathetic activity and
after surgery there is a reduction in thyroid hormone production, which normalises over a few days.

Metabolic effect of endocrine response

Carbohydrate metabolism: Hyperglycaemia is a main feature of the metabolic response to surgery.Due to increased increase in
glucose production and a reduction in glucose utilization. Catecholamines and cortisol promote glycogenolysis and
gluconeogenesis. Initial failure of insulin secretion followed by insulin resistance affects the normal responses. The proportion of
the hyperglycaemic response reflects the severity of surgery. Hyperglycaemia impairs wound healing and increase infection rates

Protein metabolism: Initially there is inhibition of protein anabolism, followed later, if the stress response is severe, by enhanced
catabolism. The amount of protein degradation is influenced by the type of surgery and also by the nutritional status of the patient.
Mainly skeletal muscle protein is affected. The amino acids released form acute phase proteins (fibrinogen, C reactive protein,
complement proteins, a2-macroglobulin, amyloid A and ceruloplasmin) and are used for gluconeogenesis. Nutritional support has
little effect on preventing catabolism

Lipid metabolism: Increased catecholamine, cortisol and glucagon secretion, and insulin deficiency, promotes lipolysis and ketone
body production.

Salt and water metabolismADH causes water retention, concentrated urine, and potassium loss and may continue for 3 to 5 days
after surgery. Renin causes sodium and water retention

Cytokines: Glycoproteins. Interleukins (IL) 1 to 17, interferons, and tumour necrosis factor. Synthesized by activated
macrophages, fibroblasts, endothelial and glial cells in response to tissue injury from surgery or trauma. IL-6 main cytokine
associated with surgery. Peak 12 to 24 h after surgery and increase by the degree of tissue damage Other effects of cytokines
include fever, granulocytosis, haemostasis, tissue damage limitation and promotion of healing.

Modifying the response: Opioids suppress hypothalamic and pituitary hormone secretion. At high doses the hormonal response
to pelvic and abdominal surgery is abolished. However, such doses prolong recovery and increase the need for postoperative
ventilatory support. Spinal anaesthesia can reduce the glucose, ACTH, cortisol, GH and epinephrine changes, although cytokine
responses are unaltered. Cytokine release is reduced in less invasive surgery. Nutrition prevents the adverse effects of the stress
response. Enteral feeding improves recovery. Growth hormone and anabolic steroids may improve outcome. Normothermia
decreases the metabolic response

A. The intrinsic pathway is the main pathway in coagulation

126
B. Heparin inhibits the activation of Factor 8
C. The activation of factor 8 is the point when the intrinsic and the extrinsic pathways meet
D. Tissue factor released by damaged tissue initiates the extrinsic pathway
E. Thrombin converts plasminogen to plasmin
Answer: D
The extrinsic pathway is the main path of coagulation. Heparin inhibits the activation of factors 2,9,10,11. The activation of factor
10 is when both pathways meet. Thrombin converts fibrinogen to fibrin. During fibrinolysis plasminogen is converted to plasmin
to break down fibrin.

Coagulation cascade: Two pathways lead to fibrin formation

Intrinsic pathway (components already present in the blood): Minor role in clotting. Subendothelial damage e.g. collagen.
Formation of the primary complex on collagen by high-molecular-weight kininogen (HMWK), prekallikrein, and Factor 12.
Prekallikrein is converted to kallikrein and Factor 12 becomes activated. Factor 12 activates Factor 11 . Factor 11 activates Factor
9, which with its co-factor Factor 8a form the tenase complex which activates Factor 10

Extrinsic pathway (needs tissue factor released by damaged tissue): Tissue damage. Factor 7 binds to Tissue factor. This
complex activates Factor 9. Activated Factor 9 works with Factor 8 to activate Factor 10

Common pathway: Activated Factor 10 causes the conversion of prothrombin to thrombin. Thrombin hydrolyses fibrinogen
peptide bonds to form fibrin and also activates factor 8 to form links between fibrin molecules

Fibrinolysis: Plasminogen is converted to plasmin to facilitate clot resorption

 Intrinsic pathway: Increased APTT: Factors 8,9,11,12


 Extrinsic pathway :Increased PT: Factor 7
 Common pathway: Increased APTT & PT: Factors 2,5,10
 Vitamin K dependent:Factors 2,7,9,10

2. Which of the following is not secreted by the parietal cells?


A. Hydrochloric acid
B. Mucus
C. Magnesium
D. Intrinsic factor
E. Calcium
Answer: B
Chief of Pepsi cola = Chief cells secrete PEPSInogen. Parietal cells: secrete HCl, Ca, Na, Mg and intrinsic factor. Chief cells:
secrete pepsinogen. Surface mucosal cells: secrete mucus and bicarbonate

Gastric secretions
A working knowledge of gastric secretions is important for surgery because peptic ulcers are common, surgeons frequently
prescribe anti secretory drugs and because there are still patients around who will have undergone acid lowering procedures
(Vagotomy) in the past.

Gastric acid: Is produced by the parietal cells in the stomach. pH of gastric acid is around 2 with acidity being maintained by the
H+/K+ ATP ase pump. As part of the process bicarbonate ions will be secreted into the surrounding vessels. Sodium and chloride
ions are actively secreted from the parietal cell into the canaliculus. This sets up a negative potential across the membrane and as a
result sodium and potassium ions diffuse across into the canaliculus.Carbonic anhydrase forms carbonic acid which dissociates
and the hydrogen ions formed by dissociation leave the cell via the H+/K+ antiporter pump. At the same time sodium ions are
actively absorbed. This leaves hydrogen and chloride ions in the canaliculus these mix and are secreted into the lumen of the
oxyntic gland.

Phases of gastric acid secretion: There are 3 phases of gastric secretion: 1. Cephalic phase (smell / taste of food): 30% acid
produced. Vagal cholinergic stimulation causing secretion of HCL and gastrin release from G cells; 2. Gastric phase (distension of
stomach ): 60% acid produced. Stomach distension/low H+/peptides causes Gastrin release; 3. Intestinal phase (food in
duodenum): 10% acid produced. High acidity/distension/hypertonic solutions in the duodenum inhibits gastric acid secretion via
enterogastrones (CCK, secretin) and neural reflexes. Regulation of gastric acid production: Factors increasing production
include:Vagal nerve stimulation. Gastrin release. Histamine release (indirectly following gastrin release) from enterchromaffin
like cells. Factors decreasing production include: Somatostatin (inhibits histamine release). Cholecystokinin. Secretin.

127
Source Stimulus Actions
Gastrin G cells in Distension of stomach, Increase HCL, pepsinogen and IF secretion, increases gastric
antrum of the extrinsic nerves motility, trophic effect on gastric mucosa
stomach Inhibited by: low
antral pH, somatostatin
CCK I cells in upper Partially digested Increases secretion of enzyme-rich fluid from pancreas, contraction
small intestine proteins and of gallbladder and relaxation of sphincter of Oddi, decreases gastric
triglycerides emptying, trophic effect on pancreatic acinar cells, induces satiety
Secretin S cells in upper Acidic chyme, fatty Increases secretion of bicarbonate-rich fluid from pancreas and
small intestine acids hepatic duct cells, decreases gastric acid secretion, trophic effect on
pancreatic acinar cells
VIP Small intestine, Neural Stimulates secretion by pancreas and intestines, inhibits acid and
pancreas pepsinogen secretion
Somatostatin D cells in the Fat, bile salts and Decreases acid and pepsin secretion, decreases gastrin secretion,
pancreas and glucose in the decreases pancreatic enzyme secretion, decreases insulin and
stomach intestinal lumen glucagon secretion
inhibits trophic effects of gastrin, stimulates gastric mucous
production

3. A 45 year old male is diagnosed with carcinoma of the head of the pancreas. He reports that his stool sticks to the
commode and will not flush away. Loss of which of the following enzymes is most likely to be responsible for this
problem?
A. Lipase
B. Amylase
C. Trypsin
D. Elastase
E. None of the above
Answer: A
Loss of lipase is one of the key features in the development of steatorrhoea which typically consists of pale and offensive stools
that are difficult to flush away.

Pancreatic cancer: Adenocarcinoma. Risk factors: Smoking, diabetes, Adenoma, Familial adenomatous polyposis. Mainly occur
in the head of the pancreas (70%). Spread locally and metastasizes to the liver. Carcinoma of the pancreas should be differentiated
from other periampullary tumours with better prognosis

Clinical features: Weight loss. Painless jaundice. Epigastric discomfort (pain usually due to invasion of the coeliac plexus is a
late feature). Pancreatitis. Trousseau's sign: migratory superficial thrombophlebitis
Investigations: USS: May miss small lesions. CT Scanning (pancreatic protocol). If unresectable on CT then no further staging
needed.. PET/CT for those with operable disease on CT alone. ERCP/ MRI for bile duct assessment. Staging laparoscopy to
exclude peritoneal disease.

Management: Head of pancreas: Whipple's resection (SE dumping and ulcers). Newer techniques include pylorus preservation
and SMA/ SMV resection. Carcinoma body and tail: poor prognosis, distal pancreatectomy if operable. Usually adjuvent
chemotherapy for resectable disease. ERCP and stent for jaundice and palliation.Surgical bypass may be needed for duodenal
obstruction.

4. Which of the following is not well absorbed following a gastrectomy?


A. Vitamin c
B. Zinc
C. Vitamin B12
D. Copper
E. Molybdenum
Answer: C
Vitamin B12. The others are unaffected
Post gastrectomy syndrome:
Rapid emptying food from stomach into the duodenum: diarrhoea, abdominal pain, hypoglycaemia
Complications: Vitamin B12 and iron malabsorption, osteoporosis
Treatment: High protein, low carbohydrate diet. Replace B12/Fe/Ca

Gastric emptying: The stomach serves both a mechanical and immunological function. Solid and liquid are retained in the
stomach during which time repeated peristaltic activity against a closed pyloric sphincter will cause fragmentation of food bolus
material. Contact with gastric acid will help to neutralise any pathogens present. The amount of time material spends in the

128
stomach is related to its composition and volume. For example a glass of water will empty more quickly than a large meal. The
presence of amino acids and fat will all serve to delay gastric emptying.

Controlling factor: Neuronal stimulation of the stomach is mediated via the vagus and the parasympathetic nervous system will
tend to favor an increase in gastric motility. It is for this reason that individuals who have undergone truncal vagotomy will tend to
routinely require either a pyloroplasty or gastro-enterostomy as they would otherwise have delayed gastric emptying. The
following hormonal factors are all involved: Delay emptying: Gastric inhibitory peptide, Cholecystokinin and Enteroglucagon.
Increase emptying :Gastrin.

Diseases affecting gastric emptying: All diseases that affect gastric emptying may result in bacterial overgrowth, retained food
and eventually the formation of bezoars that may occlude the pylorus and make gastric emptying even worse. Fermentation of
food may cause dyspepsia, reflux and foul smelling belches of gas.

Iatrogenic: Gastric surgery can have profound effects on gastric emptying. As stated above any procedure that disrupts the vagus
can cause delayed emptying. Whilst this is particularly true of Vagotomy this operation is now rarely performed. Surgeons are
divided on the importance of vagal disruption that occurs during an oesophagectomy and some will routinely perform a
pyloroplasty and other will not.
When a distal gastrectomy is performed the type of anastomosis performed will impact on emptying. When a gastro-enterostomy
is constructed, a posterior, retrocolic gastroenterostomy will empty better than an anterior one.

Diabetic gastroparesis: This is predominantly due to neuropathy affecting the vagus nerve. The stomach empties poorly and
patients may have episodes of repeated and protracted vomiting. Diagnosis is made by upper GI endoscopy and contrast studies,
in some cases a radio nucleotide scan is needed to demonstrate the abnormality more clearly. In treating these conditions drugs
such as metoclopramide will be less effective as they exert their effect via the vagus nerve. One of the few prokinetic drugs that do
not work in this way is the antibiotic erythromycin.

Malignancies: Obviously a distal gastric cancer may obstruct the pylorus and delay emptying. In addition malignancies of the
pancreas may cause extrinsic compression of the duodenum and delay emptying. Treatment in these cases is by gastric
decompression using a wide bore nasogastric tube and insertion of a stent or if that is not possible by a surgical gastroenterostomy.
As a general rule gastroenterostomies constructed for bypass of malignancy are usually placed on the anterior wall of the stomach
(in spite of the fact that they empty less well). A Roux en Y bypass may also be undertaken but the increased number of
anastomoses for this in malignant disease that is being palliated is probably not justified.

Congenital Hypertrophic Pyloric Stenosis: This is typically a disease of infancy. Most babies will present around 6 weeks of
age with projectile non bile stained vomiting. It has an incidence of 2.4 per 1000 live births and is more common in males.
Diagnosis is usually made by careful history and examination and a mass may be palpable in the epigastrium (often cited seldom
felt!). The most important diagnostic test is an ultrasound that usually demonstrates the hypertrophied pylorus. Blood tests may
reveal a hypochloraemic metabolic alkalosis if the vomiting is long standing. Once the diagnosis is made the infant is resuscitated
and a pyloromyotomy is performed (usually laparoscopically). Once treated there are no long term sequelae.

5. Which vitamin is involved in the formation of collagen?


A. Vitamin A
B. Vitamin B
C. Vitamin C
D. Vitamin D
E. Vitamin E
Answer: C
Vitamin C is needed for the hydroxylation of proline during collagen synthesis.

Collagen: One of the major connective tissue proteins. Composed of 3 polypeptide strands that are woven into a helix. Numerous
hydrogen bonds exist within molecule to provide additional strength. Many sub types but commonest sub type is I (90% of bodily
collagen). Vitamin c is important in establishing cross links

Collagen Diseases: Osteogenesis imperfect. Ehlers Danlos

Osteogenesis imperfecta: 8 Subtypes: Defect of type I collagen. In type I the collagen is normal quality but insufficient quantity.
Type II- poor quantity and quality. Type III- Collagen poorly formed, normal quantity. Type IV- Sufficient quantity but poor
quality. Patients have bones which fracture easily, loose joint and multiple other defects depending upon which sub type they
suffer from
Ehlers Danlos: Multiple sub types. Abnormality of types 1 and 3 collagen. Patients have features of hypermobility. Individuals
are prone to joint dislocations and pelvic organ prolapse. In addition to many other diseases related to connective tissue defects

129
6. A 56 year old man has long standing chronic pancreatitis and develops pancreatic insufficiency. Which of the following
will be absorbed normally?
A. Fat
B. Protein
C. Folic acid
D. Vitamin B12
E. None of the above
Answer: C
Pancreatic lipase is required for digestion of fat, Proteases facilitate protein and B12 absorption. Folate digestion is independent of
the pancreas.

Pancreatic juice: Alkaline solution pH 8. 1500ml/day. Composition: acinar secretion (ENZYMES: trypsinogen, procarboxylase,
amylase, lecithin) and ductile secretion (HCO, Na+, water). Pancreatic juice action: Trypsinogen is converted via enterokinase to
active trypsin in the duodenum. Trypsin then activates the other inactive enzymes.

7. A 56 year old male presents to the acute surgical take with severe abdominal pain. He is normally fit and well. He has
no malignancy. The biochemistry laboratory contacts the ward urgently, his corrected calcium result is 3.6 mmol/l.
What is the medication of choice to treat this abnormality?

A. IV Pamidronate
B. Oral Alendronate
C. Dexamethasone
D. Calcitonin
E. IV Zoledronate
Answer: A
IV Pamidronate is the drug of choice as it most effective and has long lasting effects. Calcitonin would need to be given with
another agent, to ensure that the hypercalcaemia is treated once its short term effects wear off. IV zoledronate is preferred in
scenarios associated with malignancy.

Management of hypercalcaemia: Free Ca is affected by pH (increased in acidosis) and plasma albumin concentration. ECG
changes include: Shortening of QTc interval. Urgent management is indicated if: Calcium > 3.5 mmol/l, reduced consciousness,
severe abdominal pain.
Pre renal failure

Management: Airway Breathing Circulation, Intravenous fluid resuscitation with 3-6L of 0.9% Normal saline in 24h, After
hydration, give frusemide (to encourage excretion of Ca), Medical therapy (usually if Corrected calcium >3.0mmol/l)

Bisphosphonates:Analogues of pryrophosphate. Prevent osteoclast attachment to bone matrix and interfere with osteoclast
activity. Inhibit bone resorption.

Agents

Drug Side effects Notes


IV Pamidronate pyrexia, leucopaenia Most potent agent
IV Zoledronate response lasts 30 days Used for malignancy associated hypercalcaemia

Calcitonin: Quickest onset of action however short duration (tachyphylaxis) therefore only given with a second agent.
Prenisolone: May be given in hypercalcaemia related to sarcoidosis, myeloma or vitamin D intoxication.

8. An over enthusiastic medical student decides to ask you questions about ECGs. Rather than admitting your dwindling
knowledge on this topic, you bravely attempt to answer her questions! One question is what segment of the ECG
represents ventricular repolarization?
A. QRS complex
B. Q-T interval
C. P wave
D. T wave
E. S-T segment

Answer: D
The T wave represents ventricular repolarization. The common sense approach to remembering this, is to acknowledge that
ventricular repolarization is the last phase of cardiac contraction and should therefore correspond the the last part of the ECG.

130
The normal ECG
P wave: Represents the wave of depolarization that spreads from the SA node throughout the atria. Lasts 0.08 to 0.1 seconds (80-
100 ms) . The isoelectric period after the P wave represents the time in which the impulse is traveling within the AV node . P-R
interval: Time from the onset of the P wave to the beginning of the QRS complex. Ranges from 0.12 to 0.20 seconds in duration.
Represents the time between the onset of atrial depolarization and the onset of ventricular depolarization. QRS complex:
Represents ventricular depolarization. Duration of the QRS complex is normally 0.06 to 0.1 seconds. ST segment: Isoelectric
period following the QRS. Represents period which the entire ventricle is depolarized and roughly corresponds to the plateau
phase of the ventricular action potential. T wave: Represents ventricular repolarization and is longer in duration than
depolarization. A small positive U wave may follow the T wave which represents the last remnants of ventricular repolarization.
Q-T interval: Represents the time for both ventricular depolarization and repolarization to occur, and therefore roughly estimates
the duration of an average ventricular action potential. Interval ranges from 0.2 to 0.4 seconds depending upon heart rate. At high
heart rates, ventricular action potentials shorten in duration, which decreases the Q-T interval. Therefore the Q-T interval is
expressed as a "corrected Q-T (QTc)" by taking the Q-T interval and dividing it by the square root of the R-R interval (interval
between ventricular depolarizations). This allows an assessment of the Q-T interval that is independent of heart rate. Normal
corrected Q-Tc interval is less than 0.44 seconds.

9. The oxygen-haemoglobin dissociation curve is shifted to the right in which of the following scenarios?
A. Hypothermia
B. Respiratory alkalosis
C. Low altitude
D. Decreased 2,3-DPG in transfused red cells
E. Chronic iron deficiency anaemia
Answer: E
Mnemonic to remember causes of right shift of the oxygen dissociation curve:
CADET face RIGHT: C O2. A cidosis. 2,3-DPG. E xercise. T emperature

The curve is shifted to the right when there is an increased oxygen requirement by the tissue. This includes: Increased
temperature. Acidosis. Increased DPG: it is found in erythrocytes and is increased during glycolysis. It binds to the Hb molecule,
thereby releasing oxygen to tissues. DPG is increased in conditions associated with poor oxygen delivery to tissues, such as
anaemia and high altitude.

Oxygen transport: Almost all oxygen is transported within erythrocytes. It has limited solubility and only 1% is carried as
solution. Therefore the amount of oxygen transported will depend upon haemoglobin concentration and its degree of saturation.

Haemoglobin: Globular protein composed of 4 subunits. Haem consists of a protoporphyrin ring surrounding an iron atom in its
ferrous state. The iron can form two additional bonds; one with oxygen and the other with a polypeptide chain. There are two
alpha and two beta subunits to this polypeptide chain in an adult and together these form globin. Globin cannot bind oxygen but is
able to bind to carbon dioxide and hydrogen ions, the beta chains are able to bind to 2,3 diphosphoglycerate. The oxygenation of
haemoglobin is a reversible reaction. The molecular shape of haemoglobin is such that binding of one oxygen molecule facilitates
the binding of subsequent molecules.

Oxygen dissociation curve: The oxygen dissociation curve describes the relationship between the percentage of saturated
haemoglobin and partial pressure of oxygen in the blood. It is not affected by haemoglobin concentration. Chronic anaemia causes
2, 3 DPG levels to increase, hence shifting the curve to the right. Bohr effect: Shifts to left = for given oxygen tension there is
increased saturation of Hb with oxygen i.e. Decreased oxygen delivery to tissues. Shifts to right = for given oxygen tension there
is reduced saturation of Hb with oxygen i.e. Enhanced oxygen delivery to tissues

Shifts to Left = Lower oxygen delivery: HbF, methaemoglobin, carboxyhaemoglobin; low [H+] (alkali); low pCO2; low 2,3-
DPG.
low temperature
Shifts to Right = Raised oxygen delivery: raised [H+] (acidic); raised pCO2; raised 2,3-DPG*; raised temperature

10. A 73 year old lady is admitted for a laparoscopic cholecystectomy. During her pre-operative assessment it is noted that
she is receiving furosemide for the treatment of hypertension. Where is the site of action of this diuretic?
A. Proximal convoluted tubule
B. Descending limb of the loop of Henle
C. Ascending limb of the loop of Henle
D. Distal convoluted tubule
E. Collecting ducts
Answer: C

131
Action of furosemide = ascending limb of the loop of Henle
Furosemide and bumetanide are loop diuretics that act by inhibiting the Na-K-Cl cotransporter in the thick ascending limb of the
loop of Henle, reducing the absorption of NaCl.
Diuretic agents: The diuretic drugs are divided into three major classes, which are distinguished according to the site at which
they impair sodium reabsorption: loop diuretics in the thick ascending loop of Henle, thiazide type diuretics in the distal tubule
and connecting segment; and potassium sparing diuretics in the aldosterone - sensitive principal cells in the cortical collecting
tubule.
In the kidney, sodium is reabsorbed through Na+/ K+ ATPase pumps located on the basolateral membrane. These pumps return
reabsorbed sodium to the circulation and maintain low intracellular sodium levels. This latter effect ensures a constant
concentration gradient.

Physiological effects of commonly used diuretics


Site of action Diuretic Carrier or channel Percentage of filtered sodium
inhibited excreted
Ascending limb of loop of Henle Frusemide Na+/K+ 2Cl - carrier Upt to 25%
Distal tubule and connecting Thiazides Na+Cl- carrier Between 3 and 5%
segment
Cortical collecting tubule Spironolactone Na+ channel Between 1 and 2%

11. A 45 year old man is referred to the breast clinic with gynaecomastia. He takes the drugs listed below. Which is least
likely to be the cause of his symptoms?
A. Spironolactone
B. Carbimazole
C. Chlorpromazine
D. Cimetidine
E. Methyldopa
Answer: B
Mnemonic for drugs causing gynaecomastia: DISCO: D igitalis, I soniazid, S pironolactone, C imentidine, O estrogen
Mnemonic for causes of gynaecomastia: METOCLOPRAMIDE: M etoclopramide. E ctopic oestrogen, T rauma skull/tumour
breast, testes, O rchitis, C imetidine, Cushings, L iver cirrhosis, O besity, P araplegia
R A, A cromegaly, M ethyldopa, I soniazid, D igoxin, E thionamide. Carbimazole is not associated with gynaecomastia.

GynaecomastiaGynaecomastia describes an abnormal amount of breast tissue in males and is usually caused by an increased
oestrogen:androgen ratio. It is important to differentiate the causes of galactorrhoea (due to the actions of prolactin on breast
tissue) from those of gynaecomastia

Causes of gynaecomastia: physiological: normal in puberty. Syndromes with androgen deficiency: Kallman's, Klinefelter's.
Testicular failure: e.g. Mumps. Liver disease. Testicular cancer e.g. Seminoma secreting hCG. Ectopic tumour secretion.
Hyperthyroidism. Haemodialysis. Drugs: see below

Drug causes of gynaecomastia: spironolactone (most common drug cause). Cimetidine. Digoxin. Cannabis. Finasteride.
Oestrogens, anabolic steroids. Very rare drug causes of gynaecomastia: Tricyclics. Isoniazid. Calcium channel blockers.
Heroin. Busulfan Methyldopa

12. 43 year old lady is recovering on the intensive care unit following a Whipples procedure. She has a central venous line
in situ. Which of the following will lead to the "y" descent on the waveform trace?
A. Ventricular contraction
B. Emptying of the right atrium
C. Emptying of the right ventricle
D. Opening of the pulmonary valve
E. Cardiac tamponade
Answer: B
JVP
3 Upward deflections and 2 downward deflections. Upward deflections: a wave = atrial contraction; c wave = ventricular
contraction; v wave = atrial venous filling. Downward deflections: x wave = atrium relaxes and tricuspid valve moves down; y
wave = ventricular filling

The 'y' descent represents the emptying of the atrium and the filling of the right ventricle.

Cardiac physiology: The heart has four chambers ejecting blood into both low pressure and high pressure systems. The pumps
generate pressures of between 0-25mmHg on the right side and 0-120 mmHg on the left. At rest diastole comprises 2/3 of the
cardiac cycle. The product of the frequency of heart rate and stroke volume combine to give the cardiac output which is typically
5-6L per minute.

132
Detailed descriptions of the various waveforms are often not a feature of MRCS A (although they are on the syllabus). However,
they are a very popular topic for surgical physiology vivas in the oral examination.

Electrical properties: Intrinsic myogenic rhythm within cardiac myocytes means that even the denervated heart is capable of
contraction. In the normal situation the cardiac impulse is generated in the sino atrial node in the right atrium and conveyed to the
ventricles via the atrioventricular node. The sino atrial node is also capable of spontaneous discharge and in the absence of
background vagal tone will typically discharge around 100x per minute. Hence the higher resting heart rate found in cardiac
transplant cases. In the SA and AV nodes the resting membrane potential is lower than in surrounding cardiac cells and will
slowly depolarise from -70mV to around -50mV at which point an action potential is generated. Differences in the depolarisation
slopes between SA and AV nodes help to explain why the SA node will depolarise first. The cells have a refractory period during
which they cannot be re-stimulated and this period allows for adequate ventricular filling. In pathological tachycardic states this
time period is overridden and inadequate ventricular filling may then occur, cardiac output falls and syncope may ensue.
Parasympathetic fibres project to the heart via the vagus and will release acetylcholine. Sympathetic fibres release nor adrenaline
and circulating adrenaline comes from the adrenal medulla. Noradrenaline binds to β 1 receptors in the SA node and increases the
rate of pacemaker potential depolarisation.

Cardiac cycle
Mid diastole: AV valves open. Ventricles hold 80% of final volume. Outflow valves shut. Aortic pressure is high. Late diastole:
Atria contract. Ventricles receive 20% to complete filling. Typical end diastolic volume 130-160ml. Early systole: AV valves
shut. Ventricular pressure rises. Isovolumetric ventricular contraction. AV Valves bulge into atria (c-wave). Aortic and pulmonary
pressure exceeded- blood is ejected. Shortening of ventricles pulls atria downwards and drops intra atrial pressure (x-descent).
Late systole: Ventricular muscles relax and ventricular pressures drop. Although ventricular pressure drops the aortic pressure
remains constant owing to peripheral vascular resistance and elastic property of the aorta. Brief period of retrograde flow that
occurs in aortic recoil shuts the aortic valve. Ventricles will contain 60ml end systolic volume. The average stroke volume is 70ml
(i.e. Volume ejected). Early diastole: All valves are closed. Isovolumetric ventricular relaxation occurs. Pressure wave associated
with closure of the aortic valve increases aortic pressure. The pressure dip before this rise can be seen on arterial waveforms and is
called the incisura. During systole the atrial pressure increases such that it is now above zero (v- wave). Eventually atrial pressure
exceed ventricular pressure and AV valves open - atria empty passively into ventricles and atrial pressure falls (y -descent ). The
negative atrial pressures are of clinical importance as they can allow air embolization to occur if the neck veins are exposed to air.
This patient positioning is important in head and neck surgery to avoid this occurrence if veins are inadvertently cut, or during
CVP line insertion.

Mechanical properties: Preload = end diastolic volume; Afterload = aortic pressure. It is important to understand the principles
of Laplace's law in surgery. It states that for hollow organs with a circular cross section, the total circumferential wall tension
depends upon the circumference of the wall, multiplied by the thickness of the wall and on the wall tension. The total luminal
pressure depends upon the cross sectional area of the lumen and the transmural pressure. Transmural pressure is the internal
pressure minus external pressure and at equilibrium the total pressure must counterbalance each other. In terms of cardiac
physiology the law explains that the rise in ventricular pressure that occurs during the ejection phase is due to physical change in
heart size. It also explains why a dilated diseased heart will have impaired systolic function.
Starlings law: Increase in end diastolic volume will produce larger stroke volume. This occurs up to a point beyond which cardiac
fibres are excessively stretched and stroke volume will fall once more. It is important for the regulation of cardiac output in
cardiac transplant patients who need to increase their cardiac output.

Baroreceptor reflexes: Baroreceptors located in aortic arch and carotid sinus. Aortic baroreceptor impulses travel via the vagus
and from the carotid via the glossopharyngeal nerve. They are stimulated by arterial stretch. Even at normal blood pressures they
are tonically active.Increase in baroreceptor discharge causes: Increased parasympathetic discharge to the SA node, Decreased
sympathetic discharge to ventricular muscle causing decreased contractility and fall in stroke volume, Decreased sympathetic
discharge to venous system causing increased compliance and Decreased peripheral arterial vascular resistance

Atrial stretch receptors: Located in atria at junction between pulmonary veins and vena cava.Stimulated by atrial stretch and are
thus low pressure sensors. Increased blood volume will cause increased parasympathetic activity.Very rapid infusion of blood will
result in increase in heart rate mediated via atrial receptors: the Bainbridge reflex. Decreases in receptor stimulation results in
increased sympathetic activity this will decrease renal blood flow-decreases GFR-decreases urinary sodium excretion-renin
secretion by juxtaglomerular apparatus-Increase in angiotensin II. Increased atrial stretch will also result in increased release of
atrial natriuretic peptide.

13. Which of the following are not characteristic features of central chemoreceptors in the control of ventilation?
A. They are located in the medulla oblongata
B. They are stimulated primarily by venous hypercapnia
C. They are relatively insensitive to hypoxia
D. They are less sensitive to changes in arterial pH than other ventillatory receptors
E. During acute hypercapnia the carotid receptors will be stimulated first
Answer: B

133
They are stimulated by arterial carbon dioxide. It takes longer to equilibrate than the peripheral chemoreceptors located in the
carotid. They are less sensitive to acidity due to the blood brain barrier.

Control of ventilation: Control of ventilation is coordinated by the respiratory centres, chemoreceptors, lung receptors and
muscles. Automatic, involuntary control of respiration occurs from the medulla. The respiratory centres control the respiratory
rate and the depth of respiration.

Respiratory centres

Medullary respiratory centre: Inspiratory and expiratory neurones. Has ventral group which controls forced voluntary
expiration and the dorsal group controls inspiration. Depressed by opiates.
Apneustic centre: Lower pons, Stimulates inspiration - activates and prolongs inhalation, Overridden by pneumotaxic control to
end inspiration, Pneumotaxic centre:Upper pons, inhibits inspiration at a certain point. Fine tunes the respiratory rate, Levels of
PCO2 most important in ventilation control, Levels of O2 are less important. Peripheral chemoreceptors: located in the
bifurcation of carotid arteries and arch of the aorta. They respond to changes in reduced pO2, increased H+ and increased pCO2 in
ARTERIAL BLOOD. Central chemoreceptors: located in the medulla. Respond to increased H+ in BRAIN INTERSTITIAL
FLUID to increase ventilation. NB the central receptors are NOT influenced by O2 levels. Lung receptors include: Stretch
receptors: respond to lung stretching causing a reduced respiratory rate. Irritant receptors: respond to smoke etc causing
bronchospasm. J (juxtacapillary) receptors

14. A 32 year old man has a glomerular filtration rate of 110ml / minute at a systolic blood pressure of 120/80. If his blood
pressure were to fall to 100/70 what would glomerular filtration rate be?
A. 110ml / minute
B. 100ml/ minute
C. 55ml/ minute
D. 25ml/ minute
E. 75ml/ minute
Answer: A

The proposed drop in blood pressure falls within the range within which the kidney autoregulates its blood supply. GFR will
therefore remain unchanged.

Each nephron is supplied with blood from an afferent arteriole that opens onto the glomerular capillary bed. Blood then flows to
an efferent arteriole, supplying the peritubular capillaries and medullary vasa recta. The kidney receives up to 25% of resting
cardiac output.

Control of blood flow:The kidney is able to autoregulate its blood flow between systolic pressures of 80- 180mmHg so there is
little variation in renal blood flow. This is achieved by myogenic control of arteriolar tone, both sympathetic input and hormonal
signals (e.g. renin) are responsible.

Glomerular structure and function: Blood inside the glomerulus has considerable hydrostatic pressure. The basement
membrane has pores that will allow free diffusion of smaller solutes, larger negatively charged molecules such as albumin are
unable to cross. The glomerular filtration rate (GFR) is equal to the concentration of a solute in the urine, times the volume of
urine produced per minute, divided by the plasma concentration (assuming that the solute is freely diffused e.g. inulin). In clinical
practice creatinine is used because it is subjected to very little proximal tubular secretion. Although subject to variability, the
typical GFR is 125ml per minute. Glomerular filtration rate = Total volume of plasma per unit time leaving the capillaries and
entering the bowman's capsule. Renal clearance = volume plasma from which a substance is removed per minute by the kidneys

Substances used to measure GFR have the following features:1. Inert; 2. Free filtration from the plasma at the glomerulus (not
protein bound); 3. Not absorbed or secreted at the tubules; 4. Plasma concentration constant during urine collection

Examples: inulin, creatinine

GFR = urine concentration (mmol/l) x urine volume (ml/min)


plasma concentration (mmol/l)

The clearance of a substance is dependent not only on its diffusivity across the basement membrane but also subsequent tubular
secretion and / or reabsorption. So glucose which is freely filtered across the basement membrane is usually reabsorbed from
tubules giving a clearance of zero.

134
Tubular function: Reabsorption and secretion of substances occurs in the tubules. In the proximal tubule substrates such as
glucose, amino acids and phosphate are co-transported with sodium across the semi permeable membrane. Up to two thirds of
filtered water is reabsorbed in the proximal tubules. This will lead to increase in urea concentration in the distal tubule allowing
for its increased diffusion. Substances to be secreted into the tubules are taken up from the peritubular blood by tubular cells.
Solutes such as paraaminohippuric acid are cleared with a single passage through the kidneys and this is why it is used to measure
renal plasma flow. Ions such as calcium and phosphate will have a tubular reabsorption that is influenced by plasma PTH levels.
Potassium may be both secreted and re-absorbed and is co-exchanged with sodium.

Loop of Henle: Approximately 60 litres of water containing 9000mmol sodium enters the descending limb of the loop of Henle in
24 hours. Loops from the juxtamedullary nephrons run deep into the medulla. The osmolarity of fluid changes and is greatest at
the tip of the papilla. The thin ascending limb is impermeable to water, but highly permeable to sodium and chloride ions. This
loss means that at the beginning of the thick ascending limb the fluid is hypo osmotic compared with adjacent interstitial fluid. In
the thick ascending limb the reabsorption of sodium and chloride ions occurs by both facilitated and passive diffusion pathways.
The loops of Henle are co-located with vasa recta, these will have similar solute compositions to the surrounding extracellular
fluid so preventing the diffusion and subsequent removal of this hypertonic fluid. The energy dependent reabsorption of sodium
and chloride in the thick ascending limb helps to maintain this osmotic gradient.

15. Which of the following does not stimulate insulin release?


A. Gastrin
B. Atenolol
C. Protein
D. Secretin
E. Vagal cholinergic action
Answer: B
Beta blockers inhibit the release of insulin. Stimulation of insulin release:Glucose, Amino acid, Vagal cholinergic,
Secretin/Gastrin/CCK, Fatty acids, Beta adrenergic drugs.

Insulin is an Anabolic hormone. Synthesis: Pro-insulin is formed by the rough endoplasmic reticulum in pancreatic beta cells.
Then pro-insulin is cleaved to form insulin and C-peptide. Insulin is stored in secretory granules and released in response to Ca.
Function: Secreted in response to hyperglycaemia, Glucose utilisation and glycogen synthesis, Inhibits lipolysis, Reduces muscle
protein loss

16. A 63 year old female is referred to the surgical clinic with an iron deficiency anaemia. Her past medical history
includes a left hemi colectomy but no other co-morbidities. At what site is most dietery iron absorbed?
A. Stomach
B. Duodenum
C. Proximal ileum
D. Distal ileum
E. Colon
Ansewr: B
Iron is best absorbed from the proximal small bowel (duodenum and jejunum) in the Fe 2+ state. Iron is transported across the
small bowel mucosa by a divalent membrane transporter protein (hence the improved absorption of F2 2+. The intestinal cells
typically store the bound iron as ferritin. Cells requiring iron will typically then absorb the complex as needed.

Iron metabolism: absorption Duodenum and upper jejunum; About 10% of dietary iron absorbed; Fe2+ (ferrous iron) much
better absorbed than Fe3+ (ferric iron); Ferrous iron is oxidized to form ferric iron, which is combined with apoferritin to form
ferritin; Absorption is regulated according to body's need; Increased by vitamin C, gastric acid; Decreased by proton pump
inhibitors, tetracycline, gastric achlorhydria, tannin (found in tea). Transport: In plasma as Fe3+ bound to transferrin.
Storage:Ferritin (or haemosiderin) in bone marrow. Excretion: Lost via intestinal tract following desquamation
Distribution in body
Total body iron:4g; Haemoglobin: 70%; Ferritin and haemosiderin: 25%; Myoglobin: 4%; Plasma iron: 0.1%

17. Which of the following drugs increases the rate of gastric emptying in the vagotomised stomach?
A. Ondansetron
B. Metoclopramide
C. Cyclizine
D. Erythromycin
E. Chloramphenicol
Answer: D
Vagotomy seriously compromises gastric emptying which is why either a pyloroplasty or gastro-enterostomy is routinely
performed at the same time.
135
Chloramphenicol has no effect on gastric emptying. Ondansetron slows gastric emptying slightly. Metoclopramide increases the
rate of gastric emptying but its effects are mediated via the vagus nerve.

18. Which of the following haemodynamic changes is not seen in hypovolaemic shock?
A. Decreased cardiac output
B. Increased heart rate
C. Reduced left ventricle filling pressures
D. Reduced blood pressure
E. Reduced systemic vascular resistance
Answer: E
Cardiogenic Shock: e.g. MI, valve abnormality

increased SVR (vasoconstriction in response to low BP); increased HR (sympathetic response); decreased cardiac; output;
decreased blood pressure

Hypovolaemic shock: blood volume depletion e.g. haemorrhage, vomiting, diarrhoea, dehydration, third-space losses during
major operations; increased SVR; increased HR; decreased cardiac output
decreased blood pressure

Septic shock: occurs when the peripheral vascular dilatation causes a fall in SVR . Similar response may occur in anaphylactic
shock, neurogenic shock. reduced SVR; increased HR; normal/increased cardiac output; decreased blood pressure; SVR will
typically increase

Shock:Shock occurs when there is insufficient tissue perfusion. The pathophysiology of shock is an important surgical topic and
may be divided into the following aetiological groups:Septic, Haemorrhagic, Neurogenic, Cardiogenic, Anaphylactic

Septic shock: Septic shock is a major problem and those patients with severe sepsis have a mortality rate in excess of 40%. In
those who are admitted to intensive care mortality ranges from 6% with no organ failure to 65% in those with 4 organ failure.
Sepsis is defined as an infection that triggers a particular Systemic Inflammatory Response Syndrome (SIRS). This is
characterised by body temperature outside 36 oC - 38 o C, HR >90 beats/min, respiratory rate >20/min, WBC count >12,000/mm3
or < 4,000/mm3. Patients with infections and two or more elements of SIRS meet the diagnostic criteria for sepsis. Those with
organ failure have severe sepsis and those with refractory hypotension -septic shock. During the septic process there is marked
activation of the immune system with extensive cytokine release. This may be coupled with or triggered by systemic circulation
of bacterial toxins. These all cause endothelial cell damage and neutrophil adhesion. The overall hallmarks are thus those of
excessive inflammation, coagulation and fibrinolytic suppression.

The surviving sepsis campaign highlights the following key areas for attention: Prompt administration of antibiotics to cover all
likely pathogens coupled with a rigorous search for the source of infection. Haemodynamic stabilisation. Many patients are
hypovolaemic and require aggressive fluid administration. Aim for CVP 8-12 cm H2O, MAP >65mmHg. Modulation of the septic
response. This includes manoeuvres to counteract the changes and includes measures such as tight glycaemic control, use of
activated protein C and sometimes intravenous steroids.In surgical patients, the main groups with septic shock include those with
anastomotic leaks, abscesses and extensive superficial infections such as necrotising fasciitis. When performing surgery the aim
should be to undertake the minimum necessary to restore physiology. These patients do not fare well with prolonged surgery.
Definitive surgery can be more safely undertaken when physiology is restored and clotting in particular has been normalised.

Haemorrhagic shock: The average adult blood volume comprises 7% of body weight. Thus in the 70 Kg adult this will equate to
5 litres. This changes in children (8-9% body weight) and is slightly lower in the elderly.
The table below outlines the 4 major classes of haemorrhagic shock and their associated physiological sequelae:

Parameter Class I Class II Class III Class IV


Blood loss ml <750ml 750-1500ml 1500-2000ml >2000ml
Blood loss % <15% 15-30% 30-40% >40%
Pulse rate <100 >100 >120 >140ml
Blood pressure Normal Decreased Decreased Decreased
Respiratory rate 14-20 20-30 30-40 >35
Urine output >30ml 20-30ml 5-15ml <5ml
Symptoms Normal Anxious Confused Lethargic

Decreasing blood pressure during haemorrhagic shock causes organ hypoperfusion and relative myocardial ishaemia. The cardiac
index gives a numerical value for tissue oxygen delivery and is given by the equation: Cardiac index= 13.4 - [Hb] - SaO2 + 0.03
PaO2. Where Hb is haemoglobin concentration in blood and SaO2 the saturation and PaO2 the partial pressure of oxygen. Detailed
knowledge of this equation is required for the MRCS Viva but not for part A, although you should understand the principle.

In patients suffering from trauma the most likely cause of shock is haemorrhage. However, the following may also be the cause or
136
occur concomitantly:Tension pneumothorax, Spinal cord injury, Myocardial contusion, Cardiac tamponade. When assessing
trauma patients it is worth remembering that in order to generate a palpable femoral pulse an arterial pressure of >65mmHg is
required. Once bleeding is controlled and circulating volume normalised the levels of transfusion should be to maintain a Hb of 7-
8 in those with no risk factors for tissue hypoxia and Hb 10 for those who have such risk factors.

Neurogenic shock: This occurs most often following a spinal cord transection, usually at a high level. There is resultant
interruption of the autonomic nervous system. The result is either decreased sympathetic tone or increased parasympathetic
tone, the effect of which is a decrease in peripheral vascular resistance mediated by marked vasodilation. This results in
decreased preload and thus decreased cardiac output (Starlings law). There is decreased peripheral tissue perfusion and shock is
thus produced. In contrast with many other types of shock peripheral vasoconstrictors are used to return vascular tone to normal.

Cardiogenic shock: In medical patients the main cause is ischaemic heart disease. In the traumatic setting direct myocardial
trauma or contusion is more likely. Evidence of ECG changes and overlying sternal fractures or contusions should raise the
suspicion of injury. Treatment is largely supportive and transthoracic echocardiography should be used to determine evidence of
pericardial fluid or direct myocardial injury. The measurement of troponin levels in trauma patients may be undertaken but they
are less useful in delineating the extent of myocardial trauma than following MI. When cardiac injury is of a blunt nature and is
associated with cardiogenic shock the right side of the heart is the most likely site of injury with chamber and or valve rupture.
These patients require surgery to repair these defects and will require cardiopulmonary bypass to achieve this. Some may require
intra aortic balloon pump as a bridge to surgery.

Anaphylactic shock: Anaphylaxis may be defined as a severe, life-threatening, generalised or systemic


hypersensitivity reaction. Anaphylaxis is one of the few times when you would not have time to look up the dose of a medication.
The Resuscitation Council guidelines on anaphylaxis have recently been updated. Adrenaline is by far the most important drug in
anaphylaxis and should be given as soon as possible. The recommended doses for adrenaline, hydrocortisone and chlorphenamine
are as follows:

Adrenaline Hydrocortisone Chlorphenamine


< 6 months 150 mcg (0.15ml 1 in 1,000) 25 mg 250 mcg/kg
6 months - 6 years 150 mcg (0.15ml 1 in 1,000) 50 mg 2.5 mg
6-12 years 300 mcg (0.3ml 1 in 1,000) 100 mg 5 mg
Adult and child 12 years 500 g (0.5ml 1 in 1,000) 200 10 g

Adrenaline can be repeated every 5 minutes if necessary. The best site for IM injection is the anterolateral aspect of the middle
third of the thigh.Common identified causes of anaphylaxis:food (e.g. Nuts) - the most common cause in children, drugs, venom
(e.g. Wasp sting)

19. A 25 year old man is undergoing respiratory spirometry. He takes a maximal inspiration and maximally exhales.
Which of the following measurements will best illustrate this process?
A. Functional residual capacity
B. Vital capacity
C. Inspiratory capacity
D. Maximum voluntary ventilation
E. Tidal volume
Answer: B
The maximum voluntary ventilation is the maximal ventilation over the course of 1 minute.

Tidal volume (TV) Is the volume of air inspired and expired during each ventilatory cycle at rest. It is normally 500mls in males
and 340mls in females. Inspiratory reserve volume (IRV) Is the maximum volume of air that can be forcibly inhaled following a
normal inspiration. 3000mls. Expiratory reserve volume (ERV): Is the maximum volume of air that can be forcibly exhaled
following a normal expiration. 1000mls.Residual volume (RV): Is that volume of air remaining in the lungs after a maximal
expiration: RV = FRC - ERV. 1500mls. Functional residual capacity (FRC) Is the volume of air remaining in the lungs at the
end of a normal expiration. FRC = RV + ERV. 2500mls. Vital capacity (VC) Is the maximal volume of air that can be forcibly
exhaled after a maximal inspiration. VC = TV + IRV + ERV. 4500mls in males, 3500mls in females. Total lung capacity (TLC)
Is the volume of air in the lungs at the end of a maximal inspiration. TLC = FRC + TV + IRV = VC + RV. 5500-
6000mls. Forced vital capacity (FVC) The volume of air that can be maximally forcefully exhaled.

20. Which of the following does not decrease the functional residual capacity?
A. Obesity
B. Pulmonary fibrosis
C. Muscle relaxants
D. Laparoscopic surgery
E. Upright position
Answer: E

137
Increased FRC: Erect position, Emphysema, Asthma. Decreased FRC: Pulmonary fibrosis, Laparoscopic surgery, Obesity,
Abdominal swelling, Muscle relaxants

When the patient is upright the diaphragm and abdominal organs put less pressure on the lung bases, allowing for an increase in
the functional residual capacity (FRC). Other causes of increased FRC include: Emphysema, Asthma.
In addition to those listed above, causes of reduced FRC include: Abdominal swelling, Pulmonary oedema, Reduced muscle tone
of the diaphragm, Age

21. Which of the following is the main site of dehydroepiandrosterone release?


A. Posterior pituitary
B. Zona reticularis of the adrenal gland
C. Zona glomerulosa of the adrenal gland
D. Juxtaglomerular apparatus of the kidney
E. Zona fasciculata of the adrenal gland
Answer: B
Adrenal cortex mnemonic: GFR - ACD
DHEA possesses some androgenic activity and is almost exclusively released from the adrenal gland.

Renin-angiotensin-aldosterone system

Adrenal cortex (mnemonic GFR - ACD): Zona glomerulosa (on outside): mineralocorticoids, mainly aldosterone; Zona
fasciculata (middle): glucocorticoids, mainly cortisol; Zona reticularis (on inside): androgens, mainly dehydroepiandrosterone
(DHEA)

Renin: Released by JGA cells in kidney in response to reduced renal perfusion, low sodium; Hydrolyses angiotensinogen to form
angiotensin I

Factors stimulating renin secretion: Low BP; Hyponatraemia; Sympathetic nerve stimulation; Catecholamines; Erect posture

Angiotensin: ACE in lung converts angiotensin I --> angiotensin II; Vasoconstriction leads to raised BP; Stimulates thirst;
Stimulates aldosterone and ADH release

Aldosterone: Released by the zona glomerulosa in response to raised angiotensin II, potassium, and ACTH levels; Causes
retention of Na+ in exchange for K+/H+ in distal tubule

22. Secretions from which of the following will contain the highest levels of potassium?
A. Rectum
B. Small bowel
C. Gallbladder
D. Pancreas
E. Stomach
Answer: A

The rectum has the potential to generate secretions rich in potassium. This is the rationale behind administration of resins for
hyperkalaemia and the development of hypokalaemia in patients with villous adenoma of the rectum.

Potassium secretions: Salivary glands: Variable may be up to 60mmol/L; Stomach: 10 mmol/L; Bile:5 mmol/L; Pancreas: 4-5
mmol/L; Small bowel: 10 mmol/L; Rectum:30 mmol/L. These values provide average figures only and the exact composition
varies depending upon the existence of disease, serum aldosterone levels and serum pH.
A key point to remember for the exam is that gastric potassium secretions are low. Hypokalaemia may occur in vomiting, usually
as a result of renal wasting of potassium, not because of potassium loss in vomit.

23. What is the typical stroke volume in a resting 70 Kg man?


A. 10ml
B. 150ml
C. 125ml
D. 45ml
E. 70ml
Answer: E
138
Stroke volumes range from 55-100ml.
The stroke volume equates to the volume of blood ejected from the ventricle during each cycle of cardiac contraction. The
volumes for both ventricles are typically equal and equate roughly to 70ml for a 70Kg man. It is calculated by subtracting the end
systolic volume from the end diastolic volume. Factors affecting stroke volume: Cardiac size, Contractility, Preload, Afterload

24. A patient loses 1.6L fresh blood from their abdominal drain. Which of the following will not decrease?
A. Cardiac output
B. Renin secretion
C. Firing of carotid baroreceptors
D. Firing of aortic baroreceptors
E. Blood pressure
Answer: B
Renin secretion will increase as systemic hypotension will cause impairment of renal blood flow. Although the kidney can
autoregulate its own blood flow over a range of systemic blood pressures a loss of 1.6 L will usually produce an increase in renin
secretion.

25. Release of vasopressin from the pituitary will result in which of the following?
A. Vasoconstriction of the afferent glomerular arteriole
B. Increased permeability of the mesangial cells to glucose
C. Reduced permeability of the inner medullary portion of the collecting duct to urea
D. Increased secretion of aldosterone from the macula densa
E. Increased water permeability of the distal tubule cells of the kidney
Answer: E
ADH (vasopressin) results in the insertion of aquaporin channels in apical membrane of the distal tubule and collecting ducts.

26. Which of the following hormones is mainly responsible for sodium - potassium exchange in the salivary ducts?
A. Vasopressin
B. Angiotensin I
C. Aldosterone
D. Somatostatin
E. Cholecystokinin
Answer: C
Aldosterone is responsible for regulating ion exchange in salivary glands. It acts on a sodium / potassium ion exchange pump.It is
a mineralocorticoid hormone derived from the zona glomerulosa of the adrenal gland.

Anatomy of the parotid gland


Location: Overlying the mandibular ramus; anterior and inferior to the ear. Salivary duct: Crosses the masseter, pierces the
buccinator and drains adjacent to the 2nd upper molar tooth (Stensen's duct). Structures passing through the gland: Facial nerve
(Mnemonic: The Zebra Buggered My Cat; Temporal Zygomatic, Buccal, Mandibular, Cervical). External carotid artery.
Retromandibular vein. Auriculotemporal nerve .
Relations:Anterior: masseter, medial pterygoid, superficial temporal and maxillary artery, facial nerve, stylomandibular
ligament. Posterior: posterior belly digastric muscle, sternocleidomastoid, stylohyoid, internal carotid artery, mastoid process,
styloid process
Arterial supply: Branches of external carotid artery. Venous drainage: Retromandibular vein. Lymphatic drainage: Deep
cervical nodes. Nerve innervation:Parasympathetic-Secretomotor. Sympathetic-Superior cervical ganglion. Sensory- Greater
auricular nerve. Parasympathetic stimulation produces a water rich, serous saliva. Sympathetic stimulation leads to the production
of a low volume, enzyme-rich saliva.

27. In a 70 Kg male, what proportion of total body fluid will be contributed by plasma?
A. 50%
B. 5%
C. 35%
D. 65%
E. 25%
Answer: B
70 Kg male = 42 L water (60% of total body weight)
Body fluid compartments comprise intracellular and extracellular compartments. The latter includes interstitial fluid, plasma and
transcellular fluid. Typical figures are based on the 70 Kg male.

Body fluid volumes


Compartment Volume in litres Percentage of total volume
Intracellular 28 L 60-65%
Extracellular 14 L 35-40%
Plasma 3L 8%
139
Interstitial 10 L 24%
Transcellular 1L 3%

28. A 23 year old man is undergoing an inguinal hernia repair under local anaesthesia. The surgeon encounters a bleeding
site which he manages with diathermy. About a minute or so later the patient complains that he is able to feel the
burning pain of the heat at the operative site. Which of the following nerve fibres is responsible for the transmission of
this signal?
A. A α fibres
B. A β fibres
C. B fibres
D. C fibres
E. None of the above
Answer: D
Slow transmission of mechanothermal stimuli is transmitted via C fibres.
A α fibres transmit information relating to motor proprioception, A β fibres transmit touch and pressure and B fibres are
autonomic fibres.
Somatic pain: Peripheral nociceptors are innervated by either small myelinated fibres (A-gamma) fibres or by unmyelinated C
fibres. The A gamma fibres register high intensity mechanical stimuli. The C fibres usually register high intensity
mechanothermal stimuli.

29. What is the approximate volume of pancreatic secretions in a 24 hour period?


A. 100ml
B. 200ml
C. 500ml
D. 1500ml
E. 3000ml
Answer: D
Typically the pancreas secretes up to 1.5L per day.

30. A 34 year old lady has just undergone a parathyroidectomy for primary hyperparathyroidism. The operation is
difficult and all 4 glands were explored. The wound was clean and dry at the conclusion of the procedure and a suction
drain inserted. On the ward she becomes irritable and develops respiratory stridor. On examination her neck is soft
and the drain empty. Which of the following treatments should be tried initially?
A. Administration of intravenous calcium gluconate
B. Administration of intravenous lorazepam
C. Removal of the skin closure on the ward
D. Direct laryngoscopy
E. Administration of calcichew D3 orally
Answer: A
Exploration of the parathyroid glands may result in impairment of the blood supply. Serum PTH levels can fall quickly and
features of hypocalcaemia may ensue, these include neuromuscular irritability and laryngospasm. Prompt administration of
intravenous calcium gluconate can be lifesaving. The absence of any neck swelling and no blood in the drain would go against a
contained haematoma in the neck (which should be managed by removal of skin closure).

Calcium homeostasis: Calcium ions are linked to a wide range of physiological processes. The largest store of bodily calcium is
contained within the skeleton. Calcium levels are primarily controlled by parathyroid hormone, vitamin D and calcitonin.
Parathyroid hormone (PTH): Increase calcium levels and decrease phosphate levels. Increases bone resorption. Immediate
action on osteoblasts to increase ca2+ in extracellular fluid. Osteoblasts produce a protein signaling molecule that activate
osteoclasts which cause bone resorption. Increases renal tubular reabsorption of calcium. Increases synthesis of 1,25(OH)2D
(active form of vitamin D) in the kidney which increases bowel absorption of Ca2+. Decreases renal phosphate reabsorption
1,25-dihydroxycholecalciferol (the active form of vitamin D):Increases plasma calcium and plasma phosphate. Increases renal
tubular reabsorption and gut absorption of calcium. Increases osteoclastic activity. Increases renal phosphate reabsorption
Calcitonin:Secreted by C cells of thyroid. Inhibits intestinal calcium absorption. Inhibits osteoclast activity Inhibits renal tubular
absorption of calcium
Both growth hormone and thyroxine also play a small role in calcium metabolism.

Theme: Interpretation of aterial blood gas results

A. pH 7.19, pCO2 10.2, pO2 16 (FiO2 85%), Bicarbonate 23.8, Base excess -2.2 mmol
B. pH 7.57, PaCO2 3.5, Pa O2 24.5 (FiO2 85%), Bicarbonate 23.5, Base excess +1.8 mmol
C. pH 7.14, PaCO2 7.4, PaO2 8.9 (FiO2 40%), Bicarbonate 14 mmol, Base excess -10.6
D. pH 7.36, PaCO2 7.3, PO2 8.9 (FiO2 40%), Bicarbonate 30.2, Base excess 5.3
140
E. pH 7.32, PCO2 3.8, PaO2 22.2 (FiO2 40%), Bicarbonate 19.1, Base excess -7.9

Which of the following arterial blood gases fit with the description below?

31. Acute respiratory acidosis


pH 7.19, pCO2 10.2, pO2 16 (FiO2 85%), Bicarbonate 23.8, Base excess -2.2 mmol
32. Metabolic acidosis with a compensatory respiratory alkalosis
pH 7.32, PCO2 3.8, PaO2 22.2 (FiO2 40%), Bicarbonate 19.1, Base excess -7.9
33. Chronic respiratory acidosis with a compensatory metabolic alkalosis
pH 7.36, PaCO2 7.3, PO2 8.9 (FiO2 40%), Bicarbonate 30.2, Base excess 5.3

In advanced life support training, a 5 step approach to arterial blood gas interpretation is advocated.
1. How is the patient?
2. Is the patient hypoxaemic? The Pa02 on air should be 10.0-13.0 kPa
3. Is the patient acidaemic (pH <7.35) or alkalaemic (pH >7.45)?
4. What has happened to the PaCO2? If there is acidaemia, an elevated PaCO2 will account for this
5. What is the bicarbonate level or base excess? A metabolic acidosis will have a low bicarbonate level and a low base excess
(< -2 mmol). A metabolic alkalosis will have a high bicarbonate and a high base excess (> +2 mmol)

34. Cortisol is predominantly produced by which of the following?


A. Zona fasciculata of the adrenal
B. Zona glomerulosa of the adrenal
C. Zona reticularis of the adrenal
D. Adrenal medulla
E. Posterior lobe of the pituitary
Answer: A
Relative Glucocorticoid activity: Hydrocortisone = 1; Prednisolone = 4; Dexamethasone = 25
Cortisol is produced by the zona fasciculata of the adrenal gland.

Cortisol: Glucocorticoid; Released by zona fasiculata of the adrenal gland; 90% protein bound; 10% active; Circadian rhythm:
High in the mornings; Negative feedback via ACTH. Actions: Glycogenolysis; Glucaneogenesis; Protein catabolism; Lipolysis;
Stress response; Anti-inflammatory; Decrease protein in bones; Increase gastric acid; Increases neutrophils/platelets/red blood
cells; Inhibits fibroblastic activity

35. Which of the following is not an intravenous colloid?


A. Gelofusine
B. Dextran 40
C. Human albumin solution
D. Hydroxyethyl starch
E. Bicarbonate 8.4%
Answer: E
Bicarbonate is a crystalloid

Pre operative fluid management: fluid management has been described in the British Consensus guidelines on IV fluid
therapy for Adult Surgical patients (GIFTASUP)

The Recommendations include: Use Ringer's lactate or Hartmann's when a crystalloid is needed for resuscitation or replacement
of fluids. Avoid 0.9% N. Saline (due to risk of hyperchloraemic acidosis) unless patient vomiting or has gastric drainage. Use
0.4%/0.18% dextrose saline or 5% dextrose in maintenance fluids. It should not be used in resuscitation or as replacement fluids.
Adult maintenance fluid requirements are: Na 50-100 mmol/day and K 40-80 mmol/day in 1.5-2.5L fluid per day. Patients for
elective surgery should NOT be nil by mouth for >2 hours (unless has disorder of gastric emptying). Patients for elective surgery
should be given carbohydrate rich drinks 2-3h before. Ideally this should form part of a normal pre op plan to facilitate recovery.
Avoid mechanical bowel preparation. If bowel prep is used, simultaneous administration of Hartmann's or Ringer's lactate should
be considered. Excessive fluid losses from vomiting should be treated with a crystalloid with potassium replacement. 0.9% N.
Saline should be given if there is hypochloraemia. Otherwise Hartmann's or Ringer lactate should be given for
diarrhoea/ileostomy/ileus/obstruction. Hartmann's should also be given in sodium losses secondary to diuretics. High risk patients
should receive fluids and inotropes. An attempt should be made to detect pre or operative hypovolaemia using flow based
measurements. If this is not available, then clinical evaluation is needed i.e. JVP, pulse volume etc. In Blood loss or infection
causing hypovolaemia should be treated with a balanced crystalloid or colloid (or until blood available in blood loss). A critically
ill patient is unable to excrete Na or H20 leading to a 5% risk of interstitial oedema. Therefore 5% dextrose as well as colloid
should be given. Give 200mls of colloid in hypovolaemia, repeat until clinical parameters improve.

141
Theme: Electrolyte disorders

A. Hypotonic hypovolaemic hyponatraemia


B. Hypotonic hypervolaemic hyponatraemia
C. Pseudohyponatraemia
D. Syndrome of inappropriate ADH secretion (SIADH)
E. Hypertonic hyponatraemia
F. Over administration 5% dextrose

Please select the most likely reason for hyponatraemia for each scenario given. Each option may be used once, more than once or
not at all.

36. A 73 year old man presents to pre operative clinic for an elective total hip replacement. He is on frusemide for
hypertension. He is found to have the following blood results: Na 120; Urine Na 10 (low); Serum osmolality 280
(normal)
Answer: Hypotonic hypovolaemic hyponatraemia

The blood results reflect extra-renal sodium loss. The body is trying to preserve the sodium by not allowing any sodium
into the urine (hence the low Na in the urine). Note with renal sodium loss the Urinary sodium is high.

37. A 67 year old man presents to pre operative clinic for an elective hernia repair. He is on frusemide for heart
failure. He is found to have the following blood results: Na 120; Urine Na 35 (high); Urine osmolality 520 (high);
Serum osmolality 265 (low)
Answer: Syndrome of inappropriate ADH secretion (SIADH)

This blood picture fits with SIADH. SIADH causes retention of fluid from the urine (concentrated urine) into the blood
vessels, therefore diluting the fluid in the blood vessels (low osmolality). Management involves removing the cause and
fluid restriction.

38. A 77 year old man presents to pre operative clinic for a total knee replacement. He is on frusemide for
hypertension. He is known to have multiple myeloma. He is found to have the following blood results:
Na 120; Serum osmolality 280 (normal); Urine osmolallity normal; Urine Na normal

Answer: Pseudohyponatraemia
Hyperlipidaemia and multiple myeloma are known to cause a pseudohyponatraemia.
SIADH: Low serum osmolalityHigh/Normal urine osmolality

Hyponatraemia
This is commonly tested in the MRCS (despite most surgeons automatically seeking medical advice if this occurs!). The most
common cause in surgery is the over administration of 5% dextrose.
Hyponatraemia may be caused by water excess or sodium depletion. Causes of pseudohyponatraemia include hyperlipidaemia
(increase in serum volume) or a taking blood from a drip arm. Urinary sodium and osmolarity levels aid making a diagnosis.

Classification
 Urinary sodium > 20 mmol/l: Sodium depletion, renal loss
Patient often hypovolaemic; Diuretics (thiazides); Addison's; Diuretic stage of renal failure; SIADH (serum osmolality
low, urine osmolality high, urine Na high); Patient often euvolaemic. Mnemonic: Syndrome of INAPPropriate Anti-
Diuretic Hormone: In creased; Na (sodium); PP (urine)

 Urinary sodium < 20 mmol/l Sodium depletion, extra-renal loss


Diarrhoea, vomiting, sweating. Burns, adenoma of rectum (if villous lesion and large

 Water excess (patient often hypervolaemic and oedematous)Secondary hyperaldosteronism: CCF, cirrhosis; Reduced
GFR: renal failure; IV dextrose, psychogenic polydipsia

Management
Symptomatic Hyponatremia: Acute hyponatraemia with Na <120: immediate therapy. Central Pontine Myelinolisis, may occur
from overly rapid correction of serum sodium. Aim to correct until the Na is > 125 at a rate of 1 mEq/h. Normal saline with
frusemide is an alternative method. The sodium requirement can be calculated as follows:
(125 - serum sodium) x 0.6 x body weight = required mEq of sodium

142
39. A 53 year old man is on the intensive care unit following an emergency abdominal aortic aneurysm repair. He develops
abdominal pain and diarrhoea and is profoundly unwell. His abdomen has no features of peritonism. Which of the
following arterial blood gas pictures is most likely to be present?
A. pH 7.45, pO2 10.1, pCO2 3.2, Base excess 0, Lactate 0
B. pH 7.35, pO2 8.0, pCO2 5.2, Base excess 2, Lactate 1
C. pH 7.20, pO2 9.0, pCO2 3.5, Base excess -10, Lactate 8
D. pH 7.29, pO2 8.9, pCO2 5.9, Base excess -4, Lactate 3
E. pH 7.30, pO2 9.2 pCO2 4.8, Base excess -2, lactate 1
Answer: C
This man is likely to have a metabolic acidosis secondary to a mesenteric infarct.
Disorders of acid- base balance are often covered in the MRCS part A, both in the SBA and EMQ sections.

Metabolic acidosis: This is the most common surgical acid - base disorder. Reduction in plasma bicarbonate levels. Two
mechanisms:1. Gain of strong acid (e.g. diabetic ketoacidosis); 2. Loss of base (e.g. from bowel in diarrhoea)
Classified according to the anion gap, this can be calculated by: (Na+ + K+) - (Cl- + HCO3-).
If a question supplies the chloride level then this is often a clue that the anion gap should be calculated. The normal range = 10-18
mmol/L

Normal anion gap ( = hyperchloraemic metabolic acidosis): Gastrointestinal bicarbonate loss: diarrhoea,
ureterosigmoidostomy, fistula; Renal tubular acidosis; Drugs: e.g. acetazolamide; Ammonium chloride injection; Addison's
disease

Raised anion gap: Lactate: shock, hypoxia; Ketones: diabetic ketoacidosis, alcohol; Urate: renal failure; Acid poisoning:
salicylates, methanol

Metabolic acidosis secondary to high lactate levels may be subdivided into two types: Lactic acidosis type A: (Perfusion disorders
e.g.shock, hypoxia, burns) or Lactic acidosis type B: (Metabolic e.g. metformin toxicity)

Metabolic alkalosis: Usually caused by a rise in plasma bicarbonate levels. Rise of bicarbonate above 24 mmol/L will typically
result in renal excretion of excess bicarbonate. Caused by a loss of hydrogen ions or a gain of bicarbonate. It is due mainly to
problems of the kidney or gastrointestinal tract

Causes: Vomiting / aspiration (e.g. Peptic ulcer leading to pyloric stenosis, nasogastric suction); Diuretics; Liquorice,
carbenoxolone; Hypokalaemia; Primary hyperaldosteronism; Cushing's syndrome; Bartter's syndrome; Congenital adrenal
hyperplasia

Mechanism of metabolic alkalosis: Activation of renin-angiotensin II-aldosterone (RAA) system is a key factor; Aldosterone
causes reabsorption of Na+ in exchange for H+ in the distal convoluted tubule; ECF depletion (vomiting, diuretics) --> Na+ and Cl-
loss --> activation of RAA system --> raised aldosterone levels; In hypokalaemia, K+ shift from cells --> ECF, alkalosis is caused
by shift of H+ into cells to maintain neutrality

Respiratory acidosis: Rise in carbon dioxide levels usually as a result of alveolar hypoventilation: Renal compensation may
occur leading to Compensated respiratory acidosis

Causes: COPD; Decompensation in other respiratory conditions e.g. Life-threatening asthma / pulmonary oedema; Sedative
drugs: benzodiazepines, opiate overdose

Respiratory alkalosis: Hyperventilation resulting in excess loss of carbon dioxide; This will result in increasing pH

Causes: Psychogenic: anxiety leading to hyperventilation; Hypoxia causing a subsequent hyperventilation: pulmonary embolism,
high altitude; Early salicylate poisoning*; CNS stimulation: stroke, subarachnoid haemorrhage, encephalitis; Pregnancy
*Salicylate overdose leads to a mixed respiratory alkalosis and metabolic acidosis. Early stimulation of the respiratory centre leads
to a respiratory alkalosis whilst later the direct acid effects of salicylates (combined with acute renal failure) may lead to an
acidosis

40. A 48 year old women suffers blunt trauma to the head and develops respiratory compromise. As a result she develops
hypercapnia. Which of the following effects is most likely to ensue?
A. Cerebral vasoconstriction
B. Cerebral vasodilation
C. Cerebral blood flow will remain unchanged
143
D. Shunting of blood to peripheral tissues will occur in preference to CNS perfusion
E. None of the above
Answer: B
Hypercapnia will tend to produce cerebral vasodilation. This is of considerable importance in patients with cranial trauma as it
may increase intracranial pressure.

Pressure within the cranium is governed by the Monroe-Kelly doctrine. This considers the skull as a closed box. Increases in mass
can be accommodated by loss of CSF. Once a critical point is reached (usually 100- 120ml of CSF lost) there can be no further
compensation and ICP rises sharply. The next step is that pressure will begin to equate with MAP and neuronal death will occur.
Herniation will also accompany this process. The CNS can autoregulate its own blood supply. Vaso constriction and dilatation of
the cerebral blood vessels is the primary method by which this occurs. Extremes of blood pressure can exceed this capacity
resulting in risk of stroke. Other metabolic factors such as hypercapnia will also cause vasodilation, which is of importance in
ventilating head injured patients. The brain can only metabolise glucose, when glucose levels fall, consciousness will be impaired.

41. A patient is seen in clinic complaining of abdominal pain. Routine bloods show:Na+ 142 mmol/l; K+ 4.0 mmol/l;
Chloride 104 mmol/l; Bicarbonate 19 mmol/l; Urea 7.0 mmol/l; Creatinine 112 µmol/l. What is the anion gap?
A. 4 mmol/L
B. 14 mmol/L
C. 20 mmol/L
D. 21 mmol/L
E. 23 mmol/L

Answer: E

The anion gap may be calculated by using (sodium + potassium) - (bicarbonate + chloride)
= (142 + 4.0) - (104 + 19) = 23 mmol/L

42. A surgeon is considering using lignocaine as local anasthesia for a minor procedure. Which of the following best
accounts for its actions?
A. Blockade of neuronal acetylcholine receptors
B. Blockade of neuronal nicotinic receptors
C. Blockade of neuronal sodium channels
D. Blockade of neuronal potassium channels
E. Blockade of neuronal calcium channels
Answer: C
Lignocaine blocks sodium channels. They will typically be activated first, hence the pain some patients experience on
administration.

Local anaesthetic agents


Lidocaine: An amide; Local anaesthetic and a less commonly used antiarrhythmic (affects Na channels in the axon); Hepatic
metabolism, protein bound, renally excreted; Toxicity: due to IV or excess administration. Increased risk if liver dysfunction or
low protein states. Note acidosis causes lidocaine to detach from protein binding.; Drug interactions: Beta blockers, ciprofloxacin,
phenytoin; Features of toxicity: Initial CNS over activity then depression as lidocaine initially blocks inhibitory pathways then
blocks both inhibitory and activating pathways. Cardiac arrhythmias. Increased doses may be used when combined with
adrenaline to limit systemic absorption.

Cocaine: Pure cocaine is a salt, usually cocaine hydrochloride. It is supplied for local anaesthetic purposes as a paste. It is
supplied for clinical use in concentrations of 4 and 10%. It may be applied topically to the nasal mucosa. It has a rapid onset of
action and has the additional advantage of causing marked vasoconstriction. It is lipophillic and will readily cross the blood brain
barrier. Its systemic effects also include cardiac arrhythmias and tachcardia. Apart from its limited use in ENT surgery it is
otherwise used rarely in mainstream surgical practice.

Bupivicaine: Bupivacaine binds to the intracellular portion of sodium channels and blocks sodium influx into nerve cells, which
prevents depolarization. It has a much longer duration of action than lignocaine and this is of use in that it may be used for topical
wound infiltration at the conclusion of surgical procedures with long duration analgesic effect. It is cardiotoxic and is therefore
contra indicated in regional blockage in case the tourniquet fails. The co-administration of adrenaline concentrates it at the site of
action and allows the use of higher doses.

Prilocaine: Similar mechanism of action to other local anaesthetic agents. However, it is far less cardiotoxic and is therefore the
agent of choice for intravenous regional anaesthesia e.g. Biers Block.

All local anaesthetic agents dissociate in tissues and this contributes to their therapeutic effect. The dissociation constant shifts in
tissues that are acidic e.g. where an abscess is present and this reduce the efficacy.

144
Doses of local anaesthetics
Agent Dose plain Dose with adrenaline
Lignocaine 3mg/Kg 7mg/Kg
Bupivicane 2mg/Kg 2mg/Kg
Prilocaine 6mg/Kg 9mg/Kg
These are a guide only as actual doses depend on site of administration, tissue vascularity and co-morbidities.

43. A 22 year old man suffers a blunt head injury. He is drowsy and has a GCS of 7 on admission. Which of the following
is the major determinant of cerebral blood flow in this situation?
A. Systemic blood pressure
B. Mean arterial pressure
C. Intra cranial pressure
D. Hypoxaemia
E. Acidosis
Answer: C

Hypoxaemia and acidosis may both affect cerebral blood flow. However, in the traumatic situation increases in intracranial
pressure are far more likely to occur especially when GCS is low. This will adversely affect cerebral blood flow.
Cerebral blood flow: CNS autoregulates its own blood supply
Factors affecting the cerebral pressure include; systemic carbon dioxide levels, CNS metabolism, CNS trauma, CNS pressure. The
PaCO2 is the most potent mediator
Acidosis and hypoxaemia will increase cerebral blood flow but to a lesser degree
Cerebral perfusion pressure may increase in patients with head injuries and this can result in impaired blood flow
Intra cerebral pressure governed by Monroe-Kelly Doctrine which considers brain as closed box, changes in pressure are offset by
loss of CSF. When this is no longer possible ICP rises

44. Which main group of receptors does dobutamine bind to?


A. α-1
B. α-2
C. ß-1
D. ß-2
E. D-1
Answer: C
Dobutamine is a sympathomimetic with both alpha- and beta-agonist properties; it displays a considerable selectivity for beta1-
cardiac receptors.

Inotrope and its receptor:Adrenaline: α-1, α-2, β-1, β-2; Noradrenaline : α-1,( α-2), (β-1), (β-2); Dobutamine: β-1, (β 2);
Dopamine : (α-1), (α-2), (β-1), D-1,D-2. Minor receptor effects in brackets

Effects of receptor binding: α-1, α-2:vasoconstriction; β-1:increased cardiac contractility and HR; β-2:vasodilatation; D-1:renal
and spleen vasodilatation; D-2:inhibits release of noradrenaline

45. Which of the following is responsible for the release and synthesis of calcitonin?
A. Parathyroid glands
B. Anterior pituitary
C. Thyroid gland
D. Posterior pituitary
E. Adrenal glands
Answer: C
Calcitonin has the opposite effect of PTH and is release from the thyroid gland.

46. What is the half life of insulin in the circulation of a normal healthy adult?
A. Less than 30 minutes
B. Between 1 and 2 hours
C. Between 2 and 3 hours
D. Between 4 and 5 hours
E. Over 6 hours
Answer: A

Insulin is degraded by enzymes in the circulation. It typically has a half life of less than 30 minutes. Abnormalities of the
clearance of insulin may occur in type 2 diabetes.

145
47. Which of the following statements about blood clotting is untrue?
A. Platelet adhesion to disrupted endothelium is dependent upon von Willebrand factor
B. Protein C is a vitamin K dependent substance
C. The bleeding time provides an assessment of platelet function
D. The prothrombin time tests the extrinsic system
E. Administration of aprotinin during liver transplantation surgery prolongs survival
Answer: E

Although aprotinin reduces fibrinolysis and thus bleeding, it is associated with increased risk of death and was withdrawn in 2007.
Protein C is dependent upon vitamin K and this may paradoxically increase the risk of thrombosis during the early phases of
warfarin treatment.

Theme: Management of vomiting

A. Ondansetron
B. Metoclopramide
C. Cyclizine
D. Erythromycin
E. Cisapride
F. Haloperidol

Please select the most appropriate drug for the given scenario. Each option may be used once, more than once or not at all.

48. A 78 year old manwith diabetes develops autonomic gastropathy and persistant a troublesome vomiting.
Answer: Erythromycin
Unlike metoclopramide the effects of erythromycin on gastric empyting are not mediated via the vagus nerve.
49. A drug which blocks the chemoreceptor trigger zone in the area postrema.
Answer: Ondansetron
5 HT3 blockers are most effective for many types of nausea for this reason.
50. A 48 year old man with oesphageal varices has a profuse haemorrhage on the ward.
Answer: Metoclopramide
Intravenous metoclopramide causes increased oesophageal pressure and this may temporarily slow the rate of
haemorrhage whilst more definitive measures are instigated.

Vomiting
Reflex oral expulsion of gastric (and sometimes intestinal) contents - reverse peristalsis and abdominal contraction The vomiting
centre is in part of the medulla oblongata and is triggered by receptors in several locations: Labyrinthine receptors of ear (motion
sickness); Overdistention receptors of duodenum and stomach; Trigger zone of CNS - many drugs (e.g., opiates) act here ; Touch
receptors in throat; Sensory innervation rich, both extrinsic and intrinsic

51. Which of the following cell types is least likely to be found in a wound 1 week following injury?
A. Macrophages
B. Fibroblasts
C. Myofibroblasts
D. Endothelial cells
E. Neutrophils
Answer: C
Myofibroblasts are differentiated fibroblasts composed, in which the cytoskeleton contains actin filaments. These cell types
facilitate wound contracture and are the hallmark of a mature wound.

Phases of wound healing

Phase Key features Cells Timeframe


 Vasospasm in adjacent vessels
Seconds/
Haemostasis  Platelet plug formation and generation of fibrin rich clot Erythrocytes
Minutes

 Neutrophils migrate into wound (function impaired in


diabetes).
Neutrophils, fibroblasts
Inflammation  Growth factors released, including basic fibroblast Days
and macrophages
growth factor and vascular endothelial growth factor.
 Fibroblasts replicate within the adjacent matrix and

146
migrate into wound.
 Macrophages and fibroblasts couple matrix regeneration
and clot substitution.

 Platelet derived growth factor and transformation growth


factors stimulate fibroblasts and epithelial cells.
 Fibroblasts produce a collagen network. Fibroblasts, endothelial
Regeneration Weeks
 Angiogenesis occurs and wound resembles granulation cells, macrophages
tissue.

 Longest phase of the healing process and may last up to


one year (or longer).
 During this phase fibroblasts become differentiated
(myofibroblasts) and these facilitate wound contraction. 6 weeks to 1
Remodelling Myofibroblasts
year
 Collagen fibres are remodelled.
 Microvessels regress leaving a pale scar.

52. The blood - brain barrier is not highly permeable to which of the following?
A. Carbon dioxide
B. Barbituates
C. Glucose
D. Oxygen
E. Hydrogen ions
Answer: E
The blood brain barrier is relatively impermeable to highly dissociated compounds.

Cerebrospinal fluid: The CSF fills the space between the arachnoid mater and pia mater (covering surface of the brain). The total
volume of CSF in the brain is approximately 150ml. Approximately 500 ml is produced by the ependymal cells in the choroid
plexus (70%), or blood vessels (30%). It is reabsorbed via the arachnoid granulations which project into the venous sinuses.

Circulation: 1. Lateral ventricles (via foramen Munro); 2. 3rd ventricle; 3. Cerebral aqueduct (aqueduct Sylvius); 4. 4th ventricle
(via foramina of Magendie and Luschka); 5. Subarachnoid space; 6. Reabsorbed into venous system via arachnoid granulations in
superior sagittal sinus

Composition: Glucose: 50-80mg/dl; Protein: 15-40 mg/dl; Red blood cells: Nil; White blood cells: 0-3 cells/ mm3

53. A 43 year old presents to the urology clinic complaining of impotence.Which of the following will occur in response to
increased penile parasympathetic stimulation?
45. Detumescence
46. Ejaculation
47. Erection
48. Vasospasm of the penile branches of the pudendal artery
49. Contraction of the smooth muscle in the epididymis and vas deferens
Answer: C
Parasympathetic stimulation causes erection. Sympathetic stimulation will produce ejaculation, detumescence and vasospasm of
the pudendal artery. It will also cause contraction of the smooth muscle in the epididymis and vas to convey the ejaculate.

Autonomic: Sympathetic nerves originate from T11-L2 and parasympathetic nerves from S2-4 join to form pelvic plexus.
Parasympathetic discharge causes erection, sympathetic discharge causes ejaculation and detumescence. Somatic nerves
Supplied by dorsal penile and pudendal nerves. Efferent signals are relayed from Onufs nucleus (S2-4) to innervate
ischiocavernosus and bulbocavernosus muscles.

Autonomic discharge to the penis will trigger the veno-occlusive mechanism which triggers the flow of arterial blood into the
penile sinusoidal spaces. As the inflow increases the increased volume in this space will secondarily lead to compression of the
subtunical venous plexus with reduced venous return. During the detumesence phase the arteriolar constriction will reduce arterial
inflow and thereby allow venous return to normalise.

Priapism: Prolonged unwanted erection, in the absence of sexual desire, lasting more than 4 hours.

Classification of priaprism
147
Low flow priaprism Due to veno-occlusion (high intracavernosal pressures): Most common type; Often painful; Often low
cavernosal flow; If present for >4 hours requires emergency treatment
High flow priaprism:Due to unregulated arterial blood flow. Usually presents as semi rigid painless erection
Recurrent priaprism Typically seen in sickle cell disease, most commonly of high flow type.

Causes: Intracavernosal drug therapies (e.g. for erectile dysfunction>; Blood disorders such as leukaemia and sickle cell disease;
Neurogenic disorders such as spinal cord transection; Trauma to penis resulting in arterio-venous malformations

Tests: Exclude sickle cell/ leukaemia; Consider blood sampling from cavernosa to determine whether high or low flow (low flow
is often hypoxic)

Management: Ice packs/ cold showers; If due to low flow then blood may be aspirated from copora or try intracavernosal alpha
adrenergic agonists.Delayed therapy of low flow priaprism may result in erectile dysfunction.

54. In class II haemorrhagic shock in a 70Kg male, one would not expect to find?
A. Blood loss greater than 750ml
B. Tachycardia
C. Decreased blood pressure
D. Urine output less than 20ml
E. Anxiety
Answer: D
Urine output in class II shock (assuming 70Kg adult) is typically between 20 and 30ml.

Theme: Acid - base disorders

A. pH 7.64 pO2 10.0 kPa pCO2 2.8 kPa HCO3 20


B. pH 7.25 pO2 8.9 pCO2 3.2 HCO3 10
C. pH 7.20 pO2 6.2 pCO2 8.2 HCO3 27
D. pH 7.60 pO2 8.2 pCO2 5.8 HCO3 40
E. pH7.45 pO2 7.2 pCO2 2.5 HCO3 24

Please match the diagnosis with the arterial blood gas result. Each option may be used once, more than once or not at all.

55. Pulmonary embolus

The correct answer is pH7.45 pO2 7.2 pCO2 2.5 HCO3 24


A combination of hypoxia and respiratory alkalosis should suggest a pulmonary embolus. The respiratory alkalosis is due
to hyperventilation associated with the pulmonary embolism.
56. Ureterosigmoidostomy

Answer: pH 7.25 pO2 8.9 pCO2 3.2 HCO3 10


There is acidosis. To compensate the patient will attempt to reduce the pH level in the blood by hyperventilating, hence
the low CO2 level .

57. Peptic ulcer causing pyloric stenosis


Answer: pH 7.60 pO2 8.2 pCO2 5.8 HCO3 40

58.Which of the following best accounts for the action of PTH in increasing serum calcium levels?
A.Activation of vitamin D to increase absorption of calcium from the small intestine.
B.Direct stimulation of oestoclasts to absorb bone with release of calcium.
C.Stimulation of phosphate absorption at the distal convoluted tubule of the kidney.
D.Decreased porosity of the vessels at Bowmans capsule to calcium.
E.Vasospasm of the afferent renal arteriole thereby reducing GFR and calcium urinary loss.
Answer: A
PTH increases the activity of 1-α-hydroxylase enzyme, which converts 25-hydroxycholecalciferol to 1,25-
dihydroxycholecalciferol, the active form of vitamin D.
Osteoclasts do not have a PTH receptor and effects are mediated via osteoblasts.

Parathyroid hormone is secreted by the chief cells of the parathyroid glands. It acts to increase serum calcium concentration by
stimulation of the PTH receptors in the kidney and bone. PTH has a plasma half life of 4 minutes.
148
Effects of PTH
Bone:Binds to osteoblasts which signal to osteoclasts to cause resorption of bone and release calcium. Kidney:Active
reabsorption of calcium and magnesium from the distal convoluted tubule. Decreases reabsorption of phosphate. Intestine via
kidney:Increases intestinal calcium absorption by increasing activated vitamin D. Activated vitamin D increases calcium
absorption.

59. Which of the following drugs does not cause syndrome of inappropriate anti diuretic hormone release?
45. Haloperidol
46. Carbamazepine
47. Amitriptylline
48. Cyclophosphamide
49. Methotrexate
Answer: E
Drugs causing SIADH: ABCD:A nalgesics: opioids, NSAIDs; B arbiturates; C yclophosphamide/ Chlorpromazine/
Carbamazepine; D iuretic (thiazides)

60. Which of the following changes are not typically seen in established dehydration?
A. Rising haematocrit
B. Urinary sodium <20mmol/ litre
C. Metabolic acidosis
D. Decreased serum urea to creatinine ratio
E. Hypernatraemia
Answer: D
Diagnosing dehydration can be complicated, laboratory features include:Hypernatraemia; Rising haematocrit; Metabolic acidosis;
Rising lactate; Increased serum urea to creatinine ratio; Urinary sodium <20 mmol/litre; Urine osmolality approaching
1200mosmol/kg

61. A 67 year old male is admitted to the surgical unit with acute abdominal pain. He is found to have a right sided
pneumonia. The nursing staff put him onto 15L O2 via a non rebreathe mask. After 30 minutes the patient is found
moribund, sweaty and agitated by the nursing staff. An arterial blood gas reveals: pH:7.15; pCO2:10.2; pO2:8;
Bicarbonate:32; Base excess:5.2. What is the most likely cause for this patients deterioration?
A. Acute respiratory alkalosis secondary to hyperventilation
B. Over administration of oxygen in a COPD patient
C. Metabolic acidosis secondary to severe pancreatitis
D. Metabolic alkalosis secondary to hypokalaemia
E. Acute respiratory acidosis secondary to pneumonia
Answer: B
This patient has an acute respiratory acidosis, however this is on a background of chronic respiratory acidosis (due to COPD) with
a compensatory metabolic alkalosis (the elevated bicarbonate is the main clue to the chronic nature of the respiratory acidosis).
This blood gas picture is typical in a COPD patient who has received too much oxygen; these patients lose their hypoxic drive for
respiration, therefore retain CO2 and subsequently hypoventilate leading to respiratory arrest. If the bicarbonate was normal, then
the answer would be acute respiratory acidosis secondary to pneumonia.

62. Which of the following statements relating to the pharmacology of warfarin is untrue?
45. Interferes with clotting factors 2,7,9 and 10
46. It may not be pharmacologically active for up to 72 hours
47. The half life of warfarin is 40 hours
48. Warfarin has a large volume of distribution
49. It is metabolized in the liver
Answer: D
Factors 2,7,9,10 affected
Warfarin interferes with fibrin formation by affecting carboxylation of glutamic acid residues in factors 2,7,9 and 10. Factor 2 has
the longest half life of approximately 60 h, therefore it can take up to 3 days for warfarin to be fully effective. Warfarin has a
small volume of distribution as it is protein bound. Warfarin is an oral anticoagulant which inhibits the reduction of vitamin K to
its active hydroquinone form, which in turn acts as a cofactor in the formation of clotting factor II, VII, IX and X (mnemonic =
1972) and protein C

Factors that may potentiate warfarin: Liver disease; P450 enzyme inhibitors, e.g.: amiodarone, ciprofloxacin; Cranberry juice;
Drugs which displace warfarin from plasma albumin, e.g. NSAIDs; Inhibit platelet function: NSAIDs

149
Side-effects: Haemorrhage; Teratogenic; Skin necrosis: when warfarin is first started biosynthesis of protein C is reduced. This
results in a temporary procoagulant state after initially starting warfarin, normally avoided by concurrent heparin administration.
Thrombosis may occur in venules leading to skin necrosis.

63. Which of the following does not cause an increased anion gap acidosis?
A. Uraemia
B. Paraldehyde
C. Diabetic ketoacidosis
D. Ethylene glycol
E. Acetazolamide
Answer: E

Causes if increased anion acidosis: MUDPILES : M – Methanol; U – Uraemia; D - DKA/AKA; P - Paraldehyde/phenformin; I


- Iron/INH; L - Lactic acidosis; E - Ethylene glycol; S - Salicylates

64. Which one of the following is least associated with thrombocytopenia?


A. Heparin therapy
B. Rheumatoid arthritis
C. Infectious mononucleosis
D. Liver disease
E. Pregnancy
Answer: B
Rheumatoid arthritis, unlike systemic lupus erythematous, is generally associated with a thrombocytosis. In some cases of Felty's
syndrome thrombocytopaenia may be seen secondary to hypersplenism. This however represents a small percentage of patients
with rheumatoid arthritis. Causes of severe thrombocytopenia: ITP; DIC; TTP; haematological malignancy. Causes of
moderate thrombocytopenia: heparin induced thrombocytopenia (HIT); drug-induced (e.g. quinine, diuretics, sulphonamides,
aspirin, thiazides); alcohol; liver disease; hypersplenism; viral infection (EBV, HIV, hepatitis); pregnancy; SLE/antiphospholipid
syndrome; vitamin B12 deficiency

65. Which of the following will increase the volume of pancreatic exocrine secretions?
A. Octreotide
B. Cholecystokinin
C. Aldosterone
D. Adrenaline
E. None of the above
Answer: B
Cholecystokinin will often increase the volume of pancreatic secretions.

66. Where is the majority of iron found in the body?


A. Bone
B. Haemoglobin
C. Ferritin and haemosiderin
D. Myoglobin
E. Plasma iron
Answer: B
Approximately 70% of body iron is found bound to haemoglobin.

67. A 44 year old man recieves a large volume transfusion of whole blood. The whole blood is two weeks old. Which of the
following best describes its handling of oxygen?
A. It will have a low affinity for oxygen
B. Its affinity for oxygen is unchanged
C. It will more readily release oxygen in metabolically active tissues than fresh blood
D. The release of oxygen in metabolically acitve tissues will be the same as fresh blood
E. It will have an increased affinity for oxygen
Answer: E

Stored blood has less 2,3 DPG and therefore has a higher affinity for oxygen, this reduces its ability to release it at metabolising
tissues.

68. Which of the following does not occur during the physiological response to surgery?
A. Glycogenolysis

150
B. Increased acute phase proteins
C. Increased cortisol production
D. Bronchoconstriction
E. Release of nitric oxide by vessels

Answer: D

Response to surgery
Sympathetic nervous system: Noradrenaline from sympathetic nerves and adrenaline from adrenal medulla; Blood diverted from
skin and visceral organs; bronchodilatation, reduced intestinal motility, increased glucagon and glycogenolysis, insulin reduced;
Heart rate and myocardial contractility are increased

Acute phase response: TNF-α, IL-1, IL-2, IL-6, interferon and prostaglandins are released; Excess cytokines may cause SIRS

Cytokines increase the release of acute phase proteins

Endocrine response: Hypothalamus, pituitary, adrenal axis. Increases ACTH and cortisol production:increases protein
breakdown and increases blood glucose levels Aldosterone increases sodium reabsorption. Vasopressin increases water
reabsorption and causes vasoconstriction

Vascular endothelium: Nitric oxide produces vasodilatation. Platelet activating factor enhances the cytokine response.
Prostaglandins produce vasodilatation and induce platelet aggregation

69. A 43 year old lady undergoes a day case laparoscopic cholecystectomy. The operation is more difficult than anticipated
and a drain is placed to the operative site. Whilst in recovery the patient loses 1800ml of frank blood into the drain.
Which of the following will not occur?
A. Release of aldosterone via the Bainbridge reflex
B. Reduced urinary sodium excretion
C. Increase in sympathetic discharge to ventricular muscle
D. Fall in parasympathetic discharge to the sino atrial node
E. Decreased stimulation from atrial pressure receptors
Answer: A

The Bainbridge reflex is the increase in heart rate mediated via atrial stretch receptors that occurs following a rapid infusion of
blood.

70. Which of the following statements are not typically true in hypokalaemia?
A. It may occur as a result of mechanical bowel preparation
B. Chronic vomiting may increase renal potassium losses
C. It may be associated with aciduria
D. It may cause hyponatraemia
E. It often accompanies acidosis
Answer: E
Potassium depletion occurs either through the gastrointestinal tract or the kidney. Chronic vomiting in itself is less prone to induce
potassium loss than diarrhoea as gastric secretions contain less potassium than those in the lower GI tract. If vomiting produces a
metabolic alkalosis then renal potassium wasting may occur as potassium is excreted in preference to hydrogen ions. The converse
may occur in potassium depletion resulting in acid urine.

Hyperkalaemia Plasma potassium levels are regulated by a number of factors including aldosterone, acid-base balance and
insulin levels. Metabolic acidosis is associated with hyperkalaemia as hydrogen and potassium ions compete with each other for
exchange with sodium ions across cell membranes and in the distal tubule. ECG changes seen in hyperkalaemia include tall-tented
T waves, small P waves, widened QRS leading to a sinusoidal pattern and asystole

Causes of hyperkalaemia: Acute renal failure; Drugs*: potassium sparing diuretics, ACE inhibitors, angiotensin 2 receptor
blockers, spironolactone, ciclosporin, heparin** Metabolic acidosis; Addison's; Tissue necrosis/rhabdomylosis: burns, trauma;
Massive blood transfusion. Foods that are high in potassium: Salt substitutes (i.e. Contain potassium rather than sodium);
Bananas, oranges, kiwi fruit, avocado, spinach, tomatoes

*beta-blockers interfere with potassium transport into cells and can potentially cause hyperkalaemia in renal failure patients -
remember beta-agonists, e.g. Salbutamol, are sometimes used as emergency treatment

**both unfractionated and low-molecular weight heparin can cause hyperkalaemia. This is thought to be caused by inhibition of
aldosterone secretion
151
71. Approximately what proportion of salivary secretions is provided by the submandibular glands?
A. 10%
B. 70%
C. 40%
D. 90%
E. 20%
Answer: B
Although they are small, the submandibular glands provide the bulk of salivary secretions and contribute 70%, the sublingual
glands provide 5% and the remainder from the parotid.

Relations of the submandibular gland

Superficial:Platysma, deep fascia and mandible, Submandibular lymph nodes, Facial vein (facial artery near mandible), Marginal
mandibular nerve, Cervical branch of the facial nerve. Deep:Facial artery (inferior to the mandible), Mylohoid muscle, Sub
mandibular duct, Hyoglossus muscle, Lingual nerve, Submandibular ganglion, Hypoglossal nerve. Submandibular duct
(Wharton's duct): Opens lateral to the lingual frenulum on the anterior floor of mouth. 5 cm length. Lingual nerve wraps around
Wharton's duct. As the duct passes forwards it crosses medial to the nerve to lie above it and then crosses back, lateral to it, to
reach a position below the nerve.

Innervation: Sympathetic innervation- Superior Cervical ganglion via the Lingual nerve. Parasympathetic innervation-
Submandibular ganglion Arterial supply: Branch of the Facial artery. The facial artery passes through the gland to groove its
deep surface. It then emerges onto the face by passing between the gland and the mandible. Venous drainage: Anterior Facial
vein (lies deep to the Marginal Mandibular nerve). Lymphatic drainage: Deep cervical and jugular chains of nodes.

72. Which is not a cause of hyperuricaemia?


A. Severe psoriasis
B. Lesch-Nyhan syndrome
C. Hyperthyroidism
D. Diabetic ketoacidosis
E. Alcohol
Answer: C
Mnemonic of the drugs causing hyperuricaemia as a result of reduced excretion of urate: 'Can't leap': C iclosporin; A lcohol; N
icotinic acid; T hiazides; L oop diuretics; E thambutol; A spirin; P yrazinamide

Hyperuricaemia:Increased levels of uric acid may be seen secondary to either increased cell turnover or reduced renal excretion
of uric acid. Hyperuricaemia may be found in asymptomatic patients who have not experienced attacks of gout. Hyperuricaemia
may be associated with hyperlipidaemia and hypertension. It may also be seen in conjunction with the metabolic syndrome
Increased synthesis: Lesch-Nyhan disease; Myeloproliferative disorders; Diet rich in purines; Exercise; Psoriasis; Cytotoxics.
Decreased excretion: Drugs: low-dose aspirin, diuretics, pyrazinamide; Pre-eclampsia; Alcohol; Renal failure; Lead.

73. A 20 year old man is hit in the face and occludes his airway. Which of the following stimuli and receptor groups would
the most potently activated as a result?
A. Receptors in the apneustic centre
B. Hypoxia of centrally located chemoreceptors
C. Hypoxia of peripherally located chemoreceptors
D. Hypercapnia of the carotid bodies
E. Hypoxia in the aortic arch receptors
Answer: D
The carotid bodies are the most vascular site and hypercapnia the most potent stimulus.

74. Which of the following statements relating to low molecular weight heparins is false?
A. They act via inhibition of Factor Xa
B. Large doses may be used prior to commencing cardiopulmonary bypass
C. They have a highly predictable pharmacokinetic profile
D. They are derivatives of unfractionated heparin
E. They have a molecular mass in the range of 3000-10000Da
Answer: B
As they are not easily reversed they are unsuitable for this purpose.

Heparin: Causes the formation of complexes between antithrombin and activated thrombin/factors 7,9,10,11 & 12

152
Advantages of low molecular weight heparin: Better bioavailability; Lower risk of bleeding; Longer half life; Little effect on
APTT at prophylactic dosages; Less risk of HIT. Complications: Bleeding; Osteoporosis; Heparin induced thrombocytopenia
(HIT): occurs 5-14 days after 1st exposure; Anaphylaxis. In surgical patients that may need a rapid return to theatre administration
of unfractionated heparin is preferred as low molecular weight heparins have a longer duration of action and are harder to reverse.

75. A 43 year old lady presents with urinary incontinence. At which of the following locations is Onufs nucleus likely to be
found?
A. Medulla oblongata
B. Anterior horn of L5 nerve roots
C. Micturition centre in the Pons
D. Anterior horn of S2 nerve roots
E. None of the above
Answer: D
Onufs nucleus is located in the anterior horn of S2 and is the origin of neurones to the external urethral sphincter.

Urinary incontinence: Involuntary passage of urine. Most cases are female (80%). It has a prevalence of 11% in those aged
greater than 65 years. The commonest variants include:Stress urinary incontinence (50%); Urge incontinence (15%); Mixed (35%)

Males: Males may also suffer from incontinence although it is a much rarer condition in men. A number of anatomical factors
contribute to this. Males have 2 powerful sphincters; one at the bladder neck and the other in the urethra. Damage to the bladder
neck mechanism is a factor in causing retrograde ejaculation following prostatectomy. The short segment of urethra passing
through the urogenital diaphragm consists of striated muscle fibres (the external urethral sphincter) and smooth muscle capable of
more sustained contraction. It is the latter mechanism that maintains continence following prostatectomy.

Females: The sphincter complex at the level of bladder neck is poorly developed in females. As a result the external sphincter
complex is functionally more important, its composition being similar to that of males. Innervation is via the pudendal nerve and
the neuropathy that may accompany obstetric events may compromise this and lead to stress urinary incontinence.

Innervation: Somatic innervation to the bladder is via the pudendal, hypogastric and pelvic nerves. Autonomic nerves travel in
these nerve fibres too. Bladder filling leads to detrusor relaxation (sympathetic) coupled with sphincter contraction. The
parasympathetic system causes detrusor contraction and sphincter relaxation. Overall control of micturition is centrally mediated
via centres in the Pons.

Stress urinary incontinence: 50% of cases, especially in females. Damage (often obstetric) to the supporting structures
surrounding the bladder may lead to urethral hypermobility. Other cases due to sphincter dysfunction, usually from neurological
disorders (e.g. Pudendal neuropathy, multiple sclerosis).

Urethral mobility:Pressure not transmitted appropriately to the urethra resulting in involuntary passage of urine during episodes of
raised intra-abdominal pressure.

Sphincter dysfunction: Sphincter fails to adapt to compress urethra resulting in involuntary passage of urine. When the sphincter
completely fails there is often to continuous passage of urine.

Urge incontinence: In these patients there is sense of urgency followed by incontinence. The detrusor muscle in these patients is
unstable and urodynamic investigation will demonstrate overactivity of the detrusor muscle at inappropriate times (e.g. Bladder
filling). Urgency may be seen in patients with overt neurological disorders and those without. The pathophysiology is not well
understood but poor central and peripheral co-ordination of the events surrounding bladder filling are the main processes.

Assessment: Careful history and examination including vaginal examination for cystocele.
Bladder diary for at least 3 days
Consider flow cystometry if unclear symptomatology or surgery considered and diagnosis is unclear.
Exclusion of other organic disease (e.g. Stones, UTI, Cancer)

Management: Conservative measures should be tried first; Stress urinary incontinence or mixed symptoms should undergo 3
months of pelvic floor exercise. Over active bladder should have 6 weeks of bladder retraining.
Drug therapy for women with overactive bladder should be offered with oxybutynin if conservative measures fail.
In women with detrusor instability who fail non operative therapy a trial of sacral neuromodulation may be considered, with
conversion to permanent implant if good response. Augmentation cystoplasty is an alternative but will involve long term
intermittent self catheterisation.
In women with stress urinary incontinence a urethral sling type procedure may be undertaken. Where cystocele is present in
association with incontinence it should be repaired particularly if it lies at the introitus.

NICE guidelines: Initial assessment urinary incontinence should be classified as stress/urge/mixed. At least 3/7 bladder diary if
153
unable to classify easily. Start conservative treatment before urodynamic studies if a diagnosis is obvious from the history.
Urodynamic studies if plans for surgery. Stress incontinence: Pelvic floor exercises 3/12, if fails consider surgery. Urge
incontinence: Bladder training >6/52, if fails for oxybutynin (antimuscarinic drugs) then sacral nerve stimulation. Pelvic floor
exercises offered to all women in their 1st pregnancy.

76. Which of the following is associated with reduced lung compliance?


A. Older age
B. Emphysematous type COPD
C. Decline in pulmonary blood flow
D. Adopting a vertical posture
E. Adjusting a ventilator to maintain high lung volumes

Answer: E

Increased lung compliance = Older age, COPD


Lung compliance is a measure of the ease of expansion of the lungs and thorax, determined by pulmonary volume and elasticity.
A high degree of compliance indicates a loss of elastic recoil of the lungs, as in old age or emphysema. This increased lung
compliance is due to loss of supportive tissue around the airways. While a normal lung has a high passive elastic recoil, the sick
lung has a decreased elasticity (i.e. decreased transpulmonary pressure) which leads to increased lung compliance.
Decreased compliance means that a greater change in pressure is needed for a given change in volume, as in atelectasis,
pulmonary fibrosis, pneumonia, or lack of surfactant.

77. Which of the following statements relating to alveolar ventilation is untrue?

A. Anatomical dead space is measured by helium dilution


B. Physiological dead space is increased in PE
C. Alveolar ventilation is defined as the volume of fresh air entering the alveoli per minute
D. Anatomical dead space is increased by noradrenaline
E. Type 2 pneumocytes in the alveoli secrete surfactant
Answer: A
Anatomical dead space is measured by Fowlers method.
A patient inhales 100% oxygen to empty the conducting zone gases of nitrogen and then exhales through a mouthpiece which
analyses the nitrogen concentration at the mouth. Initially the exhaled gases contain no nitrogen as this is dead space gas; the
nitrogen concentration will increase
as the alveolar gases are exhaled. Nitrogen which is measured following the breath of 100% oxygen must then have come only
from gas exchanging areas of the lung and not dead space.

Alveolar ventilation: Minute ventilation is the total volume of gas ventilated per minute.
MV (ml/min)= tidal volume x Respiratory rate (resps/min).

Dead space ventilation describes the volume of gas not involved in exchange in the blood. There are 2 types:

1. Anatomical dead space: 150mls: Volume of gas in the respiratory tree not involved in gaseous exchange: mouth,
pharynx, trachea, bronchi up to terminal bronchioles. Measured by Fowlers method. Increased by:Standing, increased
size of person, increased lung volume and drugs causing bronchodilatation e.g. Adrenaline
2. Physiological dead space: normal 150 mls, increases in ventilation/perfusion mismatch e.g. PE, COPD, hypotension

Volume of gas in the alveoli and anatomical dead space not involved in gaseous exchange. Alveolar ventilation is the volume of
fresh air entering the alveoli per minute. Alveolar ventilation = minute ventilation - Dead space volume

77. Which opioid receptor does morphine attach to?


A. mu
B. alpha
C. sigma
D. beta
E. kappa
Answer: A
Pethidine and other conventional opioids attach to this receptor.

Opioids: Combine to specific opiate receptors in the CNS (periaqueductal grey matter, limbic system, substantia gelatinosa).
Morphine attaches to mu1 receptors

78. Which of the following inhibits the secretion of insulin?


154
A. Adrenaline
B. Glucagon
C. Gastrin
D. Arginine
E. Vagal cholinergic activity
Answer: A

Inhibition of insulin release: Alpha adrenergic drugs, Beta blockers, Sympathetic nerves

79. What is measured to obtain renal plasma flow?


A. Creatinine
B. Para-amino hippuric acid (PAH)
C. Inulin
D. Glucose
E. Protein

Answer: B

amount of PAH in urine per unit time


Renal plasma flow =
difference in PAH concentration in the renal artery or vein

Normal value = 660ml/min

80. Which of the following is not a cause of hypercalcaemia?


A. Thiazides
B. Antacids
C. Coeliac disease
D. Sarcoidosis
E. Zolinger-Ellison syndrome
Answer: C
Mnemonic for the causes of hypercalcaemia: CHIMPANZEES: C alcium supplementation; H yperparathyroidism; I
atrogentic (Drugs: Thiazides); M ilk Alkali syndrome; P aget disease of the bone; A cromegaly and Addison's Disease; N
eoplasia; Z olinger-Ellison Syndrome (MEN Type I); E xcessive Vitamin D; E xcessive Vitamin A; S arcoidosis
Patients with coeliac disease tend to develop hypocalcaemia due to malabsorption of calcium by the bowel.

Main causes: Malignancy; Primary hyperparathyroidism. Less common: Sarcoidosis (extrarenal synthesis of calcitriol );
Thiazides, lithium; Immobilisation; Pagets disease; Vitamin A/D toxicity; Thyrotoxicosis; MEN; Milk alkali syndrome. Clinical
features: “Stones, bones, abdominal moans, and psychic groans”

81. Which of the following surgical procedures will have the greatest long term impact on a patients calcium metabolism?
A. Distal gastrectomy
B. Cholecystectomy
C. Extensive small bowel resection
D. Sub total colectomy
E. Gastric banding for obesity

Answer: C
Calcium is mainly absorbed from the small bowel and this will have a direct long term impact on calcium metabolism and
increase the risk of osteoporosis. Gastric banding and distal gastrectomy may affect a patients dietary choices but any potential
deleterious nutritional intake may be counteracted by administration of calcium supplements orally. Only 10% of calcium is
absorbed from the colon so that a sub total colectomy will only have a negligible effect.

82. A 52-year-old woman with a history of gastrectomy reports lethargy and a sore tongue. Blood tests are reported as
follows: Hb:10.7 g/dl; MCV:121 fl; Plt:177 * 10^9/l; WBC:5.4 * 10^9/l. What is the most likely cause?
A. Vitamin B12 deficiency
B. Vitamin C deficiency
C. Iron deficiency anaemia
D. Anaemia of chronic disease
E. Vitamin E deficiency
Answer: A
A history of gastrectomy and a macrocytic anaemia should indicate a diagnosis of B12 deficiency.
Vitamin B12 is mainly used in the body for red blood cell development and also maintenance of the nervous system. It is absorbed
after binding to intrinsic factor (secreted from parietal cells in the stomach) and is actively absorbed in the terminal ileum. A small
amount of vitamin B12 is passively absorbed without being bound to intrinsic factor.
155
Causes of vitamin B12 deficiency: pernicious anaemia; post gastrectomy; poor diet; disorders of terminal ileum (site of
absorption): Crohn's, blind-loop etc. Features of vitamin B12 deficiency: macrocytic anaemia; sore tongue and mouth;
neurological symptoms: e.g. Ataxia; neuropsychiatric symptoms: e.g. Mood disturbances. Management: if no neurological
involvement 1 mg of IM hydroxocobalamin 3 times each week for 2 weeks, then once every 3 months. If a patient is also deficient
in folic acid then it is important to treat the B12 deficiency first to avoid precipitating subacute combined degeneration of the cord

83. A 43 year old lady is diagnosed with primary hyperparathyroidism. Her serum PTH levels are elevated. An endocrine
surgeon performs a parathyroidectomy. How long will it take for the serum PTH levels to fall if the functioning
adenoma has been successfully removed?
A. 6 hours
B. 24 hours
C. 2 hours
D. 1 hour
E. 10 minutes
Answer: E
PTH has a very short half life usually less than 10 minutes. Therefore a demonstrable drop in serum PTH should be identified
within 10 minutes of removing the adenoma. This is useful clinically since it is possible to check the serum PTH intraoperatively
prior to skin closure and explore the other glands if levels fail to fall.

84. Which of the following statements relating to abnormal coagulation is false?


A. Warfarin affects the synthesis of factor 2,7,9,10
B. The prothrombin time is prolonged in Haemophilia A
C. Cholestatic jaundice can cause vitamin K deficiency
D. Disseminated intravascular coagulation is associated with thrombocytopenia
E. Massive transfusion is associated with reduced levels of factor 5 and 8

Answer: B

In haemophilia A the APTT is prolonged and there is reduced levels of factor 8:C. The bleeding time and PT are normal.
Cholestatic jaundice prevents the absorption of the fat soluble vitamin K. Massive transfusion (>10u blood or equivalent to the
blood volume of a person) puts the patient at risk of thrombocytopaenia, factor 5 and 8 deficiency.

Abnormal coagulation
Heparin: Prevents activation factors 2,9,10,11; Warfarin: Affects synthesis of factors 2,7,9,10; DIC: Factors 1,2,5,8,11; Liver
disease: Factors 1,2,5,7,9,10

Interpretation blood clotting test results

Disorder APTT PT Bleeding time


Haemophilia Increased Normal Normal
von Willebrand's disease Increased Normal Increased
Vitamin K deficiency Increased Increased Normal

85. Which of the following physiological changes do not occur following tracheostomy?
A. Alveolar ventilation is increased.
B. Anatomical dead space is reduced by 50%.
C. Work of breathing is increased.
D. Proportion of ciliated epithelial cells in the trachea may decrease.
E. Splinting of the larynx may lead to swallowing difficulties.
Answer: C
Work of breathing is decreased which is one reasons it is popular option for weaning ventilated patients. Humdified air in this
setting helps to reduce the viscosity of mucous that forms.

Trachea: Location:C6 vertebra to the upper border of T5 vertebra (bifurcation). Arterial and venous supply:Inferior thyroid
arteries and the thyroid venous plexus. Nerve:Branches of vagus, sympathetic and the recurrent nerves

Relations in the neck: Anterior(Superior to inferior): Isthmus of the thyroid gland, Inferior thyroid veins, Arteria thyroidea ima
(when that vessel exists), Sternothyroid, Sternohyoid, Cervical fascia, Anastomosing branches between the anterior jugular veins;
Posterior: Oesophagus. Laterally: Common carotid arteries, Right and left lobes of the thyroid gland, Inferior thyroid arteries,
Recurrent laryngeal nerves

156
Relations in the thorax: Anterior: Manubrium sterni, the remains of the thymus, the aortic arch, left common carotid arteries,
and the deep cardiac plexus; Lateral: In the superior mediastinum, on the right side is the pleura and right vagus; on its left side
are the left recurrent nerve, the aortic arch, and the left common carotid and subclavian arteries.

86. A 34 year old man presents with a peptic ulcer. Which of the following is responsible for the release of gastric acid?
A. Chief cells
B. Parietal cells
C. Brunners Glands
D. G Cells
E. None of the above
Answer: B

Parietal cells are responsible for the release of gastric acid. Brunners glands are found in the duodenum.

87. Which of the following does not lead to relaxation of the lower oesophageal sphincter?
A. Metoclopramide
B. Botulinum toxin type A
C. Nicotine
D. Alcohol
E. Theophylline
Answer: A
Metoclopramide acts directly on the smooth muscle of the LOS to cause it to contract.
Theophylline is a phosphodiesterase inhibitor (mimics action of prostaglandin E1) which causes relaxation of the LOS.

Peristalsis: Circular smooth muscle contracts behind the food bolus and longitudinal smooth muscle propels the food through the
oesophagus. Primary peristalsis spontaneously moves the food from the oesophagus into the stomach (9 seconds). Secondary
peristalsis occurs when food, which doesn't enter the stomach, stimulates stretch receptors to cause peristalsis. In the small
intestine each peristalsis waves slows to a few seconds and causes mixture of chime. In the colon three main types of peristaltic
activity are recognised (see below)

Colonic peristalsis
Segmentation contractions: Localised contractions in which the bolus is subjected to local forces to maximise mucosal
absorption. Antiperistaltic contractions towards ileum: Localised reverse peristaltic waves to slow entry into colon and
maximise absorption. Mass movement: Waves migratory peristaltic waves along the entire colon to empty the organ prior to the
next ingestion of food bolus

88. Which of the following is not released from the islets of Langerhans?
A. Pancreatic polypeptide
B. Glucagon
C. Secretin
D. Somatostatin
E. Insulin
Answer: C
Secretin is released from mucosal cells in the duodenum and jejunum.
Hormones released from the islets of Langerhans: Beta cells:Insulin (70% of total secretions); Alpha cells:Glucagon; Delta
cells:Somatostatin; F cells:Pancreatic polypeptide

89. Which of the following is not classically seen in coning resulting from raised intra cranial pressure?
A. Coma
B. Hypotension
C. Unreactive mid sized pupils
D. Cheyne Stokes style respiratory efforts
E. Bradycardia
Answer: B
Cushings triad: Widening of the pulse pressure; Respiratory changes; Bradycardia
Due to raised ICP systemic hypertension is usually seen. Compression of the respiratory centre will typically result in Cheyne
Stokes style respiration.

157
Coning: The cranial vault is a confined cavity apart from infants with a non fused fontanelle. Rises in ICP may be accommodated
by shifts of CSF. Once the CSF shifting has reached its capacity ICP will start to rise briskly. The brain autoregulates its blood
supply, as ICP rises the systemic circulation will display changes to try and meet the perfusion needs of the brain. Usually this will
involve hypertension. As CSF rises further, the brain will be compressed, cranial nerve palsies may be seen and compression of
essential centres in the brain stem will occur. When the cardiac centre is involved bradycardia will often develop.

90. Control of ventilation. Which statement is false?


A. Peripheral chemoreceptors are located in the bifurcation of the carotid arteries and arch of the aorta
B. Central chemoreceptors respond to changes in O2
C. The respiratory centres control the rate and depth of respiration
D. Involuntary control of respiration is from the medulla and pons
E. Irritant receptors cause bronchospasm
Answer: B
Central chemoreceptors: Respond to increased H+ in BRAIN INTERSTITIAL FLUID to increase ventilation.

91. A 54 year old lady has her serum calcium measured. Assuming her renal function is normal, what proportion of
calcium filtered at the glomerulus will be reabsorbed by the renal tubules?

A. 5%
B. 15%
C. 25%
D. 50%
E. 95%
Answer: E
Most filtered calcium is reabsorbed (95%) a rare disorder of familial hypercalcemic calciurea may affect this proportion.

91. Which of the following does not cause hyperkalaemia?


A. Haemolysis
B. Burns
C. Familial periodic paralysis
D. Type 4 renal tubular acidosis
E. Severe malnutrition
Answer: E
'Machine' - Causes of Increased Serum K+: M - Medications - ACE inhibitors, NSAIDS; A - Acidosis - Metabolic and
respiratory; C - Cellular destruction - Burns, traumatic injury; H - Hypoaldosteronism, haemolysis; I - Intake – Excessive; N -
Nephrons, renal failure; E - Excretion - Impaired

Familial periodic paralysis has subtypes associated with hyper and hypokalaemia.

92. Which of the following statements is true of glucagon?


A. Produced in response to hyperglycaemia
B. Released by beta cells
C. Inhibits gluconeogenesis
D. Produced in response to an increase of amino acids
E. Composed of 2 alpha polypeptide chains linked by hydrogen bonds
Answer: D
Glucagon is a protein comprised of a single polypeptide chain.
Produced by alpha cells of pancreatic islets of Langerhans in response to hypoglycaemia and amino acids.
It increases plasma glucose and ketones.

Glucagon, the hormonal antagonist to insulin, is released from the alpha cells of the Islets of Langerhans in the pancreas. It will
result in an increased plasma glucose level.Stimulation: Decreased plasma glucose; Increased catecholamines; Increased plasma
amino acids; Sympathetic nervous system; Acetylcholine; Cholecystokinin. Inhibition: Somatostatin; Insulin; Increased free fatty
acids and keto acids; Increased urea

93. A 28 year old man undergoes a completion right hemicolectomy for treatment of a 5cm appendiceal carcinoid. As part
of his follow up he is due to undergo 24 hour urine collection for 5-HIAA. Which of the following causes an elevated 5-
HIAA in a 24-hour urine collection?
A. Isoniazid
B. Oranges
C. Flucloxacillin
D. Amiodarone

158
E. Beef
Answer: A
It is important to be aware of what can falsely elevate 5-HIAA to avoid diagnosing carcinoid syndrome incorrectly. These include:
Food: spinach, cheese, wine, caffeine, tomatoes. Drugs: Isoniazid, Monoamine oxidase inhibitors

Carcinoid syndrome: Carcinoid tumours secrete serotonin; Originate in neuroendocrine cells mainly in the intestine (midgut-
distal ileum/appendix); Can occur in the rectum, bronchi; Hormonal symptoms mainly occur when disease spreads outside the
bowel
Clinical features: Onset: years. Flushing face. Palpitations. Tricuspid stenosis causing dyspnea. Asthma. Severe diarrhoea
(secretory, persists despite fasting)
Investigation: 5-HIAA in a 24-hour urine collection. Scintigraphy. CT scan
Treatment: Octreotide; Surgical removal

94. A 52 year old man develops septic shock following a Hartmans procedure for perforated diverticular disease. He is
started on an adrenaline infusion. Which of the following is least likely to occur?
A. Peripheral vasoconstriction
B. Coronary artery vasospasm
C. Gluconeogenesis
D. Lipolysis
E. Tachycardia
Answer: B
Its cardiac effects are mediated via β 1 receptors. The coronary arteries which have β 2 receptors are unaffected.

Adrenaline
Fight or Flight response: Catecholamine (phenylalanine and tyrosine); Neurotransmitter and hormone; Released by the adrenal
glands; Effects on α 1 and 2, β 1 and 2 receptors; Main effect on alpha 1 receptors in skeletal muscle-causing vasodilation;
Increase cardiac output and total peripheral resistance; This leads to vasoconstriction in the skin and kidneys causing a narrow
pulse pressure

Actions: α adrenergic receptors: Inhibits insulin secretion by the pancreas. Stimulates glycogenolysis in the liver and muscle.
Stimulates glycolysis in muscle

β adrenergic receptors: Stimulates glucagon secretion in the pancreas. Stimulates ACTH. Stimulates lipolysis by adipose tissue

95. Intra cranial pressure is governed by the principles of the Monroe-Kellie doctrine. To which of the following does this
concept not apply?
A. A 2 month old child
B. A 2 year old child
C. A 5 year old child
D. A 10 year old child
E. An adult
Answer: A

The Monroe-Kelly Doctrine assumes that the cranial cavity is a rigid box. In children with non fused fontanells this is not the case.

Pressure within the cranium is governed by the Monroe-Kelly doctrine. This considers the skull as a closed box. Increases in mass
can be accommodated by loss of CSF. Once a critical point is reached (usually 100- 120ml of CSF lost) there can be no further
compensation and ICP rises sharply. The next step is that pressure will begin to equate with MAP and neuronal death will occur.
Herniation will also accompany this process. The CNS can autoregulate its own blood supply. Vaso constriction and dilatation of
the cerebral blood vessels is the primary method by which this occurs. Extremes of blood pressure can exceed this capacity
resulting in risk of stroke. Other metabolic factors such as hypercapnia will also cause vasodilation, which is of importance in
ventilating head injured patients. The brain can only metabolise glucose, when glucose levels fall, consciousness will be impaired.

96. Which of the following is not caused by cortisol in the stress response?
A. Anti-inflammatory effects
B. Hypoglycaemia
C. Skeletal muscle protein breakdown
D. Stimulation of lipolysis
E. Mineralocorticoid effects
Answer: B
An 'anti insulin' effect occurs leading to hyperglycaemia.

97. Which of the following features does not put a patient at risk of refeeding syndrome?
159
A. BMI < 16 kg/m2
B. Alcohol abuse
C. Thyrotoxicosis
D. Chemotherapy
E. Diuretics

Answer: C

Diuretics increase the risk of re-feeding syndrome through a process of increasing the risk of depletion of key electrolytes.

Refeeding syndrome describes the metabolic abnormalities which occur on feeding a person a starved state. The metabolic
consequences include: Hypophosphataemia; Hypokalaemia; Hypomagnesaemia; Abnormal fluid balance
These abnormalities can lead to organ failure.

Re-feeding problems: If patient not eaten for > 5 days, aim to re-feed at < 50% energy and protein levels

High risk for re-feeding problems: If one or more of the following: BMI < 16 kg/m2; Unintentional weight loss >15% over 3-6
months; Little nutritional intake > 10 days; Hypokalaemia, Hypophosphataemia or hypomagnesaemia prior to feeding (unless
high). If two or more of the following: BMI < 18.5 kg/m2; Unintentional weight loss > 10% over 3-6 months; Little nutritional
intake > 5 days; PMH alcohol abuse or drug therapy including insulin, chemotherapy, diuretics, antacids

Prescription: Start at up to 10 kcal/kg/day increasing to full needs over 4-7 days; Start immediately before and during feeding:
oral thiamine 200-300mg/day, vitamin B co strong 1 tds and supplements; Give K+ (2-4 mmol/kg/day), phosphate (0.3-0.6
mmol/kg/day), magnesium (0.2-0.4 mmol/kg/day)

98. Which of the following statements relating to the regulation of renal blood flow is untrue?
A. In a healthy 70Kg male, the glomerular filtration rate will be the same at a systolic blood pressure of 120mmHg as a
systolic blood pressure of 95 mmHg
B. Over 90% of the blood supply to the kidney is distributed to the cortex
C. The kidney receives approximately 25% of the total cardiac output at rest
D. A decrease in renal perfusion pressure will cause the juxtaglomerular cells to secrete renin
E. Systolic blood pressures of less than 65mmHg will cause the mesangial cells to secrete aldosterone
Answer: E

The kidney autoregulates its blood supply over a range of systolic blood pressures. Drop in arterial pressure is sensed by the
juxtaglomerular cells and renin is released leading to the activation of the renin-angiontensin system. Mesangial cells are
contractile cells that are located in the tubule and have no direct endocrine function.

99. 39 year old lady undergoes a laparoscopic cholecystectomy as a daycase. The operation is more difficult than
anticipated and the surgeon places a drain to the liver bed. In recovery 1.5 litres of blood is seen to enter the drain.
Which of the following substances is the first to be released in this situation?
A. Angiotensinogen
B. Renin
C. Angiotensin I
D. Angiotensin II
E. Aldosterone
Answer: B
The decrease in blood pressure will be sensed by the juxtaglomerular cells in the kidney. This will cause renin secretion.

100.Which of the following drugs causes hyperkalaemia?


A. Heparin
B. Ciprofloxacin
C. Salbutamol
D. Levothyroxine
E. Codeine phosphate
Answer: A
Both unfractionated and low-molecular weight heparin can cause hyperkalaemia. This is thought to be caused by inhibition of
aldosterone secretion. Salbutamol is a recognised treatment for hyperkalaemia.

101.A 25-year-old man who has been morbidly obese for the past five years is reviewed in the surgical bariatric clinic. In
this patient, release of which of the following hormones would increase appetite?
A. Leptin
B. Thyroxine
160
C. Adiponectin
D. Ghrelin
E. Serotonin
Answer: D
Obesity hormones: leptin decreases appetite. Ghrelin increases appetite. Whilst thyroxine can increase appetite it does not fit
with the clinical picture being described.
Leptin: Leptin is thought to play a key role in the regulation of body weight. It is produced by adipose tissue and acts on satiety
centres in the hypothalamus and decreases appetite. More adipose tissue (e.g. in obesity) results in high leptin levels. Leptin
stimulates the release of melanocyte-stimulating hormone (MSH) and corticotrophin-releasing hormone (CRH). Low levels of
leptin stimulates the release of neuropeptide Y (NPY)

Ghrelin: Where as leptin induces satiety, ghrelin stimulates hunger. It is produced mainly by the fundus of the stomach and the
pancreas. Ghrelin levels increase before meals and decrease after meals

102.Which of the following bony complications is not linked to excess glucocorticoids?


A. Avascular necrosis
B. Vertebral body collapse
C. Increased susceptibility to osteomyelitis from strep viridans
D. Decreased absorption of calcium from the gut
E. Growth retardation in children
Answer: C
This infection is not typical of steroid excess, although general increased susceptibilty to infections is.

103.A 54-year-old woman is admitted to the Surgical Admissions Unit with abdominal pain. Blood tests taken on admission
show the following: Magnesium: 0.40 mmol/l. Which one of the following factors is most likely to be responsible for this
result?
A. Excessive resuscitation with intravenous saline
B. Digoxin therapy
C. Diarrhoea
D. Hypothermia
E. Rhabdomyolysis
Answer: C
Cause of low magnesium: Diuretics; Total parenteral nutrition; Diarrhoea; Alcohol; Hypokalaemia, hypocalcaemia
Features: Paraesthesia; Tetany; Seizures; Arrhythmias; Decreased PTH secretion --> hypocalcaemia; ECG features similar to
those of hypokalaemia. Exacerbates digoxin toxicity

104.A 43 year old man has a nasogastric tube inserted. The nurse takes a small aspirate of the fluid from the stomach and
tests the pH of the aspirate. What is the normal intragastric pH?
45. 0.5
46. 2
47. 4
48. 5
49. 6

Answer: B
The intragastric pH is usually 2. Administration of proton pump inhibitors can result in almost complete abolition of acidity

105.Which of the following is the equivalent of cardiac preload?


A. End diastolic volume
B. Stroke volume
C. Systemic vascular resistance
D. Mean arterial pressure
E. Peak systolic arterial pressure
Answer: A

Preload is the same as end diastolic volume. When it is increased slightly there is an associated increase in cardiac output (Frank
Starling principle). When it is markedly increased e.g. over 250ml then cardiac output falls.

106.A 73 year old female is referred to the surgical clinic with an iron deficiency anaemia. As part of the diagnostic work
up the doctor requests a serum ferritin level. Which of the conditions listed is most likely to lead to a falsely elevated
result?
A. Locally perforated sigmoid colonic adenocarcinoma

161
B. Colonic angiodysplasia
C. Dieulafoy lesion of the stomach
D. Transitional cell carcinoma of the bladder
E. Endometrial adenocarcinoma
Answer: A

A locally perforated colonic tumour will typically cause an intense inflammatory response and if peritonitis is not present
clinically then at the very least a localised abscess. This inflammatory process is the most likely (from the list) to falsely raise the
serum ferritin level. Angiodysplasia and dieulafoy lesions are mucosal arteriovenous malformations and unlikely to result in
considerable inflammatory activity.

Ferritin: Ferritin is an intracellular protein that binds iron and stores it to be released in a controlled fashion at sites where iron is
required. Because iron and ferritin are bound the total body ferritin levels may be decreased in cases of iron deficiency anaemia.
Measurement of serum ferritin levels can be useful in determining whether an apparently low haemoglobin and microcytosis is
truly caused by an iron deficiency state.
Ferritin is an acute phase protein and may be synthesised in increased quantities in situations where inflammatory activity is
ongoing. Falsely elevated results may therefore be encountered clinically and need to be taken in context of the clinical picture
and full blood count results.

107.Which of the following is least likely to cause a prolonged prothrombin time?


A. Cholestatic jaundice
B. Disseminated intravascular coagulation
C. Prolonged antibiotic treatment
D. Liver disease
E. Acquired factor 12 deficiency
Answer: E
Vitamin K deficiency results from cholestatic jaundice and prolonged antibiotic therapy. Acquired factor 12 deficiency causes
prolonged APTT.

108.Which statement about peristalsis is true?


A. Longitudinal smooth muscle propels the food bolus through the oesophagus
B. Secondary peristalsis occurs when there is no food bolus in the oesophagus
C. Food transfer from the oesophagus to the stomach is 4 seconds
D. Circular smooth muscle is not involved in peristalsis
E. Peristalsis only occurs in the oesophagus
Answer: A

109.24 year old man is injured in a road traffic accident. He becomes oliguric and his renal function deteriorates. Which of
the options below would favor acute tubular necrosis over pre renal uraemia?
A. No response to intravenous fluids
B. Urinary sodium < 20mmol/L
C. Bland coloured urinary sediment
D. Increased urine specific gravity
E. None of the above
Answer: A

In acute tubular necrosis there is no response to intravenous fluids because the damage occurs from within the renal system rather
than as a result of volume depletion.

Acute renal failure: Pre renal failure vs. acute tubular necrosis: Prerenal uraemia - kidneys retain sodium to preserve volume
Pre-renal uraemia Acute tubular necrosis
Urine sodium < 20 mmol/L > 30 mmol/L
Fractional sodium excretion* < 1% > 1%
Fractional urea excretion** < 35% >35%
Urine:plasma osmolality > 1.5 < 1.1
Urine:plasma urea > 10:1 < 8:1
Specific gravity > 1020 < 1010
Urine 'bland' sediment brown granular casts
Response to fluid challenge Yes No

162
*fractional sodium excretion = (urine sodium/plasma sodium) / (urine creatinine/plasma creatinine) x 100
**fractional urea excretion = (urine urea /blood urea ) / (urine creatinine/plasma creatinine) x 100

110.Which of the following is not an effect of cholecystokinin?


A. It causes gallbladder contraction
B. It increases the rate of gastric emptying
C. It relaxes the sphincter of oddi
D. It stimulates pancreatic acinar cells
E. It has a trophic effect on pancreatic acinar cells
Answer:B

It decreases the rate of gastric emptying.

111.Which part of the jugular venous waveform is associated with the closure of the tricuspid valve?
A. a wave
B. c wave
C. x descent
D. y descent
E. v wave

Answer: B

JVP: {C} wave - {c}losure of the tricuspid valveThe c wave of the jugular venous waveform is associated with the closure of the
tricuspid valve.

112.Which one of the following serum proteins is most likely to increase in a patient with severe sepsis?
A. Transferrin
B. Transthyretin
C. Ferritin
D. Albumin
E. Cortisol binding protein
Answer: C

Ferritin can be markedly increased during acute illness. The other parameters tend to decrease during an acute phase response.
Acute phase proteins: CRP; procalcitonin; ferritin; fibrinogen; alpha-1 antitrypsin; caeruloplasmin; serum amyloid A;
haptoglobin; complement

During the acute phase response the liver decreases the production of other proteins (sometimes referred to as negative acute
phase proteins). Examples include:albumin; transthyretin (formerly known as prealbumin); transferrin; retinol binding protein;
cortisol binding protein

Theme: Critical care

B. Hypovolaemia
C. Normal
D. Cardiogenic shock
E. Septic shock

For each of the scenarios outlined in the tables below, please select the most likely diagnosis from the list. Each option may be
used once, more than once or not at all.

113.A 45 year old man is admitted to the intensive care unit following a laparotomy. He has a central line, pulmonary artery
catheter and arterial lines inserted. The following results are obtained:

Pulmonary artery occlusion pressure: Low; Cardiac output: Low; Systemic vascular resistance:High

Answer: Hypovolaemia
Cardiac output is lowered in hypovolaemia due to decreased preload.
163
114.A 75 year old man is admitted to the intensive care unit following a laparotomy. He has a central line, pulmonary
artery catheter and arterial lines inserted. The following results are obtained: Pulmonary artery occlusion
pressure:High; Cardiac output: Low; Systemic vascular resistance:High

Answer: Cardiogenic shock


In cardiogenic shock pulmonary pressures are often high. This is the basis for the use of venodilators in the treatment of
pulmonary oedema.

115.A 22 year old lady is admitted to the intensive care unit following a laparotomy. She has a central line, pulmonary
artery catheter and arterial lines inserted. The following results are obtained: Pulmonary artery occlusion pressure:
Low; Cardiac output: High; Systemic vascular resistance: Low

Answer: Septic shock


Decreased SVR is a major feature of sepsis. A hyperdynamic circulation is often present. This is the reason for the use of
vasoconstrictors.
Pulmonary artery occlusion pressure monitoring: The pulmonary artery occlusion pressure is an indirect measure of left atrial
pressure, and thus filling pressure of the left heart. The low resistance within the pulmonary venous system allows this useful
measurement to be made. The most accurate trace is made by inflating the balloon at the catheter tip and "floating" it so that it
occludes the vessel. If it is not possible to occlude the vessel in this way then the measurement gained will be the pulmonary
artery end diastolic pressure. Interpretation of PAOP(mmHg): Normal: 8-12; Low:<5: Hypovolaemia; Low with pulmonary
oedema: <5:ARDS; High: >18:Overload. When combined with measurements of systemic vascular resistance and cardiac output
it is possible to accurately classify patients.
Systemic vascular resistance: Derived from aortic pressure, right atrial pressure and cardiac output.
SVR=80(mean aortic pressure-mean right atrial pressure)/cardiac output

116.A 73 year old man has an arterial line in situ. On studying the trace the incisura can be seen. What is the physiological
event which accounts for this process?
A. Atrial repolarisation
B. Mitral valve closure
C. Ventricular repolarisation
D. Elastic recoil of the aorta
E. Tricuspid valve closure

Answer: D

It is the temporary rise in aortic pressure occurring as a result of elastic recoil.

117.A 72-year-old woman is admitted to the acute surgical unit with profuse vomiting. Admission bloods show the
following: Na+: 131 mmol/l; K+:2.2 mmol/l; Urea: 3.1 mmol/l; Creatinine:56 µmol/l; Glucose: 4.3 mmol/l. Which one of
the following ECG features is most likely to be seen?
A. Short PR interval
B. Short QT interval
C. Flattened P waves
D. J waves
E. U waves

Answer: E

Hypokalaemia - U waves on ECG


J waves are seen in hypothermia whilst delta waves are associated with Wolff Parkinson White syndrome.

ECG features in hypokalemia: U waves; Small or absent T waves (occasionally inversion); Prolonged PR interval; ST
depression; Long QT interval. Mnemonic: In Hypokalaemia, U have no Pot and no T, but a long PR and a long QT!

118.Which of the following secretions contains the highest composition of potassium?


A. Bile
B. Small bowel
C. Pancreatic juice
D. Gastric juice
E. Saliva
Answer: E

164
Of the secretions shown above, saliva has the greatest composition of potassium. The exact amount secreted will depend upon
aldosterone levels. Potassium composition of secretions: Saliva:20-60 mmol/L; Gastric juice:5-10 mmol/L; Bile:5-8 mmol/L;
Pancreatic juice:4-5 mmol/L; Small bowel:4-10 mmol/L

119.The oxygen-haemoglobin dissociation curve is shifted to the left in:


A. With decreased 2,3-DPG in transfused red cells
B. Respiratory acidosis
C. High altitude
D. Pyrexia
E. Haemolytic anaemia

Answer: A

S shaped curve. The curve is shifted to the left when there is a decreased oxygen requirement by the tissue. This
includes:1.Hypothermia; 2. Alkalosis; 3. Reduced levels of DPG: DPG is found in erythrocytes and is reduced in non exercising
muscles, i.e. when there is reduced glycolysis. 4. Polycythaemia

120.A homeless 42 year old male had an emergency inguinal hernia repair 24 hours previously. He has a BMI of 15. He has
been put on a feeding regime of 35 kcal/kg/day with no additional medications. The nursing staff contact you as he has
become confused and unsteady. On examination the patient is disorientated to place, has diplopia and nystagmus.
What is the most likely diagnosis?
45. Cerebellar stroke
46. Acute dystonic reaction
47. Refeeding syndrome
48. Parkinsonism
49. Wernickes encephalopathy
Answer: E
Triad of Wernicke encephalopathy: Acute confusion, Ataxia, Opthalmoplegia
This patient has received a carbohydrate rich diet without any thiamine or vitamin B co strong replacement. This has led to
Wernickes encephalopathy, which classically presents with confusion, ataxia and opthalmoplegia. Characteristically it is
associated with chronic alcoholism, however it is also known to occur post bariatric surgery.

121.A 22 year old lady receives intravenous morphine for acute abdominal pain. Which of the following best accounts for
its analgesic properties?
45. Binding to δ opioid receptors in the brainstem
46. Binding to δ opioid receptors at peripheral nerve sites
47. Binding to β opioid receptors within the CNS
48. Binding to α opioid receptors within the CNS
49. Binding to µ opioid receptors within the CNS

Answer: E

Morphine: Strong opiate analgesic. It is a pro- type narcotic drug and its effects mediated via the µ opioid receptor. Its clinical
effects stem from binding to these receptor sites within the CNS and gastrointestinal tract. Unwanted side effects include nausea,
constipation, respiratory depression and, if used long term, addiction.
It may be administered orally or intravenously. It can be reversed with naloxone.

122.Which one of the following reduces the secretion of renin?


A. Erect posture
B. Adrenaline
C. Hyponatraemia
D. Hypotension
E. Beta-blockers
Answer: E
Renin is secreted by juxtaglomerular cells and hydrolyses angiotensinogen to produce angiotensin I
Factors stimulating renin secretion: Low BP; Hyponatraemia; Sympathetic nerve stimulation; Catecholamines; Erect posture.
Factors reducing renin secretion: drugs: beta-blockers, NSAIDs

123.Which of the following stimulates prolactin release?


A. Leutinising hormone

165
B. Dopamine
C. Thyrotropin releasing hormone
D. Oestrogen
E. Follicle stimulating hormone
Answer: C

TRH stimulates prolactin release. Dopamine suppresses the release of prolactin. Prolactin: is a peptide hormone released from the
anterior pituitary. It is under tonic dopamine inhibition, thyrotropin releasing hormone has a stimulatory effect on release.
Prolactin release stimulates milk production but also reduces gonadal activity. It decreases GnRH pulsatility at the hypothalamic
level and to a lesser extent, blocks the action of LH on the ovary or testis.

124.Which of the following statements are not typically true in hypokalaemia?


A. It may occur as a result of mechanical bowel preparation
B. Chronic vomiting may increase renal potassium losses
C. It may be associated with aciduria
D. It may cause hyponatraemia
E. It often accompanies acidosis
Answer: E

Potassium depletion occurs either through the gastrointestinal tract or the kidney. Chronic vomiting in itself is less prone to induce
potassium loss than diarrhoea as gastric secretions contain less potassium than those in the lower GI tract. If vomiting produces a
metabolic alkalosis then renal potassium wasting may occur as potassium is excreted in preference to hydrogen ions. The converse
may occur in potassium depletion resulting in acid urine.

Potassium and hydrogen can be thought of as competitors. Hyperkalaemia tends to be associated with acidosis because as
potassium levels rise fewer hydrogen ions can enter the cells

Hypokalaemia with alkalosis: Vomiting, Diuretics, Cushing's syndrome, Conn's syndrome (primary hyperaldosteronism)
Hypokalaemia with acidosis: Diarrhoea, Renal tubular acidosis, Acetazolamide, Partially treated diabetic ketoacidosis

Theme: Vitamin deficiency

A. Vitamin A
B. Vitamin B1
C. Vitamin B12
D. Vitamin B3
E. Vitamin C
F. Vitamin K
G. Vitamin D

Please select the vitamin deficiency most closely associated with the situation described. Each option may be used once, more
than once or not at all.

125.A 3 year old child presents with Rickets

Answer: Vitamin D
Vitamin D is needed to help mineralise bone. When this is deficient, mineralisation is inadequate and deformities mayt result.

126.A 44 year old lady presents with jaundice. Following a minor ward based surgical procedure she develops troublesome
and persistent bleeding.

Answer: Vitamin K
Patients who are jaundiced usually have impaired absorption of vitamin K. This can result in loss of the vitamin K dependent
clotting factors and troublesome bleeding.

127.A 69 year old man who has been living in sheleted accomodation for many months, with inadequate nutrition notices
that his night vision is becoming impaired.

Answer: Vitamin A
Loss of vitamin A will result in impaire rhodopsin synthesis and poor night vision.

Vitamin deficiency
166
A: Night blindness, Epithelial atrophy, Infections. B1:Beriberi; B2:Dematitis and photosensitivity; B3:Pellagra; B12:Pernicious
anaemia; C:Poor wound healing, Impaired collagen synthesis. D:Rickets (Children), Osteomalacia (Adults). K:Clotting disorders

128.Which of the following statements is true with regards to gastric secretions?


A. During the cephalic phase 40% of the total gastric secretion occurs
B. Histamine release stimulates acid secretion in the stomach
C. Secretin stimulates gastric acid secretion
D. Intrinsic factor combines with B1 in the stomach
E. G cells are only present in the gastric mucosa
Answer: B

Histamine is released from enterochromaffin cells in the stomach mucosa which stimulates acid secretion.
Intrinsic factor combines with B12 to prevent acid digestion in the stomach.
G cells can be found in the duodenum and jejunum

129.A 55 year old man undergoes a laparotomy and repair of incisional hernia. Which of the following hormones is least
likely to be released in increased quantities following the procedure?
A. Insulin
B. ACTH
C. Glucocorticoids
D. Aldosterone
E. Growth hormone
Answer: A
Insulin and thyroxine are often have reduced levels of secretion in the post operative period. This, coupled with increased
glucocorticoid release may cause difficulty in management of diabetes in individuals with insulin resistance.

130.Which of the following is not a major function of the spleen in adults?


A. Iron reutilisation
B. Storage of platelets
C. Storage of monocytes
D. Haematopoeisis in haematological disorders
E. Storage red blood cells

Answer: E

The reservoir function of the spleen is less marked in humans than other animals (e.g. pigs) and in normal individuals it can
sequester between 5 and 10% of the red cell mass. The other stated processes are major splenic functions and this accounts for the
answer provided.

Spleen: Embryology: derived from mesenchymal tissue. Shape: orange segment. Position: below 9th-12th ribs. Weight: 75-150g

Relations: Superiorly- diaphragm. Anteriorly- gastric impression. Posteriorly- kidney. Inferiorly- colon. Hilum: tail of pancreas
and splenic vessels (splenic artery divides here, branches pass to the white pulp transporting plasma). Forms apex of lesser sac
(containing short gastric vessels)

Contents: White pulp: immune function. Contains central trabecular artery. The germinal centres are supplied by arterioles called
penicilliary radicles. Red pulp: filters abnormal red blood cells

Function: Filtration of abnormal blood cells and foreign bodies such as bacteria. Immunity: IgM. Production of properdin, and
tuftsin which help target fungi and bacteria for phagocytosis. Haematopoiesis: up to 5th month gestation or in haematological
disorders. Pooling: storage of 40% platelets. Iron reutilization. Storage red blood cells-animals, not humans. Storage monocytes

Disorders of the spleen


Massive splenomegaly: Myelofibrosis. Chronic myeloid leukaemia. Visceral leishmaniasis (kala-azar). Malaria. Gaucher's
syndrome. Other causes (as above plus): Portal hypertension e.g. secondary to cirrhosis. Lymphoproliferative disease e.g. CLL,
Hodgkin's. Haemolytic anaemia. Infection: hepatitis, glandular fever. Infective endocarditis. Sickle-cell*, thalassaemia
Rheumatoid arthritis (Felty's syndrome)
*the majority of adults patients with sickle-cell will have an atrophied spleen due to repeated infarction

131.Which one of the following is associated with increased lung compliance?


A. Kyphosis
B. Pulmonary oedema
C. Emphysema
167
D. Pulmonary fibrosis
E. Pneumonectomy
Answer: C

Respiratory physiology: lung compliance: Lung compliance is defined as change in lung volume per unit change in airway
pressure. Causes of increased compliance: Age. Emphysema (this is due to loss alveolar walls and associated elastic tissue).
Causes of decreased compliance: pulmonary oedema, pulmonary fibrosis, pneumonectomy, kyphosis

132.Which of the following areas is predominantly concerned with thermoregulation?


A. Hypothalamus
B. Anterior pituitary
C. Cerebellum
D. Brain stem
E. Temporal lobe
Answer: A
The hypothalamus is primarily concerned with thermoregulation. It may relay to the cerebral cortex to induce behavioural
adaptation to facilitate the thermoregulatory process.

Thermoregulation: The hypothalamus is the main centre for thermoregulation. Peripheral and central thermoreceptors relay to
this region. Central chemoreceptors play the main role in maintenance of core temperature. Hypothalamus may initiate
involuntary motor responses to raise body temperature (e.g.shivering). It will also stimulate the sympathetic nervous system to
produce peripheral vasoconstriction and release of adrenaline from the adrenal medulla. Heat loss is governed by behavioural
responses and by autonomic responses including peripheral vasodilation. Heat loss can be maintained within the thermoneutral
zone (25 to 30 degrees) although the absolute value depends upon atmospheric humidity. Sepsis results in the release of cytokines
that reset the thermoregulatory centre resulting in fever.

133.Which of the following drugs does not interfere with the measurement of cortisol levels?
A. Dexamethasone
B. Prednisolone
C. Hydrocortisone IV
D. Hydrocortisone PO
E. Hydrocortisone IM
Asnwer: A
Dexamethasone can be given as glucorticoid replacement during testing for addisons or adrenal insufficiency as it does not
interfere with cortisol levels. For example if you have a patient with polymyalgia rheumatica and they are on longterm
prednisolone, you can replace the prednisolone with dexamethasone to undertake a short synacthen test.

134.An elderly lady who presented with weight loss and malabsorption was found to have amyloid of the small bowel. On
presentation she was found to have osteomalacia and was hypocalcaemic. Over the past seven days she has received
total parenteral nutrition with adequate calcium replacement. Despite this she remained hypocalcaemic. Deficiency of
which of the following electrolytes is most likely to account for this process?
A. Magnesium
B. Potassium
C. Sodium
D. Phosphate
E. None of the above
Answer: A
Patients with malabsorption may develop magnesium deficiency, although her TPN feeds may have contained magnesium it may
not have been sufficient to correct her losses. Sodium, phosphate and potassium would not have this effect on serum calcium.

Combined deficiency of magnesium and calcium: Magnesium is required for both PTH secretion and its action on target
tissues. Hypomagnesaemia may both cause hypocalcaemia and render patients unresponsive to treatment with calcium and
vitamin D supplementation. Magnesium is the fourth most abundant cation in the body. The body contains 1000mmol, with half
contained in bone and the remainder in muscle, soft tissues and extracellular fluid. There is no one specific hormonal control of
magnesium and various hormones including PTH and aldosterone affect the renal handling of magnesium. Magnesium and
calcium interact at a cellular level also and as a result decreased magnesium will tend to affect the permeability of cellular
membranes to calcium, resulting in hyperexcitability.

135.A 19 year old man is attacked outside a club and beaten with a baseball bat. He sustains a blow to the right side of his
head. He is brought to the emergency department and a policy of observation is adopted. His glasgow coma score
deteriorates and he becomes comatose. Which of the following haemodynamic parameters is most likely to be present?
168
A. Hypertension and bradycardia
B. Hypotension and tachycardia
C. Hypotension and bradycardia
D. Hypertension and tachycardia
E. Normotension and bradycardia
Answer: A
Hypertension and bradycardia are seen prior to coning. The brain autoregulates its blood supply by controlling systemic blood
pressure.

Head injury: Patients who suffer head injuries should be managed according to ATLS principles and extra cranial injuries should
be managed alongside cranial trauma. Inadequate cardiac output will compromise CNS perfusion irrespective of the nature of the
cranial injury.

Types of traumatic brain injury


Extradural haematoma:Bleeding into the space between the dura mater and the skull. Often results from acceleration-
deceleration trauma or a blow to the side of the head. The majority of extradural haematomas occur in the temporal region where
skull fractures cause a rupture of the middle meningeal artery.
Features: Raised intracranial pressure. Some patients may exhibit a lucid interval

Subdural haematoma:Bleeding into the outermost meningeal layer. Most commonly occur around the frontal and parietal lobes.
May be either acute or chronic. Risk factors include old age and alcoholism. Slower onset of symptoms than a extradural
haematoma.

Subarachnoid haemorrhage:Usually occurs spontaneously in the context of a ruptured cerebral aneurysm but may be seen in
association with other injuries when a patient has sustained a traumatic brain injury

Pathophysiology: Primary brain injury may be focal (contusion/ haematoma) or diffuse (diffuse axonal injury). Diffuse axonal
injury occurs as a result of mechanical shearing following deceleration, causing disruption and tearing of axons. Intra-cranial
haematomas can be extradural, subdural or intracerebral, while contusions may occur adjacent to (coup) or contralateral (contre-
coup) to the side of impact. Secondary brain injury occurs when cerebral oedema, ischaemia, infection, tonsillar or tentorial
herniation exacerbates the original injury. The normal cerebral auto regulatory processes are disrupted following trauma rendering
the brain more susceptible to blood flow changes and hypoxia. The Cushings reflex (hypertension and bradycardia) often occurs
late and is usually a pre terminal event

Management: Where there is life threatening rising ICP such as in extra dural haematoma and whilst theatre is prepared or
transfer arranged use of IV mannitol/ frusemide may be required. Diffuse cerebral oedema may require decompressive
craniotomy. Exploratory Burr Holes have little management in modern practice except where scanning may be unavailable and to
thus facilitate creation of formal craniotomy flap. Depressed skull fractures that are open require formal surgical reduction and
debridement, closed injuries may be managed non operatively if there is minimal displacement. ICP monitoring is appropriate in
those who have GCS 3-8 and normal CT scan. ICP monitoring is mandatory in those who have GCS 3-8 and Abnormal CT scan.
Hyponatraemia is most likely to be due to syndrome of inappropriate ADH secretion. Minimum of cerebral perfusion pressure of
70mmHg in adults. Minimum cerebral perfusion pressure of between 40 and 70 mmHg in children.

Interpretation of pupillary findings in head injuries


Pupil size Light response Interpretation

Unilaterally dilated Sluggish or fixed 3rd nerve compression secondary to tentorial herniation

Bilaterally dilated Sluggish or fixed Poor CNS perfusion; Bilateral 3rd nerve palsy

Unilaterally dilated or equal Cross reactive (Marcus - Gunn) Optic nerve injury
Opiates; Pontine lesions; Metabolic encephalopathy
Bilaterally constricted May be difficult to assess

Unilaterally constricted Preserved Sympathetic pathway disruption

136.Which of the following drugs does not cause pseudohaematuria?


A. Rifampicin
169
B. Quinine
C. Noradrenaline
D. Levodopa
E. Phenytoin
Answer: C
Rifampicin, phenytoin, levodopa, methyldopa, and quinine all cause pseudohaematuria.

Causes of haematuria: Trauma Injury to renal tract. Renal trauma commonly due to blunt injury (others penetrating injuries).
Ureter trauma rare: iatrogenic. Bladder trauma: due to RTA or pelvic fractures. Infection: Remember TB. Malignancy :Renal
cell carcinoma (remember paraneoplastic syndromes): painful or painless. Urothelial malignancies: 90% are transitional cell
carcinoma, can occur anywhere along the urinary tract. Painless haematuria. Squamous cell carcinoma and adenocarcinoma: rare
bladder tumours. Prostate cancer. Penile cancers: SCC. Renal disease:Glomerulonephritis. Stones: Microscopic haematuria
common. Structural abnormalities: Benign prostatic hyperplasia (BPH) causes haematuria due to hypervascularity of the
prostate gland, Cystic renal lesions e.g. polycystic kidney disease, Vascular malformations, Renal vein thrombosis due to renal
cell carcinoma. Coagulopathy: Causes bleeding of underlying lesions. Drugs: Cause tubular necrosis or interstitial nephritis:
aminoglycosides, chemotherapy. Interstitial nephritis: penicillin, sulphonamides, and NSAIDs. Anticoagulants. Benign: Exercise.
Gynaecological: Endometriosis: flank pain, dysuria, and haematuria that is cyclical. Iatrogenic: Catheterisation, Radiotherapy;
cystitis, severe haemorrhage, bladder necrosis. Pseudohaematuria

137.A 74-year-old woman with thyroid cancer is admitted due to shortness of breath. What is the best investigation to
assess for possible compression of the upper airways?
A. Arterial blood gases
B. Forced vital capacity
C. Transfer factor
D. Peak expiratory flow rate
E. Flow volume loop
Answer: E
Flow volume loop is the investigation of choice for upper airway compression.
A normal flow volume loop is often described as a 'triangle on top of a semi circle'. Flow volume loops are the most suitable way
of assessing compression of the upper airway

138.Which of the following statements relating to cerebrospinal fluid is untrue?


A. The choroid plexus is only present in the lateral ventricles
B. Total CSF volume is 100-150ml
C. CSF pressure is usually 10-15mmHg
D. The cerebral aqueduct connects the third and fourth ventricles
E. The foramen of Luschka are paired and lie laterally in the fourth ventricle
Answer: A
The choroid plexus lies in all ventricles.

139.Which substance can be used to achieve the most accurate measurement of the glomerular filtration rate?
A. Glucose
B. Protein
C. Inulin
D. Creatine
E. Para-amino hippuric acid
Answer: C
Creatinine declines with age due to decline in renal function and muscle mass. Glucose, protein (amino acids) and PAH are
reabsorbed by the kidney.

140.A 45 year old man sustains a closed head injury. He is initially alert, however, his level of consciousness deteriorates
on arrival at hospital. An intra cranial pressure monitor is inserted. What is the normal intracranial pressure?
A. 35 - 45mm Hg
B. 45 - 55mm Hg
C. <15mm Hg
D. 25 - 35mm Hg
E. 25 - 30 mm Hg
Answer: C
The normal intracranial pressure is between 7 and 15 mm Hg. The brain can accommodate increases up to 24 mm Hg, thereafter
clinical features will become evident.

141.A 55-year-old man with a history of type 2 diabetes mellitus, bipolar disorder and chronic obstructive pulmonary
disease has bloods taken during a pre operative assessment of an inguinal hernia repair:Na+:125 mmol/l; K+:3.8

170
mmol/l; Bicarbonate:24 mmol/l; Urea:3.7 mmol/l; Creatinine:92 µmol/l Due to his smoking history a chest x-ray is
ordered which is reported as normal. The Consultant asks you what is the most likely cause for the hyponatraemia?
A. Metformin
B. Lithium
C. Carbamazepine
D. Sodium valproate
E. Pioglitazone
Answer: C
SIADH - drug causes: carbamazepine, sulfonylureas, SSRIs, tricyclics
Lithium can cause diabetes insipidus but this is generally associated with a high sodium. Lithium only tends to cause raised
antidiuretic hormone levels following a severe overdosage.

Syndrome of inappropriate antidiuretic hormone (SIADH): causes: Malignancy: especially small cell lung cancer. Also:
pancreas, prostate. Neurological: stroke; subarachnoid haemorrhage; subdural haemorrhage; meningitis/encephalitis/abscess.
Infections: tuberculosis; pneumonia. Drugs: sulfonylureas; SSRIs, tricyclicsl; carbamazepine; vincristine; cyclophosphamide.
Other causes: positive end-expiratory pressure (PEEP); porphyrias

142.A 39 year old lady has recurrent attacks of biliary colic. What is the approximate volume of bile to enter the duodenum
per 24 hours?
A. 500 mL
B. 50 mL
C. 100 mL
D. 2000 mL
E. 150 mL
Answer: A
Between 500 mL and 1.5 L of bile enters the small bowel daily. Most bile salts are recycled by the enterohepatic circulation.
When the gallbladder contracts the lumenal pressure is approximately 25cm water, which is why biliary colic may be so painful.

Bile: is produced at a rate of between 500ml and 1500mL per day. Bile is composed of bile salts, bicarbonate, cholesterol, steroids
and water. There are three main factors regulating bile flow; hepatic secretion, gall bladder contraction and sphincter of oddi
resistance. Bile salts are absorbed in the terminal ileum (and recycled to the liver). Over 90% of all bile salts are recycled in this
way, such that the total pool of bile salts is recycled up to six times a day. Primary bile salts: Cholate and chenodeoxycholate.
Secondary bile salts: Formed by bacterial action on primary bile salts. These are deoxycholate and lithocholate. Of these
deoxycholate is reabsorbed, whilst lithocholate is insoluble and excreted. Pathophysiology of gallstones: Bile salts have a
detergent action. They aggregate to form micelles and these have a lipid centre in which fats may be transported. Excessive
quantities of cholesterol cannot be transported in this way and will tend to precipitate, resulting in the formation of cholesterol rich
gallstones.

143.At which of the following sites is the most water absorbed?


A. Right colon
B. Left colon
C. Stomach
D. Jejunum
E. Duodenum

Water absorption in the gastrointestinal tract predominantly occurs in the small bowel (jejunum and ileum). The colon is an
important site of water absorption, however, its overall contribution is relatively small. The importance of the colonic component
to water absorption may increase following extensive small bowel resections.

Water absorption: During a 24 hours period the average person will ingest up to 2000 ml of liquid orally. In addition a further
8000ml of fluid will enter the small bowel as gastrointestinal secretions. Intestinal water absorption is a passive process and is
related to solute load. In the jejunum the active absorption of glucose and amino acids will create a concentration gradient that
water will flow across. In the ileum most water is absorbed by a process of facilitated diffusion (with sodium). Approximately
150ml of water enters the colon daily, most is absorbed, the colon can adapt to, and increase this amount following resection.

144.Which of the following is not a characteristic of the proximal convoluted tubule in the kidney?
A. Up to 95% of filtered amino acids will be reabsorbed at this site
B. It is a risk of damage in a patient with compartment syndrome due to a tibial fracture
C. It is responsible for absorbing more than 50% of filtered water
D. Its secretory function is most effective at low systolic blood pressures (typically less than 100mmHg)

171
E. Glucose is reabsorbed by a process of facilitated diffusion
Answer: D
The proximal convoluted tubule may undergo necrosis in situations such as compartment syndrome. It is responsible for
reabsorbing up to two thirds of filtered water. Low systolic blood pressures (below the renal autoregulatory range) are a risk factor
for acute tubular necrosis. Within the autoregulatory range the absolute value of systolic BP has little effect.

145.Which of the following does not cause a normal anion gap acidosis?
45. Pancreatic fistula
46. Acetazolamide
47. Uraemia
48. Ureteric diversion
49. Renal tubular acidosis

Answer: C

Normal Gap Acidosis: HARDUP: H - Hyperalimentation/hyperventilation ; A - Acetazolamide ; R - Renal tubular acidosis; D -


Diarrhoea; U - Ureteral diversion; P - Pancreatic fistula/parenteral saline. Uraemia will typically cause a high anion gap acidosis.
It is one of the unmeasured anions.

146.Which one of the following would cause a rise in the carbon monoxide transfer factor (TLCO)?
A. Emphysema
B. Pulmonary embolism
C. Pulmonary haemorrhage
D. Pneumonia
E. Pulmonary fibrosis
Answer: C
Transfer factor: raised: asthma, haemorrhage, left-to-right shunts, polycythaemia. low: everything else
Where alveolar haemorrhage occurs the TLCO tends to increase due to the enhanced uptake of carbon monoxide by intra-alveolar
haemoglobin.

Transfer factor: The transfer factor describes the rate at which a gas will diffuse from alveoli into blood. Carbon monoxide is
used to test the rate of diffusion. Results may be given as the total gas transfer (TLCO) or that corrected for lung volume (transfer
coefficient, KCO)
Causes of a raised TLCO Causes of a lower TLCO

 asthma  pulmonary fibrosis


 pulmonary haemorrhage (Wegener's, Goodpasture's)  pneumonia
 left-to-right cardiac shunts  pulmonary emboli
 polycythaemia  pulmonary oedema
 hyperkinetic states  emphysema
 male gender, exercise  anaemia
 low cardiac output

KCO also tends to increase with age. Some conditions may cause an increased KCO with a normal or reduced TLCO

 pneumonectomy/lobectomy
 scoliosis/kyphosis
 neuromuscular weakness
 ankylosis of costovertebral joints e.g. ankylosing spondylitis

147.Release of somatostatin from the pancreas will result in which of the following?
A. Decrease in pancreatic exocrine secretions
B. Contraction of the gallbladder
C. Increase in the rate of gastric emptying
D. Increased synthesis of growth hormone
E. Increased insulin release
Answer: A
Octreotide reduces exocrine pancreatic secretions so is used to treat high output pancreatic fistulae (though parenteral feeding is
most effective). Other uses include variceal bleeding and treatment of acromegaly.
Inhibits growth hormone and insulin release (when released from pancreas).
Somatostatin is also released by the hypothalamus causing a negative feedback response on growth hormone.

172
Somatostatin: is produced in the D cells of the pancreatic islets. It is also produced in the gut (enterochromaffin cells) and is
found in brain tissue. Those substances that stimulate insulin release will also induce somatostatin secretion. It is an inhibitor of
growth hormone, it also delays gastric emptying and reduces gastrin secretion.
It reduces pancreatic exocrine secretions and may be used therapeutically to treat pancreatic fistulae.
Somatostatinomas are rare pancreatic endocrine tumours and will result in the clinical manifestations of diabetes mellitus,
gallstones and steatorrhoea.

148.A 34 year old lady develops septic shock and features of the systemic inflammatory response syndrome as a
complication of cholangitis. Which of the following is not a typical feature of this condition?
A. Body temperature less than 36oC or greater than 38oC
B. Respiratory rate >20
C. Lactate <4 mmol/L
D. High levels of tumour necrosis factor α
E. WCC >12,000 mm3
Answer: C

Septic shock will typically result in end organ hypoperfusion and as a result lactate levels will often be high. In the surviving
sepsis campaign it is suggested that elevated lactate levels are an independent indicator for vasopressor support in patient with
sepsis. The WCC may be paradoxically low in severe sepsis, although it is most often elevated.

149.Which of the following stimulates gastric acid secretion?


A. Cholecystokinin
B. Gastric inhibitory peptide
C. Secretin
D. Histamine
E. Somatostatin
Answer: D
Gastrin: From G cells: stimulates gastric acid production. Pepsin: Digestion of protein, secretion occurs simultaneously with
gastrin. Secretin: From mucosal cells in the duodenum and jejunum: inhibits gastric acid, stimulates bile and pancreatic juice
production. Gastric inhibitory peptide: (produced in response to fatty acids) inhibits gastrin release and acid secretion from
parietal cells. Cholecystokinin: From mucosal cells in the duodenum and jejunum (produced in response to fatty acids) inhibits
acid secretion from parietal cells, causes gallbladder contraction and relaxation of sphincter of Oddi. Somatostatin: From D cells

150.Which of the following statements relating to gastric acid secretions are untrue?
A. In parietal cells carbonic anhydrase generates hydrogen ions which are then actively secreted
B. The cephalic phase is abolished following truncal vagotomy
C. The intestinal phase accounts for 60% of gastric acid produced
D. Histamine acts in a paracrine manner on H2 receptors
E. H2 receptor antagonists will not completely abolish gastric acid production
Answer: C

The intestinal phase of gastric acid secretion accounts for only 10% of gastric acid produced.

151.A 22 year old man is undergoing a daycase excision of a sebaceous cyst. He is needle phobic and as the surgeon
approaches with the needle the patient begins to hyperventilate. He soon develops circumoral parasthesia and
muscular twitching. Which of the following is the most likely explanation for this event?
A. Temporal lobe epilepsy
B. Reduction in ionised calcium levels
C. Increase in ionised calcium levels
D. Fall in serum PTH levels
E. Rise in serum PTH levels
Answer: B
50% of plasma calcium is ionised. Hyperventilation will induce a state of alkalosis which will lower ionised plasma calcium
levels.

152.Which of the following inhibits gastric acid secretion?


A. Histamine
B. Nausea
C. Calcium
D. Parasympathetic vagal stimulation

173
E. Gastrin
Answer: B

Nausea inhibits gastric secretion via higher cerebral activity and sympathetic innervation.

153.73 year old lady is diagnosed with hyperaldosteronism. From which of the following structures is aldosterone released?
A. Zona fasciculata of the adrenal gland
B. Juxtaglomerular apparatus of the kidney
C. Zona reticularis of the adrenal gland
D. Adrenal medulla
E. Zona glomerulosa of the adrenal cortex
Answer: E
Aldosterone serves to conserve sodium and water. It is produced in the zona glomerulosa of the adrenal cortex.

Adosterone is secreted by the zona glomerulosa of the adrenal cortex. It is a mineralocorticoid hormone. Secretion is regulated by
the renin- angiotensin system, and by plasma levels of sodium and potassium. Aldosterone conserves sodium by stimulating the
reabsorption of sodium in the distal nephron in exchange for potassium. Lack of aldosterone release will result in hyperkalaemia
and hyponatraemia.

154.A 43 year old lady is admitted with cholestasis secondary to a stone impacted at the level of the ampulla of vater.
Which of the following tests is most likely to be predictive of bleeding diathesis at the time of ERCP in this particular
case?
A. Bleeding time
B. Prothrombin time
C. APTT
D. Platelet count
E. Factor I levels
Answer: B
PT: Vitamin K dependent factors 2, 7, 9, 10. APTT: Factors 8, 9, 11, 12

Jaundice will impair the production of vitamin K dependent clotting factors. This is most accurately tested by measuring the
prothrombin time. APTT can be affected by vitamin K deficiency (due to factor 9 deficiency), however this occurs to a lesser
extent and is normally associated with severe liver disease. The bleeding time is a measure of platelet function.

155.Which of the following mechanisms best accounts for the release of adrenaline?
A. Release from the adrenal medulla in response to increased angiotensin 1 levels
B. Release from the zona fasiculata from the adrenal gland in response to increased sympathetic discharge
C. Release from the adrenal medulla in response to increased noradrenaline levels
D. Release from the adrenal medulla in response to sympathetic stimulation from the splanchnic nerves
E. None of the above
Answer: D
The adrenal gland releases adrenaline in response to increased sympathetic discharge from preganglionic sympathetic fibres of the
splanchnic nerves. These cause the chromafin cells of the medulla to release adrenaline (which is preformed) by exocytosis.

156.The acute phase response to injury does not include:


A. Pyrexia
B. Decreased albumin
C. Hepatic sequestration of cations
D. Increased transferrin
E. Increased serum amyloid A
Answer: D

The acute phase response includes: Acute phase protein.Reduction of transport proteins (albumin, transferrin); Hepatic
sequestration cations; Pyrexia; Neutrophil leukocytosis; Increased muscle proteolysis; Changes in vascular permeability

157. Which of the following statements relating to blood transfusions in surgical patients is false?
A. Packed red cells typically have a haematocrit of between 55 and 75%
174
B. Clotting factor activity in whole blood decreases in samples stored for longer than 7 days
C. After 3 weeks of storage blood has a pH of 6.9
D. Gamma irradiated blood products are not required routinely
E. Patients should be transfused to achieve a target haemoglobin of 10 g/dl and a haematocrit of 30%
Answer: E
Patients can generally be managed without transfusion as long as the Hb is 7 or greater. The exact level depends upon patient
factors such as co-morbidities. Old blood functions less effectively and should not be used during massive transfusions.

Whole blood fractions

Packed red cells: Used for transfusion in chronic anaemia and cases where infusion of large volumes of fluid may result in
cardiovascular compromise. Product obtained by centrifugation of whole blood. Platelet rich plasma: Usually administered to
patients who are thrombocytopaenic and are bleeding or require surgery. It is obtained by low speed centrifugation. Platelet
concentrate: Prepared by high speed centrifugation and administered to patients with thrombocytopaenia. Fresh frozen plasma:
Prepared from single units of blood. Contains clotting factors, albumin and immunoglobulin.Unit is usually 200 to 250ml. Usually
used in correcting clotting deficiencies in patients with hepatic synthetic failure who are due to undergo surgery. Usual dose is 12-
15ml/Kg-1. It should not be used as first line therapy for hypovolaemia. Cryoprecipitate: Formed from supernatant of FFP. Rich
source of Factor VIII and fibrinogen. Allows large concentration of factor VIII to be administered in small volume. SAG-
Mannitol Blood: Removal of all plasma from a blood unit and substitution with: Sodium chloride, Adenine, Anhydrous glucose,
Mannitol. Up to 4 units of SAG M Blood may be administered. Thereafter whole blood is preferred. After 8 units, clotting factors
and platelets should be considered.

Cell saver devices


These collect patients own blood lost during surgery and then reinfuse it. There are two main types: Those which wash the blood
cells prior to reinfusion. These are more expensive to purchase and more complicated to operate. However, they reduce the risk
of reinfusing contaminated blood back into the patient. Those which do not wash the blood prior to reinfusion.
Their main advantage is that they avoid the use of infusion of blood from donors into patients and this may reduce risk of blood
borne infection. It may be acceptable to Jehovah's witnesses. It is contraindicated in malignant disease for risk of facilitating
disease dissemination.

Blood products used in warfarin reversal


In some surgical patients the use of warfarin can pose specific problems and may require the use of specialised blood products
Immediate or urgent surgery in patients taking warfarin (1) (2):
1. Stop warfarin. 2. Vitamin K (reversal within 4-24 hours): IV takes 4-6h to work (at least 5mg). Oral can take 24 hours to be
clinically effective. 3. Fresh frozen plasma: Used less commonly now as 1st line warfarin reversal: 30ml/kg-1. Need to give at
least 1L fluid in 70kg person (therefore not appropriate in fluid overload). Need blood group. Only use if human prothrombin
complex is not available. 4. Human Prothrombin Complex (reversal within 1 hour): Bereplex 50 u/kg. Rapid action but factor 6
short half life, therefore give with vitamin K

158.Which of the following statements relating the fluid physiology of a physiologically normal 70 Kg adult male is
false?
A. He will have more water per unit of body weight than a female of similar weight
B. Plasma will comprise 25% of his body weight
C. Interstitial fluid will account for up to 14% of body weight
D. Approximately 65% of total body water is intracellular
E. 60% of his body weight is composed of water
Answer: B
The 60-40-20 rule: 60% total body weight is water; 40% of total body weight is intracellular fluids; 20% of body weight is
extracellular fluids

Plasma typically accounts for 4-6% of body weight in healthy individuals.


Males typically have more water per unit weight than females, as females have a higher fat content.

159.A 17 year old lady with long standing anorexia nervosa is due to undergo excision of a lipoma. Which of the
following nutritional deficiencies is most likely to be implicated in poor collagen formation as the wound heals?
A. Deficiency of copper
B. Deficiency of iron
C. Deficiency of ascorbic acid
D. Deficiency of phosphate
E. None of the above

175
Answer: C
Vitamin C is involved in the cross linkage of collagen and impaired wound healing is well described in cases of vitamin C
deficiency.

160.A 45 year old man is undergoing a small bowel resection. The anaesthetist decides to administer an intravenous
fluid which is electrolyte rich. Which of the following most closely matches this requirement?
A. Dextrose / Saline
B. Pentastarch
C. Gelofusine
D. Hartmans
E. 5% Dextrose with added potassium 20mmol/ L
Answer: D
Hartmans solution is the most electrolyte rich. However, both pentastarch and gelofusine have more macromolecules.
Composition of commonly used intravenous fluids mmol-1

Na K Cl Bicarbonate Lactate
Plasma 137-147 4-5.5 95-105 22-25 -
0.9% Saline 153 - 153 - -
Dextrose / saline 30.6 - 30.6 - -
Hartmans 130 4 110 - 28

Recommendations for intra operative fluid management: Intra operative fluids are recommended to optimise cardiac stroke
volume. Patients undergoing non elective orthopaedic or abdominal surgery should receive IV fluids for the 1st 8h post
operatively. This may be supplemented by a low dose dopexamine infusion in selected cases.

161.A 16 year old girl develops pyelonephritis and is admitted in a state of septic shock. Which of the following is not
typically seen in this condition?
A. Increased cardiac output
B. Increased systemic vascular resistance
C. Oliguria may occur
D. Systemic cytokine release
E. Tachycardia
Answer: B

Septic shock:occurs when the peripheral vascular dilatation causes a fall in SVR. Similar response may occur in anaphylactic
shock, neurogenic shock

162.A man is admitted after a period of prolonged self, induced starvation. Naso gastric feeding is planned. Which of
the following is least likely to occur?
A. Hypokalaemia
B. Increased risk of cardiac arrhythmias
C. His haemoglobin will have decreased affinity for oxygen
D. Hypophosphataemia
E. None of the above
Answer: C
The process of starvation may lower DPG levels, in practice this is unlikely to occur early as it is generated during glycolysis.
Altered metabolism in starvation may be more acidotic and this would also tend to impair oxygen carriage.

163.Where does spironolactone act in the kidney?


A. Glomerulus
B. Proximal convoluted tubule
C. Descending limb of the loop of Henle
D. Ascending limb of the loop of Henle
E. Distal convoluted tubule
Answer: E

Potassium-sparing diuretics may be divided into the epithelial sodium channel blockers (amiloride and triamterene) and
aldosterone antagonists (spironolactone and eplerenone). Amiloride is a weak diuretic which blocks the epithelial sodium channel
in the distal convoluted tubule. Usually given with thiazides or loop diuretics as an alternative to potassium supplementation.
Spironolactone is an aldosterone antagonist which acts act in the distal convoluted tubule. Indications: ascites: patients with
cirrhosis develop a secondary hyperaldosteronism. Relatively large doses such as 100 or 200mg are often used; heart failure;
nephrotic syndrome; Conn's syndrome
176
164. Which receptor does noradrenaline mainly bind to?
B. α 1 receptors
C. α 2 receptors
D. β 1 receptors
E. β 2 receptors
F. G receptors
Answer: A

Noradrenaline is the precursor of adrenaline. It is a powerful α 1 stimulant (although it will increase myocardial contractility).
Infusions will produce vasoconstriction and an increase in total peripheral resistance. It is the inotrope of choice in septic shock.

165. A 47 year old lady is diagnosed as suffering from a phaeochromocytoma. From which of the following amino acids
are catecholamines primarily derived?
A. Aspartime
B. Glutamine
C. Arginine
D. Tyrosine
E. Alanine
Answer: D

Catecholamine hormones are derived from tyrosine, it is modified by a DOPA decarboxylase enzyme to become dopamine and
thereafter via two further enzymic modifications to noradrenaline and finally adrenaline.

Adrenal medulla The chromaffin cells of the adrenal medulla secrete the catecholamines noradrenaline and adrenaline. The
medulla is innervated by the splanchnic nerves; the preganglionic sympathetic fibres secrete acetylcholine causing the chromaffin
cells to secrete their contents by exocytosis.
Phaeochromocytomas are derived from these cells and will secrete both adrenaline and nor adrenaline.

Adrenal cortex: Three histologically distinct zones are recognised:

Zone Location Hormone Secreted


Zona glomerulosa Outer zone Aldosterone
Zona fasiculata Middle zone Glucocorticoids
Zona reticularis Inner zone Androgens

The glucocorticoids and aldosterone are mostly bound to plasma proteins in the circulation. Glucocorticoids are inactivated and
excreted by the liver.

165. Where are the arterial baroreceptors located?


A. Carotid sinus and aortic arch
B. Carotid sinus only
C. Superior vena cava
D. External carotid artery
E. None of the above
Answer: A

They lie in the carotid sinus and aortic arch.

166. Which one of the following cells secretes the majority of tumour necrosis factor in humans?
A. Neutrophils
B. Macrophages
C. Natural killer cells
D. Killer-T cells
E. Helper-T cells
Answer: B
Tumour necrosis factor (TNF) is a pro-inflammatory cytokine with multiple roles in the immune system. TNF is secreted
mainly by macrophages and has a number of effects on the immune system, acting mainly in a paracrine fashion: activates
macrophages and neutrophils, acts as costimulator for T cell activation, key mediator of bodies response to Gram negative
septicaemia, similar properties to IL-1, anti-tumour effect (e.g. phospholipase activation)
TNF-alpha binds to both the p55 and p75 receptor. These receptors can induce apoptosis. It also cause activation of NFkB.
Endothelial effects include increase expression of selectins and increased production of platelet activating factor, IL-1 and
177
prostaglandins. TNF promotes the proliferation of fibroblasts and their production of protease and collagenase. It is thought
fragments of receptors act as binding points in serum. Systemic effects include pyrexia, increased acute phase proteins and
disordered metabolism leading to cachexia. TNF is important in the pathogenesis of rheumatoid arthritis - TNF blockers (e.g.
infliximab, etanercept) are now licensed for treatment of severe rheumatoid

167. Which of the following is responsible for the rapid depolarisation phase of the myocardial action potential?
A. Rapid sodium influx
B. Rapid sodium efflux
C. Slow efflux of calcium
D. Efflux of potassium
E. Rapid calcium influx
Answer: A
Myocardial action potential

Phase Description Mechanism


Rapid sodium influx
0 Rapid depolarisation
These channels automatically deactivate after a few ms
1 Early repolarisation Efflux of potassium
2 Plateau Slow influx of calcium
3 Final repolarisation Efflux of potassium
Resting potential is restored by Na+/K+ ATPase
Restoration of ionic
4 There is slow entry of Na+ into the cell decreasing the potential difference until the
concentrations
threshold potential is reached, triggering a new action potential
NB cardiac muscle remains contracted 10-15 times longer than skeletal muscle

Conduction velocity
Atrial conduction Spreads along ordinary atrial myocardial fibres at 1 m/sec
AV node 0.05 m/sec
conduction
Ventricular Purkinje fibres are of large diameter and achieve velocities of 2-4 m/sec (this allows a rapid and
conduction coordinated contraction of the ventricles

168. Which of the following is not a feature of normal cerebrospinal fluid?


A. It has a pressure of between 10 and 15 mmHg.
B. It usually contains a small amount of glucose.
C. It may normally contain up to 5 red blood cells per mm3.
D. It may normally contain up to 3 white blood cells per mm3.
E. None of the above
It should not contain red blood cells.
Answer: C
169. Which of the following is not an effect of somatostatin?
A. It stimulates pancreatic acinar cells to release lipase
B. It decreases gastric acid secretion
C. It deceases gastrin release
D. It decreases pepsin secretion
E. It decreases glucagon release
Answer: A
It inhibits pancreatic enzyme secretion.
170. Which of the following is the least likely to increase acid secretion in the stomach?
A. Calcium
B. Alcohol
C. Caffeine
D. Pear
E. None of the above Answer: D
Pathology
1. A 38 year old lady presents with a recent episode of renal colic. As part of her investigations the following results are
obtained: Corrected Calcium: 3.84 mmol/l; PTH: 88pg/ml (increased). Her serum urea and electrolytes are normal.
What is the most likely diagnosis?
A. Carcinoma of the bronchus
B. Secondary hyperparathyroidism
C. Primary hyperparathyroidism
D. Tertiary hyperparathyroidism

178
E. Carcinoma of the breast

Answer: C
In this situation the most likely diagnosis is primary hyperparathyroidism. The question mentions that serum urea and
electrolytes are normal, which makes tertiary hyperparathyroidism unlikely.
In exams primary hyperparathyroidism is stereotypically seen in elderly females with an unquenchable thirst and an
inappropriately normal or raised parathyroid hormone level. It is most commonly due to a solitary adenoma
Causes of primary hyperparathyroidism: 80%: solitary adenoma; 15%: hyperplasia; 4%: multiple adenoma; 1%: carcinoma
Features - 'bones, stones, abdominal groans and psychic moans' Polydipsia, polyuria; Peptic
ulceration/constipation/pancreatitis; Bone pain/fracture; Renal stones; Depression; Hypertension. Associations; Hypertension;
Multiple endocrine neoplasia: MEN I and II
Investigations: Raised calcium, low phosphate; PTH may be raised or normal; Technetium-MIBI subtraction scan. Treatment;
Parathyroidectomy, if imaging suggests target gland then a focused approach may be used

Theme: Head and neck lumps

A. Branchial cyst
B. Cystic hygroma
C. Carotid body tumour
D. Lymphadenopathy
E. Adenolymphoma of the parotid
F. Pleomorphic adenoma of the parotid
G.Submandibular tumour
H.Thyroglossal cyst
I. Thoracic outlet syndrome
J. Submandibular gland calculus
Please select the most likely lesion to account for the clinical scenario given. Each option may be used once, more than once or
not at all.

2. A 60 year old Tibetan immigrant is referred to the surgical clinic with a painless neck swelling. On examination it is
located on the left side immediately anterior to the sternocleidomastoid muscle. There are no other abnormalities to find
on examination.
Answer: The correct answer is Carotid body tumour
Carotid body tumours typically present as painless masses. They may compress the vagus or hypoglossal nerves with symptoms
attributable to these structures. Over 90% occur spontaneously and are more common in people living at high altitude. In
familial cases up to 30% may be bilateral. Treatment is with excision.

3. A 40 year old women presents as an emergency with a painful mass underneath her right mandible. The mass has
appeared over the previous week with the pain worsening as the lump has increased in size. On examination there is a
4cm mass underneath her mandible, there is no associated lymphadenopathy.
Answer: Submandibular gland calculus
The sub mandibular gland is the most common site for salivary calculi. Patients will usually complain of pain, which is worse
on eating. When the lesion is located distally the duct may be laid open and the stone excised. Otherwise the gland will require
removal.

179
4. A 73 year old male smoker is referred to the clinic by his GP. On examination he has a 3cm soft mass immediately
anterior to his ear. It has been present for the past five years and is otherwise associated with no symptoms.
Answer: Adenolymphoma of the parotid
Warthins tumours (a.k.a. adenolymphoma) are commoner in older men (especially smokers). They are the second commonest
benign tumour of the parotid gland, they may be bilateral. They are soft and slow growing and relatively easy to excise.
Pleomorphic adenomas typically present in females aged between 40 - 60 years.

The table below gives characteristic exam question features for conditions causing neck lumps:

Reactive By far the most common cause of neck swellings. There may be a history of local infection or a
lymphadenopathy generalised viral illness
Lymphoma Rubbery, painless lymphadenopathy
The phenomenon of pain whilst drinking alcohol is very uncommon
There may be associated night sweats and splenomegaly
Thyroid swelling May be hypo-, eu- or hyperthyroid symptomatically
Moves upwards on swallowing
Thyroglossal cyst More common in patients < 20 years old
Usually midline, between the isthmus of the thyroid and the hyoid bone
Moves upwards with protrusion of the tongue
May be painful if infected
Pharyngeal pouch More common in older men
Represents a posteromedial herniation between thyropharyngeus and cricopharyngeus muscles
Usually not seen, but if large then a midline lump in the neck that gurgles on palpation
Typical symptoms are dysphagia, regurgitation, aspiration and chronic cough
Cystic hygroma A congenital lymphatic lesion (lymphangioma) typically found in the neck, classically on the left side
Most are evident at birth, around 90% present before 2 years of age
Branchial cyst An oval, mobile cystic mass that develops between the sternocleidomastoid muscle and the pharynx
Develop due to failure of obliteration of the second branchial cleft in embryonic development
Usually present in early adulthood
Cervical rib More common in adult females
Around 10% develop thoracic outlet syndrome
Carotid aneurysm Pulsatile lateral neck mass which doesn't move on swallowing

5. A 12 year old child is admitted with a 12 hour history of colicky right upper quadrant pain. On examination the child is
afebrile and is jaundiced. The abdomen is soft and non tender at the time of examination. What is the most likely cause?
A. Infectious hepatitis
B. Acute cholecystitis
C. Cholangitis
D. Hereditary spherocytosis
E. Gilberts syndrome

Answer: D
The child is most likely to have hereditary spherocytosis. In these individuals there may be disease flares precipitated by acute
illness. They form small pigment stones. These may cause biliary colic and some may require cholecystectomy.

Hereditary Spherocytosis: Most common disorder of the red cell membrane, it has an incidence of 1 in 5000. The abnormally
shaped erythrocytes are prone to splenic sequestration and destruction. This can result in hyperbilirubinaemia, jaundice and
splenomegaly. In older patients an intercurrent illness may increase the rate of red cell destruction resulting in more acute
symptoms.
Severe cases may benefit from splenectomy.

180
6. A 2 day old baby is noted to have voiding difficulties and on closer inspection is noted to have hypospadias. Which of the
following abnormalities is most commonly associated with the condition?
A. Cryptorchidism
B. Diaphragmatic hernia
C. Ventricular - septal defect
D. Bronchogenic cyst
E. Atrial septal defect

Answer: A
Hypospadias most commonly occurs as an isolated disorder. Associated urological abnormalities may be seen in up to 40% of
infants, of these cryptorchidism is the most frequent (10%).
Hypospadias: The urethral meatus opens on the ventral surface of the penis. There is also a ventral deficiency of the foreskin.
The uretral meatus may open more proximally in the more severe variants. However, 75% of the openings are distally located.
The incidence is 1 in 300 male births.

Features include: Absent frenular artery; Ventrally opened glans; Skin tethering to hypoplastic urethra; Splayed columns of
spongiosum tissue distal to the meatus; Deficiency of the foreskin ventrally
Management: No routine cultural circumcisions; Urethroplasty; Penile reconstruction. The foreskin is often utilised in the
reconstructive process. In boys with very distal disease no treatment may be needed.

Theme: Liver lesions

A. Cystadenoma
B. Hyatid cyst
C. Amoebic abscess
D. Mesenchymal hamartoma
E. Liver cell adenoma
F. Cavernous haemangioma

Please select the most likely lesion for the scenario given. Each option may be used once, more than once or not at all.

7. A 38 year old lady presents with right upper quadrant pain and nausea. She is otherwise well and her only medical
therapy is the oral contraceptive pill which she has taken for many years with no ill effects. Her liver function tests are
normal. An ultrasound examination demonstrates a hyperechoic well defined lesion in the left lobe of the liver which
measures 14 cm in diameter.
Answer: Cavernous haemangioma
Cavernous haemangioma often presents with vague symptoms and signs. They may grow to considerable size. Liver function
tests are usually normal. The lesions are typically well defined and hyperechoic on ultrasound. A causative link between OCP
use and haemangiomata has yet to be established, but is possible.

8. A 37 year old lady presents with right upper quadrant pain and nausea. She is otherwise well and her only medical
therapy is the oral contraceptive pill which she has taken for many years with no ill effects. Her liver function tests and
serum alpha feto protein are normal. An ultrasound examination demonstrates a 4cm non encapsulated lesion in the
right lobe of the liver which has a mixed echoity and heterogeneous texture.
Answer: Liver cell adenoma
Liver cell adenomas are linked to OCP use and 90% of patients with liver cell adenomas have used the OCP. Liver function
tests are often normal. The lesions will typically have a mixed echoity and heterogeneous texture.

9. A 38 year old shepherd presents to the clinic with a 3 month history of malaise and right upper quadrant pain. On
examination he is mildly jaundiced. His liver function tests demonstrate a mild elevation in bilirubin and transaminases,
his full blood count shows an elevated eosinophil level. An abdominal x-ray is performed by the senior house officer and
demonstrates a calcified lesion in the right upper quadrant of the abdomen.
Answer: Hyatid cyst

181
Hyatid disease is more common in those who work with sheep or dogs. Liver function tests may be abnormal and an
eosinophilia is often present. Plain radiographs may reveal a calcified cyst wall.

Benign liver lesions


Haemangioma: Most common benign tumours of mesenchymal origin. Incidence in autopsy series is 8%. Cavernous
haemangiomas may be enormous. Clinically they are reddish purple hypervascular lesions. Lesions are normally separated from
normal liver by ring of fibrous tissue. On ultrasound they are typically hyperechoic
Liver cell adenoma: 90% develop in women in their third to fifth decade. Linked to use of oral contraceptive pill. Lesions are
usually solitary. They are usually sharply demarcated from normal liver although they usually lack a fibrous capsule. On
ultrasound the appearances are of mixed echoity and heterogeneous texture. On CT most lesions are hypodense when imaged
prior to administration of IV contrast agents. In patients with haemorrhage or symptoms removal of the adenoma may be
required
Mesenchymal hamartomas: Congential and benign, usually present in infants. May compress normal liver
Liver abscess: Biliary sepsis is a major predisposing factor. Structures drained by the portal venous system form the second
largest source. Common symptoms include fever, right upper quadrant pain. Jaundice may be seen in 50. Ultrasound will
usually show a fluid filled cavity, hyperechoic walls may be seen in chronic abscesses
Amoebic abscess: Liver abscess is the most common extra intestinal manifestation of amoebiasis. Between 75 and 90% lesions
occur in the right lobe. Presenting complaints typically include fever and right upper quadrant pain. Ultrasonography will
usually show a fluid filled structure with poorly defined boundaries. Aspiration yield sterile odourless fluid which has an
anchovy paste consistency. Treatment is with metronidazole
Hyatid cysts: Seen in cases of Echinococcus infection. Typically an intense fibrotic reaction occurs around sites of infection.
The cyst has no epithelial lining. Cysts are commonly unilocular and may grow to 20cm in size. The cyst wall is thick and has
an external laminated hilar membrane and an internal enucleated germinal layer. Typically presents with malaise and right upper
quadrant pain. Secondary bacterial infection occurs in 10%. Liver function tests are usually abnormal and eosinophilia is present
in 33% cases. Ultrasound may show septa and hyatid sand or daughter cysts. Percutaneous aspiration is contra indicate
Treatment is by sterilisation of the cyst with mebendazole and may be followed by surgical resection. Hypertonic swabs are
packed around the cysts during surgery
Polycystic liver disease: Usually occurs in association with polycystic kidney disease. Autosomal dominant disorder.
Symptoms may occur as a result of capsular stretch
Cystadenoma: Rare lesions with malignant potential. Usually solitary multiloculated lesions. Liver function tests usually
normal. Ultrasonography typically shows a large anechoic, fluid filled area with irregular margins. Internal echos may result
from septa. Surgical resection is indicated in all cases

10. A 72 year old man presents with symptoms and signs of benign prostatic hyperplasia. Which of the following structures
is most likely to be enlarged on digital rectal examination?
A. Posterior lobe of the prostate
B. Median lobe of the prostate
C. Right lateral lobe of the prostate
D. Left lateral lobe of the prostate
E. Anterior lobe of the prostate
Answer: B
Carcinoma of the prostate typically occurs in the posterior lobe. The median lobe is usually enlarged in BPH. The anterior lobe
has little in the way of glandular tissue and is seldom enlarged.

Benign Prostatic Hyperplasia: Prostatic enlargement occurs in many elderly men . >90% of men aged over 80 will have at
least microscopic evidence of benign prostatic hyperplasia. Pathology: As part of the hyperplastic process increase in both
stromal and glandular components are seen. The changes are most notable in the central and periurethral region of the gland.
Presentation: The vast majority of men will present with lower urinary tract symptoms. These will typically be: Poor flow.
Nocturia. Hesitancy. Incomplete and double voiding. Terminal dribbling. Urgency. Incontinence. Investigation: Digital rectal
examination to assess prostatic size and morphology. Urine dipstick for infections and haematuria. Uroflowmetry (a flow rate of
>15ml/second helps to exclude BOO). Bladder pressure studies may help identify detrusor failure and whilst may not form part
of first line investigations should be included in those with atypical symptoms and prior to redo surgery. Bladder scanning to
demonstrate residual volumes. USS if high pressure chronic retention. Management: Lifestyle changes such as stopping
smoking and altering fluid intake may help those with mild symptoms. Medical therapy includes alpha blockers and 5 alpha
reductase inhibitors.

182
The former work quickly on receptor zones located at the bladder neck. Cardiovascular side effects are well documented. The
latter work on testosterone metabolising enzymes. Although they have a slower onset of action, the 5 alpha reductase inhibitors
may prevent acute urinary retention. Surgical therapy includes transurethral resection of the prostate and is the treatment of
choice in those with severe symptoms and those who fail to respond to medical therapy. More tailored bladder neck incision
procedures may be considered in those with small prostates. Retrograde ejaculation may occur following surgery. The change in
the type of irrigation solutions used has helped to minimise the TURP syndrome of electrolyte disturbances.
11. A 58 year old man has been suffering from mechanical back pain for several years. One morning he awakes from sleep
and feels a sudden onset of pain in his back radiating down his left leg. Which of the following events is most likely to
account for his symptoms?
A. Prolapse of inner annulus fibrosus
B. Prolapse of outer annulus fibrosus
C. Prolapse of nucleus pulposus
D. Rupture of the ligamentum flavum
E. None of the above

Answer: C
The symptoms would be most likely the result of intervertebral disk prolapse. In disk prolapse the nucleus pulposus is the
structure which usually herniates. Intervertebral discs: Consist of an outer annulus fibrosus and an inner nucleus pulposus.
The anulus fibrosus consists of several layers of fibrocartilage. The nucleus pulposus contains loose fibres suspended in a
mucoprotein gel with the consistency of jelly. The nucleus of the disc acts as a shock absorber. Pressure on the disc causes
posterior protrusion of the nucleus pulposus. Most commonly in the lumbrosacral and lower cervical areas. The discs are
separated by hyaline cartilage. There is one disc between each pair of vertebrae, except for C1/2 and the sacrococcygeal
vertebrae.

Theme: Paediatric neck masses

A. Cystic hygroma
B. Thyroglossal cyst
C. Rhabdomyosarcoma
D. Branchial cyst
E. Dermoid cyst

Please select the most likely underlying diagnosis for the situation that is described. Each option may be used once, more than
once, or not at all.

12. A 2 year old boy is brought to the clinic by his mother who has noticed that he has developed a small mass. On
examination a small smooth cyst is identified which is located above the hyoid bone. On ultrasound the lesion appears to
be a heterogenous and multiloculated mass.
Answer: Dermoid cyst
Dermoid cysts are usually multiloculated and heterogeneous. Most are located above the hyoid and their appearances on
imaging differentiate them from thyroglossal cysts.

13. A 22 month old baby is brought to the clinic by her mother who is concerned that she has developed a swelling in her
neck. On examination she has a soft, lesion located in the posterior triangle that transilluminates.
Answer: Cystic hygroma
Cystic hygromas are soft and transilluminate. Most are located in the posterior triangle.

14. A 3 year old boy is brought to the clinic by his mother who has noticed a mass in his neck. On examination he has a
smooth mass located on the lateral aspect of his anterior triangle, near to the angle of the mandible. On ultrasound it has
a fluid filled, anechoic, appearance.
Answer: The correct answer is Branchial cyst
Branchial cysts are usually located laterally and derived from the second branchial cleft. Unless infection has occurred they will
usually have an anechoic appearance on ultrasound.

183
Neck Masses in Children
Thyroglossal cyst: Located in the anterior triangle, usually in the midline and below the hyoid (65% cases). Derived from
remnants of the thyroglossal duct. Thin walled and anechoic on USS (echogenicity suggests infection of cyst)
Branchial cyst: Six branchial arches separated by branchial clefts. Incomplete obliteration of the branchial apparatus may result
in cysts, sinuses or fistulae. 75% of branchial cysts originate from the second branchial cleft. Usually located anterior to the
sternocleidomastoid near the angle of the mandible. Unless infected the fluid of the cyst has a similar consistency to water and
is anechoic on USS
Dermoids: Derived from pleuripotent stem cells and are located in the midline. Most commonly in a suprahyoid location. They
have heterogeneous appearances on imaging and contain variable amounts of calcium and fat
Thyroid gland: True thyroid lesions are rare in children and usually represent thyroglossal cysts or tumours like lymphoma
Lymphatic malformations: Usually located posterior to the sternocleidomastoid. Cystic hygroma result from occlusion of
lymphatic channels. The painless, fluid filled, lesions usually present prior to the age of 2. They are often closely linked to
surrounding structures and surgical removal is difficult. They are typically hypoechoic on USS
Infantile haemangioma: May present in either triangle of the neck. Grow rapidly initially and then will often spontaneously
regress. Plain x-rays will show a mass lesion, usually containing calcified phleboliths. As involution occurs the fat content of
the lesions increases
Lymphadenopathy: Located in either triangle of the neck. May be reactive or neoplastic. Generalised lymphadenopathy
usually secondary to infection in children (very common)

15. An unusually tall 43 year old lady presents to the surgical clinic with bilateral inguinal hernias. She develops chest pain
and collapses. As part of her investigations a chest x-ray shows evidence of mediastinal widening. What is the most likely
underlying diagnosis?
A. Pulmonary embolus
B. Aortic dissection
C. Tietze syndrome
D. Boerhaaves syndrome
E. Myocardial infarct
Answer:
B
Marfans syndrome may present with a variety of connective tissue disorders such as bilateral inguinal hernia. They are at high
risk of aortic dissection, as in this case. Aortic dissection: More common than rupture of the abdominal aorta. 33% of patients
die within the first 24 hours, and 50% die within 48 hours if no treatment received. Associated with hypertension: Features of
aortic dissection: tear in the intimal layer, followed by formation and propagation of a subintimal hematoma. Cystic medial
necrosis (Marfan's). Most common site of dissection: 90% occurring within 10 centimetres of the aortic valve

Stanford Classification: Type A: Ascending aorta/ aortic root: Surgery- aortic root replacement. Type: B: Descending aorta:
Medical therapy with antihypertensives. DeBakey classification: Type I:Ascending aorta, aortic arch, descending aorta; Type
II: Ascending aorta only; Type III: Descending aorta distal to left subclavian artery. Clinical features: Tearing, sudden onset
chest pain (painless 10%). Hypertension or Hypotension. A blood pressure difference greater than 20 mm Hg. Neurologic
deficits (20%)
Investigations: CXR: widened mediastinum, abnormal aortic knob, ring sign, deviation trachea/oesophagus. CT (spiral). MRI.
Angiography (95% of patients diagnosed). Management: Beta-blockers: aim HR 60- bpm and systolic BP 100-120 mm Hg.
Urgent surgical intervention: type A dissections. This will usually involve aortic root replacement.

16. A 72 year old man has just undergone an emergency repair for a ruptured abdominal aortic aneurysm. Pre operatively
he was taking aspirin, clopidogrel and warfarin. Intra operatively he received 5000 units of unfractionated heparin prior
to application of the aortic cross clamp. His blood results on admission to the critical care unit are as follows:
Full blood count: Hb: 8 g/dl; Platelets: 40 * 109/l; WBC: 7.1 * 109/l. His fibrin degradation products are measured and
found to be markedly elevated. Which of the following accounts for these results?
A. Anastomotic leak
B. Disseminated intravascular coagulation
C. Heparin induced thrombocytopenia
D. Adverse effect of warfarin
E. Adverse effects of antiplatelet agents

Answer: B
The combination of low platelet counts and raised FDP in this setting maked DIC the most likely diagnosis.

Disseminated intravascular coagulation – Diagnosis: Under homeostatic conditions, coagulation and fibrinolysis are coupled.
The activation of the coagulation cascade yields thrombin that converts fibrinogen to fibrin; the stable fibrin clot being the final
product of hemostasis. The fibrinolytic system breaks down fibrinogen and fibrin. Activation of the fibrinolytic system

184
generates plasmin (in the presence of thrombin), which is responsible for the lysis of fibrin clots. The breakdown of fibrinogen
and fibrin results in polypeptides (fibrin degradation products). In a state of homeostasis, the presence of plasmin is critical, as it
is the central proteolytic enzyme of coagulation and is also necessary for fibrinolysis.

In DIC, the processes of coagulation and fibrinolysis are dysregulated, and the result is widespread clotting with resultant
bleeding. Regardless of the triggering event of DIC, once initiated, the pathophysiology of DIC is similar in all conditions. One
critical mediator of DIC is the release of a transmembrane glycoprotein (tissue factor =TF). TF is present on the surface of many
cell types (including endothelial cells, macrophages, and monocytes) and is not normally in contact with the general circulation,
but is exposed to the circulation after vascular damage. For example, TF is released in response to exposure to cytokines
(particularly interleukin 1), tumor necrosis factor, and endotoxin. This plays a major role in the development of DIC in septic
conditions. TF is also abundant in tissues of the lungs, brain, and placenta. This helps to explain why DIC readily develops in
patients with extensive trauma. Upon activation, TF binds with coagulation factors that then triggers the extrinsic pathway (via
Factor VII) which subsequently triggers the intrinsic pathway (XII to XI to IX) of coagulation.

Fibrin degradation products are often raised.


Disorder Prothrombin time APTT Bleeding time Platelet count
Warfarin administration Prolonged Normal Normal Normal
Aspirin administration Normal Normal Prolonged Normal
Heparin Often normal (may be prolonged) Prolonged Normal Normal
DIC Prolonged Prolonged Prolonged Low

17. A 53 year old man from Hong Kong presents with symptoms of fatigue, weight loss and recurrent epistaxis. Clinical
examination reveals left sided cervical lymphadenopathy and oropharyngeal examination reveals an ulcerated mass in
the naso pharynx. Which of the following viral agents is most commonly implicated in the development of this condition?
A. Cytomegalovirus
B. Epstein Barr virus
C. Coxsackie virus
D. Herpes simplex virus
E. None of the above

Answer: B
The clinical scenario is most typical for nasopharyngeal carcinoma. An association with previous Epstein Barr Virus is well
established. Infection with the other viruses listed is not a recognised risk factor for the development of the
condition.Nasopharyngeal carcinoma: Squamous cell carcinoma of the nasopharynx. Rare in most parts of the world, apart
from individuals from Southern ChinaAssociated with Epstein Barr virus infection. Presenting features: Systemic: Cervical
lymphadenopathy. Local: Otalgia, Unilateral serous otitis media, Nasal obstruction, discharge and/ or epistaxis, Cranial nerve
palsies e.g. III-VI. Imaging: Combined CT and MRI. Treatment: Radiotherapy is first line therapy.

18. An 18 year old male presents with lethargy, night sweats and on examination is found to have left supraclavicular
lymphadenopathy. A surgical registrar performs a left supraclavicular lymph node biopsy. The pathologist identifies
Reed- Sternberg cells on the subsequent histology sections, what is the most likely diagnosis?
A. Metastatic gastric cancer
B. Hodgkins lymphoma
C. Non Hodgkins lymphoma
D. Tuberculosis
E. None of the above

Answer: B
Reed-Sternberg cells are characteristic histological cell type found in Hodgkins disease. Lymphadenopathy:
Lymphadenopathy in the neck, axillae, groins and abdomen. Need to note: solitary/multiple, defined/indistinct,
hard/rubbery/soft, tender/painless

Causes of lymphadenopathy: Mnemonic: Hodgkins disease: H aematological: Hodgkins lymphoma, NHL, Leukaemia; O
ncological: metastases; D ermatopathic lympadenitis; G aucher's disease; K awasaki disease; I nfections: TB, glandular fever,
Syphilis; N iemann Pick disease; S erum sickness; D rug reaction (phenytoin); I mmunological (SLE); S arcoidosis; E
ndocrinological (Hyperthyroidism); A ngioimmunoplastic lymphadenopathy; S LE; E osinophilic granulomatosis

19. Which of the following lesions is least likely to occur in the presence of severe atrophic gastritis?
A. Duodenal ulcer
B. Gastric cancer
C. Gastric polyp
D. Iron deficiency anaemia
185
E. Pernicious anaemia

Answer: A
Due the absence of acid a duodenal ulcer is unlikely to occur.

Gastric cancer: Overview: There are 700,000 new cases of gastric cancer worldwide each year. It is most common in Japan
and less common in western countries. It is more common in men and incidence rises with increasing age. The exact cause of
many sporadic cancer is not known, however, familial cases do occur in HNPCC families. In addition, smoking and smoked or
preserved foods increase the risk. Japanese migrants retain their increased risk (decreased in subsequent generations). The
distribution of the disease in western countries is changing towards a more proximal location (perhaps due to rising obesity).

Pathology: There is some evidence of support a stepwise progression of the disease through intestinal metaplasia progressing to
atrophic gastritis and subsequent dysplasia, through to cancer. The favoured staging system is TNM. The risk of lymph node
involvement is related to size and depth of invasion; early cancers confined to submucosa have a 20% incidence of lymph node
metastasis. Tumours of the gastro-oesophageal junction are classified: Type 1: True oesophageal cancers and may be associated
with Barrett's oesophagus. Type 2: Carcinoma of the cardia, arising from cardiac type epithelium or short segments with
intestinal metaplasia at the oesophagogastric junction. Type 3: Sub cardial cancers that spread across the junction. Involve
similar nodal stations to gastric cancer. Groups for close endoscopic monitoring: Intestinal metaplasia of columnar type.
Atrophic gastritis. Low to medium grade dysplasia. Patients who have previously undergone resections for benign peptic ulcer
disease (except highly selective vagotomy).

Table: Referral to endoscopy

Patients of any age with dyspepsia and Patients without dyspepsia Worsening dyspepsia
any of the following
Chronic gastrointestinal bleeding Dysphagia Barretts oesophagus
Dysphagia Unexplained abdominal pain Intestinal metaplasia
or weight loss
Weight loss Vomiting Dysplasia
Iron deficiency anaemia Upper abdominal mass Atrophic gastritis
Upper abdominal mass Jaundice Patient aged over 55 years with unexplained
or persistent dyspepsia

Staging: CT scanning of the chest abdomen and pelvis is the routine first line staging investigation in most centres.
Laparoscopy to identify occult peritoneal disease. PET CT (particularly for junctional tumours). Treatment: Proximally sited
disease greater than 5-10cm from the OG junction may be treated by sub total gastrectomy. Total gastrectomy if tumour is <5cm
from OG junction. For type 2 junctional tumours (extending into oesophagus) oesophagogastrectomy is usual. Endoscopic sub
mucosal resection may play a role in early gastric cancer confined to the mucosa and perhaps the sub mucosa (this is debated).
Lymphadenectomy should be performed. A D2 lymphadenectomy is widely advocated by the Japanese, the survival advantages
of extended lymphadenectomy have been debated. However, the overall recommendation is that a D2 nodal dissection be
undertaken. Most patients will receive chemotherapy either pre or post operatively. Prognsis: (UK Data) by percentage 5 year
survival: All RO resections: 54%; Early gastric cancer: 91%; Stage 1: 87%; Stage 2: 65%; Stage 3: 18%
Operative procedure: Total Gastrectomy , lymphadenectomy and Roux en Y anastomosis: General anaesthesia.
Prophylactic intravenous antibiotics. Incision: Rooftop. Perform a thorough laparotomy to identify any occult disease. Mobilise
the left lobe of the liver off the diaphragm and place a large pack over it. Insert a large self retaining retractor e.g. omnitract or
Balfour (take time with this, the set up should be perfect). Pack the small bowel away. Begin by mobilising the omentum off the
transverse colon. Proceed to detach the short gastric vessels. Mobilise the pylorus and divide it at least 2cm distally using a
linear cutter stapling device. Continue the dissection into the lesser sac taking the lesser omentum and left gastric artery flush at
its origin.
The lymph nodes should be removed en bloc with the specimen where possible.
Place 2 stay sutures either side of the distal oesophagus. Ask the anaesthetist to pull back on the nasogastric tube. Divide the
distal oesophagus and remove the stomach. The oesphago jejunal anastomosis should be constructed. Identify the DJ flexure
and bring a loop of jejunum up to the oesophagus (to check it will reach). Divide the jejunum at this point. Bring the divided
jejunum either retrocolic or antecolic to the oesophagus. Anastamose the oesophagus to the jejunum, using either interrupted 3/0
vicryl or a stapling device. Then create the remainder of the Roux en Y reconstruction distally.
Place a jejunostomy feeding tube. Wash out the abdomen and insert drains (usually the anastomosis and duodenal stump). Help
the anaesthetist insert the nasogastric tube (carefully!). Close the abdomen and skin. Enteral feeding may commence on the first
post-operative day. However, most surgeons will leave patients on free NG drainage for several days and keep them nil by
mouth.

20. A 28 year old man develops an acute paronychia and subsequent spreading sepsis. The tissue exudate has a higher
protein content than normal tissue because?

186
A. Breakdown of tissue cells release protein
B. Capillary walls are more permeable
C. Increased blood flow transports more protein into the area
D. Intracapillary pressure is raised
E. Plasma cells release gamma globulin

Answer: B
The increased permeability allows the exudation of plasma proteins.

Acute inflammation: Inflammation is the reaction of the tissue elements to injury. Vascular changes occur, resulting in the
generation of a protein rich exudate. So long as the injury does not totally destroy the existing tissue architecture, the episode
may resolve with restoration of original tissue architecture.
Vascular changes: Vasodilation occurs and persists throughout the inflammatory phase. Inflammatory cells exit the circulation
at the site of injury. The equilibrium that balances Starlings forces within capillary beds is disrupted and a protein rich exudate
will form as the vessel walls also become more permeable to proteins. The high fibrinogen content of the fluid may form a
fibrin clot. This has several important immunomodulatory functions.

Sequelae: Resolution: Typically occurs with minimal initial injury. Stimulus removed and normal tissue architecture results.
Organisation:Delayed removed of exudate. Tissues undergo organisation and usually fibrosis. Suppuration:Typically
formation of an abscess or an empyema. Sequestration of large quantities of dead neutrophils. Progression to chronic
inflammation: Coupled inflammatory and reparative activities. Usually occurs when initial infection or suppuration has been
inadequately managed
Causes: Microbacterial infections e.g. Viruses, exotoxins or endotoxins released by bacteria. Chemical agents. Physical agents
e.g. Trauma. Hypersensitivity reactions. Tissue necrosis. Presence of neutrophil polymorphs is a histological diagnostic
feature of acute inflammation

21. As a busy surgical trainee on the colorectal unit you are given the unenviable task of reviewing the unit's histopathology
results for colonic polyps. Which of the polyp types described below has the greatest risk of malignancy?
A. Hyperplastic polyp
B. Tubular adenoma
C. Villous adenoma
D. Hamartomatous polyp
E. Serrated polyp

Answer: C
Villous adenomas carry the highest risk of malignant transformation. Hyperplastic polyps carry little in the way of increased
risk. Although, patients with hamartomatous polyp syndromes may have a high risk of malignancy, the polyps themselves have
little malignant potential.

Colonic Polyps: May occur in isolation of greater numbers as part of the polyposis syndromes. In FAP greater than 100 polyps
are typically present. The risk of malignancy in association with adenomas is related to size and is the order of 10% in a 1cm
adenoma. Isolated adenomas seldom give risk of symptoms (unless large and distal). Distally sited villous lesions may produce
mucous and if very large electrolyte disturbances may occur.
Follow up of colonic polyps: Low risk: 1 or 2 adenomas <1cm. No follow up or re-colonoscopy at 5 years. Moderate risk: 3
or 4 small adenomas or 1 adenoma >1cm. Re-scope at 3 years. High risk: >5 small adenomas or >3 with 1 of them >1cm. Re
scope at 1 year. It is important to stratify patients appropriately and ensure that a complete colonoscopy with good views was
performed.
Segmental resection or complete colectomy should be considered when: 1. Incomplete excision of malignant polyp; 2.
Malignant sessile polyp; 3. Malignant pedunculated polyp with submucosal invasion; 4. Polyps with poorly differentiated
carcinoma; 5. Familial polyposis coli. Screening from teenager up to 40 years by 2 yearly sigmoidoscopy/colonoscopy.
Panproctocolectomy and Ileostomy or Restorative Panproctocolectomy. Rectal polypoidal lesions may be amenable to trans
anal endoscopic microsurgery.

22. A 23 year old man presents to the surgical clinic with an inguinal hernia. On examination he has a small direct hernia.
However, you also notice that he has pigmented spots around his mouth, on his palms and soles. In his history he
underwent a reduction of an intussusception aged 12 years. Which of the following lesions is most likely to be identified
if a colonoscopy were performed?
A. Hamartomas
B. Tubulovillous adenoma
C. Colorectal cancer
D. Crohns disease

187
E. Hyperplastic polyps

Answer: A
He is most likely to have Peutz-Jeghers syndrome which is associated with Hamartomas.

Peutz-Jeghers syndrome: Peutz-Jeghers syndrome is an autosomal dominant condition characterised by numerous benign
hamartomatous polyps in the gastrointestinal tract. It is also associated with pigmented freckles on the lips, face, palms and
soles. Around 50% of patients will have died from a gastrointestinal tract cancer by the age of 60 years.
Genetics: Autosomal dominant. Responsible gene encodes serine threonine kinase LKB1 or STK11. Features: Hamartomatous
polyps in GI tract (mainly small bowel). Pigmented lesions on lips, oral mucosa, face, palms and soles. Intestinal obstruction
e.g. intussusception (which may lead to diagnosis). Gastrointestinal bleeding. Management: Conservative unless complications
develop

23. A 56 year old surgeon has been successfully operating for many years. Over the past few weeks she has begun to notice
that her hands are becoming blistering and weepy. A latex allergy is diagnosed. Which of the following pathological
processes accounts for this scenario?
A. Type 1 hypersensitivity reaction
B. Type 2 hypersensitivity reaction
C. Type 4 hypersensitivity reaction
D. Type 3 hypersensitivity reaction
E. None of the above
Answer:
C
Hypersensitivity reactions: ACID: type 1 –Anaphylactic; type 2 –Cytotoxic; type 3 --Immune complex type 4 --Delayed
hypersensitivity
Contact dermatitis of a chronic nature is an example of a type 4 hypersensitivity reaction. Type 4 hypersensitivity reactions are
cell mediated rather than antibody mediated.

Hypersensitivity reactions: The Gell and Coombs classification divides hypersensitivity reactions into 4 types

Type I Type II Type III Type IV


Description Anaphylactic Cytotoxic Immune complex Delayed type
Mediator IgE IgG, IgM IgG, IgM T-cells
Antigen Exogenous Cell surface Soluble Tissues
Response time Minutes Hours Hours 2-3 days
Examples Asthma Autoimmune haemolytic anaemia Serum sickness Graft versus host disease
Hay fever Pemphigus SLE Contact dermatitis
Goodpasture's Aspergillosis

24. A 56 year old motorcyclist is involved in a road traffic accident and sustains a displaced femoral shaft fracture. Not
other injuries are identified on the primary or secondary surveys. The fracture is treated with closed, antegrade
intramedullary nailing. The following day the patient becomes increasingly agitated and confused. On examination he is
pyrexial, hypoxic SaO2 90% on 6 litres O2, tachycardic and normotensive. Systemic examination demonstrates a non
blanching petechial rash present over the torso. What is the most likely explanation for this?
A. Pulmonary embolism with paradoxical embolus
B. Fat embolism
C. Meningococcal sepsis
D. Alcohol withdrawl
E. Chronic sub dural haematoma

Answer: B
This man has a recent injury and physical signs that would be concordant with fat embolism syndrome. Meningococcal sepsis is
not usually associated with hypoxia initially. Pulmonary emboli are not typically associated with pyrexia. Fat embolism:
Cardiothoracic:Early persistent tachycardia. Tachypnoea, dyspnoea, hypoxia usually 72 hours following injury. Pyrexia.
Dermatological: Red/ brown impalpable petechial rash (usually only in 25-50%). Subconjunctival and oral haemorrhage/
petechiae. CNS: Confusion and agitation. Retinal haemorrhages and intra-arterial fat globules on fundoscopy. Imaging: May be
normal. Fat emboli tend to lodge distally and therefore CTPA may not show any vascular occlusion, a ground glass appearance
may be seen at the periphery. Treatment: Prompt fixation of long bone fractures. Some debate regarding benefit Vs. risk of
medullary reaming in femoral shaft/ tibial fractures in terms of increasing risk (probably does not). DVT prophylaxis. General
supportive care

25. Which of these tumour markers is most helpful in identifying an individual with hepatocellular carcinoma?
A. Serum AFP
188
B. Serum CA19-9
C. CEA
D. Beta HCG
E. CA125

Answer: A
Hepatocellular carcinoma is commonly diagnosed with imaging and an elevated alpha fetoprotein. Biopsy may seed the tumour
and should be avoided. Up to 80% of hepatocellular carcinoma arise in cirrhotic livers.

The most common primary liver tumours are cholangiocarcinoma and hepatocellular carcinoma. Overall metastatic disease
accounts for 95% of all liver malignancies making the primary liver tumours comparatively rare.
Primary liver tumours include: Cholangiocarcinoma. Hepatocellular carcinoma. Hepatoblastoma. Sarcomas (Rare).
Lymphomas. Carcinoids (most often secondary although primary may occur)

Hepatocellular carcinoma: These account for the bulk of primary liver tumours (75% cases). Its worldwide incidence reflects
its propensity to occur on a background of chronic inflammatory activity. Most cases arise in cirrhotic livers or those with
chronic hepatitis B infection, especially where viral replication is actively occurring. In the UK it accounts for less than 5% of
all cancers, although in parts of Asia its incidence is 100 per 100,000.
The majority of patients (80%) present with existing liver cirrhosis, with a mass discovered on screening ultrasound.
Diagnosis: CT/ MRI (usually both) are the imaging modalities of choice. a-fetoprotein is elevated in almost all cases. Biopsy
should be avoided as it seeds tumours cells through a resection plane. In cases of diagnostic doubt serial CT and aFP
measurements are the preferred strategy. Treatment: Patients should be staged with liver MRI and chest, abdomen and pelvic
CT scan. The testis should be examined in males (testicular tumours may cause raised AFP). PET CT may be used to identify
occult nodal disease. Surgical resection is the mainstay of treatment in operable cases. In patients with a small primary tumour
in a cirrhotic liver whose primary disease process is controlled, consideration may be given to primary whole liver resection and
transplantation.Liver resections are an option but since most cases occur in an already diseased liver the operative risks and
post-operative hepatic dysfunction are far greater than is seen following metastectomy. These tumours are not particularly
chemo or radiosensitive however, both may be used in a palliative setting. Tumour ablation is a more popular strategy.
Survival: Poor, overall survival is 15% at 5 years.

Cholangiocarcinoma: This is the second most common type of primary liver malignancy. As its name suggests these tumours
arise in the bile ducts. Up to 80% of tumours arise in the extra hepatic biliary tree. Most patients present with jaundice and by
this stage the majority will have disease that is not resectable. Primary scelerosing cholangitis is the main risk factor. In
deprived countries typhoid and liver flukes are also major risk factors. Diagnosis: Patients will typically have an obstructive
picture on liver function tests. CA 19-9, CEA and CA 125 are often elevated. CT/ MRI and MRCP are the imaging methods of
choice. Treatment: Surgical resection offers the best chance of cure. Local invasion of peri hilar tumours is a particular
problem and this coupled with lobar atrophy will often contra indicate surgical resection. Palliation of jaundice is important,
although metallic stents should be avoided in those considered for resection. Survival: Is poor, approximately 15% 5 year
survival.

26. A 39 year old man has suffered from terminal ileal Crohns disease for the past 20 years. Which condition is he least
likely to develop?
A. Gallstones
B. Malabsorption
C. Pyoderma gangrenosum
D. Amyloidosis
E. Feltys syndrome

Answer: E
Felteys syndrome: Rheumatoid disease, Splenomegaly and Neutropenia
Feltys syndrome is associated with rheumatoid disease. Individuals with long standing crohns disease are at risk of gallstones
because of impairment of the enterohepatic recycling of bile salts. Formation of entero-enteric fistulation may produce
malabsorption. Amyloidosis may complicate chronic inflammatory states.

Crohns disease: Crohns disease is a chronic transmural inflammation of a segment(s) of the gastrointestinal tract and may be
associated with extra intestinal manifestations. Frequent disease patterns observed include ileal, ileocolic and colonic disease.
Peri-anal disease may occur in association with any of these. The disease is often discontinuous in its distribution. Inflammation
may cause ulceration, fissures, fistulas and fibrosis with stricturing. Histology reveals a chronic inflammatory infiltrate that is
usually patchy and transmural.

Ulcerative colitis Vs Crohns


Crohn's disease Ulcerative colitis
Distribution Mouth to anus Rectum and colon
189
Macroscopic Cobblestone appearance, apthoid ulceration Contact bleeding
changes
Depth of disease Transmural inflammation Superficial inflammation
Distribution Patchy Continuous
pattern
Histological Granulomas (non caseating epithelioid cell aggregates Crypt abscesses, Inflammatory cells in the
features with Langhans' giant cells) lamina propria

Extraintestinal manifestations of Crohns


Related to disease extent Unrelated to disease extent
Aphthous ulcers (10%) Sacroiliiitis (10-15%)
Erythema nodosum (5-10%) Ankylosing spondylitis (1-2%)
Pyoderma gangrenosum (0.5%) Primary sclerosing cholangitis (Rare)
Acute arthropathy (6-12%) Gallstones (up to 30%)
Ocular complications (up to 10%) Renal calculi (up to 10%)

Theme: Renal stones

A. Calcium oxalate
B. Uric acid
C. Cystine
D. Struvite
E. Calcium phosphate

Please select the most likely stone type for each of the following urinary tract stone scenarios. Each option may be used once,
more than once or not at all.

27. A 73 year old lady is undergoing chemotherapy for treatment of acute leukaemia. She develops symptoms of renal colic.
Her urine tests positive for blood. A KUB x-ray shows no evidence of stones.
Answer: Uric acid
Chemotherapy and cell death can increase uric acid levels. In this acute setting the uric acid stones are unlikely to be coated
with calcium and will therefore be radiolucent.

28. A 16 year old boy presents with renal colic. His parents both have a similar history of the condition. His urine tests
positive for blood. A KUB style x-ray shows a relatively radiodense stone in the region of the mid ureter.
Answer: Cystine
Cystine stones are associated with an inherited metabolic disorder.

29. A 43 year old lady with episodes of recurrent urinary tract sepsis presents with a staghorn calculus of the left kidney.
Her urinary pH is 7.3. A KUB x-ray shows a faint outline of the calculus.
Answer: Struvite
Chronic infection with urease producing enzymes can produce an alkaline urine with formation of struvate stone.
Renal stones: Calcium oxalate: Hypercalciuria is a major risk factor (various causes). Hyperoxaluria may also increase risk
Hypocitraturia increases risk because citrate forms complexes with calcium making it more soluble. Stones are radio-opaque
(though less than calcium phosphate stones). Hyperuricosuria may cause uric acid stones to which calcium oxalate binds
Percentage of all calculi: 85%

Cystine: Inherited recessive disorder of transmembrane cystine transport leading to decreased absorption of cystine from
intestine and renal tubule. Multiple stones may form. Relatively radiodense because they contain sulphur. Percentage of all
calculi 1%. Uric acid: Uric acid is a product of purine metabolism. May precipitate when urinary pH low. May be caused by
diseases with extensive tissue breakdown e.g. malignancy. More common in children with inborn errors of metabolism.
Radiolucent. Percentage of all calculi: 5-10%. Calcium Stone type Urine acidity Mean
phosphate: May occur in renal tubular acidosis, high urinary pH urine pH
increases supersaturation of urine with calcium and phosphate. Calcium phosphate Normal- alkaline >5.5
Renal tubular acidosis types 1 and 3 increase risk of stone Calcium oxalate Variable 6
formation (types 2 and 4 do not). Radio-opaque stones Uric acid Acid 5.5
(composition similar to bone). Percentage of all calculi 10%. Struvate Alkaline >7.2
Struvite: Stones formed from magnesium, ammonium and
Cystine Normal 6.5
phosphate. Occur as a result of urease producing bacteria (and
are thus associated with chronic infections). Under the alkaline conditions produced, the crystals can precipitate. Slightly radio-
opaque. Percentage of all calculi: 2-20%>

190
Effect of urinary pH on stone formation: Urine pH will show individual variation (from pH 5-7). Post prandially the pH falls
as purine metabolism will produce uric acid. Then the urine becomes more alkaline (alkaline tide). When the stone is not
available for analysis the pH of urine may help to determine which stone was present.

30. A 64 year old man presents to the clinic with right upper quadrant discomfort. He has never attended the hospital
previously and is usually well. He has just retired from full time employment as a machinist in a PVC factory. CT
scanning shows a large irregular tumour in the right lobe of his liver. Which of the following lesions is the most likely?
A. Liposarcoma
B. Angiosarcoma
C. Hamartoma
D. Hyatid liver disease
E. Benign angioma

Answer: B
Angiosarcoma of the liver is a rare tumour. However, it is linked to working with vinyl chloride, as in this case. Although
modern factories minimise the exposure to this agent, this has not always been the case.

Occupational cancers accounted for 5.3% cancer deaths in 2005. In men the main cancers include: Mesothelioma. Bladder
cancer. Non melanoma skin cancer. Lung cancer. Sino nasal cancer. Occupations with high levels of occupational tumours
include: Construction industry. Working with coal tar and pitch. Mining. Metalworkers. Working with asbestos (accounts for
98% of all mesotheliomas). Working in rubber industry. Shift work has been linked to breast cancer in women (Health and
safety executive report RR595).
The latency between exposure and disease is typically 15 years for solid tumours and 20 for leukaemia. Many occupational
cancers are otherwise rare. For example sino nasal cancer is an uncommon tumour, 50% will be SCC. They are linked to
conditions such as wood dust exposure and unlike lung cancer is not strongly linked to cigarette smoking. Another typical
occupational tumour is angiosarcoma of the liver which is linked to working with vinyl chloride. Again in the non occupational
context this is an extremely rare sporadic tumour.

31. A 32 year old man is involved in a house fire and sustains extensive partial thickness burns to his torso and thigh. Two
weeks post operatively he develops oedema of both lower legs. The most likely cause of this is:
A. Iliofemoral deep vein thrombosis
B. Venous obstruction due to scarring
C. Hypoalbuminaemia
D. Excessive administration of intravenous fluids
E. None of the above

Answer: C
Loss of plasma proteins is the most common cause of oedema developing in this time frame.

Extensive burns: Haemolysis due to damage of erythrocytes by heat and microangiopathy. Loss of capillary membrane
integrity causing plasma leakage into interstitial space. Extravasation of fluids from the burn site causing hypovolaemic shock
(up to 48h after injury)- decreased blood volume and increased haematocrit. Protein loss. Secondary infection e.g.
Staphylococcus aureus. ARDS. Risk of Curlings ulcer (acute peptic stress ulcers). Danger of full thickness circumferential
burns in an extremity as these may develop compartment syndrome
Healing: Superficial burns: keratinocytes migrate to form a new layer over the burn site. Full thickness burns: dermal scarring.
Usually need keratinocytes from skin grafts to provide optimal coverage.

32. What is the diagnostic marker for carcinoid syndrome?


A. B-HCG
B. Histamine
C. Chromogranin A
D. 5-Hydroxyindoleacetic acid
E. 5-Hydroxytryptamine

Answer: D
Urinary measurement of 5- HIAA is an important part of clinical follow up.

Carcinoid syndrome: Carcinoid tumours secrete serotonin. Originate in neuroendocrine cells mainly in the intestine (midgut-
distal ileum/appendix). Can occur in the rectum, bronchi. Hormonal symptoms mainly occur when disease spreads outside the
bowel
Clinical features: Onset: years. Flushing face. Palpitations. Tricuspid stenosis causing dyspnea. Asthma. Severe diarrhoea
(secretory, persists despite fasting). Investigation: 5-HIAA in a 24-hour urine collection. Scintigraphy. CT scan. Treatment:
Octreotide. Surgical removal.

191
33. A 42 year old man from Southern India presents with chronic swelling of both lower legs, they are brawny and
indurated with marked skin tophic changes. Which of the following organisms is the most likely origin of this disease
process?
A. Loa loa
B. Wuchereria bancrofti
C. Trypanosoma cruzi
D. Trypanosoma gambiense
E. None of the above

Answer: B
W. Bancrofti is the commonest cause of filariasis leading to lymphatic obstruction. Infection with Loa loa typically occurs in the
African sub continent and usually results in generalised sub cutaneous infections without lymphatic obstruction. Trypanosomal
infections would not produce this clinical picture.

Wuchereria bancrofti: Parasitic filarial nematode. Accounts for 90% of cases of filariasis. Usually diagnosed by blood smears.
Usually transmitted by mosquitos. Treatment is with diethylcarbamazine.

34. A 45 year old lady has recently undergone a thyroidectomy for treatment of medullary thyroid cancer. Which of the
following tumour markers is used clinically to screen for recurrence?
a. Free T3
b. Thyroglobulin
c. Calcitonin
d. Free T4
e. Thyroid stimulating hormone
Answer: C
Calcitonin is clinically utilised to screen for medullary thyroid cancer recurrence. Thyroid function testing does not form part of
either diagnosis or follow up from a malignancy perspective. However, routine assessment of TSH may be needed in patients on
thyroxine.

Papillary carcinoma: Commonest sub-type. Accurately diagnosed on fine needle aspiration cytology. Histologically they may
demonstrate psammoma bodies (areas of calcification) and so called 'orphan Annie' nuclei. They typically metastasise via the
lymphatics and thus laterally located apparently ectopic thyroid tissue is usually a metastasis from a well differentiated papillary
carcinoma.
Follicular carcinoma: Are less common than papillary lesions. Like papillary tumours they may present as a discrete nodule.
Although they appear to be well encapsulated macroscopically there invasion on microscopic evaluation. Lymph node
metastases are uncommon and these tumours tend to spread haematogenously. This translates into a higher mortality rate.
Follicular lesions cannot be accurately diagnosed on fine needle aspiration cytology and thus all follicular FNA's will require at
least a hemi thyroidectomy.
Anaplastic carcinoma: Less common and tend to occur in elderly females. Disease is usually advanced at presentation and
often only palliative decompression and radiotherapy can be offered.
Medullary carcinoma: These are tumours of the parafollicular cells ( C Cells) and are of neural crest origin. The serum
calcitonin may be elevated which is of use when monitoring for recurrence. . They may be familial and occur as part of the
MEN -2A disease spectrum. Spread may be either lymphatic or haematogenous and as these tumours are not derived primarily
from thyroid cells they are not responsive to radioiodine.
Lymphoma: These respond well to radiotherapy. Radical surgery is unnecessary once the disease has been diagnosed on biopsy
material. Such biopsy material is not generated by an FNA and thus a core biopsy has to be obtained (with care!).

35. A 22 year old man is kicked in the head during a rugby match. He is temporarily concussed, but then regains
consciousness. Half an hour later he develops slurred speech, ataxia and loses consciousnesses. On arrival in hospital he
is intubated and ventilated. A CT Scan is performed which shows an extradural haematoma. What is the most likely
cause?
A. Basilar artery laceration
B. Middle meningeal artery laceration
C. Laceration of the sigmoid sinus
D. Laceration of the anterior cerebral artery
E. Laceration of the middle cerebral artery

Answer: B
The most likely vessel from those in the list to cause an acute extra dural haemorrhage is the middle meningeal artery. The
anterior and middle cerebral arteries may cause acute sub dural haemorrhage. Acute sub dural haemorrhages usually take
slightly longer to evolve than acute extra dural haemorrhages.

192
Middle meningeal artery: Middle meningeal artery is typically the third branch of the first part of the maxillary artery, one of
the two terminal branches of the external carotid artery. After branching off the maxillary artery in the infratemporal fossa, it
runs through the foramen spinosum to supply the dura mater (the outermost meninges) . The middle meningeal artery is the
largest of the three (paired) arteries which supply the meninges, the others being the anterior meningeal artery and the posterior
meningeal artery. The middle meningeal artery runs beneath the pterion. It is vulnerable to injury at this point, where the skull is
thin. Rupture of the artery may give rise to an extra dural hematoma. In the dry cranium, the middle meningeal, which runs
within the dura mater surrounding the brain, makes a deep indention in the calvarium. The middle meningeal artery is intimately
associated with the auriculotemporal nerve which wraps around the artery making the two easily identifiable in the dissection of
human cadavers and also easily damaged in surgery.

36. Which of the following is not characteristic of a granuloma?


A. Altered macrophages
B. Fused macrophages
C. Epithelioid cells
D. Mixture of chronic inflammatory cells
E. Polymorphnuclear leucocytes, cellular debris and fibrin
Answer:
E
These are typical components of an abscess cavity. Polymorphonuclear leucocytes may be found in a granuloma if there is a
focus of suppuration.

Chronic inflammation: it may occur secondary to acute inflammation.In most cases chronic inflammation occurs as a primary
process. These may be broadly viewed as being one of three main processes: Persisting infection with certain organisms such as
Mycobacterium tuberculosis which results in delayed type hypersensitivity reactions and inflammation. Prolonged exposure to
non-biodegradable substances such as silica or suture materials which may induce an inflammatory response. Autoimmune
conditions involving antibodies formed against host antigens.

Acute vs. Chronic inflammation: Acute inflammation: Infiltration of neutrophils. Changes to existing vascular structure and
increased permeability of endothelial cells. Process may resolve with: Suppuration. Complete resolution. Abscess formation.
Progression to chronic inflammation. Healing by fibrosis. Chronic inflammation: Angiogenesis predominates. Macrophages,
plasma cells and lymphocytes predominate. Healing by fibrosis is the main result

Granulomatous inflammation: A granuloma consists of a microscopic aggregation of macrophages (with epithelial type
arrangement =epitheliod). Large giant cells may be found at the periphery of granulomas.

Mediators: Growth factors released by activated macrophages include agents such as interferon and fibroblast growth factor
(plus many more). Some of these such as interferons may have systemic features resulting in systemic symptoms and signs,
which may be present in individuals with long standing chronic inflammation.

37. A 42 year old man presents with a painless lump in the left testicle that he noticed on self examination. Clinically there is
a firm nodule in the left testicle, ultrasound appearances show an irregular mass lesion. His serum AFP and HCG levels
are both within normal limits. What is the most likely diagnosis?
A. Yolk sack tumour
B. Seminoma
C. Testicular teratoma
D. Epididymo-orchitis
E. Adenomatoid tumour

Answer: B This mans age, presenting symptoms and normal tumour markers make a seminoma the most likely diagnosis.
Epididymo-orchitis does not produce irregular mass lesions which are painless.

Testicular cancer: Testicular cancer is the most common malignancy in men aged 20-30 years. Around 95% of cases of
testicular cancer are germ-cell tumours. Germ cell tumours may essentially be divided into Seminoma and Non seminomatous
germ cell tumours

Seminoma: Key features: Commonest subtype (50%). Average age at diagnosis = 40. Even advanced disease associated with 5
year survival of 73% AFP usually normal. Tumour markers: HCG elevated in 10% seminomas. Lactate dehydrogenase;
elevated in 10-20% seminomas (but also in many other conditions). Pathology: Sheet like lobular patterns of cells with
substantial fibrous component. Fibrous septa contain lymphocytic inclusions and granulomas may be seen.

Non seminomatous germ cell tumours (42%): Teratoma, Yolk sac tumour, Choriocarcinoma and Mixed germ cell tumours
(10%): Key features Younger age at presentation =20-30 years. Advanced disease carries worse prognosis (48% at 5 years).
193
Retroperitoneal lymph node dissection may be needed for residual disease after chemotherapy. Tumour markers: AFP
elevated in up to 70% of cases. HCG elevated in up to 40% of cases. Other markers rarely helpful. Pathology: Heterogenous
texture with occasional ectopic tissue such as hair

Risk factors for testicular cancer: Cryptorchidism. Infertility. Family history. Klinefelter's syndrome. Mumps orchitis.
Features: A painless lump is the most common presenting symptom. Pain may also be present in a minority of men. Other
possible features include hydrocele, gynaecomastia. Diagnosis: Ultrasound is first-line. CT scanning of the chest/ abdomen and
pelvis is used for staging. Tumour markers (see above) should be measured. Management: Orchidectomy (Inguinal approach).
Chemotherapy and radiotherapy may be given depending on staging. Abdominal lesions >1cm following chemotherapy may
require retroperitoneal lymph node dissection.. Prognosis is generally excellent: 5 year survival for seminomas is around 95% if
Stage I. 5 year survival for teratomas is around 85% if Stage I

Benign disease
Epididymo-orchitis: Acute epididymitis is an acute inflammation of the epididymis, often involving the testis and usually
caused by bacterial infection. Infection spreads from the urethra or bladder. In men <35 years, gonorrhoea or chlamydia are the
usual infections. Amiodarone is a recognised non infective cause of epididymitis, which resolves on stopping the drug..
Tenderness is usually confined to the epididymis, which may facilitate differentiating it from torsion where pain usually affects
the entire testis.
Testicular torsion: Twist of the spermatic cord resulting in testicular ischaemia and necrosis. Most common in males aged
between 10 and 30 (peak incidence 13-15 years). Pain is usually severe and of sudden onset.. Cremasteric reflex is lost and
elevation of the testis does not ease the pain. Treatment is with surgical exploration. If a torted testis is identified then both testis
should be fixed as the condition of bell clapper testis is often bilateral.
38. A baby is born by normal vaginal delivery at 39 weeks gestation. Initially all appears well and then the clinical staff
become concerned because the baby develops recurrent episodes of cyanosis. These are worse during feeding and
improve dramatically when the baby cries. The most likely underlying diagnosis is:
A. Choanal atresia
B. Oesophageal reflux
C. Tetralogy of Fallot
D. Oesophageal atresia
E. Congenital diaphragmatic hernia
Answer:
A
In Choanal atresia the episodes of cyanosis are usually worst during feeding. Improvement may be seen when the baby cries as the
oropharyngeal airway is used.

Choanal atresia: Congenital disorder with an incidence of 1 in 7000 births. Posterior nasal airway occluded by soft tissue or
bone. Associated with other congenital malformations e.g. coloboma. Babies with unilateral disease may go unnoticed. Babies
with bilateral disease will present early in life as they are obligate nasal breathers. Treatment is with fenestration procedures
designed to restore patency.

39. A 28 year old lady presents with a pigmented lesion on her calf. Excisional biopsy confirms a diagnosis of melanoma
measuring 1cm in diameter with a Breslow thickness of 0.5mm. The lesion is close <1 mm to all resection margins.
Which of the following surgical resection margins is acceptable for this lesion?
A. 5 cm
B. 1 cm
C. 0.5 cm
D. 2 cm
E. 3 cm

Answer: B
The main diagnostic features (major criteria): Change in size. Change in shape. Change in colour
Secondary features (minor criteria): Diameter >6mm. Inflammation. Oozing or bleeding. Altered sensation
Treatment: Suspicious lesions should undergo excision biopsy. The lesion should be removed in completely as incision biopsy
can make subsequent histopathological assessment difficult. Once the diagnosis is confirmed the pathology report should be
reviewed to determine whether further re-exicision of margins is required (see below):
Margins of excision-Related to Breslow thickness: Lesions 0-1mm thick:1cm. Lesions 1-2mm thick:1- 2cm (Depending upon
site and pathological features). Lesions 2-4mm thick:2-3 cm (Depending upon site and pathological features). Lesions >4 mm
thick:3cm
Further treatments such as sentinel lymph node mapping, isolated limb perfusion and block dissection of regional lymph node
groups should be selectively applied.

40. A 20 year old man is involved in a road traffic accident. Following the incident he is unable to extend his wrist. However,
this improves over the following weeks. Which type of injury is he most likely to have sustained?
A. Radial nerve neurotmesis

194
B. Radial nerve neuropraxia
C. Axillary nerve axonotmesis
D. Ulnar nerve neuropraxia
E. Ulnar nerve axonotmesis

Answer: B
Transient loss of function makes neuropraxia the most likely injury. The wrist extensors are innervated by the radial nerve
making this the most likely site of injury.
Neuropraxia: Nerve intact but electrical conduction is affected. Myelin sheath integrity is preserved. Full recovery. Autonomic
function preserved. Wallerian degeneration does not occur

41. A 53 year old lady has undergone a bilateral breast augmentation procedure many years previously. The implants are
tense and uncomfortable and are removed. During their removal the surgeon encounters a dense membrane
surrounding the implants, it has a coarse granular appearance. The tissue is sent for histology and it demonstrates
fibrosis with the presence of calcification. The underlying process responsible for these changes is:
A. Hyperplasia
B. Dysplasia
C. Metastatic calcification
D. Dystrophic calcification
E. Necrosis

Answer: D
Breast implants often become surrounded by a pseudocapsule and this may secondarily then be subjected to a process of
dystrophic calcification.

Pathological calcification: Dystrophic calcification:Deposition of calcium deposits in tissues that have undergone, degeneration,
damage or disease in the presence of normal serum calcium levels. Metastatic calcification:Deposition of calcium deposits in
tissues that are otherwise normal in the presence of increased serum calcium levels

42. A 4 year old girl presents with symptoms of right sided loin pain, lethargy and haematuria. On examination she is
pyrexial and has a large mass in the right upper quadrant. The most likely underlying diagnosis is:
A. Perinephric abscess
B. Nephroblastoma
C. Renal cortical adenoma
D. Grawitz tumour
E. Squamous cell carcinoma of the kidney

Answer: B
In a child of this age, with the symptoms described a nephroblastoma is the most likely diagnosis. A perinephric abscess is most
unlikely. If an abscess were to occur it would be confined to Gertotas fascia in the first instance, and hence anterior extension
would be unlikely.

Nephroblastoma (Wilms tumours): Usually present in first 4 years of life. May often present as a mass associated with
haematuria (pyrexia may occur in 50%). Often metastasise early (usually to lung). Treated by nephrectomy. Younger children
have better prognosis (<1 year of age =80% overall 5 year survival)

Theme: Thyroid neoplasms

A. Follicular carcinoma
B. Anaplastic carcinoma
C. Medullary carcinoma
D. Papillary carcinoma
E. Lymphoma
F. Hashimotos thyroiditis
G.Graves disease
For the following histological descriptions please select the most likely underlying thyroid neoplasm. Each option may be used
once, more than once or not at all.

44. A 22 year old female undergoes a thyroidectomy. The resected specimen shows a non encapsulated tumour with
papillary projections and pale empty nuclei.

195
Answer: Papillary carcinoma
The presence of papillary structures together with the cytoplasmic features described is strongly suggestive of papillary
carcinoma. They are seldom encapsulated.

45. A thyroidectomy specimen from a 43 year old lady shows a mass with prominent oxyphil cells and scanty thyroid
colloid.
Answer Follicular carcinoma
Hurthle cell tumours are a variant of follicular neoplasms in which oxyphil cells predominate. They have a poorer prognosis than
conventional follicular neoplasms

46. A 32 year old lady undergoes a thyroidectomy for a mild goitre. The resected specimen shows an intense lymphocytic
infiltrate with acinar destruction and fibrosis.
Answer: Hashimotos thyroiditis
Lymphocytic infiltrates and fibrosis are typically seen in Hashimotos thyroiditis. In Lymphoma only dense lymphatic type tissue
is usually present.

Thyroid neoplasms
Lesion:Common features
Follicular adenoma: Usually present as a solitary thyroid nodule. Malignancy can only be excluded on formal histological
assessment
Papillary carcinoma: Usually contain a mixture of papillary and colloidal filled follicles. Histologically tumour has papillary
projections and pale empty nuclei. Seldom encapsulated. Lymph node metastasis predominate. Haematogenous metastasis rare.
Account for 60% of thyroid cancers
Follicular carcinoma:May appear macroscopically encapsulated, microscopically capsular invasion is seen. Without this
finding the lesion is a follicular adenoma.Vascular invasion predominates. Multifocal disease rare. Account for 20% of all
thyroid cancers
Anaplastic carcinoma: Most common in elderly females. Local invasion is a common feature. Account for 10% of thyroid
cancers. Treatment is by resection where possible, palliation may be achieved through isthmusectomy and radiotherapy.
Chemotherapy is ineffective.
Medullary carcinoma:Tumours of the parafollicular cells (C Cells). C cells derived from neural crest and not thyroid tissue.
Serum calcitonin levels often raised. Familial genetic disease accounts for up to 20% cases. Both lymphatic and haematogenous
metastasis are recognised, nodal disease is associated with a very poor prognosis.

47. A 43 year old lady with hypertension is suspected of having a phaeochromocytoma. Which of the following investigations
is most likely to be beneficial in this situation?
A. Dexamethasone suppression test
B. Urinary 5-Hydroxyindoleacetic Acid (5-HIAA)
C. Histamine provocation test
D. Tyramine provocation test
E. Urinary vanillymandelic acid measurements
Answer:
E
Urinary VMA measurements are not completely specific but constitute first line assessment. Stimulation tests of any sort are not
justified in first line assessments.

Phaeochromocytoma: Neuroendocrine tumour of the chromaffin cells of the adrenal medulla. Hypertension and hyperglycaemia
are often found: 10% of cases are bilateral. 10% occur in children. 11% are malignant (higher when tumour is located outside the
adrenal). 10% will not be hypertensive. Familial cases are usually linked to the Multiple endocrine neoplasia syndromes
(considered under its own heading). Most tumours are unilateral (often right sided) and smaller than 10cm. Diagnosis: Urine
analysis of vanillymandelic acid (VMA) is often used (false positives may occur e.g. in patients eating vanilla ice cream!). Blood
testing for plasma metanephrine levels. CT and MRI scanning are both used to localise the lesion.

Treatment: Patients require medical therapy first. An irreversible alpha adrenoreceptor blocker should be given, although
minority may prefer reversible bockade(1). Labetolol may be co-administered for cardiac chronotropic control. Isolated beta
blockade should not be considered as it will lead to unopposed alpha activity. These patients are often volume depleted and will
often require moderate volumes of intra venous normal saline perioperatively. Once medically optimised the phaeochromocytoma
should be removed. Most adrenalectomies can now be performed using a laparoscopic approach(2). The adrenals are highly
vascular structures and removal can be complicated by catastrophic haemorrhage in the hands of the inexperienced. This is
particularly true of right sided resections where the IVC is perilously close. Should the IVC be damaged a laparotomy will be
necessary and the defect enclosed within a Satinsky style vascular clamp and the defect closed with prolene sutures. Attempting to
interfere with the IVC using any instruments other than vascular clamps will result in vessel trauma and make a bad situation
much worse.
196
Incidental adrenal lesions
Adrenal lesions may be identified on CT scanning performed for other reasons(3). Factors suggesting benign disease on CT
include(4): Size less than 3cm. Homogeneous texture. Lipid rich tissue. Thin wall to lesion

All patients with incidental lesions should be managed jointly with an endocrinologist and full work up as described above.
Patients with functioning lesions or those with adverse radiological features (Particularly size >3cm) should proceed to surgery.

48. A 46 year old lady presents with symptoms of diarrhoea, weight loss of 10 Kg and a skin rash of erythematous blisters
involving the abdomen and buttocks. The blisters have an irregular border and both intact and ruptured vesicles. What
is the most likely diagnosis?
A. Colonic adenocarcinoma
B. Pancreatic adenocarcinoma
C. Tropical sprue
D. Glucagonoma
E. Insulinoma

Answer: D
Glucagonoma Is strongly associated with necrolytic migratory erythema.

Glucagonoma: Rare pancreatic tumours arising from the alpha cells of the pancreas.Glucagon levels markedly elevated. Symptoms
include diarrhoea, weight loss and necrolytic migratory erythema. A serum level of glucagon >1000pg/ml usually suggests the
diagnosis, imaging with CT scanning is also required. Treatment is with surgical resection.

49. A 56 year old man presents with symptoms of neuropathic facial pain and some weakness of the muscles of facial
expression on the right side. On examination he has a hard mass approximately 6cm anterior to the right external
auditory meatus. What is the most likely diagnosis?
A. Pleomorphic adenoma
B. Adenocarcinoma
C. Mucoepidermoid carcinoma
D. Adenoid cystic carcinoma
E. Lymphoma

Answer: D The patient is most likely to have a malignant lesion within the parotid. Of the malignancies listed; adenoid cystic
carcinoma has the greatest tendency to perineural invasion.

Parotid gland malignancy: Most parotid neoplasms (80%) are benign lesions. Most commonly present with painless mass in
cheek region. Up to 30% may present with pain, when this is associated with a discrete mass lesion in the parotid it usually
indicates perineural invasion. Perineural invasion is very unlikely to occur in association with benign lesions. 80% of patients with
facial nerve weakness caused by parotid malignancies will have nodal metastasis and a 5 year survival of 25%
Types of malignancy
Mucoepidermoid carcinoma: 30% of all parotid malignancies. Usually low potential for local invasiveness and metastasis
(depends mainly on grade)
Adenoid cystic carcinoma:Unpredictable growth patter. Tendency for perineural spread. Nerve growth may display skip lesions
resulting in incomplete excision. Distant metastasis more common (visceral rather than nodal spread). 5 year survival 35%
Mixed tumours: Often a malignancy occurring in a previously benign parotid lesion
Acinic cell carcinoma:Intermediate grade malignancy. May show perineural invasion. Low potential for distant metastasis. 5 year
survival 80%
Adenocarcinoma:Develops from secretory portion of gland. Risk of regional nodal and distant metastasis. 5 year survival
depends upon stage at presentation, may be up to 75% with small lesions with no nodal involvement
Lymphoma:Large rubbery lesion, may occur in association with Warthins tumours. Diagnosis should be based on regional nodal
biopsy rather than parotid resection Treatment is with chemotherapy (and radiotherapy)

50. A 20 year old African lady undergoes an open appendicectomy. She is reviewed for an unrelated problem 8 months
later. On abdominal inspection the wound site is covered by shiny dark protuberant scar tissue that projects beyond the
limits of the skin incision. Which of the following is the most likely underlying process?
A. Hypertrophic scar
B. Keloid scar
C. Marjolins ulcer
D. Repeated episodes of wound sepsis

197
E. Mycosis fungoides

Answer: B
Keloid scars extend beyond the limits of the incision. Mycosis fungoides is a cutaneous T cell lymphoma.

Wound healing: Surgical wounds are either incisional or excisional and either clean, clean contaminated or dirty. Although the
stages of wound healing are broadly similar their contributions will vary according to the wound type.
The main stages of wound healing include:
Haemostasis: Vasospasm in adjacent vessels, platelet plug formation and generation of fibrin rich clot.
Inflammation: Neutrophils migrate into wound (function impaired in diabetes). Growth factors released, including basic
fibroblast growth factor and vascular endothelial growth factor. Fibroblasts replicate within the adjacent matrix and migrate into
wound. Macrophages and fibroblasts couple matrix regeneration and clot substitution.
Regeneration: Platelet derived growth factor and transformation growth factors stimulate fibroblasts and epithelial cells.
Fibroblasts produce a collagen network. Angiogenesis occurs and wound resembles granulation tissue.
Remodeling: Longest phase of the healing process and may last up to one year (or longer). During this phase fibroblasts become
differentiated (myofibroblasts) and these facilitate wound contraction.. Collagen fibres are remodeled. Microvessels regress
leaving a pale scar.
The above description represents an idealised scenario. A number of diseases may distort this process. It is obvious that one of the
key events is the establishing well vascularised tissue. At a local level angiogenesis occurs, but if arterial inflow and venous return
are compromised then healing may be impaired, or simply nor occur at all. The results of vascular compromise are all too
evidence in those with peripheral vascular disease or those poorly constructed bowel anastomoses.
Conditions such as jaundice will impair fibroblast synthetic function and overall immunity with a detrimental effect in most parts
of healing.
Problems with scars: Hypertrophic scars: Excessive amounts of collagen within a scar. Nodules may be present histologically
containing randomly arranged fibrils within and parallel fibres on the surface. The tissue itself is confined to the extent of the
wound itself and is usually the result of a full thickness dermal injury. They may go on to develop contractures. Keloid scars:
Excessive amounts of collagen within a scar. Typically a keloid scar will pass beyond the boundaries of the original injury. They
do not contain nodules and may occur following even trivial injury. They do not regress over time and may recur following
removal.
Drugs which impair wound healing: Non steroidal anti inflammatory drugs. Steroids. Immunosupressive agents. Anti neoplastic
drugs
Closure: Delayed primary closure is the anatomically precise closure that is delayed for a few days but before granulation tissue
becomes macroscopically evident. Secondary closure refers to either spontaneous closure or to surgical closure after granulation
tissue has formed.

51. The pathogenicity of the tubercle bacillus is due to which of the following?
A. Necrosis caused by expanding granulomas
B. Ability to multiply within fibroblasts
C. Delayed hypersensitivity reaction against bacteria
D. Effect of antibody response
E. Direct toxic effect on host cells

Answer: C
Mycobacteria stimulate a specific T cell response of cell mediated immunity. This is effective in reducing the infection, the
delayed hypersensitivity also damages tissues. Necrosis occurs in TB but is usually within the granuloma.

Tuberculosis pathology: Is a form of primary chronic inflammation, caused by the inability of macrophages to kill the
Mycobacterium tuberculosis. The macrophages often migrate to regional lymph nodes, the lung lesion plus affected lymph
nodes is referred to as a Ghon complex.This leads to the formation of a granuloma which is a collection of epithelioid
histiocytes. There is the presence of caseous necrosis in the centre. The inflammatory response is mediated by a type 4
hypersensitivity reaction. In healthy individuals the disease may be contained, in the immunocompromised disseminated
(miliary TB) may occur.

Diagnosis: Waxy membrane of mycobacteria prevents binding with normal stains. Ziehl - Neelsen staining is typically used.
Culture based methods take far longer.

52. A 45 year old women with a thyroid carcinoma undergoes a total thyroidectomy. The post operative histology report
shows a final diagnosis of medullary type thyroid cancer. Which of the tests below is most likely to be of clinical use in
screening for disease recurrence?
A. Serum CA 19-9 Levels
B. Serum thyroglobulin levels
C. Serum PTH levels
D. Serum calcitonin levels
E. Serum TSH levels
198
Answer: D

Medullary thyroid cancers often secrete calcitonin and monitoring the serum levels of this hormone is useful in detecting sub
clinical recurrence.

53. A 15 year old boy undergoes an emergency splenectomy for trauma. He makes a full recovery and is discharged home.
Eight weeks post operatively the general practitioner performs a full blood count with a blood film. Which of the
following is most likely to be present?
A. Myofibroblasts
B. Howell-Jolly bodies
C. Multinucleate giant cells
D. Reed Sternberg Cells
E. None of the above

Answer: B
Post splenectomy blood film features: Howell- Jolly bodies. Pappenheimer bodies. Target cells. Irregular contracted
erythrocytes
As the filtration function is the spleen is no longer present Howell-Jolly bodies are found.

Post splenectomy blood film changes: The loss of splenic tissue results in the inability to readily remove immature or
abnormal red blood cells from the circulation. The red cell count does not alter significantly. However, cytoplasmic inclusions
may be seen e.g. Howell-Jolly bodies. In the first few days after splenectomy target cells, siderocytes and reticulocytes will
appear in the circulation. Immediately following splenectomy a granulocytosis (mainly composed of neutrophils) is seen, this is
replaced by a lymphocytosis and monocytosis over the following weeks.
The platelet count is usually increased and this may be persistent, oral antiplatelet agents may be needed in some patients.

54. A 43 year old women is identified as being a carrier of a BRCA 1 mutation. Apart from breast cancer, which of the
following malignancies is she at greatest risk of developing?
A. Colonic cancer
B. Ovarian cancer
C. Follicular carcinoma of the thyroid
D. Pituitary adenoma
E. Phaeochromocytoma

Answer: B
BRCA 1 mutation patients are 55% more likely to get ovarian cancer. Those with BRCA 2 are 25% more likely. The risk of
developing other malignancies is slightly increased but not to the same extent, and not enough to justify screening.

Genetics and surgical disease: Some of the more commonly occurring genetic conditions occurring in surgical patients are
presented here.
Li-Fraumeni Syndrome: Autosomal dominant. Consists of germline mutations to p53 tumour suppressor gene. High incidence
of malignancies particularly sarcomas and leukaemias. Diagnosed when: *Individual develops sarcoma under 45 years. *First
degree relative diagnosed with any cancer below age 45 years and another family member develops malignancy under 45 years or
sarcoma at any age
BRCA 1 and 2: Carried on chromosome 17. Linked to developing breast cancer (60%) risk. Associated risk of developing
ovarian cancer (55% with BRCA 1 and 25% with BRCA2).
Lynch Syndrome: Autosomal dominant . Develop colonic cancer and endometrial cancer at young age. 80% of affected
individuals with get colonic and or endometrial cancer. High risk individuals may be identified using the Amsterdam criteria
Amsterdam criteria: Three or more family members with a confirmed diagnosis of colorectal cancer, one of whom is a first
degree (parent, child, sibling) relative of the other two. Two successive affected generations. One or more colon cancers
diagnosed under age 50 years. Familial adenomatous polyposis (FAP) has been excluded.
Gardners syndrome: Autosomal dominant familial colorectal polyposis. Multiple colonic polyps. Extra colonic diseases include:
skull osteoma, thyroid cancer and epidermoid cysts. Desmoid tumours are seen in 15%. Mutation of APC gene located on
chromosome 5. Due to colonic polyps most patients will undergo colectomy to reduce risk of colorectal cancer. Now considered a
variant of familial adenomatous polyposis coli

55. A 53 year old man is due to undergo a splenectomy as a treatment for refractory haemolytic anaemia. The underlying
pathological basis for haemolytic anaemia is thought to be a Type 2 hypersensitivity response. Which of the following
mechanisms best describes this process
A. Deposition of immune complexes
B. Cell mediated immune response
C. IgE mediated response
199
D. Formation of autoantibodies against cell surface antigens
E. None of the above

Answer: D
Mnemonic for the reactions and the mediators involved
ACID EGG-T : Type 1 Anaphylactic. Type 2 Cytotoxic. Type 3 Immune complex. Type 4 Delayed type. EGG T (mediators): IgE.
IgG. IgG. T cells. Type 2 hypersensitivity reactions (which includes haemolytic anaemia) are associated with formation of
antibody against cell surface antigens.

56. A 25 year old man is injured in a road traffic accident. His right tibia is fractured and is managed by fasciotomies and
application of an external fixator. Over the next 48 hours his serum creatinine rises and urine is sent for microscopy,
muddy brown casts are identified. What is the most likely underlying diagnosis?
A. Acute interstitial nephritis
B. Acute tubular necrosis
C. Glomerulonephritis
D. IgA Nephropathy
E. Thin basement membrane disease

Answer: B
This patient is likely to have had compartment syndrome (tibial fracture + fasciotomies) which may produce myoglobinuria. The
presence of worsening renal function, together with muddy brown casts is strongly suggestive of acute tubular necrosis. Acute
interstitial nephritis usually arises from drug toxicity and does not usually produce urinary muddy brown casts. Thin basement
membrane disease is an autosomal dominant condition that causes persistent microscopic haematuria, but not worsening renal
function.

Acute Renal Failure: Final pathway is tubular cell death. Renal medulla is a relatively hypoxic environment making it susceptible
to renal tubular hypoxia. Renovascular autoregulation maintains renal blood flow across a range of arterial pressures. Estimates of
GFR are best indices of level of renal function. Useful clinical estimates can be obtained by considering serum creatinine, age, race,
gender and body size. eGFR calculations such as the Cockcroft and Gault equation are less reliable in populations with high GFR's.
Nephrotoxic stimuli such as aminoglycosides and radiological contrast media induce apoptosis. Myoglobinuria and haemolysis
result in necrosis. Overlap exists and proinflammatory cytokines play and important role in potentiating ongoing damage. Post-
operative renal failure is more likely to occur in patients who are elderly, have peripheral vascular disease, high BMI, have COPD,
receive vasopressors, are on nephrotoxic medication or undergo emergency surgery. Avoiding hypotension will reduce risk of renal
tubular damage. There is no evidence that administration of ACE inhibitors or dopamine reduces the incidence of post-operative
renal failure.

57. A 56 year old man has undergone a radical nephrectomy. The pathologist bisects the kidney and identifies a pink fleshy
tumour in the renal pelvis. What is the most likely disease?
A. Renal cell carcinoma
B. Transitional cell carcinoma
C. Angiomyolipoma
D. Phaeochromocytoma
E. Renal adenoma
Ansewr:
B
Most renal tumours are yellow or brown in colour. TCC's are one of the few tumours to appear pink.
The finding of a TCC in the renal pelvis mandates a nephroureterectomy.

Renal cell carcinoma: Most present with haematuria (50%). Common renal tumour (85% cases). Paraneoplastic features include
hypertension and polycythaemia. Most commonly has haematogenous mestastasis. Treatment:Usually radical or partial
ephrectomy

Nephroblastoma:Rare childhood tumour. It accounts for 80% of all genitourinary malignancies in those under the age of 15
years. Up to 90% will have a mass. 50% will be hypertensive Diagnostic work up includes ultrasound and CT scanning.
Treatment:Surgical resection combined with chemotherapy (usually vincristine, actinomycin D and doxorubicin

Neuroblastoma: Most common extracranial tumour of childhood. 80% occur in those under 4 years of age. Tumour of neural
crest origin (up to 50% occur in the adrenal gland). The tumour is usually calcified and may be diagnosed using MIBG scanning.
Staging is with CT:Surgical resection, radiotherapy and chemotherapy

200
Transitional cell carcinoma: Accounts for 90% of upper urinary tract tumour, but only 10% of renal tumours. Males affected
3x more than females. Occupational exposure to industrial dyes and rubber chemicals may increase risk. Up to 80% present with
painless haematuria. Diagnosis and staging is with CT IVU. Treatment: Radical nephroureterectomy

Angiomyolipoma:80% of these hamartoma type lesions occur sporadically, the remainder are seen in those with tuberous
sclerosis. Tumour is composed of blood vessels, smooth muscle and fat. Massive bleeding may occur in 10% of cases.
Treatment: 50% of patients with lesions >4cm will have symptoms and will require surgical resection

58. A 65 year old lady presents with a lesion affecting her right breast. On examination she has a weeping, crusting lesion
overling the right nipple, the areolar region is not involved. There is no palpable mass lesion in the breast, there is a
palpable axillary lymph node. The patients general practitioner has tried treating the lesion with 1% hydrocortisone
cream, with no success. What is the most likely diagnosis?
A. Infection with Staphylococcus aureus
B. Pagets disease of the nipple
C. Phyllodes tumour
D. Nipple eczema
E. Basal cell carcinoma

Answer: B
A weeping, crusty lesion such as this is most likely to represent Pagets disease of the nipple (especially since the areolar region is
spared). Although no mass lesion is palpable, a proportion of patients will still have an underlying invasive malignancy (hence the
lymphadenopathy).

Pagets disease of the nipple: Pagets disease is an eczematoid change of the nipple associated with an underlying breast
malignancy and it is present in 1-2% of patients with breast cancer. In half of these patients, it is associated with an underlying
mass lesion and 90% of such patients will have an invasive carcinoma. 30% of patients without a mass lesion will still be found
to have an underlying carcinoma. The remainder will have carcinoma in situ.
Pagets disease differs from eczema of the nipple in that it involves the nipple primarily and only latterly spreads to the areolar
(the opposite occurs in eczema). Diagnosis is made by punch biopsy, mammography and ultrasound of the breast.
Treatment will depend on the underlying lesion.

59. A 73 year old man presents with haemoptysis and is suspected of suffering from lung cancer. On examination he has an
enlarged supraclavicular lymph node. Which of the following features is most likely to be present on histological
examination?
A. Increased mitoses
B. Apoptosis
C. Barr Bodies
D. Multinucleate giant cells
E. Granuloma

Answer: A
Increased mitoses are commonly seen in association with malignant transformation of cells. Apoptosis is not a common feature
of metastatic cancer. Barr Bodies are formed during X chromosome inactivation in female somatic cells.
Histopathology of malignancy: Abnormal tissue architecture. Coarse chromatin. Invasion of basement membrane*. Abnormal
mitoses. Angiogenesis. De-differentiation. Areas of necrosis. Nuclear pleomorphism

*= Those features that distinguish invasive malignancy from in situ disease

60. Which of the following pathological explanations best describes the initial pathological processes occurring in an
abdominal aortic aneurysm in an otherwise well 65 year old, hypertensive male?
A. Loss of elastic fibres from the adventitia
B. Loss of collagen from the adventitia
C. Loss of collagen from the media
D. Loss of elastic fibres from the media
E. Decreased matrix metalloproteinases in the adventitia
Answer:
D
In established aneurysmal disease there is dilation of all layers of the arterial wall and loss of both elastin and collagen. The
primary event is loss of elastic fibres with subsequent degradation of collagen fibres.

Pathology of abdominal aortic aneurysm: Abdominal aortic aneurysms occur primarily as a result of the failure of elastic
proteins within the extracellular matrix. Anuerysms typically represent dilation of all layers of the arterial wall. Most aneurysms
are caused by degenerative disease. After the age of 50 years the normal diameter of the infrarenal aorta is 1.5cm in females and
201
1.7cm in males. Diameters of 3cm and greater, are considered aneurysmal. The pathophysiology involved in the development of
aneurysms is complex and the primary event is loss of the intima with loss of elastic fibres from the media. This process is
associated with, and potentiated by, increased proteolytic activity and lymphocytic infiltration.

Major risk factors for the development of aneurysms include smoking and hypertension. Rare but important causes include
syphilis and connective tissues diseases such as Ehlers Danlos type 1 and Marfans syndrome.

61. A 28 year old lady has a malignant melanoma removed from her calf. Which of the following pathological criteria
carries the greatest prognostic weighting?
A. Vascular invasion
B. Abnormal mitoses
C. Breslow thickness
D. Perineural invasion
E. Lymphocytic infiltrates

Answer: C
The Breslow thickness has considerable prognostic importance. Lymphocytic infiltrates may be associated with an improved
prognosis, but do not carry nearly the same weight as increased thickness.

62. A 34 year old lady undergoes an elective cholecystectomy for attacks of recurrent cholecystitis due to gallstones.
Microscopic assessment of the gallbladder is most likely to show which of the following?
A. Dysplasia of the fundus
B. Widespread necrosis
C. Ashoff-Rokitansky sinuses
D. Metaplasia of the fundus
E. None of the above

Answer: C
Aschoff-Rokitansky sinuses are the result of hyperplasia and herniation of epithelial cells through the fibromuscular layer of the
gallbladder wall. They may be macroscopic or microscopic. Ashoff-Rokitansky sinuses may be identified in cases of chronic
cholecystitis and gallstones. Although gallstones may predispose to the development of gallbladder cancer the actual incidence
of dysplasia and metaplastic change is rare. In the elective setting described above necrosis would be rare.

Gallbladder: Fibromuscular sac with capacity of 50ml. Columnar epithelium. Relations of the gallbladder: Anterior: Liver.
Posterior: Covered by peritoneum. Transverse colon. 1st part of the duodenum. Laterally: Right lobe of liver. Medially:
Quadrate lobe of liver. Arterial supply: Cystic artery (branch of Right hepatic artery). Venous drainage: Cystic vein. Nerve
supply: Sympathetic- mid thoracic spinal cord, Parasympathetic- anterior vagal trunk
Common bile duct: Origin: Confluence of cystic and common hepatic ducts. Relations at origin: Medially - Hepatic artery.
Posteriorly- Portal vein. Relations distally:Duodenum – anteriorly. Pancreas - medially and laterally Right renal vein –
posteriorly. Arterial supply: Branches of hepatic artery and retroduodenal branches of gastroduodenal artery.

Calot's triangle: Medially: Common hepatic duct. Inferiorly:Cystic duct. Superiorly:Inferior edge of liver. Contents:Cystic
artery

63. Which of the following are not true of Li-Fraumeni syndrome?


A. It consists of mutations to the p53 tumour suppressor gene
B. Is likely to be present in a teenager presenting with a liposarcoma
C. It has an autosomal dominant inheritance pattern
D. Affected individuals are unlikely to develop acute myeloid leukaemia
E. Adrenal malignancies are more common than in normal population

Answer: D

They are at high risk of developing leukaemia.

64. A 35 year old type 1 diabetic presents with difficulty mobilising and back pain radiating to the thigh. He has a
temperature of 39 oC and has pain on extension of the hip. He is diagnosed with an iliopsoas abscess. Which of the
following statements is false in relation to his diagnosis?
A. Staphylococcus aureus is the most likely primary cause
B. Recurrence occurs in 60% cases
C. More common in males
D. Crohn's is the most likely secondary cause

202
E. CT guided drainage is preferable first line management
Answer:
B
Classical features include: a limp, back pain and fever. Recurrence rates are about 15-20%. Staphylococcus is the commonest
primary cause, others include Streptococcus and E.coli. Management is ideally by CT guided drainage.

Iliopsoas abscess: Collection of pus in iliopsoas compartment (iliopsoas and iliacus): Causes: Primary: Haematogenous spread
of bacteria. Staphylococcus aureus: most common. Secondary: Crohn's (commonest cause in this category). Diverticulitis,
Colorectal cancer. UTI, GU cancers. Vertebral osteomyelitis. Femoral catheter, lithotripsy. Endocarditis. Note the mortality rate
can be up to 19-20% in secondary iliopsoas abscesses compared with 2.4% in primary abscesses. Clinical features: Fever.
Back/flank pain. Limp. Weight loss

Clinical examination: Patient in the supine position with the knee flexed and the hip mildly externally rotated. Specific tests to
diagnose iliopsoas inflammation: Place hand proximal to the patient's ipsilateral knee and ask patient to lift thigh against your
hand. This will cause pain due to contraction of the psoas muscle. Lie the patient on the normal side and hyperextend the
affected hip. In inflammation this should elicit pain as the psoas muscle is stretched.

Investigation: CT is gold standard

Management: Antibiotics. Percutaneous drainage. Surgery is indicated if: 1. Failure of percutaneous drainage. 2. Presence of an
another intra-abdominal pathology which requires surgery

Surgical approach: The authors technique for draining these collections is given here. Review the CT scans and plan surgical
approach. An extraperitoneal approach is important. The collection usually extends inferiorly and can be accessed from an
incision at a level of L4 on the affected side. GA. Transverse laterally placed incision. Incise external oblique. Split the
subsequent muscle layers.
As you approach the peritoneum use blunt dissection to pass laterally around it. Remember the ureter and gonadal veins lie
posterior at this level. Eventually you will enter the abscess cavity, a large amount of pus is usually released at this point. Drain
the area with suction and washout with saline. Place a corrugated drain well into the abscess cavity. If you have made a small
skin incision it is reasonable to bring the drain up through the skin wound. Otherwise place a lateral exit site and close the skin
and external oblique. If you do this ensure that you use interrupted sutures. Anchor the drain with strong securely tied silk
sutures (it is extremely tiresome if it falls out!)

65. Which of the following statements relating to parathyroid neoplasms is incorrect?


A. 15% of cases are due to parathyroid carcinoma
B. 80% of cases are due to parathyroid adenomas
C. Parathyroid adenomas are often encapsulated
D. 10% of parathyroid adenomas develop in ectopically located glands
E. 85% of cases of primary hyperparathyroidism are due to solitary adenomas
Answer:
A
Parathyroid carcinomas account for up to 5% of tumours. Adenomas are often encapsulated .Lesions that are fibrotic and
densely adherent to the gland may be a carcinoma. 85% cases of primary hyperparathyroidism are due to a single adenoma and
this is the reason some surgeons favour a focussed parathyroidectomy.

Parathyroid glands and disorders of calcium metabolism

Primary hyperparathyroidism: PTH (Elevated). Ca2+ (Elevated). Phosphate (Low). Serum Calcium: Creatinine clearance
ratio > 0.01. Clinical features: May be asymptomatic if mild. Recurrent abdominal pain (pancreatitis, renal colic). Changes to
emotional or cognitive state. Causes: Most cases due to solitary adenoma (80%), multifocal disease occurs in 10-15% and
parathyroid carcinoma in 1% or less. Management: Indications for surgery: Elevated serum Calcium > 1mg/dL above normal.
Hypercalciuria > 400mg/day. Creatinine clearance < 30% compared with normal. Episode of life threatening hypercalcaemia.
Nephrolithiasis. Age < 50 years. Neuromuscular symptoms. Reduction in bone mineral density of the femoral neck, lumbar
spine, or distal radius of more than 2.5 standard deviations below peak bone mass (T score lower than -2.5)

Secondary hyperparathyroidism: PTH (Elevated). Ca2+ (Low or normal). Phosphate (Elevated). Vitamin D levels (Low).
Clinical features: May have few symptoms. Eventually may develop bone disease, osteitis fibrosa cystica and soft tissue
calcifications. Causes: Parathyroid gland hyperplasia occurs as a result of low calcium, almost always in a setting of chronic
renal failure. Management: Usually managed with medical therapy. Indications for surgery in secondary (renal)
hyperparathyroidism: Bone pain. Persistent pruritus. Soft tissue calcifications

203
Tertiary hyperparathyroidism: Ca2+ (Normal or high). PTH (Elevated). Phosphate levels (Decreased or Normal). Vitamin D
(Normal or decreased). Alkaline phosphatase (Elevated). Clinical features: Metastatic calcification. Bone pain and / or fracture.
Nephrolithiasis. Pancreatitis. Causes: Occurs as a result of ongoing hyperplasia of the parathyroid glands after correction of
underlying renal disorder, hyperplasia of all 4 glands is usually the cause. Management: Usually treatment is surgical. The
presence of an autonomously functioning parathyroid gland may require surgery. If the culprit gland can be identified then it
should be excised. Otherwise total parathyroidectomy and re-implantation of part of the gland may be required.

Differential diagnoses
It is important to consider the rare but relatively benign condition of benign familial hypocalciuric hypercalcaemia, caused by an
autosomal dominant genetic disorder. Diagnosis is usually made by genetic testing and concordant biochemistry (Serum
Calcium: Creatinine clearance ratio <0.01-distinguished from primary hyperparathyroidism).

66. A 20 year old girl presents with a thyroid cancer, she is otherwise well with no significant family history. On
examination she has a nodule in the left lobe of the thyroid with a small discrete mass separate from the gland itself.
Which of the following is the most likely cause?
A. Follicular carcinoma
B. Anaplastic carcinoma
C. Medullary carcinoma
D. Papillary carcinoma
E. B Cell Lymphoma

Answer: D
Papillary carcinoma is the most common subtype and may cause lymph node metastasis (mass separate from the gland itself)
that is rare with follicular tumours. Anaplastic carcinoma would cause more local symptoms and would be rare in this age
group.

67. A 28 year old lady is breast feeding her first child. She presents with discomfort of the right breast. Clinical examination
demonstrates erythema and an area that is fluctuant. Aspiration and culture of the fluid is most likely to demonstrate
infection with which of the following organisms?
A. Clostridium perfringens
B. Staphylococcus aureus
C. Streptococcus pyogenes
D. Staphylococcus epidermidis
E. Actinomycosis

Answer: B
Staphylococcus aureus is the commonest cause. The infants mouth is usually the source as it damages the nipple areolar
complex allowing entry of bacteria.

Breast abscess: In lactational women Staphylococcus aureus is the most common cause. Typical presentation is with a tender
mass in a lactating women. There is often tenderness and pain and a fluctuant mass. Diagnosis and treatment is performed using
USS and associated drainage of the abscess cavity. Antibiotics should also be administered. Where there is necrotic skin
overlying the abscess, the patient should undergo surgery

68. An 18 year old rock climber falls onto his left arm and sustains a large haematoma of the left upper arm. Unfortunately
the wound associated with the injury is neglected and it becomes infected. Which of the following changes is least likely
to occur?
A. Axillary lymphadenopathy
B. Leucopenia
C. Tenderness
D. Mild pyrexia
E. Local formation of yellow pus

Answer: B
Leucopenia would be unusual and should prompt a search for another cause.

Theme: Tumour markers

A. Invasive ductal carcinoma of the breast


B. Prostate cancer
C. Gastric cancer
D. Ovarian cancer
E. Colorectal cancer
204
F. Pancreatic adenocarcinoma
G.Seminoma testicular cancer
H.Non-seminomatous testicular cancer
I. Hepatocellular carcinoma

For each tumour marker please select the most likely underlying malignancy. Each option may be used once, more than once or
not at all.

69. Raised beta-human chorionic gonadotropin with a raised alpha-feto protein level
Answer: Non-seminomatous testicular cancer
A raised alpha-feto protein level excludes a seminoma

70. Elevated CA 19-9


Answer: Pancreatic adenocarcinoma

71. Raised alpha-feto protein level in a 54-year-old woman


Answer: Hepatocellular carcinoma

Tumour markers may be divided into: monoclonal antibodies against carbohydrate or glycoprotein tumour antigens. Tumour
antigens. enzymes (alkaline phosphatase, neurone specific enolase). Hormones (e.g. calcitonin, ADH)
It should be noted that tumour markers usually have a low specificity
Monoclonal antibodies: CA 125: Ovarian cancer. CA 19-9: Pancreatic cancer. CA 15-3: Breast cancer. NB: The breast cancer
tumour marker is not specific or sensitive enough to be used routinely.
Tumour antigens: Prostate specific antigen (PSA): Prostatic carcinoma. Alpha-feto protein (AFP): Hepatocellular carcinoma,
teratoma. Carcinoembryonic antigen (CEA):Colorectal cancer

Theme: Benign breast lesions

A. Fibroadenoma
B. Breast abscess
C. Cyst of Montgomery's gland
D. Galactocele
E. Lipoma
F. Duct ectasia
G.Intraductal papilloma
H.Fat necrosis

What is the most likely diagnosis for the scenario given? Each option may be used once, more than once or not at all.

72. A 64 year old obese female presents with a breast lump. She was hit on the breast by a cricket ball when playing with her
grandson.
Answer: Fat necrosis
An obese, post menopausal woman, with a history of trauma points towards fat necrosis. Trauma causes inflammation of fat
cells, leading to formation of a lump. Mammography will be needed to differentiate it from breast disease.

73. A 21 year old female notices a bloody discharge from the nipple. She is otherwise well. On examination there are no
discrete lesions to feel and mammography shows dense breast tissue but no mass lesion.
Answer: Intraductal papilloma
Intraductal papillomata are the commonest cause of blood stained nipple discharge in younger women. There is seldom any
palpable mass. An ultrasound is required and possibly a galactogram.

74. A 18 year old female notices a non tender mobile breast lump. Clinically there is a smooth lump which is not tethered to
the skin.
Answer: Fibroadenoma
Also called a breast 'mouse' due to its mobility. It is a benign condition arising from the breast lobule. May enlarge in
pregnancy.

Benign Breast lesions


Fibroadenoma: Develop from a whole lobule. Mobile, firm breast lumps. 12% of all breast masses. Over a 2 year period up to
30% will get smaller. No increase in risk of malignancy. Treatment: If >3cm surgical excision is usual, Phyllodes tumours
should be widely excised (mastectomy if the lesion is large)

205
Breast cyst: 7% of all Western females will present with a breast cyst. Usually presents as a smooth discrete lump (may be
fluctuant). Small increased risk of breast cancer (especially if younger). Treatment: Cysts should be aspirated, those which are
blood stained or persistantly refill should be biopsied or excised
Sclerosing adenosis, (radial scars and complex sclerosing lesions):Usually presents as a breast lump or breast pain. Causes
mammographic changes which may mimic carcinoma. Cause distortion of the distal lobular unit, without hyperplasia (complex
lesions will show hyperplasia). Considered a disorder of involution, no increase in malignancy risk. Treatment: Lesions should
be biopsied, excision is not mandatory
Epithelial hyperplasia:Variable clinical presentation ranging from generalised lumpiness through to discrete lump. Disorder
consists of increased cellularity of terminal lobular unit, atypical features may be present. Atypical features and family history
of breast cancer confers greatly increased risk of malignancy. Treatment :If no atypical features then conservative, those with
atypical features require either close monitoring or surgical resection
Fat necrosis: Up to 40% cases usually have a traumatic aetiology. Physical features usually mimic carcinoma. Mass may
increase in size initially. Treatment: Imaging and core biopsy
Duct papilloma: Usually present with nipple discharge. Large papillomas may present with a mass. The discharge usually
originates from a single duct. Treatment: No increase risk of malignancy: Microdochectomy

75. A 17 year old man is identified as having a Meckels diverticulum. From which of the following embryological structures
is it derived?
A. Foregut
B. Hindgut
C. Uranchus
D. Cloaca
E. Vitello-intestinal duct

Answer: E
Rule of 2's: 2% of population. 2 inches (5cm) long. 2 feet (60 cm) from the ileocaecal valve. 2 x's more common in men. 2
tissue types involved

The Meckels diverticulum is a persistence of the vitello-intestinal duct. Meckel's diverticulum: Congenital abnormality
resulting in incomplete obliteration of the vitello-intestinal duct. Normally, in the foetus, there is an attachment between the
vitello-intestinal duct and the yolk sac.This disappears at 6 weeks gestation. The tip is free in majority of cases. Associated with
enterocystomas, umbilical sinuses, and omphaloileal fistulas. Arterial supply: omphalomesenteric artery. 2% of population, 2
inches long, 2 feet from the ileocaecal valve. Typically lined by ileal mucosa but ectopic gastric mucosa can occur, with the risk
of peptic ulceration. Pancreatic and jejunal mucosa can also occur. Clinical: Normally asymptomatic and an incidental finding.
Complications are the result of obstruction, ectopic tissue, or inflammation. Removal if narrow neck or symptomatic. Options
are between wedge excision or formal small bowel resection and anastomosis.

Theme: Rectal bleeding

A. Solitary rectal ulcer syndrome


B. Haemorroidal disease
C. Fissure in ano
D. Fistula in ano
E. Anal cancer
F. Ulcerative colitis

Please select the most likely diagnosis for the scenario given. Each option may be used once, more than once or not at all.

76. A 22 year old man presents with a 6 day history of passage of bloody diarrhoea with passage of mucous and slime. He is
passing an average of 8 to 9 bowel movements per day. On digital rectal examination there is no discrete abnormality to
feel, but there is some blood stained mucous on the glove.
Answer: Ulcerative colitis
The passage of bloody diarrhoea together with mucous and a short history makes this a likely first presentation of inflammatory
bowel disease.

77. A 17 year old man presents with a 2 week history of significant pain on defecation accompanied by the presence of a
small amount of blood which is noticed on toilet paper.
Answer: Fissure in ano
Young patients with painful rectal bleeding may have a fissure. Treatment is with stool softeners and either GTN or Diltiazem
initially.

206
78. A 24 year old women presents with a long history of obstructed defecation and chronic constipation. She often strains to
open her bowels for long periods and occasionally notices that she has passed a small amount of blood. On examination
she has an indurated area located anteriorly approximately 3cm proximal to the anal verge.
Answer: Solitary rectal ulcer syndrome
Solitary rectal ulcers are associated with chronic constipation and straining. It will need to be biopsied to exclude malignancy
(the histological appearances are characteristic). Diagnostic work up should include endoscopy and probably defecating
proctogram and ano-rectal manometry studies.

Rectal bleeding: is a common cause for patients to be referred to the surgical clinic. In the clinical history it is useful to try and
localise the anatomical source of the blood. Bright red blood is usually of rectal anal canal origin, whilst dark red blood is more
suggestive of a proximally sited bleeding source. Blood which has entered the GI tract from a gastro-duodenal source will
typically resemble malaena due to the effects of the digestive enzymes on the blood itself.

In the table below we give some typical bleeding scenarios together with physical examination findings and causation.
Type of
Cause Features in history Examination findings
bleeding
Painful bleeding that occurs post Muco-epithelial defect usually in the
Fissure in Bright red
defecation in small volumes. Usually midline posteriorly (anterior fissures more
ano rectal bleeding
antecedent features of constipation likely to be due to underlying disease)
Post defecation bleeding noted both on
toilet paper and drips into pan. May be Normal colon and rectum. Proctoscopy
Bright red
Haemorroids alteration of bowel habit and history of may show internal haemorrhoids. Internal
rectal bleeding
straining. No blood mixed with stool. No haemorrhoids are usually impalpable.
local pain.
Bleeding that is accompanied by other Perineal inspection may show fissures or
Crohns Bright red or symptoms such as altered bowel habit, fistulae. Proctoscopy may demonstrate
disease mixed blood malaise, history of fissures (especially indurated mucosa and possibly strictures.
anterior) and abscesses. Skip lesions may be noted at colonoscopy.
Bright red Proctitis is the most marked finding. Peri
Ulcerative bleeding often Diarrhoea, weight loss, nocturnal anal disease is usually absent.
colitis mixed with incontinence, passage of mucous PR. Colonoscopy will show continuous
stool mucosal lesion.
Usually obvious mucosal abnormality.
Bright red Alteration of bowel habit. Tenesmus may Lesion may be fixed or mobile depending
Rectal cancer blood mixed be present. Symptoms of metastatic upon disease extent. Surrounding mucosa
volumes disease. often normal, although polyps may be
present.

Investigation: All patients presenting with rectal bleeding require digital rectal examination and procto-sigmoidoscopy as a
minimal baseline. Remember that haemorrhoids are typically impalpable and to attribute bleeding to these in the absence of
accurate internal inspection is unsatisfactory. In young patients with no other concerning features in the history a carefully
performed sigmoidoscopy that demonstrates clear haemorrhoidal disease may be sufficient. If clear views cannot be obtained
then patients require bowel preparation with an enema and a flexible sigmoidscopy performed. In those presenting with features
of altered bowel habit or suspicion of inflammatory bowel disease a colonoscopy is the best test. Patients with excessive pain
who are suspected of having a fissure may require an examination under general or local anaesthesia. In young patients with
external stigmata of fissure and a compatible history it is acceptable to treat medically and defer internal examination until the
fissure is healed. If the fissure fails to heal then internal examination becomes necessary along the lines suggested above to
exclude internal disease.

Special tests: In patients with a malignancy of the rectum the staging investigations comprise an MRI of the rectum to identify
circumferential resection margin compromise and to identify mesorectal nodal disease. In addition to this CT scanning of the
chest abdomen and pelvis is necessary to stage for more distant disease. Some centres will still stage the mesorectum with endo
rectal ultrasound but this is becoming far less common. Patients with fissure in ano who are being considered for surgical
sphincterotomy and are females who have an obstetric history should probably have ano rectal manometry testing performed
together with endo anal ultrasound. As this service is not universally available it is not mandatory but in the absence of such
information there are continence issues that may arise following sphincterotomy.

Disease Management
Fissure in ano GTN ointment 0.2% or diltiazem cream applied topically is the usual first line treatment. Botulinum
207
toxin for those who fail to respond. Internal sphincterotomy for those who fail with botox, can be
considered at the botox stage in males.
Haemorroids Lifestyle advice, for small internal haemorrhoids can consider injection sclerotherapy or rubber band
ligation. For external haemorrhoids consider haemorrhoidectomy. Modern options include HALO
procedure and stapled haemorrhoidectomy.
Inflammatory Medical management- although surgery may be needed for fistulating Crohns (setons).
bowel disease
Rectal cancer Anterior resection or abdomino-perineal excision of the colon and rectum. Total mesorectal excision is
now standard of care. Most resections below the peritoneal reflection will require defunctioning
ileostomy. Most patients will require preoperative radiotherapy.

79. Which of the following is the most common childhood brain tumour?
A. Glioblastoma multiforme
B. Astrocytoma
C. Medulloblastoma
D. Ependymoma
E. Meningioma

Answer: C
Glioblastoma multiforme is rare in childhood. In contrast, medulloblastoma (more correctly termed primitive neuroectodermal
tumours) is the commonest brain tumour in children, and the 2nd commonest malignant solid neoplasm in children.

CNS tumours: 60% = Glioma and metastatic disease. 20% = Meningioma. 10% = Pituitary lesions. In paediatric practice
medulloblastomas (neuroectodermal tumours) are the commonest lesion, they are very rare in adults. Tumours arising in right
temporal and frontal lobe may reach considerable size before becoming symptomatic. Whereas tumours in the speech and visual
areas will typically produce early symptoms. Diagnosis : MRI Scanning provides the best resolution. Treatment: Usually
surgery, even if tumour cannot be completely resected conditions such as rising ICP can be addressed with tumour debulking
and survival and quality of life prolonged. Curative surgery can usually be undertaken with lesions such as meningiomas.
Gliomas have a marked propensity to invade normal brain and resection of these lesions is nearly always incomplete.

80. A keen surgical trainee is about to embark on her first hemi arthroplasty for a fractured neck of femur. In the
anaesthetic room the patient is given 1.2g intravenous co-amoxiclav. There is a possible history of penicillin allergy but
the patient is demented and the history is not checked. The patient then develops severe respiratory compromise and
haemodynamic collapse. Which of the following pathological processes accounts for this event?
A. Binding of the drug to circulating IgG class antibodies
B. Recognition of the drug by IgE receptors on mast cells
C. Drug initiated formation of hapten-protein complexes
D. Binding of the drug to circulating IgM class antibodies
E. None of the above

Answer: B
Anaphylactic shock: Antigen recognised by IgE molecules on the surface of mast cells resulting in rapid degranulation with
release of histamine and other inflammatory cytokines. This is a case of anaphylactic shock. In anaphylaxis the mast cells
degranulate.
Anaphylactic shock: Suspect if there has been exposure to an allergen

Management: Remove allergen. ABCD. Drugs: Adrenaline 1:1000 0.5ml INTRAMUSCULARLY (not IV). Repeat after 5
mins if no response. Then Chlorpheniramine 10mg IV. Then Hydrocortisone 100-200mg IV

81. Which of the following hepatobiliary disorders are most classically associated with ulcerative colitis?
A. Gallstones
B. Primary sclerosing cholangitis
C. Bile duct stones
D. Liver hamartomas
E. Hepatocellular carcinoma

Answer: B

Primary sclerosing cholangitis is an idiopathic inflammation of the bile ducts. It may result in episodes of cholestasis and
cholangitis and ultimately result in the need for liver transplantation. It carries a 10% risk of malignant transformation. Crohns

208
disease is associated with gallstones due to impaired entero-hepatic circulation. Apart from PSC, ulcerative colitis does not
increase the risk of other liver lesions.

Ulcerative colitis: Ulcerative colitis is a form of inflammatory bowel disease. Inflammation always starts at rectum, never
spreads beyond ileocaecal valve and is continuous. The peak incidence of ulcerative colitis is in people aged 15-25 years and in
those aged 55-65 years. It is less common in smokers. The initial presentation is usually following insidious and intermittent
symptoms. Features include: bloody diarrhea. Urgency. Tenesmus. Abdominal pain, particularly in the left lower quadrant.

Extra-intestinal features: questions regarding the 'extra-intestinal' features of inflammatory bowel disease are common. Extra-
intestinal features include sclerosing cholangitis, iritis and ankylosing spondylitis. Related to disease activity: Common to
both Crohn’s disease and ulcerative colitis: Arthritis: pauciarticular, asymmetric. Erythema nodosum. Episcleritis. Osteoporosis.
Note: Arthritis is the most common extra-intestinal feature in both CD and UC
Episcleritis is more common in CD. Unrelated to disease activity: Common to both Crohn’s disease and ulcerative colitis:
Arthritis: polyarticular, symmetric. Uveitis. Pyoderma gangrenosum. Clubbing. Primary sclerosing cholangitis. Note: Primary
sclerosing cholangitis is much more common in UC. Uveitis is more common in UC

Pathology: Red, raw mucosa, bleeds easily. No inflammation beyond submucosa (unless fulminant disease). Widespread
superficial ulceration with preservation of adjacent mucosa which has the appearance of polyps ('pseudopolyps'). Inflammatory
cell infiltrate in lamina propria. Neutrophils migrate through the walls of glands to form crypt abscesses. Depletion of goblet
cells and mucin from gland epithelium. Granulomas are infrequent. Barium enema: loss of haustrations. superficial ulceration,
'pseudopolyps'. long standing disease: colon is narrow and short -'drainpipe colon'. Endoscopy: Superficial inflammation of the
colonic and rectal mucosa. Continuous disease from rectum proximally. Superifical ulceration, mucosal islands, loss of vascular
definition and continuous ulceration pattern. Management: Patients with long term disease are at increased risk of development
of malignancy. Acute exacerbations are generally managed with steroids, in chronic patients agents such as azathioprine and
infliximab may be used. Individuals with medically unresponsive disease usually require surgery- in the acute phase a sub total
colectomy and end ileostomy. In the longer term a proctectomy will be required. An ileoanal pouch is an option for selected
patients

82. Which of the following is not associated with thrombosis?


A. Endothelial cell damage
B. Use of tourniquets in surgery
C. Formation of platelet aggregates
D. Thrombocytopenia
E. Carcinoma of the stomach

Answer: D

All the other options either act directly to promote thrombosis e.g. endothelial cell damage or via changes in consistency or

Cause Factors affected Disorder APTT PT Bleeding time


Heparin Prevents activation factors 2,9,10,11 Haemophilia Increased Normal Normal
Warfarin Affects synthesis of factors 2,7,9,10 von Willebrand's Increased Normal Increased
DIC Factors 1,2,5,8,11 disease
Liver disease Factors 1,2,5,7,9,10 Vitamin K deficiency Increased Increased Normal
flow of blood.

83. A 16 year old boy develops a painful swelling of his distal femur. An osteoblastic sarcoma is diagnosed. To which of the
following sites is this lesion most likely to metastasise?
A. Inguinal lymph nodes
B. Common iliac lymph nodes
C. Liver
D. Brain
E. Lung

Answer: E
Sarcomas in which Lymphatic Metastasis is seen:

'RACE For MS': R: Rhabdomyosarcoma. A: Angiosarcoma. C: Clear cell sarcoma. E: Epithelial cell sarcoma. For:
Fibrosarcoma M: Malignant fibrous histiocytoma. S: Synovial cell sarcoma Or 'SCARE': Synovial sarcoma. Clear cell sarcoma.
Angiosarcoma. Rhabdomyosarcoma. Epithelioid sarcoma. Sarcomas often metastasise via the haematogenous route and the
lung is a common site for sarcoma metastasis. The liver and brain are often spared (at least initially). A smaller number may
develop lymphatic metastasis (see above).
209
Sarcomas: Malignant tumours of mesenchymal origin. Types: May be either bone or soft tissue in origin. Bone sarcoma
include: Osteosarcoma. Ewings sarcoma (although non bony sites recognised). Chrondrosarcoma - originate from
Chondrocytes. Soft tissue sarcoma are a far more heterogeneous group and include: Liposarcoma-adipocytes.
Rhabdomyosarcoma-striated muscle. Leiomyosarcoma-smooth muscle. Synovial sarcomas- close to joints (cell of origin not
known but not synovium). Malignant fibrous histiocytoma is a sarcoma that may arise in both soft tissue and bone. Features:
Certain features of a mass or swelling should raise suspicion for a sarcoma these include: Large >5cm soft tissue mass. Deep
tissue location or intra muscular location. Rapid growth. Painful lump. Assessment: Imaging of suspicious masses should utilise
a combination of MRI, CT and USS. Blind biopsy should not be performed prior to imaging and where required should be done
in such a way that the biopsy tract can be subsequently included in any resection.

Ewings sarcoma: Commoner in males. Incidence of 0.3 / 1, 000, 000. Onset typically between 10 and 20 years of age. Location
by femoral diaphysis is commonest site. Histologically it is a small round tumour. Blood borne metastasis is common and
chemotherapy is often combined with surgery

Osteosarcoma: Mesenchymal cells with osteoblastic differentiation. 20% of all primary bone tumours. Incidence of 5 per
1,000,000. Peak age 15-30, commoner in males. Limb preserving surgery may be possible and many patients will receive
chemotherapy

Liposarcoma: Malignancy of adipocytes. Rare approximately 2.5 per 1,000,000. They are the second most common soft tissue
sarcoma. Typically located in deep locations such as retroperitoneum. Affect older age group usually >40 years of age. May be
well differentiated and thus slow growing although may undergo dedifferentiation and disease progression. Many tumours will
have a pseudocapsule that can misleadingly allow surgeons to feel that they can 'shell out' these lesions. In reality tumour may
invade at the edge of the pseudocapsule and result in local recurrence if this strategy is adopted. Usually resistant to
radiotherapy although this is often used in a palliative setting

Malignant Fibrous Histiocytoma: Tumour with large number of histiocytes. Most common sarcoma in adults. Also described
as undifferentiated pleomorphic sarcoma NOS (i.e. Cell of origin is not known). Four major subtypes are recognised: storiform-
pleomorphic (70% cases), myxoid (less aggressive), giant cell and inflammatory. Treatment is usually with surgical resection
and adjuvant radiotherapy as this reduces the likelihood of local recurrence

84. Infection with which of the following micro-organisms may result in a clinical picture resembling achalasia of the
oesphagus?
A. Epstein Barr virus
B. Wuchereria Bancrofti
C. Candida Spp
D. Trypanosoma Cruzi
E. Helicobacter Pylori

Answer: D
Infection with Trypanosoma Cruzi may result in destruction of the ganglion cells of the myenteric plexus, resulting in a clinical
picture similar to achalasia.

Trypanosoma Cruzi: Protozoan. Causes Chagas disease. Carried by bugs which infect the skin whilst feeding. Penetrate
through open wounds and mucous membranes. Intracellular proliferation. Major infective sites include CNS, intestinal
myenteric plexus, spleen, lymph nodes and cardiac muscle. Chronic disease is irreversible, nifurtimox is used to treat acute
infection

85. A 45-year-old man presents to surgical outpatients with a long history of recurrent abdominal pain and vomiting. He is
noted to have a peripheral motor neuropathy on examination. What is the most likely diagnosis?
A. Huntington's disease
B. Myeloma
C. Acute intermittent porphyria
D. Lawrence-Moon-Biedl syndrome
E. Friedreich's ataxia

Answer: C
Neurological signs combined with abdominal pain is acute intermittent porphyria or lead poisoning until proven otherwise.

Acute intermittent porphyria: Acute intermittent porphyria (AIP) is a rare autosomal dominant condition caused by a defect in
porphobilinogen deaminase, an enzyme involved in the biosynthesis of haem. The results in the toxic accumulation of delta
aminolaevulinic acid and porphobilinogen. It characteristically presents with abdominal and neuropsychiatric symptoms in 20-
40 year olds. AIP is more common in females (5:1). Features: Abdominal: abdominal pain, vomiting. Neurological: motor
neuropathy. Psychiatric: e.g. Depression. Hypertension and tachycardia common. Diagnosis: Classically urine turns deep red on

210
standing. Raised urinary porphobilinogen (elevated between attacks and to a greater extent during acute attacks). Assay of red
cells for porphobilinogen deaminase. Raised serum levels of delta aminolaevulinic acid and porphobilinogen

86. A 56 year old man presents with episodic facial pain and discomfort whilst eating. He has suffered from halitosis
recently and he frequently complains of a dry mouth. He has a smooth swelling underneath his right mandible. What is
the most likely underlying diagnosis?
A. Stone impacted in Whartons duct
B. Stone impacted in Stensens duct
C. Benign adenoma of the submandibular gland
D. Adenocarcinoma of the submandibular gland
E. Squamous cell carcinoma of the submandibular gland

Answer: A
The symptoms are typical for sialolithiasis. The stones most commonly form in the submandibular gland and therefore may
occlude Whartons duct. Stensens duct drains the parotid gland.

Submandibular glands- disease: Physiology: The submandibular glands secrete approximately 800- 1000ml saliva per day.
They typically produce mixed seromucinous secretions. When paraympathetic activity is dominant the secretions will be more
serous. The parasympathetic fibres are derived from the chorda tympani nerves and the submandibular ganglion, they travel to
the glands via the lingual nerves.

Sialolithiasis: 80% of all salivary gland calculi occur in the submandibular gland. 70% of the these calculi are radio-opaque.
Stones are usually composed of calcium phosphate or calcium carbonate. Patients typically develop colicky pain and post
prandial swelling of the gland. Investigation involves sialography to demonstrate the site of obstruction and associated other
stones. Stones impacted in the distal aspect of Whartons duct may be removed orally, other stones and chronic inflammation
will usually require gland excision.

Sialadenitis: Usually occurs as a result of Staphylococcus aureus infection. Pus may be seen leaking from the duct, erythema
may also be noted. Development of a sub mandibular abscess is a serious complication as it may spread through the other deep
fascial spaces and occlude the airway

Submandibular tumours: Only 8% of salivary gland tumours affect the sub mandibular gland. Of these 50% are malignant
(usually adenoid cystic carcinoma). Diagnosis usually involves fine needle aspiration cytology. Imaging is with CT and MRI. In
view of the high prevalence of malignancy, all masses of the submandibular glands should generally be excised.

87. Which of the following cellular types or features is not seen in sarcoidosis?
A. Reed Sternberg Cells
B. T lymphocytes
C. Macrophages
D. Asteroid bodies
E. B lymphocytes

Answer: A
Reed Sternberg cells are seen in Hodgkins disease. All of the other cell types are seen in sarcoid.

88. Which of the following diseases is not considered a risk factor for gastric cancer?
A. Polya gastrectomy for antral ulcer
B. Atrophic gastritis
C. Intestinal metaplasia of columnar type at the gastric cardia
D. Patient with polyp showing medium grade dysplasia
E. Long term therapy with H2 blockers

Answer: E
Although some acid lowering procedures increase the risk of gastric cancer the use of H2 blockers does not, at the present time,
seem to increase the risk.

89. A 56 year old man is diagnosed as having a glioma. From which of the following cell types do these tumours usually
originate?
A. Astrocytes
B. Oligodendrocytes
C. Ependymal cells
D. Squamous cells

211
E. Neuroglial cells
Answer:
E
Gliomas originate from glial (otherwise known as neuroglial) cells. These serve a structural function in the CNS. The tumours
produced may resemble a number of CNS cell types. Tumours are therefore named according to the cells they resemble rather
than the origin. Where this is not possible they are termed gliomas.

Glioma: a tumour that is typically found in the CNS. These tumours arise from glial cells. They are sub categorised according
to the cell type they most closely resemble.

Glioma sub types:Ependymomas- Ependymal cells. Astocytomas- Astrocytes (including glioblastoma). Oligodendrogliomas-
Oligodendrocytes. Mixed- e.g. oligoastrocytomas. Gliomas are categorised as being either high or low grade lesions (the former
has the worse prognosis). They may be either supra or infra tentorial. Their symptoms will typically reflect their site of origin.
Glioblastoma multiforme has the worst prognosis and few patients will survive beyond 12 months.

90. A 78 year old man presents with unilateral deafness which has been present for the past 3 months. On examination
Webers test localises to the contralateral side and a CT scan of his head shows a thickened calvarium with areas of
sclerosis and radiolucency. His blood tests show an elevated alkaline phosphatase, normal serum calcium and normal
PTH levels. Which of the following is the most likely underlying diagnosis?
A. Multiple myeloma with skull involvement
B. Osteoporosis
C. Pagets disease with skull involvement
D. Lung cancer with skull metastasis
E. Osteopetrosis with skull involvement

Answer: C
Of the conditions listed Pagets disease is the most likely diagnosis (skull vault expansion and sensorineural hearing loss).
Multiple myeloma would typically result in multiple areas of radiolucency and usually raised calcium in this setting.
Osteopetrosis is a recognised cause of the features described. However, it is a rare inherited disorder and usually presents in
children in young adults. Presentation at this stage with no prior symptoms would be extremely rare and therefore this is not the
most likely diagnosis.

Pagets disease: a disease of increased but uncontrolled bone turnover and is characterised by architecturally abnormal bones. It
is thought to be primarily a disorder of osteoclasts, with excessive osteoclastic resorption followed by increased osteoblastic
activity causing areas of sclerosis and deformity. Paget's disease is common (UK prevalence 5%) but symptomatic in only 1 in
20 patients
Predisposing factors: increasing age. Male sex. Northern latitude. Family history. Clinical features: bone pain (e.g. pelvis,
lumbar spine, femur). Classical, untreated features: bowing of tibia, bossing of skull. Raised alkaline phosphatase (alp) -
calcium* and phosphate are typically normal. Skull x-ray: thickened vault, osteoporosis circumscripta. Indications for
treatment: bone pain, skull or long bone deformity, fracture, periarticular Paget's. bisphosphonate (either oral risedronate or IV
zoledronate). Calcitonin is less commonly used now. Complications: deafness (cranial nerve entrapment). Bone sarcoma (1% if
affected for > 10 years). Fractures. Skull thickening. High-output cardiac failure.
*usually normal in this condition but hypercalcaemia may occur with prolonged immobilisation

Theme: Genetic causes of cancer

A. Multiple endocrine neoplasia type I


B. Multiple endocrine neoplasia type II
C. Gardner's syndrome
D. Lynch Syndrome
E. Kartagener's syndrome
F. Von Recklinghausen's disease

Please select the most likely condition for the disease process described. Each option may be used once, more than once or not
at all

91. A 5 year old boy presents with recurrent episodes of sinusitis. The casualty staff are surprised to find his liver lying in
the left upper quadrant of the abdomen
Answer: Kartagener's syndrome
This is a case of Kartagener's syndrome. The primary problem is of immotile cilia syndrome. When associated with situs
inversus Kartagener's syndrome is diagnosed.

212
92. A 22 year old man presents with carcinoma of the caecum. His brother died from colorectal cancer aged 25 and his
father died from the disease aged 30.
Answer: Lynch Syndrome
This is a case of Lynch syndrome HNPCC. It is transmitted in an autosomal dominant fashion.

93. A tall 32 year old lady presents with a diffuse neck swelling a carcinoma of the thyroid medullary type is diagnosed.
Answer: Multiple endocrine neoplasia type II
This is a case MEN type IIb. It is associated with phaeochromocytomas and is transmitted in an autosomal dominant pattern if
inherited. All MEN II tend to have medullary carcinoma of the thyroid as a presenting feature

94. A 45 year old man presents with symptoms of urinary colic. In the history he has suffered from recurrent episodes of
frank haematuria over the past week or so. On examination he has a left loin mass and a varicocele. The most likely
diagnosis is:
A. Renal adenocarcinoma
B. Renal cortical adenoma
C. Squamous cell carcinoma of the renal pelvis
D. Retroperitoneal fibrosis
E. Nephroblastoma
Answer: A
Renal adenocarcinoma are the most common renal malignancy and account for 75% cases.
Patients may develop frank haematuria and have episodes of clot colic.A Grawitz tumour is an eponymous name for Renal
Adenocarcinoma. May metastasise to bone.

Renal cell carcinoma


Renal cell carcinoma is an adenocarcinoma of the renal cortex and is believed to arise from the proximal convoluted tubule. They
are usually solid lesions, up to 20% may be multifocal, 20% may be calcified and 20% may have either a cystic component or be
wholly cystic. They are often circumscribed by a pseudocapsule of compressed normal renal tissue. Spread may occur either by
direct extension into the adrenal gland, renal vein or surrounding fascia. More distant disease usually occurs via the
haematogenous route to lung, bone or brain. Renal cell carcinoma comprise up to 85% of all renal malignancies. Males are more
commonly affected than females and sporadic tumours typically affect patients in their sixth decade. Patients may present with a
variety of symptoms including; haematuria (50%), loin pain (40%), mass (30%) and up to 25% may have symptoms of
metastasis.Less than 10% have the classic triad of haematuria, pain and mass.

Investigation Many cases will present as haematuria and be discovered during diagnostic work up. Benign renal tumours are rare,
so renal masses should be investigated with multislice CT scanning. Some units will add and arterial and venous phase to the scan
to demonstrate vascularity and evidence of caval ingrowth. CT scanning of the chest and abdomen to detect distant disease should
also be undertaken. Routine bone scanning is not indicated in the absence of symptoms. Biopsy should not be performed when a
nephrectomy is planned but is mandatory before any ablative therapies are undertaken. Assessment of the functioning of the
contra lateral kidney.

Management: T1 lesions may be managed by partial nephrectomy and this gives equivalent oncological results to total radical
nephrectomy. Partial nephrectomy may also be performed when there is inadequate reserve in the remaining kidney. For T2
lesions and above a radical nephrectomy is standard practice and this may be performed via a laparoscopic or open approach.
Preoperative embolisation is not indicated nor is resection of uninvolved adrenal glands. During surgery early venous control is
mandatory to avoid shedding of tumour cells into the circulation. Patients with completely resected disease do not benefit from
adjuvant therapy with either chemotherapy or biological agents. These should not be administered outside the setting of clinical
trials. Patients with transitional cell cancer will require a nephroureterectomy with disconnection of the ureter at the bladder.

95. A 63 year old man finds that he has to stop walking after 100 yards due to bilateral calf pain. He finds that bending
forwards and walking up hill helps. He is able to ride a bike without any pain. What is the most likely underlying cause?
A. Lumbar canal stenosis
B. Diabetic neuropathy
C. Aorto-iliac occlusion
D. Occlusion of the superficial femoral artery
E. Pelvic rheumatoid arthritis

Answer: A
The positional nature of the pain and the fact that improves with walking uphill makes an underlying vascular aetiology far less
likely.

Lumbar spinal stenosis

213
Lumbar spinal stenosis is a condition in which the central canal is narrowed by tumour, disk prolapse or other similar
degenerative changes. Patients may present with a combination of back pain, neuropathic pain and symptoms mimicking
claudication. One of the main features that may help to differentiate it from true claudication in the history is the positional
element to the pain. Sitting is better than standing and patients may find it easier to walk uphill rather than downhill. The
neurogenic claudication type history makes lumbar spinal stenosis a likely underlying diagnosis, the absence of such symptoms
makes it far less likely.
Pathology: Degenerative disease is the commonest underlying cause. Degeneration is believed to begin in the intervertebral
disk where biochemical changes such as cell death and loss of proteoglycan and water content lead to progressive disk bulging
and collapse. This process leads to an increased stress transfer to the posterior facet joints, which accelerates cartilaginous
degeneration, hypertrophy, and osteophyte formation; this is associated with thickening and distortion of the ligamentum
flavum. The combination of the ventral disk bulging, osteophyte formation at the dorsal facet, and ligamentum flavum
hyptertrophy combine to circumferentially narrow the spinal canal and the space available for the neural elements. The
compression of the nerve roots of the cauda equina leads to the characteristic clinical signs and symptoms of lumbar spinal
stenosis.
Diagnosis: MRI scanning is the best modality for demonstrating the canal narrowing. Historically a bicycle test was used as true
vascular claudicants could not complete the test.Treatment: Laminectomy

96. A 73 year old lady is admitted for a laparoscopic cholecystectomy. During her pre-operative assessment it is noted that
she is receiving furosemide for the treatment of hypertension. Approximately what proportion of the sodium that is
filtered at the glomerulus will be subsequently excreted?
A. Up to 25%
B. Upt to 75%
C. Between 3 and 5%
D. <2%
E. Between 1 and 2%

Answer: A
The loop diuretics can lead to marked increases in the amount of sodium excreted. They act in the medullary and cortical
aspects of the thick ascending limb of the loop of Henle. This results in a decreased medullary osmolal gradient and increases
free water excretion (as well as loss of sodium). Because loop diuretics result in the loss of both sodium and water they are less
frequently associated with hyponatraemia than thiazide diuretics (these latter agents act in the cortex and do not affect urine
concentrating ability).

Diuretic agents: The diuretic drugs are divided into three major classes, which are distinguished according to the site at which
they impair sodium reabsorption: loop diuretics in the thick ascending loop of Henle, thiazide type diuretics in the distal tubule
and connecting segment; and potassium sparing diuretics in the aldosterone - sensitive principal cells in the cortical collecting
tubule.
In the kidney, sodium is reabsorbed through Na+/ K+ ATPase pumps located on the basolateral membrane. These pumps return
reabsorbed sodium to the circulation and maintain low intracellular sodium levels. This latter effect ensures a constant
concentration gradient.

Physiological effects of commonly used diuretics


Site of action Diuretic Carrier or channel Percentage of filtered sodium
inhibited excreted
Ascending limb of loop of Henle Frusemide Na+/K+ 2Cl - carrier Upt to 25%
Distal tubule and connecting Thiazides Na+Cl- carrier Between 3 and 5%
segment
Cortical collecting tubule Spironolactone Na+ channel Between 1 and 2%

97. A 59 year old man presents with recurrent episodes of urinary sepsis. In his history he mentions that he has suffered
from recurrent attacks of left iliac fossa pain over the past few months. He has also notices bubbles in his urine. He
undergoes a CT scan which shows a large inflammatory mass in the left iliac fossa. No other abnormality is detected.
The most likely diagnosis is:
A. Ulcerative colitis
B. Crohns disease
C. Mesenteric ischaemia
D. Diverticular disease
E. Rectal cancer

Answer: D
Recurrent attacks of diverticulitis may cause the development of local abscesses which may erode into the bladder resulting in
urinary sepsis and pneumaturia. This would be an unusual presentation from Crohns disease and rectal cancer would be more
distally sited and generally evidence of extra colonic disease would be present if the case were malignant and this advanced.

214
Diverticular disease: Diverticular disease is a common surgical problem. It consists of herniation of colonic mucosa through
the muscular wall of the colon. The usual site is between the taenia coli which vessels pierce the muscle to supply the mucosa.
Symptoms: Altered bowel habit Bleeding. Abdominal pain. Complications: Diverticulitis. Haemorrhage. Development of
fistula Perforation and faecal peritonitis. Perforation and development of abscess. Development of diverticular phlegmon
Diagnosis: Patients presenting in clinic will typically undergo either a colonoscopy or barium enema as part of their diagnostic
work up. Both tests will identify diverticular disease. It can be far more difficult to confidently exclude cancer, particularly in
diverticular strictures. Acutely unwell surgical patients should be investigated in a systematic way. Plain abdominal films and an
erect chest x-ray will identify perforation. An abdominal CT scan with oral and intravenous contrast will help to identify
whether acute inflammation is present but also the presence of local complications such as abscess formation.
Severity Classification- Hinchey: I: Para-colonic abscess. II: Pelvic abscess. III: Purulent peritonitis. IV: Faecal peritonitis
Treatment: Increase dietary fibre intake. Mild attacks of diverticulitis may be managed conservatively with antibiotics.. Peri
colonic abscesses should be drained either surgically or radiologically. Recurrent episodes of acute diverticulitis requiring
hospitalisation are a relative indication for a segmental resection. Hinchey IV perforations (generalised faecal peritonitis) will
require a resection and usually a stoma. This group have a very high risk of post operative complications and usually require
HDU admission. Less severe perforations may be managed by laparoscopic washout and drain insertion.

98. A 78 year old man is referred to the clinic by his general practitioner. For many years he noticed a smooth swelling
approximately 5cm anterior to the tragus of his right ear. Apart from being a heavy smoker he has no co-morbidities.
What is the most likely diagnosis?
A. Pleomorphic adenoma
B. Liposarcoma
C. Warthins tumour
D. Adenocarcinoma
E. None of the above

Answer: C
Warthins tumours are most common in elderly smokers. They have a relatively benign and indolent course. They are usually
well circumscribed as illustrated below:

Parotid gland clinical: Causes of bilateral parotid enlargement : Mumps: Associated with meningoencephalitis, pancreatitis,
orchitis, or deafness. Parotitis. Sialectasis - especially if related to eating. Sjogren's syndrome: dry eyes or mouth, connective
tissue disease. Sarcoidosis. Tuberculosis. Alcoholism. Myxoedema. Cushing's disease. Diabetes/insulin resistance. Liver
cirrhosis. Gout. Bulimia nervosa. Drugs. Severe dehydration. Malnutrition

Causes of unilateral parotid enlargement: Salivary calculus. Tumour

Parotid gland tumours: Pleomorphic adenomas are the most common. Incisional biopsy of parotid masses is not
recommended, so superficial parotidectomy is the usual procedure of choice. Signs of facial nerve palsy and a parotid mass
should raise suspicion of malignancy. Warthins tumours are relatively benign lesions that are slow growing and occur most
commonly in elderly male smokers.. Adenoid cystic carcinoma have a tendency for perineural invasion.

Theme: Lung cancer

A. Adenocarcinoma
B. Small cell lung cancer
C. Large cell lung cancer
D. Squamous cell carcinoma

Please select the most likely lung cancer variant for the scenario described. Each option may be used once, more than once or
not at all.

99. A 73 year old heavy smoker presents with haemoptysis. On examination he is cachectic and shows evidence of clubbing.
Imaging shows a main bronchial tumour with massive mediastinal lymphadenopathy together with widespread visceral
metastases.
Answer: Small cell lung cancer
Small cell carcinoma is associated with disseminated disease at presentation in the majority of cases. Most cases occur in the
main airways and paraneoplastic features are common.

215
100.A 68 year old female who has never smoked presents with a mass at the periphery of her right lung.
Answer: Adenocarcinoma
Adenocarcinomas are the most common tumour type present in never smokers. They are usually located at the periphery.

101.An 85 year old man presents with a cough and haemoptysis. He has a modest smoking history of 15 pack years. He is
found to have a tumour located in the right main bronchus, with no evidence of metastatic disease. He decides no
undergo any treatment and he remains well for a further 12 months before developing symptomatic metastasis.
Answer: Squamous cell carcinoma
Squamous cell carcinomas are reported to be more slow growing and are typically centrally located. Small cell carcinomas are
usually centrally located. However, small cell carcinomas would seldom be associated with a survival of a year without
treatment.

Lung cancers may be classified according to histological subtypes. The main distinction is between small cell and non small cell
lung cancer. Non small cell lung cancer is the most common variant and accounts for 80% of all lung cancers.
Non small cell lung cancer: These share common features of prognosis and management. They comprise the following
tumours: Squamous cell carcinoma (25% cases). Adenocarcinoma (40% cases). Large cell carcinoma (10% cases)
Paraneoplastic features and early disease dissemination are less likely than with small cell lung carcinoma. Adenocarcinoma is
the most common lung cancer type encountered in never smokers.

Small cell lung carcinoma: Small cell lung carcinomas are comprised of cells with a neuro endocrine differentiation. The
neuroendocrine hormones may be released from these cells with a wide range of paraneoplastic associations. These tumours are
strongly associated with smoking and will typically arise in the larger airways. They disseminate early in the course of the
disease and although they are usually chemosensitive this seldom results in long lasting remissions.

102.Which of the following cells is not found on a blood film post splenectomy?
A. Pappenheimer bodies
B. Stipple cells
C. Erythrocyte containing siderotic granules
D. Howell-Jolly bodies
E. Target cells

Answer: B

Stipple cells are found in lead poisoning/haemoglobinopathies.


Blood film in hyposplenism:
Howell-Jolly bodies. Pappenheimer bodies. Poikilocytes (Target cells). Erythrocyte containing siderotic granules. Heinz bodies

103.A 45 year old man with long standing ulcerative colitis and rectal dysplasia presents with a DALM lesion in the rectum.
What is the most appropriate management option?
A. Snare polypectomy
B. Repeat endoscopy in 2 years
C. Discharge
D. Anterior resection
E. Panproctocolectomy

Answer: E
DALM lesions complicating ulcerative colitis should be managed with panproctocolectomy. An anterior resection is inadequate
since it will only remove the rectum and ulcerative colitis affects the entire colon. Since many will be associated with invasion a
snare polypectomy is not sufficient either.

Colonic lesions – DALM: The term DALM lesion refers to a Dysplasia Associated Lesion or Mass. They may complicate
dysplasia occurring in patients with longstanding ulcerative colitis. They have a high incidence of invasive foci.When they
complicate longstanding ulcerative colitis, they should be treated by panproctocolectomy.

104.Which of the metastatic bone tumours described below is at the greatest risk of pathological fracture ?
A. Proximal humeral lesion from a prostate cancer
B. Vertebral body lesions from a prostate cancer
C. Peritrochanteric lesion from a carcinoma of the breast
D. Proximal humeral lesion from a carcinoma of the breast
E. Peritrochanteric lesion from a prostate cancer

Ansewr: C
216
Peritrochanteric lesions have the greatest risks of fracture (due to loading). The lesions from breast cancer are usually lytic and
therefore at higher risk rather than the sclerotic lesions from prostate cancer.

Metastatic bone disease- risk of fracture: Metastatic bone tumours may be described as blastic, lytic or mixed. Osteoblastic
metastatic disease has the lowest risk of spontaneous fracture when compared to osteolytic lesions of a similar size.
Lesions affecting the peritrochanteric region are most prone to spontaneous fracture (because of loading forces at that site).
The factors are incorportated into the Mirel Scoring system to stratify the risk of spontaneous fracture for bone metastasis of
varying types.

Mirel Scoring system


Score points Site Radiographic appearance Width of bone involved Pain
1 Upper extremity Blastic Less than 1/3 Mild
2 Lower extremity Mixed 1/3 to 2/3 Moderate
3 Peritrochanteric Lytic More than 2/3 Aggravated by function

Depending upon the score the treatment should be as follows:

Score Risk of fracture Treatment


9 or greater Impending (33%) Prophylactic fixation
8 Borderline Consider fixation
7 or less Not impending (4%) Non operative management

105.A 63 year old male presents with several episodes of haematuria. He suffers from COPD secondary to long term
smoking. Which is the most likely underlying cause?
A. Renal cortical adenoma
B. Renal adenocarcinoma
C. Nephroblastoma
D. Transitional cell carcinoma of the bladder
E. Adenocarcinoma of the bladder

Answer: D

TCC is the most common subtype and is strongly linked to smoking. The important point to note in this question is the term
most likely as renal adenocarcinoma may produce similar symptoms but is less likely.

Bladder cancer is the second most common urological cancer. It most commonly affects males aged between 50 and 80 years
of age. Those who are current, or previous (within 20 years), smokers have a 2-5 fold increased risk of the disease. Occupational
exposure to hydrocarbons such as alanine increases the risk. Although rare in the UK, chronic bladder inflammation arising
from Schistosomiasis infection remains a common cause of squamous cell carcinomas, in those countries where the disease is
endemic. Benign tumours of the bladder including inverted urothelial papilloma and nephrogenic adenoma are uncommon.

Bladder malignancies: Transitional cell carcinoma (>90% of cases). Squamous cell carcinoma ( 1-7% -except in regions
affected by schistosomiasis). Adenocarcinoma (2%). Transitional cell carcinomas may arise as solitary lesions, or may be
multifocal, owing to the effect of "field change" within the urothelium. Up to 70% of TCC's will have a papillary growth
pattern. These tumours are usually superficial in location and accordingly have a better prognosis. The remaining tumours show
either mixed papillary and solid growth or pure solid growths. These tumours are typically more prone to local invasion and
may be of higher grade, the prognosis is therefore worse. Those with T3 disease or worse have a 30% (or higher) risk of
regional or distant lymph node metastasis.

Staging Stage Description


Most will undergo a cystoscopy and biopsies or TURBT, this T0 No evidence of tumour
provides histological diagnosis and information relating to Ta Non invasive papillary carcinoma
depth of invasion. Locoregional spread is best determined using T1 Tumour invades sub epithelial connective tissue
pelvic MRI and distant disease CT scanning. Nodes of T2a Inner half of detrusor invaded
uncertain significance may be investigated using PET CT. T2b Outer half of detrusor invaded
T3 Tumour extends to perivesical fat
Presentation T4a Invasion of uterus, prostate or bowel
Most patients (85%) will present with painless, macroscopic T4b Invasion of other abdominal organs
haematuria. In those patients with incidental microscopic
N0 No nodal disease
haematuria, up to 10% of females aged over 50 will be found to
N1 Single lymph node metastasis (up to 2cm)
have a malignancy (once infection excluded).
N2 Single node >2cm or multiple nodes up to 5cm
N3 Nodes over 5cm
Treatment
Those with superficial lesions may be managed using TURBT M1 Distant disease
217
in isolation. Those with recurrences or higher grade/ risk on histology may be offered intravesical chemotherapy. Those with T2
disease are usually offered either surgery (radical cystectomy and ileal conduit) or radical radiotherapy.
Prognosis: T1: 90%. T2: 60%. T3: 35%. T4a: 10-25%. Any T, N1-N2: 30%

Theme: Neck lumps

A. Cystic hygroma
B. Bartonella infection
C. Mycobacterium tuberculosis infection
D. Branchial cyst
E. Thyroglossal cyst
F. Pharyngeal pouch
G.Follicular thyroid cyst
H.Parathyroid adenoma
I. None of the above

Please select the most likely underlying disease process for the scenario given. Each option may be used once, more than once
or not at all.

106.A 25 year old cat lover presents with symptoms of abdominal pain, lethargy and sweats. These have been present for the
past two weeks. On examination she has lymphadenopathy in the posterior triangle.
Answer: Bartonella infection
Bartonella infection may occur following a cat scratch. The organism is intracellular. Generalised systemic symptoms may
occur for a week or so prior to clinical presentation.

107.A 25 year old lady presents with an swelling located at the anterior border of the sternocleidomastoid muscle. The
swelling is intermittent and on examination it is soft and fluctuant.
Answer: Branchial cyst
Branchial cysts are remnants of the branchial cleft. They may become infected.

108.A 38 year old lady presents with a mass in the midline of the neck immediately below the hyoid bone. It moves upwards
on tongue protrusion.
Answer: Thyroglossal cyst
Thyroglossal cysts are usually located in the midline and are linked to the foramen caecum and will thus move upwards on
tongue protrusion.

109. A 22 year old man presents with a discharging area on his lower back. On examination there is an epithelial defect
located 6cm proximal to the tip of his coccyx and located in the midline. There are two further defects located about 2cm
superiorly in the same position. He is extremely hirsute. What is the most likely diagnosis?
A. Pre sacral tumour
B. Sacrococcygeal teratoma
C. Pilonidal sinus
D. Fistula in ano
E. Occult spina bifida

Answer: C
Pilonidal sinuses are extremely common in hirsute individuals and typically present as midline sinuses in the natal cleft.

Pilonidal sinus: Occur as a result of hair debris creating sinuses in the skin (Bascom theory). Usually in the natal cleft of male
patients after puberty. It is more common in Caucasians related to their hair type and growth patterns. The opening of the sinus
is lined by squamous epithelium, but most of its wall consists of granulation tissue. Up to 50 cases of squamous cell carcinoma
has been described in patients with chronic pilonidal sinus disease. Hairs become trapped within the sinus. Clinically the sinus
presents when acute inflammation occurs, leading to an abscess. Patients may describe cycles of being asymptomatic and
periods of pain and discharge from the sinus. Treatment is difficult and opinions differ. Definitive treatment should never be
undertaken when acute infection or abscess is present as this will result in failure. Definitive treatments include the Bascom
procedure with excision of the pits and obliteration of the underlying cavity. The Karydakis procedure involves wide excision of
the natal cleft such that the surface is recontoured once the wound is closed. This avoids the shearing forces that break off the
hairs and has reasonable results.

218
110.A 43 year old man from Greece presents with colicky right upper quadrant pain, jaundice and a pruritic, erythematous
rash over his body. He is initially treated with ciprofloxacin, but does not improve. What is the most likely diagnosis?
A. Infection with Wucheria bancrofti
B. Infection with Echinococcus granulosus
C. Type III hypersentivity reaction
D. Allergy to ciprofloxacin
E. Common bile duct stones
Answer: B
Infection with Echinococcus granulosus will typically produce a type I hypersensitivity reaction which is characterised by a
urticarial rash. With biliary rupture a classical triad of biliary colic, jaundice and urticaria occurs. Whilst jaundice and biliary
colic may be a feature of CBD stones they do not produce an urticarial rash. Antibiotic sensitivity with ciprofloxacin may
produce jaundice and a rash, however it was not present at the outset and does not cause biliary colic.

Hydatid cysts: Hydatid cysts are endemic in Mediterranean and Middle Eastern countries. They are caused by the tapeworm
parasite Echinococcus granulosus. An outer fibrous capsule is formed containing multiple small daughter cysts. These cysts are
allergens which precipitate a type 1 hypersensitivity reaction.

Clinical features are as follows: Up to 90% cysts occur in the liver and lungs. Can be asymtomatic, or symptomatic if cysts >
5cm in diameter. Morbidity caused by cyst bursting, infection and organ dysfunction (biliary, bronchial, renal and cerebrospinal
fluid outflow obstruction). In biliary ruputure there may be the classical triad of; biliary colic, jaundice, and urticarial. CT is the
best investigation to differentiate hydatid cysts from amoebic and pyogenic cysts. Surgery is the mainstay of treatment (the cyst
walls must not be ruptured during removal and the contents sterilised first).

111.A 22 year old lady presents with an episode of renal colic and following investigation is suspected of suffering from MEN
IIa. Which of the following abnormalities of the parathyroid glands are most often found in this condition?
A. Hypertrophy
B. Hyperplasia
C. Adenoma
D. Carcinoma
E. Metaplasia

Answer: B
MEN IIa: Medullary thyroid cancer. Hyperparathyroidism (usually hyperplasia). Phaeochromocytoma
In MEN IIa the commonest lesion is medullary thyroid cancer, with regards to the parathyroid glands the most common lesion
is hyperplasia. In MEN I a parathyroid adenoma is the most common lesion.

Multiple Endocrine NeoplasiaMultiple endocrine neoplasia (MEN) is inherited as an autosomal dominant disorder.
The table below summarises the three main types of MEN:

MEN type I: Mnemonic 'three P's': Parathyroid (95%): Parathyroid adenoma. Pituitary (70%): Prolactinoma/ACTH/Growth
Hormone secreting adenoma. Pancreas (50%): Islet cell tumours/Zollinger Ellison syndrome. Also: Adrenal (adenoma) and
thyroid (adenoma). MENIN gene (chromosome 11). Most common presentation = hypercalcaemia.

MEN type IIa: Phaeochromocytoma. Medullary thyroid cancer (70%). Hyperparathyroidism (60%). RET oncogene
(chromosome 10).

MEN type IIb: Same as MEN IIa with addition of: Marfanoid body habitus. Mucosal neuromas. RET oncogene (chromosome
10)

115. A male infant is born prematurely at 34 weeks gestation by emergency cesarean section. He initially appears to be
stable. However, over the ensuing 24 hours he develops worsening neurological function. Which of the following
processes is most likely to have occurred?

A. Extra dural haemorrhage


B. Sub dural haemorrhage
C. Sub arachnoid haemorrhage
D. Intraventricular haemorrhage
E. Arteriovenous malformation

Answer: D
Intraventricular haemorrhage
Intraventricular haemorrhage is a haemorrhage that occurs into the ventricular system of the brain. It is relatively rare in adult
surgical practice and when it does occur, it is typically associated with severe head injuries. In premature neonates it may occur
219
spontaneously. The blood may clot and occlude CSF flow, hydrocephalus may result.
In neonatal practice the vast majority of IVH occur in the first 72 hours after birth, the aetiology is not well understood and it is
suggested to occur as a result of birth trauma combined with cellular hypoxia, together the with the delicate neonatal CNS.
TreatmentIs largely supportive, therapies such as intraventricular thrombolysis and prophylactic CSF drainage have been
trialled and not demonstrated to show benefit. Hydrocephalus and rising ICP is an indication for shunting.

112.A 22 year old man is admitted to hospital with a lower respiratory chest infection. He had a splenectomy after being
involved in a car accident. What is the most likely infective organism?
A. Haemophilus influenzae
B. Staphylococcus aureus
C. Rhinovirus
D. Mycobacterium tuberculosis
E. Moraxella catarrhalis

Answer: A
Organisms causing post splenectomy sepsis: Streptococcus pneumonia. Haemophilus influenza. Meningococci

Encapsulated organisms carry the greatest pathogenic risk following splenectomy. The effects of sepsis following splenectomy
are variable. This may be the result of small isolated fragments of splenic tissue that retain some function following
splenectomy. These may implant spontaneously following splenic rupture (in trauma) or be surgically implanted at the time of
splenectomy.

Post splenectomy sepsis: The loss of splenic function renders individuals at increased risk of fulminant sepsis. Young children
are at the highest risk, especially in the first 2 years following surgery. Surgery for trauma is associated with a lower risk than
when splenectomy is performed as a treatment for haematological disorders.
Infection with encapsulated organisms poses the greatest risk, these organisms may be opsonised, but this then goes undetected
at an immunological level due to loss of the spleen. Prophylactic vaccinations are usually administered to reduce the risk of
pneumococcal septicaemia. Since the vaccine only covers up to 80% of pneumococcal infections, patients will usually recieve
long term, low dose penicillin prophylaxis in addition to vaccination.

113.A 24 year old man presents with symptoms of malaise, weight loss and lymphadenopathy. A lymph node biopsy is
performed and the subsequent histology report states that there is evidence of granuloma formation and central
necrosis. What is the most likely underlying cause?
A. Non Hodgkins lymphoma
B. Churg Strauss syndrome
C. Epstein Barr Virus infection
D. Rheumatoid nodule
E. Infection with Mycobacterium tuberculosis
Answer:
E
These histological features are typically seen in TB. Necrosis occurring in granulomas is usually indicative of an underlying
infective cause. Churg Strauss syndrome is a form of vasculitis, which is the usual histological finding. Granulomas are reported
in the condition, but it is rare for them to demonstrate necrosis.

114.A 20 year old man develops acute appendicitis, his appendix is removed and he makes a full recovery. Which of the
following pathological processes is least likely to be present in the acutely inflamed tissues?
A. Altered Starlings forces.
B. Seqestration of neurophils
C. Formation of fluid exudate
D. Formation of granulomas
E. None of the above

Answer: D
Neutrophil polymorphs=Acute inflammation. Granuloma = Chronic inflammation.

Acute inflammation: 3 phases: 1. Changes in blood vessel and flow: flush, flare, wheal. 2. Fluid exudates (rich in protein i.e.
Ig, coagulation factors) produced via increased vascular permeability. 3. Cellular exudates mainly containing neutrophil
polymorphs pass into extravascular space.

Neutrophils are then transported to tissues via: a. Margination of neutrophils to the peripheral plasmatic of the vessel rather
than the central axial stream. b. Pavementing: Adhesion of neutrophils to endothelial cells in venules at site of acute
inflammation. c. Emigration: neutrophils pass between endothelial cells into the tissue
220
Theme: Liver lesions

A. Haemangioma
B. Hepatocellular carcinoma
C. Hepatic metastasis
D. Polycystic liver disease
E. Simple liver cyst
F. Hyatid cyst
G.Amoebic abscess
H.Mesenchymal hamartoma

Please select the most likely liver lesion for the scenario given. Each option may be used once, more than once or not at all.

115.A 42 year old lady has suffered from hepatitis C for many years and has also developed cirrhosis. On routine follow up,
an ultrasound has demonstrated a 2.5cm lesion in the right lobe of the liver.
Answer: Hepatocellular carcinoma
In patients with cirrhosis the presence of a lesion >2cm is highly suggestive of malignancy. The diagnosis is virtually confirmed
if the AFP is >400ng/mL.

116.A 25 year old man from the far east presents with a fever and right upper quadrant pain. As part of his investigations a
CT scan shows an ill defined lesion in the right lobe of the liver.
Answer: Amoebic abscess
Amoebic abscesses will tend to present in a similar fashion to other pyogenic liver abscesses. They should be considered in any
individual presenting from a region where Entamoeba histiolytica is endemic. Treatment with metronidazole usually produces a
marked clinical response.
117.A 42 year old lady presents with right upper quadrant pain and a sensation of abdominal fullness. An ultrasound scan
demonstrates a 6.5 cm hyperechoic lesion in the right lobe of the liver. Serum AFP is normal.
Answer: Haemangioma
A large hyperechoic lesion in the presence of normal AFP is likely to be a haemangioma. An HCC of equivalent size will
almost always result in rise in AFP.

118.Which of the following disorders is associated with massive splenomegaly?


A. Acute lymphoblastic leukaemia
B. Acute myeloblastic leukaemia
C. Acute myelomonocytic leukaemia
D. Acute monoblastic leukaemia
E. Chronic granulocytic leukaemia

Answer: E
Chronic leukaemia is more likely to be associated with splenomegaly than acute leukaemia.

Spleen: Embryology: derived from mesenchymal tissue. Shape: orange segment. Position: below 9th-12th ribs. Weight: 75-
150g
Relations: Superiorly- diaphragm. Anteriorly- gastric impression. Posteriorly- kidney. Inferiorly- colon. Hilum: tail of pancreas
and splenic vessels (splenic artery divides here, branches pass to the white pulp transporting plasma). Forms apex of lesser sac
(containing short gastric vessels). Contents: White pulp: immune function. Contains central trabecular artery. The germinal
centres are supplied by arterioles called penicilliary radicles. Red pulp: filters abnormal red blood cells. Function: Filtration of
abnormal blood cells and foreign bodies such as bacteria. Immunity: IgM. Production of properdin, and tuftsin which help target
fungi and bacteria for phagocytosis. Haematopoiesis: up to 5th month gestation or in haematological disorders. Pooling: storage
of 40% platelets. Iron reutilization. Storage red blood cells-animals, not humans. Storage monocytes

Disorders of the spleen: Massive splenomegaly. Myelofibrosis. Chronic myeloid leukaemia. Visceral leishmaniasis (kala-azar).
Malaria. Gaucher's syndrome

Other causes (as above plus): Portal hypertension e.g. secondary to cirrhosis. Lymphoproliferative disease e.g. CLL, Hodgkin's.
Haemolytic anaemia. Infection: hepatitis, glandular fever. Infective endocarditis. Sickle-cell*, thalassaemia. Rheumatoid
arthritis (Felty's syndrome)
*the majority of adults patients with sickle-cell will have an atrophied spleen due to repeated infarction

119.Causes of primary chronic inflammation do not include which of the following?


A. Sarcoidosis
B. Tuberculosis
221
C. Ulcerative colitis
D. Prostheses
E. Chronic cholecystitis

Answer: E
Chronic cholecystitis is caused by recurrent episodes of acute inflammation.
Prosthetic implants may be the site of primary chronic inflammation. A common example clinically is breast implants which
may become encapsulated. The subsequent fibrosis then results in distortion and may be painful.

120.A 30 year old man is trapped in a house fire and sustains 30% partial and full thickness burns to his torso and limbs.
Three days following admission he has a brisk haematemesis. Which of the following is the most likely explanation for
this event?
A. Dieulafoy lesion
B. Curlings ulcers
C. Mallory Weiss tear
D. Depletion of platelets
E. Depletion of clotting factors

Answer: B
Stress ulcers in burns patients are referred to as Curlings ulcers and may cause haematemesis.

Burns: Types of burn


Type of burn Skin layers affected Skin Blanching Management
appearance
Epidermal/Superficial Epidermis Red, moist Yes
Superficial partial Epidermis and part of Pale, dry Yes Normally heals with no
thickness papillary dermis affected intervention
Deep partial thickness Epidermis, whole papillary Mottled red No Needs surgical intervention
dermis affected colour (depending on site)
Full thickness Whole skin layer and Dry, leathery No Burns centre
subcutaneous tissue affected hard wound

Depth of burn assessment: Bleeding on needle prick. Sensation. Appearance. Blanching to pressure

Percentage burn estimation: Lund Browder chart: most accurate even in children. Wallace rule of nines. Palmar surface:
surface area palm = 0.8% burn. >15% body surface area burns in adults needs urgent burn fluid resuscitation

Transfer to burn centre if: Need burn shock resuscitation. Face/hands/genitals affected. Deep partial thickness or full
thickness burns. Significant electrical/chemical burns

Escharotomies: Indicated in circumferential full thickness burns to the torso or limbs. Careful division of the encasing band of
burn tissue will potentially improve ventilation (if the burn involves the torso), or relieve compartment syndrome and oedema
(where a limb is involved)
Hettiaratchy S & Papini R. Initial management of a major burn: assessment and resuscitation. BMJ 2004;329:101-103

Theme: Adrenal gland disorders

A. Nelsons syndrome
B. Conns syndrome
C. Cushings syndrome
D. Benign incidental adenoma
E. Malignant adrenal adenoma
F. Waterhouse- Friderichsen syndrome
G.Metastatic lesion
H.Walker - Warburg syndrome
I. Phaeochromocytoma

Please select the most appropriate adrenal disorder for the scenario given. Each disorder may be selected once, more than once
or not at all.

121. A 19 year old lady is admitted to ITU with severe meningococcal sepsis. She is on maximal inotropic support and a CT
scan of her chest and abdomen is performed. The adrenal glands show evidence of diffuse haemorrhage.
222
Answer: Waterhouse- Friderichsen syndrome
This is often a pre-terminal event and is associated with profound sepsis and coagulopathy.

122. A 34 year old lady is admitted with recurrent episodes of non-specific abdominal pain. On each admission all blood
investigations are normal, as are her observations. On this admission a CT scan was performed. This demonstrates a
1.5cm nodule in the right adrenal gland. This is associated with a lipid rich core. Urinary VMA is within normal limits.
Other hormonal studies are normal.
Answer: Benign incidental adenoma
This is typical for a benign adenoma.Benign adenomas often have a lipid rich core that is readily identifiable on CT scanning. In
addition the nodules are often well circumscribed.

123.A 38 year old man is noted to have a blood pressure of 175/110 on routine screening. On examination there are no
physical abnormalities of note. CT scanning shows a left sided adrenal mass. Plasma metanephrines are elevated.
Answer: Phaeochromocytoma
Hypertension in a young patient without any obvious cause should be investigated. Urinary VMA and plasma metanephrines are
typically elevated.

124.A 15 year old boy is admitted with colicky abdominal pain of 6 hours duration. On examination he has a soft abdomen,
on systemic examination he has brownish spots around his mouth, feet and hands. His mother underwent surgery for
intussusception, aged 12, and has similar lesions. What is the most likely underlying diagnosis?
A. Li Fraumeni syndrome
B. Peutz-Jeghers syndrome
C. Addisons disease
D. McCune -Albright syndrome
E. Appendicitis

Answer: B

This is most likely to be Peutz-Jeghers syndrome. Addisons and McCune Albright syndrome may produce similar skin changes
but the intussusception resulting from polyps combined with the autosomal inheritance pattern makes this the most likely
diagnosis.

Peutz-Jeghers syndrome is an autosomal dominant condition characterised by numerous benign hamartomatous polyps in the
gastrointestinal tract. It is also associated with pigmented freckles on the lips, face, palms and soles. Around 50% of patients
will have died from a gastrointestinal tract cancer by the age of 60 years. Genetics: Autosomal dominant. Responsible gene
encodes serine threonine kinase LKB1 or STK11. Features: Hamartomatous polyps in GI tract (mainly small bowel).
Pigmented lesions on lips, oral mucosa, face, palms and soles. Intestinal obstruction e.g. intussusception (which may lead to
diagnosis). Gastrointestinal bleeding. Management: Conservative unless complications develop

125.Which of the following is not included in Multiple Endocrine Neoplasia Type 2b?
A. Phaeochromocytoma
B. Visceral ganglioneuromas
C. Thyroid medullary carcinoma
D. Zollinger Ellison syndrome
E. Marfanoid features

Answer: D
MEN IIB: Medullary thyroid cancer. Phaeochromocytoma. Mucosal neuroma. Marfanoid appearance

126.Which virus is associated with Kaposi's sarcoma?


A. Human herpes virus 8
B. Human papillomavirus 16
C. Human T-lymphotropic virus 1
D. Epstein-Barr virus
E. Human papillomavirus 18

Answer: A
Oncoviruses: Viruses which cause cancer. These may be detected on blood test and prevented by vaccine

These are the main types of oncoviruses and their diseases: Epstein-Barr virus: Burkitt's lymphoma, Hodgkin's lymphoma,
Post transfusion lymphoma, Nasopharyngeal carcinoma. Human papillomavirus 16/18: Cervical cancer, Anal cancer, Penile
cancer, Vulval cancer, Oropharyneal cancer. Human herpes virus 8 :Kaposi's sarcoma. Hepatitis B virus :Hepatocellular
223
carcinoma. Hepatitis C virus: Hepatocellular carcinoma. Human T-lymphotropic virus 1: Tropical spastic paraparesis
Adult T cell leukaemia

127. Which of the following is not a feature of Wallerian Degeneration?


A. May result from an axonotmesis
B. May occur in either the central or peripheral nervous systems
C. The axon remains excitable throughout the whole process
D. The distal neuronal stump is affected
E. Is a component of the healing process following neuronal injury

Answer: C
The axon loses its excitability once the process is established.

Wallerian degeneration: Is the process that occurs when a nerve is cut or crushed. It occurs when the part of the axon
separated from the neuron's cell nucleus degenerates. It usually begins 24 hours following neuronal injury and the distal axon
remains excitable up until this time. The degeneration of the axon is following by breakdown of the myelin sheath, a process
that occurs by infiltration of the site with macrophages. Eventually regeneration of the nerve may occur although recovery will
depend on the extent and manner of injury.

128.A 45 year old woman complains of painful tingling in her fingers. The pain is relieved by hanging the arm over the side
of the bed. She has a positive Tinel's sign. Which of the following is most likely to contribute to her diagnosis?
A. Methotrexate use
B. Crohn's disease
C. Hyperthyroidism
D. Tuberculosis
E. Rheumatoid arthritis
Answer:
E
This woman has a diagnosis of carpal tunnel syndrome. Rheumatological disorders are a common cause. Clinical examination
should focus on identifying stigmata of rheumatoid arthritis, such as rheumatoid nodules, vasculitic lesions and
metacarpophalangeal joint arthritis.

Carpal tunnel syndrome is caused by compression of median nerve in the carpal tunnel. History: pain/pins and needles in
thumb, index, middle finger e.g. at night. Patient flicks hand to obtain relief. Examination: weakness of thumb abduction.
Wasting of thenar eminence (NOT hypothenar). Tinel's sign: tapping causes paraesthesia. Phalen's sign: flexion of wrist causes
symptoms. Causes of carpal tunnel syndrome: MEDIAN TRAP Mnemonic: Myxoedema. Edema premenstrually. Diabetes.
Idiopathic. Agromegaly. Neoplasm. Trauma. Rheumatoid arthritis. Amyloidosis. Pregnancy.
Management: Non surgical treatment: May resolve spontaneously. Avoid precipitants and reassurance. Night-time splints.
Local steroid injections. Surgery Complete division of the flexor retinaculum and decompression of the tunnel (successful in
approximately 80% of patients)

129.Which of the following is not an oncogene?


A. ras
B. myc
C. sis
D. Ki 67
E. erb-B
Answer:
D
Ki 67 is a nuclear proliferation marker (used in immunohistochemistry). Although, Ki67 positivity is a marker of malignancy, it is
not itself, an oncogene.

Oncogenes: Oncogenes are cancer promoting genes that are derived from normal genes (proto-oncogenes). Proto-oncogenes play
an important physiological role in cellular growth. They are implicated in the development of up to 20% of human cancers. Proto-
oncogenes may become oncogenes via the following processes: Mutation (point mutation). Chromosomal translocation. Increased
protein expression. Only one mutated copy of the gene is needed for cancer to occur - a dominant effect. Classification of
oncogenes: Growth factors e.g. Sis. Transcription factors e.g. Myc. Receptor tyrosine kinase e.g. RET. Cytoplasmic tyrosine
kinase e.g. Src. Regulatory GTPases e.g. Ras

Tumour supressor genes: Tumour supressor genes restrict or repress cellular proliferation in normal cells. Their inactivation
through mutation or germ line incorporation is implicated in renal, colonic, breast, bladder and many other cancers. One of the
best known tumour supressor genes is p53. p53 gene offers protection by causing apoptosis of damaged cells. Other well known
genes include BRCA 1 and 2.
224
130.A 30 year old male presents with a painless swelling of the testis. Histologically the stroma has lymphocytic
infiltrate. The most likely diagnosis is :
A. Differentiated teratoma
B. Malignant undifferentiated teratoma
C. Classical seminoma
D. Spermatocytic seminoma
E. Anaplastic seminoma

Answer: C
Seminoma is the commonest type of testicular tumour and is more common in males aged between 30-40 years. Classical
seminoma is the commonest subtype and histology shows lymphocytic stromal infiltrate. Other subtypes include:
1. Spermatocytic: tumour cells resemble spermatocytes. Excellent prognosis. 2. Anaplastic. 3. Syncytiotrophoblast giant cells:
beta hCG present in cells. A teratoma is common in males aged 20-30 years.

131.A 48 year old women presents with recurrent loin pain and fevers. Investigation reveals a staghorn calculus of the
left kidney. Infection with which of the following organisms is most likely?
A. Staphylococcus saprophyticus
B. Proteus mirabilis
C. Klebsiella
D. E-Coli
E. Staphylococcus epidermidis

Answer: B
Infection with Proteus mirabilis accounts for 90% of all proteus infections. It has a urease producing enzyme. This will tend to
favor urinary alkalinisation which is a relative per-requisite for the formation of staghorn calculi.

132.Causes of granulomatous disease do not include:


A. Amiodarone
B. Allopurinol
C. Sulphonamides
D. Beryllium
E. Wegener's granulomatosis

Answer: A
Allopurinol and sulphonamides cause hepatic granulomas.

Theme: Facial nerve palsy

A. Adenoid cystic carcinoma


B. Cerebrovascular accident
C. Petrous temporal fracture
D. Warthins tumour
E. Sarcoidosis
F. Pleomorphic adenoma
G. Cholesteatoma

Please select the most likely cause of facial nerve palsy for the scenario given. Each option may be used once, more than once or
not at all.

133.A 22 year old man presents with symptoms of lethargy and bilateral facial nerve palsy. On examination he has
bilateral parotid gland enlargement.
Answer: Sarcoidosis
Facial nerve palsy is the commonest neurological manifestation of sarcoid. It usually resolves. The absence of ear
discharge or discrete lesion on palpation is against the other causes.

134.A 21 year old man presents with a unilateral facial nerve palsy after being hit in the head. On examination he has
a right sided facial nerve palsy and a watery discharge from his nose.
Answer: Petrous temporal fracture
Nasal discharge of clear fluid and recent head injury makes a basal skull fracture the most likely underlying diagnosis.

225
135.A 43 year old lady presents with symptoms of chronic ear discharge and a right sided facial nerve palsy. On
examination she has foul smelling fluid draining from her right ear and a complete right sided facial nerve palsy.
Answer: Cholesteatoma
Foul smelling ear discharge and facial nerve weakness is likely to be due to cholesteatoma. The presence of a
neurological deficit is a sinister feature.

Facial nerve palsy: Sarcoid: Facial nerve palsy is the most frequent neurological manifestation of sarcoid. Affects right and left
side with equal frequency, may be bilateral. Typically resolves in up to 80% of cases. Cholesteatoma: Destructive and expanding
growth of keratinised squamous epithelium. Patients often complain of chronic ear discharge. Infection with Pseudomonas may
occur resulting in foul smell to discharge. Aquired lesions usually arise from the Pars flaccida region of the tympanic membrane.
Surgical removal and mastoidectomy may be needed. Recurrence rates of 20% may be seen following surgery. Basal skull
fracture: History of head injury. Presence of features such as Battles sign on examination. Clinical presence of CSF leak strongly
supports diagnosis. Assessment is by CT and MRI scan. Prophylactic antibiotics should be given in cases of CSF leak

136.A 59 year old lady is referred from the NHS breast screening program. A recent mammogram is reported as showing
linear, branching microcalcification with coarse granules. Which disease process is the most likely underlying cause of
these appearances?
A. Invasive lobular cancer
B. Lobular carcinoma in situ
C. Cribriform type ductal carcinoma in situ
D. Comedo type ductal carcinoma in situ
E. Fibroadenosis

Answer: D
Comedo type DCIS is usually associated with microcalcifications. Cribriform lesions are usually multifocal but less likely to form
microcalcifications. Lobular cancers and in situ lesions rarely form microcalcifications and are difficult to detect using
mammography.

Breast cancer - In situ disease: Breast cancer that has yet to invade the basement membrane is referred to as in situ disease. Both
ductal and lobular in situ variants are recognised. Ductal carcinoma in situ: Sub types include; comedo, cribriform,
micropapillary and solid. Comdeo DCIS is most likely to form microcalcifications. Cribriform and micropapillary are most likely
to be multifocal. Most lesions are mixed (composed of multiple subtypes). High nuclear grade DCIS is associated with more
malignant characteristics (loss of p53, increase erbB2 expression). Local excision of low nuclear grade DCIS will usually produce
satisfactory outcomes. Multifocal lesions, large and high nuclear grade lesions will usually require mastectomy. Lobular
carcinoma in situ: Much rarer than DCIS. Does not form microcalcifications. Usually single growth pattern. When an invasive
component is found it is less likely to be associated with axillary nodal metastasis than with DCIS. Low grade LCIS is usually
treated by monitoring rather than excision

137.In patients with an annular pancreas where is the most likely site of obstruction?
A. The first part of the duodenum
B. The second part of the duodenum
C. The fourth part of the duodenum
D. The third part of the duodenum
E. The duodeno-jejunal flexure

Answer: B
The pancreas develops from two foregut outgrowths (ventral and dorsal). During rotation the ventral bud and adjacent gallbladder
and bile duct lie together and fuse. When the pancreas fails to rotate normally it can compress the duodenum with development of
obstruction. Usually occurring as a result of associated duodenal malformation. The second part of the duodenum is the
commonest site.

Pancreas: The pancreas is a retroperitoneal organ and lies posterior to the stomach. It may be accessed surgically by dividing the
peritoneal reflection that connects the greater omentum to the transverse colon. The pancreatic head sits in the curvature of the
duodenum. It's tail lies close to the hilum of the spleen, a site of potential injury during splenectomy.

Relations: Posterior to the pancreas


Pancreatic head: Inferior vena cava. Common bile duct. Right and left renal veins. Superior mesenteric vein and artery.
Pancreatic neck: Superior mesenteric vein, portal vein. Pancreatic body: Left renal vein: Crus of diaphragm. Psoas muscle.
Adrenal gland. Kidney. Aorta. Pancreatic tail: Left kidney

226
Anterior to the pancreas

Pancreatic head: 1st part of the duodenum. Pylorus. Gastroduodenal artery. SMA and SMV(uncinate process). Pancreatic
body: Stomach. Duodenojejunal flexure. Pancreatic tail: Splenic hilum. Superior to the pancreas: Coeliac trunk and its
branches common hepatic artery and splenic artery. Grooves of the head of the pancreas: 2nd and 3rd part of the duodenum.
Arterial supply: Head: pancreaticoduodenal artery. Rest: splenic artery. Venous drainage: Head: superior mesenteric vein.
Body and tail: splenic vein. Ampulla of Vater: Merge of pancreatic duct and common bile duct. Is an important landmark,
halfway along the second part of the duodenum, that marks the anatomical transition from foregut to midgut (also the site of
transition between regions supplied by coeliac trunk and SMA).

Theme: Chest pain

A. Achalasia
B. Pulmonary embolus
C. Dissection of thoracic aorta
D. Boerhaaves syndrome
E. Gastro-oesophageal reflux
F. Carcinoma of the oesophagus
G. Oesophageal candidiasis

Please select the most likely cause for chest pain for the scenario given. Each option may be used once, more than once or not at
all.

138.A 43 year old man who has a long term history of alcohol misuse is admitted with a history of an attack of
vomiting after an episode of binge drinking. After vomiting he developed sudden onset left sided chest pain, which
is pleuritic in nature. On examination he is profoundly septic and drowsy with severe epigastric tenderness and
left sided chest pain.
Answer: Boerhaaves syndrome
In patients with Boerhaaves the rupture is often on the left side. The story here is typical. All patients should have a
contrast study to confirm the diagnosis and the affected site prior to thoracotomy.

139.A 22 year old man is admitted with severe retrosternal chest pain and recurrent episodes of dysphagia. These
occur sporadically and often resolve spontaneously. On examination there are no physical abnormalities and the
patient seems well.
Answer: Achalasia
Achalasia may produce severe chest pain and many older patients may undergo cardiac investigations prior to
endoscopy.
Endoscopic injection with botulinum toxin is a popular treatment (although the benefit is not long lasting).
Cardiomyotomy is a more durable alternative.

140.An obese 53 year old man presents with symptoms of recurrent retrosternal discomfort and dyspepsia. This is
typically worse at night after eating a large meal. On examination there is no physical abnormality to find.
Answer: Gastro-oesophageal reflux

Patients with GORD often have symptoms that are worse at night. In this age group an Upper GI endoscopy should
probably be performed.

Surgical chest pain

Dissection of thoracic aorta: Tearing interscapular pain. Discrepancy in arterial blood pressures taken in both arms. May show
mediastinal widening on chest x-ray

Diffuse oesophageal spasm: Spectrum of oesophageal motility disorders. Caused by uncoordinated contractions of oesphageal
muscles. May show "nutcracker oesophagus" on barium swallow. Symptoms include dysphagia, retrosternal discomfort and
dyspepsia

Gastro-oesphageal reflux: Common cause of retrosternal discomfort. Usually associated with symptoms of regurgitation,
odynophagia and dyspepsia. Symptoms usually well controlled with PPI therapy. Risk factors include obesity, smoking and
excess alcohol consumption

227
Boerhaaves syndrome: Spontaneous rupture of the oesophagus. Caused by episodes of repeated vomiting often in association
with alcohol excess. Typically there is an episode of repetitive vomiting followed by severe chest and epigastric pain. Diagnosis is
by CT and contrast studies. Treatment is surgical; during first 12 hours primary repair, beyond this usually creation of controlled
fistula with a T Tube, delay beyond 24 hours is associated with fulminent mediastinitis and is usually fatal.

Achalasia: Difficulty swallowing, dysphagia to both liquids and solids and sometimes chest pain. Usually caused by failure of
distal oesphageal inhibitory neurons. Diagnosis is by pH and manometry studies together with contrast swallow and endoscopy.
Treatment is with either botulinum toxin, pneumatic dilatation or cardiomyotomy

141.A 32 year old woman presents with an episode of haemoptysis and is found to have metastatic tumour present
within the parenchyma of the lungs. This is biopsied and subsequent histology shows clear cells. What is the most
likely primary site?
A. Kidney
B. Breast
C. Liver
D. Adrenal
E. Bone

Answer: A
Clear cell tumours are a sub type of renal cell cancer it is associated with specific genetic changes localised to chromosome 3.

142.A laceration of the wrist produces a median nerve transaction. The wound is clean and seen immediately after
injury. Collateral soft tissue damage is absent. The patient asks what the prognosis is. You indicate that the nerve
should regrow at approximately:
A. 0.1 mm per day
B. 1 mm per day
C. 5 mm per day
D. 1 cm per day
E. None of the above

Answer: B
Transaction of a peripheral nerve results in hemorrhage and in retraction of the several nerve ends. Almost immediately,
degeneration of the axon distal to the injury begins. Degeneration also occurs in the proximal fragment back to the first node of
Ranvier. Phagocytosis of the degenerated axonal fragments leaves neurilemmal sheath with empty cylindrical spaces where the
axons were. Several days following the injury, axons from the proximal fragment begin to regrow. If they make contact with the
distal neurilemmal sheath, regrowth occurs at about the rate of 1 mm/day. However, if associated trauma, fracture, infection, or
separation of neurilemmal sheath ends precludes contact between axons, growth is haphazard and a traumatic neuroma is formed.
When neural transaction is associated with widespread soft tissue damage and hemorrhage (with increased probability of
infection), many surgeons choose to delay reapproximation of the severed nerve end for 3 to 4 weeks.

Nerve injury: There are 3 types of nerve injury: Neuropraxia: Nerve intact but electrical conduction is affected. Full recovery.
Autonomic function preserved. Wallerian degeneration does not occur. Axonotmesis: Axon is damaged and the myelin sheath is
preserved. The connective tissue framework is not affected. Wallerian degeneration occurs. Neurotmesis: Disruption of the axon,
myelin sheath and surrounding connective tissue. Wallerian degeneration occurs. Wallerian Degeneration: Axonal degeneration
distal to the site of injury. Typically begins 24-36 hours following injury. Axons are excitable prior to degeneration occurring.
Myelin sheath degenerates and is phagocytosed by tissue macrophages.

Nerve repair: Neuronal repair may only occur physiologically where nerves are in direct contact. Where a large defect is present
the process of nerve regeneration is hampered and may not occur at all or result in the formation of a neuroma. Where nerve
regrowth occurs it typically occurs at a rate of 1mm per day.

156. Which of the following statements relating to gastric cancer is untrue?

A. It is associated with chronic helicobacter pylori infection


B. 5% of gastric malignancies are due to lymphoma
C. In the Lauren classification the diffuse type of adenocarcinoma typically presents as a large exophytic growth in
the antrum
D. Smoking is a risk factor
E. It is associated with acanthosis nigricans
Answer: C
The Lauren classification describes a diffuse type of adenocarcinoma (Linitis plastica type lesion) and an intestinal type. The

228
diffuse type is often deeply infiltrative and may be difficult to detect on endoscopy. Barium meal appearances can be
characteristic.

157. Which of the following statements relating to Gardners syndrome variant of familial adenomatous polyposis coli is
false?

A. It is an autosomal dominant condition


B. Patients may develop retroperitoneal desmoid tumours
C. The vast majority of the polyps are benign and thus the risk of colorectal cancer is small
D. Patients are at increased risk of thyroid cancer
E. It is characterised by a mutation in the APC gene

Answer: C
The multiple polyps increase the risk of malignancy and most patients should undergo a colectomy.

158. A 60-year-old man presents with lower urinary tract symptoms and is offered a PSA test. Which one of the following
could interfere with the PSA level?

A. Vigorous exercise in the past 48 hours


B. Poorly controlled diabetes mellitus
C. Drinking more than 4 units of alcohol in the past 48 hours
D. Smoking
E. Recent cholecystectomy

Answer: A
PSA testing: Prostate specific antigen (PSA) is a serine protease enzyme produced by normal and malignant prostate epithelial
cells. It has become an important tumour marker but much controversy still exists regarding its usefulness as a screening tool. The
NHS Prostate Cancer Risk Management Programme (PCRMP) has published updated guidelines in 2009 on how to handle
requests for PSA testing in asymptomatic men. A recent European trial (ERSPC) showed a statistically significant reduction in the
rate of death prostate cancer by 20% in men aged 55 to 69 years but this was associated with a high risk of over-diagnosis and
over-treatment. Having reviewed this and other data the National Screening Committee have decided not to introduce a prostate
cancer screening programme yet but rather allow men to make an informed choice. Age-adjusted upper limits for PSA were
recommended by the PCRMP*:
PSA level by age (ng/ml): 50-59 years:3.0; 60-69 years:4.0; > 70 years:5.0

PSA levels may also be raised by**: benign prostatic hyperplasia (BPH). Prostatitis and urinary tract infection (NICE recommend
to postpone the PSA test for at least 1 month after treatment). Ejaculation (ideally not in the previous 48 hours). Vgorous exercise
(ideally not in the previous 48 hours). Urinary retention. Instrumentation of the urinary tract

Poor specificity and sensitivity: around 33% of men with a PSA of 4-10 ng/ml will be found to have prostate cancer. With a PSA
of 10-20 ng/ml this rises to 60% of men. Around 20% with prostate cancer have a normal PSA. Various methods are used to try
and add greater meaning to a PSA level including age-adjusted upper limits and monitoring change in PSA level with time (PSA
velocity or PSA doubling time)
*aide memoire for upper PSA limit: (age - 20) / 10
**whether digital rectal examination actually causes a rise in PSA levels is a matter of debate

159. Which of the symptoms below is least typical of pancreatic cancer?

A. Painless jaundice
B. Hyperamylasaemia
C. Hyperglycaemia
D. Weight loss
E. Classical Courvoisier syndrome

Answer: B
Raised serum amylase is relatively uncommon. The typical Courvoisier syndrome typically occurs in 20% and hyperglycaemia
occurs in 15-20%.

Pancreatic cancer: Adenocarcinoma. Risk factors: Smoking, diabetes, Adenoma, Familial adenomatous polyposis. Mainly occur
in the head of the pancreas (70%). Spread locally and metastasizes to the liver. Carcinoma of the pancreas should be differentiated
from other periampullary tumours with better prognosis. Clinical features: Weight loss. Painless jaundice. Epigastric discomfort
(pain usually due to invasion of the coeliac plexus is a late feature). Pancreatitis. Trousseau's sign: migratory superficial
thrombophlebitis. Investigations: USS: May miss small lesions. CT Scanning (pancreatic protocol). If unresectable on CT then no
229
further staging needed. PET/CT for those with operable disease on CT alone. ERCP/ MRI for bile duct assessment. Staging
laparoscopy to exclude peritoneal disease. Management: Head of pancreas: Whipple's resection (SE dumping and ulcers). Newer
techniques include pylorus preservation and SMA/ SMV resection. Carcinoma body and tail: poor prognosis, distal
pancreatectomy if operable. Usually adjuvent chemotherapy for resectable disease. ERCP and stent for jaundice and palliation.
Surgical bypass may be needed for duodenal obstruction.

160. A 53 year old man presents with dyspepsia. An upper GI endoscopy is performed and Helicobacter pylori is identified.
A duodenal ulcer is present in the first part of the duodenum. Duodenal biopsies are taken and demonstrate epithelium
that resembles cells of the gastric antrum. Which of the following is the most likely explanation for this process?

A. Hyperplasia of the crypts of Lieberkhun


B. Duodenal metaplasia
C. Duodenal dysplasia
D. Duodenal carcinoma
E. Hyptertrophy of Brunners glands

Answer: B
Metaplasia = cell type conversion

The process involved is metaplasia. During metaplasia there is no direct carcinogenesis, however the persistent presence of
precipitants of metaplasia will lead to malignant changes in cells.
Metaplastic changes in the duodenal cap are frequently seen in association with H-Pylori induced ulcers. It typically resolves after
ulcer healing and eradication therapy.

Metaplasia: Definition: reversible change of differentiated cells to another cell type. May represent an adaptive substitution of
cells that are sensitive to stress by cell types better able to withstand the adverse environment. Can be a normal physiological
response (ossification of cartilage to form bone). Most common epithelial metaplasia occurs with transformation of columnar cells
to squamous cells (smoking causes ciliated columnar cells to be replaced by squamous epithelial cells; Schistosomiasis).
Metaplasia from squamous to columnar cells occurs in Barrett oesophagus. If the metaplastic stimulus is removed, the cells will
return to their original pattern of differentiation. However, if the stimulus is not removed then progression to dysplasia may occur.
Not considered directly carcinogenic, however the factors which predispose to metaplasia, if persistent may induce malignant
transformation. The pathogenesis involves a reprogramming of stem cells that are known to exist in normal tissues, or of
undifferentiated mesenchymal cells present in connective tissue. In a metaplastic change, these precursor cells differentiate along
a new pathway.

161. A 22 year old lady presents with symptoms and signs of hyperthyroidism. Her diagnostic work up results in a
diagnosis of Graves disease. Which of the following best describes the pathophysiology of the condition?

A. Formation of IgG antibodies to the TSH receptors on the thyroid gland


B. Formation of IgG antibodies to the TRH receptors on the anterior pituitary
C. Formation of IgM antibodies to the TSH receptors on the thyroid gland
D. Formation of IgA antibodies to the TSH receptors on the thyroid gland
E. Formation of IgM antibodies to the TRH receptors on the anterior pituitary
Answer:
A
Usually IgG antibodies are formed against the TSH receptors on the thyroid gland. Which is why the TSH level is often very low
in Graves disease.

Hormones of the thyroid gland: Triiodothyronine T3: Major hormone active in target cells. Thyroxine T4: Most prevalent
form in plasma, less biologically active than T3. Calcitonin: Lowers plasma calcium

Synthesis and secretion of thyroid hormones: Thyroid actively concentrates iodide to twenty five times the plasma
concentration. Iodide is oxidised by peroxidase in the follicular cells to atomic iodine which then iodinates tyrosine residues
contained in thyroglobulin. Iodinated tyrosine residues in thyroglobulin undergo coupling to either T3 or T4. Process is stimulated
by TSH, which stimulates secretion of thyroid hormones. The normal thyroid has approximately 3 month reserves of thyroid
hormones.

LATS and Graves disease: In Graves disease patients develop IgG antibodies to the TSH receptors on the thyroid gland. This
results in chronic and long term stimulation of the gland with release of thyroid hormones. The typically situation is raised thyroid
hormones and low TSH. Thyroid receptor autoantibodies should be checked in individuals presenting with hyperthyroidism as
they are present in up to 85% cases.
230
Theme: Breast disease

A. Tuberculosis
B. Actinomycosis
C. Duct ectasia
D. Fibroadenoma
E. Fat necrosis
F. Intraductal papilloma
G. Breast abscess

What is the most likely diagnosis for each scenario given? Each diagnosis may be used once, more than once or not at all.

162.A 32 year old Indian woman presents with a tender breast lump. She has a 2 month old child. Clinically there is a
tender, fluctuant mass of the breast.
Answer: Breast abscess
This lady is likely to be breast feeding and is at risk of mastitis. This may lead to an abscess if not treated. Staphylococcus
aureus is usually the causative organism.

163.A 53 year old lady presents with a green nipple discharge. On examination she has discharge originating from multiple
ducts and associated nipple inversion.
Answer: Duct ectasia
Duct ectasia is common during the period of breast involution that occurs during the menopausal period. As the ducts shorten
they may contain insipiated material. The discharge will often discharge from several ducts.

164.A 52 year old lady presents with an episode of nipple discharge. It is usually clear in nature. On examination the
discharge is seen to originate from a single duct and although it appears clear, when the discharge is tested with a
labstix it is shown to contain blood. Imaging and examination shows no obvious mass lesion,
Answer: Intraductal papilloma
Intraductal papilloma usually cause single duct discharge. The fluid is often clear, although it may be blood stained. If the fluid
is tested with a labstix (little point in routiine practice) then it will usually contain small amounts of blood. A
microdocechtomy may be performed.

Non malignant breast disease: Duct ectasia: Mammary duct ectasia may be seen in up to 25% of normal female breasts.
Patients usually present with nipple discharge, which may be from single or multiple ducts (usually present age >50 years). The
discharge is often thick and green. Duct ectasia is a normal varient of breast involution and is not the same condition as periductal
mastitis
Periductal mastitis: Present at younger age than duct ectasia. May present with features of inflammation, abscess or mammary
duct fistula. Stongly associated with smoking. Usually treated with antibiotics, abscess will require drainage
Intraductal papilloma: Growth of papilloma in a single duct. Usually presents with clear or blood stained discharge originating
from a single duct. No increase in risk of malignancy
Breast abscess: Lactational mastitis is common. Infection is usually with Staphylococcus aureus. On examination there is usually
a tender fluctuant mass. Treatment is with antibiotics and ultrasound guided aspiration. Overlying skin necrosis is an indication for
surgical debridement, which may be complicated by the development of a subsequent mammary duct fistula.
Tuberculosis: Rare in western countries, usually secondary TB. Affects women later in child bearing period. Chronic breast or
axillary sinus is present in up to 50% cases. Diagnosis is by biopsy culture and histology

165.A 45 year old man has widespread metastatic adenocarcinoma of the colon. Which of the following tumour markers is
most likely to be elevated?
A. CA19-9
B. Carcinoembryonic antigen
C. Alpha Feto Protein
D. CA 125
E. Beta BCG

Answer: B
Screening for colonic cancer using CEA is not justified
Carcinoembryonic antigen is elevated in colonic cancer, typically in relation to disease extent with highest serum levels noted in
metastatic disease. It is falsely elevated in a number of non-malignant disease states such as cirrhosis and colitis and for this
reason it has no role in monitoring colitics for colonic cancer[1].

231
Colorectal cancer screening and diagnosis: Overview: Most cancers develop from adenomatous polyps. Screening for
colorectal cancer has been shown to reduce mortality by 16%. The NHS now has a national screening programme offering
screening every 2 years to all men and women aged 60 to 69 years. Patients aged over 70 years may request screening. Eligible
patients are sent faecal occult blood (FOB) tests through the post. Patients with abnormal results are offered a colonoscopy. At
colonoscopy, approximately: 5 out of 10 patients will have a normal exam. 4 out of 10 patients will be found to have polyps which
may be removed due to their premalignant potential. 1 out of 10 patients will be found to have cancer
Diagnosis: Essentially the following patients need referral: Altered bowel habit for more than six weeks. New onset of rectal
bleeding. Symptoms of tenesmus. Colonoscopy is the gold standard, provided it is complete and good mucosal visualisation is
achieved. Other options include double contrast barium enema and CT colonography.
Staging: Once a malignant diagnosis is made patients with colonic cancer will be staged using chest / abdomen and pelvic CT.
Patients with rectal cancer will also undergo evaluation of the mesorectum with pelvic MRI scanning.
For examination purposes the Dukes and TNM systems are preferred.
Tumour markers
Carcinoembryonic antigen (CEA) is the main tumour marker in colorectal cancer. Not all tumours secrete this, and it may be
raised in conditions such as IBD. However, absolute levels do correlate (roughly) with disease burden and whilst this marker may
not be used extensively in follow up, it can be useful for investigation of patients with cancer of unknown primary.

166. Which of the following does not cause hypercalcaemia?

A. Thiazides
B. DiGeorge syndrome
C. Vitamin A
D. Rhabdomylosis
E. Sarcoidosis
Answer: B

VITAMINS TRAP: Vitamins A & D, Immobilization, Thyrotoxicosis, Addison's disease, Milk-alkali syndrome, Inflammatory
disorders, Neoplastic diseases, Sarcoidosis, Thiazides and other drugs, Rhabdomyolysis, AIDS, Paget's disease, Parenteral
nutrition, Parathyroid disease.

The parathyroid glands don't form as a result of this syndrome.

Hypercalcaemia: Main causes: Malignancy. Primary hyperparathyroidism


Less common: Sarcoidosis (extrarenal synthesis of calcitriol ). Thiazides, lithium. Immobilisation. Pagets disease. Vitamin A/D
toxicity. Thyrotoxicosis. MEN. Milk alkali syndrome

Clinical features: “Stones, bones, abdominal moans, and psychic groans”


Theme: Diseases affecting the great vessels

A. Aortic coarctation
B. Cervical rib
C. Takayasu's arteritis
D. Subclavian steal syndrome
E. Patent ductus arteriosus
F. Aortic dissection

Please select the most likely underlying cause for the symptoms described. Each option may be used once, more than once or not
at all.

166.A 24 year old lady from Western India presents with symptoms of lethargy and dizziness, worse on turning her head.
On examination her systolic blood pressure is 176/128. Her pulses are impalpable at all peripheral sites. Auscultation of
her chest reveals a systolic heart murmur.
Answer: Takayasu's arteritis
Takayasu's arteritis most commonly affects young Asian females. Pulseless peripheries are a classical finding. The CNS
symptoms may be variable.

167.A 48 year old man notices that he is becoming increasingly dizzy when he plays squash, in addition he has also
developed cramping pain in his left arm. One day he is inflating his car tyre with a hand held pump, he collapses and is
brought to hospital.
Answer: Subclavian steal syndrome
Subclavian steal syndrome is associated with a stenosis or occlusion of the subclavian artery, proximal to the origin of the
vertebral artery. As a result the increased metabolic needs of the arm then cause retrograde flow and symptoms of CNS
vascular insufficiency.
232
168.shows evidence of rib notching. Auscultation of his chest reveals a systolic murmur which is loudest at the posterior
aspect of the fourth intercostal space.
Answer: Aortic coarctation
Coarctation of the aorta may occur due to the remnant of the ductus arteriosus acting as a fibrous constrictive band of the aorta.
Weak arm pulses may be seen, radiofemoral delay is the classical physical finding. Collateral flow through the intercostal
vessels may produce notching of the ribs, if the disease is long standing.

Vascular disease
Aortic dissection: Chest pain (anterior chest pain- ascending aorta, back pain - descending aorta). Widening of aorta on chest x-
ray. Diagnosis made by CT scanning. Treatment is either medical (Type B disease) or surgical (Type A disease)

Cervical rib: Supernumery fibrous band arising from seventh cervical vertebra. Incidence of 1 in 500. May cause thoracic outlet
syndrome. Treatment involves surgical division of rib

Subclavian steal syndrome: Due to proximal stenotic lesion of the subclavian artery. Results in retrograte flow through vertebral
or internal thoracic arteries. The result is that decrease in cerebral blood flow may occur and produce syncopal symptoms. A
duplex scan and/ or angiogram will delineate the lesion and allow treatment to be planned

Takayasu's arteritis: Large vessel granulomatous vasculitis. Results in intimal narrowing. Most commonly affects young asian
females. Patients present with features of mild systemic illness, followed by pulseless phase with symptoms of vascular
insufficiency. Treatment is with systemic steroids

Patent ductus arteriosus: Ductus arteriosus is a normal foetal vessel that closes spontaneously after birth. Results in high
pressure, oxygenated blood entering the pulmonary circuit. Untreated patients develop symptoms of congestive cardiac failure

Coarctation of the aorta: Aortic stenosis at the site of the ductus arteriosus insertion. Most common in boys and girls with
Turners syndrome. Patients may present with symptoms of arterial insufficiency, such as syncope and claudication. Blood
pressure mismatch may be seen, as may mismatch of pulse pressure in the upper and lower limbs. Treatment is either with
angioplasty or surgical resection (the former is the most common)

169.A 25 year old man is found to have carcinoid syndrome. Which of the following hormones is released by carcinoids?
A. Serotonin
B. Dopamine
C. Nor adrenaline
D. Adrenaline
E. Aldosterone
Answer: A
Rule of thirds:1/3 multiple. 1/3 small bowel. 1/3 metastasize. 1/3 second tumour

Carcinoids secrete serotonin. Carcinoid syndrome will only occur in the presence of liver metastasis as the hormone released from
primary lesions will typically be metabolised by the liver.

170.Which one of the following genes protects against neoplasms?


A. sis
B. p53
C. ras
D. myc
E. src

Answer: B
p53 is a tumour supressor gene and located on chromosome 17. It plays an important role in causing cells that are undergoing
neoplastic changes to enter an apoptotic pathway.

171.A 55 year old man with a long history of achalasia is successfully treated by a Hellers Cardiomyotomy. Several years
later he develops an oesophageal malignancy. Which of the following lesions is most likely to be present?
A. Adenocarcinoma
B. Gastrointestinal stromal tumour
C. Leiomyosarcoma
D. Rhabdomyosarcoma

233
E. Squamous cell carcinoma

Answer: E
Achalasia is a rare condition. However, even once treated there is an increased risk of malignancy. When it does occur it is most
likely to be of squamous cell type.

Oesophageal cancer: Incidence is increasing. In most cases in the Western world this increase is accounted for by a rise in the
number of cases of adenocarcinoma. In the UK adenocarcinomas account for 65% of cases. Barretts oesophagus is a major risk
factor for most cases of oesophageal adenocarcinoma. In other regions of the world squamous cancer is more common and is
linked to smoking, alcohol intake, diets rich in nitrosamines and achalasia. Surveillance of Barretts is important as it imparts a 30
fold increase in cancer risk and if invasive malignancy is diagnosed early then survival may approach 85% at 5 years.
Diagnosis: Upper GI endoscopy is the first line test. Contrast swallow may be of benefit in classifying benign motility disorders
but has no place in the assessment of tumours. Staging is initially undertaken with CT scanning of the chest, abdomen and pelvis.
If overt metastatic disease is identified using this modality then further complex imaging is unnecessary. If CT does not show
metastatic disease, then local stage may be more accurately assessed by use of endoscopic ultrasound. Staging laparoscopy is
performed to detect occult peritoneal disease. PET CT is performed in those with negative laparoscopy. Thoracoscopy is not
routinely performed.
Treatment: Operable disease is best managed by surgical resection. The most standard procedure is an Ivor- Lewis type
oesophagectomy. This procedure involves the mobilisation of the stomach and division of the oesophageal hiatus. The abdomen is
closed and a right sided thoracotomy performed. The stomach is brought into the chest and the oesophagus mobilised further. An
intrathoracic oesophagogastric anastomosis is constructed. Alternative surgical strategies include a transhiatal resection (for distal
lesions), a left thoraco-abdominal resection (difficult access due to thoracic aorta) and a total oesophagectomy (McKeown) with a
cervical oesophagogastric anastomosis.
The biggest surgical challenge is that of anastomotic leak, with an intrathoracic anastomosis this will result in mediastinitis. With
high mortality. The McKeown technique has an intrinsically lower systemic insult in the event of anastmotic leakage.
In addition to surgical resection many patients will be treated with adjuvent chemotherapy.

172.Which of the following genes is not implicated in the adenoma-carcinoma sequence in colorectal cancer?
A. src
B. c-myc
C. APC
D. p53
E. K-ras

Answer: A
Other genes involved are: MCC, DCC, c-yes, bcl-2

Colorectal cancer: Annually about 150,000 new cases are diagnosed and 50,000 deaths from the disease. About 75% will have
sporadic disease and 25% will have a family history. Colorectal tumours comprise a spectrum of disease ranging from adenomas
through to polyp cancers and frank malignancy. Polyps may be categorised into: neoplastic polyps, adenomatous polyps and non
neoplastic polyps. The majority of adenomas are polypoidal lesions, although flat lesions do occur and may prove to be dysplastic.
Non-neoplastic polyps include hyperplastic, juvenile, hamartomatous, inflammatory, and lymphoid polyps, which have not
generally been thought of as precursors of cancer. Three characteristics of adenomas that correlate with malignant potential have
been characterised. These include increased size, villous architecture and dysplasia. For this reason most polyps identified at
colonoscopy should be removed. The transformation from polyp to cancer is described by the adenoma - carcinoma sequence and
its principles should be appreciated. Essentially genetic changes accompany the transition from adenoma to carcinoma; key
changes include APC, c-myc, K RAS mutations and p53 deletions.
A 63 year old lady is suspected as having sarcoidosis. She is sent to the general surgeons and a lymph node biopsy is performed.

173.Which of the following histological features is most likely to be identified in a lymph node if sarcoid is present?
A. Psammoma bodies
B. Extensive necrosis
C. Dense eosinophillic infiltrates
D. Asteroid bodies
E. None of the above

Answer: D
Asteroid bodies are often found in the granulomas of individuals with sarcoid. Unlike the granulomata associated with
tuberculosis the granulomas of sarcoid are rarely associated with extensive necrosis.

174.Brown tumours of bone are associated with which of the following?


A. Hyperthyroidism
234
B. Hypothyroidism
C. Hyperparathyroidism
D. Hypoparathyroidism
E. Osteopetrosis
Answer:
C
Brown tumors are tumors of bone that arise in settings of excess osteoclast activity, such as hyperparathyroidism, and consist of
fibrous tissue, woven bone and supporting vasculature, but no matrix. They are radiolucent on x-ray. The osteoclasts consume the
trabecular bone that osteoblasts lay down and this front of reparative bone deposition followed by additional resorption can
expand beyond the usual shape of the bone, involving the periosteum thus causing bone pain. They appear brown because
haemosiderin is deposited at the site.

175.A 73 year old man is recovering following an emergency Hartmans procedure performed for an obstructing sigmoid
cancer. The pathology report shows a moderately differentiated adenocarcinoma that invades the muscularis propria, 3
of 15 lymph nodes are involved with metastatic disease. What is the correct stage for this?
A. Astler Coller Stage B2
B. Dukes stage A
C. Dukes stage B
D. Dukes stage C
E. Dukes stage D
Answer:
D
The involvement of lymph nodes makes this Dukes C. In the Astler Coller system the B and C subsets are split to B1 and B2 and
C1 and C2. Where C2 denotes involvement of the nodes in conjunction with penetration of the muscularis propria.

Dukes classification: Gives the extent of spread of colorectal cancer: Dukes A:Tumour confined to the mucosa (90%). Dukes
B:Tumour invading bowel wall (70%). Dukes C:Lymph node metastases (45%). Dukes D:Distant metastases (6%)(20% if
resectable)
5 year survival in brackets

176.A 55 year old man with dyspepsia undergoes an upper GI endoscopy. An irregular erythematous area is seen to protrude
proximally from the gastro-oesophageal junction. Apart from specialised intestinal metaplasia, which of the following
cell types should also be present for a diagnosis of Barretts oesophagus to be made?
A. Goblet cell
B. Neutrophil
C. Lymphocytes
D. Epithelial cells
E. Macrophages

Answer: A
Goblet cells need to be present for a diagnosis of Barrett's oesophagus to be made.

Barrett's oesophagus: Intestinal metaplasia. Squamous epithelium replaced by columnar epithelium in the lower oesophagus. 3
types of columnar epithelium: 1. Junctional. 2. Atrophic fundal. 3. Specialised. Presence of goblet cells important in identification.
Premalignant change (progress to dysplasia). Risk of adenocarcinoma. Risk factors: middle age, men, smoker, Caucasian, gastro-
oesophageal reflux, obesity
Treatment: Long term proton pump inhibitor. Consider pH and manometry studies in younger patients who may prefer to
consider an anti reflux procedure. Regular endoscopic monitoring (more frequently if moderate dysplasia). With quadrantic
biopsies every 2-3 cm. If severe dysplasia be very wary of small foci of cancer

177.Which of the following amino acids is present in all types of collagen?


A. Alanine
B. Aspartime
C. Glycine
D. Tyrosine
E. Cysteine

Answer: C
Collagen has a generic structure of Glycine- X- Y, where X and Y are variable sub units. The relatively small size of the glycine
molecule enables collagen to form a tight helical structure.

235
Collagen: One of the major connective tissue proteins. Composed of 3 polypeptide strands that are woven into a helix. Numerous
hydrogen bonds exist within molecule to provide additional strength. Many sub types but commonest sub type is I (90% of bodily
collagen). Vitamin c is important in establishing cross links
Collagen Diseases: Osteogenesis imperfect. Ehlers Danlos
Osteogenesis imperfecta: 8 Subtypes. Defect of type I collagen. In type I the collagen is normal quality but insufficient quantity.
Type II- poor quantity and quality. Type III- Collagen poorly formed, normal quantity. Type IV- Sufficient quantity but poor
quality
Patients have bones which fracture easily, loose joint and multiple other defects depending upon which sub type they suffer from
Ehlers Danlos: Multiple sub types. Abnormality of types 1 and 3 collagen. Patients have features of hypermobility. Individuals are
prone to joint dislocations and pelvic organ prolapse. In addition to many other diseases related to connective tissue defects

178.An 8 year old boy presented with a painless swelling on the superotemporal aspect of his orbit. It was smooth on
examination, produced no visual disturbances. Following excision it was found to be lined by squamous epithelium and
hair follicles. Which of the following lesions most closely matches these findings?
A. Dermoid cyst
B. Desmoid tumour
C. Lipoma
D. Sebaceous cyst
E. Schwannoma

Answer: A
Dermoid cysts are embryological remnants and may be lined by hair and squamous epithelium (like teratomas). They are often
located in the midline and may be linked to deeper structures resulting in a dumbbell shape to the lesion. Complete excision is
requires as they have a propensity to local recurrence if not excised.
Desmoid tumours are a different entity, they most commonly develop in ligaments and tendons. They are also referred to as
aggressive fibromatosis and consist of fibroblast dense lesions (resembling scar tissue). They should be managed in a similar
manner to soft tissue sarcomas.

Skin Diseases: Skin lesions may be referred for surgical assessment, but more commonly will come via a dermatologist for
definitive surgical management. Skin malignancies include basal cell carcinoma, squamous cell carcinoma and malignant
melanoma.

Basal Cell Carcinoma: Most common form of skin cancer. Commonly occur on sun exposed sites apart from the ear. Sub types
include nodular, morphoeic, superficial and pigmented. Typically slow growing with low metastatic potential. Standard surgical
excision, topical chemotherapy and radiotherapy are all successful. As a minimum a diagnostic punch biopsy should be taken if
treatment other than standard surgical excision is planned.

Squamous Cell Carcinoma: Again related to sun exposure. May arise in pre - existing solar keratoses. May metastasise if left.
Immunosupression (e.g. Following transplant), increases risk. Wide local excision is the treatment of choice and where a
diagnostic excision biopsy has demonstrated SCC, repeat surgery to gain adequate margins may be required.

Malignant Melanoma
The main diagnostic features (major criteria):Change in size. Change in shape. Change in colour
Secondary features (minor criteria): Diameter >6mm. Inflammation. Oozing or bleeding. Altered sensation

Treatment: Suspicious lesions should undergo excision biopsy. The lesion should be removed in completely as incision biopsy
can make subsequent histopathological assessment difficult. Once the diagnosis is confirmed the pathology report should be
reviewed to determine whether further re-exicision of margins is required (see below):

Margins of excision-Related to Breslow thickness: Lesions 0-1mm thick:1cm. Lesions 1-2mm thick:1- 2cm (Depending upon
site and pathological features). Lesions 2-4mm thick:2-3 cm (Depending upon site and pathological features). Lesions >4 mm
thick:3cm

Further treatments such as sentinel lymph node mapping, isolated limb perfusion and block dissection of regional lymph node
groups should be selectively applied.

Kaposi Sarcoma: Tumour of vascular and lymphatic endothelium. Purple cutaneous nodules. Associated with immunosupression.
Classical form affects elderly males and is slow growing. Immunosupression form is much more aggressive and tends to affect
those with HIV related disease.

Non malignant skin disease: Dermatitis Herpetiformis: Chronic itchy clusters of blisters. Linked to underlying gluten
enteropathy (coeliac disease).

236
Dermatofibroma: Benign lesion. Firm elevated nodules. Usually history of trauma. Lesion consists of histiocytes, blood vessels
and fibrotic changes.

Pyogenic granuloma: Overgrowth of blood vessels. Red nodules. Usually follow trauma. May mimic amelanotic melanoma.

Acanthosis nigricans: Brown to black, poorly defined, velvety hyperpigmentation of the skin. Usually found in body folds such
as the posterior and lateral folds of the neck, the axilla, groin, umbilicus, forehead, and other areas. The most common cause of
acanthosis nigricans is insulin resistance, which leads to increased circulating insulin levels. Insulin spillover into the skin results
in its abnormal increase in growth (hyperplasia of the skin). In the context of a malignant disease, acanthosis nigricans is a
paraneoplastic syndrome and is then commonly referred to as acanthosis nigricans maligna. Involvement of mucous membranes is
rare and suggests a coexisting malignant condition

179.A 55 year old man from Hong Kong presents with left sided otalgia and recurrent episodes of epistaxis. On
examination his pharynx appears normal. Examination of his neck reveals left sided cervical lymphadenopathy. What
is the most likely underlying diagnosis?
A. Antrochoanal polyp
B. Nasopharyngeal carcinoma
C. Adenocarcinoma of the tonsil
D. Angiofibroma
E. Globus syndrome

Answer: B
Given this mans ethnic origin and presenting features a nasopharyngeal carcinoma is the most likely underlying diagnosis.

180.Patients with suspected temporal arteritis are often sent for temporal artery biopsy. Which statement is true?
A. Temporal artery biopsy is only diagnostic if there is visual loss
B. Biopsy is typically taken from the non-symptomatic side to avoid the risk of blindness
C. Pre-operative localisation with duplex is mandatory
D. Biopsies may be non diagnostic in over 50% of cases
E. Biopsies are usually performed under general anaesthesia

Answer: D
Temporal artery biopsies are frequently non diagnostic. They should be taken from the symptomatic side and though not
mandatory a duplex ultrasound is a helpful investigation, particularly if they mark the artery. It is usually performed under local
anaesthetic.

Temporal artery biopsy: Superficial temporal artery is a terminal branch of the external carotid artery

Main indication: Temporal arteritis


American College of Rheumatology guidelines recommend a temporal artery biopsy if: Age of onset older than 50 years. New-
onset headache or localized head pain. Temporal artery tenderness to palpation or reduced pulsation. ESR > 50 mm/h

Histopathology: Vessel wall granulomatous arteritis with mononuclear cell infiltrates and giant cell formation

Procedure: Position: supine, head 45 degrees. USS doppler to locate the superficial temporal artery or palpate. Local anaesthetic.
Artery within temporoparietal fascia. Clamp and ligate the vessel. Cut 3-5cm. Ligate the remaining ends with absorbable suture.
Close the skin

Contraindication
Glucocorticoid therapy > 30 days
Risks
Injury to facial or auriculotemporal nerve

181.Which of the following best describes the processes underpinning type IV hypersensitivity reactions?
A. Deposition of immune complexes of IgG and antigen at the site of inflammation
B. Deposition of IgA complexes at the site of inflammation
C. Deposition of IgM and IgG complexes at the site of inflammation
D. Degranulation of mast cells at the site of inflammation
E. T cell mediated response at the site of inflammation

Answer: E
Theme: Bone tumours
237
A. Osteosarcoma
B. Fibrosarcoma
C. Osteoclastoma
D. Ewings sarcoma
E. Leiomyosarcoma
F. Chondrosarcoma
G. Rhabdomyosarcoma
H. Osteoid osteoma
I. Malignant fibrous histiocytoma

Please select the most appropriate lesion for the clinical scenario given. Each option may be used once, more than once or not at
all.

182.A 16 year-old boy presents to his GP with loss of weight, pain and fever. On examination, a soft tissue mass is palpable
over the mid-thigh region
Answer: Ewings sarcoma
Ewing's sarcoma is a malignant round cell tumour occurring in the diaphysis of the long bones in the children. These are not
confined to the ends of long bones. x Rays often show a large soft-tissue mass with concentric layers of new bone formation (
'onion-peel' sign). The ESR may be elevated, thus suggesting an inflammatory or an infective cause such as osteomyelitis;
although osteomyelitis usually affects the metaphyseal region in children. Treatment is with chemotherapy and surgical
excision, an endoprothesis may be used to conserve the limb.

183. A 75 year old lady presents with weight loss, pain and a swelling over her left knee. She has been treated for Pagets
disease of the bone for some time.
Answer: Osteosarcoma
Osteosarcoma may complicate Pagets disease of bone in up to 10% cases. Radiological appearances include bone destruction
coupled with new bone formation, periosteal elevation may also occur. Surgical resection is the main treatment.

184. A 17-year-old girl presents with weight loss, fever and a swelling over her right knee. Movements of her knee are
restricted. A plain x-ray of the affected site shows multiple lytic and lucent lesions with clearly defined borders.
You answered Malignant fibrous histiocytoma
Answer: Osteoclastoma
Osteoclastoma has a characteristic appearance on x-ray with multple lytic and lucent areas (Soap bubble) appearances.
Pathological fractures may occur. The disease is usually indolent.

185. A 22 year old man is undergoing an abdominal ultrasound scan as part of a series of investigations for abdominal
pain. The radiologist notes that there is evidence of splenic atrophy. What is the most likely cause?

A. Letterer-Siwe disease
B. Coeliac disease
C. Malaria
D. Niemann-Pick disease
E. Sarcoidosis
Answer:
B
Splenic atrophy may occur in coeliac disease together with the appearance of Howell-Jolly bodies in erythrocytes. Letterer - Siwe
disease is a form of Histiocytosis X in which macrophages proliferate.

186. Which statement relating to phaeochromocytoma is untrue?

A. They are tumours of chromaffin cells in the adrenal medulla.


B. They are bilateral in 10% of cases.
C. When located in an extra adrenal location have a higher incidence of malignancy.
D. May be associated with an elevated urinary VMA.
E. Up to 40% may have a blood pressure within the normal range.

Answer: E
Normotension is seen in around 10% cases. The remainder show a degree of hypertension. Neuroendocrine tumour of the
chromaffin cells of the adrenal medulla. Hypertension an

187. A 69 year old man presents with a purple lesion on his forearm. It is excised and an a 3 cm Merkel cell tumour is
diagnosed. Which of the following statements relating to this diagnosis is false?

238
A. He should undergo a sentinel lymph node biopsy.
B. Lymphovascular invasion is typically seen histologically
C. They are more common in immunosupressed patients
D. Histologically they may resemble pyogenic granuloma
E. They are associated with visceral metastasis
Answer: D
Merkel cell tumours are rare cutaneous tumours. Histologically they consist of sheets and nodules of hyperchromatic epithelial
cells, with high rates of mitosis and apoptosis. As such they are relatively easy to distinguish from pyogenic granuloma which has
no features of malignancy and would not show lymphovascular invasion.

Merkel cell tumours of the skin: Rare but aggressive tumour. Develops from intra epidermal Merkel cells. Usually presents on
elderly, sun damaged skin. The periorbital area is the commonest site. Histologically these tumours appear within the dermis and
subcutis. The lesions consist of sheets and nodules of small hyperchromatic epithelial cells with high rates of mitosis and
apoptosis. Lymphovascular invasion is commonly seen. Pre-existing infection with Merkel Cell Polyomavirus is seen in 80%
cases.
TreatmentSurgical excision is first line. Margins of 1cm are required. Lesions >10mm in diameter should undergo sentinel lymph
node biopsy. Adjuvant radiotherapy is often given to reduce the risk of local recurrence.
Prognosis: With lymph node metastasis 5 year survival is 50% or less. Small lesions without nodal spread are usually associated
with a 5 year survival of 80%.

188. A 58 year old lady undergoes a screening mammogram and appearances are suspicious for ductal carcinoma in situ.
A stereotactic core biopsy is performed. If ductal carcinoma in situ is to be diagnosed, which of the following pathological
features must not be present?

A. Nuclear pleomorphism
B. Coarse chromatin
C. Abnormal mitoses
D. Angiogenesis
E. Dysplastic cells infiltrating the suspensory ligaments of the breast

Answer: E

The presence of invasion is a hallmark of invasive disease and thus would not be a feature of DCIS. Angiogenesis may occur in
association with high grade DCIS.

189. Which of the following does not occur as a pathological response to extensive burns?

A. Plasma leakage into interstitial space


B. Polycythaemia
C. Increased haematocrit
D. Keratinocyte migration during healing
E. Cardiac output reduction by 50% in first 30 minutes

Answer: B
Haemolysis is the main pathological response.

Theme: Chest pain

A. Pulmonary embolism
B. Acute exacerbation asthma
C. Physiological
D. Mitral valve stenosis
E. Aortic dissection
F. Mitral regurgitation
G. Bronchopneumonia
H. Tuberculosis
I. None of the above

239
What is the most likely diagnosis for the scenario given? Each option may be used once, more than once or not at all.

190.A 28 year old Indian woman, who is 18 weeks pregnant, presents with increasing shortness of breath, chest pain and
coughing clear sputum. She is apyrexial, blood pressure is 140/80 mmHg, heart rate 130 bpm and saturations 94% on
15L oxygen. On examination there is a mid diastolic murmur, there are bibasal crepitations and mild pedal oedema.
She suddenly deteriorates and has a respiratory arrest. Her chest x-ray shows a whiteout of both of her lungs.
Answer: Mitral valve stenosis
Mitral stenosis is the commonest cause of cardiac abnormality occurring in pregnant women. Mitral stenosis is becoming
less common in the UK population, however should be considered in women from countries were there is a higher incidence
of rheumatic heart disease. Mitral stenosis causes a mid diastolic murmur which may be difficult to auscultate unless the
patient is placed into the left lateral position. These patients are at risk of atrial fibrillation (up tp 40%), which can also
contribute to rapid decompensation. Physiological changes in pregnancy may cause an otherwise asymptomatic patient to
suddenly deteriorate. Balloon valvuloplasty is the treatment of choice.

191.A 28 year old woman, who is 30 weeks pregnant, presents with sudden onset chest pain associated with loss of
consciousness. Her blood pressure is 170/90 mmHg, saturations on 15L oxygen 93%, heart rate 120 bpm and she is
apyrexial. On examination there is an early diastolic murmur, occasional bibasal creptitations and mild peal oedema.
An ECG shows ST elevation in leads II, III and aVF.
Answer: Aortic dissection
Aortic dissection is associated with the 3rd trimester of pregnancy, connective tissue disorders (Marfan's, Ehlers- Danlos)
and bicuspid valve. Patients may complain of a tearing chest pain or syncope. Clinically they may be hypertensive. The right
coronary artery may become involved in the dissection, causing myocardial infarct in up to 2% cases (hence ST elevation in
the inferior leads). An aortic regurgitant murmur may be auscultated.

192.A 28 year old woman, who is 18 weeks pregnant, presents with sudden chest pain. Her blood pressure is 150/70
mmHg, saturations are 92% on 15L oxygen and her heart rate is 130 bpm. There are no murmurs and her chest is
clear. There is signs of thrombophlebitis in the left leg.
Answer: Pulmonary embolism
Chest pain, hypoxia and clear chest on auscultation in pregnancy should lead to a high suspicion of pulmonary embolism.
Pregnant women can decompensate rapidly from cardiac compromise.

Chest pain in pregnancy

Aortic dissection: Predisposing factors in pregnancy are hypertension, congenital heart disease and Marfan's syndrome. Mainly
Stanford type A dissections. Sudden tearing chest pain, transient syncope. Patient may be cold and clammy, hypertensive and have
an aortic regurgitation murmur. Involvement of the right coronary artery may cause inferior myocardial infarction

Surgical management (by gestational timeframe): < 28/40: Aortic repair with the fetus kept in utero; 28-32/40:Dependent on
fetal condition; > 32/40:Primary Cesarean section followed by aortic repair at the same operation

Mitral stenosis: Most cases associated with rheumatic heart disease. Becoming less common in British women; suspect in
Immigrant women. Commonest cardiac condition in pregnancy. Commonly associated with mortality. Valve surgery; balloon
valvuloplasty preferable

Pulmonary embolism: Leading cause of mortality in pregnancy. Half dose scintigraphy; CT chest if underlying lung disease
should aid diagnosis. Treatment with low molecular weight heparin throughout pregnancy and 4-6 weeks after childbirth.
Warfarin is contra indicated in pregnancy

193.A 67 year old man is investigated for biliary colic and a 4.8 cm abdominal aortic aneurysm is identified. Which of the
following statements relating to this condition is untrue?
A. The wall will be composed of dense fibrous tissue only
B. The majority are located inferior to the renal arteries
C. They occur most often in current or former smokers
D. He should initially be managed by a process of active surveillance
E. Aortoduodenal fistula is a recognised complication following repair.
Answer:
A
They are true aneurysms and have all 3 layers of arterial wall.

240
Abdominal aorta aneurysm: Abdominal aortic aneurysms are a common problem in vascular surgery. They may occur as either
true or false aneurysm. With the former all 3 layers of the arterial wall are involved, in the latter only a single layer of fibrous
tissue forms the aneurysm wall. True abdominal aortic aneurysms have an approximate incidence of 0.06 per 1000 people. They
are commonest in elderly men and for this reason the UK is now introducing the aneurysm screening program with the aim of
performing an abdominal aortic ultrasound measurement in all men aged 65 years.

Causes: Several different groups of patients suffer from aneurysmal disease. The commonest group is those who suffer from
standard arterial disease, i.e. Those who are hypertensive, have diabetes and have been or are smokers. Other patients such as
those suffering from connective tissue diseases such as Marfan's may also develop aneurysms. In patients with abdominal aortic
aneurysms the extracellular matrix becomes disrupted with a change in the balance of collagen and elastic fibres.

Management: Most abdominal aortic aneurysms are an incidental finding. Symptoms most often relate to rupture or impending
rupture. 20% rupture anteriorly into the peritoneal cavity. Very poor prognosis. 80% rupture posteriorly into the retroperitoneal
space. The risk of rupture is related to aneurysm size, only 2% of aneurysms measuring less than 4cm in diameter will rupture
over a 5 year period. This contrasts with 75% of aneurysms measuring over 7cm in diameter. This is well explained by La Places'
law which relates size to transmural pressure. For this reason most vascular surgeons will subject patients with an aneurysm size
of 5cm or greater to CT scanning of the chest, abdomen and pelvis with the aim of delineating anatomy and planning treatment.
Depending upon co-morbidities, surgery is generally offered once the aneurysm is between 5.5cm and 6cm.
Indications for surgery: Symptomatic aneurysms (80% annual mortality if untreated). Increasing size above 5.5cm if
asymptomatic. Rupture (100% mortality without surgery)

Surgical procedures
Abdominal aortic aneurysm repair: Procedure: GA. Invasive monitoring (A-line, CVP, catheter). Incision:Midline or
transverse. Bowel and distal duodenum mobilised to access aorta. Aneurysm neck and base dissected out and prepared for cross
clamp. Systemic heparinisation. Cross clamp (distal first). Longitudinal aortotomy
Atherectomy. Deal with back bleeding from lumbar vessels and inferior mesenteric artery. Insert graft either tube or bifurcated
depending upon anatomy. Suture using Prolene (3/0 for proximal , distal anastomosis suture varies according to site)
Clamps off: End tidal CO2 will rise owing to effects of reperfusion, at this point major risk of myocardial events. Haemostasis.
Closure of aneurysm sac to minimise risk of aorto-enteric fistula. Closure: Loop 1 PDS or Prolene to abdominal wall. Skin-
surgeons preference. Post operatively: ITU (Almost all). Greatest risk of complications following emergency repair.
Complications: Embolic- gut and foot infarcts. Cardiac - owing to premorbid states, reperfusion injury and effects of cross clamp.
Wound problems. Later risks related to graft- infection and aorto-enteric fistula

Special groups: Supra renal AAA: These patients will require a supra renal clamp and this carries a far higher risk of
complications and risk of renal failure. Ruptured AAA: Preoperatively the management depends upon haemodynamic instability.
In patients with symptoms of rupture (typical pain, haemodynamic compromise and risk factors) then ideally prompt laparotomy.
In those with vague symptoms and haemodynamic stability the ideal test is CT scan to determine whether rupture has occurred or
not. Most common rupture site is retroperitoneal 80%. These patients will tend to develop retroperitoneal haematoma. This can be
disrupted if Bp is allowed to rise too high so aim for Bp 100mmHg. Operative details are similar to elective repair although
surgery should be swift, blind rushing often makes the situation worse. Plunging vascular clamps blindly into a pool of blood at
the aneurysm neck carries the risk of injury the vena cava that these patients do not withstand. Occasionally a supracoeliac clamp
is needed to effect temporary control, although leaving this applied for more than 20 minutes tends to carry a dismal outcome.
EVAR: Increasingly patients are now being offered endovascular aortic aneurysm repair. This is undertaken by surgeons and
radiologists working jointly. The morphology of the aneurysm is important and not all are suitable. Here is a typical list of those
features favoring a suitable aneurysm: Long neck. Straight iliac vessels. Healthy groin vessels
Clearly few AAA patients possess the above and compromise has to be made. The use of fenestrated grafts can allow supra renal
AAA to be treated.
Procedure: GA. Radiology or theatre. Bilateral groin incisions. Common femoral artery dissected out. Heparinisation. Arteriotomy
and insertion of guide wire. Dilation of arteriotomy. Insertion of EVAR Device. Once in satisfactory position it is released.
Arteriotomy closed once check angiogram shows good position and no endoleak
Complications: Endoleaks depending upon site are either Type I or 2. These may necessitate re-intervention and all EVAR
patients require follow up . Details are not needed for MRCS.

194.Which of the following statements in relation to the p53 tumour suppressor protein is false?
A. It may induce necrosis of cells with non repairable DNA damage
B. It is affected in Li Fraumeni syndrome
C. It can induce DNA repair
D. It can halt the cell cycle
E. It may inhibit angiogenesis

Answer: A
When DNA cannot be repaired it will induce cellular apoptosis (not necrosis)

195.Which of the following cell types is most likely to be identified in the wall of a fistula in ano?
241
A. Squamous cells
B. Goblet cells
C. Columnar cells
D. Ciliated columnar cells
E. None of the above

Answer: A
A fistula is an abnormal connection between two epithelial lined surfaces, in the case of a fistula in ano it will be lined by
squamous cells.

Fistulas: A fistula is defined as an abnormal connection between two epithelial surfaces. There are many types ranging from
Branchial fistulae in the neck to entero-cutaneous fistulae abdominally. In general surgical practice the abdominal cavity generates
the majority and most of these arise from diverticular disease and Crohn's. As a general rule all fistulae will resolve
spontaneously as long as there is no distal obstruction. This is particularly true of intestinal fistulae.

The four types of fistulae are: Enterocutaneous


These link the intestine to the skin. They may be high (>1L) or low output (<1L) depending upon source. Duodenal /jejunal
fistulae will tend to produce high volume, electrolyte rich secretions which can lead to severe excoriation of the skin. Colo-
cutaneous fistulae will tend to leak faeculent material. Both fistulae may result from the spontaneous rupture of an abscess cavity
onto the skin (such as following perianal abscess drainage) or may occur as a result of iatrogenic input. In some cases it may even
be surgically desirable e.g. mucous fistula following sub total colectomy for colitis.Suspect if there is excess fluid in the drain.

Enteroenteric or Enterocolic: This is a fistula that involves the large or small intestine. They may originate in a similar manner
to enterocutaneous fistulae. A particular problem with this fistula type is that bacterial overgrowth may precipitate malabsorption
syndromes. This may be particularly serious in inflammatory bowel disease.

Enterovaginal: Aetiology as above.

Enterovesicular: This type of fistula goes to the bladder. These fistulas may result in frequent urinary tract infections, or the
passage of gas from the urethra during urination.

Management: Some rules relating to fistula management: They will heal provided there is no underlying inflammatory bowel
disease and no distal obstruction, so conservative measures may be the best option. Where there is skin involvement, protect the
overlying skin, often using a well fitted stoma bag- skin damage is difficult to treat. A high output fistula may be rendered more
easily managed by the use of octreotide, this will tend to reduce the volume of pancreatic secretions. Nutritional complications are
common especially with high fistula (e.g. high jejunal or duodenal) these may necessitate the use of TPN to provide nutritional
support together with the concomitant use of octreotide to reduce volume and protect skin. When managing perianal fistulae
surgeons should avoid probing the fistula where acute inflammation is present, this almost always worsens outcomes. When
perianal fistulae occur secondary to Crohn's disease the best management option is often to drain acute sepsis and maintain that
drainage through the judicious use of setons whilst medical management is implemented. Always attempt to delineate the fistula
anatomy, for abscesses and fistulae that have an intra abdominal source the use of barium and CT studies should show a track. For
perianal fistulae surgeons should recall Goodsall's rule in relation to internal and external openings.
196.A 22 year old man is referred to the surgical clinic. He has been complaining of varicose veins for many years. On
examination he has extensive varicosities of the right leg, there are areas of marked port wine staining. The
saphenofemoral junction is competent on doppler assessment. The most likely underlying diagnosis is:
A. Deep vein thrombosis
B. Klippel-Trenaunay syndrome
C. Varicose veins due to sapheno-popliteal junction incompetence
D. Sturge - Weber syndrome
E. Angiosarcoma

Answer: B
Sturge - Weber syndrome is a an arteriovenous malformation affecting the face and CNS, the peripheral vessels are not affected.
Simple varicose veins should not typically be associated with port wine staining, not should a DVT or angiosarcoma.

Klippel-Trenaunay syndrome: Klippel-Trenaunay-Weber syndrome generally affects a single extremity, although cases of
multiple affected limbs have been reported. The leg is the most common site followed by the arms, the trunk, and rarely the head
and the neck

Signs and symptoms: The birth defect is diagnosed by the presence of a combination of these symptoms: One or more distinctive
port-wine stains with sharp borders. Varicose veins. Hypertrophy of bony and soft tissues, that may lead to local gigantism or
shrinking. An improperly developed lymphatic system. In some cases, port-wine stains (capillary port wine type) may be absent.
Such cases are very rare and may be classified as "atypical Klippel-Trenaunay syndrome". KTS can either affect blood vessels,

242
lymph vessels, or both. The condition most commonly presents with a mixture of the two. Those with venous involvement
experience increased pain and complications.

197.A 68 year old man presents with an ulcerated lesion on his right cheek. It is excised and on histological assessment a
squamous cell carcinoma is diagnosed. It measures 25mm in diameter and is 4mm deep. Which of the following
statements relating to this condition is false?
A. In this particular case margins of at least 6mm are required
B. Use of cryosurgery to treat this patients lesion would have been unsafe
C. Use of radiotherapy to treat this lesion would have been unsafe
D. This patients local recurrence rate may approach 15%
E. The disese usually spreads via lymphatics
Answer:
C

Poor prognostic factors in SCC: Size >20mm (local recurrence rate of up to 15%). Depth greater than 4mm (risk of metastasis up
to 30%)

This man has an SCC with significant risk of metastasis. Although cryotherapy may be used to treat SCC it would be most unsafe
in this setting as the lesion extends deeply. However, radiotherapy is a safe treatment modality for SCC and may be used in
selected cases. It is unwise to use radiotherapy in areas prone to radionecrosis e.g. the nose.

Squamous cell carcinoma of the skin: Second most common skin malignancy. Derived from epidermal keratinocytes.
Commonest in fair skinned individuals in sun exposed sites. May occur in perianal and genital skin especially in association with
Human Papilloma Virus 16 and 18 infections.

Groups at high risk: Renal transplant and on immunosuppression. Individuals with HIV. Those who have received psoralen
UVA therapy. Chronic wounds (Marjolins ulcer). Xeroderma pigmentosum. Oculocutaneous albinism

Prognosis: Good Prognosis: Well differentiated tumours. <20mm diameter. <2mm deep. No associated diseases. Poor
prognosis: Poorly differentiated tumours. >20mm in diameter. >4mm deep. Immunosupression for whatever reason

Treatment: Surgical excision with 4mm margins if lesion <20mm in diameter. If tumour >20mm then margins should be 6mm.

198.A 23 year old man presents with weight loss fatigue and lymphadenopathy. He is diagnosed with tuberculosis. Which of
the following processes most closely matches the underlying pathological process?
A. Type 1 hypersensitivity reaction
B. Type 2 hypersensitivity reaction
C. Type 3 hypersensitivity reaction
D. Type 4 hypersensitivity reaction
E. None of the above

Answer: D
Granulomas (which occur in tuberculosis) are a feature of Type 4 hypersensitivity reactions.

199.A 73 year old man undergoes an emergency amputation for severe lower limb sepsis and gangrene. Post operatively he
develops disseminated intravascular coagulation. Which of the following clotting factors will be most rapidly consumed
in this process?
A. Factor V and VIII
B. Factor I
C. Factor I and III
D. Factor III and VII
E. Factor VI and VIII
Answer: A
DISSEM-NATED: D-Dx: D dimer, I-Immune complexes, S-Snakebite, shock, heatstroke, S-SLE, E-Eclampsia, HELLP
syndrome, M-Massive tissue damage, I-Infections: viral and bacterial, N-Neoplasms, A-Acute promyelocytic leukemia , T-Tumor
products: Tissue Factor (TF) and TF-like factors released by carcinomas of pancreas, prostate, lung,
colon, stomach, E-Endotoxins (bacterial), D-Dead fetus (retained). DIC Will tend to consume factors five and eight intially (and
platelets).

243
Disseminated intravascular coagulation: Simultaneous coagulation and haemorrhage caused by initially formation of thrombi
which consume clotting factors (factors 5,8) and platelets, ultimately leading to bleeding
Causes include:Infection. Malignancy. Trauma e.g. major surgery, burns, shock, dissecting aortic aneurysm. Liver disease.
Obstetric complications
Key points: Clinically bleeding is usually a dominant feature, bruising, ischaemia and organ failure. Blood tests: prolonged
clotting times, thrombocytopenia, decreased fibrinogen, increased fibrinogen degradation products. Treat the underlying cause and
supportive management

200.A 63 year old Japanese man presents with epigastric discomfort and iron deficiency anaemia. He undergoes an upper
GI endoscopy, where the following appearances are found: The most likely diagnosis is:
A. Squamous cell carcinoma
B. Linitis plastica
C. Leiomyosarcoma
D. Gastric varices
E. None of the above
Answer:
B
During upper GI endoscopy, a linitis plastica lesion may prevent gastric distension.
Linitis plastica produces a diffuse infiltrating lesion, the stomach is fibrotic and rigid and will not typically distend. This may be
described as a 'leather bottle stomach'. Diagnosis is made with a combination of pathology examination with endoscopy,
radiological or surgical assessment. Pathologically signet-ring cell proliferation occurs.

Theme: Genetics and cancer

A. Multiple endocrine neoplasia type I


B. Multiple endocrine neoplasia type II
C. Gardner's syndrome
D. Lynch Syndrome
E. Kartagener's syndrome
F. Neurofibromatosis Type I
G. Neurofibromatosis Type II

Please select the most likely condition for the disease process described. Each option may be used once, more than once or not at
all.

201.A 40 year old male is found to have multiple colonic polyps during a colonoscopy. He mentions that he has extra
teeth.
Answer: Gardner's syndrome
Gardner's syndrome is an AD disorder, characterised by: Colonic polyps, supernumerary teeth, jaw osteomas, congenital
hypertrophy of retinal pigment. osteomas of the skull, thyroid cancer, epidermoid cysts, fibromas and sebaceous cysts.

202.A 10 year old boy who has learning difficulties, reports a difference in size between his two legs.
Answer: Neurofibromatosis Type I
Neurofibromatosis type I. A hallmark finding is a plexiform neurofibroma, which is a sheet of neurofibromatosis tissue
which encases major nerves. In children this attracts extra blood circulation, which can accelerate growth of the affected
limb.
Other features include: Schwannoma, > 6. Cafe au lait spots, axillary freckling, Lisch nodules, Optic glioma.
Meningiomas,Glioma, or Schwannoma.

203.A 22 year old is found to have bilateral acoustic neuromas.


Answer: Neurofibromatosis Type II
In NF2 bilateral acoustic neuromas are characteristic with a FH of Neurofibroma,

204.A 52 year old lady is referred to the breast clinic with symptoms of nipple discharge. The discharge is usually thick
and green. Which of the following statements relating to the most likely underlying diagnosis is untrue?
A. The majority of patients will be smokers
B. Typically produces blood stained nipple discharge
C. It is not associated with increased risk of breast cancer
D. May result in development of mammary duct fistula
E. May require total duct excision (Hadfields operation) if it fails to resolve
Answer:
B

244
Blood stained nipple discharge should always be investigated.
Nipple fluid cytology is generally unhelpful.
Discharge of this type of material is most likely to be due to duct ectasia. Green or brown discharge is most common. Blood
stained discharge should raise concern of intraductal papilloma or cancer.

Causes of nipple discharge: Physiological: During breast feeding. GalactorrhoeaCommonest cause may be response to
emotional events, drugs such as histamine receptor anatagonists are also implicated. Hyperprolactinaemia: Commonest type of
pituitary tumour. Microadenomas <1cm in diameter. Macroadenomas >1cm in diameter. Pressure on optic chiasm may cause
bitemporal hemianopia. Mammary duct ectasia: Dilatation breast ducts. Most common in enopausal women. Discharge typically
thick and green in colour. Most common in smokers. Carcinoma: Often blood stained. May be underlying mass or axillary
lymphadenopathy
Intraductal papilloma: Commoner in younger patients. May cause blood stained discharge. There is usually no palpable lump

Assessment of patients: Examine breast and determine whether there is mass lesion present. All mass lesions should undergo
Triple assessment.

Reporting of investigations
Where a mass lesion is suspected or investigations are requested these are prefixed using a system that denotes the investigation
type e.g. M for mammography, followed by a numerical code as shown below: 1:No abnormality; 2:Abnormality with benign
features; 3:Indeterminate probably benign; 4:Indeterminate probably malignant; 5:Malignant

Management of non malignant nipple discharge: Exclude endocrine disease. Nipple cytology unhelpful. Smoking cessation
advice for duct ectasia. For duct ectasia with severe symptoms, total duct excision may be warrented.

205.Which of the following statements relating to gas gangrene is untrue?


A. There is necrosis with putrefaction
B. The causative pathogens may be detected on normal perineal skin
C. Treatment with low dose penicillin is indicated
D. Hyperbaric oxygen may be beneficial
E. Clostridium perfringens is a recognised cause

Answer: C
Rapid surgery and high dose antibiotics are indicated in the treatment of gas gangrene.

Meleney's Gangrene and Necrotising Fasciitis.


Necrotising fasciitis: Advancing soft tissue infection associated with fascial necrosis. Uncommon, but can be fatal. In many cases
there is underlying background immunosuppression e.g. Diabetes. Caused by polymicrobial flora (aerobic and anaerobic) and
MRSA is seen increasingly in cases of necrotising fasciitis. Streptococcus is the commonest organism in isolated pathogen
infection (15%). Meleneys gangrene: Meleneys is a similar principle but the infection is more superficially sited than necrotising
fasciitis and often confined to the trunk. Fournier gangrene: Necrotising fasciitis affecting the perineum. Polymicrobial with
E.coli and Bacteroides acting in synergy. Clinical features: Fever. Pain. Cellulitis. Oedema. Induration. Numbness. Late
findings: Purple/black skin discolouration. Blistering. Haemorrhagic bullae. Crepitus. Dirty Dishwater fluid discharge. Septic
shock. Diagnosis is mainly clinical
Management: Radical surgical debridement forms the cornerstone of management. Sterile dressing is used to dress the wound.
Reconstructive surgery is considered once the infection is completely treated

206.A 30 year old man presents with abdominal distension, a laparotomy is performed, at operation the abdomen is filled
with a large amount of gelatinous fluid. What is the most likely underlying diagnosis?
A. Infection with entamoeba histolytica
B. Pseudomxyoma peritonei
C. Metastatic colonic cancer
D. Chylous ascites
E. None of the above

Answer: B Pseudomyxoma is associated with the deposition of large amounts of gelatinous material. The appendix is the
commonest organ or origin.

Pseudomyxoma Peritonei: Rare mucinous tumour. Most commonly arising from the appendix (other abdominal viscera are also
recognised as primary sites). Incidence of 1-2/1,000,000 per year. The disease is characterised by the accumulation of large
amounts of mucinous material in the abdominal cavity. Treatment: Is usually surgical and consists of cytoreductive surgery (and

245
often peritonectomy c.f Sugarbaker procedure) combined with intra peritoneal chemotherapy with mitomycin C.
Survival is related to the quality of primary treatment and in Sugarbakers own centre 5 year survival rates of 75% have been
quoted. Patients with disseminated intraperitoneal malignancy from another source fare far worse.
In selected patients a second look laparotomy is advocated and some practice this routinely.

207.A 30 year old man is suspected of having appendicitis. At operation an inflamed Meckels diverticulum is found. Which
of the following vessels is responsible for the blood supply to a Meckels diverticulum?
A. Right colic artery
B. Vitelline artery
C. Appendicular artery
D. Internal iliac artery
E. External iliac artery

Answer: B
The vitelline arteries supply a Meckels these are usually derived from the ileal arcades.

208.A 23 year old man suffers a thermal injury to his left hand. It becomes red and painful. Which of the following
mediators are not involved in this process?
A. Histamine
B. Free radicals
C. Prostaglandins
D. Leukotrienes
E. Serotonin

Answer: B
Acute inflammation is not mediated by free radicals. Chemical mediators facilitate the spread of inflammation into normal tissue
Chemical mediators include: Lysosomal compounds. Chemokines such as serotinin and histamine (released by platelets and mast
cells). Other enzyme cascades producing inflammatory mediators include: Complement, kinin, coagulation system & fibrinolytic
system

209.Which of the following associations are incorrect?


A. Afro-Caribbean skin and keloid scarring
B. Extensive third degree burns and wound contraction
C. Chemotherapy and dehisence of healed wounds
D. Poor healing at the site of previous radiotherapy
E. Zinc deficiency and delayed healing
Answer: C

Wound healing: Surgical wounds are either incisional or excisional and either clean, clean contaminated or dirty. Although the
stages of wound healing are broadly similar their contributions will vary according to the wound type.

210.A 23 year old man fractures his right tibia in a sporting accident. At which point in the healing process is fracture
callus most likely to be visible radiologically?
A. 1 day
B. 7 days
C. 8 weeks
D. 6 weeks
E. 3 weeks

Answer: E
Fracture callus is composed of fibroblasts and chondroblasts and the synthesis of fibrocartilage. It is typically visible on
radiographs approximately 3 weeks following injury. If delayed then there may be risk of non union.

Fracture healing

Bone fracture: Bleeding vessels in the bone and periosteum. Clot and haematoma formation. The clot organises over a week
(improved structure and collagen). The periosteum contains osteoblasts which produce new bone. Mesenchymal cells produce
cartilage (fibrocartilage and hyaline cartilage) in the soft tissue around the fracture. Connective tissue + hyaline cartilage = callus.
As the new bone approaches the new cartilage, endochondral ossification occurs to bridge the gap. Trabecular bone forms.
Trabecular bone is resorbed by osteoclasts and replaced with compact bone

Factors Affecting Fracture Healing: Age, Malnutrition, Bone disorders: osteoporosis. Systemic disorders: diabetes. Marfan's
syndrome and Ehlers-Danlos syndrome cause abnormal musculoskeletal healing. Drugs: steroids, non steroidal anti inflammatory
agents. Type of bone: Cancellous (spongy) bone fractures are usually more stable, involve greater surface areas, and have a better

246
blood supply than cortical (compact) bone fractures. Degree of Trauma: The more extensive the injury to bone and surrounding
soft tissue, the poorer the outcome.Vascular injury: Especially the femoral head, talus, and scaphoid bones. Degree of
immobilization.
Intra-articular Fractures: These fractures communicate with synovial fluid, which contains collagenases that retard bone
healing. Separation of Bone Ends: Normal apposition of fracture fragments is needed for union to occur. Inadequate reduction,
excessive traction, or interposition of soft tissue will prevent healing. Infection.

211.Of the options below, which does not cause lymphadenopathy?


A. Kawasaki disease
B. Systemic Lupus Erthematosus
C. Phenytoin
D. Hydrallazine
E. Amiodarone
Answer:
E
Lymphadenopathy: Lymphadenopathy in the neck, axillae, groins and abdomen. Need to note: solitary/multiple,
defined/indistinct, hard/rubbery/soft, tender/painless

212.A 23 year old man is reviewed on the ward 10 days following a laparotomy. The wound is inspected and is healing well.
Which of the following processes is least likely to be occurring in the wound at this stage?
A. Angiogenesis
B. Synthesis of collagen
C. Necrosis of fibroblasts
D. Secretion of matrix metalloproteinases by fibroblasts
E. Proliferation of fibroblasts
Answer:
C

Vasculogenesis vs Angiogenesis: Vascu is new. Angi is pre. Vasculogenesis is new vessels developing in situ from existing
mesenchyme. Angiogenesis is vessels develop from sprouting off pre-existing arteries. Fibroblasts are an important cell type in
healing wounds. They typically proliferate in the early phases of wound healing. They release matrix metalloproteinases and these
facilitate in the remodelling of the matrix within the healing wound. Necrosis in a healing wound would be unusual as wounds
will tend to show clinical evidence of angiognesis by this time.

213.A 25 year old women presents with a slowly enlarging mass on the side of the face. Clinical examination demonstrates
that the mass is located in the tail of the parotid gland. There is no evidence of facial nerve involvement. What is the
most likely cause?
A. Sialolithiasis
B. Adenocarcinoma
C. Warthins tumour
D. Oncocytoma
E. Pleomorphic adenoma

Answer: E
Pleomorphic adenomas are the commonest tumours of the parotid gland and are often slow growing, smooth and mobile. Warthins
tumours are typically found in elderly males and are composed of multiple cysts and solid components consisting of lymphoid
tissue. Warthins tumours are most often found in the tail of the parotid gland, but not in 25 year old females, where a pleomorphic
adenoma remains the most likely lesion.

214.Beta-naphthalamine is associated with which of the following malignancies?


A. Lung cancer
B. Bowel cancer
C. Bladder cancer
D. Liver cancer
E. Renal cancer

Answer: C
Beta-naphthalamine is used in the rubber industry.
The following factors are associated with the development of bladder cancer: smoking. Occupational: aniline dyes used in printing
and textile industry, rubber manufacture. Schistosomiasis. Drugs: cyclophosphamide

247
215.A 56 year old man with Wilsons disease presents with right upper quadrant discomfort. An ultrasound scan is
performed and this demonstrates a mass lesion in the right lobe of the liver. What is the most appropriate method of
establishing the underlying diagnosis?
A. PET CT scan
B. Ultrasound guided biopsy
C. Measurement of serum alpha feto protein
D. MRI scan of the liver
E. CT scan of the liver
Answer:
C
High AFP + chronic liver inflammation = Hepatocellular carcinoma.
This is likely to be a hepatocellulcar carcinoma. Diagnosis is usually made by AFP measurement (with further imaging depending
on the result). Biopsy should not be performed as it may seed the tumour. Chronic liver diseases such as Wilsons disease (Hepato-
lenticular degeneration) increase the risk.

216.A 55 year old man has suffered from reflux oesophagitis for many years. During a recent endoscopy a biopsy is taken
from the distal oesophagus. The histopathology report indicates that cells are identified with features of coarse
chromatin and abnormal mitoses. The cells are confined to the superficial epithelial layer only. Which of the following
accounts for this process?
A. Metaplasia
B. Apoptosis
C. Autoimmune oesophagitis
D. Dysplasia
E. Infection with Helicobacter pylori
Answer:
D
Dysplasia = pre cancerous
Dysplasia tends to develop as a result of prolonged stimulation by precipitants. Removal of these precipitants may possibly
reverse these changes. Replacement of differentiated cells wth another cell type describes metaplasia rather than dysplasia. The
absence of invasion distinguishes this from malignancy.

Dysplasia: Premalignant condition. Disordered growth and differentiation of calls. Alteration in size, shape, and organization of
cells. Features increased abnormal cell growth (increased number of mitoses/abnormal mitoses and cellular differentiation).
Underlying connective tissue is not invaded. Causes include smoking, Helicobacter pylori, Human papilloma virus. Main
differences to metaplasia is that dysplasia is considered to be part of carcinogenesis (pre cancerous) and is associated with a delay
in maturation of cells rather than differentiated cells replacing one another. The absence of invasion differentiates dysplasia from
invasive malignancy. Severe dysplasia with foci of invasion are well recognised.

217.A 63 year old man has a history of claudication that has been present for many years. He is recently evaluated in the
clinic and a duplex scan shows that he has an 85% stenosis of the superficial femoral artery. Two weeks later he
presents with a 1 hour history of severe pain in his leg. On examination he has absent pulses in the affected limb and it
is much cooler than the contra-lateral limb. Which process best accounts for this presentation?
A. Thrombosis
B. Embolus
C. Atheroma growth
D. Sub intimal dissection
E. Anaemia

Answer: A
In an existing lesion a complication such as thrombosis is more likely than embolus. These patients should receive heparin and
imaging with duplex scanning. Whilst an early surgical bypass or intra-arterial thrombolysis may be indicated, an embolectomy
should not generally be performed as the lesion is not an embolus and the operation therefore ineffective.

Claudication: claudication is a condition in which patients develop pain in a limb during periods of exercise. The underlying
disorder is usually that of arterial insufficiency. Atheroma develops in the arterial wall and once this occludes >50-75% of the
lumenal diameter the supply to metabolising tissues distally may become compromised. The typical claudicant complains of calf
pain that is worse on exercise and relieved by rest. This typical description assumes that the SFA is the site of disease, more
proximal disease may present with other symptoms such as buttock claudication and impotence.
The history is usually a progressive one, patients presenting as an emergency with severe pain, diminished sensation, pallor and
absent pulses have critical limb ischaemia. This may complicate claudication and usually indicates a plaque related complication,
such as thrombosis. Risk factors: Risk factors for claudication include smoking, diabetes and hyperlipidaemia. Diagnosis:
Diagnostic work -up includes measurement of ankle- brachial pressure indices, duplex scanning and formal angiography.
Treatment: Those with long claudication distances, no ulceration or gangrene may be managed conservatively. Patients with rest

248
pain, ulceration or gangrene will almost always require intervention. All patients should receive an antiplatelet agent and a statin
unless there are compelling contra-indications.

218.The following are true of carcinoid tumours except:


A. When present in the appendix tip and measure less than 2 cm have an excellent prognosis
B. Even when metastatic disease is present it tends to follow a protracted course
C. When present in the appendix body tend to present with carcinoid syndrome even when liver metastases are not present
D. May be imaged using 5 HIAA radionucleotide scanning
E. Advanced appendiceal carcinoids may require right hemicolectomy.

Answer: C

219.During a difficult femoro-popliteal bypass operation the surgeon inadvertently places a clamp across the femoral
nerve. It remains there for most of the procedure. At the end of the operation the nerve is inspected, it is in continuity
but has evidence of being crushed. Which of the following is most likely to occur over the following months?
A. Wallerian degeneration
B. Rapid restoration of neuronal function because the axon itself is intact
C. Normal but delayed neuronal transmission due to disruption of the myelin
D. Absence of neuroma formation
E. None of the above

Answer: A
A neuronal injury such as this will result in Wallerian degeneration even though the nerve remains in continuity. Neuromas may
well form.

220.A 38 year old lady who smokes heavily presents with recurrent episodes of infection in the right breast. On
examination she has an indurated area at the lateral aspect of the nipple areaolar complex. Imaging shows no mass
lesions. What is the most likely diagnosis?
A. Duct ectasia
B. Periductal mastitis
C. Pagets disease of the nipple
D. Mondors disease of the breast
E. Radial scar

Answer: B
Periductal mastitis is common in smokers and may present with recurrent infections. Treatment is with co-amoxyclav. Mondors
disease of the breast is a localised thrombophlebitis of a breast vein.

Duct ectasia: Duct ectasia is a dilatation and shortening of the terminal breast ducts within 3cm of the nipple. It is common and
the incidence increases with age. It typically presents with nipple retraction and occaisionally creamy nipple discharge. It may be
confused with periductal mastitis, which presents in younger women, the vast majority of which are smokers. Periductal mastitis
typically presents with periareolar or sub areolar infections and may be recurrent.
Patients with troublesome nipple discharge may be treated by microdochectomy (if young) or total duct excision (if older).

221.Which of the following statements relating to chronic inflammation is true?


A. Chronic inflammation is mainly secondary to acute inflammation
B. Neutrophils are the predominant cells involved
C. Growth factors are not involved in the process
D. Appendicitis is mainly a form of chronic inflammation
E. Fibrosis is a macroscopic feature
Answer: E
Macroscopic features include: Ulcers, Fibrosis, Granulomatous process. It most commonly occurs as a primary event rather than as
a result of acute inflammation.

Theme: Paediatric ano-rectal disorders

A. Ulcerative colitis
B. Juvenile polyps
C. Haemorroids
D. Intussceception
E. Rectal cancer
F. Anal fissure
G. Arteriovenous malformation
249
Please select the most likely cause for the condition described. Each option may be used once, more than once or not at all.

222.A 4 year old boy is brought to the clinic. He gives a history of difficult, painful defecation with bright red rectal
bleeding.
Answer: Anal fissure
Painful rectal bleeding in this age group is typically due to a fissure. Treatment should include stool softeners and lifestyle
advice.

223.A 2 year old has a history of rectal bleeding. The parents notice that post defecation, a cherry red lesion is present at
the anal verge.
Answer: Juvenile polyps
These lesions are usually hamartomas and this accounts for the colour of the lesions. Although the lesions are not themselves
malignant they serve as a marker of an underlying polyposis disorder.

224.A 12 year old is brought to the colorectal clinic with a history of rectal bleeding, altered bowel habit, weight loss and
malaise. Abdominal examination is normal.
Answer: Ulcerative colitis
The systemic features in the history are strongly suggestive of inflammatory bowel disease rather than the other causes.

Paediatric proctology
Children may present with altered bowel habit and/ or rectal bleeding. Classical haemorroidal disease is relatively rare in children.
Painful bright red rectal bleeding is much more common since constipation is a relatively common childhood disorder. The hard
stool causes a tear of the ano-rectal mucosa with subsequent fissure. The pain from the fissure must be addressed promptly or the
child will delay defecation and this fissure will worsen. Inflammatory bowel disease may present in a similar pattern in paediatric
practice with altered bowel habit (usually diarrhoea) and bleeding. Systemic features may be present and investigation with an
endoscopy may be required. Children with intussceception usually present at a relatively young age and the history is usually one
of colicky abdominal pain, together with a mass on clinical examination.The often cited red current jelly type stool is a rare but
classical feature.
Juvenile polyps may occur as part of the familial polyposis coli syndromes. The lesions, which are hamartomas, are often cherry
red if they protrude externally.

225.You review a 42-year-old woman 8 months following a renal transplant for focal segmental glomerulosclerosis. She is
on a combination of tacrolimus, mycophenolate, and prednisolone. She has now presented with a five day history of
feeling generally unwell with jaundice, fatigue and arthralgia. On examination she has jaundice, widespread
lymphadenopathy and hepatomegaly. What is the most likely diagnosis?

A. Hepatitis C
B. Epstein-Barr virus
C. HIV
D. Hepatitis B
E. Cytomegalovirus
Answer: B
Post transplant complications: CMV: 4 weeks to 6 months post transplant. EBV: post transplant lymphoproliferative disease. > 6
months post transplant. Post transplant lymphoproliferative disorder is most commonly associated with Epstein-Barr virus. It
typically occurs 6 months post transplant and is associated with high dose immunosupressant therapy. Remember
cytomegalovirus presents within the first 4 weeks to 6 months post transplant.

Renal transplant:HLA typing and graft failure: The human leucocyte antigen (HLA) system is the name given to the major
histocompatibility complex (MHC) in humans. It is coded for on chromosome 6.
Some basic points on the HLA system: Class 1 antigens include A, B and C. Class 2 antigens include DP,DQ and DR. When HLA
matching for a renal transplant the relative importance of the HLA antigens are as follows DR > B > A

Graft survival: 1 year = 90%, 10 years = 60% for cadaveric transplants. 1 year = 95%, 10 years = 70% for living-donor transplants
Post-op problems: ATN of graft. Vascular thrombosis. Urine leakage. UTI. Hyperacute acute rejection: Due to antibodies
against donor HLA type 1 antigens. Rarely seen due to HLA matching. Acute graft failure (< 6 months): Usually due to
mismatched HLA. Other causes include cytomegalovirus infection. Management: give steroids, if resistant use monoclonal
antibodies. Causes of chronic graft failure (> 6 months): Chronic allograft nephropathy. Ureteric obstruction. Recurrence of
original renal disease (MCGN > IgA > FSGS)

226.Which of the following processes facilitates phagocytosis?


A. Apoptosis
250
B. Opsonisation
C. Proteolysis
D. Angiogenesis
E. Necrosis
Answer: B
Opsonisation will facilitate phagocytosis. The micro-organism becomes coated with antibody, C3b and certain acute phase
proteins. The macrophages and neutrophils have up regulation of phagocytic cell surface receptors in these circumstances, a
process mediated by pro-inflammatory cytokines. These cells then engulf the micro organism.

Phagocytosis: Ingestion of pathogens or foreign materials by cells. First step is opsonisation whereby the organism is coated by
antibody. Second step is adhesion to cell surface. Third step is pseudopodial extension to form a phagocytic vacuole. Lysosomes
fuse with vacuole and degrade contents.

227.Features which are evaluated for the grading of breast cancer include all the following, except:
A. Tubule formation
B. Mitoses
C. Nuclear pleomorphism
D. Tumour necrosis
E. Coarse chromatin
Answer: D
The necrosis of a tumour may be suggestive of a high grade tumour which has out grown its blood supply. However, the grading of
breast cancer which classically follows the Bloom -Richardson grading model will tend to favor nuclear appearances (which include
mitoses, coarse chromatin and pleomorphism). Tubule formation is an important marker of the degree of differentiation with
formation of tubular structures being associated with well differentiated tumours.

Tumour grading: Tumours may be graded according to their degree of differentiation, mitotic activity and other features. Grade 1
tumours are the most differentiated and grade 4 the least. The assessment is subjective, in most cases high grade equates to poor
prognosis, or at least rapid growth. Tumours of glandular epithelium will tend to arrange themselves into acinar type structures
containing a central lumen. Well differentiated tumours may show excellent acinar formation and poorly differentiated tumours
simply clumps of cells around a desmoplastic stroma. Sometimes tumours demonstrate mucous production without evidence of
acinar formation. Since mucous production is evidence of a glandular function such tumours are often termed mucinous
adenocarcinoma.
Squamous cell tumours will typically produce structures resembling epithelial cell components. Well differentiated tumours may
also produce keratin (depending upon tissue of origin).

228.Which of the following is associated with poor wound healing?


A. Jaundice
B. Patients taking carbamazepine
C. General anaesthesia using thiopentone
D. General anaesthesia using ketamine
E. Multiple sclerosis
Answer: A
Mnemonic to remember factors affecting wound healing: DID NOT HEAL: D iabetes, I nfection, irradiation, D rugs eg steroids,
chemotherapy, N utritional deficiencies (vitamin A, C & zinc, manganese), Neoplasia, O bject (foreign material), T issue necrosis,
H ypoxia, E xcess tension on wound, A nother wound, L ow temperature, Liver jaundice
Multiple sclerosis is associated with pressure sores, however the cellular healing process is not affected.

229.A 55 year old man undergoes a colonoscopy and a colonic polyp is identified. It has a lobular appearance and is located
on a stalk in the sigmoid colon. Which of the processes below best accounts for this disease?
A. Apoptosis
B. Metaplasia
C. Dysplasia
D. Calcification
E. Degeneration
Answer: C
Most colonic polyps described above are adenomas. These may have associated dysplasia. The more high grade the dysplasia the
greater the level of clinical concern.

230.56 year old lady has just undergone a colonoscopy and a lesion was identified in the caecum. The histology report states
that biopsies have been taken from a sessile serrated polyp with traditional features? What is the best management
option?
A. Perform a right hemicolectomy
B. List the patient for colonoscopic polypectomy
251
C. Discharge the patient
D. Re scope the patient in 6 months
E. Re scope the patient at 3 years

Answer: B
These polyps represent an alternative pathway to progression to carcinoma and may be diagnostically confused with hyperplastic
polyps. Hyperplastic polyps are more common in the left colon and confer no increased risk. SSA's are more common in the right
colon and are usually larger. Those with "traditional features" on histology have dysplasia with increased risk of malignant
transformation.

231.A 30 year old male presents with gynaecomastia. Clinically he is noted to have a nodule in the left testis. The most likely
diagnosis is:
A. Oestrogen abuse
B. Seminoma with syncytiotrophoblast giant cells
C. Teratoma
D. Choriocarcinoma
E. Leydig cell tumour
Answer: E
Leydig cell tumours are rare testicular sex cord stromal tumours (which also include sertoli cell tumours) which are associated
with hormonal activity. Patients with Leydig cell tumours may present with gynaecomastia before they notice testicular
enlargement. Majority are benign. Histology: eosinophilic cells in columns

232.Which of the following features are not typical of Crohns disease?


A. Complex fistula in ano
B. Small bowel strictures
C. Skip lesions
D. 'Rose thorn ulcers' on barium studies
E. Pseudopolyps on colonoscopy

Answer: E
Pseudopolyps are a feature of ulcerative colitis and occur when there is severe mucosal ulceration. The remaining islands of mucosa

233.A 32 year old lady presents with a 1.5cm pigmented lesion on her back. The surgeon is concerned that this may be a
melanoma. What is the most appropriate course of action?
A. 2mm punch biopsy from the centre of the lesion
B. 4mm punch biopsy from the centre of the lesion
C. Wide excision of the lesion with 3cm margins
D. Excisional biopsy of the lesion
E. Wide excision of the lesion with 1cm margins
Answer:
D
Suspicious naevi should NOT be partially sampled as histological interpretation is severely compromised. Complete excision is
mandatory where lesions fulfil diagnostic criteria. However, wide excision for margins may be deferred until definitive histology
is available. Lesions that are suspicious for melanoma should be excised with complete margins. Radical excision is not routinely
undertaken for diagnostic purposes and therefore if subsequent histopathological assessment determines that the lesion is a
melanoma a re-exicision of margins may be required. Incisional punch biopsies of potential melanomas makes histological
interpretation difficult and is best avoided.

234.A 70 year old male presents with painless frank haematuria. Clinical examination is unremarkable. Routine blood tests
reveal a haemoglobin of 18g/dl but are otherwise normal. What is the most likely underlying diagnosis?
A. Squamous cell carcinoma of the bladder
B. Adenocarcinoma of the prostate
C. Adenocarcinoma of the kidney
D. Wilms tumour
E. Transitional cell carcinoma of the renal pelvis
Answ

er: C

Theme from April 2012 Exam. Polycythaemia is a recognised feature of renal cell carcinoma. Wilms tumours most commonly
occur in children.

Haematuria: Causes of haematuria. Trauma: Injury to renal tract. Renal trauma commonly due to blunt injury (others
penetrating injuries). Ureter trauma rare: iatrogenic. Bladder trauma: due to RTA or pelvic fractures. Infection: Remember TB.
252
Malignancy: Renal cell carcinoma (remember paraneoplastic syndromes): painful or painless. Urothelial malignancies: 90% are
transitional cell carcinoma, can occur anywhere along the urinary tract. Painless haematuria. Squamous cell carcinoma and
adenocarcinoma: rare bladder tumours. Prostate cancer. Penile cancers: SCC. Renal disease: Glomerulonephritis.
Stones:Microscopic haematuria common. Structural abnormalities: Benign prostatic hyperplasia (BPH) causes haematuria due
to hypervascularity of the prostate gland. Cystic renal lesions e.g. polycystic kidney disease. Vascular malformations. Renal vein
thrombosis due to renal cell carcinoma. Coagulopathy: Causes bleeding of underlying lesions. Drugs: Cause tubular necrosis or
interstitial nephritis: aminoglycosides, chemotherapy. Interstitial nephritis: penicillin, sulphonamides, and NSAIDs.
Anticoagulants. Benign: Exercise. Gynaecological:Endometriosis: flank pain, dysuria, and haematuria that is cyclical.
Iatrogenic: Catheterisation. Radiotherapy; cystitis, severe haemorrhage, bladder necrosis. Pseudohaematuri

Theme: Thyroid neoplasms

A. Follicular carcinoma
B. Follicular adenoma
C. Papillary carcinoma
D. Papillary adenoma
E. Anaplastic carcinoma
F. Medullary carcinoma

Please select the most likely underlying diagnosis for the thyroid masses described. Each option may be used once, more than
once or not at all.

235.A 78 year old lady presents to the surgical clinic with symptoms of both dysphagia and dyspnoea. On examination
there is a large mass in the neck that moves on swallowing. CT scanning of the neck shows a locally infiltrative lesion
arising from the thyroid and invading the strap muscles.
Answer: Anaplastic carcinoma
Marked local invasion is a feature of anaplastic carcinoma. These tumours are more common in elderly females.

236.A 25 year old female presents with a lump in her neck. On examination she has a discrete nodule in the right lobe of the
thyroid. A fine needle aspirate shows papillary cells. An adjacent nodule is also sampled which shows similar well
differentiated papillary cells.
Answer: Papillary carcinoma
Multifocal disease is a recognised feature of papillary lesions. Papillary adenomas are not really recognised and most well
differentiated lesions are papillary carcinomas.

237.A 45 year old man presents with a fracture of his right humerus. On examination there is a lytic lesion of the proximal
humerus and a mass in the neck, this moves on swallowing.
Answer: Follicular carcinoma
Follicular carcinomas are a recognised source of bone metastasis. Up to 60% will show vascular invasion histologically.

238.Which of the following lesions is most closely associated with Barretts oesophagus?
A. Squamous cell carcinoma
B. Gastro intestinal stromal tumours
C. Carcinoid tumours
D. Leiomyosarcoma
E. Adenocarcinoma

Answer: E
Barretts oesphagus is most closely associated with adenocarcinoma, and it confers at 30 fold increased risk of developing the
condition.

239.A 56 year old man presents with epigastric discomfort and episodes of migratory thrombophlebitis. On examination
he is mildly jaundiced. A CT scan shows peri hilar lymphadenopathy and a mass in the pancreatic head. Which of the
following is the most likely underlying diagnosis?
A. Squamous cell carcinoma of the pancreas
B. Adenocarcinoma of the pancreas
C. Insulinoma
D. Glucagonoma
E. Gastrinoma
Answer:
B

253
Adenocarcinoma of the pancreas is the most likely diagnosis and migratory thrombophlebitis is associated with the condition.
Squamous cells carcinoma is extremely uncommon in the pancreas. Gastrinoma are extremely rare and thus not the most likely
diagnosis.

240.A 22 year old man presents with symptoms of headache, lethargy and confusion. On examination he is febrile and has a
right sided weakness. A CT scan shows a ring enhancing lesion affecting the left motor cortex. Which of the following is
the most likely diagnosis?
A. Arteriovenous malformation
B. Cerebral abscess
C. Herpes simplex encephalitis
D. Metastatic renal adenocarcinoma
E. Glioblastoma multiforme

Answer: B
The combination of rapidly progressive neurology, fever and headache is highly suggestive of cerebral abscess. CT scanning will
show a ring enhancing lesion because the intravenous contrast cannot penetrate the centre of the abscess cavity. HSV encephalitis
does not produce ring enhancing lesions.

Brain abscess: CNS abscesses may result from a number of causes including, extension of sepsis from middle ear or sinuses,
trauma or surgery to the scalp, penetrating head injuries and embolic events from endocarditis. The presenting symptoms will
depend upon the site of the abscess (those in critical areas e.g.motor cortex) will present earlier. Abscesses have a considerable
mass effect in the brain and raised intra cranial pressure is common. Although fever, headache and focal neurology are highly
suggestive of a brain abscess the absence of one or more of these does not exclude the diagnosis, fever may be absent and even if
present, is usually not the swinging pyrexia seen with abscesses at other sites. Assessment of the patient includes imaging with CT
scanning. Treatment is usually surgical, a craniotomy is performed and the abscess cavity debrided. The abscess may reform
because the head is closed following abscess drainage.

241.A 56 year old man from Egypt has suffered from recurrent attacks of haematuria for many years. He presents with
suprapubic discomfort and at cystoscopy is found to have a mass lesion within the bladder. What is the most likely
diagnosis?
A. Squamous cell carcinoma
B. Transitional cell papilloma
C. Adenocarcinoma
D. Leiomyosarcoma
E. Rhabdomyosarcoma

Answer: A
In Egypt Schistosomiasis is more common than in the UK and may cause recurrent episodes of haematuria. In those affected with
the condition who develop a bladder neoplasm, an SCC is the most common type.

242.A newborn infant is noted to have a unilateral cleft lip only. What is the most likely explanation for this process?
A. Incomplete fusion of the second branchial arch
B. Incomplete fusion of the nasolabial muscle rings
C. Incomplete fusion of the first branchial arch
D. Incomplete fusion of the third branchial arch
E. Incomplete fusion of the secondary palate

Answer: B
Theme from April 2012 Exam: Unilateral isolated cleft lip represents a failure of nasolabial ring fusion. It is not related to
branchial arch fusion. Arch disorders have a far more profound phenotype and malformation sequences.

Cleft lip and palate: Cleft lip and palate are the most common congenital deformity affecting the orofacial structures. Whilst they
may be an isolated developmental malformation they are also a recognised component of more than 200 birth defects. The
incidence is as high as 1 in 600 live births. The commonest variants are: Isolated cleft lip (15%), Isolated cleft palate (40%).
Combined cleft lip and palate (45%). The aetiology of the disorder is multifactorial; both genetic (affected first degree relative
increases risk) and environmental factors play a role.
Cleft lip: Cleft lip occurs as a result of disruption of the muscles of the upper lip and nasolabial region. These muscles comprise a
chain of muscles viz; nasolabial, bilabial and labiomental. Defects may be unilateral or bilateral.
Cleft palate: The primary palate consists of all anatomical structures anterior to the incisive foramen. The seconday palate lies
more posteriorly and is sub divided into the hard and soft palate. Cleft palate occurs as a result of non fusion of the two palatine
shelves. Both hard and soft palate may be involved. Complete cases are associated with complete separation of the nasal septum
and vomer from the palatine processes.
Treatment: Surgical reconstruction is the mainstay of management. The procedures are planned according to the extent of
254
malformation and child age. Simple defects are managed as a single procedure. Complex malformations are usually corrected in
stages. Affected individuals have a higher incidence of hearing and speech problems.

243.A 72 year old woman with back pain and chronic renal failure has the following results: Ca2+ 2.03 (2.15-2.55 mmol/l);
Parathyroid hormone 10.4 (1-6.5 pmol/l); Phosphate 0.80 (0.6-1.25 mmol/l). What is the most likely diagnosis?
A. Hypoparathyroidism
B. Primary hyperparathyroidism
C. Secondary hyperparathyroidism
D. Tertiary hyperparathyroidism
E. Pseudohypoparathyroidism

Answer: C
In relation to secondary hyperparathyroidism; there is a HIGH PTH and the Ca2+ is NORMAL or LOW. In secondary
hyperparathyroidism there in hyperplasia of the parathyroid glands in response to chronic hypocalcaemia (or hyperphosphataemia)
and is a normal physiological response. Calcium is restored from bone, kidneys and the gastrointestinal system.

Theme: Pharyngitis

A. Infectious mononucleosis

B. Acute bacterial tonsillitis

C. Quinsy

D. Lymphoma

E. Diptheria

Please select the most likely underlying cause for the following patients presenting with pharyngitis. Each option may be used
once, more than once or not at all.

244.An 8 year old child presents with enlarged tonsils that meet in the midline and are covered with a white film that
bleeds when you attempt to remove it. He is pyrexial but otherwise well.
Answer: Acute bacterial tonsillitis
Theme from April 2012 Exam: In acute tonsillitis the tonsils will often meet in the midline and may be covered with a
membrane. Individuals who are systemically well are unlikely to have diptheria.

245.A 10 year old child presents with enlarged tonsils that meet in the midline. Oropharyngeal examination confirms this
finding and you also notice peticheal haemorrhages affecting the oropharynx. On systemic examination he is noted to
have splenomegaly.
Answer: Infectious mononucleosis
A combination of pharyngitis and tonsillitis is often seen in glandular fever. Antibiotics containing penicillin may produce a
rash when given in this situation, leading to a mistaken label of allergy.

246.A 19 year old man has had a sore throat for the past 5 days. Over the past 24 hours he has notices increasing and
severe throbbing pain in the region of his right tonsil. He is pyrexial and on examination he is noted to have a swelling
of this area.
Answer: Quinsy
Unilateral swelling and fever is usually indicative of quinsy. Surgical drainage usually produces prompt resolution of
symptoms.

Acute tonsillitis: Characterised by pharyngitis, fever, malaise and lymphadenopathy. Over half of all cases are bacterial with
Streptococcus pyogenes the most common organism. The tonsils are typically oedematous and yellow or white pustules may be
present. Infectious mononucleosis may mimic the condition.Treatment with penicillin type antibiotics is indicated for bacterial
tonsillitis. Bacterial tonsillitis may result in local abscess formation (quinsy)

247.A 4 year old boy is brought to the clinic by his mother who has noticed a small lesion at the external angle of his eye.
On examination there is a small cystic structure which has obviously been recently infected. On removal of the scab,
there is hair visible within the lesion. What is the most likely diagnosis?
A. Dermoid cyst
B. Desmoid cyst
C. Sebaceous cyst
255
D. Epidermoid cyst
E. Keratoacanthoma
Answer: A
Dermoid cysts occur at sites of embryonic fusion and may contain multiple cell types. They occur most often in children.
The lesion is unlikely to be a desmoid cyst as these are seldom located either at this site or in this age group. In addition they do
not contain hair. Sebaceous cysts will usually have a punctum and contain a cheesy material. Epidermoid cysts contain keratin
plugs.

Dermoid cysts: A cutaneous dermoid cyst may develop at sites of embryonic developmental fusion. They are most common in
the midline of the neck, external angle of the eye and posterior to the pinna of the ear. They typically have multiple inclusions
such as hair follicles that bud out from its walls. They may develop at other sites such as the ovary and in these sites are
synonymous with teratomas. A desmoid tumour is a completely different entity and may be classified either as low grade
fibrosarcomas or non aggressive fibrous tumours. They commonly present as large infiltrative masses. They may be divided into
abdominal, extra abdominal and intra abdominal. All types share the same biological features. Extra abdominal desmoids have an
equal sex distribution and primarily arise in the musculature of the shoulder, chest wall, back and thigh. Abdominal desmoids
usually arise in the musculoaponeurotic structures of the abdominal wall. Intra abdominal desmoids tend to occur in the mesentery
or pelvic side walls and occur most frequently in patients with familial adenomatous polyposis coli syndrome.

248.Which of the following tumours are most likely to give rise to para-aortic nodal metastasis early?
A. Ovarian
B. Bladder
C. Rectal
D. Caecal
E. Cervical

Answer: A
Theme from 2009 Exam. Ovarian tumours are supplied by the ovarian vessels, these branch directly from the aorta. The cervix
drains to the internal and external iliac nodes.

Para-aortic lymphadenopathy: Organ sites that may metastasise (early) to the para-aortic lymph nodes: Testis. Ovary. Uterine
fundus. Many other organs may result in para-aortic nodal disease. However, these deposits will represent a much later stage of
the disease, since other nodal stations are involved earlier.

249.Which of the following statements relating to malignant mesothelioma is false?


A. It may be treated by extrapleural pneumonectomy.
B. It is linked to asbestos exposure.
C. It is linked to cigarette smoking independent of asbestos exposure.
D. It may occur intra abdominally.
E. It is relatively resistant to radiotherapy
Answer:
C
It is not linked to cigarette smoking. When identified at an early stage a radical resection is the favored option. Radiotherapy is
often given perioperatively. However, it is not a particularly radiosensitive tumour. Combination chemotherapy gives some of the
best results and most regimes are cisplatin based.

Theme: Thyroid disorders

A. Sick euthyroid
B. Hyperthyroidism
C. Hypothyroidism
D. Normal euthyroid
E. Anxiety state
F. Factitious hyperthyroidism

For each of the scenarios please match the scenario with the most likely underlying diagnosis. Each answer may be used once,
more than once or not at all.

250.A 33 year old man is recovering following a protracted stay on the intensive care unit recovering from an anastomotic
leak following a difficult Trans hiatal oesophagectomy. His progress is slow, and the intensive care doctors receive the
following thyroid function test results: TSH 1.0 u/L, Free T48, T3 1.0 (1.2-3.1 normal)
256
Answer: Sick euthyroid
Theme from April 2012 Exam
Sick euthyroid syndrome is caused by systemic illness. With this, the patient may have an apparently low total and free T4 and
T3, with a normal or low TSH. Note that the levels are only mildly below normal.

251.A 28 year old female presents to the general practitioner with symptoms of fever and diarrhoea. As part of her
diagnostic evaluation the following thyroid function tests are obtained: TSH< 0.01, Free T4 30, T3 4.0
Answer: Hyperthyroidism
The symptoms are suggestive of hyperthyroidism. This is supported by the abnormal blood results; suppressed TSH with an
elevated T3 and T4.

252.A 19 year old lady presents with palpitations. The medical officer takes a blood sample for thyroid function tests.
The following results are obtained:TSH > 6.0, Free T4 20, T3 2.0
Answer: Hypothyroidism
An elevated TSH with normal T4 indicates partial thyroid failure. This is caused by Hashimotos, drugs (lithium,
antithyroids) and dyshormogenesis.

Hyperthyroidism
Causes of hyperthyroidism include: Diffuse toxic goitre (Graves Disease). Toxic nodular goiter. Toxic nodule. Rare causes
Graves disease: Graves disease is characterised by a diffuse vascular goitre that appears at the same time as the clinical
manifestations of hyperthyroidism. It is commonest in younger females and may be associated with eye signs. Thyrotoxic
symptoms will predominate. Up to 50% of patients will have a familial history of autoimmune disorders. The glandular
hypertrophy and hyperplasia occur as a result of the thyroid stimulating effects of the TSH receptor antibodies.
Toxic nodular goiter: In this disorder the goitre is present for a long period of time prior to the development of clinical
symptoms. In most goitres the nodules are inactive and in some cases it is the internodular tissue that is responsible for the goitre.
Toxic nodule: Overactive, autonomously functioning nodule. It may occur as part of generalised nodularity or be a true toxic
adenoma. The TSH levels are usually low as the autonomously functioning thyroid tissue will exert a negative feedback effect.

Clinical features: Symptoms: Lethargy. Emotionally labile. Heat intolerance. Weight loss. Excessive appetite. Palpitations.
Signs: Tachycardia. Agitation. Hot, moist palms. Exopthalmos. Thyroid goitre and bruit. Lid lag/retraction. Diagnosis: The most
sensitive test for diagnosing hyperthyroidism is plasma T3 (which is raised). Note in hypothyroidism the plasma T4 and TSH are
the most sensitive tests. A TSH level of <0.5U/L suggests hyperthyroidism. TSH receptor antibodies may be tested for in the
diagnosis of Graves. Treatment: First line treatment for Graves disease is usually medical and the block and replace regime is the
favored option. Carbimazole is administered at higher doses and thyroxine is administered orally. Patient are maintained on this
regime for between 6 and 12 months. Attempts are then made to wean off medication. Where relapse then occurs the options are
between ongoing medical therapy, radioiodine or surgery.

253.Which of the following conditions is least likely to exhibit the Koebner phenomenon?
A. Vitiligo
B. Molluscum contagiosum
C. Lichen planus
D. Psoriasis
E. Lupus vulgaris

Answer: E
Lupus vulgaris is not associated with the Koebner phenomenon.

Koebner phenomenon: The Koebner phenomenon describes skin lesions which appear at the site of injury. It is seen in:
Psoriasis. Vitiligo. Warts. Lichen planus. Lichen sclerosus. Molluscum contagiosum

254.Which of the tumour types listed below is found most frequently in a person with aggressive fibromatosis?
A. Medullary thyroid cancer
B. Basal cell carcinoma of the skin
C. Desmoid tumours
D. Dermoid tumours
E. Malignant melanoma
Answer: C
Aggressive fibromatosis: Aggressive fibromatosis is a disorder consisting of desmoid tumours, which behave in a locally
aggressive manner. Desmoid tumours may be identified in both abdominal and extra-abdominal locations. Metastatic disease is
rare. The main risk factor (for abdominal desmoids) is having APC variant of familial adenomatous polyposis coli. Most cases are
257
sporadic.
Treatment is by surgical excision.

255.A 20 year old man is suspected of having an inflamed Meckels diverticulum. At which of the following locations is it
most likely to be found?
A. Approximately 60 cm distal to the ileo-caecal valve
B. Approximately 60 cm proximal to the ileocaecal valve
C. Approximately 200cm distal to the ileocaecal valve
D. Approximately 200cm proximal to the ileocaecal valve
E. 50cm distal to the DJ flexure

Answer: B
256.A 43 year old lady presents with severe chest pain. Investigations demonstrate a dissecting aneurysm of the ascending
aorta which originates at the aortic valve. What is the optimal long term treatment?
A. Endovascular stent
B. Medical therapy with beta blockers
C. Medical therapy with ACE inhibitors
D. Sutured aortic repair
E. Aortic root replacement
Answer: E
Proximal aortic dissections are generally managed with surgical aortic root replacement. The proximal origin of the dissection
together with chest pain (which may occur in all types of aortic dissection) raises concerns about the possibility of coronary ostial
involvement (which precludes stenting). There is no role for attempted suture repair in this situation.

257.A 45 year old lady presents with a pathological fracture of her femoral shaft. She is a poor historian, but it transpires
that she underwent a thyroidectomy 1 year previously. She has no other illness or co-morbidities. What is the most
likely underlying diagnosis?
A. Hyperparathyroidism
B. Metastatic papillary carcinoma of the thyroid
C. Metastatic medullary carcinoma of the thyroid
D. Metastatic follicular carcinoma of the thyroid
E. None of the above
Answer:
D
Follicular carcinomas are a recognised cause of bone metastasis. Papillary lesions typically spread via the lymphatics.

Theme: Cardiac murmurs

A. Pulmonary stenosis
B. Mitral regurgitation
C. Tricuspid regurgitation
D. Aortic stenosis
E. Mitral stenosis
F. Aortic sclerosis

What is the most likely cause of the cardiac murmur in the following patients? Each option may be used once, more than once or
not at all.

258.A 35 year old Singaporean female attends a varicose vein pre operative clinic. On auscultation a mid diastolic
murmur is noted at the apex. The murmur is enhanced when the patient lies in the left lateral position.
Answer: Mitral stenosis
Theme from September 2011 exam
A mid diastolic murmur at the apex is a classical description of a mitral stenosis murmur. The most common cause is
rheumatic heart disease. Complications of mitral stenosis include atrial fibrillation, stroke, myocardial infarction and
infective endocarditis.

259.A 22 year old intravenous drug user is found to have a femoral abscess. The nursing staff contact the on call doctor as
the patient has a temperature of 39oC. He is found to have a pan systolic murmur loudest at the left sternal edge at
the 4th intercostal space.
Answer: Tricuspid regurgitation
Intravenous drug users are at high risk of right sided cardiac valvular endocarditis. The character of the murmur fits with a
diagnosis of tricuspid valve endocarditis.
258
260.An 83 year old woman is admitted with a left intertrochanteric neck of femur fracture. On examination the patient is
found to have an ejection systolic murmur loudest in the aortic region. There is no radiation of the murmur to the
carotid arteries. Her ECG is normal.
Answer: Aortic sclerosis
The most likely diagnosis is aortic sclerosis. The main differential diagnosis is of aortic stenosis, however as there is no
radiation of the murmur to the carotids and the ECG is normal, this is less likely.

Cardiac murmurs: Ejection systolic: Aortic stenosis. Pulmonary stenosis, HOCM. ASD, Fallot's. Pan-systolic: Mitral
regurgitation. Tricuspid regurgitation. VSD. Late systolic:Mitral valve prolapse. Coarctation of aorta. Early diastolic: Aortic
regurgitation. Graham-Steel murmur (pulmonary regurgitation). Mid diastolic: Mitral stenosis. Austin-Flint murmur (severe
aortic regurgitation)

261.A 55 year old lady presents with discomfort in the right breast. On clinical examination a small lesion is identified
and clinical appearances suggest fibroadenoma. Imaging confirms the presence of a fibroadenoma alone. A core
biopsy is taken, this confirms the presence of the fibroadenoma. However, the pathologist notices that a small area of
lobular carcinoma in situ is also present in the biopsy. What is the best management?
A. Whole breast irradiation
B. Simple mastectomy
C. Mastectomy and sentinal lymph node biopsy
D. Wide local excision and sentinel lymph node biopsy
E. Breast MRI scan
Answer:
E
Lobular carcinoma in situ has a low association with invasive malignancy. It is seldom associated with microcalcification and
therefore MRI is the best tool for determining disease extent. Resection of in situ disease is not generally recommended and most
surgeons would simply pursue a policy of close clinical and radiological follow up.

Lobular carcinoma of the breast: Lobular breast cancers are less common than their ductal counterparts. They typically present
differently, the mass is usually more diffuse and less obvious on the usual imaging modalities of ultrasound and mammography.
This is significant since the disease may be understaged resulting in inadequate treatment when wide local excision is undertaken.
In women with invasive lobular carcinoma it is usually safest to perform an MRI scan of the breast, if breast conserving surgery is
planned. Lobular carcinomas are also more likely to be multifocal and metastasise to the contralateral breast. Lobular carcinoma
in situ is occasionally diagnosed incidentally on core biopsies. Unlike DCIS, lobular carcinoma in situ is far less strongly
associated with foci of invasion and is usually managed by close monitoring.

262.Which one of the following may be associated with an increased risk of venous thromboembolism?
A. Diabetes
B. Cannula
C. Hyperthyroidism
D. Tamoxifen
E. Amiodarone
Answer: D

Consider thromboembolism in breast cancer patients on tamoxifen!

Venous thromboembolism: risk factors: Common predisposing factors include malignancy, pregnancy and the period following
an operation. The comprehensive list below is partly based on the 2010 SIGN venous thromboembolism (VTE) guidelines:
General: increased risk with advancing age. Obesity. Family history of vte. Pregnancy (especially puerperium). Immobility.
Hospitalisation. Anaesthesia. Central venous catheter: femoral >> subclavian

Underlying conditions: Malignancy. Thrombophilia: e.g. Activated protein c resistance, protein c and s deficiency. Heart failure.
Antiphospholipid syndrome. Behcet's. Polycythaemia. Nephrotic syndrome. Sickle cell disease. Paroxysmal nocturnal
haemoglobinuria. Hyperviscosity syndrome. Homocystinuria

Medication: combined oral contraceptive pill: 3rd generation more than 2nd generation. Hormone replacement therapy.
Raloxifene and tamoxifen. Antipsychotics (especially olanzapine) have recently been shown to be a risk factor

SIGN also state that the following are risk factors for recurrent VTE: Previous unprovoked VTE. Male sex. Obesity.
Thrombophilias
259
263.A 42 year old lady is investigated for symptoms of irritability and altered bowel habit. On examination she is noted to
have a smooth enlargement of the thyroid gland. As part of her investigations thyroid function tests are requested, these
are as follows: TSH: 0.1 mug/l, Free T4:35 pmol/l. The most likely underlying diagnosis is:
A. Multinodular goitre
B. Follicular carcinoma of the thyroid gland
C. Graves disease
D. Pregnancy
E. None of the above

Answer: C
TSH receptor antibodies will cause stimulation of the thyroid to synthesise T4. However, this will have a negative feedback effect
on the pituitary causing decrease in TSH levels.
Where hyperthyroidism occurs secondary to pregnancy the TSH is typically elevated.

Thyroid disease: Patients may present with a number of different manifestations of thyroid disease. They can be broadly sub
classified according to whether they are euthyroid or have clinical signs of thyroid dysfunction. In addition it needs to be
established whether they have a mass or not.
Assessment: History: Examination including USS. If a nodule is identified then it should be sampled ideally via an image guided
fine needle aspiration. Radionucleotide scanning is of limited use

Thyroid Tumours: Papillary carcinoma. Follicular carcinoma. Anaplastic carcinoma. Medullary carcinoma. Lymphoma's

Multinodular goiter: One of the most common reasons for presentation. Provided the patient is euthyroid and asymptomatic and
no discrete nodules are seen, they can be reassured. In those with compressive symptoms surgery is required and the best
operation is a total thyroidectomy. Sub total resections were practised in the past and simply result in recurrent disease that
requires a difficult revisional resection.

Endocrine dysfunction: In general these patients are managed by physicians initially. Surgery may be offered alongside radio
iodine for patients with Graves disease that fails with medical management or in patients who would prefer not to be irradiated
(e.g. pregnant women).Patients with hypothyroidism do not generally get offered a thyroidectomy. Sometimes people
inadvertently get offered resections during the early phase of Hashimotos thyroiditis, however, with time the toxic phase passes
and patients can simply be managed with thyroxine.

Complications following surgery: Anatomical such as recurrent laryngeal nerve damage. Bleeding. Owing to the confined space
haematoma's may rapidly lead to respiratory compromise owing to laryngeal oedema. Damage to the parathyroid glands resulting
in hypocalcaemia.

264.Which of the following is not a risk factor for developing tuberculosis?


A. Gastrectomy
B. Solid organ transplantation with immunosupression
C. Intravenous drug use
D. Haematological malignancy
E. Amiodarone

Answer: E
Risk factors for developing active tuberculosis include: Silicosis. Chronic renal failure. Hiv positive. Solid organ transplantation
with immunosuppression. Intravenous drug use. Haematological malignancy. Anti-tnf treatment. Previous gastrectomy

Tuberculosis: is an infection caused by Mycobacterium tuberculosis that most commonly affects the lungs. Understanding the
pathophysiology of TB can be difficult - the key is to differentiate between primary and secondary disease. Primary
tuberculosis: A non-immune host who is exposed to M. tuberculosis may develop primary infection of the lungs. A small lung
lesion known as a Ghon focus develops. The Ghon focus is composed of tubercle-laden macrophages. The combination of a Ghon
focus and hilar lymph nodes is known as a Ghon complex. In immunocompotent people the intially lesion usually heals by
fibrosis. Those who are immunocompromised may develop disseminated disease (miliary tuberculosis). Secondary (post-
primary) tuberculosis
If the host becomes immunocompromised the initial infection may become reactivated. Reactivation generally occurs in the apex
of the lungs and may spread locally or to more distant sites. Possible causes of immunocomprise include: immunosuppressive
drugs including steroids. HIV. Malnutrition

The lungs remain the most common site for secondary tuberculosis. Extra-pulmonary infection may occur in the following areas:
central nervous system (tuberculous meningitis - the most serious complication). Vertebral bodies (Pott's disease). Cervical lymph
nodes (scrofuloderma). Renal. Gastrointestinal tract
260
265.What is the most common presentation of a parotid gland tumour?
A. Parapharyngeal mass
B. Mass at anterior border of masseter
C. Mass inferior to the angle of the mandible
D. Mass behind the angle of the mandible
E. Mass anterior to the ear

Answer: D Parotid tumours may present at any region in the gland. However, most lesions will be located behind the angle of the
mandible, inferior to the ear lobe. Tumours of the deep lobe of the parotid may present as a parapharyngeal mass and large lesions
may displace the tonsil.

266.A 52 year old male is referred to urology clinic with impotence. He is known to have hypertension. He does not have
any morning erections. On further questioning the patient reports pain in his buttocks, this worsens on mobilising. On
examination there is some muscle atrophy. The penis and scrotum are normal. What is the most likely diagnosis?
A. Leriche syndrome
B. S4-S5 cord lesion
C. Pudendal nerve lesion
D. Psychological impotence
E. Beta blocker induced impotence
Answer: A
Leriche syndrome: Classically, it is described in male patients as a triad of symptoms: 1. Claudication of the buttocks and thighs
2.Atrophy of the musculature of the legs 3. Impotence (due to paralysis of the L1 nerve)
Leriche syndrome, is atherosclerotic occlusive disease involving the abdominal aorta and/or both of the iliac arteries. Management
involves correcting underlying risk factors such as hypercholesterolaemia and stopping smoking. Investigation is usually with
angiography.

Leriche syndrome: Atheromatous disease involving the iliac vessels. Blood flow to the pelvic viscera is compromised. Patients
may present with buttock claudication and impotence (in this particular syndrome). Diagnostic work up will include angiography,
where feasible iliac occlusions are usually treated with endovascular angioplasty and stent insertion.

267.A 23 year old man presents with diarrhoea and passage of mucous. He is suspected of having ulcerative colitis. Which
of the following is least likely to be associated with this condition?
A. Superficial mucosal inflammation in the colon
B. Significant risk of dysplasia in long standing disease
C. Epsiodes of large bowel obstruction during acute attacks
D. Haemorrhage
E. Disease sparing the anal canal

Answer: C
Large bowel obstruction is not a feature of UC, patients may develop megacolon. However, this is a different entity both
diagnostically and clinically. Ulcerative colitis does not affect the anal canal and the anal transitional zone. Inflammation is
superficial. Dysplasia can occur in 2% overall, but increases significantly if disease has been present over 20 years duration.
Granulomas are features of crohn's disease.
Other features:Disease maximal in the rectum and may spread proximally. Contact bleeding. Longstanding UC crypt atrophy and
metaplasia/dysplasia

268.A 52 year old woman attends clinic for investigation of abdominal pain and constipation. On examination you note blue
lines on the gum margin. She mentions that her legs have become weak in the past few days. What is the most likely
diagnosis?
A. Acute intermittent porphyria
B. Lead poisoning
C. Constipation
D. Guillan Barre syndrome
E. Rectal carcinoma

Answer: B
This would be an impressive diagnosis to make in the surgical out patient department! The combination of abdominal pain and a
motor periperal neuropathy, should indicate this diagnosis. The blue line along the gum margin can occur in up to 20% patients
with lead poisoning.

261
Lead poisoning: Along with acute intermittent porphyria, lead poisoning should be considered in questions giving a combination
of abdominal pain and neurological signs. Features: Abdominal pain. Peripheral neuropathy (mainly motor). Fatigue.
Constipation. Blue lines on gum margin (only 20% of adult patients, very rare in children)

Investigations: The blood lead level is usually used for diagnosis. Levels greater than 10 mcg/dl are considered significant. Full
blood count: microcytic anaemia. Blood film shows red cell abnormalities including basophilic stippling and clover-leaf
morphology. Raised serum and urine levels of delta aminolaevulinic acid may be seen making it sometimes difficult to
differentiate from acute intermittent porphyria. Urinary coproporphyrin is also increased (urinary porphobilinogen and
uroporphyrin levels are normal to slightly increased). Management: various chelating agents are currently used:
Dimercaptosuccinic acid (DMSA). D-penicillamine. EDTA. Dimercaprol

Theme: Vasculitis

A. Wegeners granulomatosis
B. Polyarteritis nodosa
C. Giant cell arteritis
D. Takayasu's arteritis
E. Buergers disease

For each of the scenarios provided please select the most likely underlying diagnosis from the list below. Each option may be used
once, more than once or not at all.

269.A 20 year old lady is referred to the vascular clinic. She has been feeling generally unwell for the past six weeks. She
works as a typist and has noticed increasing pain in her forearms whilst working. On examination she has absent upper
limb pulses. Her ESR is measured and mildly elevated.
Answer: Takayasu's arteritis
Takayasus arteritis may be divided into acute systemic phases and the chronic pulseless phase. In the latter part of the disease
process the patient may complain of symptoms such as upper limb claudication. In the later stages of the condition the vessels
will typically show changes of intimal proliferation, together with band fibrosis of the intima and media.

270.A 32 year old man presents to the vascular clinic with symptoms of foot pain during exertion. He is a heavy smoker and
has recently tried to stop smoking. On examination he has normal pulses to the level of the popliteal. However, foot
pulses are absent. A diagnostic angiogram is performed which shows an abrupt cut off at the level of the anterior tibial
artery, together with the formation of corkscrew shaped collateral vessels distally.
Answer: Buergers disease
Buergers disease is most common in young male smokers. This demographic is changing in those areas where young female
smokers are more common. In the acute lesion the internal elastic lamina of the vessels is usually intact. As the disease
progresses the changes progress to hypercellular occlusive thrombus. Tortuous corkscrew collaterals may reconstitute patent
segments of the distal tibial or pedal vessels.

271.A 78 year old man presents with symptoms of headaches and deteriorating vision. He notices that there is marked pain
on the right hand side of his face when he combs his hair.
Answer: Giant cell arteritis
Temporal arteritis may present acutely with symptoms of headache and visual loss, or with a less acute clinical picture. Sight
may be threatened and treatment with immunosupressants should be started promptly. The often requested temporal artery
biopsy (which can be the bane of many surgeons) is often non diagnostic and unhelpful.

The vasculitides are a group of conditions characterised by inflammation of the blood vessel walls. This may, in turn, compromise
vessel integrity. Constitutional symptoms may be present. Whilst certain disease subtypes are reported to affect specific vessels,
there is often a degree of overlap clinically.

Vessel diameter and vasculitis classification


Aorta and branches: Takayasu's arteritis. Buergers disease. Giant cell arteritis. Large and medium sized arteries: Buergers
disease. Giant cell arteritis. Polyarteritis nodosa. Medium sized muscular arteries: Polyarteritis nodosa. Wegeners
granulomatosis. Small muscular arteries: Wegeners granulomatosis. Rheumatoid vasculitis

Specific conditions: Takyasu's arteritis: Inflammatory, obliterative arteritis affecting aorta and branches. Females> Males.
Symptoms may include upper limb claudication. Clinical findings include diminished or absent pulses. ESR often affected during
the acute phase. Buergers disease: Segmental thrombotic occlusions of the small and medium sized lower limb vessels.
Commonest in young male smokers. Proximal pulses usually present, but pedal pulses are lost. An acuter hypercellular occlusive
thrombus is often present. Tortuous corkscrew shaped collateral vessels may be seen on angiography. Giant cell arteritis:
Systemic granulomatous arteritis that usually affects large and medium sized vessels. Females > Males. Temporal arteritis is

262
commonest type. Granulomatous lesions may be seen on biopsy (although up to 50% are normal). Polyarteritis nodosa: Systemic
necrotising vasculitis affecting small and medium sized muscular arteries. Most common in populations with high prevalence of
hepatitis B. Renal disease is seen in 70% cases. Angiography may show saccular or fusiform aneurysms and arterial stenosis.
Wegeners granulomatosis: Predominantly affects small and medium sized arteries. Systemic necrotising granulomatous
vasculitis. Cutaneous vascular lesions may be seen (ulceration, nodules and purpura). Sinus imaging may show mucosal
thickening and air fluid levels. TreatmentConditions such as Buergers disease are markedly helped by smoking cessation.
Immunosupression is the main treatment for vasculitides.

272.A 28 year old man presents with right upper quadrant pain and hydatid disease is suspected. Which of the following
statements relating to the disease is untrue?
A. First line treatment is with pentavalent antimony.
B. Peritoneal contamination with active daughter cysts may complicate surgery.
C. CT scanning of the liver may show a floating membrane.
D. Biliary communication with the cysts may occur.
E. It is caused by Echinococcus granulosus.

Answer: A
Drug treatment is with albendazole or mebendazole. Praziquantzel may be used in the pre operative stages.

273.Which of the following statements relating to neutrophil polymorphs is true?


A. Produce nitrogen peroxide as a microbicidal agent
B. Not involved in opsonisation
C. Deficiency leads to AIDS
D. Neutrophil disorders cause chronic granulomatous diseases
E. Have a lifespan of 9 hours
Answer: D

Neutrophils are the main cells of acute inflammation, important action against gram -ve and +ve bacteria. Appearance of
segmented nucleus and granulated cytoplasm. Have a lifespan of 1-3 days (shorter when consumed during septic process, though
9 hours is unusual). Actions include: movement, opsonise microorganisms, phagocytosis & intracellular killing of microorganisms
via aerobic (produce HYDROGEN PEROXIDE) & anaerobic mechanisms. Neutrophil disorders include chronic granulomatous
diseases: rare. AIDS associated with T cell deficiency

274.A 33 year old man is involved in a road traffic accident. He is initially stable and transferred to the accident and
emergency department. On arrival he is catheterised. One minute later he becomes hypotensive, with evidence of
angioedema surrounding his penis. What is the most likely explanation for this event?
A. Type V latex hypersensitivity reaction
B. Type IV latex hypersensitivity reaction
C. Type III latex hypersensitivity reaction
D. Type I latex hypersensitivity reaction
E. Type II latex hypersensitivity reaction
Answer: D
Theme from April 2012 Exam. Sudden collapse and angioedema following exposure to latex (of which most urinary catheters are
manufactured) suggests a type I hypersensitivity reaction.

275.A 43 year old female develops severe chest wall cellulitis following a mastectomy. On examination the skin is markedly
erythematous. Which of the acute inflammatory mediators listed below is least likely to produce vasodilation?
A. Complement component C5a
B. Lysosomal compounds
C. Histamine
D. Serotonin
E. Prostaglandins
Answer: D
Erythema is a classical feature of acute inflammation. Potent mediators of vascular dilatation include; histamine, prostaglandins,
nitric oxide, platelet activating factor, complement C5a (and C3a) and lysosomal compounds. Although serotonin is associated
with acute inflammation it is a vasoconstrictor. The effects of serotonin are dependant upon the state of the vessels in the tissues.
Intact and healthy tissues and vessels will respond to a serotonin infusion with vasodilation (hence the flushing seen in carcinoid
syndrome). In contrast it worsens cardiac ischaemia in myocardial infarcts when released from damaged platelets.

276.A 43 year old man presents with a 3 week history of malaise, sore throat, odynophagia and dysphagia. On examination
he is found to have patchy white spots in his oropharynx. An upper GI endoscopy is performed and similar lesions are
identified in the oesophagus. Which investigation is most likely to identify the underlying pathology in this case?
263
A. Serum urea and electrolytes
B. Oesophageal biopsy for culture
C. Oesophageal biopsy for histology
D. Glucose tolerance testing
E. Viral serology

Answer: E
Oesophageal candidiasis is associated with immunosupression; mainly in patients on chemotherapy, with haematological
malignancy, HIV or inhaled steroids. In patients with HIV, oesophageal candidiasis is part of the spectrum of AIDS defining
illnesses and usually occurs when the CD4 count is less than 200. Others include PCP pneumonia and CMV infections.

Oesophageal candidiasis: Characterised by white spots in the oropharynx with extension into the oesophagus. Associated with
broad spectrum antibiotic usage, immunosupression and immunological disorders. Patients may present with oropharyngeal
symptoms, odynophagia and dysphagia. Treatment is directed both at the underlying cause (which should be investigated for) and
with oral antifungal agents.

Theme: Colonic obstruction

A. Malignant obstruction
B. Olgilvies syndrome
C. Volvulus
D. Diverticular stricture
E. Ischaemic stricture

Please select the most likely cause of obstruction for the situation described. Each option may be used once, more than once or not
at all.

277.A 78 year old man has undergone a hemi-arthroplasty for a intracapsular hip fracture. Post operatively he develops
electrolyte derangement and receives intravenous fluids. Over the previous 24 hours he develops marked abdominal
distension. On examination he has a tense, tympanic abdomen which is not painful. A contrast enema shows flow of
contrast through to the caecum and through the ileocaecal valve.
Answer: Olgilvies syndrome
Patients with electrolyte disturbance and previous surgery may develop colonic pseudo-obstruction (olgilvies syndrome). The
diagnosis is made using a contrast enema and treatment is usually directed at the underlying cause with colonic decompression
if indicated.

278.A 67 year old man has had multiple episodes with fever and left iliac fossa pain. These have usually resolved with
courses of intravenous antibiotics. He is admitted with a history of increasing constipation and abdominal distension. A
contrast x-ray is performed which shows flow of contrast to the sigmoid colon, here the contrast flows through a long
narrow segment of colon into dilated proximal bowel.
Answer: Diverticular stricture
The long history of left iliac fossa pain and development of bowel obstruction suggests a diverticular stricture. These may
contain a malignancy and most will require resection.

279.A 78 year old lady from a nursing home is admitted with a 24 hour history of absolute constipation and abdominal
pain. On examination she has a distended soft mass in her left iliac fossa. An x-ray is performed which shows a large
dilated loop of bowel in the left iliac fossa which contains a fluid level.

Answer: Volvulus
Sigmoid volvulus may present with an asymmetrical mass in an elderly patient. It may contain a fluid level, visible on plain
films.

Colonic obstruction
Cancer: Usually insidious onset. History of progressive constipation. Systemic features (e.g. anaemia). Abdominal distension
Absence of bowel gas distal to site of obstruction. Establish diagnosis (e.g. contrast enema/ endoscopy). Treatment: Laparotomy
and resection, stenting, defunctioning colostomy or bypass
Diverticular stricture: Usually history of previous acute diverticulitis. Long history of altered bowel habit
Evidence of diverticulosis on imaging or endoscopy. Once diagnosis established, usually surgical resection. Colonic stenting
should not be performed for benign disease

264
Volvulus: Twisting of bowel around its mesentery. Sigmoid colon affected in 76% cases. Patients usually present with abdominal
pain, bloating and constipation. Examination usually shows asymmetrical distension. Plain X-rays usually show massively dilated
sigmoid colon, loss of haustra and "U" shape are typical, the loop may contain fluid levels. Treatment: Initial treatment is to
untwist the loop, a flexible sigmoidoscopy may be needed. Those with clinical evidence of ischaemia should undergo surgery.
Patient with recurrent volvulus should undergo resection
Acute colonic pseudo-obstruction: Symptoms and signs of large bowel obstruction with no lesion. Usually associated with
metabolic disorders. Usually a cut off in the left colon (82% cases). Although abdomen tense and distended, it is usually not
painful. Treatment: All patients should undergo contrast enema (may be therapeutic!). Colonoscopic decompression. Correct
metabolic disorders. IV neostigmine. Surgery.

280.A 52 year old man with dyspepsia is found to have a duodenal ulcer. A CLO test is taken and is positive. Which
statement relating to the likely causative organism is false?
A. It is a gram negative organism
B. It lives only on gastric type mucosa
C. It may occupy areas of ectopic gastric metaplasia
D. In patients who are colonised there is commonly evidence of fundal gastritis on endoscopy
E. It produces a powerful urease that forms the basis of the Clo test
Answer: D
Helicobacter pylori accounts for >75% cases of duodenal ulceration. It may be diagnosed with either serology, microbiology,
histology or CLO testing.

Theme from January 2011 Exam. Helicobacter pylori rarely produces any typical features on endoscopy. Where infection is
suspected the easiest course of action is to take an antral biopsy for Clo testing in the endoscopy suite.

Helicobacter Pylori

Infection with Helicobacter Pylori is implicated in many cases of duodenal ulceration and up to 60% of patients with gastric
ulceration. Gram negative, helix shaped rod, microaerophillic. Produces hydrogenase that can derive energy from hydrogen
released by intestinal bacteria. Flagellated and mobile. Those carrying the cag A gene may cause ulcers. It secretes urease that
breaks down gastric urea> Carbon dioxide and ammonia> ammonium>bicarbonate (simplified!) The bicarbonate can neutralise
the gastric acid. Usually colonises the gastric antrum and irritates resulting in increased gastrin release and higher levels of gastric
acid. These patients will develop duodenal ulcers. In those with more diffuse H-Pylori infection gastric acid levels are lower and
ulcers develop by local tissue damage from H-Pylori- these patients get gastric ulcers. Diagnosis may be made by serology
(approx. 75% sensitive). Biopsy urease test during endoscopy probably the most sensitive. In patients who are colonised 10-20%
risk of peptic ulcer, 1-2% risk gastric cancer, <1% risk MALT lymphoma.

281.A 34 year old male presents with painful rectal bleeding and a fissure in ano is suspected. On examination he has an
epithelial defect at the mucocutaenous defect that is located anteriorly. Approximately what proportion of patients with
fissure in ano will present with this pattern of disease?
A. 90%
B. 10%
C. 50%
D. 25%
E. 100%

Answer: B
Only a minority of patients with fissure in ano will have an anteriorly sited fissure. They are particularly rare in males and an
anterior fissure in a man should prompt a search for an underlying cause.

Anal fissure: Anal fissures are a common cause of painful, bright red, rectal bleeding. Most fissures are idiopathic and present as
a painful mucocutaneous defect in the posterior midline (90% cases). Fissures are more likely to be anteriorly located in females,
particularly if they are multiparous. Multiple fissures and those which are located at other sites are more likely to be due to an
underlying cause. Diseases associated with fissure in ano include: Crohns disease. Tuberculosis . Internal rectal prolapse

Diagnosis: In most cases the defect can be visualised as a posterior midline epithelial defect. Where symptoms are highly
suggestive of the condition and examination findings are unclear an examination under anaesthesia may be helpful. Atypical
disease presentation should be investigated with colonoscopy and EUA with biopsies of the area.

Treatment: Stool softeners are important as the hard stools may tear the epithelium and result in recurrent symptoms. The most
effective first line agents are topically applied GTN (0.2%) or Diltiazem (2%) paste. Side effects of diltiazem are better tolerated.
Resistant cases may benefit from injection of botulinum toxin or lateral internal sphincterotomy (beware in females).
265
Advancement flaps may be used to treat resistant cases.Sphincterotomy produces the best healing rates. It is associated with
incontinence to flatus in up to 10% of patients in the long term.

282.The pathogenesis of osteopetrosis is best explained by a defect in which of the following?


A. Osteoclast function
B. PTH receptors
C. Osteoblast function
D. Calcium resorption in proximal tubule
E. Calcium absorption
Answer: A
Osteopetrosis: aka marble bone disease. Rare disorder of defective osteoclast function resulting in failure of normal bone
resorption. Stem cell transplant and interferon-gamma have been used for treatment

283.A 13 month old boy is brought to the surgical clinic by his mother because his left testicle is not located in the scrotum.
At which of the following sites would the testicle be located if it were an ectopic testis?
A. Canalicular
B. Inguinal
C. External inguinal ring
D. Superficial inguinal pouch
E. High scrotal
Answer: D
Theme from September 2011 Exam. Theme from January 2012 Exam. Ectopic testes are those that come to lie outside the normal
range of embryological descent (i.e. in the superficial inguinal pouch). Other sites of ectopic testes include; base of penis, femoral
and perineal.

Testicular disorders-paediatric

Testicular disorders
Testicular disorders are some of the commonest conditions present in paediatric urological practice.

Cryptorchidism: The embryological descent of the testicle from within the abdominal cavity may be subject to a number of
variations. Distinctions need to be made clinically from a non descended testis and a testis that is retractile. Testis that lie outside
the normal path of embryological descent are termed ectopic testis. Undescended testis occurs in 1% of male infants. Where the
testis does not lie in an intra scrotal location, its location should be ascertained. Where both testes are absent the infant may be
intersex. MRI scanning may reveal intra-abdominal testes; however a GA is often needed to perform this investigation in this age
group. Testes that are undescended should be placed in the scrotum after 1 year of age as the testosterone surge that may facilitate
descent occurs at 6 months of age. Where the testes lie distally e.g. Superficial inguinal pouch an open orchidopexy is the
procedure of choice. With abdominal testes a laparoscopy should be performed. The risk of seminoma is increased in individuals
with a non descended testes and this risk is not reduced by orchidopexy.

Testicular torsion: Typically the patient has severe sudden onset of scrotal pain. The difficulty in paediatric practice is the lack of
clear history. On examination the testis is tender and enlarged. Management is by surgical exploration. Delay beyond 6 hours is
associated with low salvage rates. A torted hyatid produces pain that is far more localised and the testis itself should feel normal.
However, diagnostic doubt often exists and in such cases surgical exploration is warranted.

284.A splenectomy increases the risk of infection from all the following organisms except?
A. Pneumococcus
B. Klebsiella
C. Haemophilus influenzae
D. Staphylococcus aureus
E. Neisseria meningitides
Answer: D
Theme from 2010 Exam. Staphylococcus aureus infection following splenectomy is no more common than in non splenectomised
individuals. The other organisms are encapsulateed, which is why they are more likely to cause overwhelming post splenectomy
sepsis.

285.Which of the following is not an extraintestinal feature Crohns disease?


A. Iritis
B. Clubbing
C. Aphthous ulcers
D. Erythema multiforme
266
E. Pyoderma gangrenosum
Answer: D
Extraintestinal manifestation of inflammatory bowel disease: A PIE SAC: Aphthous ulcers. Pyoderma gangrenosum. Iritis.
Erythema nodosum. Sclerosing cholangitis. Arthritis. Clubbing

286.Which of the following is not considered a risk factor for the development of oesophageal malignancy?
A. Oesophageal metaplasia
B. Smoking
C. Excessive intake of alcoholic spirits
D. Achalasia
E. Blood group O
Answer: E
Blood group O is not a risk factor for oesophageal cancer. Achalasia is associated with the risk of developing squamous cell
carcinoma of the oesophagus.

Theme: Thyroid blood testing

A. Measurement of antibodies to TSH receptor


B. Thyroid peroxidase antibodies
C. Thyroglobulin antibodies
D. Serum calcitonin

Please select the blood test most commonly performed for the diagnosis or assessment of the thyroid disorder described. Each
answer may be used once, more than once or not at all.

287.A 32 year old lady is diagnosed with Medullary carcinoma of the thyroid and has undergone resection of the tumour.
Answer: Serum calcitonin
Measurement of basal or stimulated calcitonin concentrations is used to assess the completeness of surgical resection, and is of
use in detecting diseases recurrences during follow up.

288.A 20 year old lady has undergone a total thyroidectomy for a well differentiated papillary carcinoma. She attends clinic
and is well and the surgeon wishes to screen for disease recurrence.
Answer: Thyroglobulin antibodies
Antibodies to thyroglobulin, the major constituent of colloid and precursor of thyroid hormones may be elevated in those with
metastatic or recurrent thyroid cancer. Results may be erronoeous in those with other thyroid disorders.

289.A 33 year old lady presents with a recently diagnosed goitre and a diagnosis of Hashimotos thyroiditis is suspected.
Answer: Thyroid peroxidase antibodies
Antibodies to thyroid peroxidase are found in most patients with Graves disease or Hashimotos thyroiditis.

Blood testing in thyroid disease: Thyroid peroxidase (microsomal) antibodies:Found in autoimmune disease affecting the
thyroid (Hashimotos 100%) and Graves (70%). Antibodies to TSH receptor:Individuals with Graves disease (95%).
Thyroglobulin antibodies:Not useful for clinically distinguishing between different types of thyroid disease, may be used as part
of thyroid cancer follow up. Calcitonin:Released from the parafollicular cells. Usually found in patient with Medullary carcinoma
of the thyroid

290.Which one of the following complications is least associated with ventricular septal defects?
A. Right heart failure
B. Aortic regurgitation
C. Eisenmenger's complex
D. Infective endocarditis
E. Atrial fibrillation

Answer: E
Atrial fibrillation is associated more with atrial septal defects

Ventricular septal defect: Ventricular septal defects are the most common cause of congenital heart disease. They close
spontaneously in around 50% of cases. Non-congenital causes include post myocardial infarction. Features: classically a pan-
systolic murmur which is louder in smaller defects. Complications: aortic regurgitation*. Infective endocarditis. Eisenmenger's
complex. Right heart failure
*aortic regurgitation is due to a poorly supported right coronary cusp resulting in cusp prolapse
267
291.A 24 year old man from Sudan presents with a lymphadenopathy and weight loss. A diagnosis of tuberculosis is
suspected and a lymph node biopsy is performed. Staining with which of the agents below is most likely to facilitate
identification of the causative organism?
A. Gram stain
B. Ziehl-Neelsen stain
C. Von Kossa stain
D. Van Gieson stain
E. Masson Trichrome stain
Answer: B

Ziehl-Neelsen stain is typically used to identify mycobacteria. They are not stained in the Gram staining process. Van Gieson and
Masson trichrome are histological staining methods for identification of connective tissues. The Von Kossa technique is useful for
identifying tissue mineralisation.

292.Which of the following is not a pathological feature of breast cancer?


A. Resemblance to ductal epithelial cells
B. Angiogenesis
C. Nuclear pleomorphism
D. Metastatic calcification
E. Vascular invasion

Answer: D
Dystrophic calcification may be present in breast malignancy and is the basis for the breast screening programme. Metastatic
calcification is calcification which occurs in otherwise normal tissues, usually as a result of hypercalcaemia. Invasive ductal
carcinoma is the most common type of breast cancer, unless the tumour is very poorly differentiated there is usually some
resemblance to ductal epithelial cells.

Breast cancer pathology:


The histological features of breast cancer depend upon the underlying diagnosis. The invasive component is usually comprised of
ductal cells (unless it is an invasive lobular cancer). In situ lesions may co-exist (such as DCIS). Typical changes seen in
conjunction with invasive breast cancer include: 1. Nuclear pleomorphism; 2. Coarse chromatin; 3. Angiogenesis; 4. Invasion of
the basement membrane; 5. Dystrophic calcification (may be seen on mammography); 6. Abnormal mitoses; 7. Vascular invasion;
8. Lymph node metastasis
The primary tumour is graded on a scale of 1-3 where 1 is the most benign lesion and 3 the most poorly differentiated.
Immunohistochemistry for oestrogen receptor and herceptin status is routinely performed.
The grade, lymph node stage and size are combined to provide the Nottingham prognostic index.

Theme: Renal lesions

A. Renal cell carcinoma


B. Renal transitional cell carcinoma
C. Nephroblastoma
D. Neuroblastoma
E. Angiomyolipoma
F. Renal squamous cell carcinoma
G. Retroperitoneal fibrosis

For each scenario please select the most likely underlying diagnosis. Each option may be used once, more than once or not at all.

293.A 69 year old male presents with haematuria. He worked in the textile industry. He has a left flank mass. A CT IVU
shows a lesion of the left renal pelvis.
Answer: Renal transitional cell carcinoma
TCC is a rare form of renal cancer, accounting for approximately 7% of all renal tumours. Risk factors include exposure to
chemicals in the textile, plastic and rubber industry.

294.A 2 year old boy presents with a right renal mass. On examination he has an irregular mass arisong from the right
flank and is hypertensive. A CT scan shows a non calcified irregular lesion affecting the apex of the right kidney and
the right adrenal gland.
Answer: Nephroblastoma
Wilm's tumour of the kidney is the most common renal tumour in children. Both nephroblastoma and neuroblastoma may
occupy the adrenal and apex of the kidney. In the case of neuroblastoma the lesion will have arisen from the adrenal, in the
case of nephroblastoma the lesion will have arisen from the kidney. Hypertension is more commonly associated with

268
nephroblastoma. Neuroblastomas are usually calcified, whereas nephroblastomas are not and this may be of diagnostic
usefulness pre operatively.

295.A 35 year old male presents with haematuria. He is found to have bilateral masses in the flanks. He has a history of
epilepsy and learning disability.
Answer: Angiomyolipoma
This patient has tuberous sclerosus. This is associated with angiomyolipoma, which is present in 60-80% patients. It is a
benign lesion.

296.An 18 month old boy presents with recurrent urinary tract infections. As part of the diagnostic work-up he is noted to
have abnormal renal function. An ultrasound scan is performed and shows bilateral hydronephrosis. What is the most
likely underlying diagnosis?
A. Urethral valves
B. Meatal stenosis
C. Hydronephrosis
D. Pelvico-ureteric junction obstruction
E. Benign prostatic hyperplasia

Answer: A
Theme from April 2012 Exam. A posterior urethral valve is an obstructive, developmental uropathy that usually affects male
infants (incidence 1 in 8000). Diagnostic features include bladder wall hypertrophy, hydronephrosis and bladder diverticula.

Urethral valves: Posterior urethral valves are the commonest cause of infravesical outflow obstruction in males. They may be
diagnosed on ante natal ultrasonography. Because the bladder has to develop high emptying pressures in utero the child may
develop renal parenchymal damage. This translates to renal impairment noted in 70% of boys at presentation. Treatment is with
bladder catheterisation. Endoscopic valvotomy is the definitive treatment of choice with cystoscopic and renal follow up.

297.At which of the following anatomical sites does dormant tuberculosis most frequently reactivate?
A. Apex of the lung
B. Base of the lung
C. Brain
D. Terminal ileum
E. Lumbar spine
Answer: A
TB reactivation most commonly occurs at the lung apex. This site is better oxygenated than elsewhere allowing the
mycobacteria to multiply more rapidly and then spread both locally and distantly.

298.What is the commonest tumour type encountered in the colon?


A. Squamous cell carcinoma
B. Adenocarcinoma
C. Lymphoma
D. Anaplastic carcinoma
E. Sarcoma
Answer: B
Adenocarcinoma are the most common and typically arise as a result of the adenoma - carcinoma sequence.

299.Which of the following changes are most likely to be identified in the aortic wall of a 38 year old lady with a Marfans
syndrome and a dissecting aortic aneurysm?
A. Transmural aortitis
B. Cystic medial necrosis
C. Foamy macrophages
D. Dense dystrophic calcification
E. None of the above

Answer: B
Cystic medial necrosis ( or cystic medial degeneration) occurs when basophils and mucoid material lie in between the intimal

269
elastic fibres of the aorta. It is typically found in the aortic degeneration of Marfans syndrome, but may also be seen in aortic
degeneration in older adults.

300.A 58 year old man undergoes an upper GI endoscopy for the investigation of odynophagia. At endoscopy a reddish
area is seen to protrude up into the oesophagus from the gastrooesophageal junction. Which of the following
pathological processes is most likely to explain this process?
A. Metaplasia
B. Anaplasia
C. Dysplasia
D. Hypoplasia
E. Hyperplasia

Answer: A
This is most likely to represent Barretts oesphagus and is thus metaplasia. Dysplasia is less likely in this setting although biopsies
are mandatory.

301.A male infant is born by emergency cesarean section at 39 weeks gestation for foetal distress. Soon after the birth the
baby becomes progressively hypoxic and on examination is found to have a scaphoid abdomen. What is the most likely
underlying diagnosis?
A. Intestinal malrotation
B. Hiatus hernia
C. Foramen of Bochdalek hernia
D. Foramen of Morgagni hernia
E. Tracheooesphageal fistula
Answer: C

The finding of a scaphoid abdomen and respiratory distress suggests extensive intra thoracic herniation of the abdominal contents.
This is seen most frequently with Bochdalek hernias. Morgagni hernias seldom present in such a dramatic fashion. The other
options do not typically present with the symptoms and signs described.

Embryology: The diaphragm is formed between the 5th and 7th weeks of gestation through the progressive fusion of the septum
transversum, pleuroperitoneal folds and via lateral muscular ingrowth. The pleuroperitoneal membranes from which the
diaphragm originates originate at somites located in cervical segments 3 to 5, which accounts for the long path taken by the
phrenic nerve. The components contribute to the following diaphragmatic segments: Septum transversum - Central tendon.
Pleuroperitoneal membranes - Parietal membranes surrounding viscera. Cervical somites C5 to C7 - Muscular component of the
diaphragm

Diaphragmatic hernia: Morgagni: Anteriorly located. Minimal compromise on lung development. Minimal signs on antenatal
ultrasound. Usually present later. Usually good prognosis. Bochdalek hernia: Posteriorly located. Larger defect. Often diagnosed
antenatally. Associated with pulmonary hypoplasia. Poor prognosis.
The posterior hernias of Bochdalek are the most common type and if not diagnosed antenatally will typically present soon after
birth with respiratory distress. The classical finding is that of a scaphoid abdomen on clinical examination because of herniation of
the abdominal contents into the chest. Bochdalek hernias are associated with a number of chromosomal abnormalities such as
Trisomy 21 and 18. Infants have considerable respiratory distress due to hypoplasia of the developing lung. Historically this was
considered to be due to direct compression of the lung by herniated viscera. This view over simplifies the situation and the
pulmonary hypoplasia occurs concomitantly with the hernial development, rather than as a direct result of it. The pulmonary
hypoplasia is associated with pulmonary hypertension and abnormalities of pulmonary vasculature. The pulmonary hypertension
renders infants at risk of right to left shunting (resulting in progressive and worsening hypoxia). Diagnostic work up of these
infants includes chest x-rays/ abdominal ultrasound scans and cardiac echo. Surgery forms the mainstay of treatment and both
thoracic and abdominal approaches may be utilised. Following reduction of the hernial contents a careful search needs to be made
for a hernial sac as failure to recognise and correct this will result in a high recurrence rate. Smaller defects may be primarily
closed, larger defects may require a patch to close the defect. Malrotation of the viscera is a recognised association and may
require surgical correct at the same procedure (favoring an abdominal approach). The mortality rate is 50-75% and is related to the
degree of lung compromise and age at presentation (considerably better in infants >24 hours old).

302.A 72 year old lady falls and lands on her left hip. She attends the emergency department and is given some paracetamol
by the junior doctor and discharged. Several months later she presents with ongoing pain and discomfort of the hip.
Avascular necrosis of the femoral head is suspected. Which of the following features is least likely to be present?
A. Non union of the fracture
B. Angiogenesis at the fracture site
C. Increased numbers of fibroblasts at the fracture site
270
D. Osteochondritis dissecans
E. Apoptosis of osteoblasts

Answer: E
Apoptosis is not a feature of necrotic cell death. By this stage there would usually be attempted repair so angiogenesis and
proliferation of fibroblasts would be expected. These cells may differentiate further to become osteoblasts which in turn will lay
down new matrix.

Avascular necrosis: Cellular death of bone components due to interruption of the blood supply, causing bone destruction. Main
joints affected are hip, scaphoid, lunate and the talus. It is not the same as non union. The fracture has usually united. Radiological
evidence is slow to appear. Vascular ingrowth into the affected bone may occur. However, many joints will develop secondary
osteoarthritis.
Causes: P ancreatitis. L upus. A lcohol. S teroids. T rauma. I diopathic, infection. C aisson disease, collagen vascular disease. R
adiation, rheumatoid arthritis. A myloid. G aucher disease. S ickle cell disease
Presentation: Usually pain. Often despite apparent fracture union. Investigation: MRI scanning will show changes earlier than
plain films. Treatment: In fractures at high risk sites anticipation is key. Early prompt and accurate reduction is essential.
Non weight bearing may help to facilitate vascular regeneration.
Joint replacement may be necessary, or even the preferred option (e.g. Hip in the elderly).

303.Which one of the following is least associated with the development of colorectal cancer in patients with ulcerative
colitis?
A. Unremitting disease
B. Disease duration > 10 years
C. Onset before 15 years old
D. Poor compliance to treatment
E. Disease confined to the rectum

Answer: E
Ulcerative colitis and colorectal cancer: Overview: risk of colorectal cancer is 10-20 times that of general population. The
increased risk is mainly related to chronic inflammation. Worse prognosis than patients without ulcerative colitis (partly due to
delayed diagnosis). Lesions may be multifocal
Factors increasing risk of cancer: Disease duration > 10 years. Patients with pancolitis. Onset before 15 years old. Unremitting
disease. Poor compliance to treatment

304.Which of the following statements about wound healing is false?


A. During the course of the first 2 weeks the fibrin plug is replaced by a collagen rich mesh.
B. Scar contraction is a late feature and is mediated by myofibroblasts.
C. May be impaired when an infected foreign body is present.
D. Is more cosmetically acceptable when incisions are made parallel to Langer's lines.
E. Has features of chronic inflammation from the beginning.

Answer: E
Although wounds often contain cell types (macrophages) and processes (angiogenesis) these typically take time to occur. They are
not present in the immature wound.

Theme: Causes of chest pain

A. Pulmonary embolism
B. Anterior myocardial infarction
C. Inferior myocardial infarction
D. Proximal aortic dissection
E. Distal aortic dissection
F. Boerhaave Syndrome
G. Mallory weiss tear
H. Perforated gastric ulcer

Please select the most likely cause of chest pain for the scenario given. Each option may be used once, more than once or not at
all.

305.A 52 year old male presents with tearing central chest pain. On examination he has an aortic regurgitation murmur.
An ECG shows ST elevation in leads II, III and aVF.
271
Answer: Proximal aortic dissection
An inferior myocardial infarction and AR murmur should raise suspicions of an ascending aorta dissection rather than an
inferior myocardial infarction alone. Also the history is more suggestive of a dissection. Other features may include pericardial
effusion, carotid dissection and absent subclavian pulse.

306.A 52 year old male presents with central chest pain and vomiting. He has drunk a bottle of vodka. On examination
there is some mild crepitus in the epigastric region.
Answer: Boerhaave Syndrome
The Mackler triad for Boerhaave syndrome: vomiting, thoracic pain, subcutaneous emphysema. It commonly presents in
middle aged men with a background of alcohol abuse.

307.A 52 year old male presents with central chest pain. On examination he has an mitral regurgitation murmur. An ECG
shows ST elevation in leads V1 to V6. There is no ST elevation in leads II, III and aVF.
Answer: Anterior myocardial infarction
The most likely diagnosis is an anterior MI. As there are no ST changes in the inferior leads, aortic dissection is less likely.

Chest pain
Aortic dissection: This occurs when there is a flap or filling defect within the aortic intima. Blood tracks into the medial layer and
splits the tissues with the subsequent creation of a false lumen. It most commonly occurs in the ascending aorta or just distal to the
left subclavian artery (less common). It is most common in Afro-carribean males aged 50-70 years. Patients usually present with a
tearing intrascapular pain, which may be similar to the pain of a myocardial infarct. The dissection may spread either proximally
or distally with subsequent disruption to the arterial branches that are encountered. In the Stanford classification system the
disease is classified into lesions with a proximal origin (Type A) and those that commence distal to the left subclavian (Type B).
Diagnosis may be suggested by a chest x-ray showing a widened mediastinum. Confirmation of the diagnosis is usually made by
use of CT angiography. Proximal (Type A) lesions are usually treated surgically, type B lesions are usually managed non
operatively.

Pulmonary embolism: Typically sudden onset of chest pain, haemoptysis, hypoxia and small pleural effusions may be present.
Most patients will have an underlying deep vein thrombosis. Diagnosis may be suggested by various ECG findings including S
waves in lead I, Q waves in lead III and inverted T waves in lead III. Confirmation of the diagnosis is usually made through use of
CT pulmonary angiography. Treatment is with anticoagulation, in those patients who develop a cardiac arrest or severe
compromise from their PE, consideration may be given to thrombolysis.

Myocardial infarction: Traditionally described as sudden onset of central, crushing chest pain. It may radiate into the neck and
down the left arm. Signs of autonomic dysfunction may be present. The presenting features may be atypical in the elderly and
those with diabetes. Diangosis is made through identification of new and usually dynamic ECG changes (and cardiac enzyme
changes). Inferior and anterior infarcts may be distinguished by the presence of specific ECG changes (usually II, III and aVF for
inferior, leads V1-V5 for anterior). Treatment is with oral antiplatelet agents, primary coronary angioplasty and/ or thrombolysis.

Perforated peptic ulcer: Patients usually develop sudden onset of epigastric abdominal pain, it may be soon followed by
generalised abdominal pain. There may be features of antecendant abdominal discomfort, the pain of gastric ulcer is typically
worse immediately after eating. Diagnosis may be made by erect chest x-ray which may show a small amount of free intra-
abdominal air (very large amounts of air are more typically associated with colonic perforation). Treatment is usually with a
laparotomy, small defects may be excised and overlaid with an omental patch, larger defects are best managed with a partial
gastrectomy.

Boerhaaves syndrome: Spontaneous rupture of the oesophagus that occurs as a result of repeated episodes of vomiting. The
rupture is usually distally sited and on the left side. Patients usually give a history of sudden onset of severe chest pain that may
complicate severe vomiting. Severe sepsis occurs secondary to mediastinitis. Diagnosis is CT contrast swallow. Treatment is with
thoracotomy and lavage, if less than 12 hours after onset then primary repair is usually feasible, surgery delayed beyond 12 hours
is best managed by insertion of a T tube to create a controlled fistula between oesophagus and skin. Delays beyond 24 hours are
associated with a very high mortality rate.

308.A 78 year old lady presents with a tender swelling in her right groin. On examination there is a tender swelling that lies
lateral to the pubic tubercle. It has a cough impulse. What is the most likely underlying diagnosis?
A. Thrombophlebitis of the great saphenous vein
B. Femoral hernia
C. Thrombophlebitis of saphena varix
D. Inguinal hernia
E. Obturator hernia

272
Answer: B Theme from April 2012 Exam. Whilst a thrombophlebitis of a saphena varix may cause a tender swelling at
this site, it would not usually be associated with a cough impulse.

Femoral canal: The femoral canal lies at the medial aspect of the femoral sheath. The femoral sheath is a fascial tunnel
containing both the femoral artery laterally and femoral vein medially. The canal lies medial to the vein.
Borders of the femoral canal: Laterally:Femoral vein. Medially:Lacunar ligament. Anteriorly: Inguinal ligament.
Posteriorly:Pectineal ligament
Contents: Lymphatic vessels. Cloquet's lymph node
Physiological significance: Allows the femoral vein to expand to allow for increased venous return to the lower limbs.
Pathological significance: As a potential space, it is the site of femoral hernias. The relatively tight neck places these at high risk
of strangulation.

309.A 3 month old boy is suspected of having hypospadias. At which of the following locations is the urethral opening most
frequently located in boys suffering from the condition?
A. On the distal ventral surface of the penis
B. On the proximal ventral surface of the penis
C. On the distal dorsal surface of the penis
D. On the proximal dorsal surface of the penis
E. At the base of the scrotum
Answer:
A
The defect is located ventrally and most often distally. Proximally located urethral openings are well recognised. Circumcision
may compromise reconstruction.

310.A 52 year old male attends for a preoperative assessment for an inguinal hernia repair. You notice that the chest x-ray
shows a loculated left pleural effusion. On further questioning the patient reports that he worked as a builder 30 years
ago. What is the most likely cause for the effusion?
A. Asbestosis
B. Pneumonia
C. Mesothelioma
D. Silicosis
E. Left ventricular failure
Answer: C

This patient has a risk of asbestos exposure through his occupation as a builder. As there a is latent period of 30 years and a
complicated effusion, the most likely cause is mesothelioma.

Mesothelioma: Features: Dyspnoea, weight loss, chest wall pain. Clubbing. 30% present as painless pleural effusion. Only 20%
have pre-existing asbestosis. History of asbestos exposure in 85-90%, latent period of 30-40 years
Basics: Malignancy of mesothelial cells of pleura. Metastases to contralateral lung and peritoneum. Right lung affected more
often than leftManagement: Investigation: pleural biopsy, CT Scanning, (PET Scanning if surgery considered). Symptomatic.
Industrial compensation. Chemotherapy, Surgery if operable. Prognosis poor, median survival 12 months

311.A 64-year-old woman who is reviewed due to multiple non-healing leg ulcers. She reports feeling generally unwell for
many months. Examination findings include a blood pressure of 138/72 mmHg, pulse 90 bpm, pale conjunctivae and
poor dentition associated with bleeding gums. What is the most likely underlying diagnosis?
A. Thyrotoxicosis
B. Vitamin B12 deficiency
C. Vitamin C deficiency
D. Diabetes mellitus
E. Sarcoidosis
Answer: C

Bleeding gums and poor healing are suggestive of vitamin C deficiency.

Vitamin C deficiency: Vitamin C deficiency (scurvy) leads to defective synthesis of collagen resulting in capillary fragility
(bleeding tendency) and poor wound healing. Features: Gingivitis. Loose teeth. Poor wound healing. Bleeding from gums,
haematuria, epistaxis. General malaise

312.Which of the following is not a typical feature of neuropraxia?


A. Transient delay in neuronal transmission
B. Axonal degeneration distal to the site of injury
273
C. Absence of neuroma formation
D. Preservation of autonomic function
E. Absence of axonal degeneration proximal to the site of injury

Answer: B

Full recovery may occur 6-8 weeks after nerve injury in neuropraxia. Wallerian degeneration does not usually occur in simple
neuropraxia. Autonomic function is usually preserved.

313.A 44 year old lady presents with a pathological fracture of the left femur. She has previously undergone a renal
transplant for end stage renal failure. Her blood test results are as follows: Serum Ca2+ 2.80, PTH 88pg/ml, Phosphate
0.30. A surgeon decides to perform a parathyroidectomy on the basis of these results. When the glands are assessed
histologically, which of the appearances is most likely to be identified?
A. Metaplasia the gland
B. Hypertrophy of the gland
C. Hyperplasia of the gland
D. Parathyroid carcinoma
E. Necrosis of the parathyroid gland
Answer: C

This is likely to be a case of tertiary hyperparathyroidism (high Calcium, high PTH, low phosphate). Therefore the glands will be
hyperplastic. Hypertrophy is not correct as this implies an increase in size without an increase in cellularity. This mistake has cost
many candidates marks in the MRCS exams over the years!

314.A 56 year old man presents with lethargy, haematuria and haemoptysis. On examination he is hypertensive and has a left
loin mass. A CT scan shows a lesion affecting the upper pole of the right kidney, it has a small cystic centre. Which of the
options below is the most likely diagnosis?
A. Squamous cell carcinoma of the kidney
B. Nephroblastoma
C. Renal adenocarcinoma
D. Transitional cell carcinoma of the kidney
E. Polycystic kidney disease
Answer:
C

Renal adenocarcinoma are the most common renal tumours. These will typically affect the renal parenchyma. Transitional cell
carcinoma will usually affect urothelial surfaces. Nephroblastoma would be very rare in this age group. Renal adenocarcinoma
may produce cannon ball metastasis in the lung which cause haemoptysis, this is not a feature of PKD.

315.A 34-year-old man is taken immediately to theatre with aortic dissection. You note he is tall with pectus excavatum and
arachnodactyly. His condition is primarily due to a defect in which one of the following proteins?
A. Polycystin-1
B. Fibrillin
C. Type IV collagen
D. Type I collagen
E. Elastin

Answer: B
Although fibrillin is the primary protein affected (due to a defect in the fibrillin-1 gene) it should be noted that fibrillin is used as a
substrate of elastin.Marfan's syndrome: is an autosomal dominant connective tissue disorder. It is caused by a defect in the
fibrillin-1 gene on chromosome 15 and affects around 1 in 3,000 people.
Features: Tall stature with arm span to height ratio > 1.05. High-arched palate. Arachnodactyly. Pectus excavatum. Pes planus.
Scoliosis of > 20 degrees. Heart: dilation of the aortic sinuses (seen in 90%) which may lead to aortic aneurysm, aortic dissection,
aortic regurgitation, mitral valve prolapse (75%). Lungs: repeated pneumothoraces. Eyes: upwards lens dislocation
(superotemporal ectopia lentis), blue sclera, myopia. Dural ectasia (ballooning of the dural sac at the lumbosacral level)

The life expectancy of patients used to be around 40-50 years. With the advent of regular echocardiography monitoring and beta-
blocker/ACE-inhibitor therapy this has improved significantly over recent years. Aortic dissection and other cardiovascular
problems remain the leading cause of death however.

316.Which of the following are not typical of Lynch syndrome?


A. It is inherited in an autosomal recessive manner
B. Affected patients are more likely to develop right colon mucinous tumours than the general population
C. Affected individuals have an 80% lifetime risk of colon cancer
274
D. Endometrial cancer is seen in 80%.
E. Gastric cancers are more common

Answer: A
Lynch syndrome is inherited in an autosomal dominant fashion. It is characterised by microsatellite instability in the DNA
mismatch repair genes. Colonic tumours in patients with Lynch syndrome are more likely to be right sided tumours and to be
poorly differentiated.

317.An enthusiastic medical student approaches you with a list of questions about blood transfusion reactions. Which of
her following points is incorrect?
A. Graft versus host disease involves neutrophil proliferation
B. Thrombocytopaenia may occur in women with a prior pregnancy
C. IgA antibodies may cause blood pressure compromise during transfusion
D. Hypocalcaemia can occur
E. Iron overload can be avoided by chelation therapy

Answre: A
Mnemonic for transfusion reactions:

Got a bad unit: G raft vs. Host disease. O verload. T hrombocytopaenia. A lloimmunization. B lood pressure unstable. A cute
haemolytic reaction. D elayed haemolytic reaction. U rticaria. N eutrophilia. I nfection. T ransfusion associated lung injury
GVHD results from lymphocytic proliferation. The patient's own lymphocytes are similar to the donor's lymphocytes, therefore
don't perceive them as being foreign. The donor lymphocytes, however, sees the recipient lymphocytes as being foreign.
Therefore they proliferate causing severe complications. Thrombocytopaenia occurs a few days after transfusion and may resolve
spontaneously.
Patients with IGA antibodies need IgA deficient blood transfusions.

Blood transfusion reactions


Immune mediated: Pyrexia. Alloimmunization. Thrombocytopaenia. Transfusion associated lung injury. Graft vs Host disease.
Urticaria. Acute or delayed haemolysis. ABO incompatibility. Rhesus incompatibility.
Non immune mediated: Hypocalcaemia. CCF. Infections. Hyperkalaemia
Notes: GVHD: lymphocyte proliferation causing organ failure. Transfusion associated lung injury: neutrophil mediated allergic
pulmonary oedema. ABO and Rhesus incompatibility: causes acute haemolytic transfusion reaction leading to agglutination and
haemolysis

318.An 82 year old lady presents with a carcinoma of the caecum. Approximately what proportion of patients presenting
with this diagnosis will have synchronous lesions?
A. <1%
B. 60%
C. 50%
D. 20%
E. 5%
Answer: E
Synchronous colonic tumours are seen in 5% cases and all patients having a flexible sigmoidoscopy should have completion
colonoscopy if tumours or polyps are found
Synchronous lesions may occur in up to 5% of patients with colorectal cancer. A full and complete lumenal study with either
colonoscopy, CT cologram or barium enema is mandatory in all patients being considered for surgery.

319.A 22 year old man undergoes a splenectomy for an iatrogenic splenic injury. On the second post operative day a full
blood count is performed. Which of the following components of the full blood count is the first to be affected ?
A. Erythrocyte count
B. Reticulocyte count
C. Eosinophil count
D. Monocyte count
E. Lymphocyte count
Answer: B

Theme from January 2012 Exam.The granulocyte and platelet count are the first to be affected following splenectomy. Then
reticulocytes increase. Although a lymphocytosis and monocytosis are reported, these take several weeks to develop.

320.A 28 year old lady presents with benign cyclical mastalgia. Which of the following is not a recognised treatment for the
condition?
A. Evening primrose oil
B. Bromocriptine

275
C. Methotrexate
D. Danazol
E. Tamoxifen
Answer: C

Surgical excision of tender breast tissue is inappropriate. Methotrexate is used for the treatment of breast cancer. Whilst the use of
tamoxifen is of benefit other agents such as flaxseed oil or evening primrose oil should be tried first. Danazol is effective, but
many women dislike the side effects.
Benign cyclical mastalgia is a common cause of breast pain in younger females. It varies in intensity according to the phase of
the menstrual cycle. It is not associated with point tenderness of the chest wall (more likely to be Tietze's syndrome).
The underlying cause is difficult to pinpoint, examination should focus on identifying focal lesions (such as cysts) that may be
treated to provide symptomatic benefit. Women should be advised to wear a supportive bra. Conservative treatments include flax
seed oil and evening primrose oil. There is slightly more evidence in favor of flax seed oil, though neither has performed much
better than placebo in RCT's. Hormonal agents such as bromocriptine and danazol may be more effective. However, many
women discontinue these therapies due to adverse effects.

321.A 39 year old lady has undergone surgery for breast cancer. As part of the histopathology report the pathologist
provides the surgeon with a Nottingham Prognostic Index score of 6.4. He also states that the tumour size is 2cm.
Which of the following inferences can be made in relation to this statement?
A. The tumour is likely to be grade 1
B. Vascular invasion is present
C. Lymph node metastasis are definitely present
D. The tumour is oestrogen receptor positive
E. None of the above

Answer: C
A score of this value is unlikely to be reached with a grade 1 tumour and a size of 2cm. Therefore lymph node metastasis are
definitely present. In addition since the maximal score for lymph node metastasis is 3 the tumour is likely be of a higher grade (see
below). The Nottingham Prognostic Index provides no information about oestrogen receptor status or the presence or absence of
vascular invasion.

Nottingham prognostic index: The Nottingham Prognostic Index can be used to give an indication of survival. In this system the
tumour size is weighted less heavily than other major prognostic parameters. Calculation of NPI: Tumour Size x 0.2 + Lymph
node score(From table below)+Grade score(From table below).
Lymph nodes involved Grade
Score
1 0 1
2 1-3 2
3 >3 3

Prognosis: Score to Percentage 5 year survival: 2.0 to 2.4: 93%. 2.5 to 3.4:85%. 3.5 to 5.4:70%. >5.4:50%. This data was
originally published in 1992. It should be emphasised that other factors such as vascular invasion and receptor status also impact
on survival and are not included in this data and account for varying prognoses often cited in the literature.

322.In patients with multiple endocrine neoplasia type IIb which of the following clinical appearances is the patient most
likely to display?
A. Acromegalic facies
B. Turners type features
C. Profound kyphoscoliosis
D. Multiple bony exostoses
E. Marfanoid features

Answer: E
Patients with MEN IIb may display Marfanoid features. It is unclear at the present time whether they have discrete changes in the
microfibrils of elastic fibres that are present in Marfans.

323.32 year old man undergoes an appendicectomy. A large carcinoid tumour is identified and a completion right
hemicolectomy is performed. He is well for several months and then develops symptoms of palpitations and facial
flushing. Which of the following diagnostic markers should be requested?
A. Alpha feto protein
B. 5-Hydroxyindoleacetic acid
C. Urinary catecholamines
D. Urinary VMA measurements
276
E. None of the above
Answer: B
5 HIAA is the most commonly used diagnostic marker for carcinoid syndrome.

324.Which of the following breast tumours is most commonly associated with a risk of metastasis to the contralateral
breast?
A. Invasive ductal carcinoma
B. Invasive lobular carcinoma
C. Phyllodes tumour
D. Pagets disease of the breast
E. Atypical ductal hyperplasia

Answer: B

Risk of metastasis to the contralateral breast is a classical feature of invasive lobular carcinoma.

325.With respect to oncogenes which statement is false?


A. Mutations in oncogenes lead to cell survival
B. Cells with oncogene mutations are resistant to cell necrosis
C. Oncogene mutations must typically involve two allelic mutations for biological effects to become manifest
D. May prevent cellular apoptosis
E. Include MYC mutations in the development of Burkitts Lymphoma

Answer: C
Tumour suppressor gene mutations tend to be recessive and thus twin allelic mutations are required as exemplified in the Knudson
two hit hypothesis in the development of retinoblastoma. Necrosis will occur at the centre of tumours that outgrow their blood
supply and induction of angiogenesis is a key feature of tumour progression.

326.Which is the characteristic finding on a blood film post splenectomy?


A. Stipple cell
B. Tear drop cell
C. Reticulocytes
D. Howell-Jolly bodies
E. Schistocyte
Answer: D
Blood film in hyposplenism: Howell-Jolly bodies. Pappenheimer bodies. Poikilocytes (Target cells). Erythrocyte containing
siderotic granules. Heinz bodies
Indications: Trauma: 1/4 are iatrogenic. Spontaneous rupture: EBV. Hypersplenism: hereditary spherocytosis or elliptocytosis
etc. Malignancy: lymphoma or leukaemia. Splenic cysts, hydatid cysts, splenic abscesses. Post splenectomy changes: Platelets
will rise first (therefore in ITP should be given after splenic artery clamped). Blood film will change over following weeks,
Howell Jolly bodies will appear. Other blood film changes include target cells and Pappenheimer bodies. Increased risk of post
splenectomy sepsis, therefore prophylactic antibiotics and pneumococcal vaccine should be given. Post splenectomy sepsis:
Typically occurs with encapsulated organisms. Opsonisation occurs but then not recognized

Microbiology
1. A 48 year old lady is admitted with crampy abdominal pain and diarrhoea. She has been unwell for the past 12 hours. In
the history she complains that her milk bottles have been pecked repeatedly by birds, she otherwise has had no dietary
changes. Which of the following is the most likely causative organism?
A. Staphylococcus aureus
B. Campylobacter jejuni
C. Clostridium difficile
D. Norovirus
E. Clostridium botulinum
Answer: B
Birds are a recognised reservoir of campylobacter.

Bacterial Gastroenteritis: Causative organisms

Campylobacter jejuni: Most common cause of acute infective diarrhea. Spiral, gram negative rods. Usually infects terminal ileum
but spreads to involve colon and rectum. Local lymphadenopathy is common. May mimic appendicitis as it has marked right iliac
fossa pain. Reactive arthritis is seen in 1-2% of cases

277
Shigella spp.: Members of the enterobacteriaceae. Gram negative bacilli. Clinically causes dysentery. Shigella soneii is the
commonest infective organism (mild illness). Usually self limiting, ciprofloxacin may be required if individual is in a high risk
group

Salmonella spp: Facultatively anaerobic, gram negative, enterobacteriaceae. Infective dose varies according to subtype.
Salmonellosis: usually transmitted by infected meat (especially poultry) and eggs

E. coli: Enteropathogenic. Enteroinvasive: dysentery, large bowel necrosis/ulcers. Enterotoxigenic: small intestine, travelers
diarrhea. Enterohaemorrhagic: 0157, cause a haemorrhagic colitis, haemolytic uraemic syndrome and thrombotic
thrombocytopaenic purpura

Yersinia enterocolitica: Gram negative, coccobacilli. Enterocolitis, acute mesenteric lymphadenitis or terminal ileitis. Differential
diagnosis acute appendicitis. May progress to septicaemia in susceptible individuals. Usually sensitive to quinolone or
tetracyclines

Vibrio cholera: Short, gram negative rods. Transmitted by contaminated water, seafood. Symptoms include sudden onset of
effortless vomiting and profuse watery diarrhea. Correction of fluid and electrolyte losses are the mainstay of treatment. Most
cases will resolve, antibiotics are not generally indicated

2. A 50-year-old female with a history of rheumatoid presents with a suspected septic knee joint. A diagnostic aspiration is
performed and sent to microbiology. Which of the organisms below is most likely to be responsible?
A. Staphylococcus aureus
B. Staphylococcus epidermidis
C. Escherichia coli
D. Neisseria gonorrhoeae
E. Streptococcus pneumoniae
Answer: A
Septic arthritis - most common organism: Staphylococcus aureus

Septic arthritis: Overview: Most common organism overall is Staphylococcus aureus. In young adults who are sexually active
Neisseria gonorrhoeae should also be considered

Management: Synovial fluid should be obtained before starting treatment. Intravenous antibiotics which cover Gram-positive
cocci are indicated. The BNF currently recommends flucloxacillin or clindamycin if penicillin allergic. Antibiotic treatment is
normally be given for several weeks (BNF states 6-12 weeks). Needle aspiration should be used to decompress the joint.
Arthroscopic lavage may be required

3. A 22 year old man presents with crampy abdominal pain diarrhoea and bloating. He has just returned from a holiday in
Egypt. He had been swimming in the local pool a few days ago. He reports that he is opening his bowels 5 times a day.
The stool floats in the toilet water, but there is no blood. What is the most likely cause?
A. Cryptosporidium
B. Salmonella sp
C. E.coli sp
D. Chronic pancreatitis
E. Giardia lamblia
Answer: E
Giardia causes fat malabsorption, therefore greasy stool can occur. It is resistant to chlorination, hence risk of transfer in
swimming pools.
Diarrhoea: World Health Organisation definitions: Diarrhoea: > 3 loose or watery stool per day. Acute diarrhoea < 14 days.
Chronic diarrhoea > 14 days

Acute Diarrhoea
Gastroenteritis May be accompanied by abdominal pain or nausea/vomiting
Diverticulitis Classically causes left lower quadrant pain, diarrhoea and fever
Antibiotic therapy More common with broad spectrum antibiotics
Clostridium difficile is also seen with antibiotic use
278
Constipation causing overflow A history of alternating diarrhoea and constipation may be given
May lead to faecal incontinence in the elderly

Chronic Diarrhoea
Irritable bowel Extremely common. The most consistent features are abdominal pain, bloating and change in bowel habit.
syndrome Patients may be divided into those with diarrhoea predominant IBS and those with constipation predominant
IBS.
Features such as lethargy, nausea, backache and bladder symptoms may also be present
Ulcerative colitis Bloody diarrhoea may be seen. Crampy abdominal pain and weight loss are also common. Faecal urgency
and tenesmus may occur
Crohn's disease Crampy abdominal pains and diarrhoea. Bloody diarrhoea less common than in ulcerative colitis. Other
features include malabsorption, mouth ulcers perianal disease and intestinal obstruction
Colorectal Symptoms depend on the site of the lesion but include diarrhoea, rectal bleeding, anaemia and constitutional
cancer symptoms e.g. Weight loss and anorexia
Coeliac disease  In children may present with failure to thrive, diarrhoea and abdominal distension
 In adults lethargy, anaemia, diarrhoea and weight loss are seen. Other autoimmune conditions may
coexist

Other conditions associated with diarrhoea include: Thyrotoxicosis. Laxative abuse. Appendicitis with pelvic abscess or pelvic
appendix. Radiation enteritis

Diagnosis: Stool culture. Abdominal and digital rectal examination. Consider colonoscopy (radiological studies unhelpful).
Thyroid function tests, serum calcium, anti endomysial antibodies, glucose

4. A 54-year-old female is admitted one week following a cholecystectomy with profuse diarrhoea. Apart from a minor
intra-operative bile spillage incurred during removal of the gallbladder, the procedure was uncomplicated. What is the
most likely diagnosis?
A. Campylobacter infection
B. E. coli infection
C. Clostridium difficile infection
D. Salmonella infection
E. Pelvic abscess
Answer: C
Antibiotics are not routinely administered during an uncomplicated cholecystectomy. Indications for administration of broad
spectrum antibiotics include intraoperative bile spillage. Delayed pelvic abscesses following bile spills are extremely rare since
most surgeons will manage these intra-operatively.

Clostridium difficile: Clostridium difficile is a Gram positive rod often encountered in hospital practice. It produces an exotoxin
which causes intestinal damage leading to a syndrome called pseudomembranous colitis. Clostridium difficile develops when the
normal gut flora are suppressed by broad-spectrum antibiotics. Clindamycin is historically associated with causing Clostridium
difficile but the aetiology has evolved significantly over the past 10 years. Second and third generation cephalosporins are now the
leading cause of Clostridium difficile.

Features: Diarrhoea. Abdominal pain. A raised white blood cell count is characteristic

 If severe, toxic megacolon may develop

Diagnosis is made by detecting Clostridium difficile toxin (CDT) in the stool

Management: First-line therapy is oral metronidazole for 10-14 days. If severe, or not responding to metronidazole, then oral
vancomycin may be used. For life-threatening infections a combination of oral vancomycin and intravenous metronidazole should
be used

5. Which of the following is not a feature of Campylobacter jejuni infection?


A. Infection may present in a similar manner to acute appendicitis
B. Pyrexia is unusual
C. They are gram negative organisms
D. Infection accounts for 26% case of Guillain-Barre syndrome
E. It is the commonest cause of infective diarrhoea
Answer: B
279
A prodromal period of fever and generalised malaise precedes abdominal pain (which may mimic appendicitis) and diarrhoea.

6. A 53-year-old woman is diagnosed with cellulitis surrounding her stoma site. A swab is taken and oral flucloxacillin is
started. The following result is obtained: Skin swab:Group A streptococcus. How should the antibiotic therapy be
changed?
A. No change
B. Add topical fusidic acid
C. Add clindamycin
D. Add penicillin
E. Add erythromycin
Answer: D
7. Penicillin is the antibiotic of choice for group A streptococcal infections. The BNF suggests stopping flucloxacillin if
streptococcal infection is confirmed in patients with cellulitis, due to the high sensitivity. This should be balanced however with
the variable absorption of phenoxymethylpenicillin.

Streptococci: Streptococci may be divided into alpha and beta haemolytic types

Alpha haemolytic streptococci: The most important alpha haemolytic streptococcus is Streptococcus pneumoniae
(pneumococcus). Pneumococcus is a common cause of pneumonia, meningitis and otitis media. Another clinical example is
Streptococcus viridans

Beta haemolytic streptococci: These can be subdivided into group A and B

Group A: most important organism is Streptococcus pyogenes. responsible for erysipelas, impetigo, cellulitis, type 2 necrotizing
fasciitis and pharyngitis/tonsillitis. immunological reactions can cause rheumatic fever or post-streptococcal glomerulonephritis.
erythrogenic toxins cause scarlet fever

Group B: Streptococcus agalactiae may lead to neonatal meningitis and septicaemia


Theme: Infectious disease

A. Clostridium difficile
B. Clostridium perfringens
C. Clostridium tetani
D. Streptococcus pyogenes
E. Steptococcus Bovis
F. Staphylococcus aureus
G. Staphylococcus epidermidis
H. Bacteroides fragilis
I. None of the above

Please select the most likely infective organism for the scenario given. Each option may be used once, more than once or not at
all.

8. A 23 year old man is readmitted following a difficult appendicectomy. His wound is erythematous and on incision foul
smelling pus is drained.

The correct answer is Bacteroides fragilis. Bacteroides is commonly present in severe peritoneal infections and as it is
facultatively anaerobic may be present in pus. It smells foul!
9. A 62 year old lady is unwell following a difficult acute cholecystectomy for acute cholecystitis. Her gallbladder spilled
stones intraoperatively and she has been on ciprofloxacin intravenously for this for the past 4 days. She now has colicky
abdominal pain and profuse, foul smelling diarrhoea.

Clostridium difficile. C. difficile may complicate administration of broad spectrum antibiotics.


10. A 21 year old man is admitted with crampy abdominal pain and diarrhoea. He attended a large wedding earlier in the
day. Several other guests are also affected with the same illness.

The correct answer is Clostridium perfringens. C. Perfringens is a common cause of food borne illness and its ability to form
spores may make it relatively resistant to cooking. The timing of onset would favor C. Perfringens which typically evolves over
several hours, rather than staphylococcus aureus poisening which may occur sooner.

Surgical Microbiology: An extensive topic so an overview is given here. Organisms causing common surgical infections are
reasonable topics in the examination. However, microbiology is less rigorously tested than anatomy, for example.
280
Common organisms: Staphylococcus aureus: Facultative anaerobe. Gram positive coccus. Haemolysis on blood agar plates.
Catalase positive. 20% population are long term carriers. Exo and entero toxin may result in toxic shock syndrome and
gastroenteritis respectively. Ideally treated with penicillin although many strains now resistant through beta Lactamase production.
In the UK less than 5% of isolates are sensitive to penicillin. Resistance to methicillin (and other antibiotics) is mediated by the
mec operon , essentially penicillin binding protein is altered and resistance to this class of antibiotics ensues. Common cause of
cutaneous infections and abscesses

Streptococcus pyogenes: Gram positive, forms chain like colonies, Lancefield Group A Streptococcus. Produces beta haemolysis
on blood agar plates. Rarely part of normal skin microflora. Catalase negative. Releases a number of proteins/ virulence factors
into host including hyaluronidase, streptokinase which allow rapid tissue destruction. Releases superantigens such as pyogenic
exotoxin A which results in scarlet fever. Remains sensitive to penicillin, macrolides may be used as an alternative.

Escherichia coli: Gram negative rod. Facultative anaerobe, non sporing. Wide range of subtypes and some are normal gut
commensals. Some subtypes such as 0157 may produce lethal toxins resulting in haemolytic-uraemic syndrome. Enterotoxigenic
E-Coli produces an enterotoxin (ST enterotoxin) that results in large volume fluid secretion into the gut lumen (Via cAMP
activation). Enteropathogenic E-Coli binds to intestinal cells and cause structural damage, this coupled with a moderate (or in
case of enteroinvasive E-Coli significant) invasive component produces enteritis and large volume diarrhoea together with fever.
They are resistant to many antibiotics used to treat gram positive infections and acquire resistance rapidly and are recognised as
producing beta lactamases

Campylobacter jejuni: Curved, gram negative, non sporulating bacteria. One of the commonest causes of diarrhoea worldwide.
Produces enteritis which is often diffuse and blood may be passed. Remains a differential for right iliac fossa pain with diarrhea.
Self limiting infection so antibiotics are not usually advised. However, the quinolones are often rapidly effective.

Helicobacter pylori: Gram negative, helix shaped rod, microaerophilic. Produces hydrogenase that can derive energy from
hydrogen released by intestinal bacteria. Flagellated and mobile. Those carrying the cag A gene may cause ulcers. It secretes
urease that breaks down gastric urea> Carbon dioxide and ammonia> ammonium>bicarbonate (simplified!) The bicarbonate can
neutralise the gastric acid. Usually colonises the gastric antrum and irritates resulting in increased gastrin release and higher levels
of gastric acid. These patients will develop duodenal ulcers. In those with more diffuse H-Pylori infection gastric acid levels are
lower and ulcers develop by local tissue damage from H-Pylori- these patients get gastric ulcers. Diagnosis may be made by
serology (approx. 75% sensitive). Biopsy urease test during endoscopy probably the most sensitive. In patients who are colonised
10-20% risk of peptic ulcer, 1-2% risk gastric cancer, <1% risk MALT lymphoma.

11. Which of the following statements related to necrotising fasciitis is false?


A. Mainly polymicrobial
B. A feature may include 'dirty dishwater fluid' in the wound
C. The presence of crepitus is needed to make the diagnosis
D. Further surgery is mandatory 24-48h after initial surgery to review extension of infection
E. The muscles are relatively spared
Answer: C
Never attempt primary closure after the initial debridement of necrotising fasciitis.

Crepitus may be present in only 35% of cases, therefore its absence should not exclude a diagnosis of necrotising fasciitis.

Meleney's Gangrene and Necrotising Fasciitis: Necrotising fasciitis: Advancing soft tissue infection associated with fascial
necrosis. Uncommon, but can be fatal. In many cases there is underlying background immunosuppression e.g. Diabetes. Caused
by polymicrobial flora (aerobic and anaerobic) and MRSA is seen increasingly in cases of necrotising fasciitis. Streptococcus is
the commonest organism in isolated pathogen infection (15%)

Meleneys gangrene: Meleneys is a similar principle but the infection is more superficially sited than necrotising fasciitis and
often confined to the trunk

Fournier gangrene: Necrotising fasciitis affecting the perineum. Polymicrobial with E.coli and Bacteroides acting in synergy

Clinical features: Fever. Pain. Cellulitis. Oedema. Induration. Numbness

Late findings: Purple/black skin discolouration. Blistering. Haemorrhagic bullae. Crepitus. Dirty Dishwater fluid discharge. Septic
shock. Diagnosis is mainly clinical

Management: Radical surgical debridement forms the cornerstone of management. Sterile dressing is used to dress the wound.
Reconstructive surgery is considered once the infection is completely treated
281
12. A surgical trainee is incising a groin "abscess" in an intravenous drug abuser. Unfortunately the "abscess" is a false
aneurysm and torrential bleeding ensues. In the panic of the situation the doctor then stabs himself in the finger. It
transpires that the patient is a Hepatitis B carrier and the doctor is not immunised! What type of virus is Hepatitis B?
A. Double stranded DNA virus
B. Single stranded DNA virus
C. Double stranded RNA virus
D. Single stranded RNA virus
E. Retrovirus
Answer: A
Hepatitis B is a double-stranded DNA virus and is spread through exposure to infected blood or body fluids, including vertical
transmission from mother to child. The incubation period is 6-20 weeks.

Immunisation against hepatitis B : Contains HBsAg absorbed onto aluminium hydroxide adjuvant and is prepared from yeast
cells using recombinant DNA technology. Most schedules give 3 doses of the vaccine with a recommendation for a one-off
booster 5 years following the initial primary vaccination. At risk groups who should be vaccinated include: healthcare workers,
intravenous drug users, sex workers, close family contacts of an individual with hepatitis B, individuals receiving blood
transfusions regularly, chronic kidney disease patients who may soon require renal replacement therapy, prisoners, chronic liver
disease patients. Around 10-15% of adults fail to respond or respond poorly to 3 doses of the vaccine. Risk factors include age
over 40 years, obesity, smoking, alcohol excess and immunosuppression. Testing for anti-HBs is only recommended for those at
risk of occupational exposure (i.e. Healthcare workers) and patients with chronic kidney disease. In these patients anti-HBs
levels should be checked 1-4 months after primary immunization. The table below shows how to interpret anti-HBs levels:

Anti-HBs level Response


(mIU/ml)
> 100 Indicates adequate response, no further testing required. Should still receive booster at 5 years
10 - 100 Suboptimal response - one additional vaccine dose should be given. If immunocompetent no further testing is
required
< 10 Non-responder. Test for current or past infection. Give further vaccine course (i.e. 3 doses again) with testing
following. If still fails to respond then HBIG would be required for protection if exposed to the virus

Complications of hepatitis B infection: Chronic hepatitis (5-10%). Fulminant liver failure (1%). Hepatocellular carcinoma.
Glomerulonephritis. Polyarteritis nodosa. Cryoglobulinaemia

Management of hepatitis B: Pegylated interferon-alpha used to be the only treatment available. It reduces viral replication in up to
30% of chronic carriers. A better response is predicted by being female, < 50 years old, low HBV DNA levels, non-Asian, HIV
negative, high degree of inflammation on liver biopsy. However, due to the side-effects of pegylated interferon it is now used less
commonly in clinical practice. Oral antiviral medication is increasingly used with an aim to suppress viral replication (not in
dissimilar way to treating HIV patients). Examples include lamivudine, tenofovir and entecavir

13. Which virus is implicated in cervical carcinoma?


A. Human papilloma virus 16
B. Human papilloma virus 2
C. Human herpes virus 8
D. Human herpes virus 2
E. Epstein-Barr virus
Answer: A

Oncoviruses: Viruses which cause cancer. These may be detected on blood test and prevented by vaccine. These are the main
types of oncoviruses and their diseases:
Oncovirus Cancer
Epstein-Barr virus Burkitt's lymphoma
Hodgkin's lymphoma
Post transfusion lymphoma
Nasopharyngeal carcinoma
Human papillomavirus 16/18 Cervical cancer
Anal cancer
Penile cancer
Vulval cancer
Oropharyneal cancer
Human herpes virus 8 Kaposi's sarcoma
Hepatitis B virus Hepatocellular carcinoma
282
Hepatitis C virus Hepatocellular carcinoma
Human T-lymphotropic virus 1 Tropical spastic paraparesis
Adult T cell leukaemia

14. A young woman is admitted to hospital with E-coli 0157 after visiting Germany during an outbreak. Which of the
following is not true of the condition?
A. It may be complicated by micro-angiopathic haemolytic anaemia.
B. Adults typically develop haemolytic uraemic syndome.
C. It is most commonly transmitted by consumption of contaminated food.
D. Plasmids typically confer antibiotic resistance.
E. E-Coli is a gram negative organism.
Answer: B
Children typically develop this complication.

Theme: Infections

A. Staphylococcus aureus
B. Streptococcus bovis
C. Clostridium perfringens
D. Clostridium dificile
E. Clostridium tetani
F. Klebsiella
G. Streptococcus pyogenes
H. Yersinia enterocolitica
I. None of the above

Please select the most likely pathogen to account for the scenario given. Each option may be used once, more than once or not at
all.

15. A 72 year old man with peripheral vascular disease develops a gangrenous toe. This becomes infected and there is
evidence of infection in the surrounding tissues.

Clostridium perfringens. Clostridium perfringens is the most likely pathogen to be associated with gangrene.

16. A 22 year old lady is breastfeeding her first child. One week post partum she presents with a tender indurated mass in
the right breast.

The correct answer is Staphylococcus aureus.Staphylococcus aureus is the commonest cause of lactational mastitis.

17. A 45 year old man is recovering in hospital following a total hip replacement. He develops a profuse and watery
diarrhoea. Several other patients have been suffering from similar symptoms.

Clostridium dificile. Clostridium dificile can spread rapidly on surgical wards. The use of broad spectrum prophylactic
antibiotics during arthroplasty surgery can increase the risk.

18. A 27-year-old male presents to urology for investigation of pyelonephritis. He reports malaise, pyrexia,
lymphadenopathy and a maculopapular rash. The Monospot test is negative. Given a history of high-risk sexual
behaviour you are asked to exclude a HIV seroconversion illness. What is the most appropriate investigation?
A. Antibodies to HIV-2
B. gp120 polymerase chain reaction
C. p24 antigen test
D. CCR5 polymerase chain reaction
E. Antibodies to HIV-1
Answer: C

HIV testing: HIV seroconversion is symptomatic in 60-80% of patients and typically presents as a glandular fever type illness.
Increased symptomatic severity is associated with poorer long term prognosis. It typically occurs 3-12 weeks after infection

Features: sore throat. Lymphadenopathy. malaise, myalgia, arthralgia. Diarrhea. maculopapular rash. mouth ulcers. rarely
meningoencephalitis

283
Diagnosis: antibodies to HIV may not be present. HIV PCR and p24 antigen tests can confirm diagnosis

HIV antibody test: most common and accurate test. usually consists of both a screening ELISA (Enzyme Linked Immuno-Sorbent
Assay) test and a confirmatory Western Blot Assay. most people develop antibodies to HIV at 4-6 weeks but 99% do by 3 months
p24 antigen test: usually positive from about 1 week to 3 - 4 weeks after infection with HIV. sometimes used as an additional
screening test in blood banks

19. Which statement relating to actinomycosis is false?


A. They are gram positive bacilli
B. They are strict aerobes
C. It may be a cause of chronic multiple abscesses
D. Abdominal cases may develop in the appendix
E. Open biopsy of the lesions is the best diagnostic test
Answer: B
They are facultative anaerobes and may be difficult to culture. Direct visualisation of organisms and sulphur granules from
lesions themselves is the easiest way to make a diagnosis. It remains a differential of conditions such as hydradenitis supprativa,
particularly if it is occurring in odd locations and with deeper abscesses than usual.

Abdominal Surgery
1. A 53 year old man undergoes a reversal of a loop colostomy. He recovers well and is discharged home. He is
readmitted 10 days later with symptoms of vomiting and colicky abdominal pain. On examination he has a swelling of
the loop colostomy site and it is tender. What is the most likely underlying diagnosis?
A. Haematoma
B. Intra abdominal adhesions
C. Anastomotic leak
D. Anastomotic stricture
E. Obstructed incisional hernia
Answer: E
Theme from September 2011 Exam: In this scenario the most likely diagnosis would be obstructed incisional hernia. The tender
swelling coupled with symptoms of obstruction point to this diagnosis. Prompt surgical exploration is warranted. Loop colostomy
reversals are at high risk of this complication as the operative site is at increased risk of the development of post operative wound
infections.

Acute incisional hernia: Any surgical procedure involving entry into a cavity containing viscera may be complicated by post
operative hernia. The abdomen is the commonest site. The deep layer of the wound has usually broken down, allowing internal
viscera to protrude through. Management is dictated by the patients clinical status and the timing of the hernia in relation to recent
surgery. Bowel obstruction or tenderness at the hernia site both mandate early surgical intervention to reduce the risk of bowel
necrosis. Mature incisional hernias with a wide neck and no symptoms may be either left or listed for elective repair. Risk factors
for the development of post operative incisional hernias include post operative wound infections, long term steroid use, obesity
and chronic cough

Theme: Abdominal stomas

A. End ileostomy
B. End colostomy
C. Loop ileostomy
D. Loop colostomy
E. End jejunostomy
F. Loop jejunostomy
G. Caecostomy

For each of the following scenarios, please select the most appropriate type of stoma to be constructed. Each option may be
selected once, more than once or not at all.

2. A 56 year old man is undergoing a low anterior resection for carcinoma of the rectum. It is planned to restore intestinal
continuity.
Answer: Loop ileostomy. Colonic resections with an anastomosis below the peritoneal reflection may have an anastomotic leak

284
rate (both clinical and radiological) of up to 15%. Therefore most surgeons will defunction such an anastomosis to reduce the
clinical severity of an anastomotic leak. A loop ileostomy will achieve this end point and is relatively easy to reverse.

3. A 23 year old man with uncontrolled ulcerative colitis is undergoing an emergency sub total colectomy.
Answer: End ileostomy. Following a sub total colectomy the immediate surgical options include an end ileostomy or ileorectal
anastomosis. In the emergency setting an ileorectal anastomosis would be unsafe.

4. A 63 year old women presents with large bowel obstruction. On examination she has a carcinoma 10cm from the anal
verge.
Answer: Loop colostomy. Large bowel obstruction resulting from carcinoma should be resected, stented or defunctioned. The
first two options typically apply to tumours above the peritoneal reflection. Lower tumours should be defunctioned with a loop
colostomy and then formal staging undertaken prior to definitive surgery. An emergency attempted rectal resection carries a
high risk of involvement of the circumferential resection margin and is not recommended.

Abdominal stomas: Stomas may be sited during a range of abdominal procedures and involve bringing the lumen or visceral
contents onto the skin. In most cases this applies to the bowel. However, other organs or their contents may be diverted in case of
need. With bowel stomas the type method of construction and to a lesser extent the site will be determined by the contents of the
bowel. In practice, small bowel stomas should be spouted so that their irritant contents are not in contact with the skin. Colonic
stomas do not need to be spouted as their contents are less irritant. In the ideal situation the site of the stoma should be marked
with the patient prior to surgery. Stoma siting is important as it will ultimately influence the ability of the patient to manage their
stoma and also reduce the risk of leakage. Leakage of stoma contents and subsequent maceration of the surrounding skin can
rapidly progress into a spiraling loss of control of stoma contents.

Types of stomas
Name of stoma Use Common sites
Gastrostomy  Gastric decompression or fixation Epigastrium
 Feeding

Loop jejunostomy Seldom used as very high output Any location according to need
 May be used following emergency laparotomy with
planned early closure

Percutaneous  Usually performed for feeding purposes and site in the Usually left upper quadrant
jejunostomy proximal bowel

Loop ileostomy  Defunctioning of colon e.g. following rectal cancer Usually right iliac fossa
surgery
 Does not decompress colon (if ileocaecal valve
competent)

End ilestomy  Usually following complete excision of colon or where Usually right iliac fossa
ileo-colic anastomosis is not planned
 May be used to defunction colon, but reversal is more
difficult

End colostomy Where a colon is diverted or resected and anastomosis is Either left or right iliac fossa
not primarily achievable or desirable
Loop colostomy  To defunction a distal segment of colon May be located in any region of the abdomen,
 Since both lumens are present the distal lumen acts as a depending upon colonic segment used
vent

Caecostomy Stoma of last resort where loop colostomy is not possible Right iliac fossa
Mucous fistula  To decompress a distal segment of bowel following May be located in any region of the abdomen
colonic division or resection according to clinical need
 Where closure of a distal resection margin is not safe or
achievable

285
Theme: Acute abdominal pain

A. Ruptured abdominal aortic aneurysm


B. Perforated peptic ulcer
C. Perforated appendicitis
D. Mesenteric infarction
E. Small bowel obstruction
F. Large bowel obstruction
G. Pelvic inflammatory disease
H. Mesenteric adenitis
I. Pancreatitis
J. None of the above

Please select the most likely cause of abdominal pain for the scenario given. Each option may be used once, more than once or not
at all.

5. A 75 year old man is admitted with sudden onset severe generalised abdominal pain, vomiting and a single episode of
bloody diarrhoea. On examination he looks unwell and is in uncontrolled atrial fibrillation. Although diffusely tender his
abdomen is soft.
Answer: Mesenteric infarction. In mesenteric infarction there is sudden onset of pain together with vomiting and occasionally
passage of bloody diarrhoea. The pain present is usually out of proportion to the physical signs.

6. A 19 year old lady is admitted with lower abdominal pain. On examination she is diffusely tender. A laparoscopy is
performed and at operation multiple fine adhesions are noted between the liver and abdominal wall. Her appendix is
normal.
Answer: Pelvic inflammatory disease. This is Fitz Hugh Curtis syndrome in which pelvic inflammatory disease (usually
Chlamydia) causes the formation of fine peri hepatic adhesions.

7. A 78 year old man is walking to the bus stop when he suddenly develops severe back pain and collapses. On examination
he has a blood pressure of 90/40 and pulse rate of 110. His abdomen is distended and he is obese. Though tender his
abdomen itself is soft.
Answer: Ruptured abdominal aortic aneurysm. This will be a retroperitoneal rupture (anterior ones generally don't survive to
hospital). The debate regarding CT varies, it is the authors opinion that a systolic BP of <100mmHg at presentation mandates
immediate laparotomy.
Acute mesenteric ischaemia- Pain out of proportion to the physical signs.
Atrial fibrillation is often present. Fitz Hugh Curtis = Fine Hepatic Connections

Acute abdominal pain-diagnoses: Conditions presenting with acute abdominal pain


Condition Features Investigations Management
Appendicitis History of migratory pain. Differential white cell count Appendicectomy
Fever. Pregnancy test
Anorexia. C-Reactive protein
Evidence of right iliac fossa Amylase
tenderness. Urine dipstick testing
Mild pyrexia.
Mesenteric Usually recent upper Full blood count- may show Conservative management-
adenitis respiratory tract infection. slightly raised white cell count appendicectomy if diagnostic doubt
High fever. Urine dipstick often normal
Generalised abdominal Abdominal ultrasound scan -
discomfort- true localised pain usually no free fluid
and signs are rare.
Mittelschmerz Only seen in females. Full blood count- normal Manage conservatively if doubt or
Mid cycle pain. Urine dipstick- normal symptoms fail to settle then
Usually occurs two weeks after Abdominal and pelvic ultrasound- laparoscopy
last menstrual period. may show a trace of pelvic free
Pain is usually has a supra- fluid
pubic location.
Usually subsides over a 24-48
hour period.
Fitz-Hugh Curtis Disseminated infection with Abdominal ultrasound scan- may Usually medically managed-
syndrome Chlamydia. show free fluid doxycycline or azithromycin
Usually seen in females. High vaginal swabs - may show
Consists of evidence of pelvic evidence of sexually transmitted
286
inflammatory disease together infections
with peri-hepatic inflammation
and subsequent adhesion
formation.
Abdominal aortic Sudden onset of abdominal Patients who are Unstable patients should undergo
aneurysm pain radiating to the back in haemodynamically stable should immediate surgery (unless it is not
(ruptured) older adults (look for risk have a CT scan in their best interests).
factors). Those with evidence of contained
Collapse. leak on CT should undergo
May be moribund on arrival in immediate surgery
casualty, more stable if Increasing aneurysmal size is an
contained haematoma. indication for urgent surgical
Careful clinical assessment intervention (that can wait until the
may reveal pulsatile mass. next working day)
Perforated peptic Sudden onset of pain (usually Erect CXR may show free air. A Laparotomy (laparoscopic surgery
ulcer epigastric). CT scan may be indicated where for perforated peptic ulcers is both
Often preceding history of there is diagnostic doubt safe and feasible in experienced
upper abdominal pain. hands)
Soon develop generalised
abdominal pain.
On examination may have
clinical evidence of peritonitis.
Intestinal Colicky abdominal pain and A plain abdominal film may help In those with a virgin abdomen and
obstruction vomiting (the nature of which with making the diagnosis. A CT lower and earlier threshold for
depends on the level of the scan may be useful where laparotomy should exist than in
obstruction). diagnostic uncertainty exists those who may have adhesional
Abdominal distension and obstruction
constipation (again depending
upon site of obstruction).
Features of peritonism may
occur where local necrosis of
bowel loops is occurring.
Mesenteric Embolic events present with Arterial pH and lactate Immediate laparotomy and resection
infarction sudden pain and forceful Arterial phase CT scanning is the of affected segments, in acute
evacuation. most sensitive test embolic events SMA embolectomy
Acute on chronic events may be needed.
usually have a longer history
and previous weight loss.
On examination the pain is
typically greater than the
physical signs would suggest.

Theme: Gastrointestinal bleeding

A. Haemorroids
B. Meckels diverticulum
C. Angiodysplasia
D. Colonic cancer
E. Diverticular bleed
F. Ulcerative colitis
G. Ischaemic colitis

Please select the most likely cause of colonic bleeding for the scenario given. Each option may be used once, more than once or
not at all

8. A 73 year old lady is admitted with a brisk rectal bleed. She is otherwise well and the bleed settles. On examination her
abdomen is soft and non tender. Elective colonoscopy shows a small erythematous lesion in the right colon, but no other
abnormality.
Angiodysplasia: Angiodysplasia can be difficult to identify and treat. The colonoscopic stigmata are easily missed by poor
bowel preparation.

9. A 23 year old man complains of passing bright red blood rectally. It has been occurring over the past week and tends to
occur post defecation. He also suffers from pruritus ani.
Haemorroids: Classical haemorroidal symptoms include bright red rectal bleeding, it typically occurs post defecation and is
287
noticed on the toilet paper and in the toilet pan. It is usually painless, however, thrombosed external haemorroids may be very
painful.

10. A 63 year old man presents with episodic rectal bleeding the blood tends to be dark in colour and may be mixed with
stool. His bowel habit has been erratic since an abdominal aortic aneurysm repair 6 weeks previously.
Ischaemic colitis: The inferior mesenteric artery may have been ligated and being an arteriopath collateral flow through the
marginal may be imperfect.

Colonic bleeding: This typically presents as bright red or dark red blood per rectum. Colonic bleeding rarely presents as malaena
type stool, this is because blood in the colon has a powerful laxative effect and is rarely retained long enough for transformation to
occur and because the digestive enzymes present in the small bowel are not present in the colon. Up to 15% of patients presenting
with haemochezia will have an upper gastrointestinal source of haemorrhage. As a general rule right sided bleeds tend to present
with darker coloured blood than left sided bleeds. Haemorrhoidal bleeding typically presents as bright red rectal bleeding that
occurs post defecation either onto toilet paper or into the toilet pan. It is very unusual for haemorrhoids alone to cause any degree
of haemodynamic compromise.

Colitis: Bleeding may be brisk in advanced cases, diarrhoea is commonly present. Abdominal x-ray may show featureless colon.

Diverticular disease: Acute diverticulitis often is not complicated by major bleeding and diverticular bleeds often occur
sporadically. 75% all will cease spontaneously within 24-48 hours. Bleeding is often dark and of large volume.

Cancer :Colonic cancers often bleed and for many patients this may be the first sign of the disease. Major bleeding from early
lesions is uncommon

Haemorrhoidal bleeding :Typically bright red bleeding occurring post defecation. Although patients may give graphic
descriptions bleeding of sufficient volume to cause haemodynamic compromise is rare.

Angiodysplasia :Apart from bleeding, which may be massive, these arteriovenous lesions cause little in the way of symptoms.
The right side of the colon is more commonly affected.

Management: Prompt correction of any haemodynamic compromise is required. Unlike upper gastrointestinal bleeding the first
line management is usually supportive. This is because in the acute setting endoscopy is rarely helpful. When haemorrhoidal
bleeding is suspected a proctosigmoidoscopy is reasonable as attempts at full colonoscopy are usually time consuming and often
futile. In the unstable patient the usual procedure would be an angiogram (either CT or percutaneous), when these are performed
during a period of haemodynamic instability they may show a bleeding point and may be the only way of identifying a patch of
angiodysplasia. In others who are more stable the standard procedure would be a colonoscopy in the elective setting. In patients
undergoing angiography attempts can be made to address the lesion in question such as coiling. Otherwise surgery will be
necessary. In patients with ulcerative colitis who have significant haemorrhage the standard approach would be a sub total
colectomy, particularly if medical management has already been tried and is not effective.

Indications for surgery: Patients > 60 years. Continued bleeding despite endoscopic intervention. Recurrent bleeding. Known
cardiovascular disease with poor response to hypotension

Surgery: Selective mesenteric embolisation if life threatening bleeding. This is most helpful if conducted during a period of
relative haemodynamic instability. If all haemodynamic parameters are normal then the bleeding is most likely to have stopped
and any angiography normal in appearance. In many units a CT angiogram will replace selective angiography but the same
caveats will apply. If source of colonic bleeding unclear perform a laparotomy, on table colonic lavage and following this attempt
a resection. A blind sub total colectomy is most unwise, for example bleeding from an small bowel arterio-venous malformation
will not be treated by this manoeuvre.

Summary of Acute Lower GI bleeding recommendations


Consider admission if: Over 60 years. Haemodynamically unstable/profuse PR bleeding. On aspirin or NSAID. Significant co
morbidity. Management: All patients should have a history and examination, PR and proctoscopy. Colonoscopic haemostasis
aimed for in post polypectomy or diverticular bleeding

Theme: Surgical signs

A. Rovsing's sign

288
B. Boas' sign
C. Psoas stretch sign
D. Cullen's sign
E. Grey-Turner's sign
F. Murphy's sign
G. None of the above

Please select the most appropriate eponymous abdominal sign for the scenario given. Each option may be used once, more than
once or not at all.

11. Severe acute peri-umbilical bruising in the setting of acute pancreatitis.


Cullen's sign: Cullens sign occurs when there has been intraabdominal haemorrage. It is seen in cases of severe haemorrhagic
pancreatitis and is associated with a poor prognosis. It is also seen in other cases of intraabdominal haemorrhage (such as
ruptured ectopic pregnancy).

12. In acute cholecystitis there is hyperaesthesia beneath the right scapula.


Boas' sign: Boas sign refers to this hyperaesthesia. It occurs because the abdominal wall innervation of this region is from the
spinal roots that lie at this level.

13. In appendicitis palpation of the left iliac fossa causes pain in the right iliac fossa.
Rovsing's sign: Rovsings sign elicits tenderness because the deep palpation induces shift of the appendix (which is inflamed)
against the peritoneal surface. This has somatic innervation and will therefore localise the pain. It is less reliable in pelvic
appendicitis and when the appendix is truly retrocaecal

Abdominal signs: A number of eponymous abdominal signs are noted. These include: Rovsings sign- appendicitis. Boas sign –
cholecystitis. Murphys sign- cholecystitis. Cullens sign- pancreatitis (other intraabdominal haemorrhage). Grey-Turners sign-
pancreatitis (or other retroperitoneal haemorrhage)

Theme: Surgical access

A. Gridiron
B. Lanz
C. McEvedy
D. Midline abdominal
E. Rutherford Morrison
F. Battle (abdominal)
G. Lower midline

Please select the most appropriate incision for the procedure required. Each option may be used once, more than once or not at all.

14. A 78 year old lady is admitted with a tender lump in her right groin. It is within the femoral triangle and there is
concern that there may be small bowel obstruction developing.
McEvedy: This is one approach to an obstructed femoral hernia. It is possible to undertake a small bowel resection through this
approach. Although recourse to laparotomy may be needed if access is difficult.

15. A 45 year old woman with end stage renal failure is due to undergo a cadaveric renal transplant. This will be her first
transplant.
Rutherford Morrison: This is the incision of choice for the extraperitoneal approach to the iliac vessels which will be required
for a renal transplant.

16. A slim 20 year old lady is suffering from appendicitis and requires an appendicectomy.
Lanz: Either a Lanz or Gridiron incision will give access for appendicectomy. However, in the case described a Lanz incision
will give better cosmesis and can be extended should pelvic surgery be required eg for gynaecological disease.

Abdominal incisions: Theme in January 2012 exam


Midline incisionCommonest approach to the abdomen. Structures divided: linea alba, transversalis fascia, extraperitoneal fat,
peritoneum (avoid falciform ligament above the umbilicus). Bladder can be accessed via an extraperitoneal approach through the
space of Retzius

289
Paramedian incision: Parallel to the midline (about 3-4cm). Structures divided/retracted: anterior rectus sheath, rectus (retracted),
posterior rectus sheath, transversalis fascia, extraperitoneal fat, peritoneum. Incision is closed in layers

Battle: Similar location to paramedian but rectus displaced medially (and thus denervated). Now seldom used

Kocher's: Incision under right subcostal margin e.g. Cholecystectomy (open)

Lanz: Incision in right iliac fossa e.g. Appendicectomy

Gridiron: Oblique incision centered over McBurneys point- usually appendicectomy (less cosmetically acceptable than Lanz

Gable: Rooftop incision

Pfannenstiel's: Transverse supra pubic, primarily used to access pelvic organs

McEvedy's: Groin incision e.g. Emergency repair strangulated femoral hernia

Rutherford Morrison: Extraperitoneal approach to left or right lower quadrants. Gives excellent access to iliac vessels and is the
approach of choice for first time renal transplantation.

Theme: Hernias

A. Littres hernia
B. Richters hernia
C. Bochdalek hernia
D. Morgagni hernia
E. Spigelian hernia
F. Lumbar hernia
G. Obturator hernia

Please select the type of hernia that most closely matches the description given. Each option may be used once, more than once or
not at all.

17. A 73 year old lady presents with peritonitis and tenderness of the left groin. At operation she has a left femoral hernia
with perforation of the anti mesenteric border of ileum associated with the hernia.
Richters hernia: When part of the bowel wall is trapped in a hernia such as this it is termed a Richters hernia and may
complicate any hernia although femoral and obturator hernias are most typically implicated.
18. A 22 year old man is operated on for a left inguinal hernia, at operation the sac is opened to reveal a large Meckels
diverticulum.
Littres hernia: Hernia containing Meckels diverticulum is termed a Littres hernia.

19. A 45 year old man has recurrent colicky abdominal pain. As part of a series of investigations he undergoes a CT scan
and this demonstrates a hernia lateral to the rectus muscle at the level of the arcuate line.
Spigelian hernia: This is the site for a spigelian hernia.

Theme: Groin masses

A. Femoral hernia
B. Lymphadenitis
C. Inguinal hernia
D. Psoas abscess
E. Saphenous varix
F. Femoral artery aneurysm
G. Metastatic lymphadenopathy
H. Lymphangitis
I. False femoral artery aneurysm

What is the likely diagnosis for groin mass described? Each option may be used once, more than once, or not at all.

290
20. A 52 year old obese lady reports a painless grape sized mass in her groin area. She has no medical conditions apart from
some varicose veins. There is a cough impulse and the mass disappears on lying down.
Saphenous varix. The history of varicose veins should indicate a more likely diagnosis of a varix. The varix can enlarge during
coughing/sneezing. A blue discolouration may be noted.

21. A 32 year old male is noted to have a tender mass in the right groin area. There are also red streaks on the thigh,
extending from a small abrasion.
Lymphadenitis. The red streaks are along the line of the lymphatics, indicating infection of the lymphatic vessels.
Lymphadenitis is infection of the local lymph nodes.

22. A 23 year old male suffering from hepatitis C presents with right groin pain and swelling. On examination there is a
large abscess in the groin. Adjacent to this is an expansile swelling. There is no cough impulse.
False femoral artery aneurysm: False aneurysms may occur following arterial trauma in IVDU. They may have associated blood
borne virus infections and should undergo duplex scanning prior to surgery. False aneurysms do not contain all layers of the
arterial wall.

Groin masses are common and include: Herniae. Lipomas. Lymph nodes. Undescended testis. Femoral aneurysm. Saphena varix
(more a swelling than a mass!)In the history features relating to systemic illness and tempo of onset will often give a clue as to the
most likely underlying diagnosis.

Groin lumps- some key questions: Is there a cough impulse? Is it pulsatile AND is it expansile (to distinguish between false and
true aneurysm)? Are both testes intra scrotal? Any lesions in the legs such as malignancy or infections (?lymph nodes)? Examine
the ano rectum as anal cancer may metastasise to the groin. Is the lump soft, small and very superficial (?lipoma)?

Scrotal lumps - some key questions: Is the lump entirely intra scrotal? Does it transilluminate (?hydrocele)? Is there a cough
impulse (?hernia)? In most cases a diagnosis can be made clinically. Where it is not clear an ultrasound scan is often the most
convenient next investigation.

Theme: Right iliac fossa pain

A. Urinary tract infection


B. Appendicitis
C. Mittelschmerz
D. Mesenteric adenitis
E. Crohns disease
F. Ulcerative colitis
G. Meckels diverticulum

Please select the most likely cause for right iliac fossa pain for the scenario given. Each option may be used once, more than once
or not at all.

23. A 17 year old male is admitted with lower abdominal discomfort. He has been suffering from intermittent right iliac
fossa pain for the past few months. His past medical history includes a negative colonoscopy and gastroscopy for iron
deficiency anaemia. The pain is worse after meals. Inflammatory markers are normal.
Meckels diverticulum. This scenario should raise suspicion for Meckels as these may contain ectopic gastric mucosa which may
secrete acid with subsequent bleeding and ulceration.

24. A 14 year old female is admitted with sudden onset right iliac fossa pain. She is otherwise well and on examination has
some right iliac fossa tenderness but no guarding. She is afebrile. Urinary dipstick is normal. Her previous menstrual
period two weeks ago was normal and pregnancy test is negative.
Mittelschmerz: Typical story and timing for mid cycle pain. Mid cycle pain typically occurs because a small amount of fluid is
released at the time of ovulation. It will usually resolve over 24-48 hours.

25. A 21 year old male is admitted with a 3 month history of intermittent right iliac fossa pain. He suffers from episodic
diarrhoea and has lost 2 kilos in weight. On examination he has some right iliac fossa tenderness and is febrile.
Crohns disease: Weight loss and chronic symptoms coupled with change in bowel habit should raise suspicion for Crohns. The
presence of intermittent right iliac fossa pain is far more typical of terminal ileal Crohns disease. Both UC and Crohns may be
associated with a low grade pyrexia. The main concern here would be locally perforated Crohns disease with a small associated
abscess.

291
Right iliac fossa pain

Appendicitis: Pain radiating to right iliac fossa. Anorexia. Typically short history. Diarrhoea and profuse vomiting rare

Crohn's disease: Often long history. Signs of malnutrition. Change in bowel habit, especially diarrhoea

Mesenteric adenitis: Mainly affects children. Causes include Adenoviruses, Epstein Barr Virus, beta-haemolytic Streptococcus,
Staphylococcus spp., Escherichia coli, Streptococcus viridans and Yersinia spp. Patients have a higher temperature than those
with appendicitis. If laparotomy is performed, enlarged mesenteric lymph nodes will be present

Diverticulitis: Both left and right sided disease may present with right iliac fossa pain. Clinical history may be similar, although
some change in bowel habit is usual. When suspected a CT scan may help in refining the diagnosis

Meckel's diverticulitis: A Meckel's diverticulum is a congenital abnormality that is present in about 2% of the population.
Typically 2 feet proximal to the ileocaecal valve: May be lined by ectopic gastric mucosal tissue and produce bleeding.

Perforated peptic ulcer: This usually produces upper quadrant pain but pain may be lower. Perforations typically have a sharp
sudden onset of pain in the history

Incarcerated right inguinal or femoral hernia: Usually only right iliac fossa pain if right sided or bowel obstruction.

Bowel perforation secondary to caecal or colon carcinoma: Seldom localised to right iliac fossa, although complete large
bowel obstruction with caecal distension may cause pain prior to perforation.

Gynaecological causes: Pelvic inflammatory disease/salpingitis/pelvic abscess/Ectopic pregnancy/Ovarian torsion/Threatened or


complete abortion/Mittelschmerz

Urological causes: Ureteric colic/UTI/Testicular torsion

Other causes: TB/Typhoid/Herpes Zoster/AAA/Situs inversus

26. A 78 year old lady presents with colicky abdominal pain and a tender mass in her groin. On examination there is a small
firm mass below and lateral to the pubic tubercle. Which of the following is the most likely underlying diagnosis?
A. Incarcerated inguinal hernia
B. Thrombophlebitis of a saphena varix
C. Incarcerated femoral hernia
D. Incarcerated obturator hernia
E. Deep vein thrombosis
Answer: C
Femoral hernia = High risk of strangulation (repair urgently)
Femoral herniae account for <10% of all groin hernias. In the scenario the combination of symptoms of intestinal compromise
with a mass in the region of the femoral canal points to femoral hernia as the most likely cause.

Femoral canal: The femoral canal lies at the medial aspect of the femoral sheath. The femoral sheath is a fascial tunnel
containing both the femoral artery laterally and femoral vein medially. The canal lies medial to the vein.

Borders of the femoral canal: Laterally: Femoral vein. Medially: Lacunar ligament. Anteriorly: Inguinal ligament.
Posteriorly: Pectineal ligament. Contents: Lymphatic vessels. Cloquet's lymph node. Physiological significance
Allows the femoral vein to expand to allow for increased venous return to the lower limbs. Pathological significance
As a potential space, it is the site of femoral hernias. The relatively tight neck places these at high risk of strangulation.

27. Which of the following is not a typical feature of acute appendicitis?


A. Neutrophilia
B. Profuse vomiting
C. Anorexia
D. Low grade pyrexia
E. Small amounts of protein on urine analysis
Answer: B
Profuse vomiting and diarrhoea are rare in early appendicitis

292
Whilst patients may vomit once or twice, profuse vomiting is unusual, and would fit more with gastroenteritis or an ileus. A
trace of protein is not an uncommon occurrence in acute appendicitis. A free lying pelvic appendix may result in localised
bladder irritation, with inflammation occurring as a secondary phenomena. This latter feature may result in patients being
incorrectly diagnosed as having a urinary tract infection. A urine dipstick test is useful in differentiating between the two
conditions.

Appendicitis : History: Peri umbilical abdominal pain (visceral stretching of appendix lumen and appendix is mid gut
structure) radiating to the right iliac fossa due to localised parietal peritoneal inflammation. Vomit once or twice but marked and
persistent vomiting is unusual. Diarrhoea is rare. However, pelvic appendicitis may cause localised rectal irritation of some
loose stools. A pelvic abscess may also cause diarrhoea. Mild pyrexia is common - temperature is usually 37.5 -38oC. Higher
temperatures are more typical of conditions like mesenteric adenitis. Anorexia is very common. It is very unusual for patients
with appendicitis to be hungry.

Examination: Generalised peritonitis if perforation has occurred or localised peritonism. Retrocaecal appendicitis may have
relatively few signs. Digital rectal examination may reveal boggy sensation if pelvic abscess is present, or even tenderness with
a pelvic appendix.

Diagnosis: Typically raised inflammatory markers coupled with compatible history and examination findings should be enough
to justify appendicectomy. Urine analysis may show mild leucocytosis but no nitrites. Ultrasound is useful if females where
pelvic organ pathology is suspected. Although it is not always possible to visualise the appendix on ultrasound the presence of
free fluid (always pathological in males) should raise suspicion.

Treatment: Appendicectomy which can be performed via either an open or laparoscopic approach. Administration of
metronidazole reduces wound infection rates. Patients with perforated appendicitis require copious abdominal lavage. Patients
without peritonitis who have an appendix mass should receive broad spectrum antibiotics and consideration given to performing
an interval appendicectomy. Be wary in the older patients who may have either an underlying caecal malignancy or perforated
sigmoid diverticular disease.

28. An 28 year old man presents with a direct inguinal hernia. A decision is made to perform an open inguinal hernia repair.
Which of the following is the best option for abdominal wall reconstruction in this case?
A. Suture plication of the transversalis fascia using PDS only
B. Suture plication of the hernial defect with nylon and placement of prolene mesh anterior to external oblique
C. Suture plication of the hernia defect using nylon and re-enforcing with a sutured repair of the abdominal wall
D. Sutured repair of the hernial defect with prolene and placement of prolene mesh over the cord structures in the inguinal canal
E. Sutured repair of the hernial defect using nylon and placement of a prolene mesh posterior to the cord structures
Answer: E
Laparoscopic repair- bilateral and recurrent cases. During an inguinal hernia repair in males the cord structures will always lie
anterior to the mesh. In the conventional open repairs the cord structures are mobilised and the mesh placed behind them, with a
slit made to allow passage of the cord structures through the deep inguinal ring. Placement of the mesh over the cord structures
results in chronic pain and usually a higher risk of recurrence. Laparoscopic inguinal hernia repair is the procedure of choice for
bilateral inguinal hernias.

Types of surgery include: Onlay mesh repair (Litchenstein style). Inguinal herniorrhaphy. Shouldice repair. Darn repair.
Laparoscopic mesh repair

Open mesh repair and laparoscopic repair are the two main procedures in mainstream use. The Shouldice repair is a useful
procedure in cases where a mesh repair would be associated with increased risk of infection, e.g. repair of case with strangulated
bowel, as it avoids the use of mesh. It is, however, far more technically challenging to perform.

Inguinal hernia surgery: occur when the abdominal viscera protrude through the anterior abdominal wall into the inguinal
canal. They may be classified as being either direct or indirect. The distinction between these two rests on their relation to
Hesselbach's triangle.

Boundaries of Hesselbach's Triangle: Medial: Rectus abdominis. Lateral: Inferior epigastric vessels. Inferior: Inguinal
ligament
Hernias occurring within the triangle tend to be direct and those outside - indirect.

Diagnosis: Most cases are diagnosed clinically, a reducible swelling may be located at the level of the inguinal canal. Large
hernia may extend down into the male scrotum, these will not trans-illuminate and it is not possible to "get above" the swelling.
Cases that are unclear on examination, but suspected from the history, may be further investigated using ultrasound or by
performing a herniogram.

293
Treatment: Hernias associated with few symptoms may be managed conservatively. Symptomatic hernias or those which are at
risk of developing complications are usually treated surgically.
First time hernias may be treated by performing an open inguinal hernia repair; the inguinal canal is opened, the hernia reduced
and the defect repaired. A prosthetic mesh may be placed posterior to the cord structures to re-inforce the repair and reduce the
risk of recurrence.
Recurrent hernias and those which are bilateral are generally managed with a laparoscopic approach. This may be via an intra or
extra peritoneal route. As in open surgery a mesh is deployed. However, it will typically lie posterior to the deep ring.

Inguinal hernia in children: Inguinal hernias in children are almost always of an indirect type and therefore are usually dealt
with by herniotomy, rather than herniorraphy. Neonatal hernias especially in those children born prematurely are at highest risk
of strangulation and should be repaired urgently. Other hernias may be repaired on an elective basis.

Theme: Abdominal closure methods

A. Looped 1/0 PDS (polydiaxone)


B. Looped 1/0 silk
C. 1/0 Vicryl (polyglactin)
D. 1/0 Vicryl rapide
E. 2/0 Prolene (Polypropylene)
F. Re-inforced 1/0 Nylon
G. Re-inforced 1/0 Silk
H. Application of VAC system without separation film
I. Application of VAC System with separation film
J. Application of a 'Bogota Bag'

Please select the most appropriate wound closure method (for the deep layer) for the abdominal surgery described.

29. A 59 year old man with morbid obesity undergoes a laparotomy and Hartmans procedure for perforated sigmoid
diverticular disease. At the conclusion of the procedure the abdomen cannot be primarily closed. The Vac system is not
available for use.
Application of a 'Bogota Bag': this is safest as attempted closure will almost certainly fail. Repeat look at 48 hours to determine
the best definitive option is needed.

30. A 73 year old lady undergoes a low anterior resection for carcinoma of the rectum.
Looped 1/0 PDS (polydiaxone): Mass closure obeying Jenkins rule is required and this states that the suture must be 4 times the
length of the wound with tissue bites 1cm deep and 1 cm apart.

31. A 67 year old is returned to theatre after developing a burst abdomen on the ward. She has originally undergone a right
hemicolectomy and the SHO who closed the wound had failed to tie the midline suture correctly. The wound edges
appear healthy.
The correct answer is Re-inforced 1/0 Nylon
Attempt at re-closing the wound is reasonable in which case 1/0 nylon (reinforced with drainage tubing) is often used.

Abdominal wound dehiscence: This is a significant problem facing all surgeons who undertake abdominal surgery on a regular
basis. Traditionally it is said to occur when all layers of an abdominal mass closure fail and the viscera protrude externally
(associated with 30% mortality). It can be subdivided into superficial, in which the skin wound alone fails and complete,
implying failure of all layers.

Factors which increase the risk are: Malnutrition. Vitamin deficiencies. Jaundice. Steroid use. Major wound contamination (e.g.
faecal peritonitis). Poor surgical technique (Mass closure technique is the preferred method-Jenkins Rule)

When sudden full dehiscence occurs the management is as follows: Analgesia. Intravenous fluids. Intravenous broad spectrum
antibiotics. Coverage of the wound with saline impregnated gauze (on the ward). Arrangements made for a return to theatre

Surgical strategy: Correct the underlying cause (eg TPN or NG feed if malnourished). Determine the most appropriate strategy
for managing the wound.

Resuturing of the wound: This may be an option if the wound edges are healthy and there is enough tissue for sufficient
coverage. Deep tension sutures are traditionally used for this purpose.

Application of a wound manager: This is a clear dressing with removable front. Particularly suitable when some granulation
tissue is present over the viscera or where there is a high output bowel fistula present in the dehisced wound.
294
Application of a 'Bogota bag': This is a clear plastic bag that is cut and sutured to the wound edges and is only a temporary
measure to be adopted when the wound cannot be closed and will necessitate a return to theatre for definitive management.

Application of a VAC dressing system: These can be safely used BUT ONLY if the correct layer is interposed between the
suction device and the bowel. Failure to adhere to this absolute rule will almost invariably result in the development of multiple
bowel fistulae and create an extremely difficult management problem.

Theme: Hernias

A. Umbilical hernia
B. Para umbilical hernia
C. Morgagni hernia
D. Littres hernia
E. Bochdalek hernia
F. Richters hernia
G. Obturator hernia

Please select the hernia that most closely matches the description given. Each option may be used once, more than once or not at
all.

32. A 1 day old infant is born with severe respiratory compromise. On examination he has a scaphoid abdomen and an
absent apex beat.
Bochdalek hernia: Theme from 2011 exam. The large hernia may displace the heart although true dextrocardia is not present.
The associated pulmonary hypoplasia will compromise lung development.

33. A 2 month old infant is troubled by recurrent colicky abdominal pain and intermittent intestinal obstruction. On
imaging the transverse colon is herniated into the thoracic cavity, through a mid line defect.
Morgagni hernia: Morgagni hernia may contain the transverse colon. Unless there is substantial herniation pulmonary
hypoplasia is uncommon.

34. A 78 year old lady is admitted with small bowel obstruction, on examination she has a distended abdomen and the leg is
held semi flexed. She has some groin pain radiating to the ipsilateral knee.
Obturator hernia: The groin swelling in obturator hernia is subtle and hard to elicit clinically.

Hernia: Hernias occur when a viscus or part of it protrudes from within its normal anatomical cavity. Specific hernias are
covered under their designated titles the remainder are addressed here.

Spigelian hernia: Interparietal hernia occurring at the level of the arcuate line. Rare. May lie beneath internal oblique muscle.
Usually between internal and external oblique. Equal sex distribution. Position is lateral to rectus abdominis. Both open and
laparoscopic repair are possible, the former in cases of strangulation.

Lumbar hernia: The lumbar triangle (through which these may occur) is bounded by: Crest of ilium (inferiorly). External
oblique (laterally). Latissimus dorsi (medially). Primary lumbar herniae are rare and most are incisional hernias following renal
surgery. Direct anatomical repair with or without mesh re-enforcement is the procedure of choice.

Obturator hernia: Herniation through the obturator canal. Commoner in females. Usually lies behind pectineus muscle.
Elective diagnosis is unusual most will present acutely with obstruction. When presenting acutely most cases with require
laparotomy or laparoscopy (and small bowel resection if indicated).

Richters hernia: Condition in which part of the wall of the small bowel (usually the anti mesenteric border) is strangulated
within a hernia (of any type). They do not present with typical features of intestinal obstruction as luminal patency is preserved.
Where vomiting is prominent it usually occurs as a result of paralytic ileus from peritonitis (as these hernias may perforate).

Incisional hernia: Occur through sites of surgical access into the abdominal cavity. Most common following surgical wound
infection. To minimise following midline laparotomy Jenkins Rule should be followed and this necessitates a suture length 4x
length of incision with bites taken at 1cm intervals, 1 cm from the wound edge. Repair may be performed either at open surgery
or laparoscopically and a wide variety of techniques are described.

295
Bochdalek hernia: Typically congenital diaphragmatic hernia. 85% cases are located in the left hemi diaphragm. Associated
with lung hypoplasia on the affected side. More common in males. Associated with other birth defects. May contain stomach.
May be treated by direct anatomical apposition or placement of mesh. In infants that have severe respiratory compromise
mechanical ventilation may be needed and mortality rate is high.

Morgagni Hernia: Rare type of diaphragmatic hernia (approx 2% cases). Herniation through foramen of Morgagni. Usually
located on the right and tend to be less symptomatic. More advanced cases may contain transverse colon. As defects are small
pulmonary hypoplasia is less common. Direct anatomical repair is performed.

Umbilical hernia: Hernia through weak umbilicus. Usually presents in childhood. Often symptomatic. Equal sex incidence.
95% will resolve by the age of 2 years. Thereafter surgical repair is warranted.

Paraumbilical hernia: Usually a condition of adulthood. Defect is in the linea alba. More common in females. Multiparity and
obesity are risk factors. Traditionally repaired using Mayos technique - overlapping repair, mesh may be used though not if
small bowel resection is required owing to acute strangulation.

Littres hernia: Hernia containing Meckels diverticulum. Resection of the diverticulum is usually required and this will
preclude a mesh repair.

Theme: Right iliac fossa pain

A. Open Appendicectomy
B. Laparoscopic appendicectomy
C. Laparotomy
D. CT Scan
E. Colonoscopy
F.Ultrasound scan abdomen/pelvis
G. Active observation

For each scenario please select the most appropriate management option from the list. Each option may be used once, more than
once or not at all.

35. A 21 year old women is admitted with a 48 hour history of worsening right iliac fossa pain. She has been nauseated and
vomited twice. On examination she is markedly tender in the right iliac fossa with localised guarding. Vaginal
examination is unremarkable. Urine dipstick is negative. Blood tests show a WCC of 13.5 and CRP 70.
Appendicectomy: She is likely to have appendicitis. In women of this age there is always diagnostic uncertainty. With a normal
vaginal exam laparoscopy would be preferred over USS.

36. An 8 year old boy presents with a 4 hour history of right iliac fossa pain with nausea and vomiting. He has been back at
school for two days after being kept home with a flu like illness. On examination he is tender in the right iliac fossa,
although his abdomen is soft. Temperature is 38.3oc. Blood tests show a CRP of 40 and a WCC of 8.1.
Answer: Active observation: This is mesenteric adenitis. Note history of flu like illness and temp > 38o c.
The decision as to how to manage this situation is based on the abdominal findings. Patients with localising signs such as
guarding or peritonism should undergo surgery.

37. A 21 year old women presents with right iliac fossa pain. She reports some bloodstained vaginal discharge. She has a HR
of 65 bpm.
Ultrasound scan abdomen/pelvisThis patient is suspected of having an ectopic pregnancy. She needs an urgent β HCG and USS
of the pelvis. If she were haemodynamically unstable then laparotomy would be indicated.

38. Which of the following is not a typical feature of irritable bowel syndrome?
A. A change in the consistency of stools
B. Abdominal pain relieved with defecation
C. A change in frequency of defecation
D. Abdominal bloating
E. Pain at a single fixed site

296
Answer: E
The pain or discomfort of IBS is typically migratory and variable in intensity. Pain at a fixed site is suggestive of malignancy.
Abdominal bloating is an extremely common feature.

Irritable bowel syndrome: The diagnosis of irritable bowel syndrome is made according to the ROME III diagnostic criteria
which state:

Recurrent abdominal pain or discomfort at 3 days per month for the past 3 months associated with two or more of the
following: Improvement with defecation. Onset associated with a change in the frequency of stool. Onset associated with a
change in the form of the stool. Features such as lethargy, nausea, backache and bladder symptoms may also support the
diagnosis

Red flag features should be inquired about: Rectal bleeding. Unexplained/unintentional weight loss. Family history of bowel
or ovarian cancer. Onset after 60 years of age

Suggested investigations are: Full blood count. ESR/CRP. Coeliac disease screen (tissue transglutaminase antibodies).
Colonoscopy (if worrying symptoms, positive family history). Thyroid function tests. Glucose (ensure not diabetic)

The NICE criteria state that blood tests alone will suffice in people fulfilling the diagnostic criteria. We would point out that
luminal colonic studies should be considered early in patients with altered bowel habit referred to hospital and a diagnosis of
IBS should still be largely one of exclusion.

Treatment: Usually reduce fibre intake. Tailored prescriptions of laxatives or loperamide according to clinical picture. Dietary
modification (caffeine avoidance, less carbonated drinks). Consider low dose tricyclic antidepressants if pain is a dominant
symptom. Biofeedback may help.

Theme: Causes of abdominal pain

A. Acute on chronic mesenteric ischaemia


B. Ruptured aortic aneurysm
C. Acute Pancreatitis
D. Acute mesenteric embolus
E. Acute appendicitis
F.Chronic pancreatitis
G. Mesenteric vein thrombosis

Please select the most likely underlying diagnosis from the list above. Each option may be used once, more than once or not at
all.

39. A 41 year old man is admitted with peritonitis secondary to a perforated appendix. He is treated with a laparoscopic
appendicectomy but has a stormy post operative course. He in now developing increasing abdominal pain and has been
vomiting. A laparotomy is performed and at operation a large amount of small bowel shows evidence of patchy areas of
infarction.
Mesenteric vein thrombosis: Mesenteric vein thrombosis may complicate severe intra abdominal sepsis and when it progresses
may impair bowel perfusion. The serosa is quite resistant to ischaemia so in this case the appearances are usually patchy.

40. A 68 year old man is admitted with abdominal pain and vomiting of 48 hours duration, the pain radiates to his back and
he has required a considerable amount of volume replacement. Amylase is 741.
Acute Pancreatitis: Although back pain and abdominal pain coupled with haemodynamic compromise may suggest ruptured
AAA the 48 hour history and amylase >3 times normal go against this diagnosis.

41. A 79 year old lady develops sudden onset of abdominal pain and collapses, she has passed a large amount of diarrhoea.
In casualty her pH is 7.35 and WCC is 18.
The correct answer is Acute mesenteric embolus. Although mesenteric infarct may raise the lactate the pH may be raised often
secondary to vomiting.

Mesenteric vessel disease: Mesenteric ischaemia accounts for 1 in 1000 acute surgical admissions. It is primarily caused by
arterial embolism resulting in infarction of the colon. It is more likely to occur in areas such as the splenic flexure that are
located at the borders of the territory supplied by the superior and inferior mesenteric arteries.

297
Types: Acute mesenteric embolus (commonest 50%): Sudden onset abdominal pain followed by profuse diarrhoea.May be
associated with vomiting. Rapid clinical deterioration. Serological tests: WCC, lactate, amylase may all be abnormal
particularly in established disease. These can be normal in the early phases.

Acute on chronic mesenteric ischaemia: Usually longer prodromal history. Post prandial abdominal discomfort and weight
loss are dominant features. Patients will usually present with an acute on chronic event, but otherwise will tend not to present
until mesenteric flow is reduced by greater than 80%. When acute thrombosis occurs presentation may be as above. In the
chronic setting the symptoms will often be those of ischaemic colitis (mucosa is the most sensitive area to this insult).

Mesenteric vein thrombosis: Usually a history over weeks. Overt abdominal signs and symptoms will not occur until venous
thrombosis has reached a stage to compromise arterial inflow. Thrombophilia accounts for 60% of cases.

Low flow mesenteric infarction: This occurs in patients with multiple co morbidities in whom mesenteric perfusion is
significantly compromised by overuse of inotropes or background cardiovascular compromise. The end result is that the bowel
is not adequately perfused and infarcts occur from the mucosa outwards.

Diagnosis: Serological tests: WCC, lactate, CRP, amylase (can be normal in early disease). Cornerstone for diagnosis of arterial
AND venous mesenteric disease is CT angiography scanning in the arterial phase with thin slices (<5mm). Venous phase
contrast is not helpful. SMA duplex USS is useful in the evaluation of proximal SMA disease in patients with chronic
mesenteric ischaemia. MRI is of limited use due to gut peristalsis and movement artefact.

Management: Overt signs of peritonism: Laparotomy. Mesenteric vein thrombosis: If no peritonism: Medical management
with IV heparin. At operation limited resection of frankly necrotic bowel with view to relook laparotomy at 24-48h. In the
interim urgent bowel revascularisation via endovascular (preferred) or surgery.

Prognosis: Overall poor. Best outlook is from an acute ischaemia from an embolic event where surgery occurs within 12h.
Survival may be 50%. This falls to 30% with treatment delay. The other conditions carry worse survival figures.

Theme: Groin masses

A. Femoral aneurysm
B. Lymphadenitis
C. Saphena varix
D. Femoral hernia
E. Indirect inguinal hernia
F. Direct inguinal hernia
G. Psoas abscess

What is the likely diagnosis for the groin mass described? Each option may be used once, more than once or not at all.

42. A 3 year old boy is referred to the clinic with a scrotal swelling. On examination the mass does not transilluminate and it
is impossible to palpate normal cord above it.
Indirect inguinal hernia. This is likely to be an indirect hernia. In children these arise from persistent processus vaginalis and
require herniotomy.

43. A 52 year old obese lady reports a painless mass in the groin area. A mass is noted on coughing. It is below and lateral to
the pubic tubercle.
Femoral hernia. A mass below and lateral to the pubic tubercle is indicative of a femoral hernia.

44. A 21 year old man is admitted with a tender mass in the right groin, fevers and sweats. He is on multiple medical
therapy for HIV infection. On examination he has a swelling in his right groin, hip extension exacerbates the pain.
Psoas abscess. Psoas abscesses may be either primary or secondary. Primary cases often occur in the immunosuppressed and
may occur as a result of haematogenous spread. Secondary cases may complicated intra abdominal diseases such as Crohns.
Patients usually present with low back pain and if the abscess is extensive a mass that may be localised to the inguinal region or
femoral triangle . Smaller collections may be percutaneously drained. If the collection is larger, or the percutaneous route fails,
then surgery (via a retroperitoneal approach) should be performed.

45. A 56 year old lady is admitted with colicky abdominal pain. A plain x-ray is performed. Which of the following should
not show fluid levels on a plain abdominal film?

298
A. Stomach
B. Jejunum
C. Ileum
D. Caecum
E. Descending colon
Answer: E
Fluid levels in the distal colon are nearly always pathological. In general contents of the left colon transit quickly and are
seldom held in situ for long periods, the content is also more solid.

Abdominal radiology: Plain abdominal x-rays are often used as a first line investigation in patients with acute abdominal pain.
A plain abdominal film may demonstrate free air, evidence of bowel obstruction and possibly other causes of pain (e.g. renal or
gallbladder stones).
Investigation of potential visceral perforation is usually best performed by obtaining an erect chest x-ray, as this is a more
sensitive investigation for suspected visceral perforation.

Features which are usually abnormal: Large amounts of free air (colonic perforation), smaller volumes seen with more
proximal perforations. A positive Riglers sign (gas on both sides of the bowel wall). Caecal diameter of >8cm. Fluid levels in
the colon. Ground glass appearance to film (usually due to large amounts of free fluid). Sentinel loop in patients with
inflammation of other organs (e.g. pancreatitis).

Features which should be expected/ or occur without pathology: In Chialditis syndrome, a loop of bowel may be interposed
between the liver and diaphragm, giving the mistaken impression that free air is present. Following ERCP (and sphincterotomy)
air may be identified in the biliary tree. Free intra abdominal air following laparoscopy / laparotomy, although usually dissipates
after 48-72 hours.

46. A 56 year old lady presents with a large bowel obstruction and abdominal distension. Which of the following
confirmatory tests should be performed prior to surgery
A. Abdominal ultrasound scan
B. Barium enema
C. Rectal MRI Scan
D. Endoanal ultrasound scan
E. Gastrograffin enema
Answer: E
Patients with clinical evidence of large bowel obstruction, should have the presence or absence of an obstructing lesion
confirmed prior to surgery. This is because colonic pseudo-obstruction may produce a similar radiological picture. A
gastrograffin enema is the traditional test, as barium is too toxic if it spills into the abdominal cavity. An MRI scan will not
provide the relevant information, unless the lesion is rectal and below the peritoneal reflection.

Theme: Management of splenic trauma

A. Splenectomy
B. Angiography
C. CT Scan
D. Admit for bed rest and observation
E. Ultrasound scan
F.Splenic conservation
G. MRI of the abdomen

Please select the most appropriate intervention for the scenario given. Each option may be used once, more than once or not at
all.

47. A 7 year old boy falls off a wall the distance is 7 feet. He lands on his left side and there is left flank bruising. There is no
haematuria. He is otherwise stable and haemoglobin is within normal limits.
Ultrasound scan: This will demonstrate any overt splenic injury. A CT scan carries a significant dose of radiation. In the
absence of haemodynamic instability or other major associated injuries the use of USS to exclude intraabdominal free fluid
(blood) would seem safe when coupled with active observation. An USS will also show splenic haematomas.

48. A 42 year old motorcyclist is involved in a road traffic accident. A FAST scan in the emergency department shows free
intrabdominal fluid and a laparotomy is performed. At operation there is evidence of small liver laceration that has
stopped bleeding and a tear to the inferior pole of the spleen.
Splenic conservation: As minimum damage, attempt conservation.

299
49. An 18 year old man is involved in a road traffic accident. A CT scan shows disruption of the splenic hilum and a
moderate sized perisplenic haematoma.
Splenectomy: Hilar injuries usually mandate splenectomy. The main risk with conservative management here is that he will
rebleed and with hilar injuries this can be dramatic.

Splenic trauma: The spleen is one of the more commonly injured intra abdominal organs. In most cases the spleen can be
conserved. The management is dictated by the associated injuries, haemodynamic status and extent of direct splenic injury.

Management of splenic trauma: Conservative: Small subcapsular haematoma. Minimal intra abdominal blood. No hilar
disruption

Laparotomy with conservation: Increased amounts of intraabdominal blood. Moderate haemodynamic compromise. Tears or
lacerations affecting <50%

Resection: Hilar injuries. Major haemorrhage. Major associated injuries

Splenectomy: GA. Long midline incision. If time permits insert a self retaining retractor (e.g. Balfour/ omnitract). Large
amount of free blood is usually present. Pack all 4 quadrants of the abdomen. Allow the anaesthetist to 'catch up'. Remove the
packs and assess the viability of the spleen. Hilar injuries and extensive parenchymal lacerations will usually require
splenectomy. Divide the short gastric vessels and ligate them. Clamp the splenic artery and vein. Two clamps on the patient side
are better and allow for double ligation and serve as a safety net if your assistant does not release the clamp smoothly. Be
careful not to damage the tail of the pancreas, if you do then this will need to be formally removed and the pancreatic duct
closed. Wash out the abdomen and place a tube drain to the splenic bed. Some surgeons implant a portion of spleen into the
omentum, whether you decide to do this is a matter of personal choice. Post operatively the patient will require prophylactic
penicillin V and pneumococcal vaccine.

Elective: Elective splenectomy is a very different operation from that performed in the emergency setting. The spleen is often
large (sometimes massive). Most cases can be performed laparoscopically. The spleen will often be macerated inside a
specimen bag to facilitate extraction.

Complications: Haemorrhage (may be early and either from short gastrics or splenic hilar vessels. Pancreatic fistula (from
iatrogenic damage to pancreatic tail). Thrombocytosis: prophylactic aspirin. Encapsulated bacteria infection e.g. Strep.
pneumoniae, Haemophilus influenzae and Neisseria meningitidis

50. Which of the following does not increase the risk of abdominal wound dehiscence following laparotomy?
A. Jaundice
B. Abdominal compartment syndrome
C. Poorly controlled diabetes mellitus
D. Administration of intravenous steroids
E. Use of Ketamine as an anaesthetic agent
Answer: E
Ketamine does not affect healing. All the other situations in the list carry a strong association with poor healing and risk of
dehisence.

Theme: Causes of diarrhoea

A. Campylobacter jejuni infection


B. Salmonella gastroenteritis infection
C. Crohns disease
D. Ulcerative colitis
E. Irritable bowel syndrome
F. Ischaemic colitis
G. Laxative abuse
H. Clostridium difficile infection

Please select the most likely cause of diarrhoea for each scenario given. Each option may be used once, more than once or not at
all.

51. A 23 year old lady has suffered from diarrhoea for 8 months, she has also lost 2 Kg in weight. At colonoscopy
appearances of melanosis coli are identified and confirmed on biopsy

300
The correct answer is Laxative abuse: This may occur as a result of laxative abuse and consists of lipofuschin laden
marcophages that appear brown.

52. A 68 year old lady has recently undergone an abdominal aortic aneurysm repair. The operation was performed
electively and was uncomplicated. Since surgery she has had repeated episodes of diarrhoea.
Ischaemic colitis: The IMA is commonly ligated during and AAA repair and this may then render the left colon relatively
ischaemic, thereby causing mesenteric colitis. Treatment is supportive and most cases will settle with conservative management.

53. A 23 year old man is admitted to hospital with diarrhoea and severe abdominal pain. He was previously well and his
illness has lasted 18 hours.
The correct answer is Campylobacter jejuni infection: Severe abdominal pain tends to favour Campylobacter infection.

Diarrhoea: World Health Organisation definitions: Diarrhoea: > 3 loose or watery stool per day. Acute diarrhoea < 14 days.
Chronic diarrhoea > 14 days

Acute Diarrhoea: Gastroenteritis: May be accompanied by abdominal pain or nausea/vomiting


Diverticulitis: Classically causes left lower quadrant pain, diarrhoea and fever.

Antibiotic therapy: More common with broad spectrum antibiotics


Clostridium difficile is also seen with antibiotic use.

Constipation causing overflow: A history of alternating diarrhoea and constipation may be given
May lead to faecal incontinence in the elderly

Chronic Diarrhoea
Irritable bowel syndrome: Extremely common. The most consistent features are abdominal pain, bloating and change in bowel
habit. Patients may be divided into those with diarrhoea predominant IBS and those with constipation predominant IBS.
Features such as lethargy, nausea, backache and bladder symptoms may also be present

Ulcerative colitis: Bloody diarrhoea may be seen. Crampy abdominal pain and weight loss are also common. Faecal urgency
and tenesmus may occur

Crohn's disease: Crampy abdominal pains and diarrhoea. Bloody diarrhoea less common than in ulcerative colitis. Other
features include malabsorption, mouth ulcers perianal disease and intestinal obstruction

Colorectal cancer: Symptoms depend on the site of the lesion but include diarrhoea, rectal bleeding, anaemia and constitutional
symptoms e.g. Weight loss and anorexia

Coeliac disease: In children may present with failure to thrive, diarrhoea and abdominal distension. In adults lethargy, anaemia,
diarrhoea and weight loss are seen. Other autoimmune conditions may coexist

Other conditions associated with diarrhoea include: Thyrotoxicosis. Laxative abuse. Appendicitis with pelvic abscess or pelvic
appendix. Radiation enteritis

Diagnosis: Stool culture. Abdominal and digital rectal examination. Consider colonoscopy (radiological studies unhelpful)
Thyroid function tests, serum calcium, anti endomysial antibodies, glucose.

54. A 6 year old child presents with colicky abdominal pain, vomiting and the passage of red current jelly stool per rectum.
On examination the child has a tender abdomen and a palpable mass in the right upper quadrant. Imaging shows an
intussusception. Which of the conditions below is least recognised as a precipitant
A. Inflammation of Payers patches
B. Cystic fibrosis
C. Meckels diverticulum
D. Mesenteric cyst
E. Mucosal polyps
Answer: D

301
Mesenteric cysts may be associated with intra abdominal catastrophes where these occur they are typically either intestinal
volvulus or intestinal infarction. They seldom cause intussusception. Cystic fibrosis may lead to the formation of meconium
ileus equivalent and plugs may occasionally serve as the lead points for an intussusception.

Intussusception: Paediatric: Intussusception typcially presents with colicky abdominal pain and vomiting. The telescoping of
the bowel produces mucosal ischaemia and bleeding may occur resulting in the passage of "red current jelly" stools. Recognised
causes include lumenal pathologies such as polyps, lymphadenopathy and diseases such as cystic fibrosis. Idiopathic
intussceception of the ileocaecal valve and terminal ileum is the most common variant and typically affects young children and
toddlers.
The diagnosis is usually made by abdominal ultrasound investigation. The decision as to the optimal treatment is dictated by the
patients physiological status and abdominal signs. In general children who are unstable with localising peritoneal signs should
undergo laparotomy as should those in whom attempted radiological reduction has failed.
In relatively well children without localising signs attempted hydrostatic reduction under fluroscopic guidance is the usual
treatment.

55. Which one of the following is least likely to cause malabsorption?


A. Primary biliary cirrhosis
B. Ileo-colic bypass
C. Chronic pancreatitis
D. Whipples disease
E. Hartmans procedure
Answer: E
In a Hartmans procedure the sigmoid colon is removed and an end colostomy is fashioned. The bowel remains in continuity and
no absorptive ability is lost.
An ileo-colic bypass leaves a redundant loop of small bowel in continuity, where the contents will stagnate and bacterial
overgrowth will occur. Therefore this is recognised cause of malabsorption.

Malabsorption: is characterised by diarrhoea, steatorrhoea and weight loss. Causes may be broadly divided into intestinal (e.g.
villous atrophy), pancreatic (deficiency of pancreatic enzyme production or secretion) and biliary (deficiency of bile-salts
needed for emulsification of fats)
Intestinal causes of malabsorption: coeliac disease. Crohn's disease. tropical sprue. Whipple's disease. Giardiasis. brush border
enzyme deficiencies (e.g. lactase insufficiency)

Pancreatic causes of malabsorption: chronic pancreatitis. Cystic fibrosis. Pancreatic cancer.

Biliary causes of malabsorption: biliary obstruction. primary biliary cirrhosis.

Other causes: bacterial overgrowth (e.g. systemic sclerosis, diverticulae, blind loop). Short bowel syndrome. lymphoma

Theme: Intra abdominal malignancies

A. Metastatic adenocarcinoma of the pancreas


B. Metastatic appendiceal carcinoid
C. Metastatic colonic cancer
D. Pseudomyxoma peritonei
E. MALT lymphoma
F.Retroperitoneal liposarcoma
G. Retroperitoneal fibrosis

For the disease given please give the most likely primary disease process. Each option may be used once, more than once or not
at all.

56. A 32 year old man is admitted with a distended tense abdomen. He previously underwent a difficult appendicectomy 1
year previously and was discharged. At laparotomy the abdomen is filled with a gelatinous substance.
Pseudomyxoma peritonei. Pseudomyxoma is classically associated with mucin production and the appendix is the commonest
source.

57. A 62 year old man is admitted with dull lower back pain and abdominal discomfort. On examination he is hypertensive
and a lower abdominal fullness is elicited on examination. An abdominal ultrasound demonstrates hydronephrosis and
intravenous urography demonstrated medially displaced ureters. A CT scan shows a periaortic mass.
Retroperitoneal fibrosis. Retroperitoneal fibrosis is an uncommon condition and its aetiology is poorly understood. In a

302
significant proportion the ureters are displaced medially. In most retroperitoneal malignancies they are displaced laterally.
Hypertension is another common finding. A CT scan will often show a para-aortic mass

58. A 48 year old lady is admitted with abdominal distension. On examination she is cachectic and has ascites. Her CA19-9
returns highly elevated.
Metastatic adenocarcinoma of the pancreas. Although not specific CA 19-9 in the context of this history is highly suggestive of
pancreatic cancer over the other scenarios. Pseudomyxoma peritoneii: Curative treatment is peritonectomy (Sugarbaker
procedure) and heated intra peritoneal chemotherapy.

Pseudomyxoma Peritonei: Rare mucinous tumour. Most commonly arising from the appendix (other abdominal viscera are
also recognised as primary sites). Incidence of 1-2/1,000,000 per year. The disease is characterised by the accumulation of large
amounts of mucinous material in the abdominal cavity
Treatment: Is usually surgical and consists of cytoreductive surgery (and often peritonectomy c.f Sugarbaker procedure)
combined with intra peritoneal chemotherapy with mitomycin C. Survival is related to the quality of primary treatment and in
Sugarbakers own centre 5 year survival rates of 75% have been quoted. Patients with disseminated intraperitoneal malignancy
from another source fare far worse. In selected patients a second look laparotomy is advocated and some practice this routinely.

Theme: Abdominal pain

A. Acute mesenteric embolus


B. Acute on chronic mesenteric ischaemia
C. Mesenteric vein thrombosis
D. Ruptured abdominal aortic aneurysm
E. Pancreatitis
F. Appendicitis
G. Acute cholecystitis

Please select the most likely underlying diagnosis from the list above. Each option may be used once, more than once or not at
all.

59. A 72 year old man collapses with sudden onset abdominal pain. He has been suffering from back pain recently and has
been taking ibuprofen.
Ruptured abdominal aortic aneurysm. Back pain is a common feature with expanding aneurysms and may be miss classified as
being of musculoskeletal origin.

60. A 73 year old women collapses with sudden onset of abdominal pain and the passes a large amount of diarrhoea. On
admission she is vomiting repeatedly. She has recently been discharged from hospital following a myocardial infarct but
recovered well.
The correct answer is Acute mesenteric embolusSudden onset of abdominal pain and forceful bowel evacuation are features of
acute mesenteric infarct.

61. A 66 year old man has been suffering from weight loss and develops severe abdominal pain. He is admitted to hospital
and undergoes a laparotomy. At operation the entire small bowel is infarcted and only the left colon is viable.
The correct answer is Acute on chronic mesenteric ischaemia. This man is likely to have underlying chronic mesenteric vascular
disease. Only 15% of emboli will occlude SMA orifice leading to entire small bowel infarct. The background history of weight
loss also favours an acute on chronic event.

Theme: Surgical incisions

A. Lanz incision
B. Gridiron incision
C. Kochers incision
D. Rutherford Morrison
E. Rooftop incision
303
F. McEvedy Incision
G. Lothissen Incision

Please select the most appropriate incision for the procedure described. Each option may be used once, more than once or not at
all.

62. A 78 year old lady is admitted with an incarcerated femoral hernia. Abdominal signs are absent and there are no
symptoms of obstruction. AXR is normal.
McEvedy Incision: From the list the McEvedy approach is the most appropriate. The Lothissen incision may compromise the
posterior wall of the inguinal canal and is best avoided. The author prefers a limited pfannenstial type incision for this
procedure, as it gives better control of the hernia, but this is not on the list.

63. A 15 year old girl presents with right iliac fossa pain and guarding, pregnancy test is negative and WCC is 16.
Lanz incision: She requires an appendicectomy although there is an increasing vogue for performing this procedure
laparoscopically an open procedure is entirely suitable. However, although both a Gridiron and Lanz incision are suitable for
appendicectomy a Lanz will give a superior cosmetic result and would be the preferred option for most young females.

64. A 45 year old man is due to undergo a live donor renal transplant. This will be his first procedure.
Rutherford Morrison: The Rutherford Morrison incision will typically give access to the iliac vessels and bladder for the
procedure

Theme: Acute abdominal pain

A. Appendicitis
B. Henoch Schonlein purpura
C. Diabetes mellitus
D. Intussusception
E. Mittelschmerz
F.Pneumonia
G. Sickle cell crisis
H. Spontaneous bacterial peritonitis
I. Rupure of follicular cyst

Please select the most likely cause of abdominal pain for the scenario given. Each option may be used once, more than once or
not at all.

65. An 11 month-old girl develops sudden onset abdominal pain. She has a high pitched scream and draws up her legs. Her
BP is 90/40 mm/Hg, her pulse 118/min and abdominal examination is normal.
Intussusception: Intussusception should be considered in toddlers and infants presenting with screaming attacks. The child often
has a history of being unwell for one to three days prior to presentation. The child may pass bloody mucus stool, which is a late
sign. Examination of the abdomen is often normal as the sausage mass in the right upper quadrant is difficult to feel.

66. An 8 year-old West Indian boy presents with periumbilical abdominal pain. He has vomited twice and is refusing fluids.
His temperature is 38.1oC and blood tests are as follows: Haemoglobin 8 g/dl, WCC 13 x 109/l, with a neutrophilia.
Sickle cell crisis: Sickle cell anaemia is characterised by severe chronic haemolytic anaemia resulting from poorly formed
erythrocytes. Painful crises result from vaso-occlusive episodes, which may occur spontaneously or may be precipitated by
infection. Consider this diagnosis in all children of appropriate ethnic background.

67. A 15-month-old girl presents with a three day history of periorbital oedema. She is brought to hospital. On examination
she has facial oedema and a tender distended abdomen. Her temperature is 39oC and her blood pressure is 90/45 mmHg.
There is clinical evidence of poor peripheral perfusion.
Spontaneous bacterial peritonitis: The 15-month-old girl is a patient with nephrotic syndrome. Patients with this condition are at
risk of septicaemia and peritonitis from Streptococcus pneumoniae, due to the loss of immunoglobulins and opsonins in the
urine.

(Question was not found)

A. Peritoneal lavage with cetrimide following elective right hemicolectomy


B. Use of a laparoscopic approach over open surgery

304
C. Use of talc to coat surgical gloves
D. Performing a Nobles plication of the small bowel
E. Using stapled rather than a hand sewn anastomosis
Answer: B
Laparoscopy results in fewer adhesions. When talc was used to coat surgical gloves it was a major cause of adhesion formation
and withdrawn for that reason. A Nobles plication is an old fashioned operation which has no place in the prevention of
adhesion formation. Use of an anastamotic stapling device will not influence the development of adhesions per se although
clearly an anastamotic leak will result in more adhesion formation

Surgical complications: Complications occur in all branches of surgery and require vigilance in their detection. In many cases
anticipating the likely complications and appropriate avoidance will minimise their occurrence. For the purposes of the MRCS
the important principles to appreciate are:The anatomical principles that underpin complications. The physiological and
biochemical derangements that occur. The most appropriate diagnostic modalities to utilize. The principles which underpin their
management
This is clearly a very broad area and impossible to cover comprehensively. There is considerable overlap with other topic areas
within the website.

Avoiding complications: Some points to hopefully avert complications: World Health Organisation checklist- now mandatory
prior to all operations. Prophylactic antibiotics - right dose, right drug, right time. Assess DVT/ PE risk and ensure adequate
prophylaxis. MARK site of surgery. Use tourniquets with caution and with respect for underlying structures. Remember the
danger of end arteries and in situations where they occur avoid using adrenaline containing solutions and monopolar diathermy.
Handle tissues with care- devitalised tissue serves as a nidus for infection. Be very wary of the potential for coupling injuries
when using diathermy during laparoscopic surgery. The inferior epigastric artery is a favourite target for laparoscopic ports and
surgical drains!

Anatomical principles: Understanding the anatomy of a surgical field will allow appreciation of local and systemic
complications that may occur. For example nerve injuries may occur following surgery in specific regions the table below lists
some of the more important nerves to consider and mechanisms of injury

Nerve Mechanism
Accessory Posterior triangle lymph node biopsy
Sciatic Posterior approach to hip
Common peroneal Legs in Lloyd Davies position
Long thoracic Axillary node clearance
Pelvic autonomic nerves Pelvic cancer surgery
Recurrent laryngeal nerves During thyroid surgery
Hypoglossal nerve During carotid endarterectomy
Ulnar and median nerves During upper limb fracture repairs

These are just a few. The detailed functional sequelae are particularly important and will often be tested. In addition to nerve
injuries certain procedures carry risks of visceral or structural injury. Again some particular favourites are given below:

Structure Mechanism
Thoracic duct During thoracic surgery e.g. Pneumonectomy, oesphagectomy
Parathyroid glands During difficult thyroid surgery
Ureters During colonic resections/ gynaecological surgery
Bowel perforation Use of Verres Needle to establish pneumoperitoneum
Bile duct injury Failure to delineate Calots triangle carefully and careless use of diathermy
Facial nerve Always at risk during Parotidectomy
Tail of pancreas When ligating splenic hilum
Testicular vessels During re-do open hernia surgery
Hepatic veins During liver mobilisation

Again many could be predicted from the anatomy of the procedure.

Physiological derangements: A very common complication is bleeding and this is covered under the section of haemorrhagic
shock. Another variant is infection either superficial or deep seated. The organisms are covered under microbiology and the
features of sepsis covered under shock. Do not forget that immunocompromised and elderly patients may present will atypical
physiological parameters.

Selected physiological and biochemical issues are given below:

Complication Physiological/ Biochemical Problem


305
Arrhythmias following cardiac Susceptibility to hypokalaemia (K+ <4.0 in cardiac patients)
surgery
Neurosurgical electrolyte SIADH following cranial surgery causing hyponatraemia
disturbance
Ileus following gastrointestinal Fluid sequestration and loss of electrolytes
surgery
Pulmonary oedema following Loss of lung volume makes these patients very sensitive to fluid overload
pneumonectomy
Anastamotic leak Generalised sepsis causing mediastinitis or peritonitis depending on site of leak
Myocardial infarct May follow any type of surgery and in addition to direct cardiac effects the decreased
cardiac output may well compromise grafts etc.

Try making a short list of problems and causes specific to your own clinical area.

Diagnostic modalities: Depends largely on the suspected complication. In the acutely unwell surgical patient the following
baseline investigations are often helpful: Full blood count, urea and electrolytes, C- reactive protein (trend rather than absolute
value), serum calcium, liver function tests, clotting (don't forget to repeat if on-going bleeding). Arterial blood gases. ECG
(+cardiac enzymes if MI suspected). Chest x-ray to identify collapse/ consolidation. Urine analysis for UTI. These will often
identify the most common complications.

Special tests: CT scanning for identification of intra-abdominal abscesses, air and if luminal contrast is used an anastamotic
leak.. Gatrograffin enema- for rectal anastamotic leaks. Doppler USS of leg veins- for identification of DVT. CTPA for PE.
Sending peritoneal fluid for U+E (if ureteric injury suspected) or amylase (if pancreatic injury suspected). Echocardiogram if
pericardial effusion suspected post cardiac surgery and no pleural window made.

Management of complications: The guiding principal should be safe and timely intervention. Patients should be stabilised and
if an operation needs to occur in tandem with resuscitation then generally this should be of a damage limitation type procedure
rather than definitive surgery (which can be more safely undertaken in a stable patient the following day). Remember that
recent surgery is a contra indication to thrombolysis and that in some patients IV heparin may be preferable to a low molecular
weight heparin (easier to reverse). As a general rule laparotomies for bleeding should follow the core principle of quadrant
packing and then subsequent pack removal rather than plunging large clamps into pools of blood. The latter approach invariable
worsens the situation is often accompanied by significant visceral injury particularly when done by the inexperienced. If
packing controls a situation it is entirely acceptable practice to leak packs in situ and return the patient to ITU for pack removal
the subsequent day.

Theme: Abdominal pain

A. Appendicitis
B. Threatened miscarriage
C. Ectopic pregnancy
D. Irritable bowel syndrome
E. Mittelschmerz
F. Pelvic inflammatory disease
G. Adnexial torsion
H. Endometriosis
I. Degenerating fibroid

Please select the most likely cause of abdominal pain for the clinical scenario given. Each option may be used once, more than
once or not at all.

68. An 18 year-old girl presents to the Emergency Department with sudden onset sharp, tearing pelvic pain associated with
a small amount of vaginal bleeding. She also complains of shoulder tip pain. On examination she is hypotensive,
tachycardic and has marked cervical excitation.
Ectopic pregnancy: The history of tearing pain and haemodynamic compromise in a women of child bearing years should
prompt a diagnosis of ectopic pregnancy.

69. A 25 year-old lady presents to her GP complaining of a two day history of right upper quadrant pain, fever and a white
vaginal discharge. She has seen the GP twice in 12 weeks complaining of pelvic pain and dyspareunia.
Pelvic inflammatory disease: The most likely diagnosis is pelvic inflammatory disease. Right upper quadrant pain occurs as part
of the Fitz Hugh Curtis syndrome in which peri hepatic inflammation occurs.

306
70. A 16 year old female presents to the emergency department with a 12 hour history of pelvic discomfort. She is otherwise
well and her last normal menstrual period was 2 weeks ago. On examination she has a soft abdomen with some mild
supra Pubic discomfort: Mittelschmerz: Mid cycle pain is very common and is due to the small amount of fluid released
during ovulation. Inflammatory markers are usually normal and the pain typically subsides over the next 24-48 hours.

Gynaecological causes of abdominal pain: A number of women will present with abdominal pain and subsequently be
diagnosed with a gynaecological disorder. In addition to routine diagnostic work up of abdominal pain, all female patients
should also undergo a bimanual vaginal examination, urine pregnancy test and consideration given to abdominal and pelvic
ultrasound scanning.
When diagnostic doubt persists a laparoscopy provides a reliable method of assessing suspected tubulo-ovarian pathology.

Differential diagnoses of abdominal pain in females

Mittelschmerz: Usually mid cycle pain. Often sharp onset. Little systemic disturbance.May have recurrent episodes. Usually
settles over 24-48 hours. Investigation: Full blood count- usually normal. Ultrasound- may show small quantity of free fluid
Treatment: Conservative

Endometriosis: 25% asymptomatic, in a further 25% associated with other pelvic organ pathology. Remaining 50% may have
menstrual irregularity, infertility, pain and deep dyspareurina. Complex disease may result in pelvic adhesional formation with
episodes of intermittent small bowel obstruction. Intra-abdominal bleeding may produce localised peritoneal inflammation.
Recurrent episodes are common. Investigation: Ultrasound- may show free fluid Laparoscopy will usually show lesions.
Treatment: Usually managed medically, complex disease will often require surgery and some patients will even require formal
colonic and rectal resections if these areas are involved

Ovarian torsion: Usually sudden onset of deep seated colicky abdominal pain. Associated with vomiting and distress.
Vaginal examination may reveal adnexial tenderness. Investigaion: Ultrasound may show free fluid. Laparoscopy is usually
both diagnostic and therapeutic. Treatment: Laparoscopy.

Ectopic gestation: Symptoms of pregnancy without evidence of intra uterine gestation. Present as an emergency with evidence
of rupture or impending rupture. Open tubular ruptures may have sudden onset of abdominal pain and circulatory collapse, in
other the symptoms may be more prolonged and less marked. Small amount of vaginal discharge is common. There is usually
adnexial tenderness. Investigation: Ultrasound showing no intra uterine pregnancy and beta HCG that is elevated
May show intra abdominal free fluid: Laparoscopy or laparotomy is haemodynamically unstable. A salphingectomy is usually
performed.

Pelvic inflammatory disease: Bilateral lower abdominal pain associated with vaginal discharge. Dysuria may also be present.
Peri-hepatic inflammation secondary to Chlamydia (Fitz Hugh Curtis Syndrome) may produce right upper quadrant discomfort.
Fever >38. Full blood count- Leucocytosis. Pregnancy test negative (Although infection and pregnancy may co-exist). Amylase
- usually normal or slightly raised. High vaginal and urethral swabs. Treatment: Usually medical management

71. Which of the following statements relating to a burst abdomen is false?


A. Is seen in 1-2% of modern laparotomies
B. Is more common in faecal peritonitis
C. Is less common when a 'mass closure' technique is used
D. When it does occur is most common at 15 days
E. Is similar in incidence regardless of whether 1/0 polydiaxone or 1/0 polypropylene are used
Answer: D

When it does occur a burst abdomen is most common at 6 days and is usually the result of technical error when Jenkins rule is
not followed and sutures are placed in the zone of collagenolysis. The choice of materials given above does not influence
dehisence rates.

72. Which of the following statements about diarrhoea is false?


A. Nocturnal diarrhoea is uncommon in irritable bowel syndrome
B. World Health Organisation definition of diarrhoea is greater than 3 episodes of loose or watery stool a day
C. Pancreatic disease causes osmotic diarrhoea
D. Vitamin C deficiency causes diarrhoea
E. The World Health Organisation definition of chronic diarrhoea is greater than 14 days of diarrhea
Answer: D
Vitamin C toxicity causes osmotic diarrhoea.

307
73. A 40 year old man presents with a long standing inguinal hernia. On examination he has a small, direct inguinal hernia.
He inquires as to the risk of strangulation over the next twelve months should he decide not to undergo surgery. Which
of the following most closely matches the likely risk of strangulation over the next 12 months?
A. 50%
B. 40%
C. 25%
D. 15%
E. <5%
Answer: E
The annual probability of strangulation is up to 3% and is more common in indirect hernias. Elective repair poses few risks.
However, emergency repair is associated with increased mortality, particularly in the elderly.

74. A 60 year old women has fully recovered from an attack of pancreatitis. Over the following 12 months she develops
episodic epigastric discomfort. Un upper GI endoscopy shows gastric varices only. An abdominal CT scan demonstrates
a splenic vein thrombosis. What is the treatment of choice?
A. Splenectomy
B. Insertion of transjugular porto-systemic shunt
C. Surgical bypass of the splenic vein
D. Gastrectomy
E. Stapling of the gastro-oesophgeal junction
Answer: A
Splenic vein thrombosis: Thrombosis of the splenic vein may complicate pancreatitis, pancreatic carcinoma, iatrogenic trauma
and hypercoagulable diseases. The condition may predispose to the development of gastric varices, oesophageal varices are
uncommon in splenic vein thrombosis alone. Diagnosis is made by CT angiography. Treatment is with splenectomy.

Theme: Surgical signs

A. Rovsing's sign
B. Boas' sign
C. Psoas stretch sign
D. Cullen's sign
E. Grey-Turner's sign
F. Murphy's sign
G. None of the above

Please match the clinical sign to the clinical scenario described. Each option may be used once, more than once or not at all.

75. Acute retrocaecal appendicitis is indicated when the right thigh is passively extended with the patient lying on their side
with their knees extended.
Psoas stretch sign

76. In acute pancreatitis there is bruising in the flanks.


Grey-Turner's sign: Grey-Turners sign occurs in patients with severe haemorrhagic pancreatitis. In this situation the major
vessels surrounding the pancreas bleed. The pancreatitis process also results in local fat destruction, this results in blood
tracking in the tissue planes of the retroperitoneum and appearing as flank bruising.

77. In cholecystitis there is pain/catch of breath elicited on palpation of the right hypochondrium during inspiration.
Murphy's sign: Invariably present when patients are assessed in the emergency department! This sign occurs because the
inflamed gallbladder irritates the parietal peritoneum in this manoeuvre.

78. Which of the following is commonest cause of acute abdominal pain in acute unselected surgical 'take'?
A. Non specific abdominal pain
B. Biliary colic
C. Acute appendicitis
D. Ureteric colic
E. Pancreatitis
308
Answer: A
Non specific abdominal pain is a commonly recorded diagnosis for patients presenting with acute abdominal pain. Following
careful diagnostic work up a proportion of patients may be identified with disorders such as coeliac disease and the diagnosis of
non specific abdominal pain should be used lightly.

Acute abdominal pain: this is a common cause of admission to hospital. The relative proportions of conditions presenting with
abdominal pain is given below:

 Non specific abdominal pain (35%)


 Appendicitis (17%)
 Intestinal obstruction (15%)
 Urological disease (6%)
 Gallstone disease (5%)
 Colonic diverticular disease (4%)
 Abdominal trauma (3%)
 Perforated peptic ulcer (3%)
 Pancreatitis (2%)

Non specific abdominal pain should really be a diagnosis of exclusion and if care is taken in excluding organic disease the
proportion of cases labeled such should decline. It should also be appreciated that a proportion of patients may have an
underlying medical cause for their symptoms such as pneumonia or diabetic ketoacidosis.

Key points in management: Early administration of adequate analgesia (including opiates). Abdominal ultrasound is safe, non
invasive and cheap and yields significantly more information than plain radiology. However, plain radiology is still the main
test for suspected perforated viscus, especially out of hours. In up to 50% cases with perforated peptic ulcer, the plain x-rays
may show no evidence of free air. If clinical signs suggest otherwise, then a CT scan may be a more accurate investigation, if
plain films are normal. Plain film radiology usually cannot detect <1mm free air, and is 33% sensitive for detection of 1-13mm
pockets of free air. Think of strangulated intestine when there is fever, raised white cell count, tachycardia and peritonism. In
suspected large bowel obstruction a key investigation is either a water soluble contrast enema or CT scan. Where need for
surgery is difficult to define and imaging is inconclusive the use of laparoscopy as a definitive diagnostic test is both safe and
sensible.

79. A 72 year old obese man undergoes and emergency repair of a ruptured abdominal aortic aneurysm. The wound is
closed with an onlay prolene mesh to augment the closure. Post operatively he is taken to the intensive care unit. Over
the following twenty four hours his nasogastric aspirates increase, his urine output falls and he has a metabolic acidosis.
What is the most likely underlying cause?
A. Colonic ischaemia
B. Abdominal compartment syndrome
C. Peritonitis
D. Reactionary haemorrhage
E. Aorto-duodenal fistula
Answer: B
Obese patients with ileus following major abdominal surgery are at increased risk of intra abdominal compartment
syndrome.The risk is increased by the use of prosthetic meshes, which some surgeons favor following a major vascular case as
they may reduce the incidence of incisional hernia. They prevent abdominal distension and may increase the risk of intra
abdominal hypertension in the short term. Although colonic ischaemia may occur following major aortic surgery it would not
typically present in this way.

Abdominal compartment syndrome

Background: Intra-abdominal pressure is the steady state pressure concealed within the abdominal cavity. In critically ill adults
the normal intra abdominal pressure = 5-7mmHg. Intra abdominal hypertension has pressures of 12-25mmHg. Changes
>15mmHg are associated with microvascular hypoperfusion. Abdominal compartment syndrome is defined as sustained intra
abdominal pressure >20mmHg coupled with new organ dysfunction / failure. It may occur either primarily without
previous surgical intervention e.g. Following intestinal ischaemia or secondarily following a surgical procedure. Diagnosis is
typically made by transvesical pressure measurements coupled with an index of clinical suspicion.

Management: Once the diagnosis is made non operative measures should be instituted including: Gastric decompression.
Improve abdominal wall compliance e.g. muscle relaxants/ sedation. Drain abdominal fluid collections. Consider fluid
restriction/ diuretics if clinically indicated. In those whom non operative treatment is failing; the correct treatment is laparotomy

309
and laparostomy. Options for laparostomy are many although the Bogota bag or VAC techniques are the most widely practised.
Re-look laparotomy and attempts at delayed closure will follow in due course.

Theme: Surgical drains

A. Redivac suction drain


B. Corrugated drain
C. Wallace Robinson drain
D. Penrose tubing
E. Latex T Tube drain
F. Silastic T Tube drain

Please select the most appropriate surgical drainage system for the indication given. Each option may be used once, more than
once or not at all.

80. A 56 year old lady undergoes and open cholecystectomy and exploration of common bile duct. The bile duct is closed
over a drain.
Latex T Tube drain: Latex is used for this indication as it will encourage track formation.

81. A 48 year old lady undergoes a mastectomy and axillary node clearance for an invasive ductal cancer of the breast with
lymph node metastasis.
Redivac suction drain: Suction drains are commonly used following mastectomy and axillary surgery to prevent haematoma
formation. Not all surgeons routinely drain the axilla.

82. A 75 year old man undergoes a hartmans procedure for sigmoid diverticular disease with pericolic abscess and
colovesical fistula.
Wallace Robinson drain: These tube drains are often used in abdominal surgery to drain abscess cavities. Debate might occur
around the use of low pressure vs no suction in this setting so this option is deliberately omitted.

Surgical drains
Drains are inserted in many surgical procedures and are of many types. As a broad rule they can be divided into those using
suction and those which do not. The diameter of the drain will depend upon the substance being drained, for example smaller
lumen drain for pneumothoraces vs haemothorax. Drains can be associated with complications and these begin with insertion
when there may be iatrogenic damage. When in situ they serve as a route for infections. In some specific situations they may
cause other complications, for example suction drains left in contact with bowel for long periods may carry a risk of inducing
fistulation.Drains should be inserted for a defined purpose and removed once the need has passed.

A brief overview of types of drain and sites is given below

CNS: Low suction drain or free drainage systems may be used for situations such as drainage of sub dural haematomas.

CVS: Following cardiothoracic procedures of thoracic trauma underwater seal drains are often placed. These should be
carefully secured. When an air leak is present they may be placed on suction whilst the air leak settles

Orthopaedics and trauma: In this setting drains are usually used to prevent haematoma formation (with associated risk of
infection). Some orthopaedic drains may also be specially adapted to allow the drained blood to be auto transfused.

Gastro-intestinal surgery: Surgeons often place abdominal drains either to prevent or drain abscesses, or to turn an anticipated
complication into one that can be easily controlled such as a bile leak following cholecystectomy. The type of drain used will
depend upon the indication.

Drain types
Redivac: Suction type of drain. Closed drainage system. High pressure vacuum system

Low pressure drainage systems: Consist of small systems such as the lantern style drain that may be used for short term
drainage of small wounds and cavities. Larger systems are sometimes used following abdominal surgery, they have a lower
pressure than the redivac system, which decreases the risks of fistulation. May be emptied and re-pressurised

310
Latex tube drains: May be shaped (e.g. T Tube) or straight. Usually used in non pressurised systems and act as sump drains.
Most often used when it is desirable to generate fibrosis along the drain trach (e.g. following exploration of the CBD)

Chest drains: May be large or small diameter (depending on the indication). Connected to underwater seal system to ensure
one way flow of air

Corrugated drain: Thin, wide sheet of plastic, usually soft. Contains corrugations, along which fluids can track

Theme: Right iliac fossa pathology

A. Appendicitis
B. Mesenteric adenitis
C. Inflammatory bowel disease
D. Irritable bowel syndrome
E. Mesenteric cyst
F. Campylobacter infection
G. Appendix abscess

Please select the most likely diagnosis for the scenario given. Each option may be used once, more than once or not at all.

83. An 8 year old boy is examined by his doctor as part of a routine clinical examination. The doctor notices a smooth
swelling in the right iliac fossa. It is mobile and the patient is otherwise well.
Mesenteric cyst: Mesenteric cysts are often smooth. Imaging with ultrasound and CT is usually sufficient. Although rare, they
most often occur in young children (up to 30% present before the age of 15). Many are asymptomatic and discovered
incidentally. Acute presentations are recognised and may occur following cyst torsion, infarction or rupture. Most cysts will be
surgically resected. Theme from April 2012 Exam

84. An 8 year old boy presents with abdominal pain,a twelve hour history of vomiting, a fever of 38.3 oC and four day
history of diarrhoea. His abdominal pain has been present for the past week.
Appendix abscess: The high fever and diarrhoea together with vomiting all point to a pelvic abscess. The presence of pelvic pus
is highly irritant to the rectum, and many patients in this situation will complain of diarrhoea.

85. A 7 year old boy presents with a three day history of right iliac fossa pain and fever. On examination he has a
temperature of 39.9o C. His abdomen is soft and mildly tender in the right iliac fossa.
Mesenteric adenitis: High fever and mild abdominal signs in a younger child should raise suspicion for mesenteric adenitis. The
condition may mimic appendicitis and many may require surgery.

86. A 56 year old man undergoes a difficult splenectomy and is left with a pancreatic fistula. There are ongoing problems
with very high fistula output. Which of the following agents may be administered to reduce the fistula output?
A. Metoclopramide
B. Erthyromycin
C. Octreotide
D. Loperamide
E. Omeprazole
Answer: C
Octreotide is a useful agent in reducing the output from pancreatic fistulae. Prokinetic agents will increase fistula output and
should be avoided.

Fistulas: A fistula is defined as an abnormal connection between two epithelial surfaces. There are many types ranging from
Branchial fistulae in the neck to entero-cutaneous fistulae abdominally. In general surgical practice the abdominal cavity
generates the majority and most of these arise from diverticular disease and Crohn's. As a general rule all fistulae will resolve
spontaneously as long as there is no distal obstruction. This is particularly true of intestinal fistulae.
The four types of fistulae are:

Enterocutaneous: These link the intestine to the skin. They may be high (>1L) or low output (<1L) depending upon source.
Duodenal /jejunal fistulae will tend to produce high volume, electrolyte rich secretions which can lead to severe excoriation of
the skin. Colo-cutaneous fistulae will tend to leak faeculent material. Both fistulae may result from the spontaneous rupture of
an abscess cavity onto the skin (such as following perianal abscess drainage) or may occur as a result of iatrogenic input. In
some cases it may even be surgically desirable e.g. mucous fistula following sub total colectomy for colitis. Suspect if there is
excess fluid in the drain.

311
Enteroenteric or Enterocolic: This is a fistula that involves the large or small intestine. They may originate in a similar manner
to enterocutaneous fistulae. A particular problem with this fistula type is that bacterial overgrowth may precipitate
malabsorption syndromes. This may be particularly serious in inflammatory bowel disease.

Enterovaginal: Aetiology as above.

Enterovesicular: This type of fistula goes to the bladder. These fistulas may result in frequent urinary tract infections, or the
passage of gas from the urethra during urination.

Management: Some rules relating to fistula management: They will heal provided there is no underlying inflammatory bowel
disease and no distal obstruction, so conservative measures may be the best option. Where there is skin involvement, protect the
overlying skin, often using a well fitted stoma bag- skin damage is difficult to treat. A high output fistula may be rendered more
easily managed by the use of octreotide, this will tend to reduce the volume of pancreatic secretions. Nutritional complications
are common especially with high fistula (e.g. high jejunal or duodenal) these may necessitate the use of TPN to provide
nutritional support together with the concomitant use of octreotide to reduce volume and protect skin. When managing perianal
fistulae surgeons should avoid probing the fistula where acute inflammation is present, this almost always worsens outcomes.
When perianal fistulae occur secondary to Crohn's disease the best management option is often to drain acute sepsis and
maintain that drainage through the judicious use of setons whilst medical management is implemented. Always attempt to
delineate the fistula anatomy, for abscesses and fistulae that have an intra abdominal source the use of barium and CT studies
should show a track. For perianal fistulae surgeons should recall Goodsall's rule in relation to internal and external openings.

Theme: Inguinal hernia management

A. Bassini repair
B. Inguinal herniotomy
C. Lichtenstein repair
D. Laparoscopic hernia repair
E. Shouldice repair
F. McVey repair

For the herniae described please select the most appropriate procedure from the list. Each option may be used once, more than
once or not at all.

87. A 11 month old child presents with intermittent groin swelling, it has a cough impulse and is easily reducible.
Inguinal herniotomy: Infants usually suffer from a patent processus vaginalis (a congential problem). As a result a simple
herniotomy is all that is required. A mesh is not required as there is not specific muscle weakness.

88. A 25 year old builder presents with a reducible swelling in the right groin, it is becoming larger and has not been
operated on previously.
Lichtenstein repair: An open Lichtenstein repair using mesh is appropriate. There is a 0.77% recurrence rate with this technique.
A Shouldice repair is an acceptable alternative if the surgeon is experienced

89. A 28 year old man presents with a recurrent inguinal hernia on the left side of his abdomen and a newly diagnosed
inguinal hernia on the right side.
Laparoscopic hernia repairLaparoscopic hernia repairs are specifically indicated where there are bilateral hernias or recurrence
of a previous open repair.

Theme: Surgical access

A. Kocher's
B. Lanz
C. Rooftop
D. Pfannenstiel's
E. Midline

312
F. Paramedian incision
G. Mcevedy

Please select the most appropriate incision for the procedure described. Each option may be used once, more than once or not at
all.

90. A 19 year old girl who is 39 weeks pregnant goes into labour. The labour is prolonged and she is found to have an
undiagnosed breech baby.
Pfannenstiel's: This patient needs an emergency cesarean section.

91. A 49 year old woman presents with jaundice and abdominal pain. She is haemodynamically unstable. An USS shows a
dilated common bile duct and gallstones in the gallbladder.
Kocher's: This lady needs a cholecystectomy and bile duct exploration.

92. A 42 year old man with history of alcohol abuse is diagnosed with pancreatic cancer and requires a Whipples resection.
Rooftop:A pancreatectomy is usually performed through a roof top incision. This provides excellent access to the upper
abdomen.

Upper GI Surgery
1. A 55 year old man presents with symptoms of dyspepsia and on upper GI endoscopy an area of patchy erythematous
tissue is identified protruding proximally from the gastro oesophageal junction. Biopsies are diagnostic of Barretts
oesophagus with low grade dysplasia. Which of the following is the most appropriate management?
A. Distal oesophagectomy
B. Upper GI endoscopy with quadrantic biopsies from the region
C. Photodynamic therapy
D. Endoscopic sub mucosal resection of the area
E. Argon plasma coagulation
Answer: B
Low grade dysplasia in conjunction with Barretts oesphagus should be monitored with regular (6 monthly) upper GI endoscopy
and quadrantic biopsies. If the disease remains static at 2 years then the screening frequency may be decreased.

Barrett's oesophagus: Intestinal metaplasia. Squamous epithelium replaced by columnar epithelium in the lower oesophagus. 3
types of columnar epithelium: 1. Junctional. 2. Atrophic fundal. 3. Specialised. Presence of goblet cells important in
identification. Premalignant change (progress to dysplasia). Risk of adenocarcinoma. Risk factors: middle age, men, smoker,
Caucasian, gastro-oesophageal reflux, obesity

Treatment: Long term proton pump inhibitor. Consider pH and manometry studies in younger patients who may prefer to
consider an anti reflux procedure. Regular endoscopic monitoring (more frequently if moderate dysplasia). With quadrantic
biopsies every 2-3 cm. If severe dysplasia be very wary of small foci of cancer.

Theme: Causes of dysphagia

A. Adenocarcinoma of the oesophagus


B. Plummer Vinson syndrome
C. Squamous cell carcinoma of the oesophagus
D. Achalasia
E. Benign oesophageal stricture
F. Pharyngeal pouch
G. Tracheo-oesophageal fistula

Please select the most likely underlying cause for dysphagia from the scenario given. Each option may be used once, more than
once or not at all.

2. A 78 year old lady presents with episodic dysphagia and halitosis, occasionally she complains of regurgitation. A recent
attempted upper GI endoscopy was poorly tolerated and abandoned.

313
Pharyngeal pouch. This is a pulsion type diverticulum through Killians dehiscence. Patients often complain of regurgitation of
foul smelling liquid. Upper GI endoscopy is potentially hazardous and may result in iatrogenic perforation.

3. An overweight 56 year old man with longstanding Barrett's oesophagus complains of worsening dysphagia to solids over
the past 6 weeks.

Adenocarcinoma of the oesophagus. Barrett's metaplasia places patients at increased risk of adenocarcinoma of the oesophagus.
Small localised lesions and in situ disease has been treated by endoscopic mucosal resection and photodynamic therapy. In
many centres oesophagectomy is still offered as the safest and standard treatment. Where dysphagia is present a significant
lesion (requiring resection if operable) is invariably found.

4. A 24 year old man complains of occasional retrosternal chest pain and dysphagia that occurs to both liquids and solids.
He is otherwise well.

Achalasia. This is an atypical cause of these symptoms. Diagnosis is made by upper GI endoscopy, barium swallow and
manometry/ pH studies. Treatments range from botulinum toxin injection through to Hellers Cardiomyotomy that may be
performed laparoscopically.

Causes of dysphagia

 Extrinsic: Mediastinal masses. Cervical spondylosis


 Oesophageal wall: Achalasia. Diffuse oesophageal spasm. Hypertensive lower oesophageal sphincter
 Intrinsic: Tumours. Strictures. Oesophageal web. Schatzki rings
 Neurological: CVA. Parkinson's disease. Multiple Sclerosis. Brainstem pathology. Myasthenia Gravis

Investigation: All patients require an upper GI endoscopy unless there are compelling reasons for this not to be performed.
Motility disorders may be best appreciated by undertaking fluoroscopic swallowing studies. A full blood count should be
performed. Ambulatory oesophageal pH and manometry studies will be required to evaluate conditions such as achalasia and
patients with GORD being considered for fundoplication surgery.

5. A 63 year old man undergoes an upper GI endoscopy and adrenaline injection for a large actively bleeding duodenal
ulcer. He remains stable for 6 hours and the nurses then call because he has passed 400ml malaena and has become
tachycardic (pulse rate 120) and hypotensive (Bp 80/40). Which is the best option:
A. Reassure that blood trapped in the upper portion of the gastrointestinal system will pass and that this episode will resolve with
phosphate enema
B. Perform a repeat upper GI endoscopy
C. Perform a laparotomy and underrunning of the ulcer
D. Administer tranexamic acid and intravenous proton pump inhibitors
E. Insert a Minnesota tube
Answer: C
Whilst gastroenterologists might argue for repeat OGD in the face of impending haemodynamic instability and signs of ongoing
bleeding a laparotomy is safest. The OGD may be repeated in theatre prior to making skin incision if the surgeon wishes.

Upper gastrointestinal bleeding: Patients may present with the following: Haematemesis and/ or malaena. Epigastric discomfort.
Sudden collapse

The extent to which these will occur will depend upon the source. Mortality is higher in patients presenting with haematemesis
than malaena alone.

Oesophageal bleeding
Cause Presenting features
Oesophagitis Small volume of fresh blood, often streaking vomit. Malaena rare. Often ceases spontaneously. Usually
history of antecedent GORD type symptoms.
Cancer Usually small volume of blood, except as pre terminal event with erosion of major vessels. Often associated
symptoms of dysphagia and constitutional symptoms such as weight loss. May be recurrent until malignancy
managed.
Mallory Weiss Typically brisk small to moderate volume of bright red blood following bout of repeated vomiting. Malaena
Tear rare. Usually ceases spontaneously.
Varices Usually large volume of fresh blood. Swallowed blood may cause malaena. Often associated with
314
haemodynamic compromise. May stop spontaneously but re-bleeds are common until appropriately managed.

Gastric Bleeding
Cause Presenting features
Gastric cancer May be frank haematemesis or altered blood mixed with vomit. Usually prodromal features of dyspepsia and
may have constitutional symptoms. Amount of bleeding variable but erosion of major vessel may produce
considerable haemorrhage.
Dilafeuoy Often no prodromal features prior to haematemesis and malaena, but this arteriovenous malformation may
Lesion produce quite considerable haemorrhage and may be difficult to detect endoscopically
Diffuse erosive Usually haematemesis and epigastric discomfort. Usually there is an underlying cause such as recent NSAID
gastritis usage. Large volume haemorrhage may occur with considerable haemodynamic compromise
Gastric ulcer Small low volume bleeds more common so would tend to present as iron deficiency anaemia. Erosion into a
significant vessel may produce considerable haemorrhage and haematemesis.

Duodenum: Most common cause of major haemorrhage is a posteriorly sited duodenal ulcer. However, ulcers at any site in the
duodenum may present with haematemesis, malaena and epigastric discomfort. The pain of duodenal ulcer is slightly different to
that of gastric ulcers and often occurs several hours after eating. Peri ampullary tumours may bleed but these are rare. In patients
with previous abdominal aortic aneurysm surgery aorto-enteric fistulation remains a rare but important cause of major
haemorrhage associated with high mortality.

Management: Admission to hospital careful monitoring, cross match blood, check FBC, LFTs, U+E and Clotting (as a
minimum). Patients with on-going bleeding and haemodynamic instability are likely to require O negative blood pending cross
matched blood. Early control of airway is vital (e.g. Drowsy patient with liver failure). Patients with suspected varices should
receive terlipressin prior to endoscopy. Ideally all patients admitted with upper gastrointestinal haemorrhage should undergo
Upper GI endoscopy within 24 hours of admission. In those who are unstable this should occur immediately after resuscitation or
in tandem with it. The endoscopy department is a potentially dangerous place for unstable patients and it may be safer to perform
the endoscopy in theatre with an anaesthetist present. Varices should be banded or subjected to sclerotherapy. If this is not
possible owing to active bleeding then a Sengaksten- Blakemore tube (or Minnesota tube) should be inserted. This should be done
with care; gastric balloon should be inflated first and oesophageal balloon second. Remember the balloon with need deflating after
12 hours (ideally sooner) to prevent necrosis. Portal pressure should be lowered by combination of medical therapy +/- TIPSS.
Patients with erosive oesophagitis / gastritis should receive a proton pump inhibitor. Mallory Weiss tears will typically resolve
spontaneously. Identifiable bleeding points should receive combination therapy of injection of adrenaline and either a thermal or
mechanical treatment. All who have received intervention should receive a continuous infusion of a proton pump inhibitor (IV
omeprazole for 72 hours) to reduce the re-bleeding rate. Patients with diffuse erosive gastritis who cannot be managed
endoscopically and continue to bleed may require gastrectomy. Bleeding ulcers that cannot be controlled endoscopically may
require laparotomy and ulcer underruning

Indications for surgery: Patients > 60 years. Continued bleeding despite endoscopic intervention. Recurrent bleeding. Known
cardiovascular disease with poor response to hypotension

Surgery: Duodenal ulcer: Laparotomy, duodenotomy and under running of the ulcer. If bleeding is brisk then the ulcer is almost
always posteriorly sited and will have invaded the gastroduodenal artery. Large bites using 0 Vicryl are taken above and below the
ulcer base to occlude the vessel. The duodenotomy should be longitudinal but closed transversely to avoid stenosis.

For gastric ulcer: Partial gastrectomy-antral ulcer. Partial gastrectomy or under running the ulcer- lesser curve ulcer (involving left
gastric artery). Total gastrectomy if bleeding persists

Summary of Acute Upper GI bleeding recommendations: All patients should have a pre-endoscopic Rockall score. Consider
admission if: Aged over 60 years. Witnessed haematemesis. Systolic BP < 100mmHg or HR > 100 bpm. Liver disease/known
varices

Resuscitation: In shock, Give fluid. Transfuse if 30% circulating volume is lost. Administration of proton pump inhibitors prior to
endoscopy may reduce evidence of haemorrhage. Give IV PPI e.g. 80mg Omeprazole bolus then 8mg/h over 72h, if bleeding
ulcer identified. If post endoscopy Rockall score < 3 consider discharge and follow-up

Theme: Oesophageal disease

A. Oesophagectomy
B. Endoscopic sub mucosal dissection
C. Photodynamic therapy
D. Insertion of oesophageal stent
E. Chemotherapy
F. Radiotherapy

315
Please select the most appropriate treatment modality for the scenario given. Each option may be used once, more than once or not
at all.

6. A 52 year old man with long standing Barretts oesophagus is diagnosed with high grade dysplasia on recent endoscopy.
The lesions are multifocal and mainly distally sited.

The correct answer is Oesophagectomy. Some may argue for local therapy. However, in young patients who are otherwise fit,
multifocal disease such as this should probably be resected. Similar theme in 2011 exams

7. A 72 year old man presents with dysphagia. He is investigated and found to have an adenocarcinoma of the distal
oesophagus. His staging investigations have revealed a solitary metastatic lesion in the right lobe of his liver.

The correct answer is Insertion of oesophageal stent. Although he may be palliated with chemotherapy a stent will produce the
quickest clinical response. Metastatic disease is usually a contra indication to oesophageal resection. Similar theme in 2011
exam

8. An 83 year old lady with long standing Barretts oesophagus is diagnosed with a 1cm focus of high grade dysplasia 3cm
from the gastrooesophageal junction.

Endoscopic sub mucosal dissection. As she is elderly and the disease localised EMR is an appropriate first line step.
The technique involves raising the mucosa containing the lesion and then using an endoscopic snare to remove it. This
technique is therefore minimally invasive. However, it is only suitable for early superficial lesions. Deeper invasion would carry
a high risk of recurrence.

Treatment of oesophageal cancer: In general resections are not offered to those patients with distant metastasis, and usually not
to those with N2 disease. Local nodal involvement is not in itself a contra indication to resection. Surgical resection is the
mainstay of treatment. Neoadjuvent chemotherapy is given in most cases prior to surgery. In situ disease may be managed by
endoscopic mucosal resection, although this is still debated. In patients with lower third lesions an Ivor - Lewis type procedure is
most commonly performed. Very distal tumours may be suitable to a transhiatal procedure. Which is an attractive option as the
penetration of two visceral cavities required for an Ivor- Lewis type procedure increases the morbidity considerably. More
proximal lesions will require a total oesphagectomy (Mckeown type) with anastomosis to the cervical oesophagus. Patients with
unresectable disease may derive benefit from local ablative procedures, palliative chemotherapy or stent insertion.

Operative details of Ivor- Lewis procedure: Combined laparotomy and right thoracotomy

Indication: Lower and middle third oesophageal tumours

Preparation: Staging with a combination of CT chest abdomen and pelvis- if no metastatic disease detected then patients will
undergo a staging laparoscopy to detect peritoneal disease. If both these modalities are negative then patients will finally undergo
a PET CT scan to detect occult metastatic disease. Only in those whom no evidence of advanced disease is detected will proceed
to resection. Patients receive a GA, double lumen endotracheal tube to allow for lung deflation, CVP and arterial monitoring.

Procedure: A rooftop incision is made to access the stomach and duodenum.

Laparotomy To mobilize the stomach. The greater omentum is incised away from its attachment to the right gastroepiploic
vessels along the greater curvature of the stomach.Then the short gastric vessels are ligated and detached from the greater
curvature from the spleen.The lesser omentum is incised, preserving the right gastric artery. The retroperitoneal attachments of the
duodenum in its second and third portions are incised, allowing the pylorus to reach the oesophageal hiatus. Some surgeons
perform a pyloroplasty at this point to facilitate gastric emptying. The left gastric vessels are then ligated, avoiding any injury to
the common hepatic or splenic arteries. Care must be taken to avoid inadvertently devascularising the liver owing to variations in
anatomy.

Right Thoracotomy Oesophageal resection and oesophagogastric anastomosis. Through 5th intercostal space. Dissection
performed 10cm above the tumour. This may involve transection of the azygos vein.The oesophagus is then removed with the
stomach creating a gastric tube. An anastomosis is created.

The chest is closed with underwater seal drainage and tube drains to the abdominal cavity.

Post operatively: Patients will typically recover in ITU initially. A nasogastric tube will have been inserted intraoperatively and
must remain in place during the early phases of recovery.Post operatively these patients are at relatively high risk of developing
complications:

316
* Atelectasis- due to the effects of thoracotomy and lung collapse
* Anastomotic leakage. The risk is relatively high owing to the presence of a relatively devascularised stomach. Often the only
blood supply is from the gastroepiploic artery as all others will have been divided. If a leak does occur then many will attempt to
manage conservatively with prolonged nasogastric tube drainage and TPN. The reality is that up to 50% of patients developing an
anastomotic leak will not survive to discharge.
* Delayed gastric emptying (may be avoided by performing a pyloroplasty).

Theme: Pancreatic tumours

A. Lymphoma
B. Gastrinoma
C. Insulinoma
D. Glucagonoma
E. Phaeochromocytoma
F. Carcinoid syndrome
G. Vasoactive Intestinal Peptide secreting tumour
H. Pancreatic adenocarcinoma

Please select the most likely diagnosis for the scenario given. Each option may be used once, more than once or not at all.

9. A 65 year old male attends surgical out patients with epigastric discomfort. He has recently been diagnosed with diabetes
by the GP and is a heavy smoker. An OGD is normal.

The correct answer is Pancreatic adenocarcinoma. The dominant differential diagnosis should be of pancreatic adenocarcinoma
in this setting. Glucagonomas are very rare and may be associated with a bullous rash.

10. A 50 year old male presents with recurrent episodes of abdominal pain and diarrhoea. Blood tests reveal mild iron
deficiency anaemia and an upper GI endoscopy demonstrates multiple ulcers in the first part of the duodenum.

Gastrinoma. Diarrhoea, abdominal pain and multiple ulcers should raise the suspicion of Zollinger Ellison syndrome cause by
gastrinoma.

11. An obese 40 year old male presents with episodes of anxiety, confusion and one convulsive episode. CT brain is normal.
An abdominal CT scan shows a small 1.5cm lesion in the head of the pancreas.

Insulinoma. These episodes are due to hypoglycaemia. Insulinomas are normally solitary tumours and may not be seen by
radiological imaging. Resection is the treatment of choice.

Pancreatic cancer: Adenocarcinoma. Risk factors: Smoking, diabetes, Adenoma, Familial adenomatous polyposis. Mainly occur
in the head of the pancreas (70%). Spread locally and metastasizes to the liver. Carcinoma of the pancreas should be differentiated
from other periampullary tumours with better prognosis

Clinical features: Weight loss. Painless jaundice. Epigastric discomfort (pain usually due to invasion of the coeliac plexus is a
late feature). Pancreatitis. Trousseau's sign: migratory superficial thrombophlebitis

Investigations: USS: May miss small lesions. CT Scanning (pancreatic protocol). If unresectable on CT then no further staging
needed. PET/CT for those with operable disease on CT alone. ERCP/ MRI for bile duct assessment. Staging laparoscopy to
exclude peritoneal disease.

Management: Head of pancreas: Whipple's resection (SE dumping and ulcers). Newer techniques include pylorus preservation
and SMA/ SMV resection. Carcinoma body and tail: poor prognosis, distal pancreatectomy if operable. Usually adjuvent
chemotherapy for resectable disease. ERCP and stent for jaundice and palliation. Surgical bypass may be needed for duodenal
obstruction.

12. A 45 year old man has a 4 week history of epigastric discomfort which is relieved by eating. He develops haematemesis
and undergoes an upper GI endoscopy. An actively bleeding ulcer is noted in the first part of the duodenum. What is
the best management?
A. Whipples procedure
B. Truncal vagotomy and drainage
C. Distal gastrectomy
D. Injection with tranexamic acid
317
E. Injection with adrenaline
Answer: E
Bleeding duodenal ulcers will usually undergo adrenaline injection. This may be augmented by the placement of endoscopic clips
or heat therapy with endoscopic heater probes. Following these interventions patients should recieve a proton pump inhibitor
infusion. Those who rebleed, may require surgery. For ulcers in this location, laparotomy, duodenotomy and underrunning of the
ulcer is usually performed.

13. Which of the following patients should be referred for 1st line bariatric surgery?

A. BMI 35 kg/m2 and hypertension


B. BMI 40 kg/m2
C. BMI 40 kg/m2 and type 2 diabetes
D. BMI 50kg/m2
E. BMI 40kg/m2, COPD and type 2 diabetes
Answer: D
With all the other options the patient should have conservative management for a minimum of 6 months first.

Bariatric surgery: Obesity is a major health problem in the Western world. Surgical solutions to the problem have evolved
dramatically over the past few years. Randomised controlled trials have shown that dramatic weight loss can be achieved
following surgical interventions compared with standard medical therapy. The weight loss process is also more durable following
surgery than with non surgical interventions.

Case selection: BMI >/= 40 kg/m2 or between 35-40 kg/m2 and other significant disease (for example, type 2 diabetes,
hypertension) that could be improved with weight loss.

Pre-requisites to surgery (NICE UK Guidelines: All non-surgical measures have failed to achieve or maintain adequate
clinically beneficial weight loss for at least 6 months.Will receive intensive specialist management. They are generally fit for
anaesthesia and surgery. They commit to the need for long-term follow-up. First-line option for adults with a BMI > 50 kg/m2 in
whom surgical intervention is considered appropriate; consider orlistat if there is a long waiting list.

Surgical options: Adjustable gastric band: Laparoscopic placement of adjustable band around proximal stomach. Contains an
adjustable filling port. Effective method for lifestyle control. Reversible. Takes longer to achieve target weight. Complications
such as band erosion (rare), slippage or loss of efficacy may require re-intervention

Gastric bypass: Combines changes to reservoir size with malabsorptive procedure for more enduring weight loss. Technically
more challenging. Risks related to anastomoses (2% leak rate). Irreversible. Up to 50% may become B12 deficient

Sleeve gastrectomy: Resection of stomach using stapling devices. Less popular now as initial promising results not sustaine

Theme: Management of gastric cancer

A. Radical radiotherapy
B. Endoscopic submucosal resection
C. Polya Gastrectomy
D. Distal gastrectomy and anterior gastrojejunostomy
E. Distal gastrectomy and posterior gastrojejunostomy
F. Besley Mark IV procedure
G. Sub total gastrectomy and Roux and Y reconstruction
H. Total gastrectomy and Roux en Y reconstruction

Please select the most appropriate management for the gastric cancer case described. Each option may be used once, more than
once or not at all.

14. An otherwise fit 73 year old man presents with gastric outlet obstruction. An upper GI endoscopy shows a prepyloric
tumour occluding the pylorus. Staging investigations show nodal disease at D2 and an involved paraaortic lymph node.

Distal gastrectomy and anterior gastrojejunostomy. This man does not have disease amenable to curative surgical resection.
However, good palliation can be achieved with a resection and chemotherapy. He is likely to have recurrent disease in the
gastric bed and an anterior gastrojejunostomy is therefore preferred.

15. A 40 year old lady presents with a gastric carcinoma of the greater curvature of the stomach. Her staging investigations
are negative for metastatic disease.

318
Sub total gastrectomy and Roux and Y reconstruction. This is amenable to potentially curative resection. The proximal stomach
can be conserved.

16. A 62 year old man presents with dyspepsia and a tumour of the gastric cardia is diagnosed. He has no evidence of
metastatic disease.

The correct answer is Total gastrectomy and Roux en Y reconstruction. This will require a total gastrectomy. Retention of a
gastric remnant is unlikely to achieve acceptable resection margins

Gastric cancer: Overview: There are 700,000 new cases of gastric cancer worldwide each year. It is most common in Japan and
less common in western countries. It is more common in men and incidence rises with increasing age. The exact cause of many
sporadic cancer is not known, however, familial cases do occur in HNPCC families. In addition, smoking and smoked or
preserved foods increase the risk. Japanese migrants retain their increased risk (decreased in subsequent generations). The
distribution of the disease in western countries is changing towards a more proximal location (perhaps due to rising obesity).

Pathology: There is some evidence of support a stepwise progression of the disease through intestinal metaplasia progressing to
atrophic gastritis and subsequent dysplasia, through to cancer. The favoured staging system is TNM. The risk of lymph node
involvement is related to size and depth of invasion; early cancers confined to submucosa have a 20% incidence of lymph node
metastasis. Tumours of the gastro-oesophageal junction are classified as below:

Type 1 True oesophageal cancers and may be associated with Barrett's oesophagus.
Type 2 Carcinoma of the cardia, arising from cardiac type epithelium
or short segments with intestinal metaplasia at the oesophagogastric junction.
Type 3 Sub cardial cancers that spread across the junction. Involve similar nodal stations to gastric cancer.

Groups for close endoscopic monitoring: Intestinal metaplasia of columnar type. Atrophic gastritis. Low to medium grade
dysplasia. Patients who have previously undergone resections for benign peptic ulcer disease (except highly selective vagotomy).

Referral to endoscopy

Patients of any age with dyspepsia and Patients without dyspepsia Worsening dyspepsia
any of the following
Chronic gastrointestinal bleeding Dysphagia Barretts oesophagus
Dysphagia Unexplained abdominal pain or Intestinal metaplasia
weight loss
Weight loss Vomiting Dysplasia
Iron deficiency anaemia Upper abdominal mass Atrophic gastritis
Upper abdominal mass Jaundice Patient aged over 55 years with unexplained or
persistent dyspepsia

Staging: CT scanning of the chest abdomen and pelvis is the routine first line staging investigation in most centres. Laparoscopy
to identify occult peritoneal disease. PET CT (particularly for junctional tumours)

Treatment: Proximally sited disease greater than 5-10cm from the OG junction may be treated by sub total gastrectomy. Total
gastrectomy if tumour is <5cm from OG junction. For type 2 junctional tumours (extending into oesophagus)
oesophagogastrectomy is usual. Endoscopic sub mucosal resection may play a role in early gastric cancer confined to the mucosa
and perhaps the sub mucosa (this is debated). Lymphadenectomy should be performed. A D2 lymphadenectomy is widely
advocated by the Japanese, the survival advantages of extended lymphadenectomy have been debated. However, the overall
recommendation is that a D2 nodal dissection be undertaken. Most patients will receive chemotherapy either pre or post
operatively.

Prognosis: UK Data

Disease extent Percentage 5 year survival


All RO resections 54%
Early gastric cancer 91%
Stage 1 87%
Stage 2 65%

319
Stage 3 18%

Operative procedure: Total Gastrectomy , lymphadenectomy and Roux en Y anastomosis

General anaesthesia. Prophylactic intravenous antibiotics. Incision: Rooftop. Perform a thorough laparotomy to identify any occult
disease. Mobilise the left lobe of the liver off the diaphragm and place a large pack over it. Insert a large self retaining retractor
e.g. omnitract or Balfour (take time with this, the set up should be perfect). Pack the small bowel away.

Begin by mobilising the omentum off the transverse colon. Proceed to detach the short gastric vessels. Mobilise the pylorus and
divide it at least 2cm distally using a linear cutter stapling device. Continue the dissection into the lesser sac taking the lesser
omentum and left gastric artery flush at its origin. The lymph nodes should be removed en bloc with the specimen where possible.

Place 2 stay sutures either side of the distal oesophagus. Ask the anaesthetist to pull back on the nasogastric tube. Divide the distal
oesophagus and remove the stomach. The oesphago jejunal anastomosis should be constructed. Identify the DJ flexure and bring a
loop of jejunum up to the oesophagus (to check it will reach).

Divide the jejunum at this point. Bring the divided jejunum either retrocolic or antecolic to the oesophagus. Anastamose the
oesophagus to the jejunum, using either interrupted 3/0 vicryl or a stapling device. Then create the remainder of the Roux en Y
reconstruction distally.

Place a jejunostomy feeding tube.Wash out the abdomen and insert drains (usually the anastomosis and duodenal stump). Help
the anaesthetist insert the nasogastric tube (carefully!). Close the abdomen and skin. Enteral feeding may commence on the first
post-operative day. However, most surgeons will leave patients on free NG drainage for several days and keep them nil by mouth.

Theme: Causes of upper gastrointestinal haemorrhage

A. Antral gastric ulcer


B. Mallory Weiss tear
C. Oesphageal varices
D. Dieulafoy lesion
E. Gastritis
F. Duodenal ulcer anterior wall
G. Duodenal ulcer posterior wall

Please select the most appropriate likely diagnosis for the scenario given. Each option may be used once, more than once or not at
all.

17. A 35 year old man is admitted with an episode of collapse and passage of malaena. He has been suffering from post
prandial abdominal pain for 5 weeks and this is most marked several hours after eating.

Duodenal ulcer posterior wall. Patients with duodenal ulcers will usually have a history of epigastric pain that occurs several
hours after eating. The pain is often improved by eating food. They are most frequently located i nthe first part of the
duodenum. Anteriorly sited ulcers may perforate and result in peritonitis, posteriorly sited ulcers may erode the
gastroduodenal artery and present with haematemesis and/ or malaena.

18. A 72 year old man is admitted with an episode of brisk haematemesis. Following resuscitation an upper GI endoscopy
is performed and a small mucosal defect approximately 6 cm from the O-G junction on the lesser curve of the stomach
is identified.

The correct answer is Dieulafoy lesion. These small arterial lesions are a rare cause of bleeding and are characterised by a
single large tortuous arteriole in the sub mucosa. Extra gastric lesions may occur.

19. A 56 year old man is admitted with a profuse upper gastro intestinal haemorrhage. He is relatively malnourished and
has evidence of gynaecomastia.

Oesphageal varices. Patients presenting with gastrointestinal bleeding and evidence of established liver disease may have
portal hypertension and develop variceal haemorrhage. The patient may have evidence of jaundice, gynaecomastia, spider
naevia, caput medusae and ascites. The bleeding is usually profuse and painless.

Theme: Dysphagia

A. Mallory Weiss Tear


320
B. Hiatus hernia
C. Squamous cell carcinoma of the oesophagus
D. Adenocarcinoma of the oesophagus
E. Peptic stricture
F. Motility disorder

For each of the following scenarios please select the most likely underlying disorder. Each option may be used once, more than
once or not at all.

20. A 56 year old lady presents with a 6 month history of dysphagia to solids. She has a long history of retrosternal chest
pain that is worse on lying flat and bending forwards. She undergoes an upper GI endoscopy where a smooth stricture is
identified.

Peptic stricture. A six month history of dysphagia is a relatively long history and makes malignancy less likely. The lesion
should be biopsied for histological confirmation. Long standing oesophagitis may be complicated by the development of
strictures, Barretts oesophagus or both. Theme from April 2012 Exam

21. A 76 year old man presents with a 5 week history of progressive dysphagia. An upper GI endoscopy is performed and
the surgeon notices changes that are compatible with Barretts oesophagus. The oesophagus is filled with food debris that
cannot be cleared and the endoscope encounters a resistance that cannot be passed.

Adenocarcinoma of the oesophagus. A short history of dysphagia together with food debris and Barretts changes makes
adenocarcinoma the most likely diagnosis.

22. A 22 year old man presents with a 5 month history of episodic retrosternal chest pain together with episodes of
dysphagia to liquids. An upper GI endoscopy is performed and no mucosal abnormality is seen.

Motility disorder. Dysphagia that is episodic and varies between solids and liquids is more likely to represent a motility
disorder.

Oesophageal disease: Mallory-Weiss Tear: Usually history of antecedent vomiting. This is then followed by the vomiting of a
small amount of blood. There is usually little in the way of systemic disturbance or prior symptoms.

Hiatus hernia of gastric cardia: Often longstanding history of dyspepsia, patients are often overweight. Uncomplicated hiatus
hernias should not be associated with dysphagia or haematemesis.

Squamous cell carcinoma of the oesophagus: History of progressive dysphagia. Often signs of weight loss. Usually little or no
history of previous GORD type symptoms.

Adenocarcinoma of the oesophagus: Progressive dysphagia, may have previous symptoms of GORD or Barretts oesophagus.

Peptic stricture: Longer history of dysphagia, often not progressive. Usually symptoms of GORD. Often lack systemic features
seen with malignancy

Dysmotility disorder: May have dysphagia that is episodic and non progressive. Retrosternal pain may accompany the episodes.

Diagnosis: Most of the differential diagnoses listed above can be accurately categorised by upper GI endoscopy (usually most
patients). Where this fails to demonstrate a mechanical stricture the use of pH and manometry studies together with radiological
contrast swallows will facilitate the diagnosis.

Theme: Management of dysphagia

A. Nissen Fundoplication
B. Belsey Mark IV procedure
C. Heller's Cardiomyotomy
D. Ivor Lewis oesophagectomy
E. Mckeown oesophagectomy
F. Palliation
G. Stent insertion
H. Pneumatic dilatation

321
Select the most appropriate treatment for the oesophageal disorder described from the list above. Each option may be used once,
more than once or not at all.

23. A 28 year old man with longstanding dysphagia that is worse to liquids than solids. He has a barium swallow that shows
a 'birds beak' disorder of the distal oesophagus. Non operative treatments have been undertaken for this before and
surgery is planned.

Heller's Cardiomyotomy. This man has achalasia and as the questions suggests other measures have been tried a laparoscopic
Heller's Myotomy is the next step.

24. A 78 year old man had reflux many years previously. He was treated with an anti-reflux operation that left him with a
left sided thoracotomy scar.

The correct answer is Belsey Mark IV procedure. The Belsey Mark IV procedure employs a thoracic approach.

25. A 56 year old man with an adenocarcinoma of the proximal oesophagus. Staging investigations show no metastatic
disease and he is otherwise fit.

Mckeown oesophagectomy. He requires a total (3 oesophagectomy). This is also called a Mckeown oesophagectomy.

26. A 73 year old lady presents with progressive dysphagia and is diagnosed with oesophageal cancer and liver metastases, it
is located 8cm proximal to the gastro-oesophageal junction. Which of the following treatment options would be the the
most appropriate management?
A. Insertion of Celestin tube
B. Insertion of Minnesota tube
C. Insertion of self expanding metal stent
D. Photodynamic therapy
E. Trans hiatal oesphagectomy
Answer: C
Most cases of malignant oesophageal obstruction can be managed by the placement of self expanding metal stents. The Celestin
tube requires a laparotomy and is therefore obsolete. A resectional procedure would be inappropriate in the presence of liver
metastasis. The main contra indication to metallic stent placement are very proximal tumours as it can be difficult to get proximal
control in this situation and chemotherapy may be more appropriate.

27. Which of the following statements relating to gastric banding for obesity is false?
A. It is one of the safest anti obesity operations
B. If successful up to 55% of excess weight may be lost over 2 years
C. Excessively tight gastric bands have increased risk of long term complications
D. It is associated with early satiety
E. It is contra indicated in patients with polycystic ovaries who are trying to conceive
Answer: E
Adjustable gastric bands are one of the most widely performed anti obesity procedures in the UK. They are relatively easy to
insert. Weight loss is slightly slower than with some of the other weight loss procedures. Up to 15% patients may require
revisional surgery.

28. Which of the following statements relating to gastric banding for obesity is false?
A. It is one of the safest anti obesity operations
B. If successful up to 55% of excess weight may be lost over 2 years
C. Excessively tight gastric bands have increased risk of long term complications
D. It is associated with early satiety
E. It is contra indicated in patients with polycystic ovaries who are trying to conceive
Answer: E
Adjustable gastric bands are one of the most widely performed anti obesity procedures in the UK. They are relatively easy to
insert. Weight loss is slightly slower than with some of the other weight loss procedures. Up to 15% patients may require
revisional surgery.

29. Which of the following procedures is not performed for obesity?


A. Sleeve gastrectomy
B. Gastric band
C. Intra gastric balloon
D. Mckeown procedure
E. Small bowel bypass
322
Answer: D
A McKeown procedure is a total oesophagectomy.

30. Which of the following criteria is not an indication for bariatric surgery to be performed in the UK National Institute of
Clinical Excellence Guidelines?
A. Patients must be non smoking for at least one year
B. Patients must have tried conservative management for at least 6 months
C. Commitment to long-term follow up
D. Surgery to be performed in a specialist unit
E. BMI > 40 kg/m2
Answer: A
Being a non smoker is not included in the criteria, however poor respiratory function due to smoking may affect fitness for
surgery.

31. A 34-year-old HIV positive man is referred to the surgical out patient department with jaundice and abnormal liver
function tests. Liver function tests are as follows: Albumin 34 g/l, ALP 540 iu/l, Bilirubin 67 µmol/l, ALT 45 iu/l. What is
the most likely diagnosis?
A. Hepatic abscess
B. Fungal obstruction of the bile duct
C. Duodenal adenoma
D. Primary biliary cirrhosis
E. Sclerosing cholangitis
Answer: E
The LFTs clearly show a cholestatic picture. Given the background of HIV the most likely cause is sclerosing cholangitis.

HIV: biliary and pancreatic disease: The most common cause of biliary disease in patients with HIV is sclerosing cholangitis
due to infections such as CMV, Cryptosporidium and Microsporidia. Pancreatitis in the context of HIV infection may be
secondary to anti-retroviral treatment (especially didanosine) or by opportunistic infections e.g. CMV

32. A 42 year old woman with known multiple gastric ulcers attends the surgical out patient unit. She has not improved
despite 2 months of proton pump inhibitor treatment. She is found to have a gastrinoma. Which of the following is false
in relation to her diagnosis?
A. Most commonly found in the pancreas
B. Associated with multiple endocrine neoplasia I
C. Somatostatin sensitive scintigraphy is the most senstive non invasive test for localizing primary tumours
D. Primary tumours can occur in the ovary
E. Secretory diarrhoea is a feature
Answer: A
Most commonly found in the duodenum (in up to 50% patients), then the pancreas (approximately 20%) . Other ectopic areas
include stomach, spleen, gallbladder and ovary

Greater than 4/5 of gastrinomas are found within the triangle bounded by: Cystic and common bile duct (Top). 2nd and 3rd part of
the duodenum (Bottom). Neck and body of pancreas (Medial). Pancreatic gastrinomas are normally solitary and highly malignant.
Zollinger Ellison syndrome is composed of the triad of: 1. Non beta islet cell tumours of the pancreas 2. Hypergastrinaemia 3.
Severe ulcer disease. Clinical features related to peptic ulcer disease. Diagnosis is based on 3 criteria:1. Fasting
hypergastrinaemia. 2. Increased basal acid output. 3. Secretin stimulation test positive. Management: Resection if localised disease

Pancreatic cancer: Adenocarcinoma. Risk factors: Smoking, diabetes, Adenoma, Familial adenomatous polyposis. Mainly occur
in the head of the pancreas (70%). Spread locally and metastasizes to the liver. Carcinoma of the pancreas should be differentiated
from other periampullary tumours with better prognosis

Clinical features: Weight loss. Painless jaundice. Epigastric discomfort (pain usually due to invasion of the coeliac plexus is a
late feature). Pancreatitis. Trousseau's sign: migratory superficial thrombophlebitis

Investigations: USS: May miss small lesions. CT Scanning (pancreatic protocol). If unresectable on CT then no further staging
needed. PET/CT for those with operable disease on CT alone. ERCP/ MRI for bile duct assessment. Staging laparoscopy to
exclude peritoneal disease.

Management: Head of pancreas: Whipple's resection (SE dumping and ulcers). Newer techniques include pylorus preservation
and SMA/ SMV resection. Carcinoma body and tail: poor prognosis, distal pancreatectomy if operable. Usually adjuvent
chemotherapy for resectable disease. ERCP and stent for jaundice and palliation. Surgical bypass may be needed for duodenal
obstruction.

323
33. A 63 year old man undergoes a total gastrectomy for carcinoma of the stomach. Which of the sequelae below is least
likely to occur?
A. Metabolic bone disease
B. Bile reflux
C. Dumping syndrome
D. Zinc deficiency
E. B12 deficiency
Answer: D
Zinc is mainly absorbed in the duodenum and jejunum. Bile reflux may occur post gastrectomy. The risk of bile reflux is lower if
a Roux en Y reconstruction is used.

Post gastrectomy syndromes: Post gastrectomy syndromes may vary slightly depending upon whether a total of partial
gastrectomy is performed. A Roux en Y reconstruction generally gives the best functional outcomes. Where a gastrojejunostomy
is performed as reconstruction following a distal gastrectomy the gastric emptying is generally better if the jejunal limbs are
tunneled in the retrocolic plane.

The following may occur following gastrectomy: Small capacity (early satiety). Dumping syndrome. Bile gastritis. Afferent loop
syndrome. Efferent loop syndrome. Anaemia (B12 deficiency). Metabolic bone disease

34. A 34 year old women with morbid obesity is referred for consideration of bariatric surgery. Which of the following
options is associated with the highest long term failure rates?
A. Gastric band
B. Intra gastric balloon
C. Roux en Y bypass
D. Sleeve gastrectomy
E. Duodenal switch
Answer: B
Intragastric balloon is really only suitable as a bridge to a more definitive surgical solution.

Bariatric surgery: the main operations

Gastric banding: band applied to upper stomach which can be inflated or deflated with normal saline. This affects satiety. Over a
5 year period complications requiring further surgery occur in up to 15% cases.

Roux-en-Y gastric bypass: a gastric pouch is formed and connected to the jejunum. Patients achieve greater and more longterm
weight loss than gastric banding.

Sleeve gastrectomy: body and fundus resected to leave a small section of stomach

Biliopancreatic diversion +/- duodenal switch: bypass the small bowel. Greatest weight loss but a very complex procedure
associated with malnutrition and diarrhoea.

Vertical banded gastroplasty (stomach stapling): rarely performed due to longterm failure rate.

35. Which of the following variables is not included in the Rockall score?
A. Congestive cardiac failure
B. Liver failure
C. Systolic blood pressure < 100mmHg
D. Aspirin usage
E. Age
Answer: D

Rockall Score

Applies to upper gastrointestinal bleeding


Variable Score 0 Score 1 Score 2 Score 3
Age <60 60-79 >80
Shock None Pulse Hypotension (systolic
>100 <100mmHg)
Nil or Major organ disease e.g. IHD, Renal or liver failure, metastatic
Co-Morbidity minimal CCF cancer
Diagnosis Mallory- All GI Cancer
Weiss
324
Evidence of None Clot, Blood, spurting vessel
Bleeding

Score <3 = Good prognosis (mortality approx. 2%). Score >8= High mortality (Mortality approx. 40%)

Theme: Oesophageal disease

A. Nissen Fundoplication
B. Belsey Mark IV procedure
C. Heller's Cardiomyotomy
D. Ivor Lewis oesophagectomy
E. McKeown oesophagectomy
F. Palliation
G. Stent insertion
H. Pneumatic dilatation

Select the most appropriate treatment for the oesophageal disorder described from the list above. Each option may be used once,
more than once or not at all.

36. A 56 year old man presents with an adenocarcinoma of the mid oesophagus. Staging investigations show no metastatic
disease and he is otherwise fit.

McKeown oesophagectomy. He requires a total (3 stage oesophagectomy). This is also called a McKeown oesophagectomy.
Ivor Lewis type resections are concluded in the mid third of the oesophagus and would not adequately treat this disease

37. A 43 year old man with an adenocarcinoma of the distal oesophagus. His staging investigations are negative for
metastatic disease.

Ivor Lewis oesophagectomy. This requires an Ivor Lewis oesophagectomy with resection of the distal oesophagus and an
intrathoracic anastomosis

38. A 28 year old man with long standing symptoms of dyspepsia. His 24 hour pH and Manometry study show a low resting
pressure of the lower oesophageal sphincter and multiple episodes where the pH falls below 4. He has declined to
continue with PPI therapy.

Nissen Fundoplication. The Nissen fundoplication is the standard. A Belsey Mark IV is an older anti reflux procedure that is not
currently in routine practice

Oesophagus: 25cm long. Starts at C6 vertebra, pierces diaphragm at T10 and ends at T11. Squamous epithelium

Constrictions of the oesophagus


Structure Distance from incisors
Cricoid cartilage 15cm
Arch of the Aorta 22.5cm
Left principal bronchus 27cm
Diaphragmatic hiatus 40cm

Relations
 Trachea to T4  Thoracic duct to left at T5
 Recurrent laryngeal nerve  Hemiazygos to the left T8
 Left bronchus, Left atrium  Descending aorta
Anterior Posterior
 Diaphragm  First 2 intercostal branches of aorta

 Thoracic duct  Azygos vein


Left  Left subclavian artery Right

Arterial, venous and lymphatic drainage of the oesophagus


Artery Vein Lymphatics Muscularis externa
325
Upper third Inferior thyroid Inferior thyroid Deep cervical Striated muscle
Mid third Aortic branches Azygos branches Mediastinal Smooth & striated muscle
Lower third Left gastric Posterior mediastinal and coeliac Gastric Smooth muscle

Nerve supply: Upper half is supplied by recurrent laryngeal nerve. Lower half by oesophageal plexus (vagus)

Histology: Mucosa :Nonkeratinized stratified squamous epithelium. Submucosa: glandular tissue. Muscularis externa
(muscularis): composition varies. See table. Adventitia

Theme: Gastric ulceration

A. Acute peptic ulcer


B. Adenocarcinoma
C. Carcinoid Tumour
D. Gastrointestinal stromal tumour
E. Chronic peptic ulcer
F. Lymphosarcoma
G. Leiomyoma

Please select the most likely cause of gastric ulceration for the scenario given. Each option may be used once, more than once or
not at all.

39. A 53 year old man presents with dyspepsia. At upper GI endoscopy he has a punched out ulcer on the lesser curve of the
stomach. It measures approximately 2cm in diameter and is seen to penetrate muscle with fibrosis present at the base.

Chronic peptic ulcer. Fibrosis is usually a sign of chronic ulcer. It should be biopsied carefully, a proton pump inhibitor started
and re endoscopy should occur at 6 weeks.

40. A 42 year old man presents with epigastric pain. At endoscopy he is found to have a punched out ulcer on the anterior
wall of the stomach, it is shallow and measures 1cm in diameter

The correct answer is Acute peptic ulcer. The absence of fibrosis coupled with small size suggests a more acute ulcer.
Management should include biopsy, PPI and repeat endoscopy at 6 weeks.

41. A 65 year old man presents with epigastric discomfort. At endoscopy he is found to have an ulcer at the antrum, is has
thick rolled edges and measures 3cm in diameter.

The correct answer is Adenocarcinoma. Rolled edges and location favor tumour. Careful biopsies should be taken and staging
CT performed.

Theme: Causes of dysphagia

A. Oesophageal cancer
B. Post radiotherapy fibrosis
C. Benign stricture
D. Plummer Vinson syndrome
E. Oesophageal candidiasis
F. Neuropathy
G. Globus

Please select the most likely cause of dysphagia for the scenarios given. Each option may be used once, more than once or not at
all.

42. A cachectic 32 year old man with severe perineal Crohns disease is receiving treatment with intravenous antibiotics.
Over the past 72 hours he has complained of intermittent dysphagia and odynophagia.

Oesophageal candidiasis. Treatment with systemic antibiotics may result in development of candidiasis. Patients may present
with odynophagia and episodic dysphagia. Endoscopic appearances are usually diagnostic and treatment is with an oral anti
fungal agent

43. A 78 year old lady presents 6 years following a successfully treated squamous cell carcinoma of the oesophagus. She has
a long history of dysphagia but it is not progressive.

326
Post radiotherapy fibrosis. SCC of the oesophagus is commonly treated with chemoradiotherapy. Fibrosis and dysphagia may
occur in survivors.

44. A 32 year old lady presents with dysphagia. She has a 10 year history of anaemia secondary to menorrhagia and has
been strongly resistant to treatment.

Plummer Vinson syndrome. Plummer Vinson syndrome (oesophageal web) may occur in association with iron deficiency
anaemia (although rare!).

 Dysphagia: Causes of dysphagia


 Extrinsic: Mediastinal masses. Cervical spondylosis
 Oesophageal wall: Achalasia. Diffuse oesophageal spasm. Hypertensive lower oesophageal sphincter
 Intrinsic: Tumours. Strictures. Oesophageal web. Schatzki rings
 Neurological: CVA. Parkinson's disease. Multiple Sclerosis. Brainstem pathology. Myasthenia Gravis

Investigation: All patients require an upper GI endoscopy unless there are compelling reasons for this not to be performed.
Motility disorders may be best appreciated by undertaking fluoroscopic swallowing studies.

A full blood count should be performed. Ambulatory oesophageal pH and manometry studies will be required to evaluate
conditions such as achalasia and patients with GORD being considered for fundoplication surgery.

45. Which of the following is false during the pre operative preparation for surgery in pancreatic cancer?
A. IV antibiotics should be given intra operatively
B. Endotoxamia can be reduced with lactulose
C. Subcutaneous heparin should be avoided
D. Endotoxamia can be reduced with IV mannitol
E. There is a higher risk of complications if the bilirubin is greater than 150
Answer: C
Vitamin K should be given to correct abnormal clotting initially, however there is still a risk of thrombosis so low molecular
weight heparin should be used. Bile salts can also be given to reduce endotoxaemia. Biliary obstruction should be relieved. In the
case of biliary obstruction, if a stent is used it should be a made of plastic. Metallic stents will become embedded and may
compromise attempts at resection.

Hepatobiliary System
Theme: Management of jaundice

A. ERCP
B. MRCP
C. Percutaneous transhepatic cholangiogram
D. Laparotomy
E. Laparotomy and formation of hepatico-jejunostomy
F. Laparoscopic biliary bypass
G. CT scan

For each of the following scenarios please select the most appropriate next stage of management. Each option may be used once,
more than once or not at all.

1. A 65 year old man is admitted with jaundice and investigations demonstrate a carcinoma of the pancreatic head. An
ERCP is attempted but the surgeon is unable to cannulate the ampulla.

The correct answer is Percutaneous transhepatic cholangiogram. Cancer of the pancreatic head will cause and obstructed
jaundice and intrahepatic duct dilatation. When an ERCP has failed the most appropriate option is to attempt a PTC. This
procedure is always preceded by an ultrasound (which presumably this patient has already had or they would not be undergoing
an ERCP). Prior to performing the PTC it is important to stage the disease and establish resectability or not. This is because the
PTC drains frequently dislodge and fall out. It is usually desirable to pass a stent at the time of doing the PTC to mitigate the
effects of this problem. Theme from September 2012 Exam

2. A 48 year old lady is admitted with attacks of biliary colic and investigations show gallstones. A laparoscopic
cholecystectomy is performed. The operation is technically challenging due to a large stone impacted in Hartmans
327
pouch. Following the operation she fails to settle and becomes jaundiced and has bile draining into a drain placed at the
surgical site.

The correct answer is ERCP. In this scenario it must be assumed that the bile duct has been damaged. In most cases an ERCP is
the most appropriate investigation. This can also allow the passage of a stent if this is deemed to be safe and sensible.

3. A 34 year old lady is admitted with jaundice and undergoes an ERCP. The procedure is technically difficult and she is
returned to the ward still jaundiced. Unfortunately she now has severe generalised abdominal pain.

The correct answer is CT scan. There are two main differential diagnoses here. One is pancreatitis, repeated trauma to the
ampulla and duct (if partially cannulated) is a major risk factor for pancreatitis. The second is the possibility that the duodenum
has been perforated. ERCP is performed using a side viewing endoscope, the manipulation of which can be technically
challenging for the inexperienced operator in a patient with abnormal anatomy. A CT scan is the best investigation to
distinguish between these two differential diagnoses.

Surgical jaundice: Jaundice can present in a manner of different surgical situations. As with all types of jaundice a carefully
history and examination will often give clues as the most likely underlying cause. Liver function tests whilst conveying little in
the way of information about liver synthetic function, will often facilitate classification as to whether the jaundice is pre hepatic,
hepatic or post hepatic. The typical LFT patterns are given below:

Location Bilirubin ALT/ AST Alkaline phosphatase


Pre hepatic Normal or high Normal Normal
Hepatic High Elevated (often very high) Elevated but seldom to very high levels
Post hepatic High-very high Moderate elevation High- very high

In post hepatic jaundice the stools are often of pale colour and this feature should be specifically addressed in the history.

Modes of presentation: these are addressed in the table below:

Diagnosis Typical features Pathogenesis


Gallstones Typically history of biliary colic Usually small calibre gallstones which can pass through the cystic
or episodes of chlolecystitis. duct. In Mirizzi syndrome the stone may compress the bile duct
Obstructive type history and test directly- one of the rare times that cholecystitis may present with
results. jaundice
Cholangitis Usually obstructive and will Ascending infection of the bile ducts usually by E. coli and by
have Charcots triad of definition occurring in a pool of stagnant bile.
symptoms (pain, fever,
jaundice)
Pancreatic cancer Typically painless jaundice with Direct occlusion of distal bile duct or pancreatic duct by tumour.
palpable gallbladder Sometimes nodal disease at the portal hepatis may be the culprit in
(Courvoisier's Law) which case the bile duct may be of normal calibre.
TPN associated Usually follows long term use Often due to hepatic dysfunction and fatty liver which may occur with
jaundice and is usually painless with non long term TPN usage.
obstructive features
Bile duct injury Depending upon the type of Often due to a difficult cholecystectomy when anatomy in Calots
injury may be of sudden or triangle is not appreciated. In the worst scenario the bile duct is
gradual onset and is usually of excised and jaundice offers rapidly post operatively. More insidious is
obstructive type that of bile duct stenosis which may be caused by clips or diathermy
injury.
Cholangiocarcinoma Gradual onset obstructive Direct occlusion by disease and also extrinsic compression by nodal
pattern disease at the porta hepatis.
Septic surgical Usually hepatic features Combination of impaired biliary excretion and drugs such as
patient ciprofloxacin which may cause cholestasis.
Metastatic disease Mixed hepatic and post hepatic Combination of liver synthetic failure (late) and extrinsic compression
by nodal disease and anatomical compression of intra hepatic
structures (earlier)

Diagnosis: An ultrasound of the liver and biliary tree is the most commonly used first line test. This will establish bile duct
calibre, often ascertain the presence of gallstones, may visualise pancreatic masses and other lesions. The most important clinical
question is essentially the extent of biliary dilatation and its distribution.

Where pancreatic neoplasia is suspected the next test should be a pancreatic protocol CT scan. With liver tumours and

328
cholangiocarcinoma an MRI/ MRCP is often the preferred option. PET scans may be used to stage a number of malignancies but
do not routinely form part of first line testing.

Where MRCP fails to give adequate information and ERCP may be necessary. In many cases this may form part of patient
management. It is however, invasive and certainly not without risk and highly operator dependent.

Management: Clearly this will depend to an extent upon the underlying cause but relief of jaundice is important even if surgery
forms part of the planned treatment as patients with unrelieved jaundice have a much higher incidence of septic complications,
bleeding and death.

Screen for and address any clotting irregularities

In patients with malignancy a stent will need to be inserted. These come in two main types; metal and plastic. Plastic stents are
cheap and easy to replace and should be used if any surgical intervention (e.g. Whipples) is planned. However, they are prone to
displacement and blockage. Metal stents are much more expensive and may compromise a surgical resection. However, they are
far less prone to displacement and to a lesser extent blockage than their plastic counterparts.

If malignancy is in bile duct/ pancreatic head and stenting has been attempted and has failed, then an alternative strategy is to
drain the biliary system percutaneously via a transhepatic route. It may also be possible to insert a stent in this way. One of the
main problems with temporary PTC's is their propensity to displacement which may result in a bile leak.

In patients who have a bile duct injury surgery will be required to repair the defect. If the bile duct has been inadvertently excised
then a hepatico-jejunostomy will need to be created (difficult!)

If gallstones are the culprit then these may be removed by ERCP and a cholecystectomy performed. Where there is doubt about
the efficacy of the ERCP an operative cholangiogram should be performed and bile duct exploration undertaken where stones
remain. When the bile duct has been formally opened the options are between closure over a T tube, a choledochoduodenostomy
or choledochojejunostomy.

Patients with cholangitis should receive high dose broad spectrum antibiotics via the intravenous route. Biliary decompression
should follow soon afterwards and instrumenting the bile duct of these patients will often provoke a septic episode (but should be
done anyway).

4. Which of the following is the most sensitive blood test for diagnosis of acute pancreatitis?
A. Amylase
B. Lipase
C. C-peptide
D. Trypsin
E. Trysinogen
Answer: B
The serum amylase may rise and fall quite quickly and lead to a false negative result. Should the clinical picture not be concordant
with the amylase level then serum lipase or a CT Scan should be performed.

Management of Acute Pancreatitis in the UK

Diagnosis: Traditionally hyperamylasaemia has been utlilised with amylase being elevated three times the normal range.
However, amylase may give both false positive and negative results. Serum lipase is both more sensitive and specific than serum
amylase. It also has a longer half life. Serum amylase levels do not correlate with disease severity.

Differential causes of hyperamylasaemia: acute pancreatitis, pancreatic pseudocyst, mesenteric infarct, perforated viscus, acute
cholecystitis, diabetic ketoacidosis

Assessment of severity: Glasgow, Ranson scoring systems and APACHE II. Biochemical scoring e.g. using. CRP

Features that may predict a severe attack within 48 hours of admission to hospital

 Initial assessment: Clinical impression of severity. Body mass index >30. Pleural effusion. APACHE score >8
 ours after admission: Clinical impression of severity. APACHE II >8. Glasgow score of 3 or more. Persisting multiple
organ failure. CRP>150
 ours after admission: Glasgow Score of >3. CRP >150. Persisting or progressive organ failure

329
Table adapted from UK guidelines for management of acute pancreatitis. GUT 2005, 54 suppl III

Management: Nutrition: There is reasonable evidence to suggest that the use of enteral nutrition does not worsen the outcome in
pancreatitis. Most trials to date were underpowered to demonstrate a conclusive benefit. The rationale behind feeding is that it
helps to prevent bacterial translocation from the gut, thereby contributing to the development of infected pancreatic necrosis.

Use of antibiotic therapy: Many UK surgeons administer antibiotics to patients with acute pancreatitis. A recent Cochrane
review highlights the potential benefits of administering Imipenem to patients with established pancreatic necrosis in the hope of
averting the progression to infection.There are concerns that the administration of antibiotics in mild attacks of pancreatitis will
not affect outcome and may contribute to antibiotic resistance and increase the risks of antibiotic associated diarrhoea.

Surgery: Patients with acute pancreatitis due to gallstones should undergo early cholecystectomy. Patients with obstructed biliary
system due to stones should undergo early ERCP. Patients who fail to settle with necrosis and have worsening organ dysfunction
may require debridement, fine needle aspiration is still used by some. Patients with infected necrosis should undergo either
radiological drainage or surgical necrosectomy. The choice of procedure depends upon local expertise.

Theme: Jaundice

A. Gilberts syndrome
B. Crigler Najjar syndrome
C. Hepatocellular carcinoma
D. Mirizzi syndrome
E. Hepatitis A
F. Hepatitis E
G. Bile duct stones
H. Multi cystic liver disease

Please select the most likely cause of jaundice for the scenario given. Each option may be used once, more than once or not at all.

5. A 22 year old man returns to the UK from holiday in India. He presents with painless jaundice. On examination he is not
deeply jaundiced and there is no organomegaly.

Hepatitis A.Infective hepatitis is the most likely cause. In the UK, foreign travel is a common cause of developing infectious
hepatitis, of which hepatitis A is the most common.

6. A 56 year old man presents with jaundice. He has a long history of alcohol misuse. On examination he is jaundiced and
ultrasound shows multiple echo dense lesions in both lobes of the liver. His alpha feto protein is elevated 6 times the
normal range

Hepatocellular carcinoma.HCC may complicate cirrhosis. AFP is often raised in HCC.

7. A 32 year old man who has suffered from Crohns disease for many years presents with intermittent jaundice. When it
occurs it is obstructive in nature. It then usually resolves spontaneously.

The correct answer is Bile duct stones. Bile salts are absorbed in the terminal ileum. When this process is impaired as in Crohns
the patient may develop gallstones, if these pass into the CBD then obstructive jaundice will result.

Theme: Surgical jaundice

A. Carcinoma of the head of the pancreas


B. Bile duct stricture
C. Mirizzi syndrome
D. Bile duct stones
E. Chronic cholecystitis
F. Peri hilar lymphadenopathy
G. Fitz - Hugh Curtis syndrome

Please select the most appropriate cause of the jaundice scenario given. Each option may be used once, more than once or not at
all.

8. A 63 year old man is admitted with obstructive jaundice that has developed over the past 3 weeks. He was previously
well and on examination has a smooth mass in his right upper quadrant.
330
Carcinoma of the head of the pancreas. Carcinoma of the pancreas (Courvoisiers law!). The development of jaundice in
association with a smooth right upper quadrant mass is typical of distal biliary obstruction secondary to pancreatic malignancy.
A bile duct stricture would not present in this way, all the other choices are related to gallstones and Fitz Hugh Curtis syndrome
is a complication of pelvic inflammatory disease.

9. A 41 year old lady is admitted with colicky right upper quadrant pain. On clinical examination she has a mild pyrexia
and is clinically jaundiced. An ultrasound scan is reported as showing gallstones and the patient is taken to theatre for
an open cholecystectomy. At operation, Calots triangle is almost completely impossible to delineate.

The correct answer is Mirizzi syndrome. In Mirizzi syndrome the gallstone becomes impacted in Hartmans pouch. Episodes of
recurrent inflammation occur and this causes compression of the bile duct. In severe cases this then progresses to fistulation.
Surgery is extremely difficult as Calots triangle is often completely obliterated and the risks of causing injury to the CBD are
high.

10. A 72 year old man undergoes a distal gastrectomy for carcinoma of the stomach. He presents with jaundice
approximately 8 months post operatively. Ultrasound of the liver and bile ducts shows no focal liver lesion and normal
calibre common bile duct with intra hepatic duct dilatation.

The correct answer is Peri hilar lymphadenopathy. Unfortunately metastatic disease is the most likely event. Peri hilar
lymphadenopathy would be a common culprit.

Courvoisiers Law: Obstructive jaundice in the presence of a palpable gallbladder is unlikely to be due to stones.
This is due to the fibrotic effect that stones have on the gallbladder. Like all these laws there are numerous exceptions and many
cases will not present in the typical manner.

Bile duct injury: Inadvertent bile duct injury during laparoscopic surgery should be referred to a specialist hepatobiliary surgeon.
Outcomes are far worse when repair in undertaken by a non specialist surgeon in a district hospital.

Theme: Management of biliary disease

A. Acute laparoscopic cholecystectomy


B. Delayed laparoscopic cholecystectomy
C. Percutaneous cholecystostomy
D. Elective cholecystectomy and intra operative cholangiogram
E. Endoscopic retrograde cholangiopancreatography
F. Choledochoduodenostomy
G. Bile duct excision and hepatico-jejunostomy
H. Operative cholecystostomy

For each scenario please select the most appropriate management option. Each option may be used once, more than once or not at
all.

11. A 72 year old lady underwent an open cholecystectomy 12 years previously. She has been admitted since with 2 episodes
of cholangitis and stones were retrieved at ERCP. She has just recovered from a further episode of sepsis and MRCP has
shown further biliary stones.

The correct answer is Choledochoduodenostomy. A patient with long standing common bile duct stones is at risk of developing
duct fibrosis and ductal disproportion. This can result in impaired biliary drainage. Not only may further stones be formed in the
bile that is present, but because of the ductal disproportion the tendency will be for the stones to accumulate (rather than pass
spontaneously, as would usually be the case post ERCP and sphincterotomy). A biliary bypass procedure is the standard method
dealing with this and a choledochoduodenstomy is one procedure that can be used.

12. A 26 year old women is admitted with acute cholecystitis of 24 hours duration. LFT's are normal and Ultrasound shows
a thick walled gallbladder containing stones.

Acute laparoscopic cholecystectomy. This is an ideal case for an acute cholecystectomy, provided that surgery can be
undertaken promptly. After 48 -72 hours the patient should receive parenteral antibiotics and delayed cholecystectomy
performed.

13. A 32 year old lady is seen in the outpatients. She has had multiple episodes of biliary colic and ultrasound shows thin
walled gallbladder with multiple calculi. Her ALT is slightly raised but other parameters are normal.

331
Elective cholecystectomy and intra operative cholangiogram. The easiest option is to perform an intraoperative cholangiogram.
It is unlikely to reveal any stones. If is does then either laparoscopic bile duct exploration or urgent ERCP can be performed. An
MRCP pre op is an alternative strategy.

14. 42 year old female presents with symptoms of biliary colic and on investigation is identified as having gallstones. Of the
procedures listed below, which is most likely to increase the risk of gallstone formation?
A. Partial gastrectomy
B. Jejunal resection
C. Liver lobectomy
D. Ileal resection
E. Left hemicolectomy
Answer: D
Bile salt reabsorption occurs at the ileum. Therefore cholesterol gallstones form as a result of ileal resection.

Theme: Management of pancreatitis

A. Non Contrast enhanced CT scan


B. USS abdomen
C. ERCP alone
D. ERCP with Sphincterotomy and biliary drainage
E. Fine needle aspiration of necrosis
F. Pancreatic necrosectomy
G. Contrast enhanced CT scan

What is the next best step in management for the scenario given? Each option may be used once, more than once or not at all.

15. A 56 year old man is admitted with an attack of severe acute pancreatitis. He is managed on the intensive care unit and
is making progress. He then deteriorates and a CT scan shows extensive pancreatic necrosis (>40%). On return from the
radiology department he remains febrile and tachycardic with falling urine output.

Pancreatic necrosectomy. In patients with systemic features of sepsis and extensive necrosis a necrosectomy is usually
indicated.An FNA will not change his immediate management.

16. A 22 year old teacher is admitted with severe epigastric pain. Serum amylase is normal. You wish to exclude a
perforated viscus, and determine whether pancreatitis is present.

The correct answer is Contrast enhanced CT scan. An ultrasound will not accurately answer this question. Therefore a CT scan
is required. Oral and IV contrast would usually be given.

17. A 55 year old accountant has jaundice and a temperature of 39oC. He is known to have gallstones. Blood cultures have
grown a gram negative bacilli.

ERCP with Sphincterotomy and biliary drainage. You should suspect cholangitis in a patient with fevers and jaundice. Charcot's
triad may only be present in 20% of patients. This patient needs biliary drainage with an ERCP.

Infected pancreatic necrosis is one of the few indications for surgery in pancreatitis

Theme: Liver tumours

A. Rhabdomyosarcoma
B. Yolk sac tumour
C. Hepatocellular carcinoma
D. Metastatic lesion
E. Haemangioendothelioma
332
F. Cholangiocarcinoma
G. Hepatoblastoma
H. Angiosarcoma

Please select the most likely diagnosis for the scenario given. Each answer may be used once, more than once or not at all.

18. A 56 year old man with long standing ulcerative colitis and a DALM lesion in the rectum is admitted with jaundice. On
CT scanning the liver has 3 nodules in the right lobe and 1 nodule in the left lobe. Carcinoembryonic antigen levels are
elevated.
Metastatic lesion. This is likely to be due to metastatic lesions from a colonic primary. DALM lesions should be excised by
oncological colectomy for this reason. This burden of metastatic disease is unlikely to precipitate jaundice directly and nodal
disease at the porta hepatis is the most likely cause in this case.

19. A 48 year old lady with chronic hepatitis B infection is noted to have worsening liver function tests and progressive
jaundice. Her alpha feto protein levels are grossly elevated.

Hepatocellular carcinoma. This is most likely to be hepatocellular carcinoma and markedly elevated AFP levels in association
with a compatible risk factor should make this the diagnosis.

20. A 55 year old man with long standing ulcerative colitis is admitted with cholangitis and weight loss. Blood tests reveal a
markedly elevated Ca 19-9.

Cholangiocarcinoma. This is most likely a cholangiocarcinoma. UC with sclerosing cholangitis. Increases the risk of
cholangiocarcinoma. CA19-9 is elevated in approximately 80% cases.

Liver tumours: Primary liver tumours


The most common primary tumours are cholangiocarcinoma and hepatocellular carcinoma. Overall metastatic disease accounts
for 95% of all liver malignancies making the primary liver tumours comparatively rare.

Primary liver tumours include: cholangiocarcinoma, hepatocellular carcinoma, hepatoblastoma, sarcomas (rare), lymphomas,
carcinoids (most often secondary although primary may occur)

Hepatocellular carcinoma: these account for the bulk of primary liver tumours (75% cases). Its worldwide incidence reflects its
propensity to occur on a background of chronic inflammatory activity. Most cases arise in cirrhotic livers or those with chronic
hepatitis B infection, especially where viral replication is actively occurring. In the UK it accounts for less than 5% of all cancers,
although in parts of Asia its incidence is 100 per 100,000.
The majority of patients (80%) present with existing liver cirrhosis, with a mass discovered on screening ultrasound.

Diagnosis: CT/ MRI (usually both) are the imaging modalities of choice. a-fetoprotein is elevated in almost all cases. Biopsy
should be avoided as it seeds tumours cells through a resection plane. In cases of diagnostic doubt serial CT and aFP
measurements are the preferred strategy.

Treatment: Patients should be staged with liver MRI and chest, abdomen and pelvic CT scan. The testis should be examined in
males (testicular tumours may cause raised AFP). PET CT may be used to identify occult nodal disease. Surgical resection is the
mainstay of treatment in operable cases. In patients with a small primary tumour in a cirrhotic liver whose primary disease process
is controlled, consideration may be given to primary whole liver resection and transplantation. Liver resections are an option but
since most cases occur in an already diseased liver the operative risks and post-operative hepatic dysfunction are far greater than is
seen following metastectomy. These tumours are not particularly chemo or radiosensitive however, both may be used in a
palliative setting. Tumour ablation is a more popular strategy.

Survival: poor, overall survival is 15% at 5 years.

Cholangiocarcinoma: this is the second most common type of primary liver malignancy. As its name suggests these tumours
arise in the bile ducts. Up to 80% of tumours arise in the extra hepatic biliary tree. Most patients present with jaundice and by this
stage the majority will have disease that is not resectable.
Primary scelerosing cholangitis is the main risk factor. In deprived countries typhoid and liver flukes are also major risk factors.

Diagnosis: Patients will typically have an obstructive picture on liver function tests. CA 19-9, CEA and CA 125 are often
elevated. CT/ MRI and MRCP are the imaging methods of choice.

Treatment: Surgical resection offers the best chance of cure. Local invasion of peri hilar tumours is a particular problem and this
coupled with lobar atrophy will often contra indicate surgical resection. Palliation of jaundice is important, although metallic
stents should be avoided in those considered for resection.
333
Survival: Is poor, approximately 15% 5 year survival.

21. A 45 year old man presents with an episode of alcoholic pancreatitis. He makes slow but steady progress. He is reviewed
clinically at 6 weeks following admission. He has a diffuse fullness of his upper abdomen and on imaging a collection of
fluid is found to be located behind the stomach. His serum amylase is mildly elevated. Which of the following is the most
likely explanation?
A. Early fluid collection
B. Pancreatic abscess
C. Peripancreatic necrosis
D. Psuedocyst
E. Sterile necrosis
Answer: D
Psuedocysts are unlikely to be present less than 4 weeks after an attack of acute pancreatitis. However, they are more common at
this stage and are associated with a raised amylase.

Pancreatitis: sequelae

Peripancreatic fluid collections: Occur in 25% cases. Located in or near the pancreas and lack a wall of granulation or fibrous
tissue. May resolve or develop into pseudocysts or abscesses. Since most resolve aspiration and drainage is best avoided as it may
precipitate infection

Pseudocysts: in acute pancreatitis result from organisation of peripancreatic fluid collection. They may or may not communicate
with the ductal system. The collection is walled by fibrous or granulation tissue and typically occurs 4 weeks or more after an
attack of acute pancreatitis. Most are retrogastric. 75% are associated with persistent mild elevation of amylase. Investigation is
with CT, ERCP and MRI or Endoscopic USS. Symptomatic cases may be observed for 12 weeks as up to 50% resolve. Treatment
is either with endoscopic or surgical cystogastrostomy or aspiration

Pancreatic necrosis: Pancreatic necrosis may involve both the pancreatic parenchyma and surrounding fat. Complications are
directly linked to extent of parenchymal necrosis and extent of necrosis overall. Early necrosectomy is associated with a high
mortality rate (and should be avoided unless compelling indications for surgery exist). Sterile necrosis should be managed
conservatively (at least initially). Some centres will perform fine needle aspiration sampling of necrotic tissue if infection is
suspected. False negatives may occur and the extent of sepsis and organ dysfunction may be a better guide to surgery

Pancreatic abscess: Intra abdominal collection of pus associated with pancreas but in the absence of necrosis. Typically occur as
a result of infected pseudocyst. Transgastric drainage is one method of treatment, endoscopic drainage is an alternative

Haemorrhage: Infected necrosis may involve vascular structures with resultant haemorrhage that may occur de novo or as a
result of surgical necrosectomy. When retroperitoneal haemorrhage occurs Grey Turners sign may be identified

22. A 34 year old lady is admitted with pancreatitis. The aetiology is unclear and it is classified as an attack of moderate
severity according to the Glasgow criteria. Her imaging shows no gallstones and fluid around the pancreas. Which of the
following is the most appropriate initial management option?
A. Laparotomy
B. Laparoscopy
C. Radiological aspiration of the fluid
D. Active observation
E. Administration of octreotide
Answer: D
LEARN THIS!
Mnemonic for the assessment of the severity of pancreatitis: PANCREAS: P a02 < 60 mmHg. A ge > 55 years. N eutrophils > 15
x 10/l. C alcium < 2 mmol/l. R aised urea > 16 mmol/l. E nzyme (lactate dehydrogenase) > 600 units/l. A lbumin < 32 g/l. S ugar
(glucose) > 10 mmol/l
> 3 positive criteria indicates severe pancreatitis.

Acute early fluid collections are seen in 25% of patients with pancreatitis and require no specific treatment. Attempts at drainage
may introduce infection and result in pancreatic abscess formation.

23. A 43 year old lady presents with an attack of acute pancreatitis. It is classified as a mild attack on severity scoring.
Imaging identifies gallstones but a normal calibre bile duct, and a peripancreatic fluid collection. Which of the following
management options is most appropriate?
A. Intravenous octreotide
334
B. Cholecystectomy within 4 weeks
C. Nasogastric tube drainage of the stomach
D. Insertion of a radiological drain
E. Avoidance of enteral feeding
Answer: B
Patients with gallstone pancreatitis should undergo early cholecystectomy.
Enteral feeding helps minimise gut bacterial translocation and should be given to most patients with pancreatitis. Many studies
have evaluated the role of octreotide in reducing pancreatic secretions and shown no benefit (Uhl W et al Gut 1999 45:97-104,
McKay C et al. Int J Pancreatol 1997; 21: 13-19).
The use of antibiotics in pancreatitis is controversial. However, a recent Cochrane review has presented

24. A 43 year old lady presents with jaundice and is diagnosed as having a carcinoma of the head of the pancreas. Although
she is deeply jaundiced, her staging investigations are negative for metastatic disease. What is the best method of biliary
decompression in this case?
A. ERCP and placement of metallic stent
B. ERCP alone
C. ERCP and placement of plastic stent
D. Cholecystostomy
E. Choledochoduodenostomy
Answer: C
Metallic stents are contraindicated in resectable biliary disease
A plastic stent is the best option for biliary decompression in resectable disease. Surgical bypasses have no place in the
management of operable malignancy as a bridge to definitive surgery.

Pancreatic stents: Both benign and malignant biliary obstruction may be treated by placement of stents. These may be either
plastic tubes or self expanding metallic stents. They can be placed either percutaneously, at ERCP, or, less commonly now, open
surgery. Complications include blockage, displacement and those related to the method of insertion.

Metallic Vs Plastic stents


Metallic stents Plastic stents
Expensive Cheap
Embed in surrounding tissues Do not usually embed
Displacement rare Displacement common
Blockage rare Blockage common
Contraindicated in resectable malignant disease May be used as a bridge to resectional surgery
Theme: Management of biliary diseases

A. Acute laparoscopic cholecystectomy


B. Delayed laparoscopic cholecystectomy
C. Percutaneous cholecystostomy
D. Lithotripsy
E. Endoscopic retrograde cholangiopancreatography
F. Choledochoduodenostomy
G. Bile duct excision and hepatico-jejunostomy
H. Operative cholecystostomy

Please select the most appropriate management option for the scenario given. Each option may be used once, more than once or
not at all.

25. A 43 year old women is admitted with acute cholecystitis and fails to settle. A laparoscopic cholecystectomy is
performed, at operation the gallbladder has evidence of an empyema and Calots triangle is inflamed and the surgeon
suspects that a Mirizzi syndrome has occurred.

The correct answer is Operative cholecystostomy. This will address the acute sepsis and resolve the situation. Attempts at
completing the surgery at this stage, even in expert hands carries a very high risk of bile duct injury.

26. Following a difficult cholecystectomy a surgeon leaves a drain. 24 hours later bile is seen to be accumulating in the drain
and this fails to resolve over the next 48 hours. The patient is otherwise well.

Endoscopic retrograde cholangiopancreatography. This will delineate the presence of potential bile duct injury. Usually this is
result of leakage from the cystic duct and placement of a stent will allow free biliary drainage and the leak should settle.

335
27. A 40 year old woman is admitted with abdominal pain. She has suffered from repeated episodes of this colicky right
upper quadrant pain. On examination she is pyrexial with right upper quadrant peritonism. Her blood tests show a
white cell count of 23. However, the liver function tests are normal. An abdominal ultrasound scan shows multiple
gallstones in a thick walled gallbladder, the bile duct measures 4mm.

Acute laparoscopic cholecystectomy. This lady has acute cholecystitis and needs an acute cholecystectomy. This operation
should usually be performed within 48 hours of admission. Delay beyond this timeframe will usually result in increased
operative complications and most surgeons would administer antibiotics and perform and interval cholecystectomy if the early
window for an acute procedure is missed. A bile duct measuring 4mm is usually normal.

Theme: Pancreatitis management

A. Pancreatic necrosectomy
B. Staging laparotomy to assess severity
C. Endoscopic retrograde cholangiopancreatography
D. Emergency cystogastrostomy
E. Cholecystectomy within 4 weeks
F. Elective cystogastrostomy
G. Parenteral nutrition

Please select the most appropriate next stage in management for the scenario given. Each option may be used once, more than
once or not at all.

28. A 34 year old women is admitted with cholangitis. Her bilirubin is 180 and alkaline phosphatase is 348. She becomes
progressively more unwell and develops abdominal pain. The houseman checks her amylase which is elevated at 1080.
Standard treatment is initiated and her Glasgow score is 3.

Endoscopic retrograde cholangiopancreatography. She requires urgent decompression of her biliary system. An ERCP is the
conventional method of performing this. It is important to ensure that her coagulation status is normalised prior to performing
this procedure.

29. A 63 year old man is admitted to ITU with an attack of severe gallstone pancreatitis. He requires ventillatory support
for ARDS. Over the past few days he has become more unwell and a CT scan is organised. This demonstrates an area
of necrosis. His CRP is 400 and WCC 25.1.

Pancreatic necrosectomy. This man requires necrosectomy as he has infected pancreatic necrosis and is haemodynamically
unstable. A radiological drainage procedure is unlikely to be sufficient.

30. A 53 year old alcoholic develops acute pancreatitis and is making slow but reasonable progress. He is troubled by
persisting ileus and for this reason a CT scan is undertaken. This demonstrates a large pancreatic pseudocyst. This is
monitored by repeat CT scanning which shows no resolution and he is now complaining of early satiety.

Elective cystogastrostomy. Drainage of this man's pseudocyst is required. This could be accomplished radiologically or
endoscopically or surgically. As the other options are not on the list this is the best option from those available.

31. What proportion of patients presenting for cholecystectomy for treatment of biliary colic due to gallstones will have
stones in the common bile duct?
A. 10%
B. 30%
C. 2%
D. 50%
E. 25%
Answer: A
Up to 10% of all patients may have stones in the CBD. Therefore, all patients should have their liver function tests checked prior
to embarking on a cholecystectomy.

Colorectal Surgery
Theme: Proctology

A. Fissure in ano
B. Fistula in ano
C. Rectal prolapse
336
D. Juvenile polyps
E. Rectal adenoma
F. Intersphincteric abscess
G. Haemorroids

Please select the most likely underlying cause for the presentations described. Each option may be used once, more than once or
not at all.

1. A 21 year old female presents with a 24 hour history of increasingly severe ano-rectal pain. On examination she is febrile
and the skin surrounding the anus looks normal. She did not tolerate an attempted digital rectal examination.

The correct answer is Intersphincteric abscess. The presence of fever and severe pain makes an abscess more likely than a
fissure. Although fissures may be painful they do not, in themselves, cause fever. The usual management for this condition is
examination of the ano-rectum under general anaesthesia and drainage of the sepsis. Theme from September 2012 Exam

2. A 21 year old male presents with a 4 week history of frank, bright red, rectal bleeding. This typically occurs post
defecation into the toilet pan. He has a long standing history of constipation and a previous fissure in ano. On
examination the skin surrounding the anus is normal and digital rectal examination is normal.

Haemorroids. Haemorroids are a common cause of bright red rectal bleeding. The bleeding is typically painless. A history of
constipation is usual and may have been previously associated with a fissure (though this is less common). Haemorroids are not
always associated with external features and digital rectal examination is usually unremarkable.

3. A 21 year old lady presents with a 6 month history of an offensive discharge from the anus. She is otherwise well, but is
increasingly annoyed at the need to wear pads. On examination she has a small epithelial defect in the 5 o'clock position,
approximately 3cm from the anal verge.

Fistula in ano. Fistulas usually occur following previous ano-rectal sepsis. The discharge may be foul smelling and troublesome.
Patients should be listed for examination under anaesthesia. Fistulas which are low and have little or no sphincter involvement
are usually laid open.

Ano rectal disease


Location: 3, 7, 11 o'clock position
Haemorrhoids Internal or external
Treatment: Conservative, Rubber band ligation, Haemorrhoidectomy
Fissure in ano Location: midline 6 (posterior midline 90%) & 12 o'clock position. Distal to the dentate line
Chronic fissure > 6/52: triad: Ulcer, sentinel pile, enlarged anal papillae
Proctitis Causes: Crohn's, ulcerative colitis, Clostridium difficile
Ano rectal E.coli, staph aureus
abscess Positions: Perianal, Ischiorectal, Pelvirectal, Intersphincteric
Anal fistula Usually due to previous ano-rectal abscess
Intersphincteric, transsphincteric, suprasphincteric, and extrasphincteric. Goodsalls rule determines location
Rectal prolapse Associated with childbirth and rectal intussceception. May be internal or external
Pruritus ani Systemic and local causes
Anal neoplasm Squamous cell carcinoma commonest unlike adenocarcinoma in rectum
Solitary rectal Associated with chronic straining and constipation. Histology shows mucosal thickening, lamina propria
ulcer replaced with collagen and smooth muscle (fibromuscular obliteration)

Rectal prolapse: Common especially in multiparous women. May be internal or external. Internal rectal prolapse can present
insidiously. External prolapse can ulcerate and in long term impair continence. Diagnostic work up includes colonoscopy,
defecating proctogram, ano rectal manometry studies and if doubt exists and examination under anaesthesia.

Treatments for prolapse: In the acute setting reduce it (covering it with sugar may reduce swelling. Delormes procedure which
excises mucosa and plicates the rectum (high recurrence rates) may be used for external prolapse. Altmeirs procedure which
resects the colon via the perineal route has lower recurrence rates but carries the risk of anastamotic leak. Rectopexy is an
abdominal procedure in which the rectum is elevated and usually supported at the level of the sacral promontory. Post operative
constipation may be reduced by limiting the dissection to the anterior plane (laparoscopic ventral mesh rectopexy).

Pruritus ani: Extremely common. Check not secondary to altered bowel habits (e.g. Diarrhoea). Associated with underlying
diseases such as haemorrhoids. Examine to look for causes such as worms. Proctosigmoidoscopy to identify associated
haemorrhoids and exclude cancer. Treatment is largely supportive and patients should avoid using perfumed products around the
area.

337
Fissure in ano: Typically painful PR bleeding (bright red). Nearly always in the posterior midline. Usually solitary.

Treatment: Stool softeners. Topical diltiazem (or GTN). If topical treatments fail then botulinum toxin should be injected. If
botulinum toxin fails then males should probably undergo lateral internal sphincterotomy and females and advancement flap.

Theme: Management of colonic polypoidal lesions

A. Reassure and discharge


B. Pan proctocolectomy
C. Hot biopsy
D. Snare polypectomy
E. Segmental colonic resection
F. Repeat endoscopy at 3 years
G. Repeat endoscopy at 1 year
H. Repeat endoscopy at 5 years

Please select the most appropriate management for the scenario given. Each option may be used once, more than once or not at all.

4. A 43 year old man is investigated for altered bowel habit. At colonoscopy he is found to have a 2cm polyp on a long stalk
in the proximal sigmoid colon. The rest of the colonoscopy is normal. It bears no macroscopic features of malignancy.

Snare polypectomy. Polyps on long stalks are best managed by snare excision. It is important to retrieve the polyp for histology.

5. A 60 year old lady is investigated for abdominal pain. A polyp is identified at the proximal descending colon, three small
polyps are also noted in the sigmoid colon. The largest lesion is removed by snare polypectomy and the pathology report
states that this polyp is a low grade dysplastic adenoma measuring 3cm in diameter. The remaining lesions are ablated
using diathermy.

The correct answer is Repeat endoscopy at 1 year. She is at high risk of malignancy and should be closely followed up.
Fulguration of polyps without histology is unhelpful.

6. A 73 year old lady is investigated for anaemia. At colonoscopy she is found to have a flat broad based lesion in the
caecum. This is biopsied and the histology report states that these have diagnostic features of an adenoma with high
grade dysplasia.

The correct answer is Segmental colonic resection. This is most likely a malignancy and should be resected. Polypectomy of
flat broad lesions in the right colon is difficult and where concern arises a right hemicolectomy is probably the safest option.

Colonic Polyps: May occur in isolation of greater numbers as part of the polyposis syndromes. In FAP greater than 100 polyps
are typically present. The risk of malignancy in association with adenomas is related to size and is the order of 10% in a 1cm
adenoma. Isolated adenomas seldom give risk of symptoms (unless large and distal). Distally sited villous lesions may produce
mucous and if very large electrolyte disturbances may occur.

Follow up of colonic polyps:


Low risk: 1 or 2 adenomas <1cm. No follow up or re-colonoscopy at 5 years.
Moderate risk: 3 or 4 small adenomas or 1 adenoma >1cm. Re-scope at 3 years.
High risk: >5 small adenomas or >3 with 1 of them >1cm. Re scope at 1 year.
It is important to stratify patients appropriately and ensure that a complete colonoscopy with good views was performed.

Segmental resection or complete colectomy should be considered when:

1. Incomplete excision of malignant polyp


2. Malignant sessile polyp
3. Malignant pedunculated polyp with submucosal invasion
4. Polyps with poorly differentiated carcinoma
5. Familial polyposis coli
-Screening from teenager up to 40 years by 2 yearly sigmoidoscopy/colonoscopy
-Panproctocolectomy and Ileostomy or Restorative Panproctocolectomy.

Rectal polypoidal lesions may be amenable to trans anal endoscopic microsurger

338
Theme: Colonic resections

A. End ileostomy
B. Loop ileostomy
C. Ileo anal pouch
D. Loop colostomy
E. Pan proctocelectomy
F. Extended right hemicolectomy
G. Right hemicolectomy
H. Anterior resection
I. Anterior resection with covering loop ileostomy

Please select the most appropriate procedure from the list, each option may be used once, more than once or not at all.

7. A 75 year old man requires resection of an obstructing carcinoma of the splenic flexure.

The correct answer is Extended right hemicolectomy. Carcinoma of the splenic flexure requires extended right hemicolectomy.
Or a left hemicolectomy. The ileocolic anastomosis has a lower leak rate, particularly when the bowel is obstructed.

8. A patient presenting with a large bowel obstruction from a low rectal cancer.

Loop colostomy. This patient should be defunctioned-definitive surgery should wait until staging is completed. A loop
ileostomy will not satisfactorily decompress an acutely obstructed colon. Low rectal cancers that are obstructed should not
usually be primarily resected. The obstructed colon that would be used for anastomosis would carry a high risk of anastomotic
dehisence. In addition, as this is an emergency presentation, staging may not be completed, an attempted resection may
therefore compromise the circumferential resection margin, with an associated risk of local recurrence.

9. A 45 year old man presents with a carcinoma 10cm from the anal verge, he has completed a long course of
chemoradiotherapy and has achieved downstaging with no evidence of threatened circumferential margin on MRI
scanning.

The correct answer is Anterior resection with covering loop ileostomy. Low rectal cancer is usually treated with a low anterior
resection. Contraindications to this include involvement of the sphincters (unlikely here) and poor sphincter function that would
lead to unsatisfactory function post resection. Most colorectal surgeons defunction resections below the peritoneal reflection as
they have an intrinsically high risk of anastomotic leak. A loop ileostomy provides a safe an satisfactory method of
defunctioning these patients. A contrast enema should be performed prior to stoma reversal.

Colorectal cancer treatment: essentially this is surgical. Patients undergoing elective colonic resection are increasingly being
operated on laparoscopically and many centres now utilise enhanced recovery programmes. These encourage prompt recovery by
early mobilization, judicious administration of fluid, carbohydrate loading drinks on day of surgery, early resumption of normal
diet and avoidance of mechanical bowel preparation

In many elective cases mechanical bowel preparation can be avoided; this is universally true for right sided colonic surgery.
Controversy exists as to whether it is needed for left sided surgery.

Which operation is best?


The operations for cancer are segmental resections based on blood supply and lymphatic drainage. In the elective situation the
following operations are recommended:

Site of cancer Type of resection Anastomosis Risk of leak


Right colon Right hemicolectomy Ileo-colic Low <5%
Transverse Extended right hemicolectomy Ileo-colic Low <5%
Splenic flexure Extended right hemicolectomy Ileo-colic Low <5%
Splenic flexure Left hemicolectomy Colo-colon 2-5%
Left colon Left hemicolectomy Colo-colon 2-5%
Sigmoid colon High anterior resection Colo-rectal 5%
Upper rectum Anterior resection (TME) Colo-rectal 5%
Low rectum Anterior resection (Low TME) Colo-rectal 10%
(+/- Defunctioning stoma)
Anal verge Abdomino-perineal excision of colon and rectum None n/a

339
In the emergency setting where the bowel has perforated the risk of an anastomosis is much greater, particularly when the
anastomosis is colon-colon. In this situation an end colostomy is often safer and can be reversed later. When resection of the
sigmoid colon is performed and an end colostomy is fashioned the operation is referred to as a Hartmans procedure. Whilst left
sided resections are more risky, ileo-colic anastomoses are relatively safe even in the emergency setting and do not need to be
defunctioned.

10. Which of the following statements in relation to fistula in ano is untrue?


A. High fistulae are safest treated with a seton insertion
B. Low fistulae may be laid open
C. Are typically probed with Lockhart Mummary probes
D. When discovered during incision and drainage of peri anal abscess should always be probed to locate the internal opening
E. When complicating Crohns disease may respond to infliximab
Answer: D
Probing fistulae during acute sepsis is associated with a high complication rate and should not be undertaken routinely.

11. You embark on a laparoscopic appendicectomy and find an appendix mass. There is no free fluid and the patient has no
evidence of peritonitis. Which is the best option?
A. Convert to a midline laparotomy and perform a limited right hemicolectomy and end ileostomy
B. Convert to midline laparotomy and perform and appendicectomy after taking down the adhesions
C. Place a drain laparoscopically and administer parenteral antibiotics
D. Send the patient for CT guided drainage
E. Wrap omentum around the area and avoid drainage
Answer: C
Attempt conservative management for appendix mass without peritonitis.
In the absence of overt peritonitis minimalist approach to these is safest, they can always return to theatre in the event of clinical
deterioration.

Appendicitis: peri umbilical abdominal pain (visceral stretching of appendix lumen and appendix is mid gut structure) radiating
to the right iliac fossa due to localised parietal peritoneal inflammation. Vomit once or twice but marked and persistent vomiting
is unusual. Diarrhoea is rare. However, pelvic appendicitis may cause localised rectal irritation of some loose stools. A pelvic
abscess may also cause diarrhoea. Mild pyrexia is common - temperature is usually 37.5 -38oC. Higher temperatures are more
typical of conditions like mesenteric adenitis. Anorexia is very common. It is very unusual for patients with appendicitis to be
hungry.

Examination: generalised peritonitis if perforation has occurred or localised peritonism. Retrocaecal appendicitis may have
relatively few signs. Digital rectal examination may reveal boggy sensation if pelvic abscess is present, or even tenderness with a
pelvic appendix.

Diagnosis: typically raised inflammatory markers coupled with compatible history and examination findings should be enough to
justify appendicectomy. Urine analysis may show mild leucocytosis but no nitrites. Ultrasound is useful if females where pelvic
organ pathology is suspected. Although it is not always possible to visualise the appendix on ultrasound the presence of free fluid
(always pathological in males) should raise suspicion.

Treatment: Appendicectomy which can be performed via either an open or laparoscopic approach. Administration of
metronidazole reduces wound infection rates. Patients with perforated appendicitis require copious abdominal lavage. Patients
without peritonitis who have an appendix mass should receive broad spectrum antibiotics and consideration given to performing
an interval appendicectomy. Be wary in the older patients who may have either an underlying caecal malignancy or perforated
sigmoid diverticular disease.

Theme: Colonic obstruction

A. Active observation
B. Hartmans procedure
C. Defunctioning loop colostomy
D. Colonic stent insertion
E. Sub total colectomy and ileostomy
F. Water soluble contrast enema
G. Defunctioning loop ileostomy
H. Low anterior resection

340
Please select the most appropriate management option from the list below. Each option may be used once, more than once or not
at all.

12. A 59 year old man presents with symptoms and signs of absolute constipation. On investigation he is found to have large
bowel obstruction and an obstructing tumour of the distal sigmoid colon.

The correct answer is Colonic stent insertion.Colonic stents are indicated in the temporary treatment of malignant large bowel
obstruction. Lesions of the mid to low rectum are too low to stent. More proximal lesions are more suited to this treatment
modality. Complications include perforation at the time of stent insertion.

13. A 73 year old lady is admitted with colicky abdominal pain and vomiting. On examination she has a tense distended
abdomen and some mild right iliac fossa tenderness. Rectal examination is unremarkable. Abdominal x-ray shows
dilated large bowel with no small bowel dilatation. WCC is 15 but other blood tests are normal. At laparotomy she has a
tumour of the sigmoid colon. The caecum appears viable, however, the bowel is dilated.

The correct answer is Hartmans procedure. This is safest. Some may advocate on table lavage and primary anastomosis, this
option is not on the list.

14. A 65 year old man is admitted with colicky abdominal pain and vomiting. On examination he has a distended abdomen
which is soft. Digital rectal examination reveals a mass at the tip of the finger. Abdominal x-ray shows dilated large
bowel with no small bowel dilatation.

The correct answer is Defunctioning loop colostomy. He has an obstructing rectal cancer. His obstruction should be relieved. A
loop ileostomy is less desirable here as the history suggests a competent ileocaecal valve. The lesion would be too low for a
colonic stent.

Patients with suspected large bowel obstruction should have pseudobstruction excluded with CT scan, gastrograffin enema or
endoscopy prior to intervention.

15. A 28 year old male presents with painful, bright red, rectal bleeding. On examination he is found to have a posteriorly
sited, midline, fissure in ano. What is the most appropriate treatment?
A. Topical GTN paste
B. Sub lingual GTN paste
C. Anal stretch
D. Advancement flap
E. Tailored division of the external anal sphincter
Answer: A
Topical vasodilator therapy is the most commonly utilised treatment for fissure in ano. Surgical division of the internal anal
sphincter is a reasonable treatment option in a young male. Division of the external sphincter will almost certainly result in
incontinence and is not performed. Anal stretches were associated with a high rate of external sphincter injuries and have been
discontinued for this reason.

Anal fissures: are a common cause of painful, bright red, rectal bleeding.
Most fissures are idiopathic and present as a painful mucocutaneous defect in the posterior midline (90% cases). Fissures are more
likely to be anteriorly located in females, particularly if they are multiparous. Multiple fissures and those which are located at
other sites are more likely to be due to an underlying cause.
Diseases associated with fissure in ano include: crohns disease, tuberculosis and internal rectal prolapse

Diagnosis: In most cases the defect can be visualised as a posterior midline epithelial defect. Where symptoms are highly
suggestive of the condition and examination findings are unclear an examination under anaesthesia may be helpful. Atypical
disease presentation should be investigated with colonoscopy and EUA with biopsies of the area.

Treatment: Stool softeners are important as the hard stools may tear the epithelium and result in recurrent symptoms. The most
effective first line agents are topically applied GTN (0.2%) or Diltiazem (2%) paste. Side effects of diltiazem are better tolerated.
Resistant cases may benefit from injection of botulinum toxin or lateral internal sphincterotomy (beware in females).
Advancement flaps may be used to treat resistant cases.
Sphincterotomy produces the best healing rates. It is associated with incontinence to flatus in up to 10% of patients in the long
term.

341
16. A 73 year old lady presents with constipation and no organic disease is identified on investigation. Which of the
following types of laxatives works by direct bowel stimulation?
A. Magnesium sulphate
B. Lactulose
C. Potassium sodium tatrate
D. Methylcellulose
E. Sodium docusate
Answer: E
Laxatives
 Bulk forming laxatives: bran, psyllium and methylcellulose.
 Osmotic laxatives: magnesium sulphate, magnesium citrate, sodium phosphate, sodium sulphate, potassium, sodium
tatrate and polyethylene glycol
 Stimulant laxatives: docusates, bisacodyl, sodium picosulphate
senna, ricinoleic acid

17. A 53 year old man has a 2cm polyp identified and completely removed during a colonoscopy. Histology confirms a low
grade adenoma. What is the correct follow up?
A. Discharge.
B. Repeat endoscopy in 5 years.
C. Repeat endoscopy in 3 years.
D. Segmental resection of the affected area.
E. Barium enema at 5 years.
Answer: C
It would be unsafe to discharge. Follow up with barium enemas for polyps is counter intuitive.

Theme: Bowel cancer management

A. Loop colostomy
B. Loop ileostomy
C. Ileo-colic bypass
D. Hartman's procedure
E. Sub total colectomy
F. Right hemicolectomy
G. Left hemicolectomy
H. Abdomino-perineal excision of the colon and rectum
I. Anterior resection

Please select the most appropriate management option for the scenario given. Each option may be used once, more than once or
not at all.

18. A 67 year old man is admitted with acute abdominal pain. He has features of large bowel obstruction. At laparotomy he
has a carcinoma of the sigmoid colon and perforation of the caecum

The correct answer is Sub total colectomy. Separate resection of right colon and sigmoid is not a good idea

19. A 89 year old lady is admitted with large bowel obstruction. She has tenderness of the right side of her abdomen and CT
scanning shows a sigmoid lesion with liver metastasis. Her caecum measures 11cm.

The correct answer is Loop colostomy. A loop colostomy is the safest option. A stent would be ideal (but is not on the list).

20. A patient has a tumour 8cm from the anal verge. Staging investigations show localised disease only.

Anterior resection. This should be manageable with a low anterior resection. Loop colostomy remains the traditional method for
relieving inoperable large bowel obstruction. Colonic stents are becoming increasing popular alternatives, especially as a bridge
to surgery.

Theme: Causes of rectal bleeding

A. Ulcerative colitis proctitis


B. Diversion proctitis
C. Haemorrhoidal disease
342
D. Fissure in ano
E. Crohns Proctitis
F. Diverticular bleed
G. Ischaemic colitis
H. Rectal intussceception

Please select the most likely cause of bleeding for the scenario given. Each option may be used once, more than once or not at all.

21. A previously well 21 year old man is admitted with 2 week history of diarrhoea and passage of blood and mucous
rectally. He has previously undergone an ileocaecal resection in the past for an inflammatory bowel disorder and takes
mesalazine.

Crohns Proctitis. His previous right sided resection makes crohns disease the most likely scenario.

22. A 56 year old lady has undergone a Hartman's procedure for diverticulitis. 6 months post operatively she complains of
painless passage of blood stained mucous per rectum.

The correct answer is Diversion proctitis. Rectal diversion may result in proctitis.

23. A 74 year old lady has been admitted with sudden onset profuse dark red rectal bleeding. She was previously well. At the
time of assessment her bleeding had stopped but haemoglobin was 10.5.

The correct answer is Diverticular bleed. This pattern of sudden onset profuse bleeding is typical of diverticular bleeding. This
often ceases spontaneously.

Rectal bleeding is a common cause for patients to be referred to the surgical clinic. In the clinical history it is useful to try and
localise the anatomical source of the blood. Bright red blood is usually of rectal anal canal origin, whilst dark red blood is more
suggestive of a proximally sited bleeding source. Blood which has entered the GI tract from a gastro-duodenal source will
typically resemble malaena due to the effects of the digestive enzymes on the blood itself.

In the table below we give some typical bleeding scenarios together with physical examination findings and causation.
Cause Type of Features in history Examination findings
bleeding
Fissure in Bright red Painful bleeding that occurs post defecation Muco-epithelial defect usually in the midline
ano rectal bleeding in small volumes. Usually antecedent features posteriorly (anterior fissures more likely to be
of constipation due to underlying disease)
Haemorroids Bright red Post defecation bleeding noted both on toilet Normal colon and rectum. Proctoscopy may
rectal bleeding paper and drips into pan. May be alteration of show internal haemorrhoids. Internal
bowel habit and history of straining. No haemorrhoids are usually impalpable.
blood mixed with stool. No local pain.
Crohns Bright red or Bleeding that is accompanied by other Perineal inspection may show fissures or
disease mixed blood symptoms such as altered bowel habit, fistulae. Proctoscopy may demonstrate
malaise, history of fissures (especially indurated mucosa and possibly strictures. Skip
anterior) and abscesses. lesions may be noted at colonoscopy.
Ulcerative Bright red Diarrhoea, weight loss, nocturnal Proctitis is the most marked finding. Peri anal
colitis bleeding often incontinence, passage of mucous PR. disease is usually absent. Colonoscopy will
mixed with show continuous mucosal lesion.
stool
Rectal cancer Bright red Alteration of bowel habit. Tenesmus may be Usually obvious mucosal abnormality. Lesion
blood mixed present. Symptoms of metastatic disease. may be fixed or mobile depending upon
volumes disease extent. Surrounding mucosa often
normal, although polyps may be present.

Investigation: All patients presenting with rectal bleeding require digital rectal examination and procto-sigmoidoscopy as a
minimal baseline. Remember that haemorrhoids are typically impalpable and to attribute bleeding to these in the absence of
accurate internal inspection is unsatisfactory. In young patients with no other concerning features in the history a carefully
performed sigmoidoscopy that demonstrates clear haemorrhoidal disease may be sufficient. If clear views cannot be obtained then
patients require bowel preparation with an enema and a flexible sigmoidscopy performed. In those presenting with features of
altered bowel habit or suspicion of inflammatory bowel disease a colonoscopy is the best test. Patients with excessive pain who
are suspected of having a fissure may require an examination under general or local anaesthesia. In young patients with external
stigmata of fissure and a compatible history it is acceptable to treat medically and defer internal examination until the fissure is
healed. If the fissure fails to heal then internal examination becomes necessary along the lines suggested above to exclude internal
disease.
343
Special tests: in patients with a malignancy of the rectum the staging investigations comprise an MRI of the rectum to identify
circumferential resection margin compromise and to identify mesorectal nodal disease. In addition to this CT scanning of the chest
abdomen and pelvis is necessary to stage for more distant disease. Some centres will still stage the mesorectum with endo rectal
ultrasound but this is becoming far less common. Patients with fissure in ano who are being considered for surgical
sphincterotomy and are females who have an obstetric history should probably have ano rectal manometry testing performed
together with endo anal ultrasound. As this service is not universally available it is not mandatory but in the absence of such
information there are continence issues that may arise following sphincterotomy.

Management

Disease Management
Fissure in ano GTN ointment 0.2% or diltiazem cream applied topically is the usual first line treatment. Botulinum toxin
for those who fail to respond. Internal sphincterotomy for those who fail with botox, can be considered at the
botox stage in males.
Haemorroids Lifestyle advice, for small internal haemorrhoids can consider injection sclerotherapy or rubber band
ligation. For external haemorrhoids consider haemorrhoidectomy. Modern options include HALO procedure
and stapled haemorrhoidectomy.
Inflammatory Medical management- although surgery may be needed for fistulating Crohns (setons).
bowel disease
Rectal cancer Anterior resection or abdomino-perineal excision of the colon and rectum. Total mesorectal excision is now
standard of care. Most resections below the peritoneal reflection will require defunctioning ileostomy. Most
patients will require preoperative radiotherapy.

Theme: Diverticular disease management

A. Active observation
B. Colonoscopy acutely
C. Intravenous antibiotics
D. Abdominal CT Scan
E. Ultrasound scan
F. Defecating proctogram
G. Flexible sigmoidoscopy
H. Laparotomy

Please select the most appropriate immediate management for the diverticular presentations given. Each option may be used once,
more than once or not at all.

24. A 40 year old man with known diverticular disease diagnosed on colonoscopy 1 year previously is admitted with acute
abdominal pain. His abdomen is maximally tender in the left iliac fossa and he describes pneumaturia. His GP has been
giving him metronidazole for 2 days.

The correct answer is Abdominal CT Scan. A colovesical fistula has formed and CT will help to delineate the other
complications which may have occurred

25. An 83 year old lady with known diverticular disease is admitted with a brisk PR bleed. On assessment the bleeding is
settling and her abdomen is soft. Hb 10.2, other blood tests are normal

The correct answer is Active observation. Diverticular bleeds often settle spontaneously. Acute colonoscopy is rarely helpful.
She may require an elective endoscopy. Isolated diverticular bleeds without evidence of infection do not necessarily require
antibiotics.

26. A 72 year old man is admitted with large bowel obstruction and CT scan suggests diverticular stricture in the sigmoid
colon.

The correct answer is Laparotomy. The stricture could be benign or malignant and although a lumenal study to establish
aetiology the opportunity for that intervention has passed. 70% of diverticular bleeds will settle with conservative management.

Diverticular disease: diverticular disease is a common surgical problem. It consists of herniation of colonic mucosa through the
muscular wall of the colon. The usual site is between the taenia coli which vessels pierce the muscle to supply the mucosa.

344
Symptoms: altered bowel habit, bleeding and abdominal pain.

Complications: diverticulitis, haemorrhage, development of fistula, perforation and faecal peritonitis, perforation and
development of abscess and development of diverticular phlegmon.

Diagnosis: patients presenting in clinic will typically undergo either a colonoscopy or barium enema as part of their diagnostic
work up. Both tests will identify diverticular disease. It can be far more difficult to confidently exclude cancer, particularly in
diverticular strictures.

Acutely unwell surgical patients should be investigated in a systematic way. Plain abdominal films and an erect chest x-ray will
identify perforation. An abdominal CT scan with oral and intravenous contrast will help to identify whether acute inflammation is
present but also the presence of local complications such as abscess formation.

Severity Classification- Hinchey: I: Para-colonic abscess; II: Pelvic abscess; III: Purulent peritonitis; IV: Faecal peritonitis

Treatment: Increase dietary fibre intake. Mild attacks of diverticulitis may be managed conservatively with antibiotics. Peri
colonic abscesses should be drained either surgically or radiologically. Recurrent episodes of acute diverticulitis requiring
hospitalisation are a relative indication for a segmental resection. Hinchey IV perforations (generalised faecal peritonitis) will
require a resection and usually a stoma. This group have a very high risk of post operative complications and usually require HDU
admission. Less severe perforations may be managed by laparoscopic washout and drain insertion.

27. A 34 year old man presents with symptoms attributable to a fistula in ano. He is examined in the lithotomy position and
the external opening of the fistula is identified in the 7 o'clock position. At which of the following locations is the internal
opening most likely to be identified?
A. 7 o'clock
B. 12 o'clock
C. 9 o'clock
D. 3 o'clock
E. 6 o'clock
Answer: E
Goodsals rule: Anterior fistulae will tend to have an internal opening opposite the external opening. Posterior fistulae will tend to
have a curved track that passes towards the midline.
According to Goodsalls rule the track of a posteriorly sited fistula will track to the posterior midline (i.e. 6 o'clock)

Theme: Proctology

A. Haemorrhoids
B. Rectal intussceception
C. Fistula in ano
D. Fissure in ano
E. Peri-anal abscess
F. Solitary rectal ulcer
G. Marjolins ulcer

Please select the most likely disorder for the scenario given. Each option may be used once, more than once or not at all.

28. A 38 year old lady presents with symptoms of obstructed defecation that date back to the birth of her second child by
use of ventouse. She passes mucous and suffers from pelvic pain. Digital rectal examination and barium enema are
normal.

Rectal intussceception. Rectal intussceception (internal rectal prolapse) typically presents with symptoms of obstructed
defecation. The pathology is best demonstrated by a defecating procotogram rather than barium enema.

29. A 23 year old male presents with bright red rectal bleeding that occurs post defecation onto the toilet paper. He has been
suffering from severe pain associated with this. On examination there is a skin tag at 6 O'clock.

Fissure in ano. This is a typical story for fissure and should be treated with laxatives and topical vasodilator (eg GTN) in the
first instance.

30. A 19 year old male presents with bright red rectal bleeding that occurs post defecation onto the paper and into the pan.
Apart from constipation his bowel habit is normal. Digital rectal examination is normal.
345
Haemorrhoids. This is likely to be haemorrhoidal disease. A sigmoidoscopy should always be performed to exclude more
sinister pathology.

Theme: Management of inflammatory bowel disease

A. Ileo-anal pouch
B. Panproctocolectomy
C. Sub total colectomy
D. Hartmans procedure
E. Right hemicolectomy
F. Intravenous steroids
G. Infliximab
H. Proctectomy

Please select the most appropriate management option from the list. Each option may be used once, more than once or not at all.

31. A 20 year old man is admitted with bloody diarrhoea. He has been passing 10 stools per day, Hb-8.0, albumin-20. Stool
culture negative. Evidence of colitis on endoscopy. He has been on intravenous steroids for 5 days and has now
developed megacolon. His haemoglobin is falling and inflammatory markers are static.

The correct answer is Sub total colectomy. This man requires a sub total colectomy. Conservative management has failed.
Patients with ulcerative colitis should undergo colectomy if there is no significant improvement in 5-7 days after initiating
medical therapy if they have a severe attack of the disease.

32. A 19 year old lady has a long standing history of diarrhoea and weight loss. She is investigated with an upper gastro
intestinal endoscopy which is normal. A small bowel contrast study shows a terminal ileal stricture. A colonoscopy was
performed which was normal but the endoscopist was unable to intubate the terminal ileum. One week after the
colonoscopy she is admitted with small bowel obstruction. Steroids are administered but despite this she fails to improve.

Right hemicolectomy. It is likely that this lady has terminal ileal disease. Although , first presentation of Crohns disease is
usually managed with IV steroids, these have been trialled here and failed. A resection will remove the stricturing disease. If
proximal small bowel disease has not been excluded pre-operatively then this must be evaluated "on table" during surgery to
exclude other small bowel strictures.

33. A 28 year old man is reviewed in the clinic. He has suffered from Crohns disease for many years, he has recently
undergone a sub total colectomy. However, he has residual Crohns in his rectum and this is the cause of ongoing
symptoms. Medical therapy is proving ineffective.

Proctectomy. In Crohns patients who have rectal disease and a previous sub total colectomy, a proctectomy is the best option.
An ileo-anal pouch is contra indicated in Crohns as they may fistulate and have major post operative complications.

IBD: Ulcerative colitis Vs Crohns


Crohn's disease Ulcerative colitis
Distribution Mouth to anus Rectum and colon
Macroscopic Cobblestone appearance, apthoid ulceration Contact bleeding
changes
Depth of disease Transmural inflammation Superficial inflammation
Distribution Patchy Continuous
pattern
Histological Granulomas (non caseating epithelioid cell aggregates with Crypt abscesses, Inflammatory cells in the
features Langhans' giant cells) lamina propria

Surgical treatment: Ulcerative colitis: in UC the main place for surgery is when medical treatment has failed, in the emergency
setting this will be a sub total colectomy, end ileostomy and a mucous fistula. Electively it will be a pan proctocolectomy, an
ileoanal pouch may be a selected option for some. Remember that longstanding UC increases colorectal cancer risk.

Crohn's disease: unlike UC Crohn's patients need to avoid surgeons, minimal resections are the rule. They should not have

346
ileoanal pouches as they will do poorly with them. Management of Crohn's ano rectal sepsis is with a minimal approach, simply
drain sepsis and use setons to facilitate drainage. Definitive fistula surgery should be avoided.

34. A 19 year old man presents with painful rectal bleeding and is found to have an anal fissure. Which of the following is
least associated with this condition?
A. Leukaemia
B. Syphilis
C. Tuberculosis
D. Sickle cell disease
E. Crohn's disease
Answer: D

Anal fissures are associated with: sexually transmitted diseases (syphilis, hiv), inflammatory bowel disease (crohn's up to 50%),
leukaemia (25% of patients), tuberculosis and previous anal surgery.

35. A 53 year old man presents with a full thickness external rectal prolapse. Which of the following procedures would be
the most suitable surgical option?
A. Rectopexy
B. Delormes
C. Altmeirs
D. Thirsch tape
E. Abdomino-perineal excision of the rectum
Answer: A

As this man is young and has full thickness prolapse a rectopexy is the most appropriate procedure. It will give the lowest
recurrence rates. Theme from 2012 Exam

Rectal prolapse: rectal prolapse may be divided into internal and external prolapse. Patients with the former condition may have
internal intussceception of the rectum and present with constipation, obstructed defecation and occasionally faecal incontinence.
Patients with external rectal prolapse have a full thickness external protrusion of the rectum. Risk factors for the condition include
multiparity, pelvic floor trauma and connective tissue disorders.

Diagnosis: external prolapse is usually evident. Internal prolapse may be identified by defecating proctography and examination
under anaesthesia.
Sinister pathology should be excluded with endoscopy

Treatment: perineal approaches include the Delormes operation, this avoids resection and is relatively safe but is associated with
high recurrence rates. An Altmeirs operation involves a perineal excision of the sigmoid colon and rectum, it may be a more
effective procedure than a Delormes but carries the risk of anastomotic leak. Rectopexy - this is an abdominal procedure. The
rectum is mobilised and fixed onto the sacral promotary. A prosthetic mesh may be inserted. The recurrence rates are low and the
procedure is well tolerated (particularly if performed laparoscopically). Thirsch tape- this is a largely historical procedure and
involves encircling the rectum with tape or wire. It may be of use in a palliative setting.

36. A patient has an appendicectomy and a 1.2cm carcinoid tumour is identified in the tip of the appendix. What is the
most appropriate management?
A. Watchful waiting
B. Discharge
C. Right hemicolectomy
D. Limited ileocaecal resection
E. Radioisotope scan
Answer: B
Individuals with small carcinoids can be discharged (<2cm and limited to the appendix). Larger tumours should have a
radioisotope scan. Where the resection margin is positive or where the isotope scan suggests lymphatic metastasis a right
hemicolectomy should be performed.

Carcinoid syndrome: Carcinoid tumours secrete serotonin. Originate in neuroendocrine cells mainly in the intestine (midgut-
distal ileum/appendix). Can occur in the rectum, bronchi. Hormonal symptoms mainly occur when disease spreads outside the
bowel

Clinical features: onset: years, flushing face, palpitations, tricuspid stenosis causing dyspnea, asthma and severe diarrhoea
(secretory, persists despite fasting).

347
Investigation: 5-hiaa in a 24-hour urine collection, scintigraphy and CT scan.
Treatment: octreotide. Surgical removal.

Theme: Management of colonic disease

A. Reassure and discharge


B. Right hemicolectomy
C. Left hemicolectomy
D. Hot biopsy
E. Snare polypectomy
F. Single colonoscopy
G. Annual colonoscopy
H. Colonoscopy every 3-5 years
I. Panproctocolectomy

For each scenario given please select the most appropriate management option. Each option may be used once, more than once or
not at all.

37. A 25 year old male presents with altered bowel habit. He is known to have familial polyposis coli. A colonoscopy shows
widespread polyps, with high grade dysplasia in a polyp removed from the rectum.

Panproctocolectomy. There is a high risk of conversion to malignancy, therefore panproctocolectomy is the safest option.

38. A 19 year old female presents with colicky abdominal pain, bloating and alternating constipation/diarrhoea. Her
grandmother died from colon cancer at the age of 77 years. A digital rectal examination and general physical
examination are normal.

The correct answer is Reassure and discharge. This girl fulfills the Rome criteria for irritable bowel syndrome. Examination is
normal, therefore there is no indication for further investigation.

39. A 62 year old man is being investigated for iron deficiency anaemia. During a colonoscopy a flat polypoidal lesion is
identified in the caecum. Biopsies of this lesion demonstrate high grade dysplasia.

The correct answer is Right hemicolectomy. High grade dysplasia in a flat villous lesion of the right colon is highly likely to be
associated with an invasive lesion at this site. Hot biopsy of right sided colonic lesions is unwise an snare polypectomy would
be unlikely to remove the entire lesion.

Theme: Fistula management

A. No further action needed


B. Intravenous fluids
C. Intravenous fluids and nasogastric tube
D. Total parenteral nutrition and octreotide
E. Defunctioning stoma
F. Insertion of seton
G. Intravenous octreotide
H. Lay open fistula

What is the best management for the following types of fistula? Each option may be used once, more than once or not at all.

40. A 45 year old man develops a colocutaneous fistulae following reversal of a loop colostomy fashioned for the
defunctioning of an anterior resection. Pre-operative gastrograffin enema showed no distal obstruction or anastamotic
stricture.

The correct answer is No further action needed. Colocutaneous fistulae may occur as a result of anastomotic leakage following
loop colostomy reversal. In the absence of abdominal signs a laparotomy is not necessarily required. Signs of wound sepsis may
require antibiotics. Because there is not any distal obstruction (note normal pre-operative gastrograffin enema) these fistulae
will usually close spontaneously.

41. A 43 year old man has suffered from small bowel Crohns disease for 15 years. Following a recent stricturoplasty he
develops an enterocutaneous fistula which is high output. Small bowel follow through shows it to be 15 cm from the DJ
flexure. His overlying skin is becoming excoriated.
348
The correct answer is Total parenteral nutrition and octreotide. This man has a high output and anatomically high fistula. Drying
up the fistula with octreotide will not suffice, his nutrition is compromised and TPN will help.

42. A 33 year old lady presented with jaundice secondary to common bile duct stones. A cholecystectomy and common bile
duct exploration is performed and the bile duct closed over a T tube. Six weeks post operatively a T tube cholangiogram
is performed and shows no residual stones. The T tube is removed and five hours after removal a small amount of bile is
noted to be draining from the T tube site.

No further action needed. When the bile duct is closed over a T Tube the latex in the T tube encourages tract fibrosis. This
actually encourages a fistula to develop. The result is that when the tube is removed any bile which leaks will usually drain
through the tract. Provided that there are no residual stones in the duct the fistula will slowly close. Persistent high volume
drainage may be managed with ERCP and sphincterotomy. Fistulas are more likely to heal in the absence of distal lumenal
obstruction

Theme: Surgery for inflammatory bowel disease

A. Proctectomy
B. Anterior resection
C. Panproctocolectomy
D. Panproctocolectomy and ileoanal pouch
E. Sub total colectomy
F. Right hemicolectomy

Please select the most appropriate surgical modality for treating the inflammatory bowel disease scenarios described. Each option
may be used once, more than once or not at all.

43. A 22 year old man presents with his first presentation of ulcerative colitis. Despite aggressive medical management with
steroids, azathioprine and infliximab his symptoms remain unchanged and he has developed a megacolon.

The correct answer is Sub total colectomy. In patients with fulminant UC a sub total colectomy is the safest treatment option.
The rectum will be left in situ as resection of the rectum in these acutely unwell patients carries an extremely high risk of
complications.
44. A 22 year old lady has a long history of severe perianal Crohns disease with multiple fistulae. She is keen to avoid a
stoma. However, she has progressive disease and multiple episodes of rectal bleeding. A colonoscopy shows rectal disease
only and a small bowel study shows no involvement with Crohns.

Proctectomy. Severe rectal Crohns that has developed complications such as haemorrhage and multiple fistulae is usually best
managed with proctectomy. Although a diverting stoma may reduce the risk of local sepsis it is unlikely to reduce the bleeding.
She is keen to conserve a rectum, however, an ileoanal pouch in this setting is unwise.

45. A 22 year old man has a long history of ulcerative colitis. His symptoms are well controlled with steroids. However,
attempts at steroid weaning and use of steroid sparing drugs have repeatedly failed. He wishes to avoid a permanent
stoma.

Panproctocolectomy and ileoanal pouch. In patients with UC where medical management is not successful, surgical resection
may offer a chance of cure. Those patients wishing to avoid a permanent stoma may be considered for an ileoanal pouch.
However, this procedure is only offered in the elective setting.

Surgery for inflammatory bowel disease: patients with inflammatory bowel disease (UC and Crohns) frequently present in
surgical practice. Ulcerative colitis may be cured by surgical resection (Proctocolectomy), this is not the case in Crohns disease
which may recur and affect other areas of the gastrointestinal tract.

Ulcerative colitis: elective indications for surgery include disease that is requiring maximal therapy, or prolonged courses of
steroids. Longstanding UC is associated with a risk of malignant transformation. Dysplastic transformation of the colonic
epithelium with associated mass lesions is an absolute indication for a proctocolectomy. Emergency presentations of poorly
controlled colitis that fails to respond to medical therapy should usually be managed with a sub total colectomy. Excision of the
rectum is a procedure with a higher morbidity and is not generally performed in the emergency setting. An end ileostomy is
usually created and the rectum either stapled off and left in situ, or, if the bowel is very oedematous, may be brought to the surface
as a mucous fistula. Patients with IBD have a high incidence of DVT and appropriate thromboprophylaxis is mandatory.
Restorative options in UC include an ileoanal pouch. This procedure can only be performed whilst the rectum is in situ and cannot

349
usually be undertaken as a delayed procedure following proctectomy. Ileoanal pouch complications include, anastomotic
dehiscence, pouchitis and poor physiological function with seepage and soiling.

Crohns disease: Surgical resection of Crohns disease does not equate with cure, but may produce substantial symptomatic
improvement. Indications for surgery include complications such as fistulae, abscess formation and strictures. Extensive small
bowel resections may result in short bowel syndrome and localised stricturoplasty may allow preservation of intestinal length.
Staging of Crohns will usually involve colonoscopy and a small bowel study (e.g. MRI enteroclysis). Complex perianal fistulae
are best managed with long term draining seton sutures, complex attempts at fistula closure e.g. advancement flaps, may be
complicated by non healing and fistula recurrence. Severe perianal and / or rectal Crohns may require proctectomy, ileoanal
pouch reconstruction in Crohns carries a high risk of fistula formation and pouch failure and is not recommended. Terminal ileal
Crohns remains the commonest disease site and these patients may be treated with limited ileocaecal resections. Terminal ileal
Crohns may affect enterohepatic bile salt recycling and increase the risk of gallstones.

46. A 55 year old man develops an acute colonic pseudo-obstruction following a laminectomy. Despite correction of his
electrolytes and ongoing supportive care he fails to settle. Which of the drugs listed below may improve the situation?
A. Buscopan
B. Neostigmine
C. Metoclopramide
D. Mebevrine
E. Sodium picosulphate
Answer: B
Neostigmine affects the degradation of acetylcholine and will therefore stimulate both nicotinic and muscarinic receptors. It
may produce symptomatic bradycardia and should therefore only be administered in a monitored environment. In colonic
pseudo-obstruction it produces generalised colonic contractions and its onset is usually rapid.

Colonic pseudo-obstruction: this is characterised by the progressive and painless dilation of the colon. The abdomen may
become grossly distended and tympanic. Unless a complication such as impending bowel necrosis or perforation occurs, there is
usually little pain.
Diagnosis involves excluding a mechanical bowel obstruction with a plain film and contrast enema. The underlying cause is
usually electrolyte imbalance and the condition will resolve with correction of this and supportive care.
Patients who do not respond to supportive measures should be treated with attempted colonoscopic decompression and/ or the
drug neostigmine. In rare cases surgery may be required.

Theme: Causes of rectal bleeding

A. Fissure in ano
B. Intersphincteric abscess
C. Haemorroidal disease
D. Proctitis
E. Solitary rectal ulcer syndrome
F. Rectal cancer
G. Anal cancer

Please select the most likely cause of rectal bleeding from the list above. Each option may be used once, more than once or not at
all.

47. An 18 year old man with a previous history of constipation presents with bright red rectal bleeding and diarrhoea. He
has suffered episodes of faecal incontinence, which have occurred randomly throughout the day and night.

The correct answer is Proctitis. Nocturnal diarrhoea and incontinence are typical of inflammatory bowel disease.
48. A 56 year old man presents with episodes of pruritus ani and bright red rectal bleeding. On examination there is a mass
in the ano rectal region and biopsies confirm squamous cell cancer.

Anal cancer. These are features of anal cancer. Anal cancers arise from the cutaneous epithelium and are therefore typically
squamous cell. They are usually sensitive to chemoradiotherapy.

49. A 19 year old man presents with bright red rectal bleeding. He has a longstanding history of irritable bowel syndrome.
At flexible sigmoidoscopy a lesion is biopsied and reported as showing 'fibromuscular obliteration'.

Solitary rectal ulcer syndrome. This is the typical presentation of SRUS. These patients require careful diagnostic work up to
elicit the underlying cause of their altered bowel habit. The histological appearances of solitary rectal ulcers are characteristic
and extensive collagenous deposits are often seen. This is usually termed fibromuscular obliteration.
350
50. A 56 year old man presents with his first attack of diverticulitis. Which of the following complications is least likely to
ensue?
A. Formation of colonic strictures
B. Malignant transformation
C. Development of colovesical fistula
D. Formation of a pericolic abscess
E. Formation of a phlegmon
Answer: B
Diverticulitis may result in a number of complications. However, whilst malignant disease may coincide with diverticulitis it is not,
in itself, a risk factor for colonic cancer.

Theme: Cancer management

A. Right hemicolectomy and ileocolic anastomosis


B. Extended right hemicolectomy and ileocolic anastomosis
C. Left hemicolectomy and colon-colon anastomosis
D. High anterior resection and colo-rectal anastomosis
E. Anterior resection and colo-rectal anastomosis
F. Abdomino-perineal excision of colon and rectum
G. Hartman's procedure
H. Pan proctocolectomy

Please select the most appropriate operation for the scenario given. Each option may be used once, more than once or not at all.

51. A 28 year old man with a large (>5cm) appendiceal carcinoid tumour.

Right hemicolectomy and ileocolic anastomosis. This will require formal resection.

52. A 68 year old lady has an adenocarcinoma of the rectum that invades to the dentate line distally.

Abdomino-perineal excision of colon and rectum. Intestinal continuity cannot be restored here.

53. Emergency resection of perforated sigmoid tumour.

Hartman's procedure

54. A 34 year old lady presents to her general practitioner with peri anal discomfort. The general practitioner diagnoses
pruritus ani, which of the following is least associated with the condition?
A. Hyperbilirubinaemia
B. Anal fissure
C. Leukaemia
D. Syphilis
E. Tuberculosis
Answer: E

Systemic (DM, Hyperbilirubinaemia, aplastic anaemia). Mechanical (diarrhoea, constipation, anal fissure). Infections (STDs).
Dermatological. Drugs (quinidine, colchicine). Topical agents

Theme: Fistula

A. Recurrent urinary tract infection secondary to catheter


B. Nephroenteric fistula
C. Enterovesical fistula
D. High Enterocutaneous fistula
E. Low Enterocutaneous fistula
F. Enteroenteric fistula
G. Colocutaneous fistula
H. Wound infection

Please select the most likely diagnosis for the scenario given. Each option may be used once, more than once or not at all.

351
55. You are the specialist trainee asked to review a 39 year old man post gastrectomy for bleeding duodenal ulcers. He is
hypotensive and tachycardic. His drain has increased output, contains pus and has bubbles. There is excoriated skin
around the drain site.

The correct answer is High Enterocutaneous fistula. Suspect an enterocutaneous fistula if there is excessive drainage and
bubbles. Pus may confuse surgeons, leading them to make a diagnosis of wound infection. If there is any uncertainty, methylene
blue can be given. If methylene blue is found in the drain, this confirms a fistula.

56. A 43 year old female presents with recurrent urinary tract infections. She describes blood and frothy urine. She is 6
weeks post operative for a left hemicolectomy for crohn's disease.

Enterovesical fistula. A nephroenteric fistula causes a chronic urinary tract infection, hence develops over a longer time frame.

57. A 2 week infant has faeculent material discharging from the umbilicus.

The correct answer is Low Enterocutaneous fistula. This baby has an enterocutaneous fistula at the umbilicus due to complete
failure of the omphalomesenteric duct to obliterate. This is treated with resection.

58. A 28 year old male presents with a discharging sinus in his natal cleft. He is found to have a pilonidal sinus. Which
statement is false?
A. Can occur in webs of fingers and the axilla
B. After drainage pilonidal abscesses should not be primarily closed
C. A rare complication is squamous cell carcinoma
D. In a patient with an acute abscess the Bascoms procedure is the treatment of choice.
E. Treatment involves excising or laying open the sinus tract.
Answer: D

When performing incision and drainage for pilonidal abscess try to avoid making the incision in the midline of the natal cleft.
Acute pilonidal abscesses should receive simple incision and drainage. Definitive treatments such as a Bascoms procedure should
not be undertaken when acute sepsis is present.

Pilonidal sinus: occur as a result of hair debris creating sinuses in the skin (Bascom theory). Usually in the natal cleft of male
patients after puberty. It is more common in Caucasians related to their hair type and growth patterns. The opening of the sinus is
lined by squamous epithelium, but most of its wall consists of granulation tissue. Up to 50 cases of squamous cell carcinoma has
been described in patients with chronic pilonidal sinus disease. Hairs become trapped within the sinus. Clinically the sinus
presents when acute inflammation occurs, leading to an abscess. Patients may describe cycles of being asymptomatic and periods
of pain and discharge from the sinus. Treatment is difficult and opinions differ. Definitive treatment should never be undertaken
when acute infection or abscess is present as this will result in failure. Definitive treatments include the Bascom procedure with
excision of the pits and obliteration of the underlying cavity. The Karydakis procedure involves wide excision of the natal cleft
such that the surface is recontoured once the wound is closed. This avoids the shearing forces that break off the hairs and has
reasonable results.

Theme: Colonic resections

A. Right hemicolectomy and ileocolic anastomosis


B. Extended right hemicolectomy and ileocolic anastomosis
C. Hartman's procedure
D. Defunctioning loop ileostomy only
E. Anterior resection and colo-rectal anastomosis
F. Abdomino-perineal excision of colon and rectum

Please select the most appropriate resection for the scenario given. Each option may be used once, more than once or not at all.

59. A 58 year old man with a tumour of the splenic flexure that requires resection.

Extended right hemicolectomy and ileocolic anastomosis. Beware of the anatomy at the base of the middle colic which will
require high ligation.

60. A 63 year old man presents with a carcinoma of the upper rectum. Staging investigations demonstrate localised disease
and he is not deemed to require and neo adjuvent therapy.

352
Anterior resection and colo-rectal anastomosis. Upper rectal tumours are usually amenable to restoration of intestinal continuity
and therefore an anterior resection is a reasonable treatment option.

61. A 66 year old lady presents with a tumour of the low rectum. There is a projection inferior to within 1cm of the dentate
line.

Abdomino-perineal excision of colon and rectum. Low rectal tumours such as this will require APER.

Theme: Colonic resections

A. End ileostomy
B. Loop ileostomy
C. Ileo anal pouch
D. Loop colostomy
E. Panproctocolectomy
F. Extended right hemicolectomy
G. Right hemicolectomy
H. Anterior resection
I. Anterior resection with covering loop ileostomy

Please select the most appropriate operation from the list, each option may be used once, more than once or not at all.

62. A 75 year old man with a rectal cancer at 6 cm from the anal verge. Staging shows localised disease.

The correct answer is Anterior resection with covering loop ileostomy. Low anterior resection and covering loop ileostomy.
These resections have a 5% leak rate and should be defunctioned.

63. A 65 year old man with carcinoma of the rectosigmoid junction, staging shows localised disease.

The correct answer is Anterior resection. An anterior resection is the operation of choice. The distal resection margin will be
higher than in the first case and so will not routine require defunctioning

64. A 29 year old man with ulcerative colitis. He has previously had a sub total colectomy but still suffers from proctitis. He
is keen to avoid a long term stoma.

Ileo anal pouch. This man should be considered for an ileoanal pouch

Theme: Colonic resections

A. Loop colostomy
B. Loop ileostomy
C. Colonic stent
D. Hartmann's procedure
E. Sub total colectomy
F. Right hemicolectomy
G. Left hemicolectomy
H. Abdomino-perineal excision of the colon and rectum
I. Anterior resection

Please select the most appropriate management for the colonic case indicated. Each option may be used once, more than once or
not at all.

65. A 68 year old man is admitted with large bowel obstruction. On examination he has a large tumour palpable 4cm from
the anal verge. Imaging shows a caecal diameter of 10cm and no evidence of perforation.

The correct answer is Loop colostomy. This patient needs to be defunctioned. The tumour is too low for a colonic stent. Primary
resection in this group is unwise as CRM may be involved.

66. A 72 year old lady is admitted with large bowel obstruction. On examination she has an empty rectum. CT scan shows a
10cm caecum and a tumour present at the rectosigmoid junction.

353
The correct answer is Colonic stent. Stenting will avoid the need for emergency surgery and once she has been stabilised a
primary resection and anastomosis performed

67. A 65 year old patient is admitted with acute abdominal pain. An erect CXR shows free air. At laparotomy a perforated
sigmoid cancer is found. There is no evidence of metastatic disease.

Hartmann's procedure. A Hartman's procedure is standard. A primary anastomosis in this setting would be unsafe (high risk of
leak). Rectal cancer with a threatened CRM should be downstaged with chemoradiotherapy prior to resection

68. A 23 year old lady presents with a posteriorly sited fissure in ano. Treatment with stool softeners and topical GTN has
failed to improve matters. Which of the following would be the most appropriate next management step?
A. Lords procedure
B. Fissurectomy and injection of botulinum toxin
C. Lateral internal sphincterotomy
D. Endoanal advancement flap
E. Surgical division of the external anal sphincter
Answer: B
The next most appropriate management option when GTN or other topical nitrates has failed is to consider botulinum toxin
injection. In males a lateral internal sphincterotomy would be an acceptable alternative. In a female who has yet to conceive this
may predispose to delayed increased risk of sphincter dysfunction. Division of the external sphincter will result in faecal
incontinence and is not a justified treatment for fissure.

Theme: Gastro intestinal haemorrhage.

A. Proctoscopy and injection sclerotherapy


B. IV terlipressin
C. Pan Proctocolectomy
D. Sub total colectomy
E. Colonscopy and bleeding therapy
F. Barium enema
G. Angiography of mesenteric artery

Please select the most appropriate management option for the scenario given. Each option may be used once more than once or
not at all

46. A 56 year old man is admitted with passage of a large volume of blood per rectum. On examination he is tachycardic, his
abdomen is soft, although he has marked dilated veins on his abdominal wall. Proctoscopy reveals large dilated veins
with stigmata of recent haemorrhage.

The correct answer is IV terlipressin. Rectal varices are a recognised complication of portal hypertension. In the first instance
they can be managed with medical therapy to lower pressure in the portal venous system. TIPSS may be considered. Whilst
band ligation is an option, attempting to inject these in same way as haemorroids would carry a high risk of precipitating further
haemorrhage.

47. A 73 year old lady is admitted with dark red PR bleeding. She undergoes an OGD which is normal. Digital rectal
examination shows blood but no masses. She becomes tachycardic and BP is 95/40.

Angiography of mesenteric artery. This women is actively bleeding and mesenteric angiography may localise the bleeding.
Colonoscopy in this situation is seldom helpful or successful.

48. A 68 year old man with ulcerative colitis is admitted with an exacerbation. You are called to see him because he is having
brisk PR bleeding. He has been on Intravenous hydrocortisone for 5 days. The gastroenterologists have done an OGD to
exclude a duodenal ulcer, this was normal.

The correct answer is Sub total colectomy. This man requires surgery to remove the bleeding segment of bowel. Medical
management has failed here.

Post-Operative Care

354
1. A 44 year old man undergoes a distal gastrectomy for cancer. He is slightly anaemic and therefore receives a transfusion
of 4 units of packed red cells to cover both the existing anaemia and associated perioperative blood loss. He is noted to
develop ECG changes that are not consistent with ischaemia. What is the most likely cause?
A. Hyponatraemia
B. Hyperkalaemia
C. Hypercalcaemia
D. Metabolic alkalosis
E. Hypernatraemia
Answer: B
The transfusion of packed red cells has been shown to increase serum potassium levels. The risk is higher with large
volume transfusions and with old blood.
Blood transfusion reactions: Immune mediated: Pyrexia. Alloimmunization. Thrombocytopaenia. Transfusion associated lung
injury. Graft vs Host disease. Urticaria. Acute or delayed haemolysis. ABO incompatibility. Rhesus incompatibility.
Non immune mediated: Hypocalcaemia. CCF. Infections. Hyperkalaemia

Notes: GVHD: lymphocyte proliferation causing organ failure. Transfusion associated lung injury: neutrophil mediated allergic
pulmonary oedema. ABO and Rhesus incompatibility: causes acute haemolytic transfusion reaction leading to agglutination and
haemolysis

2. Which of the following muscle relaxants will tend to incite neuromuscular excitability following administration?
A. Atracurium
B. Suxamethonium
C. Vecuronium
D. Pancuronium
E. None of the above
Answer: B
Suxamethonium may induce generalised muscular contractions following administration. This may raise serum potassium
levels.

Muscle relaxants: Suxamethonium: Depolarising neuromuscular blocker. Inhibits action of acetylcholine at the neuromuscular
junction. Degraded by plasma cholinesterase and acetylcholinesterase. Fastest onset and shortest duration of action of all muscle
relaxants. Produces generalised muscular contraction prior to paralysis. Adverse effects include hyperkalaemia, malignant
hyperthermia and lack of acetylcholinesterase

Atracurium: Non depolarising neuromuscular blocking drug. Duration of action usually 30-45 minutes. Generalised histamine
release on administration may produce facial flushing, tachycardia and hypotension. Not excreted by liver or kidney, broken down
in tissues by hydrolysis. Reversed by neostigmine

Vecuronium: Non depolarising neuromuscular blocking drug. Duration of action approximately 30 - 40 minutes. Degraded by
liver and kidney and effects prolonged in organ dysfunction. Effects may be reversed by neostigmine

Pancuronium: Non depolarising neuromuscular blocker. Onset of action approximately 2-3 minutes. Duration of action up to 2
hours. Effects may be partially reversed with drugs such as neostigmine

3. A 23 year old man is recovering from an appendicectomy. The operation was complicated by the presence of
perforation. He is now recovering on the ward. However, his urine output is falling and he has been vomiting. Which of
the following intravenous fluids should be initially administered, pending analysis of his urea and electrolyte levels?
A. Hartmans solution
B. Dextran 70
C. Pentastarch
D. Gelofusin
E. 5% Dextrose
Answer: A
He will have sequestration of electrolyte rich fluids in the abdomen and gut lumen. These are best replaced by use of Hartmans
solution in the first instance. Theme from January 2011 Exam.
Post operative fluid management: Composition of commonly used intravenous fluids mmol-1
Na K Cl Bicarbonate Lactate
Plasma 137-147 4-5.5 95-105 22-25 -
0.9% Saline 153 - 153 - -
Dextrose / saline 30.6 - 30.6 - -
Hartmans 130 4 110 - 28

355
A summary of the recommendations for post operative fluid management: Fluids given should be documented clearly and
easily available. Assess the patient's fluid status when they leave theatre. If a patient is haemodynamically stable and euvolaemic,
aim to restart oral fluid intake as soon as possible. Review patients whose urinary sodium is < 20. If a patient is oedematous,
hypovolaemia if present should be treated first. This should then be followed by a negative balance of sodium and water,
monitored using urine Na excretion levels. Solutions such as Dextran 70 should be used in caution in patients with sepsis as there
is a risk of developing acute renal injury.

4. A 32 year old man presents to the acute surgical unit with acute pancreatitis. He suddenly becomes dyspnoeic and his
saturations are 89% on air. A CXR shows bilateral pulmonary infiltrates. His CVP pressure is 16mmHg. What is the
most likely diagnosis?
A. Pulmonary oedema
B. Pneumococcal pneumonia
C. Staphylococcal pneumonia
D. Pneumocystis carinii
E. Adult respiratory distress syndrome
Answer: E
Acute pancreatitis is known to precipitate ARDS. ARDS is characterised by bilateral pulmonary inflitrates and hypoxaemia.
Note that pulmonary oedema is excluded by the CVP reading < 18mmHg. Theme from January 2012 Exam

Adult respiratory distress syndrome : Defined as an acute condition characterized by bilateral pulmonary infiltrates and severe
hypoxemia (PaO2/FiO2 ratio < 200) in the absence of evidence for cardiogenic pulmonary oedema (clinically or pulmonary
capillary wedge pressure of less than 18 mm Hg).
In is subdivided into two stages. Early stages consist of an exudative phase of inury with associated oedema. The later stage is one
of repair and consists of fibroproliferative changes. Subsequent scarring may result in poor lung function.

Causes: Sepsis. Direct lung injury. Trauma. Acute pancreatitis. Long bone fracture or multiple fractures (through fat embolism).
Head injury (causes sympathetic nervous stimulation which leads to acute pulmonary hypertension)

Clinical features: Acute dyspnoea and hypoxaemia hours/days after event. Multi organ failure

Management: Treat the underlying cause. Antibiotics. Negative fluid balance i.e. Diuretics. Mechanical ventilation strategy using
low tidal volumes as conventional tidal volumes may cause lung injury (only treatment found to improve survival rates)

5. Which of the anaesthetic agents below is most likely to induce adrenal suppression?
A. Sodium thiopentone
B. Midazolam
C. Propofol
D. Etomidate
E. Ketamine
Answer: D
Etomidate is a recognised cause of adrenal suppression, this has been associated with increased mortality when used as a
sedation agent in the critically ill.

Anaesthetic agents :Below are some of the more commonly used IV induction agents

Propofol: Rapid onset of anaesthesia. Pain on IV injection. Rapidly metabolised with little accumulation of metabolites. Proven
anti emetic properties. Moderate myocardial depression. Widely used especially for maintaining sedation on ITU, total IV
anaesthesia and for daycase surgery

Sodium thiopentone: Extremely rapid onset of action making it the agent of choice for rapid sequence of induction. Marked
myocardial depression may occur. Metabolites build up quickly. Unsuitable for maintenance infusion. Little analgesic effects

Ketamine: May be used for induction of anaesthesia. Has moderate to strong analgesic properties. Produces little myocardial
depression making it a suitable agent for anaesthesia in those who are haemodynamically unstable. May induce state of
dissociative anaesthesia resulting in nightmares

Etomidate: Has favorable cardiac safety profile with very little haemodynamic instability. No analgesic properties. Unsuitable for
maintaining sedation as prolonged (and even brief) use may result in adrenal suppression. Post operative vomiting is common

356
6. A patient with tachycardia and hypotension is to receive inotropes. Which of the following conditions are most likely to
be treated with inotropes?
A. Hypovolaemic shock
B. Septic shock
C. Neurogenic shock
D. Cardiogenic shock
E. None of the above
Answer: B
The term septic shock has a precise meaning and refers to refractory systemic arterial hypotension in spite of fluid resuscitation.
Patients will therefore usually require inotropes. Individuals suffering from neurogenic shock will usually receive intravenous
fluids to achieve a mean arterial pressure of 90mmHg. If this target cannot be achieved then these patients will receive inotropes.
Hypovolaemic shock requires fluids and the management of cardiogenic shock is multifactorial and includes inotropes,
vasodilators and intra-aortic balloon pumps. Theme from April 2012 Exam

Effects of receptor binding Inotropes and cardiovascular receptors


α-1, α-2 vasoconstriction Inotrope Cardiovascular receptor action
β-1 increased cardiac contractility and HR Adrenaline α-1, α-2, β-1, β-2
β-2 vasodilatation Noradrenaline α-1,( α-2), (β-1), (β-2)
D-1 renal and spleen vasodilatation Dobutamine β-1, (β 2)
D-2 inhibits release of noradrenaline Dopamine (α-1), (α-2), (β-1), D-1,D-2
Minor receptor effects in brackets

Theme: Feeding options

A. Feeding jejunostomy
B. Percutaneous endoscopic gastrostomy
C. Total parenteral nutrition
D. Naso gastric feeding tube
E. Naso jejunal feeding tube
F. Normal oral intake

Please select the most appropriate method of delivering nutrition in each of the following scenarios. Each option may be used
once, more than once or not at all.

7. A 28 year old man is comatose, from head injuries, on the neurosurgical intensive care unit. He is recovering well and
should be extubated soon.

Naso gastric feeding tube. The feeding of head injured patients was reviewed in a 2008 Cochrane report. They concluded that
the overall evidence base was poor. However, there was a trend for the enteral route, with NG feeding in the later stages
following injury. This is contra indicated if there are signs of basal skull fractures. Theme from April 2012 Exam

8. A 56 year old man has undergone a potentially curative oesophagectomy for carcinoma.

Feeding jejunostomy. Feeding jejunostomy is the standard of care in most centres. Naso jejunal tubes are preferred by some
surgeons. However, if they become displaced the only alternative then becomes TPN.

9. A 43 year old man is recovering from a laparoscopic low anterior resection with loop ileostomy.

The correct answer is Normal oral intake. Early feeding in this situation is both safe and will enhance recovery.

Nutrition options in surgical patients

Oral intake: Easiest option. May be supplemented by calorie rich dietary supplements. May contra indicated following certain
procedures

Naso gastric feeding: Usually administered via fine bore naso gastric feeding tube. Complications relate to aspiration of feed or
misplaced tube. May be safe to use in patients with impaired swallow. Often contra indicated following head injury due to risks
associated with tube insertion
357
Naso jejunal feeding: Avoids problems of feed pooling in stomach (and risk of aspiration). Insertion of feeding tube more
technically complicated (easiest if done intra operatively). Safe to use following oesophagogastric surgery

Feeding jejunostomy: Surgically sited feeding tube. May be used for long term feeding. Low risk of aspiration and thus safe for
long term feeding following upper GI surgery. Main risks are those of tube displacement and peritubal leakage immediately
following insertion, which carries a risk of peritonitis

Percutaneous endoscopic gastrostomy: Combined endoscopic and percutaneous tube insertion. May not be technically possible
in those patients who cannot undergo successful endoscopy. Risks include aspiration and leakage at the insertion site

Total parenteral nutrition: The definitive option in those patients in whom enteral feeding is contra indicated. Individualised
prescribing and monitoring needed. Should be administered via a central vein as it is strongly phlebitis. Long term use is
associated with fatty liver and deranged LFT's

Theme: Anaesthetic agents

A. Etomidate
B. Ketamine
C. Propofol
D. Sodium thiopentone
E. Methohexitone
F. Metaraminol
G. Midazolam

Please select the most appropriate anaesthetic induction agent for the procedure described. Each option may be used once, more
than once or not at all.

10. A 32 year old man is admitted for a trendelenberg procedure for varicose veins. He is known to have porphyria.

Propofol. This is a daycase procedure for which propofol is ideal. Sodium thiopentone and etomidate are contraindicated in
porphyria.

11. A 77 year old lady with unstable ischaemic heart disease requires an emergency femoral hernia repair. She is volume
depleted and slightly hypotensive.

The correct answer is Ketamine. Ketamine is not negatively inotropic and will not depress cardiac output. Propofol and Sodium
thiopentone will produce myocardial depression. Some doctors may also consider etomidate. However, it may cause adrenal
suppression and post operative vomiting- which she is at high risk of developing.

12. A 22 year old man is brought to theatre for an emergency apppendicectomy for generalised peritonitis. He is vomiting.

The correct answer is Sodium thiopentone. Most anaesthetists would use sodium thiopentone for a rapid sequence induction
(which this man will need). Propofol- Ideal agent for daycase- wears off rapidly, good antiemetic effect.

Sodium thiopentone- Fast onset of action- prone to accumulation. Depresses cardiac output.
Ketamine- Little haemodynamic instability. Good analgesic properties. Nightmares and restlessness.

The table below summarises some of the more commonly used IV induction agents

13. A 54-year-old man is admitted for an elective hip replacement. Three days post operatively you suspect he has had a
pulmonary embolism. He has no past medical history of note. Blood pressure is 120/80 mmHg with a pulse of 90/min.
The chest x-ray is normal. Following treatment with low-molecular weight heparin, what is the most appropriate initial
lung imaging investigation to perform?
A. Pulmonary angiography
B. Echocardiogram
C. MRI thorax
D. Ventilation-perfusion scan
E. Computed tomographic pulmonary angiography
Answer: E
CTPA is the first line investigation for PE according to current BTS guidelines

This is a difficult question to answer as both computed tomographic pulmonary angiography (CTPA) and ventilation-perfusion
358
scanning are commonly used in UK clinical practice. The 2003 British Thoracic Society (BTS) guidelines, however,
recommended that CTPA is now used as the initial lung imaging modality of choice. Pulmonary angiography is of course the 'gold
standard' but this is not what the question asks for

Pulmonary embolism: investigation: The British Thoracic Society (BTS) published guidelines in 2003 on the management of
patients with suspected pulmonary embolism (PE)

Key points from the guidelines include: computed tomographic pulmonary angiography (CTPA) is now the recommended
initial lung-imaging modality for non-massive PE. Advantages compared to V/Q scans include speed, easier to perform out-of-
hours, a reduced need for further imaging and the possibility of providing an alternative diagnosis if PE is excluded. If the CTPA
is negative then patients do not need further investigations or treatment for PE. Ventilation-perfusion scanning may be used
initially if appropriate facilities exist, the chest x-ray is normal, and there is no significant symptomatic concurrent
cardiopulmonary disease

Some other points: Clinical probability scores based on risk factors and history and now widely used to help decide on further
investigation/management

D-dimers: sensitivity = 95-98%, but poor specificity

V/Q scan: sensitivity = 98%; specificity = 40% - high negative predictive value, i.e. if normal virtually excludes PE. Other causes
of mismatch in V/Q include old pulmonary embolisms, AV malformations, vasculitis, previous radiotherapy. COPD gives
matched defects

CTPA: peripheral emboli affecting subsegmental arteries may be missed

Pulmonary angiography: the gold standard. Significant complication rate compared to other investigations

Theme: Surgical analgesia

A. Amitriptylline
B. Pregabalin
C. Duloxetine
D. Paracetamol
E. Diclofenac
F. Pethidine
G. Morphine

Please select the most appropriate analgesic modality for the scenario given. Each option may be used once, more than once or not
at all.

14. A 72 year old man attends vascular clinic after having an amputation 2 months ago. He is having difficulty sleeping at
night due to persistent tingling at the amputation site. He is known to have orthostatic hypotension.

Pregabalin. This patient has phantom limb pain which is a neuropathic pain. First line management is with amitriptylline or
pregabalin. However this patient has orthostatic hypotension, which is a side effect of amitriptylline, therefore pregabalin is the
treatment of choice.

15. A 64 year old type 2 diabetic is referred to vascular clinic with painful foot ulcers. His ABPI is 0.6. On further
questioning the patient reports a burning sensation in both of his feet.

The correct answer is Duloxetine. This NICE guidelines state that duloxetine should be used as a 1st line agent in diabetic
neuropathic pain.

16. A 24 year old man has had a fracture of the tibia after playing football. He arrives in the emergency room distressed and
in severe pain.

Morphine. This type of injury will require morphine. However, timely fracture splinting will have a significant analgesic effect.

359
Management of pain: World Health Organisation Analgesic Ladder: Initially peripherally acting drugs such as paracetamol or
non-steroidal anti-inflammatory drugs (NSAIDs) are given. If pain control is not achieved, the second part of the ladder is to
introduce weak opioid drugs such as codeine or dextropropoxyphene together with appropriate agents to control and minimise
side effects. The final rung of the ladder is to introduce strong opioid drugs such as morphine. Analgesia from peripherally acting
drugs may be additive to that from centrally-acting opioids and thus, the two are given together.

The World Federation of Societies of Anaesthesiologists (WFSA) Analgesic Ladder: For management of acute pain. Initially,
the pain can be expected to be severe and may need controlling with strong analgesics in combination with local anaesthetic
blocks and peripherally acting drugs. The second rung on the postoperative pain ladder is the restoration of the use of the oral
route to deliver analgesia. Strong opioids may no longer be required and adequate analgesia can be obtained by using
combinations of peripherally acting agents and weak opioids. The final step is when the pain can be controlled by peripherally
acting agents alone.

Local anaesthetics: Infiltration of a wound with a long-acting local anaesthetic such as Bupivacaine. Analgesia for several hours.
Further pain relief can be obtained with repeat injections or by infusions via a thin catheter. Blockade of plexuses or peripheral
nerves will provide selective analgesia in those parts of the body supplied by the plexus or nerves. Can either be used to provide
anaesthesia for the surgery or specifically for postoperative pain relief. Especially useful where a sympathetic block is needed to
improve postoperative blood supply or where central blockade such as spinal or epidural blockade is contraindicated.

Spinal anaesthesia: Provides excellent analgesia for surgery in the lower half of the body and pain relief can last many hours
after completion of the operation if long-acting drugs containing vasoconstrictors are used. Side effects of spinal anaesthesia
include: hypotension, sensory and motor block, nausea and urinary retention.

Epidural anaesthesia: An indwelling epidural catheter inserted. This can then be used to provide a continuous infusion of
analgesic agents. It can provide excellent analgesia. They are still the preferred option following major open abdominal
procedures and help prevent post operative respiratory compromise resulting from pain. Disadvantages of epidurals is that they
usually confine patients to bed, especially if a motor block is present. In addition an indwelling urinary catheter is required. Which
may not only impair mobility but also serve as a conduit for infection. They are contraindicated in coagulopathies.

Transversus Abdominal Plane block (TAP): In this technique an ultrasound is used to identify the correct muscle plane and
local anaesthetic (usually bupivicaine) is injected. The agent diffuses in the plane and blocks many of the spinal nerves. It is an
attractive technique as it provides a wide field of blockade but does not require the placement of any indwelling devices. There is
no post operative motor impairment. For this reason it is the preferred technique when extensive laparoscopic abdominal
procedures are performed. They will then provide analgesia immediately following surgery but as they do not confine the patient
to bed, the focus on enhanced recovery can begin sooner. The main disadvantage is that their duration of action is limited to the
half life of the local anaesthetic agent chosen. In addition some anaesthetists do not have the USS skills required to site the
injections.

Patient Controlled Analgesia (PCA): Patients administer their own intravenous analgesia and titrate the dose to their own end-
point of pain relief using a small microprocessor - controlled pump. Morphine is the most popular drug used.

Strong Opioids: Severe pain arising from deep or visceral structures requires the use of strong opioids

Morphine: Short half life and poor bioavailability. Metabolised in the liver and clearance is reduced in patients with liver disease,
in the elderly and the debilitated. Side effects include nausea, vomiting, constipation and respiratory depression. Tolerance may
occur with repeated dosage

Pethidine: Synthetic opioid which is structurally different from morphine but which has similar actions. Has 10% potency of
morphine. Short half life and similar bioavailability and clearance to morphine. Short duration of action and may need to be given
hourly. Pethidine has a toxic metabolite (norpethidine) which is cleared by the kidney, but which accumulates in renal failure or
following frequent and prolonged doses and may lead to muscle twitching and convulsions. Extreme caution is advised if
pethidine is used over a prolonged period or in patients with renal failure.

Weak opioids: Codeine: markedly less active than morphine, has predictable effects when given orally and is effective against
mild to moderate pain.

Non opioid analgesics: Mild to moderate pain.

Paracetamol: Inhibits prostaglandin synthesis. Analgesic and antipyretic properties but little anti-inflammatory effect. It is well
absorbed orally and is metabolised almost entirely in the liver. Side effects in normal dosage and is widely used for the treatment
of minor pain. It causes hepatotoxicity in over dosage by overloading the normal metabolic pathways with the formation of a toxic
metabolite.

360
NSAIDs: Analgesic and anti-inflammatory actions. Inhibition of prostaglandin synthesis by the enzyme Cyclooxygenase which
catalyses the conversion of arachidonic acid to the various prostaglandins that are the chief mediators of inflammation. All
NSAIDs work in the same way and thus there is no point in giving more than one at a time. NSAIDs are, in general, more useful
for superficial pain arising from the skin, buccal mucosa, joint surfaces and bone. Relative contraindications: history of peptic
ulceration, gastrointestinal bleeding or bleeding diathesis; operations associated with high blood loss, asthma, moderate to severe
renal impairment, dehydration and any history of hypersensitivity to NSAIDs or aspirin.

Neuropthic pain: National Institute of Clinical Excellence (UK) guidelines: First line: Amitriptyline (Imipramine if cannot
tolerate) or pregabalin. Second line: Amitriptyline AND pregabalin. Third line: refer to pain specialist. Give tramadol in the
interim (avoid morphine). If diabetic neuropathic pain: Duloxetine

17. A 17 year old man undergoes an elective right hemicolectomy. Post operatively he receives a total of 6 litres of 0.9%
sodium chloride solution, over 24 hours. Which of the following complications may ensue?
A. Hyperchloraemiac acidosis.
B. Hypochloraemic alkalosis
C. Hyperchloraemic alkalosis
D. Acute renal failure
E. None of the above
Answer: A
Excessive infusions of any intravenous fluid carry the risk of development of tissue oedema and potentially cardiac failure.
Excessive administration of sodium chloride is a recognised cause of hyperchloraemic acidosis and therefore Hartmans solution
may be preferred where large volumes of fluid are to be administered.

18. A 28 year old man with Crohn's disease has undergone a number of resections. His BMI is currently 18 and his albumin
is 18. He feels well but does have a small localised perforation of his small bowel. The gastroenterologists are giving
azathioprine. What is the most appropriate advice regarding feeding?
A. Nil by mouth
B. Nil by mouth and continuous intra venous fluids until surgery
C. Enteral feeding
D. Parenteral feeding
E. Nutritional supplements
Answer: D
This man is malnourished, although he is likely to require surgery it is best for him to be nutritionally optimised first. As he may
have reduced surface area for absorption and has a localised perforation TPN is likely to be the best feeding modality.

Parenteral feeding-NICE guidelines: Parenteral nutrition: NICE guidelines summary: Identify patients as malnourished
or at risk

Patients identified as being malnourished- BMI < 18.5 kg/m2. Unintentional weight loss of > 10% over 3-6/12. BMI < 20 kg/m2
and unintentional weight loss of > 5% over 3-6/12

AT RISK of malnutrition- eaten nothing or little > 5 days, who are likely to eat little for a further 5 days. Poor absorptive capacity.
High nutrient losses. High metabolism

Identify unsafe/inadequate oral intake OR a non functional GI tract/perforation/inaccessible

Consider parenteral nutrition: for feeding < 14 days consider feeding via a peripheral venous catheter. For feeding > 30 days
use a tunneled subclavian line. Continuous administration in severely unwell patients. If feed needed > 2 weeks consider changing
from continuous to cyclical feeding. Don't give > 50% of daily regime to unwell patients in first 24-48h

Surgical patients: if malnourished with unsafe swallow OR a non functional GI tract/perforation/inaccessible then consider peri
operative parenteral feeding.

19. A 51 year old man is shot in the abdomen and sustains a significant intra abdominal injury. A laparotomy, bowel
resection and end colostomy are performed. An associated vascular injury necessitates a 6 unit blood transfusion. He has
a prolonged recovery and is paralysed and ventilated for 2 weeks on intensive care. He receives total parenteral nutrition
and is eventually weaned from the ventilator and transferred to the ward. On reviewing his routine blood tests the
following results are noted: Full blood count: Hb 11.3 g/dl, Platelets 267 x 109/l, WBC 10.1 x109/l. Urea and electrolytes:

361
Na+ 131 mmol/l, K+ 4.6 mmol/l, Urea 2.3 mmol/l, Creatinine 78 µmol/l. Liver function tests: Bilirubin 25 µmol/l, ALP 445
u/l, ALT 89 u/l, γGT 103 u/l. What is the most likely underlying cause for the abnormalities noted?
A. Delayed type blood transfusion reaction
B. Bile leak
C. Anastomotic leak
D. Total parenteral nutrition
E. Gallstones
Answer: D
TPN is known to result in derangement of liver function tests. Although, cholestasis may result from TPN, it would be very
unusual for gallstones to form and result in the picture above. Blood transfusion reactions typically present earlier and with
changes in the haemoglobin and although they may cause hepatitis this is rare nowadays.

Total parenteral nutrition: Commonly used in nutritionally compromised surgical patients. Bags contain combinations of
glucose, lipids and essential electrolytes, the exact composition is determined by the patients nutritional requirements. Although it
may be infused peripherally, this may result in thrombophlebitis. Longer term infusions should be administered into a central vein
(preferably via a PICC line). Complications are related to sepsis, re-feeding syndromes and hepatic dysfunction.

20. Which statement regarding post operative cognitive impairment is true?


A. Use of Benzodiazepines preoperatively reduces long-term post operative cognitive dysfunction
B. Pain does not cause delirium
C. Delirium has no impact on length of hospital stay
D. A regional anaesthetic rather than a general anaesthetic is more likely to contribute to post operative cognitive impairment
E. Visual hallucinations are not a feature of delirium
Answer: A
Anaesthetic technique and Post operative cognitive impairment (POCD): Use of benzodiazepines preoperatively reduces long-
term POCD (9.9% vs. 5%). Do not stop drugs for cognitive function. Regional techniques reduce POCD in first week, but no
difference at 3 months

Postoperative cognitive management

Definition: Deterioration in performance in a battery of neuropsychological tests that would be expected in < 3.5% of controls
Or: Long term, possibly permanent disabling deterioration in cognitive function following surgery

Early POCD: Increasing age. GA rather than regional. Duration of anaesthesia. Reoperation. Postoperative infection

Late POCD: Increasing age. Emboli. Biochemical disturbances

21. Which statement is false about pethidine?


A. Has approximately 10% efficacy of morphine
B. Structurally similar to morphine
C. Pethidine has a toxic metabolite (norpethidine) which is cleared by the kidney
D. Pethidine is metabolized by the liver
E. Can be given intramuscularly
Answer: B
It has a different structure.

22. Which of the following anaesthetic agents is least likely to be associated with depression of myocardial contractility?
A. Propofol
B. Etomidate
C. Sodium thiopentone
D. Ether
E. None of the above
Answer: B
Of the agents mentioned, etomidate has the most favorable cardiac safety profile.

Theme: Intravenous fluids

A. Dextran 40
B. Human albumin solution 4.5%
362
C. Dextran 70
D. Dextrose 4%/ Saline 0.19%
E. Dextrose 5%
F. Hartmans solution
G. Dextrose 10%
H. Gelofusin

Please select the most appropriate intravenous fluid for the scenario given. Each option may be used once, more than once or not
at all.

23. A 45 year old lady with cirrhosis of the liver is recovering following an emergency para umbilical hernia repair. She has
been slow to resume oral intake and has been receiving regular boluses of normal saline for oliguria

The correct answer is Human albumin solution 4.5%. In patients who are hypoalbuminaemic the use of albumin solution may
help promote a diuresis and manage fluid overload.

24. A 23 year old lady is severely unwell with pyelonephritis. She is hypotensive and clinically has septic shock.

Gelofusin. In this situation gelofusin will remain in the circulation for a long time. Starches are not a popular choice in sepsis as
they have been shown to be an independent risk factor for the development of renal failure.

25. A 24 year old man is recovering from a right hemicolectomy for Crohns disease. He is oliguric and dehydrated owing to
a high output ileostomy. His electrolytes are normal.

The correct answer is Hartmans solution. Of the solutions given Hartmans is the most suitable. Consideration should also be
given to potassium supplementation. Starches increase risk of renal failure when used in septic shock.

Theme: Intravenous access

A. 14 G peripheral cannula
B. Intraosseous infusion
C. Triple lumen central line (internal jugular route)
D. Triple lumen central line (femoral vein route)
E. Swann Ganz Catheter
F. Swann Ganz Introducer (7G)
G. 22 G peripheral cannula
H. Hickman line

Please select the most appropriate modality of intravenous access for the scenario given. Each option may be used once, more than
once or not at all.

26. A 45 year old man with liver cirrhosis is admitted with a brisk upper GI bleed. Multiple infusions are required and he is
peripherally shut down.

The correct answer is Triple lumen central line (femoral vein route). A central line is the most sensible option. He is highly
likely to be coagulopathic and a femoral insertion route is safest in these circumstances. Multiple infusions and absence of
peripheral veins are the compelling indications for central access in this case.

27. A 3 year old is injured in a road traffic accident and is hypotensive and tachycardic due to a suspected splenic injury,
she is peripherally shut down.

Intraosseous infusion. Intraosseous infusions are the preferred route in this situation as peripheral cannulation will be difficult
and unreliable.

28. A 73 year old man with Dukes C colonic cancer requires a long course of chemotherapy. He has poor peripheral veins.

Hickman line. A Hickman line is the most reliable long term option. Most Hickman lines are inserted under local anaesthesia

363
with image guidance. They have a cuff that usually becomes integrated with the surrounding tissues. This requires a brief
dissection during line removal.

Intravenous access: Venous access: A number of routes for establishing venous access are available.

Peripheral venous cannula: Easy to insert with minimal morbidity. Wide lumen cannulae can provide rapid fluid infusions.
When properly managed infections may be promptly identified and the cannula easily re sited. Problems relate to their peripheral
sites and they are unsuitable for the administration of vaso active drugs, such as inotropes and irritant drugs such as TPN (except
in the very short term setting).

Central lines: Insertion is more difficult and most operators and NICE advocate the use of ultra sound. Coagulopathies may lead
to haemorrhage following iatrogenic arterial injury. Femoral lines are easier to insert and iatrogenic injuries easier to manage in
this site however they are prone to high infection rates. Internal jugular route is preferred. They have multiple lumens allowing for
administration of multiple infusions. The lumens are relatively narrow and thus they do not allow particularly rapid rates of
infusion.

Intraosseous access This is typically undertaken at the anteromedial aspect of the proximal tibia and provides access to the
marrow cavity and circulatory system. Although traditionally preferred in paediatric practice they may be used in adults and a
wide range of fluids can be infused using these devices.

29. Which of the following is a not a diagnostic criteria for brain death?
A. No response to sound
B. No corneal reflex
C. Absent oculo-vestibular reflexes
D. No response to supraorbital pressure
E. No cough reflex to bronchial stimulation
Ansewr: A
Brain death: Criteria for brain death: Fixed pupils which do not respond to sharp changes in the intensity of incident light. No
corneal reflex. Absent oculo-vestibular reflexes - no eye movements following the slow injection of at least 50ml of ice-cold water
into each ear in turn (the caloric test). No response to supraorbital pressure. No cough reflex to bronchial stimulation or gagging
response to pharyngeal stimulation. No observed respiratory effort in response to disconnection of the ventilator for long enough
(typically 5 minutes) to ensure elevation of the arterial partial pressure of carbon dioxide to at least 6.0 kPa (6.5 kPa in patients
with chronic carbon dioxide retention). Adequate oxygenation is ensured by pre-oxygenation and diffusion oxygenation during the
disconnection (so the brain stem respiratory centre is not challenged by the ultimate, anoxic, drive stimulus). The test should be
undertaken by two appropriately experienced doctors on two separate occasions.

Theme: Airway management

A. Cricothyroidotomy
B. Laryngeal mask
C. Endotracheal intubation
D. Tracheostomy
E. Oropharyngeal airway

Please select the most appropriate method of airway access for the scenario given. Each option may be used once, more than once
or not at all.

30. A 63 year old man has been on the intensive care unit for a week with adult respiratory distress syndrome complicating
acute pancreatitis. He has required ventilation and is still being mechanically ventilated.

Tracheostomy. Tracheostomy is often used to facilitate long term weaning. The percutaneous devices are popular. These
involve a seldinger type insertion of the tube. A second operator inserts a bronchoscope to ensure the device is not advanced
through the posterior wall of the trachea. Complications include damage to adjacent structures and bleeding (contra indication in
coagulopathy).

31. A 23 year old man is undergoing an inguinal hernia repair as a daycase procedure and is being given sevoflurane.

The correct answer is Laryngeal mask. This procedure will be associated with requirement for swift onset of anaesthesia and
recovery. Muscle paralysis is not required and this would an ideal case for laryngeal mask airway.

364
32. A 48 year old man is due to undergo a laparotomy for small bowel obstruction.

Endotracheal intubation. Patients who are due to undergo laparotomies for bowel obstruction have either been vomiting or at
high risk of regurgitation of gastric contents on induction of anaesthesia. A rapid sequence induction with cricothyroid pressure
applied to occlude the oesophagus is performed. A cuffed endotracheal tube is then inserted. Once correct placement of the ET
tube is confirmed the cricothyroid pressure can be removed.

Airway management: Oropharyngeal airway: Easy to insert and use. No paralysis required. Ideal for very short procedures.
Most often used as bridge to more definitive airway

Laryngeal mask: Widely used. Very easy to insert. Device sits in pharynx and aligns to cover the airway. Poor control against
reflux of gastric contents. Paralysis not usually required. Commonly used for wide range of anaesthetic uses, especially in day
surgery. Not suitable for high pressure ventilation (small amount of PEEP often possible)

Tracheostomy: Reduces the work of breathing (and dead space). May be useful in slow weaning. Percutaneous tracheostomy
widely used in ITU. Dries secretions, humidified air usually required

Endotracheal tube: Provides optimal control of the airway once cuff inflated. May be used for long or short term ventilation.
Errors in insertion may result in oesophageal intubation (therefore end tidal CO2 usually measured). Paralysis often required.
Higher ventilation pressures can be used

33. You are the cardiothoracic surgical registrar reviewing a patient referred for an aortic valve replacement. The 40-year-
old man is being investigated for progressive breathlessness in a previous respiratory clinic. The notes show he has
smoked for the past 25 years. Pulmonary function tests reveal the following: FEV1 1.4 L, FVC 1.7 L and FEV1/FVC
82%. What is the most likely explanation?
A. Asthma
B. Bronchiectasis
C. Kyphoscoliosis
D. Chronic obstructive pulmonary disease
E. Laryngeal malignancy
Answer: C
These results show a restrictive picture, which may result from a number of conditions including kyphoscoliosis. The other
answers cause an obstructive picture.
Pulmonary function tests

Pulmonary function tests can be used to determine whether a respiratory disease is obstructive or restrictive. The table below
summarises the main findings and gives some example conditions:

Obstructive lung disease Restrictive lung disease


FEV1 - significantly reduced FEV1 - reduced
FVC - reduced or normal FVC - significantly reduced
FEV1% (FEV1/FVC) - reduced FEV1% (FEV1/FVC) - normal or increased
Asthma Pulmonary fibrosis
COPD Asbestosis
Bronchiectasis Sarcoidosis
Bronchiolitis obliterans Acute respiratory distress syndrome
Infant respiratory distress syndrome
Kyphoscoliosis
Neuromuscular disorders
Theme: Management of pain

A. Paracetamol
B. Non steroidal anti inflammatory drugs
C. Fentanyl patch
D. Carbamazepine
E. Pregabalin
F. Duloxetine
G. Radiotherapy
H. Chemotherapy
I. Spinal block

365
For each scenario please select the most appropriate analgesic modality. Each option may be used once, more than once or not at
all.

34. A 52 year old man with prostate cancer is admitted to urology with urinary retention. He complains of back pain which
is not responding to ward analgesia. A lumbar xray confirms lumbar spine metastases.

Radiotherapy. This patient needs radiotherapy for pain relief. Bisphosphonates may also be effective.

35. A 42 year old woman complains of shooting pains in her left arm after a mastectomy.

The correct answer is Pregabalin. Pregabalin is the 1st line treatment described in the NICE guidelines.

36. A 2 year old boy is recovering following an uncomplicated appendicectomy.

Paracetamol. Paracetamol is an extremely effective analgesic in children.

37. Which of the following statements relating to the use of human albumin solution is false?
A. When administered in the peri operative period it does not increase the length of stay compared with crystalloid solutions
B. Concentrated solutions may produce diuresis in patients with liver failure
C. It may restore plasma volume in cases of sodium and water overload
D. It may be associated with risk of acquiring new variant Creutzfeld-Jacob disease
E. Hepatitis C remains a concern when large volumes are infused
Answer: E
Human albumin solution went out of vogue following the Cochrane review in 2004 that showed it increased mortality. This
view has been challenged and subsequent studies have confirmed it to be safe for use. Viruses are inactivated during the
preparation process. However, theoretical risks regarding new varient CJD still exist. Outcomes in the peri operative setting are
similar whether colloid, crystalloid or albumin are used.

38. Which statement is true when prescribing nutritional support?


A. For severely ill patients aim to give < 50% energy needs in the first 24-48 hours
B. For patients at risk of refeeding syndrome, they should be given 35 kcal/kg/day initially
C. For severely ill patients aim to give the full energy needs in the first 24-48 hours
D. Patients on diuretics are unlikely to need thiamine
E. Patients on chemotherapy are unlikely to need thiamine
Answer: C
In the NICE guidelines they recommend that enteral or parenteral nutrition is cautiously introduced in seriously ill or injured
people. They suggest that nutrition is started at no more than 50% of the estimated target energy and protein needs and build up
to meet full needs over the first 2448 hours. Provide full requirements of fluid, electrolytes, vitamins and minerals from the
outset. Diuretics and chemotherapy increase the risk of refeeding syndrome.

Nutrition prescriptions: National institute of clinical excellence (NICE) guidelines: For people not severely ill and not at
risk of refeeding syndrome aim to give: 25-35 kcal/kg/day (lower if BMI > 25). 0.8-1.5g protein /kg/day. 30-35 ml fluid/kg/day.
Adequate electrolytes, minerals, vitamins. Severely ill patients aim to give < 50% of the energy and protein levels over the first
24-48h.

For people at high risk of refeeding syndrome: Start at up to 10 kcal/kg/day increasing to full needs over 4-7 days. Start
immediately before and during feeding: oral thiamine 200-300mg/day, vitamin B co strong 1 tds and supplements. Give K+ (2-4
mmol/kg/day), phosphate (0.3-0.6 mmol/kg/day), magnesium (0.2-0.4 mmol/kg/day)

39. A 78 year old man presents with a ruptured aortic aneurysm. This is repaired but the operation is difficult as it has a
juxtarenal location. A supra renal cross clamp is applied. Post operatively he is found to be oliguric and acute renal
failure is suspected. Which of the following statements relating to acute post-operative renal failure are untrue?
A. Intravenous dopamine does not prevent acute renal failure.
B. It is more common after emergency surgery.
C. Use of excessive amounts of intravenous fluids may lead to falsely normal serum creatinine measurements.
D. Vasopressor drugs have a strong renoprotective effect
E. It is minimised by normalisation of haemodynamic status.
Answer: D
Key points : Renal injury and acute renal failure: RIFLE Classification. R=Risk (Serum Creatinine x1.5). I=Injury (Serum
Creatinine x 2). F=Failure (Serum Creatinine x3). L=Loss (Loss of renal function >4weeks). E=End stage kidney disease
366
Vasopressor use is linked to renal failure as they are a marker of haemodynamic compromise.

Acute Renal Failure: Final pathway is tubular cell death. Renal medulla is a relatively hypoxic environment making it
susceptible to renal tubular hypoxia. Renovascular autoregulation maintains renal blood flow across a range of arterial pressures.
Estimates of GFR are best indices of level of renal function. Useful clinical estimates can be obtained by considering serum
creatinine, age, race, gender and body size. eGFR calculations such as the Cockcroft and Gault equation are less reliable in
populations with high GFR's. Nephrotoxic stimuli such as aminoglycosides and radiological contrast media induce apoptosis.
Myoglobinuria and haemolysis result in necrosis. Overlap exists and proinflammatory cytokines play and important role in
potentiating ongoing damage. Post-operative renal failure is more likely to occur in patients who are elderly, have peripheral
vascular disease, high BMI, have COPD, receive vasopressors, are on nephrotoxic medication or undergo emergency surgery.
Avoiding hypotension will reduce risk of renal tubular damage. There is no evidence that administration of ACE inhibitors or
dopamine reduces the incidence of post-operative renal failure.

40. A 45 year old man develops acute respiratory distress syndrome during an attack of severe acute pancreatitis. Which of
the following is not a feature of adult respiratory distress syndrome?
A. It usually consists of type I respiratory failure.
B. Patients typically require high ventillatory pressures.
C. A Swann Ganz Catheter would typically have a reading in excess of 18mmHg.
D. It may complicate acute pancreatitis.
E. It may heal with fibrosis.
Answer: C
Right heart pressure should be normal.

41. Which of the following fulfills the criteria for malnutrition based on the NICE guidelines?
A. Hypoalbuminaemia
B. BMI < 17.5 kg/m2 and unintentional weight loss of > 5% over 3-6/12
C. BMI < 17.5 kg/m2
D. Reduced skin turgor
E. Unintentional weight loss of > 10% over 3- 6 months
Answer: C

Nutrition Screening-NICE guidelines

NICE Screening for malnutrition: A summary: To be performed by an appropriate professional. All new hospital admissions,
new GP patients, new care home patients and patients attending their first clinic should be screened. Afterwards hospital in
patients should be screened weekly.

Nutritional support i.e. oral, enteral or parenteral: Given to patients identified as being malnourished: BMI < 18.5 kg/m2.
Unintentional weight loss of > 10% over 3-6/12. BMI < 20 kg/m2 and unintentional weight loss of > 5% over 3-6/12

NB BMI= weight (kg)/height (m<sup<2< sup="">): Considered in people identified as being AT RISK of malnutrition: Eaten
nothing or little > 5 days, who are likely to eat little for a further 5 days. Poor absorptive capacity High nutrient losses. High
metabolism. NB if considering feed withdrawal refer to GMC guidance 'withholding and withdrawing life prolonging treatment'.
</sup<2<>

42. A 52 year old man is recovering following an elective right hemicolectomy for carcinoma of the caecum. His surgery is
uncomplicated, when should oral intake resume?
A. Only once bowels have been opened to stool
B. Only once the patient has passed flatus
C. Between 24 and 48 hours of surgery
D. More than 48 hours after surgery
E. Within 24 hours of surgery
Answer: E
As part of the enhanced recovery principles oral intake in this setting should resume soon after surgery. Administration of liquid
and even light diet does not increase the risk of anastomotic leak.

Theme: Muscle relaxants

A. Atracurium
B. Suxamethonium
C. Pancuronium
D. Vecuronium
367
E. Curare

Please select the muscle relaxant that applies to the scenario or description supplied. Each option may be used once, more than
once or not at all.

43. An agent that is degraded by hydrolysis and may produce histamine release.

Atracurium. Atracurium is degraded by a process of ester hydrolysis. This uses non specific plasma esterases.

44. An agent which should be avoided in a 23 year old man with burn and bilateral tibial fractures are being trapped in a
car accident for 2 hours.

The correct answer is Suxamethonium. Suxamethonium may induce hyperkalaemia as it induces generalised muscular
contractions. In patients with likely extensive tissue necrosis this may be sufficient to produce cardiac arrest.

45. An agent with a half life of less than 10 minutes.

Suxamethonium. It is extremely rapidly metabolised, acetylcholinesterases degrade the drug within minutes. In patients who
lack this enzyme the drug may last far longer.

46. Which of the following is a recognised feature of ketamine when used as an anaesthetic agent?
A. Malignant hyperpyrexia
B. Adrenal suppression
C. Myocardial depression
D. Dissociative anaesthesia
E. Marked respiratory depression
Answer: D
Unlike most anaesthetic agents ketamine does not cause myocardial or marked respiratory depression. It is not associated with
the adrenal suppression that may occur with etomidate. It is however, associated with a state of dissociative anaesthesia which
patients may find distressing.

47. A 73 year old man undergoes a right below knee amputation for end stage peripheral vascular disease. He is reviewed in
the clinic 8 weeks post operatively and complains of a persistent, burning discomfort over his amputation site stump. On
examination his wound has healed and proximal pulses have a biphasic signal on doppler ultrasound. What is the post
appropriate management?
A. Commence amitryptyline
B. Commence fentanyl patch
C. Arrange duplex scan
D. Arrange MRI scan of the stump
E. Commence carbamazepine
Answer: A
This patient has neuropathic pain. Amitryptyline is the treatment of choice. Carbamazepine is mainly used for trigeminal
neuralgia.

Neuropathic pain: Neuropathic pain may be defined as pain which arises following damage or disruption of the nervous system.
It is often difficult to treat and responds poorly to standard analgesia.

Examples include: diabetic neuropathy. Post-herpetic neuralgia. Trigeminal neuralgia. Prolapsed intervertebral disc

NICE issued guidance in 2010 on the management of neuropathic pain: first-line treatment*: oral amitriptyline or pregabalin. If
satisfactory pain reduction is obtained with amitriptyline but the person cannot tolerate the adverse effects, consider oral
imipramine or nortriptyline as an alternative. Second-line treatment: if first-line treatment was with amitriptyline, switch to or
combine with pregabalin. If first-line treatment was with pregabalin, switch to or
combine with amitriptyline. Other options: pain management clinic, tramadol (not other strong opioids), topical lidocaine for
localised pain if patients unable to take oral medication

*please note that for some specific conditions the guidance may vary. For example carbamazepine is used first-line for trigeminal
neuralgia, duloxetine for diabetic neuropathy

48. A homeless 42 year old male had an emergency inguinal hernia repair 24 hours previously. He has a BMI of 15. His
electrolytes are normal. What is the best initial feeding regime?
A. Give 10 kcal/kg/day initially, oral thiamine 200-300mg/day, vitamin B co strong1 tds and supplements.
368
B. Give 35 kcal/kg/day initially, oral thiamine 200-300mg/day, vitamin B co strong 1 tds and supplements.
C. No change to diet needed
D. Oral thiamine 200-300mg/day, vitamin B co strong1 tds and supplements.
E. Give 35 kcal/kg/day initially
Answer: A
This patient is at high risk of refeeding syndrome.

Nutrition - Refeeding syndrome: Refeeding syndrome describes the metabolic abnormalities which occur on feeding a person a
starved state. The metabolic consequences include: Hypophosphataemia. Hypokalaemia. Hypomagnesaemia. Abnormal fluid
balance. These abnormalities can lead to organ failure.

Re-feeding problems: If patient not eaten for > 5 days, aim to re-feed at < 50% energy and protein levels

High risk for re-feeding problems: If one or more of the following: BMI < 16 kg/m2. Unintentional weight loss >15% over 3-6
months. Little nutritional intake > 10 days. Hypokalaemia, Hypophosphataemia or hypomagnesaemia prior to feeding (unless
high)

If two or more of the following: BMI < 18.5 kg/m2. Unintentional weight loss > 10% over 3-6 months. Little nutritional intake > 5
days. PMH alcohol abuse or drug therapy including insulin, chemotherapy, diuretics, antacids

Prescription: Start at up to 10 kcal/kg/day increasing to full needs over 4-7 days. Start immediately before and during feeding:
oral thiamine 200-300mg/day, vitamin B co strong 1 tds and supplements. Give K+ (2-4 mmol/kg/day), phosphate (0.3-0.6
mmol/kg/day), magnesium (0.2-0.4 mmol/kg/day)

49. A 48 year old man is recovering on the high dependency unit following a long and complex laparotomy. His preoperative
medication includes an ACE inhibitor for blood pressure control. For the past two hours he has been oliguric with a
urine output of 10ml/hr-1. What the most appropriate course of action?
A. Stop the ACE inhibitor
B. Administer a fluid challenge
C. Start an infusion of nor adrenaline
D. Administer intravenous frusemide
E. Insert a Swann-Ganz Catheter
Answer: B
Theme from April 2012 Exam
Hypovolaemia is the most likely cause for oliguria and a fluid challenge is the most appropriate action. Blind administration of
inotropes to hypovolaemic patients is unwise, with the possible exception of cardiac patients.

Hypovolaemia and the surgical patient: Hypovolaemia often represents the end point of multiple pathological processes. It may
be divided into the following categories; overt compensated hypovolaemia, covert compensated hypovolaemia and
decompensated hypovolaemia. Of these three categories the covert compensated subtype of hypovolaemia remains the commonest
and is accounted for by the fact that class I shock will often produce no overtly discernible clinical signs. This is due, in most
cases, to a degree of splanchnic autotransfusion. The most useful diagnostic test for detection of covert compensated
hypovolaemia remains urinanalysis. This often shows increased urinary osmolality and decreased sodium concentration.

In overt compensated hypovolaemia the blood pressure is maintained although other haemodynamic parameters may be affected.
This correlates to class II shock. In most cases assessment can be determined clinically. Where underlying cardopulmonary
disease may be present the placement of a CVP line may guide fluid resuscitation. Severe pulmonary disease may produce
discrepancies between right and left atrial filling pressures. This problem was traditionally overcome through the use of Swann-
Ganz catheters.

Untreated, hypovolaemia may ultimately become uncompensated with resultant end organ dysfunction. Microvascular
hypoperfusion may result in acidosis with a subsequent myocardial depressive effect, thereby producing a viscous circle.

The treatment of hypovolaemia is with intravenous fluids. In the first instance a fluid challenge such as the rapid infusion of
250ml of crystalloid will often serve as both a diagnostic and resuscitative measure. In the event that this fails to produce the
desired response the patient will need to be re-evaluated clinically. More fluid may be needed. However, it is important not to
overlook mechanical ureteric obstruction in the anuric, normotensive patient.

Theme: Muscle relaxants

A. Gallamine
369
B. Benzquinonium
C. Tubocurarine
D. Vecuronium
E. Pancuronium
F. Suxamethonium
G. Decamethonium halides

Please select the most appropriate neuromuscular blocking drugs for the procedure described. Each option may be used once,
more than once or not at all.

50. A 56 year old man is undergoing a distal gastrectomy and just as the surgeon begins to close the deep abdominal muscle
layer the patient develops marked respiratory efforts and closure cannot continue.

Suxamethonium. Suxamethonium has a rapid onset with short duration of action. As this is the final stage of the procedure only
brief muscle relaxation is needed.

51. An agent that is associated with a risk of malignant hyperthermia.

Suxamethonium. Suxamethonium may cause malignant hyperthermia and 1 in 2800 will have abnormal cholinesterase enzyme
and prolonged clinical effect.

52. An agent that may be absorbed from multiple bodily sites and causes histamine release.

The correct answer is Tubocurarine. It can be absorbed orally and rectally, though few would choose this route of
administration. It is now rarely used.

53. A 52 year old man undergoes a laparotomy for perforated bowel after a colonoscopy. 2 days after surgery the nursing
staff report there is pink, serous fluid discharging from the wound. What is the next most appropriate management
step?
A. IV antibiotics for wound infection
B. No further management
C. Examine the wound for separation of the rectus fascia
D. Insert a drain into the wound
E. CT abdomen
Answer: C
The seepage of pink serosanguineous fluid through a closed abdominal wound is an early sign of abdominal wound dehiscence
with possible evisceration. If this occurs, you should remove one or two sutures in the skin and explore the wound manually,
using a sterile glove. If there is separation of the rectus fascia, the patient should be taken to the operating room for primary
closure.

Abdominal wound dehiscence: This is a significant problem facing all surgeons who undertake abdominal surgery on a regular
basis. Traditionally it is said to occur when all layers of an abdominal mass closure fail and the viscera protrude externally
(associated with 30% mortality). It can be subdivided into superficial, in which the skin wound alone fails and complete,
implying failure of all layers.

Factors which increase the risk are: Malnutrition, Vitamin deficiencies, Jaundice, Steroid use, Major wound contamination (e.g.
faecal peritonitis) and Poor surgical technique (Mass closure technique is the preferred method-Jenkins Rule)

When sudden full dehiscence occurs the management is as follows: Analgesia, Intravenous fluids, Intravenous broad spectrum
antibiotics, Coverage of the wound with saline impregnated gauze (on the ward). Arrangements made for a return to theatre

Surgical strategy: Correct the underlying cause (eg TPN or NG feed if malnourished), Determine the most appropriate strategy
for managing the wound

Options
Resuturing of the This may be an option if the wound edges are healthy and there is enough tissue for sufficient coverage.
wound Deep tension sutures are traditionally used for this purpose.
Application of a This is a clear dressing with removable front. Particularly suitable when some granulation tissue is present
wound manager over the viscera or where there is a high output bowel fistula present in the dehisced wound.
Application of a This is a clear plastic bag that is cut and sutured to the wound edges and is only a temporary measure to be
'Bogota bag' adopted when the wound cannot be closed and will necessitate a return to theatre for definitive
management.

370
Application of a These can be safely used BUT ONLY if the correct layer is interposed between the suction device and the
VAC dressing bowel. Failure to adhere to this absolute rule will almost invariably result in the development of multiple
system bowel fistulae and create an extremely difficult management problem.

54. A 63 year old man undergoes a subtotal colectomy and iatrogenic injury to both ureters is sustained. He develops renal
failure and his serum potassium is found to be elevated at 6.9 mmol/L. An ECG is performed, what is the most likely
finding?
A. Increased PR interval
B. Prominent U waves
C. Narrow QRS complexes
D. Peaked T waves
E. Low ST segments
Answer: D
Peaked T waves are the first and most common finding in hyperkalaemia.

ECG features in hyperkalaemia: Peaking of T waves (occurs first). Loss of P waves. Broad QRS complexes. Ventricullar
fibrillation

Theme: Use of vasoactive drugs

A. Dopexamine
B. Dobutamine
C. Noradrenaline
D. Adrenaline
E. Milrinone
F. Dopamine

Please select the most appropriate inotrope for the scenario given. Each option may be used once, more than once or not at all.

55. An inotrope with mixed vaso dilating and vaso constricting properties.

Dopamine. Dilating in the case of renal circulating and constricting in other areas. Overall increases cardiac output. Concept of
renal dose dopamine is out of date.
56. An inotrope that is a phosphodiesterase inhibitor.

Milrinone. Milrinone works by increasing intracellular cAMP concentration.

57. An inotrope that would be useful in a 23 year old female with sepsis secondary to pyelonephritis. She has an increased
cardiac output and decreased systemic vascular resistance.

Noradrenaline. In a setting of septic shock with normal or high cardiac output and decreased SVR, a peripherally acting
vasoconstrictor such as noradrenaline would be the primary choice. . Theme from September 2011 Exam

Circulatory support of the critically ill: Circulatory support: Impaired tissue oxygenation may occur as a result of circulatory
shock. Shock is considered further under its own topic heading.

Patients requiring circulatory support require haemodynamic monitoring. At its simplest level this may simply be in the form of
regular urine output measurements and blood pressure monitoring. In addition ECG monitoring with allow the identification of
cardiac arrhythmias. Pulse oximeter measurements will allow quick estimation haemoglobin oxygen saturation in arterial blood.

Invasive arterial blood pressure monitoring is undertaken by the use of an indwelling arterial line. Most arterial sites can be used
although the radial artery is the commonest. It is important not to cannulate end arteries. The arterial trace can be tracked to
ventilation phases and those patients whose systolic pressure varies with changes in intrathoracic pressure may benefit from
further intravenous fluids.

Central venous pressure is measured using a CVP line that is usually sited in the superior vena cava via the internal jugular route.
The CVP will demonstrate right atrial filling pressure and volume status. When adequate intra vascular volume is present a fluid
challenge will typically cause a prolonged rise in CVP (usually greater than 6-8mmHg).

To monitor the cardiac output a Swan-Ganz catheter is traditionally inserted (other devices may be used and are less invasive).
Inflation of the distal balloon will provide the pulmonary artery occlusion pressure and the pressure distal to the balloon will
371
equate to the left atrial pressure. This gives a measure of left ventricular preload. Because the Swan-Ganz catheter can measure
several variables it can be used to calculate: Stroke volume. Systemic vascular resistance. Pulmonary artery resistanc. Oxygen
delivery (and consumption)

Inotropes: In patients with an adequate circulating volume but on-going circulatory compromise a vasoactive drug may be
considered. These should usually be administered via the central venous route. Commonly used inotropes include:
Noradrenaline- A vasopressor with little effect on cardiac output. Acts as an α agonist.
Adrenaline-Acts on both α and β receptors thereby increasing cardiac output and increasing systemic vascular resistance.
Dopamine- Acts as a β 1 agonist and increases contractility and rate. Renal dose dopamine is an obsolete concept.
Dobutamine- Has both β 1 and β 2 effects and will increase cardiac output and cause decrease in systemic vascular resistance.
Milrinone- Phosphodiesterase inhibitor with a positive inotropic effect. It has a short half-life (1-2 hours) and may precipitate
arrhythmias. Vasopressors often co-administered as it is a vasodilator.

58. A 56 year old man with chronic schizophrenia undergoes a cholecystectomy. He receives metoclopramide for post
operative nausea. Twenty minutes later he becomes agitated and develops marked oculogyric crises and oromandibular
dystonia. Which of the following drugs may best alleviate his symptoms?
A. Procyclidine
B. Lorazepam
C. Chlorpromazine
D. Haloperidol
E. Sulpiride
Answer: A
This man has developed an acute dystonic reaction. Administration of further anti dopaminergic drugs will worsen the situation.
Procyclidine will help to reverse the event. This is most likely to have occurred because the patient is on long term anti
psychotics and has then received metoclopramide.

Acute dystonic reaction: The anti dopaminergic drugs (such as antipsychotics) may result in extrapyramidal side effects. These
may range from mild parkinsonian symptoms such as resting tremor and bradykinesia. Through to acute dystonic reactions which
are characterised by abnormal and involuntary facial and bodily movements, such as spasmodic torticollis, oculogyric crisis and
oromandibular dystonia.

Chronic cases are generally only encountered in psychiatric units. In surgical practice the administration of the anti dopaminergic
drug metoclopramide may be sufficient to precipitate an attack. Treatment may be required if symptoms are sufficiently
troublesome; benzhexol and procyclidine are two drugs which may be used.

Theme: Surgical analgesia

A. TAP block
B. Epidural anaesthatic
C. Spinal block
D. Patient controlled analgesia
E. Paracetamol and diclofenac
F. Pethidine as required
G. Regular nefopam

Please select the most appropriate analgesic modality for the scenario given. Each option may be used once, more than once or not
at all.

59. A 63 year old man with carcinoma of the splenic flexure undergoes an extended right hemicolectomy through a midline
excision. He suffers from COPD.

The correct answer is Epidural anaesthatic. This man is at high risk of atelectasis, hypoventilation can be avoided by minimising
post operative pain through an epidural. A spinal block is short acting, therefore not appropriate.

60. A 63 year old man with rectal cancer is due to undergo an anterior resection by laparoscopic approach. He is otherwise
well.

TAP block. This is a localised infiltration of the abdominal wall with long acting local anaesthetic. This will provide optimal
analgesia for the more limited pain that may occur with a laparoscopic procedure.

372
61. A 52 year old man undergoes an appendicectomy through a lower midline abdominal incision as the initial laparoscopy
shows an appendix mass. He is otherwise well.

The correct answer is Patient controlled analgesia. This is more painful than a conventional appendicectomy, but conversion to
a limited laparotomy was not anticipated. A PCA is the most effective and practically applicable modality in this case.

62. Which of the following anaesthetic agents has the strongest analgesic effect?
A. Sodium thiopentone
B. Ketamine
C. Midazolam
D. Etomidate
E. None of the above
Answer: B
Ketamine has a moderate to strong analgesic effect. It may be used for emergency procedures outside the hospital environment
to induce anaesthesia for procedures such as emergency amputation.

63. Which statement is true on enteral feeding?


A. A PEG can be used 12h after insertion
B. A motility agent is avoided for ITU patients with an Nasogastric tube
C. A regime of 24h continuous feeding is recommended for ITU patients
D. A long-term gastrostomy is recommended if feeding is likely to be > 8 weeks
E. Enteral feeding is not possible in upper GI dysfunction
Answer: C

64. Which of the following does not need monitoring during home parenteral nutritional support?
A. Folate levels
B. Zinc levels
C. Vitamin D
D. Thyroid function
E. Bone densitometry
Answer: D

Nutrition Monitoring-NICE guidelines:


 Weight: daily if fluid balance concerns, otherwise weekly reducing to monthly.
 BMI: at start of feeding and then monthly. If weight cannot be obtained: monthly mid arm circumference or triceps skin
fold thickness.
 Daily: electrolytes until levels stable. Then once or twice a week.
 Weekly: glucose, phosphate, magnesium, LFTs, Ca, albumin, FBC, MCV levels if stable.
 2-4 weekly: Zn, Folate, B12 and Cu levels if stable
 3-6 monthly: iron and ferritin levels, manganese (if on home parenteral regime)
 6 monthly: vitamin D
 Bone densitometry: initially on starting home parenteral nutrition then every 2 years

65. Which of the following is not typically included in total parenteral nutritional solutions?
A. Fibre
B. Lipid
C. Potassium
D. Glucose
E. Magnesium
Answer: A
There is no indication for inclusion of fibre in solutions of TPN, nor would it be safe to do so.

66. A 28 year old man with poorly controlled Crohns disease is nutritionally compromised. The decision is made to start
TPN, via which of the following routes should it be best administered?
A. Internal jugular vein via a central venous catheter
B. Internal carotid artery
C. Cephalic vein via peripheral cannula
D. Basilic vein via peripheral cannula
E. Common femoral vein via a central venous catheter
Answer: A
Since TPN solutions are irritant to veins they are best administered via a central line. The femoral route has a higher incidence
of line associated sepsis and is thus best avoided in this setting.

373
Theme: Wound management

A. VAC Device
B. Packing with alginate ribbon
C. Packing with ribbon gauze
D. Application of silver nitrate
E. Application of potassium permangenate
F. Use of iodine soaked gauze
G. Gauze soaked in proflavin

For each wound please select the most appropriate management option. Each option may be used once, more than once, or not at
all.

67. A 56 year old man has a superficial dehisence of a midline sternotomy wound following an aortic valve replacement.

The correct answer is VAC Device. Provided the sternum is stable a VAC device should promote granulation and healing. It is
not indicated where the sternum has come apart.

68. A 72 year old man has discharge from a healed abdomino-perineal resection wound. On examination it has almost
completely healed but there is prominent granulation tissue at the apex of the wound. There is no evidence of an
underlying collection and he is otherwise well.

The correct answer is Application of silver nitrate. Silver nitrate will cauterise the exuberant granulation tissue and promote
healing.

69. A 23 year old man has an incision and drainage of an axillary abscess, there is no residual surrounding tissue infection.

The correct answer is Packing with alginate ribbon. Use of gauze is inappropriate and will be painful to redress.

Methods of wound closure: Primary closure: Clean wound, usually surgically created or following minor trauma. Standard
suturing methods will usually suffice. Wound heals by primary intention

Delayed primary closure: Similar methods of actual closure to primary closure. May be used in situations where primary closure
is either not achievable or not advisable e.g. infection

Vacuum assisted closure: Uses negative pressure therapy to facilitate wound closure. Sponge is inserted into wound cavity and
then negative pressure applied. Advantages include removal of exudate and versatility. Disadvantages include cost and risk of
fistulation if used incorrectly on sites such as bowel

Split thickness skin grafts: Superficial dermis removed with Watson knife or dermatome (commonly from thigh). Remaining
epithelium regenerates from dermal appendages. Coverage may be increased by meshing

Full thickness skin grafts: Whole dermal thickness is removed. Sub dermal fat is then removed and graft placed over donor site.
Better cosmesis and flexibility at recipient site. Donor site "cost"

Flaps: Viable tissue with a blood supply. May be pedicled or free. Pedicled flaps are more reliable, but limited in range. Free flaps
have greater range but carry greater risk of breakdown as they require vascular anastomosis

70. A 22 year old fit and well male undergoes an emergency appendicectomy. He is given suxamethonium. An inflamed
appendix is removed and the patient is returned to recovery. One hour post operatively the patient develops a
tachycardia of 120 bpm and a temperature of 40 ºC. He has generalised muscular rigidity. What is the most likely
diagnosis?
A. Acute dystonic reaction
B. Malignant hyperthermia
C. Pelvic abscess
D. Epilepsy
E. Serotonin syndrome
Answer: B
Anaesthetic agents, such as suxamethonium, can cause malignant hyperthermia in patients with a genetic defect. Acute dystonic
reaction normally is associated with antipsychotics (haloperidol) and metoclopramide. These lead to marked extrapyramidal
effects. Serotonin syndrome is associated with the antidepressants selective serotonin reuptake inhibitors (SSRIs) and selective
374
serotonin/norepinephrine reuptake inhibitors (SSNRIs). This causes a syndrome of agitation, tachycardia, hallucinations and
hyper-reflexia.

Malignant hyperthermia: Overview: Condition often seen following administration of anaesthetic agents. Characterised by
hyperpyrexia and muscle rigidity. Cause by excessive release of Ca2+ from the sarcoplasmic reticulum of skeletal muscle.
Associated with defects in a gene on chromosome 19 encoding the ryanodine receptor, which controls Ca2+ release from the
sarcoplasmic reticulum. Neuroleptic malignant syndrome may have a similar aetiology

Causative agents: Halothane. Suxamethonium. Other drugs: antipsychotics (neuroleptic malignant syndrome)

Investigations: CK raised. Contracture tests with halothane and caffeine

Management: Dantrolene - prevents Ca2+ release from the sarcoplasmic reticulum

Trauma
1. A 76 year old woman with a body weight of 50 kg is undergoing an excision of a lipoma from her
forehead. It is the first time the senior house officer has performed the procedure. He administers
30ml of 2% lignocaine to the area. The procedure is complicated by bleeding and the patient
experiences discomfort, a further 10ml of the same anaesthetic formulation is then administered.
Over the following 5 minutes the patient complains of tinnitus and becomes drowsy. Which of the
drugs listed below should be administered?
A. Temazepam
B. Lorazepam
C. Naloxone
D. Intralipid 20%
E. Sodium bicarbonate 20%
Answer: D
Local anaesthetic toxicity treatment = Intralipid

Intralipid is indicated for the treatment of local anaesthetic toxicity. In this case the safe dose of local
anaesthetic has been exceeded and is thus this lady's symptoms are likely to represent toxicity.

Local anaesthetic toxicity: Toxicity results from either accidental intravascular injection (rapid onset of
symptoms-usually correct dose), or from excessive dosage (slower onset). Local anaesthetic agents not only
exert a membrane stabilising effect on peripheral nerves but will also act on excitable membranes within the
CNS and Heart. The inhibitory neurones in the CNS are suppressed before the central ones. As a result the
early symptoms will typically be those of circumoral paraesthesia and tinnitus, followed by falling GCS and
eventually coma.

Management of toxicity: Stop injecting the anaesthetic agent. High flow 100% oxygen via face mask.
Cardiovascular monitoring. Administer lipid emulsion (Intralipid 20%) at 1.5ml/Kg over 1 minute as a
bolus. Consider lipid emulsion infusion, at 0.25ml/ Kg/ minute

Safe doses: 10ml of lignocaine 1% contains 100mg of drug, this would constitute 70% of the maximum safe
dose in a 50 kg patient. Up to 7mg / kg can be administered if adrenaline is added to the solution.

Doses of local anaesthetics


Agent Dose plain Dose with adrenaline
Lignocaine 3mg/Kg 7mg/Kg
Bupivicane 2mg/Kg 2mg/Kg
Prilocaine 6mg/Kg 9mg/Kg
These are a guide only as actual doses depend on site of administration, tissue vascularity and co-
morbidities.
375
2. A patient is brought to the emergency department following a motor vehicle accident. He is
unconscious and has a deep scalp laceration. His heart rate is 120/min, blood pressure is 80/40
mmHg, and respiratory rate is 35/min. Despite rapid administration of 2 litres of Hartmans
solution, the patient's vital signs do not change significantly. The injury likely to explain this
patient's hypotension is:
A. Epidural haematoma
B. Sub dural haematoma
C. Intra parenchymal brain haemorrhage
D. Base of skull fracture
E. None of the above
Answer: E
In the patient described, hypotension and tachycardia should not be uncritically attributed to the head injury,
since these findings in the setting of blunt trauma are suggestive of serious thoracic, abdominal, or pelvic
hemorrhage. When cardiovascular collapse occurs as a result of rising intracranial pressure, it is generally
accompanied by hypertension, bradycardia, and respiratory depression.

Head injury: Patients who suffer head injuries should be managed according to ATLS principles and extra
cranial injuries should be managed alongside cranial trauma. Inadequate cardiac output will compromise CNS
perfusion irrespective of the nature of the cranial injury.

Types of traumatic brain injury: Extradural haematoma: Bleeding into the space between the dura
mater and the skull. Often results from acceleration-deceleration trauma or a blow to the side of the head.
The majority of extradural haematomas occur in the temporal region where skull fractures cause a rupture of
the middle meningeal artery. Features: Raised intracranial pressure. Some patients may exhibit a lucid
interval

Subdural haematoma: Bleeding into the outermost meningeal layer. Most commonly occur around the
frontal and parietal lobes. May be either acute or chronic. Risk factors include old age and alcoholism.
Slower onset of symptoms than a extradural haematoma.

Subarachnoid haemorrhageUsually occurs spontaneously in the context of a ruptured cerebral aneurysm


but may be seen in association with other injuries when a patient has sustained a traumatic brain injury

Pathophysiology: Primary brain injury may be focal (contusion/ haematoma) or diffuse (diffuse axonal
injury). Diffuse axonal injury occurs as a result of mechanical shearing following deceleration, causing
disruption and tearing of axons. Intra-cranial haematomas can be extradural, subdural or intracerebral, while
contusions may occur adjacent to (coup) or contralateral (contre-coup) to the side of impact.

Secondary brain injury occurs when cerebral oedema, ischaemia, infection, tonsillar or tentorial herniation
exacerbates the original injury. The normal cerebral auto regulatory processes are disrupted following
trauma rendering the brain more susceptible to blood flow changes and hypoxia. The Cushings reflex
(hypertension and bradycardia) often occurs late and is usually a pre terminal event

Management: Where there is life threatening rising ICP such as in extra dural haematoma and whilst theatre
is prepared or transfer arranged use of IV mannitol/ frusemide may be required. Diffuse cerebral oedema
may require decompressive craniotomy. Exploratory Burr Holes have little management in modern practice
except where scanning may be unavailable and to thus facilitate creation of formal craniotomy flap.

Depressed skull fractures that are open require formal surgical reduction and debridement, closed injuries
may be managed non operatively if there is minimal displacement. ICP monitoring is appropriate in those
who have GCS 3-8 and normal CT scan. ICP monitoring is mandatory in those who have GCS 3-8 and

376
Abnormal CT scan. Hyponatraemia is most likely to be due to syndrome of inappropriate ADH secretion.
Minimum of cerebral perfusion pressure of 70mmHg in adults. Minimum cerebral perfusion pressure of
between 40 and 70 mmHg in children.

Interpretation of pupillary findings in head injuries


Pupil size Light response Interpretation
Unilaterally dilated Sluggish or fixed 3rd nerve compression secondary to tentorial
herniation
Bilaterally dilated Sluggish or fixed  Poor CNS perfusion
 Bilateral 3rd nerve palsy

Unilaterally dilated or Cross reactive (Marcus - Optic nerve injury


equal Gunn)
Bilaterally constricted May be difficult to assess  Opiates
 Pontine lesions
 Metabolic encephalopathy

Unilaterally constricted Preserved Sympathetic pathway disruption

3. A 42 year old man is admitted to surgery with acute appendicitis. He is known to have hypertension,
psoriatic arthropathy and polymyalgia rheumatica. His medical therapy includes: Paracetamol 1g
qds, Codeine phosphate 30mg qds, Bendrofluazide 2.5 mg od, Ramipril 10mg od, Methotrexate
7.5mg once a week, Prednisolone 5mg od, You are called by the Senior House Officer to assess this
man as he has become delirious and hypotensive 2h after surgery. His blood results reveal: Na +132
mmol/l, K+ 5.2 mmol/l, Urea 10 mmol/l, Creatinine 111 µmol/l, Glucose 3.5, CRP 158, Hb 10.2 g/dl,
Platelets 156 * 109/l, WBC 14 * 109/l. What is the most likely diagnosis?
A. Septic shock secondary to appendicitis
B. Neutropenic sepsis
C. Phaeochromocytoma
D. Perforated bowel
E. Addisonian crisis
Answer: E
Features of an addisonian crisis: Hyponatraemia, Hyperkalaemia, Hypoglycaemia

This man is on steroids for polymyalgia rheumatica. Surgery can precipitate acute adrenal deficiency. The
diagnosis is further confirmed by the blood results of hyponatraemia, hyperkalaemia and hypoglycaemia.
This patient urgently needs Hydrocortisone.

Addisonian crisis: Causes: Sepsis or surgery causing an acute exacerbation of chronic insufficiency
(Addison's, Hypopituitarism). Adrenal haemorrhage eg Waterhouse-Friderichsen syndrome (fulminant
meningococcemia). Steroid withdrawal

Management: Hydrocortisone 100 mg IM or IV. 1 litre normal saline infused over 30-60 mins or with
dextrose if hypoglycaemic. Continue hydrocortisone 6 hourly until the patient is stable. No fludrocortisone
is required because high cortisol exerts weak mineralocorticoid action. Oral replacement may begin after 24
hours and be reduced to maintenance over 3-4 days

4. A 32 year old man is involved in a motorcycle accident and sustains a closed unstable spiral tibial
fracture. This is managed with an intramedullary nail. On return to the ward he is noted to have
increasing pain in the limb and on examination the limb is swollen and tender with pain on passive
stretching of the toes. The most likely diagnosis is:
A. Tibial nerve neuropraxia

377
B. Displaced tibial nail
C. Compartment syndrome
D. Deep vein thrombosis
E. Sciatic nerve injury
Answer: C
Severe pain in a limb should raise suspicions of compartment syndrome especially in tibial fractures
following fixation with intra medullary devices. Theme from 2009 Exam and September 2012 Exam

Compartment syndrome: This is a particular complication that may occur following fractures (or
following ischaemia reperfusion injury in vascular patients). It is characterised by raised pressure within a
closed anatomical space. The raised pressure within the compartment will eventually compromise tissue
perfusion resulting in necrosis. The two main fractures carrying this complication include supracondylar
fractures and tibial shaft injuries.

Symptoms and signs: Pain, especially on movement (even passive). Parasthesiae. Pallor may be present.
Arterial pulsation may still be felt as the necrosis occurs as a result of microvascular compromise. Paralysis
of the muscle group may occur

Diagnosis: Is made by measurement of intracompartmental pressure measurements. Pressures in excess of


20mmHg are abnormal and >40mmHg is diagnostic.

Treatment: This is essentially prompt and extensive fasciotomies. In the lower limb the deep muscles may
be inadequately decompressed by the inexperienced operator when smaller incisions are performed.
Myoglobinuria may occur following fasciotomy and result in renal failure and for this reason these patients
require aggressive IV fluids. Where muscle groups are frankly necrotic at fasciotomy they should be
debrided and amputation may have to be considered. Death of muscle groups may occur within 4-6 hours

5. A 28 year old man is involved in a road traffic accident and sustains a flail chest injury. On arrival
in the emergency department he is hypotensive. On examination he has an elevated jugular venous
pulse and auscultation of the heart reveals quiet heard sounds. What is the most likely diagnosis?
A. Pneumothorax
B. Myocardial contusion
C. Cardiac tamponade
D. Haemothorax
E. Ventricular septal defect
Answer: C
The presence of a cardiac tamponade is suggested by Becks Triad: Hypotension. Muffled heart sounds.
Raised JVP. Theme from 2010 exam

Thoracic trauma: Types of thoracic trauma: Tension pneumothorax: Often laceration to lung
parenchyma with flap. Pressure develops in thorax. Most common cause is mechanical ventilation in patient
with pleural injury. Symptoms overlap with cardiac tamponade, hyper-resonant percussion note is more
likely in tension pneumothorax

Flail chest: Chest wall disconnects from thoracic cage. Multiple rib fractures (at least two fractures per rib
in at least two ribs). Associated with pulmonary contusion. Abnormal chest motion. Avoid over hydration
and fluid overload

Pneumothorax: Most common cause is lung laceration with air leakage. Most traumatic pneumothoraces
should have a chest drain. Patients with traumatic pneumothorax should never be mechanically ventilated
until a chest drain is inserted

378
Haemothorax: Most commonly due to laceration of lung, intercostal vessel or internal mammary artery.
Haemothoraces large enough to appear on CXR are treated with large bore chest drain. Surgical exploration
is warranted if >1500ml blood drained immediately

Cardiac tamponade Beck's triad: elevated venous pressure, reduced arterial pressure, reduced heart heart
sounds. Pulsus paradoxus. May occur with as little as 100ml blood

Pulmonary contusion: Most common potentially lethal chest injury. Arterial blood gases and pulse
oximetry important. Early intubation within an hour if significant hypoxia

Blunt cardiac injury: Usually occurs secondary to chest wall injury. ECG may show features of myocardial
infarction. Sequelae: hypotension, arrhythmias, cardiac wall motion abnormalities

Aorta disruption: Deceleration injuries. Contained haematoma. Widened mediastinum

Diaphragm disruption: Most due to motor vehicle accidents and blunt trauma causing large radial tears
(laceration injuries result in small tears). More common on left side. Insert gastric tube, which will pass into
the thoracic cavity

Mediastinal traversing wounds: Entrance wound in one haemothorax and exit wound/foreign body in
opposite haemothorax. Mediastinal haematoma or pleural cap suggests great vessel injury. Mortality is 20%

6. A 52 year old male type 2 diabetic is admitted to the vascular ward for a femoral popliteal bypass.
He suddenly develops expressive dysphasia and marked right sided weakness. The Senior house
officer arranges a CT head scan which shows a 60% left middle cerebral artery territory infarct.
There are no beds on the stroke unit. Overnight the patient becomes unresponsive and a CT head
confirms no bleed. What is the next best management option?
A. IV heparin
B. Clopidogrel
C. Burr hole surgery
D. Aspirin
E. Hemicraniectomy
Answer: E
The likely cause for the reduced consciousness is raised intracranial pressure due to increasing cerebral
oedema related to the infarct. In this situation, urgent neurosurgical review is needed for possible
decompressive hemicraniectomy to relieve the pressure. Ideally no further antiplatelet or anticoagulation
therapy should be given until a plan for surgery is confirmed.

Indications for hemicraniectomy include: Age under 60 years. Clinical deficit in middle cerebral artery
territory. Decreased consciousness. >50% territory infarct

Stroke: types: Primary intracerebral haemorrhage (PICH, c. 10%):Presents with headache, vomiting,
loss of consciousness

Total anterior circulation infarcts (TACI, c. 15%): Involves middle and anterior cerebral arteries:
Hemiparesis/hemisensory loss, Homonymous hemianopia, Higher cognitive dysfunction e.g. Dysphasia

Partial anterior circulation infarcts (PACI, c. 25%): Involves smaller arteries of anterior circulation e.g.
upper or lower division of middle cerebral artery. Higher cognitive dysfunction or two of the three TACI
features

379
Lacunar infarcts (LACI, c. 25%) Involves perforating arteries around the internal capsule, thalamus and
basal ganglia. Present with either isolated hemiparesis, hemisensory loss or hemiparesis with limb ataxia

Posterior circulation infarcts (POCI, c. 25%): Vertebrobasilar arteries: Presents with features of
brainstem damage. Ataxia, disorders of gaze and vision, cranial nerve lesions

Lateral medullary syndrome (posterior inferior cerebellar artery): Wallenberg's syndrome. Ipsilateral:
ataxia, nystagmus, dysphagia, facial numbness, cranial nerve palsy e.g. Horner's. Contralateral: limb sensory
loss

Weber's syndrome: Ipsilateral III palsy. Contralateral weakness

Anterior cerebral artery: Contralateral hemiparesis and sensory loss, lower extremity > upper.
Disconnection syndrome

Middle cerebral artery: Contralateral hemiparesis and sensory loss, upper extremity > lower. Contralateral
hemianopia. Aphasia (Wernicke's). Gaze abnormalities

Posterior cerebral artery: Contralateral hemianopia with macular sparing. Disconnection syndrome

Lacunar: Present with either isolated hemiparesis, hemisensory loss or hemiparesis with limb ataxia

Lateral medulla (posterior inferior cerebellar artery): Ipsilateral: ataxia, nystagmus, dysphagia, facial
numbness, cranial nerve palsy e.g.

Horner's: Contralateral: limb sensory loss

Pontine: VI nerve: horizontal gaze palsy. VII nerve. Contralateral hemiparesis

Theme: Trauma

A. Tension pneumothorax
B. Haemopericardium
C. Haemothorax
D. Aortic transection
E. Ruptured spleen
F. Duodeno-jejunal flexure disruption
G. Aorto iliac disruption
H. Ileo-colic junction disruption

For each scenario please select the most likely injury. Each option may be used once, more than once or not
at all.

7. A 24 year old motorist is involved in a road traffic accident in which he collides with the wall of a
tunnel in a head on car crash, speed 85mph. He is wearing a seatbelt and the airbags have deployed.
When rescuers arrive he is lucid and conscious and then dies suddenly.

Aortic transection. Aortic transections typically occur distal to the ligamentum arteriosum. A temporary

380
haematoma may prevent the immediate death that usually occurs. This is a deceleration injury. A widened
mediastinum may be seen on x-ray.

8. A 30 year old women is involved in a road traffic accident she is a passenger in a car involved in a
head on collision with another vehicle. Her car is travelling at 60mph. She has been
haemodynamically stable throughout with only minimal tachycardia. On examination she has
marked abdominal tenderness and a large amount of intra abdominal fluid on CT scan

The correct answer is Duodeno-jejunal flexure disruption. This is another site of sudden deceleration
injury. Given the large amount of free fluid, if it were blood, then a greater degree of haemodynamic
instability would be expected.

9. A 17 year old boy is involved in a motorcycle accident in which he is thrown from his
motorcycle. On admission he has distended neck veins and a weak pulse. The trachea is central.

Haemopericardium. This is most likely a cardiac tamponade produced by haemopericardium. As little as


100ml of blood may result in tamponade as the pericardial sac is not distensible. Diagnosis is suggested by
muffled heart sounds, paradoxical pulse and jugular vein distension.

10. A 14-year-old boy is admitted to the acute surgical unit with appendicitis. He is normally fit and
well. Apart from metoclopramide, the patient has had no other medications. The nursing staff
contact you as the patient is acting strange. On examination he is agitated, has a clenched jaw and
his eyes are deviated upwards. What is the most likely diagnosis?
A. Functional disorder
B. Malignant hyperthermia
C. Oculogyric crisis
D. Epilepsy
E. Serotonin syndrome
Answer: C
This is a classic description of an oculogyric crisis, a form of extrapyramidal disorder. An oculogyric
crisis is an acute dystonic reaction. This is precipitated by antipsychotics (haloperidol) and
metoclopramide in susceptible individuals with a genetic predisposition to this. Treatment is with
procyclidine IM.

Oculogyric crisis: An oculogyric crisis is a dystonic reaction to certain drugs or medical conditions

Features: Restlessness, agitation. Involuntary upward deviation of the eyes

Causes: Phenothiazines. Haloperidol. Metoclopramide. Postencephalitic Parkinson's disease

Management: Procyclidine

11. A 6 year old boy pulls over a kettle and suffers superficial partial thickness burns to his legs. Which
of the following will not occur?
A. Preservation of hair follicles
B. Formation of vesicles or bullae
C. Damage to sweat glands
D. Healing by re-epithelialisation
E. The burn area will be painful
Answer: C
Partial thickness burns are divided into superficial and deep burns, however, this is often not possible on
initial assessment and it may be a week or more before the distinction is clear cut. Dermal appendages are,
381
by definition, intact. Superficial partial thickness burns will typically heal by re-epithelialisation, deeper
burns will heal with scarring.

Burns: Types of burn


Type of burn Skin layers affected Skin Blanching Management
appearance
Epidermal/Superficial Epidermis Red, moist Yes
Superficial partial Epidermis and part of Pale, dry Yes Normally heals with no
thickness papillary dermis intervention
affected
Deep partial thickness Epidermis, whole Mottled red No Needs surgical
papillary dermis colour intervention (depending
affected on site)
Full thickness Whole skin layer and Dry, leathery No Burns centre
subcutaneous tissue hard wound
affected

Depth of burn assessment: Bleeding on needle prick. Sensation. Appearance. Blanching to pressure

Percentage burn estimation


Lund Browder chart: most accurate even in children
Wallace rule of nines
Palmar surface: surface area palm = 0.8% burn

>15% body surface area burns in adults needs urgent burn fluid resuscitation

Transfer to burn centre if: Need burn shock resuscitation. Face/hands/genitals affected. Deep partial
thickness or full thickness burns. Significant electrical/chemical burns

Escharotomies: Indicated in circumferential full thickness burns to the torso or limbs. Careful division of
the encasing band of burn tissue will potentially improve ventilation (if the burn involves the torso), or
relieve compartment syndrome and oedema (where a limb is involved)

12. You are called to the acute surgical unit. A patient who has short gut syndrome has developed a
broad complex tachycardia. You suspect a diagnosis of ventricular tachycardia. What is the most
likely precipitant?
A. Hypoglycaemia
B. Bisoprolol
C. Hypomagnesaemia
D. Dehydration
E. Hyperthyroidism
Answer: C

Ventricular tachycardia: Ventricular tachycardia (VT)is broad-complex tachycardia originating from a


ventricular ectopic focus. It has the potential to precipitate ventricular fibrillation and hence requires urgent
treatment. There are two main types of VT: monomorphic VT: most commonly caused by myocardial
infarction. Polymorphic VT: A subtype of polymorphic VT is torsades de pointes which is precipitated by
prolongation of the QT interval. The causes of a long QT interval are listed below

Causes of a prolonged QT interval

Congenital: Jervell-Lange-Nielsen syndrome (includes deafness and is due to an abnormal potassium


channel). Romano-Ward syndrome (no deafness)
382
Drugs: amiodarone, sotalol, class 1a antiarrhythmic drugs. Tricyclic antidepressants, fluoxetine.
Chloroquine. Terfenadine*. Erythromycin

Other: electrolyte: hypocalcaemia, hypokalaemia, Hypomagnesaemia. Acute myocardial infarction.


Myocarditis. Hypothermia. Subarachnoid haemorrhage

13. Based on the current guidelines, which option regarding management of head injuries is false?
A. Opiates should be avoided
B. Consider intubation if the GCS is <8 or = 8
C. Immediate CT head if there is > 1 episode of vomiting
D. Half hourly GCS assessment until GCS is 15
E. Contact neurosurgeons if suspected penetrating injury
Answer: A
Pain should be controlled, with opiates preferably, as this avoids distress and hypertension post injury.

Head injury management- NICE Guidelines: Summary of guidelines: All patients should be assessed
within 15 minutes on arrival to A&E. Document all 3 components of the GCS. If GCS <8 or = to 8, consider
stabilising the airway. Treat pain with low dose IV opiates (if safe). Full spine immobilisation until
assessment if: GCS < 15. Neck pain/tenderness. Paraesthesia extremities. Focal neurological deficit.
Suspected c-spine injury.
If a c-spine injury is suspected a 3 view c-spine x-ray is indicated. CT c-spine is preferred if: Intubated. GCS
<13. Normal x-ray but continued concerns regarding c-spine injury

Immediate CT head (within 1h) if: GCS < 13 on admission. GCS < 15 2h after admission. Suspected open
or depressed skull fracture. Suspected skull base fracture (panda eyes, Battle's sign, CSF from nose/ear,
bleeding ear). Focal neurology. Vomiting > 1 episode. Post traumatic seizure. Coagulopathy

Contact neurosurgeon if: Persistent GCS < 8 or = 8. Unexplained confusion > 4h. Reduced GCS after
admission. Progressive neurological signs. Incomplete recovery post seizure. Penetrating injury.
Cerebrospinal leak.

Observations: 1/2 hourly GCS until 15

14. A 22 year old man suffers 20% partial and full thickness burns in a house fire. There is an
associated inhalational injury. It is decided to administer intravenous fluids to replace fluid losses.
Which of the following intravenous fluids should be used for initial resuscitation?
A. Dextran 40
B. 5% Dextrose
C. Fresh frozen plasma
D. Hartmans solution
E. Blood
Answer: D
In most units a crystalloid such as Hartmans (Ringers lactate) is administered initially. Controversy does
remain and some units do prefer colloid. Should this leak in the interstial tissues this may increase the risk
of oedema.

Fluid resuscitation burns: Indication: >15% total body area burns in adults (>10% children): The main
aim of resuscitation is to prevent the burn deepening. Most fluid is lost 24h after injury. First 8-12h fluid
shifts from intravascular to interstitial fluid compartments. Therefore circulatory volume can be
compromised. However fluid resuscitation causes more fluid into the interstitial compartment especially
colloid (therefore avoided in first 8-24h). Protein loss occurs.
383
Fluid resuscitation formula: Parkland formula
(Crystalloid only e.g. Hartman's solution/Ringers' lactate)
Total fluid requirement in 24 hours = 4 ml x (total burn surface area (%)) x (body weight (kg)). 50%
given in first 8 hours. 50% given in next 16 hours.

Resuscitation endpoint:Urine output of 0.5-1.0 ml/kg/hour in adults (increase rate of fluid to achieve this)

Points to note: Starting point of resuscitation is time of injury. Deduct fluids already given

After 24 hours: Colloid infusion is begun at a rate of 0.5 ml x(total burn surface area (%))x(body weight
(kg)). Maintenance crystalloid (usually dextrose-saline) is continued at a rate of 1.5 ml x(burn area)x(body
weight). Colloids used include albumin and FFP. Antioxidants, such as vitamin C, can be used to minimize
oxidant-mediated contributions to the inflammatory cascade in burns. High tension electrical injuries and
inhalation injuries require more fluid. Monitor: packed cell volume, plasma sodium, base excess, and lactate
15. A 23 year old man sustains a severe facial fracture and reconstruction is planned. Which of the
following investigations will facilitate pre-operative planning?
A. Mandibular tomography
B. Magnetic resonance scan of face
C. Skull X-ray
D. Computerised tomography of the head
E. Orthopantomogram
Answer: D
Significant facial fractures may have intracranial communication. CT scanning will allow delineation of
injury extent and 3D reconstruction images can be created. An Orthopantomogram (OPT) will provide
good images of mandible and surrounding bony structures but will not give intracranial detail. A skull x-
ray lacks the detail for modern practice. Theme from 2011 Exam

Craniomaxillofacial injuries in the UK are due to: Interpersonal violence (52%). Motor vehicle accidents
(16%). Sporting injuries (19%). Falls (11%)

Le Fort Fractures: Grade Feature


Le Fort 1: The fracture extends from the nasal septum to the lateral pyriform rims, travels horizontally
above the teeth apices, crosses below the zygomaticomaxillary junction, and traverses the pterygomaxillary
junction to interrupt the pterygoid plates.

Le Fort 2: These fractures have a pyramidal shape and extend from the nasal bridge at or below the
nasofrontal suture through the frontal process of the maxilla, inferolaterally through the lacrimal bones and
inferior orbital floor and rim through or near the inferior orbital foramen, and inferiorly through the anterior
wall of the maxillary sinus; it then travels under the zygoma, across the pterygomaxillary fissure, and
through the pterygoid plates.

Le Fort 3: These fractures start at the nasofrontal and frontomaxillary sutures and extend posteriorly along
the medial wall of the orbit through the nasolacrimal groove and ethmoid bones. The thicker sphenoid bone
posteriorly usually prevents continuation of the fracture into the optic canal. Instead, the fracture continues
along the floor of the orbit along the inferior orbital fissure and continues superolaterally through the lateral
orbital wall, through the zygomaticofrontal junction and the zygomatic arch. Intranasally, a branch of the
fracture extends through the base of the perpendicular plate of the ethmoid, through the vomer, and through
the interface of the pterygoid plates to the base of the sphenoid. This type of fracture predisposes the patient
to CSF rhinorrhea more commonly than the other types.

384
Ocular injuries: Superior orbital fissure syndrome
Severe force to the lateral wall of the orbit resulting in compression of neurovascular structures. Results in :
Complete opthalmoplegia and ptosis (Cranial nerves 3, 4, 6 and nerve to levator palpebrae superioris).
Relative afferent pupillary defect. Dilatation of the pupil and loss of accommodation and corneal reflexes.
Altered sensation from forehead to vertex (frontal branch of trigeminal nerve)

Orbital blow out fracture


Typically occurs when an object of slightly larger diameter than the orbital rim strikes the incompressible
eyeball. The bone fragment is displaced downwards into the antral cavity, remaining attached to the orbital
periosteum. Periorbital fat may be herniated through the defect, interfering with the inferior rectus and
inferior oblique muscles which are contained within the same fascial sheath. This prevents upward
movement and outward rotation of the eye and the patient experiences diplopia on upward gaze. The initial
bruising and swelling may make assessment difficult and patients should usually be reviewed 5 days later.
Residual defects may require orbital floor reconstruction.

Nasal Fractures: Common injury. Ensure new and not old deformity. Control epistaxis. CSF rhinorrhoea
implies that the cribriform plate has been breached and antibiotics will be required. Usually best to allow
bruising and swelling to settle and then review patient clinically. Major persistent deformity requires
fracture manipulation, best performed within 10 days of injury.

Retrobulbar haemorrhage
Rare but important ocular emergency. Presents with: Pain (usually sharp and within the globe). Proptosis
Pupil reactions are lost. Paralysis (eye movements lost). Visual acuity is lost (colour vision is lost first)
May be the result of Le Fort type facial fractures.

Management: Mannitol 1g/Kg as 20% infusion, Osmotic diuretic, Contra-indicated in congestive heart
failure and pulmonary oedema. Acetazolamide 500mg IV, (Monitor FBC/U+E) Reduces aqueous pressure
by inhibition of carbonic anhydrase (used in glaucoma). Dexamethasone 8mg orally or intravenously. In a
traumatic setting an urgent catholysis may be needed prior to definitive surgery.

Consider: Papaverine 40mg smooth muscle relaxant. Dextran 40 500mls IV improves perfusion

Theme: Treatment of burns

A. 3500ml Human albumin solution


B. 3300ml Hartmans solution
C. 4500ml Hartmans solution
D. 6600ml Hartmans solution
E. 5500ml Hartmans solution
F. 5500ml Human albumin solution
G. 6000ml Human albumin solution
H. Fluid resuscitation not required
I. 5500ml Fresh frozen plasma

For the burns patients described below please determine the correct volume and type of intravenous fluid
(if required) for the first 8 hours of care. Each option may be used once, more than once or not at all.

16. A 25 year old male is admitted having been involved in a house fire. His weight on admission is 75 kg
and he has suffered 22% full thickness burns to his torso and limbs.

3300ml Hartmans solution.. Using the Parkland formula calculated below:

385
4 x 22 x 75/2= 3300ml. Remember half the total is administered in the first 8 hours. Hartmans is the
preferred agent. Theme from April 2012 Exam

17. A 25 year old male is admitted having been involved in a house fire. His weight on admission is 85 kg
and he has suffered 19% full thickness burns to his torso and limbs.

3300ml Hartmans solution. Using the Parkland formula calculated below: 4 x 19 x 85/2= 3230ml Therefore
3300ml is the nearest approximate amount.

18. A 25 year old male is admitted having been involved in a house fire. His weight on admission is 65 kg
and he has suffered 25% full thickness burns to his torso and limbs.

3300ml Hartmans solution. Using the Parkland formula calculated below: 4 x 25 x 65/2= 3250ml
Therefore 3300ml is the nearest approximate amount.

19. A 23 year old man is stabbed in the right upper quadrant and is haemodynamically unstable. A
laparotomy is performed and the liver has some extensive superficial lacerations and is bleeding
profusely. The patient becomes progressively more haemodynamically unstable. Which of the
following is the best management option?
A. Pack the liver and close the abdomen
B. Occlude the hepatic inflow with a pringles manoeuvre and suture the defects
C. Occlude vascular inflow and resect the most severely affected area anatomically
D. Perform a portosystemic shunt procedure
E. Suture the defects without vascular occlusion
Answer: A
Packing of the liver is the safest option and resection or repair considered later when the physiology is
normalised. Often when the packs are removed all the bleeding has ceased and the abdomen can be closed
without further action. Definitive attempts at suturing or resection at the primary laparotomy are often
complicated by severe bleeding.

Trauma management: The cornerstone of trauma management is embodied in the principles of ATLS.
Following trauma there is a trimodal death distribution: Immediately following injury. Typically as result of
brain or high spinal injuries, cardiac or great vessel damage. Salvage rate is low.

In early hours following injury. In this group deaths are due to phenomena such as splenic rupture, sub
dural haematomas and haemopneumothoraces

In the days following injury. Usually due to sepsis or multi organ failure.
Aspects of trauma management: ABCDE approach. Tension pneumothoraces will deteriorate with
vigorous ventilation attempts. External haemorrhage is managed as part of the primary survey. As a rule
tourniquets should not be used. Blind application of clamps will tend to damage surrounding structures and
packing is the preferred method of haemorrhage control. Urinary catheters and naso gastric tubes may need
inserting. Be wary of basal skull fractures and urethral injuries. Patients with head and neck trauma should
be assumed to have a cervical spine injury until proven otherwise.

Thoracic injuries: Simple pneumothorax. Mediastinal traversing wounds. Tracheobronchial tree injury.
Haemothorax. Blunt cardiac injury. Diaphragmatic injury. Aortic disruption. Pulmonary contusion

Management of thoracic trauma

386
 Simple pneumothorax insert chest drain. Aspiration is risky in trauma as pneumothorax may be from
lung laceration and convert to tension pneumothorax.
 Mediastinal traversing wounds These result from situations like stabbings. Exit and entry wounds in
separate hemithoraces. The presence of a mediastinal haematoma indicates the likelihood of a great
vessel injury. All patients should undergo CT angiogram and oesophageal contrast swallow. Indications
for thoracotomy are largely related to blood loss and will be addressed below.
 Tracheobronchial tree injury Unusual injuries. In blunt trauma most injuries occur within 4cm of the
carina. Features suggesting this injury include haemoptysis and surgical emphysema. These injuries have
a very large air leak and may have tension pneumothorax.
 Haemothorax Usually caused by laceration of lung vessel or internal mammary artery by rib fracture.
Patients should all have a wide bore 36F chest drain. Indications for thoracotomy include loss of more
than 1.5L blood initially or ongoing losses of >200ml per hour for >2 hours.
 Cardiac contusions Usually cardiac arrhythmias, often overlying sternal fracture. Perform
echocardiography to exclude pericardial effusions and tamponade. Risk of arrhythmias falls after 24
hours.
 Diaphragmatic injury Usually left sided. Direct surgical repair is performed.
 Traumatic aortic disruption Commonest cause of death after RTA or falls. Usually incomplete
laceration near ligamentum arteriosum. All survivors will have contained haematoma. Only 1-2% of
patients with this injury will have a normal chest x-ray.
 Pulmonary contusion Common and lethal. Insidious onset. Early intubation and ventilation.

Abdominal trauma: Deceleration injuries are common. In blunt trauma requiring laparotomy the spleen is
most commonly injured (40%). Stab wounds traverse structures most commonly liver (40%). Gunshot
wounds have variable effects depending upon bullet type. Small bowel is most commonly injured (50%).
Patients with stab wounds and no peritoneal signs up to 25% will not enter the peritoneal cavity. Blood at
urethral meatus suggests a urethral tear. High riding prostate on PR = urethral disruption. Mechanical testing
for pelvic stability should only be performed once

Investigations in abdominal trauma

Diagnostic Peritoneal Abdominal CT scan USS


Lavage
Indication Document bleeding if Document organ injury if Document fluid if
hypotensive normotensive hypotensive
Advantages Early diagnosis and Most specific for localising Early diagnosis, non
sensitive; 98% accurate injury; 92 to 98% accurate invasive and repeatable; 86
to 95% accurate
Disadvantages Invasive and may miss Location of scanner away from Operator dependent and
retroperitoneal and facilities, time taken for may miss retroperitoneal
diaphragmatic injury reporting, need for contrast injury

Amylase may be normal following pancreatic trauma. Urethrography if suspected urethral injury

20. A 62 year old woman presents with acute bowel obstruction. She has been vomiting up to 15 times a
day and is taking erythromycin. She suddenly collapses. Her ECG shows torsades de pointes. What
is the management of choice?
A. IV Atropine
B. IV Potassium
C. IV Magnesium sulphate
D. IV Bicarbonate
E. IV Adrenaline
Answer: C

387
Torsades de pointes: Treatment IV magnesium sulphate

This woman is likely to have hypokalaemia and hypomagnasaemia as a result of vomiting. In addition to
this, the erythromycin will predispose her to torsades de pointes. Knowledge of the management of this peri
arrest diagnosis is hence important in surgical practice.

Torsades de pointes: Torsades de pointes ('twisting of the points') is a rare arrhythmia associated with a
long QT interval. It may deteriorate into ventricular fibrillation and hence lead to sudden death

Causes of long QT interval: Congenital: Jervell-Lange-Nielsen syndrome, Romano-Ward syndrome.


Antiarrhythmics: amiodarone, sotalol, class 1a antiarrhythmic drugs. Tricyclic antidepressants.
Antipsychotics. Chloroquine. Terfenadine. Erythromycin. Electrolyte: hypocalcaemia, hypokalaemia,
hypomagnesaemia. Myocarditis. Hypothermia. Subarachnoid haemorrhage

Management: IV magnesium sulphate

21. A 27-yrs-old man sustains a single gunshot wound to the left thigh. In the emergency department, he
is noted to have a large haematoma of his medial thigh. He complains of parasthesia in his foot. On
examination, there are weak pulses palpable distal to the injury and the patient is unable to move
his foot. The appropriate initial management of this patient is:
A. Angiography
B. Immediate exploration and repair
C. Fasciotomy of the anterior compartment
D. Observation for resolution of spasm
E. Local wound exploration
Answer: B
The five P's of arterial injury include pain, parasthesias, pallor, pulselessness and paralysis. In the
extremities, the tissues most sensitive to anoxia are the peripheral nerves and striated muscle. The early
developments of paresthesias and paralysis are signals that there is significant ischemia present, and
immediate exploration and repair are warranted. The presence of palpable pulse does not exclude an
arterial injury because this presence may represent a transmitted pulsation through a blood clot. When
severe ischemia is present, the repair must be completed within 6 to 8 h to prevent irreversible muscle
ischemia and loss of limb function. Delay to obtain an angiogram or to observe for change needlessly
prolongs the ischemic time. Fasciotomy may be required but should be done in conjunction with and after
reestablishment of arterial flow. Local wound exploration is not recommended because brisk hemorrhage
may be encountered without the securing of prior vascular control.

Vascular trauma: Peripheral and central vessels may be injured by blunt, penetrating or shearing injuries.
Fractures of bones close to vessels may also be associated with vascular injury or vessel occlusion.

Assessment: Check for signs of distal perfusion. Doppler signal distally (monophasic/ biphasic or triphasic).
Anatomical location (which vessel is likely to be involved). Duplex scanning and angiography are "gold
standard" tests but may not be immediately available in the trauma setting

Management: Almost always operative. Obtaining proximal and distal control of affected vessels is crucial.
Simple lacerations of arteries may be directly closed, or a vein patch applied if there is a risk of subsequent
stenosis. Transection of the vessel should be treated by either end to end anastomosis (often not possible) or
an interposition vein graft. Use of PTFE in traumatic open injuries will invariably result in infection.

Theme: Paediatric emergencies

A. Manage conservatively

388
B. Immediate emergency theatre
C. Treat in emergency department
D. Treat in emergency department under sedation
E. Operate on next emergency list

Please select the most appropriate intervention for the scenario given. Each option may be used once, more
than once or not at all.

22. A 3 year old child inserts a crayon into their external auditory meatus. Attempts to remove it have
not been successful.

The correct answer is Operate on next emergency list.They would not tolerate removal in the emergency
department. The tympanic membrane should be carefully inspected and again this will be easier under
general anaesthesia. Theme from September 2011 Exam

23. A 2 year old accidentally inhales a peanut. They arrive in the emergency department extremely
distressed and cyanotic. Imaging shows it to be lodged in the left main bronchus.

The correct answer is Immediate emergency theatre. As they are cyanosed it requires immediate removal
and this should be undertaken in a fully staffed theatre. Ideally a rigid bronchoscopy should be performed.

24. A 10 year old boy is shot in the head with an airgun pellet. He is concerned that he will get into
trouble and the injury remains concealed for 10 days. Imaging using CT scanning shows it to be
lodged in the frontal lobe.

Manage conservatively. The pellet is small and no serious injury has occurred at this stage. This should
therefore be managed conservatively.

Management of acute cases- Paediatric: Children will often insert objects into orifices such as the nose
and external auditory meatus. Assessment includes assessment of airway and haemodynamic status. Where
the airways is not immediately threatened decisions can be made as to whether to manage in the emergency
department or transfer to theatre. In general children do not tolerate procedures well and it is usually safer to
remove objects in theatre and under general anaesthesia with a secure airway.

A chest x-ray is required to ensure that no object is present in the chest, not all objects are radiolucent.
However, signs such as focal consolidation may indicate small airway obstruction. In the case of small bore
missile injuries the decision relating to surgery depends on the size of the missile and its location. Airgun
pellets are a common culprit, if there is a long time interval between the incident and presentation and the
object has not caused any significant problems then it may be best left alone

Airgun pellets lodged in the soft tissues (and glass) are usually notoriously difficult to localise and extract,
no matter how superficial. Removal in theatre is usually the best option. If the object is radiolucent then an
image intensifier should be used

25. A 32 year old male is receiving a blood transfusion after being involved in a road traffic accident. A
few minutes after the transfusion he complains of loin pain. His observations show temperature 39
oC, HR 130bpm and blood pressure is 95/40mmHg. What is the best test to confirm his diagnosis?

A. USS abdomen
B. Direct Coomb's test
C. Blood cultures
D. Blood film
389
E. Sickle cell test
Answer: B
Mnemonic for transfusion reactions: Got a bad unit
G raft vs. Host disease. O verload. T hrombocytopaenia. A lloimmunization. B lood pressure unstable. A
cute haemolytic reaction. D elayed haemolytic reaction. U rticaria. N eutrophilia. I nfection. T ransfusion
associated lung injury

The diagnosis is of an acute haemolytic transfusion reaction, normally due to ABO incompatibility.
Haemolysis of the transfused cells occurs causing the combination of shock, haemoglobinaemia and loin
pain. This may subsequently lead to disseminated intravascular coagulation. A Coomb's test should confirm
haemolysis. Other tests for haemolysis include: unconjugated bilirubin, haptoglobin, serum and urine free
haemoglobin.

Note that delayed haemolytic reactions are normally associated with antibodies to the Rh system and occur
5-10 days after transfusion.

Blood transfusion reactions

Immune mediated Non immune mediated


Pyrexia Hypocalcaemia
Alloimmunization CCF
Thrombocytopaenia Infections
Transfusion associated lung injury Hyperkalaemia
Graft vs Host disease
Urticaria
Acute or delayed haemolysis
ABO incompatibility
Rhesus incompatibility
Notes:
GVHD: lymphocyte proliferation causing organ failure
Transfusion associated lung injury: neutrophil mediated allergic pulmonary oedema
ABO and Rhesus incompatibility: causes acute haemolytic transfusion reaction leading to agglutination and
haemolysis

26. A 45 year old man complains of sharp chest pain. He is due to have elective surgery to replace his
left hip. He has been bed bound for 3 months. He suddenly collapses; his blood pressue is
70/40mmHg, heart rate 120 bpm and his saturations are 74% on air. He is deteriorating in front of
you. What is the next best management plan?
A. Aspirin
B. Thrombolysis with Alteplase
C. Unfractionated heparin
D. Thrombolysis with streptokinase
E. Clopidogrel
Answer: B
This man is peri arrest with the diagnosis of pulmonary embolism (chest pain,bedbound, collapse, low
saturations). He needs urgent thrombolysis with alteplase (he may not survive if you wait for the medical
Spr/ITU to arrive!).

Pulmonary embolism: management: A summary of the British Thoracic Society guidelines

Heparin should be given if intermediate or high clinical probability before imaging. Unfractionated heparin
(UFH) should be considered (a) as a first dose bolus, (b) in massive PE, or (c) where rapid reversal of effect
may be needed. Otherwise, low molecular weight heparin (LMWH) should be considered as preferable to
390
UFH, having equal efficacy and safety and being easier to use. Oral anticoagulation should only be
commenced once VTE has been reliably confirmed. The target INR should be 2.0-3.0; when this is
achieved, heparin can be discontinued. The standard duration of oral anticoagulation is: 46 weeks for
temporary risk factors, 3 months for first idiopathic, and at least 6 months for other; the risk of bleeding
should be balanced with that of further VTE.

Massive PE: CTPA or echocardiography will reliably diagnose clinically massive PE. Thrombolysis is 1st
line for massive PE (ie circulatory failure) and may be instituted on clinical grounds alone if cardiac arrest is
imminent; a 50 mg bolus of alteplase is recommended. Invasive approaches (thrombus fragmentation and
IVC filter insertion) should be considered where facilities and expertise are readily available.

27. A 30 year old woman, who is 30 weeks pregnant, attends the varicose vein clinic. She suddenly
complains of shortness of breath and chest pain. She has no underlying lung condition. Her
saturations are 92 % air, blood pressure 150/80 mmHg and her chest sounds clear. What is the main
investigation of choice to confirm her diagnosis?
A. Lung spirometry
B. Half dose scintigraphy
C. CTPA
D. CXR
E. Full dose scintigraphy
Answer: B
The main differential diagnosis is pulmonary embolism. CXR should be performed first in second and
third trimester to exclude other diagnoses such as pneumothorax or pneumonia. Concerns surrounding
radiation exposure have been discounted at this stage, as the need to establish a diagnosis is the major
priority. If the chest x-ray is normal, then half dose scintigraphy or CTPA (if the patient has underlying
lung pathology) is performed.

Chest pain in pregnancy: Aortic dissection: Predisposing factors in pregnancy are hypertension,
congenital heart disease and Marfan's syndrome. Mainly Stanford type A dissections. Sudden tearing chest
pain, transient syncope. Patient may be cold and clammy, hypertensive and have an aortic regurgitation
murmur. Involvement of the right coronary artery may cause inferior myocardial infarction

Surgical management
Gestational timeframe Management
< 28/40 Aortic repair with the fetus kept in utero
28-32/40 Dependent on fetal condition
> 32/40 Primary Cesarean section followed by aortic repair at the same operation

Mitral stenosis: Most cases associated with rheumatic heart disease. Becoming less common in British
women; suspect in Immigrant women. Commonest cardiac condition in pregnancy. Commonly associated
with mortality. Valve surgery; balloon valvuloplasty preferable

Pulmonary embolism: Leading cause of mortality in pregnancy. Half dose scintigraphy; CT chest if
underlying lung disease should aid diagnosis. Treatment with low molecular weight heparin throughout
pregnancy and 4-6 weeks after childbirth. Warfarin is contra indicated in pregnancy

Theme: Bleeding disorders

A. Vitamin K deficiency
B. von Willebrand's disease
C. Acquired haemophilia
391
D. Haemophilia B
E. Protein C deficiency
F. Disseminated intravascular coagulation
G. Factor V Leiden
H. Excess heparin
I. Warfarin overdose

What is the most likely diagnosis for the scenario given? Each option may be used once, more than once or
not at all.

28. A 33 year old female is admitted for varicose vein surgery. She is fit and well. After the procedure
she is persistently bleeding. She is known to have menorrhagia. Investigations show a prolonged
bleeding time and increased APTT. She has a normal PT and platelet count.

Von Willebrand's disease. Bleeding post operatively, epistaxis and menorrhagia may indicate a diagnosis
of vWD. Haemoarthroses are rare. The bleeding time is usually normal in haemophilia (X-linked) and
vitamin K deficiency.

29. A 70 year old coal miner presents with 3 weeks of haematuria and bruising. He is normally fit and
well. He is on no medications. His results reveal: Hb 9.0, WCC 11, Pl 255, PT 16 (normal), APTT, 58
(increased) and Thrombin time 20 (normal).

The correct answer is Acquired haemophilia. Factor 8 acquires an inhibitor. The elderly, pregnancy,
malignancy and autoimmune conditions are associated with acquired haemophilia. Prolonged APTT is key
to the diagnosis. Management involves steroids.

30. A 33 year old female is attends the day unit for elective varicose vein surgery. She has previously
had recurrent pulmonary embolic events. After the procedure she is persistently bleeding. Her
APTT is 52 (increased).

The correct answer is Factor V Leiden. A combination of thromboembolism and bleeding in a young
woman should raise the possibility of antiphospholipid syndrome. Other features may include foetal loss,
venous and arterial thrombosis and thrombocytopenia. Protein C is not associated with a prolonged APTT.
Theme from January 2012 exam

Abnormal coagulation
Cause Factors affected
Heparin Prevents activation factors 2,9,10,11
Warfarin Affects synthesis of factors 2,7,9,10
DIC Factors 1,2,5,8,11
Liver disease Factors 1,2,5,7,9,10

Interpretation blood clotting test results


Disorder APTT PT Bleeding time
Haemophilia Increased Normal Normal
von Willebrand's disease Increased Normal Increased
Vitamin K deficiency Increased Increased Normal

31. A 54-year-old man is brought to the Emergency Department after being found collapsed in the
street. He is known to have a history of alcoholic liver disease. Blood tests reveal the following:

392
Calcium 1.62 mmol/l and Albumin 33 g/l. Which one of the following is the most appropriate
management of the calcium result?
A. 10ml of 10% calcium chloride over 10 minutes
B. 20% albumin infusion
C. 10ml of 10% calcium gluconate over 10 minutes
D. No action
E. 10ml of 10% calcium chloride over 4 hours
Answer: C
Even after correction for the low albumin level this patient has significant hypocalcaemia which should be
corrected.

Hypocalcaemia: causes and management: The clinical history combined with parathyroid hormone levels
will reveal the cause of hypocalcaemia in the majority of cases

Causes: Vitamin D deficiency (osteomalacia). Acute pancreatitis. Chronic renal failure.


Hypoparathyroidism (e.g. post thyroid/parathyroid surgery). Pseudohypoparathyroidism (target cells
insensitive to PTH). Rhabdomyolysis (initial stages). Magnesium deficiency (due to end organ PTH
resistance)

Management: Acute management of severe hypocalcaemia is with intravenous replacement. The preferred
method is with intravenous calcium gluconate, 10ml of 10% solution over 10 minutes. Intravenous calcium
chloride is more likely to cause local irritation. ECG monitoring is recommended. Further management
depends on the underlying cause

32. A 25 year old male pedestrian is hit by a van on a busy road. He is brought to the Emergency
Department by ambulance. On examination he is dyspneoic, and hypoxic despite administration of
high flow 100% oxygen. His blood pressure is 110/70 and pulse rate is 115 bpm. The right side of his
chest is hyper-resonant on percussion and has decreased breath sounds. The trachea is deviated to
the left. What is the most likely underlying diagnosis?
A. Fat embolism
B. Tension pneumothorax
C. Rupture of the right main bronchus
D. Rupture of the diaphragm
E. Pulmonary contusion
Answer: B
Blunt or penetrating chest trauma that creates a flap type defect on the surface of the lung can result in a
tension pneumothorax. Typical features include dyspnoea, progressive hypoxia, hyperresonance and
tracheal deviation. Treatment is with needle decompression and chest tube insertion.

Theme: Management of skin injuries

A. Wound excision and primary closure


B. Simple primary closure
C. Delayed primary closure
D. Debridement and healing by secondary intention
E. Split thickness skin graft
F. Full thickness skin graft
G. Free flap
H. Pedicled flap

For the injuries described please select the most appropriate management. Each option may be used once,
more than once or not at all.

393
33. A 25 year old man is playing with a Pit Bull terrier which bites off a substantial portion of his
nose.

The correct answer is Debridement and healing by secondary intention. Theme from April 2012 Exam
Dog bites are an extremely dirty form of wound and as a result may be most safely managed by being left
to heal by secondary intention. The cosmetic results of this strategy will be significant and major
reconstructive surgery will be needed at a later stage. Some plastic surgeons will attempt a debridement
and primary repair with either a pedicled flap or full thickness graft. The traditional teaching would be for
secondary intention healing.

34. A 7 year old boy falls over and sustains a 6cm laceration to his head. On inspection his wound
contains some dirt in it.

The correct answer is Wound excision and primary closure. By debriding the wound, the area can then be
primarily closed. Prophylactic antibiotics should be administered.

35. A 45 year old man is gardening is puts a fork into his foot. On examination there are cutaneous
defects and the surrounding skin looks dusky.

Debridement and healing by secondary intention. The skin changes described here should be debrided.
Closure would not be safe with the skin changes documented and the wound should be left open.

Methods of wound closure

Primary closure: Clean wound, usually surgically created or following minor trauma. Standard suturing
methods will usually suffice. Wound heals by primary intention

Delayed primary closure: Similar methods of actual closure to primary closure. May be used in situations
where primary closure is either not achievable or not advisable e.g. infection

Vacuum assisted closure: Uses negative pressure therapy to facilitate wound closure. Sponge is inserted
into wound cavity and then negative pressure applied. Advantages include removal of exudate and
versatility. Disadvantages include cost and risk of fistulation if used incorrectly on sites such as bowel

Split thickness skin grafts: Superficial dermis removed with Watson knife or dermatome (commonly from
thigh). Remaining epithelium regenerates from dermal appendages. Coverage may be increased by meshing

Full thickness skin grafts: Whole dermal thickness is removed. Sub dermal fat is then removed and graft
placed over donor site. Better cosmesis and flexibility at recipient site. Donor site "cost"

Flaps: Viable tissue with a blood supply. May be pedicled or free. Pedicled flaps are more reliable, but
limited in range. Free flaps have greater range but carry greater risk of breakdown as they require vascular
anastomosis

36. A 19 year old student is involved in a head on car collision. He complains of severe chest pain. A
Chest x-ray performed as part of a trauma series shows widening of the mediastinum. Which is the
most likely injury in this scenario?
A. Rupture of the distal oesophagus
B. Rupture of the left main bronchus
C. Rupture of the aorta proximal to the left subclavian artery
D. Rupture of the aorta distal to the left subclavian artery
394
E. Rupture of the inferior vena cava
Answer: D
The aorta may be injured in deceleration accidents. In the setting of deceleration injury, chest pain and
mediastinal widening the most likely problem is aortic rupture. This will typically occur distal to the left
subclavian artery. Rupture of the proximal aorta may occur. However, survival is unlikely. It is important
to note that the question uses the term Most likely injury as this is the component that distinguishes an
ascending rupture from a descending rupture.

Thoracic aorta rupture: Mechanism of injury: Decelerating force i.e. RTA, fall from a great height. Most
people die at scene. Survivors may have an incomplete laceration at the ligamentum arteriosum of the aorta.

Clinical features: Contained haematoma: persistent hypotension. Detected mainly by history, CXR changes

CXR changes: Widened mediastinum. Trachea/Oesophagus to right. Depression of left main stem
bronchus. Widened paratracheal stripe/paraspinal interfaces. Space between aorta and pulmonary artery
obliterated. Rib fracture/left haemothorax

Diagnosis: Angiography, usually CT aortogram.

Treatment: Repair or replacement. Ideally they should undergo endovascular repair.

Theme: Management of head and neck trauma

A. Observation
B. CT head within 1h
C. CT head within 8h
D. Urgent neurosurgical review (even before CT head performed)
E. 3 view c-spine xray
F. 2 view c-spine xray
G. CT c-spine

What is the best initial management plan for the injuries described? Each option may be used once, more
than once or not at all.

37. A 22 year old mechanic is involved in a fight. He is hit on the head with a hammer. On examination
he had clinical evidence of an open depressed skull fracture and a GCS of 6/15.

Urgent neurosurgical review (even before CT head performed). A patient with GCS <8 or = to 8 needs
urgent neurosurgical review. Especially when an open fracture is present.

38. A 67 year old retired lawyer falls down the stairs. His GCS is 15/15 and he has some bruising over
the mastoid.

The correct answer is CT head within 1h. This patient has a basal skull fracture, which is indicated by a
positive Battle's sign. He should have a CT head within 1h.

39. A 52 year old secretary falls down the stairs. She complains of neck pain. She has a GCS of 15/15
and no neurology. She is unable to rotate her c-spine 45 degrees to the left and right.

The correct answer is 3 view c-spine xray. She should have her c-spine immobilised and a 3 view c-spine
xray.

395
40. A 28 year old African man is admitted with acute severe abdominal pain. He has just flown into the
UK long haul and the pain developed whilst in flight. On examination he is tender in the left upper
quadrant. His blood tests are as shown. Hb 6 g/dl, Reticulocyte count 15%.
Ultrasound shows a spleen with a heterogeous texture and a few small gallstones but is otherwise
normal. What is the most likely diagnosis?
A. Pancreatitis
B. Parvovirus infection
C. Sickle cell anaemia
D. Pulmonary embolism
E. Beta Thalassaemia minor
Answer: C
A combination of a high reticulocyte count and severe anaemia indicates sickle cell anaemia, however
another differential can be of a transient aplastic crisis due to parvovirus. This is less likely as this causes a
reticulocytopenia rather than a reticulocytosis.

Parvovirus B19 infects erythroid progenitor cells in the bone marrow and causes temporary cessation of
red blood cell production, patients who have underlying hematologic abnormalities are at risk of cessation
of red blood cell production if they become infected. This can result in a transient aplastic crisis. Thus,
patients with sickle cell anaemia are at risk. Typically, these patients have a viral prodrome followed by
anaemia, often with haemoglobin concentrations falling below 5.0 g/dL and reticulocytosis.

Sickle cell anaemia: Autosomal recessive. Single base mutation. Deoxygenated cells become sickle in
shape. Causes: short red cell survival, obstruction of microvessels and infarction. Sickling is precipitated by:
dehydration, infection, hypoxia. Manifest at 6 months age. Africans, Middle East, Indian. Diagnosis: Hb
electrophoresis

Sickle crises: Bone pain. Pleuritic chest pain: acute sickle chest syndrome commonest cause of death. CVA,
seizures. Papillary necrosis. Splenic infarcts. Priapism. Hepatic pain

Hb does not fall during a crisis, unless there is: Aplasia: parvovirus. Acute sequestration. Haemolysis

Long-term complications: Infections: Streptococcus pnemoniae. Chronic leg ulcers. Gallstones:


haemolysis. Aseptic necrosis of bone. Chronic renal disease. Retinal detachment, proliferative retinopathy

Surgical complications: Bowel ischaemia. Cholecystitis. Avascular necrosis

Management: Supportive. Hydroxyurea. Repeated transfusions pre operatively. Exchange transfusion in


emergencies

Sickle cell trait: Heterozygous state. Asymptomatic. Symptoms associated with extreme situations ie
anaesthesia complications. Protective against Plasmodium falciparum

Theme: Blood transfusion reactions

A. Neutrophilic febrile reaction


B. Acute haemolytic transfusion reaction
C. Delayed haemolytic transfusion reaction
D. Pulmonary oedema
E. Sickle cell crisis

396
F. Transfusion associated lung injury
G. Graft vs. Host disease

Please select the most likely underlying cause for each scenario. Each option may be used once, more than
once or not at all.

41. A 22 year old man is having a blood transfusion after losing blood from haemorrhoids. He is
normally fit and well. After 4h, during the transfusion, he complains of sudden onset shortness of
breath and chest pain. On examination his temperature is 37.2, saturations are 88% on air, blood
pressure 100/55 mmHg and HR 110 bpm. He has crepitations bilaterally up to the midzones. He is
given IV frusemide, but deteriorates and is admitted to ITU. A pulmonary catheter is inserted and
the PCWP is 10 mmHg.

The correct answer is Transfusion associated lung injury. The pulmonary catheter reading indicates that
this is not a case of fluid overload (the PCWP should be high, normal values PCWP systolic 7mmHg,
diastolic 10mmHg). Transfusion associated lung injury is a rare reaction causing neutrophilic mediated
allergic pulmonary oedema. Patient's have antibodies to donor leukocytes. It is important to consider this
as a diagnosis when patients don't respond to treatment for pulmonary oedema. Patients normally respond
to supportive therapy including fluids and oxygen.

42. A 32 year male with leukaemia attends the day unit for a blood transfusion. Five days after the
transfusion he attends A&E with a temperature of 38.5, erythroderma and desquamation.

The correct answer is Graft vs. Host disease. This is associated with transfusion of unirradiated blood in
immunosupressed patients. Transfusion associated GVHD can occur 4-30 days after a transfusion and
follows a sub acute pathway. Patients may also have diarrhoea and abnormal liver function tests.
Management involves steroid therapy.

43. A 22 year old man is having a blood transfusion after losing blood from haemorrhoids. He is
normally fit and well. 3h during the transfusion he complains of sudden onset abdominal pain and
nausea. His temperature is 39 degrees, Blood pressure 98/42 mmHg, HR 105 bpm and saturations
94% air. His urine appears dark.

Acute haemolytic transfusion reaction. Rapid intravascular haemolysis leading to shock, DIC and death
can occur with this reaction.

44. A 49-year-old male sustained a severe blunt injury just below the bridge of the nose with industrial
machinery. Imaging demonstrates a fracture involving the superior orbital fissure. On examination
an ipsilateral pupillary defect is present and loss of the corneal reflexes. In addition to these
examination findings, which of the following will not be present?
A. Altered cutaneous sensation from the forehead to the vertex
B. Ptosis
C. Complete opthalmoplegia
D. Nystagmus
E. Enopthalmos
Answer: D
Orbital apex syndrome
This is an extension of superior orbital fissure syndrome and includes compression of the optic nerve
passing through the optic foramen. It is indicated by features of superior orbital fissure syndrome and
ipsilateral afferent pupillary defect.

397
This type of injury will result in the orbital apex syndrome (See above). As such opthalmoplegia will be
present and nystagmus cannot occur.

45. Which option is not recommended during the management of compartment syndrome?
A. Anticoagulation
B. Keep limb level with the body
C. Intravenous fluids
D. Pain control
E. Fasciotomy
Answer: A
Anticoagulation will worsen compartment syndrome.

Theme: Complications of burns

A. Deep vein thrombosis


B. Curlings Ulcer
C. Contracture
D. Type I respiratory failure
E. Type II respiratory failure
F. Toxic shock syndrome
G. Compartment syndrome
H. Rhabdomyolysis
I. Disseminated intravascular coagulation

For each clinical scenario please select the most likely complication to have occurred. Each option may be
used once, more than once or not at all.

46. A 10 year old child is admitted with severe 30% burns following a house fire. After wound cleaning
and dressings he is admitted to critical care. 1 day following skin grafts he becomes tachycardic and
hypotensive. He vomits twice and this shows evidence of haematemesis

Curlings Ulcer. Stress ulcers may occur in the duodenum of burns patients and are more common in
children.

47. A 26 year old electrician suffers a full thickness high voltage burn to his leg. On routine urine
analysis he has + blood. His U+E's show mild hyperkalaemia and a CK of 3000

Rhabdomyolysis. Electrical high voltage burns are associated with rhabdomyolysis. Acute tubular necrosis
may occur. Aggressive IV fluids should be given
48. A 45 year old man is admitted after his clothing caught fire. He suffers a full thickness
circumferential burn to his lower thigh. He complains of increasing pain in lower leg and on
examination there is parasthesia and severe pain in the lower leg. Foot pulses are normal

Compartment syndrome. Circumferential burns may constrict the limb and cause a compartment syndrome
to develop. Eshcarotomy is required, and compartmental decompression.

49. A 23 year old man who plays rugby for a hobby presents with recurrent anterior dislocation of the
shoulder. Which of the following abnormalities is most likely to be present to account for this?
A. Rotator cuff tear
B. Biceps tendon rupture
C. Bankart lesion
D. Axillary nerve injury
398
E. Infraspinatus tendinitis
Answer: C
Anterior dislocations are the most common. When recurrent a Bankart lesion is the most common underlying
abnormality. This is usually visualised by CT and MRI scanning and often repaired arthroscopically.

Proximal humerus fractures: Very common. Usually through the surgical neck. Number of classification
systems though for practical purposes describing the number of fracture fragments is probably easier. Some
key points: It is rare to have fractures through the anatomical neck. Anatomical neck fractures which are
displaced by >1cm carry a risk of avascular necrosis to the humeral head. In children the commonest injury
pattern is a greenstick fracture through the surgical neck. Impacted fractures of the surgical neck are usually
managed with a collar and cuff for 3 weeks followed by physiotherapy. More significant displaced fractures
may require open reduction and fixation or use of an intramedullary device.

Types of shoulder dislocation: Glenohumeral dislocation (commonest): anterior shoulder dislocation most
common. Acromioclavicular dislocation (12%): clavicle loses all attachment with the scapula.
Sternoclavicular dislocation (uncommon)

Types of glenohumeral dislocation:

Anterior shoulder dislocation: External rotation and abduction. 35-40% recurrent (it is the commonest
disorder). Assocociated with greater tuberosity fracture, Bankart lesion, Hill-Sachs defect

Inferior shoulder dislocation: Luxatio erecta

Posterior shoulder dislocation: Proportion misdiagnosed. Rim's sign, light bulb sign. Assocociated with
Trough sign

Superior shoulder dislocation: Rare and usually follow major trauma.

Treatment: Prompt reduction is the mainstay of treatment and is usually performed in the emergency
department. Neurovascular status must be checked pre and post reduction and x-rays should be performed
again post reduction to ensure no fracture has occurred. In recurrent anterior dislocation there is usually a
Bankart lesion and this may be repaired surgically. Recurrent posterior dislocations may be repaired in a
similar manner to anterior lesions but using a posterior (or arthroscopic) approach.

50. A 44 year old man is involved in a road traffic accident. He suffers significant injuries to his thorax,
he has bilateral haemopneumothoraces and a suspected haemopericardium. He is to undergo
surgery, what is the best method of accessing these injuries?
A. Bilateral thoracoscopy and mediastinoscopy
B. Midline sternotomy
C. Bilateral posterolateral thoracotomy
D. Clam shell thoracotomy
E. None of the above
Answer: D
Patients with significant mediastinal and lung injuries are best operated on using a Clam shell
thoracotomy. All modes of access involve a degree of compromise. A sternotomy would give good access
to the heart. However, it takes longer to perform and does not provide good access to the lungs. Trauma
should not be managed using laparoscopy.

51. A 63 year old male is admitted to the surgical ward for an elective femoral popliteal bypass. He
suddenly starts complaining of central, crushing chest pain which is radiating to the left arm. The

399
Nursing staff start high flow oxygen and give a spray of glyceryl trinitrate spray. Unfortunately
there is no relief of symptoms. What is the next agent to be administered?
A. Aspirin 75mg
B. Clopidogrel 75mg
C. Aspirin 300mg
D. Clopidogrel 300mg
E. Direct admission to angiography suite
Answer: C
Aspirin 300mg should be given as soon as possible. If the patient has a moderate to high risk of
myocardial infarction, then Clopidogrel should be given with a low molecular weight heparin.
Thromboloysis or urgent percutaneous intervention should be given if there are significant ECG changes.

Management of acute coronary syndrome: NICE produced guidelines in 2010 on the management of
unstable angina and non-ST elevation myocardial infarction (NSTEMI). They advocate managing patients
based on the early risk assessment using a recognised scoring system such as GRACE (Global Registry of
Acute Cardiac Events) to calculate a predicted 6 month mortality.

All patients should receive: aspirin 300mg. Nitrates or morphine to relieve chest pain if required. Whilst it is
common that non-hypoxic patients receive oxygen therapy there is little evidence to support this approach.
The 2008 British Thoracic Society oxygen therapy guidelines advise not giving oxygen unless the patient is
hypoxic.

Antithrombin: treatment. Low molecular weight heparin should be offered to patients who are not at a high
risk of bleeding and who are not having angiography within the next 24 hours. If angiography is likely
within 24 hours or a patients creatinine is > 265 umol/l unfractionated heparin should be given.

Clopidogrel:300mg should be given to patients with a predicted 6 month mortality of more than 1.5% or
patients who may undergo percutaneous coronary intervention within 24 hours of admission to hospital.
Clopidogrel should be continued for 12 months.

Intravenous glycoprotein IIb/IIIa receptor antagonists: (eptifibatide or tirofiban): should be given to


patients who have an intermediate or higher risk of adverse cardiovascular events (predicted 6-month
mortality above 3.0%), and who are scheduled to undergo angiography within 96 hours of hospital
admission.

Coronary angiography: should be considered within 96 hours of first admission to hospital to patients
who have a predicted 6-month mortality above 3.0%. It should also be performed as soon as possible in
patients who are clinically unstable.

52. Which of the following is not a change found on an ECG in acute pulmonary embolism?
A. No changes
B. J waves
C. P pulmonale
D. Right ventricular strain
E. T wave inversion in the inferior leads
Answer: B
S1, Q3, T3. J waves are pathognomonic of hypothermia.

Pulmonary Embolism: ECG changes: No changes. S1, Q3, T3. Tall R waves: V1. P pulmonale (peaked P
waves): inferior leads. Right axis deviation, Right bundle branch block. Atrial arrhythmias. T wave
inversion: V1, V2, V3. Right ventricular strain: if identified is associated with adverse short-term outcome
and adds prognostic value to echocardiographic evidence of right ventricular dysfunction in patients with
acute pulmonary embolism and normal blood pressure.
400
53. A 42 year old woman is admitted to surgery with acute cholecystitis. She is known to have
hypertension, rheumatoid arthritis and polymyalgia rheumatica. Her medical therapy includes:
Paracetamol 1g qds, Codeine phosphate 30mg qds, Bendrofluazide 2.5 mg od, Ramipril 10mg od,
Methotrexate 7.5mg once a week, Prednisolone 5mg od, You are called by the Senior House Officer
to assess this lady as she has become delirious and hypotensive 2h after surgery. Her blood results
reveal: Na+ 132 mmol/l, K+ 5.3 mmol/l, Urea 7 mmol/l, Creatinine 108 µmol/l, Hb 12.4 g/dl,
Platelets 178 * 109/l, WBC 15.4 * 109/l. What management is needed immediately?
A. Ceftriaxone IV
B. Hydrocortisone 50mg IV
C. CT scan abdomen
D. Urgent exploration laparotomy
E. Hydrocortisone 100mg IV
Answer: E
This patient has acute adrenal insufficiency and urgently needs steroid replacement.

Theme: Thoracic trauma

A. Thoracotomy
B. Manage conservatively
C. Intercostal tube drain insertion
D. CT scanning
E. Bronchoscopy
F. Negative pressure intercostal tube drainage
G. Video assisted thoracoscopy and pleurectomy

For each of the following scenarios please select the most appropriate management option from the list.
Each option may be used once, more than once or not at all.

54. A 28 year old male is involved in a road traffic accident he is thrown from his motorbike onto the
pavement and sustains a haemopneumothorax and flail segment of the right chest

The correct answer is Intercostal tube drain insertion. He requires a chest drain and analgesia. In general all
haemopneumothoraces should be managed by intercostal chest drain insertion as they have a risk of
becoming a tension pneumothorax until the lung laceration has sealed.
55. A 19 year old man is stabbed in the chest at a nightclub. He develops a cardiac arrest in casualty
following an attempted transfer to the CT scanning room

Thoracotomy. This is one indication for an 'emergency room' thoracotomy, there are not many others!
Typical injuries include ventricular penetration, great vessel disruption and hilar lung injuries.
56. A 32 year old male falls over and sustains a small pneumothorax following a simple rib fracture. He
has no physiological compromise

The correct answer is Intercostal tube drain insertion. Unlike spontaneous pneumothoraces most would
advocate chest tube drainage in the context of pneumothorax resulting from trauma. This is because of the
risk of the lung laceration developing a tension. Once there is no further evidence of air leak the chest drain
may be removed and a check x-ray performed to check there is no reaccumulation prior to discharge.

57. A 21 year old man is undergoing an inguinal hernia repair and receives a does of intravenous co-
amoxyclav. He is reported to have a penicillin allergy. Over the next few minutes his vital signs are:
Pulse - 130bpm, blood pressure- 60/40mmHg. What is the first line treatment?
401
A. Hydrocortisone 100mg IV
B. Adrenaline 1:1000 IV
C. Chlorpheniramine 10mg IV
D. Adrenaline 1:1000 IM
E. Adrenaline 1:10000 IV
Answer: D
Theme from 2009 Exam. The first line treatment of anaphylactic shock is intra muscular adrenaline.
Anaphylactic shock: Suspect if there has been exposure to an allergen
Management: Remove allergen. ABCD. Drugs:
Adrenaline 1:1000 0.5ml INTRAMUSCULARLY (not IV). Repeat after 5 mins if no response. Then
Chlorpheniramine 10mg IV. Then Hydrocortisone 100-200mg IV

Theme: Types of stroke

A. Anterior cerebral artery infarct


B. Middle cerebral artery infarct
C. Posterior cerebral artery infarct
D. Pituitary mass
E. Lacunar infarct
F. Lateral medullary syndrome
G. Pontine infarct
H. Horner's syndrome
I. Cerebellar infarct

Please select the most likely cause for the symptoms given. Each option may be used once, more than once
or not at all.

58. A 53 year old teacher is admitted to the vascular ward for a carotid endarterectomy. Your
houseman does a preoperative assessment and notes that there is a right homonymous hemianopia.
There is no other neurology.

Posterior cerebral artery infarct. This patient has had a left occipital infarct, as there is only a
homonymous hemianopia. If this patient had a temporal or parietal lobe infarct, there would be associated
hemiparesis and higher cortical dysfunction. This is important to differentiate, as the carotid
endarterectomy is inappropriate in this patient as the lesion is in the posterior cerebral artery.

59. A 52 year man is admitted to the vascular ward for an amputation. The patient complains of
unsteadiness. On further examination you detect right facial numbness and right sided nystagmus.
There is sensory loss of the left side.

The correct answer is Lateral medullary syndrome. A combination of ipsilateral ataxia, nystagmus,
dysphagia, facial numbness, cranial nerve palsy with contralateral hemisensory loss indicates this
diagnosis.

60. A 48 year old type 2 diabetic complains of numbness in his left arm and leg. Otherwise there is no
other neurological signs.

Lacunar infarct. Isolated hemisensory loss is a feature of a lacunar infarct.

61. A 22 year old man has a full thickness burn on his chest. It is well circumscribed. In A&E his
saturations are reduced to 92% on 15L Oxygen, Blood pressure 102/66 mmHg and HR 105bpm.
What is the best management?
A. Haemodialysis
B. Escharotomy
402
C. Fasciotomy
D. Cardiac bypass
E. Non invasive ventilation
Answer: B
The chest burn and its associated oedema is limiting respiration. Therefore an escharotomy of the chest is
indicated, allowing the breast plate to mobilise causing ventilation.

62. A 16 year old man sustains a basal skull fracture and is suspected of having CSF rhinorrhoea.
Which of the following laboratory tests would most accurately identify whether CSF is present or
not?
A. Microscopy to identify red blood cells
B. Lab stix testing for glucose
C. Lab stix testing for protein
D. Beta 2 transferrin assay
E. Microscopy, gram stain and culture
Answer: D
Beta 2 transferrin is a carbohydrate free form of transferrin that is almost exclusively found in the CSF.
Although lab stix testing for glucose is traditional it is associated with false positive results secondary to
contamination with other glucose containing bodily secretions.

63. A 66 year old male is admitted to the vascular ward for an amputation. He reports episodes of
vertigo and dysarthria to the house officer. He sudden;y collapses with a glasgow coma score of 3.
What is the most likely diagnosis?
A. Cerebral haemorrhage in left temporal parietal area
B. Opiate overdose
C. Cerebral haemorrhage in right temporal parietal area
D. Diazepam overdose
E. Basilar artery occlusion
Answer: E
Vertigo and dysarthria suggest a posterior circulation event. In the scenario of a patient complaining of
posterior symptoms and a sudden deterioration in consciousness, the main differential diagnosis is of a
basilar artery occlusion.

64. A 19 year intravenous drug abuser is recovering following a surgical drainage of a psoas abscess. He
is found collapsed in the ward toilet unresponsive and with pinpoint pupils. What is the most
appropriate immediate management?
A. Intravenous flumazenil
B. Intravenous nalaxone
C. Intravenous benxhexol
D. No further management
E. Intravenous glycopyrolate
Answer: B
Intravenous nalaxone is needed to treat the patient who has had an overdose of opiate. Naloxone has the
quickest onset of action, however it is important to be aware of its short acting duration and the need for
further administration. There is also the risk of rebound pain once naloxone is given.

Opioid misuse: Opioids are substances which bind to opioid receptors. This includes both naturally
occurring opiates such as morphine and synthetic opioids such as buprenorphine and methadone.

Features of opioid misuse: Rhinorrhoea. Needle track marks. Pinpoint pupils. Drowsiness. Complications of
intravenous opioid misuse: Viral infection secondary to sharing needles: HIV, hepatitis B & C. Bacterial
infection secondary to injection: infective endocarditis, septic arthritis, septicaemia, necrotising fasciitis,

403
groin abscess. Pseudoaneurysm. Venous thromboembolism. Osteomyelitis. Overdose may lead to
respiratory depression and death

Emergency management of opioid overdose: IV or IM naloxone: has a rapid onset and relatively short
duration of action

65. A 68 year old male is admitted to the surgical ward for assessment of severe epigastric pain. His
abdomen is soft and non tender. However the Nurse forces you to look at the ECG. It looks
abnormal. Which of the following features is an indication for urgent coronary thrombolysis or
percutaneous intervention?
A. Right bundle branch block
B. ST elevation of 1mm in leads V1 to V6
C. Ventricular tachycardia
D. Q waves in leads V1 to V6
E. ST elevation of 1mm in leads II, III and aVF
Answer: E
ECG changes for thrombolysis or percutaneous intervention:
ST elevation of > 2mm (2 small squares) in 2 or more consecutive anterior leads (V1-V6) OR

ST elevation of greater than 1mm (1 small square) in greater than 2 consecutive inferior leads (II,
III, avF, avL) OR

New Left bundle branch block: ST elevation of 1mm in leads II, III and aVF reflects significant cardiac
ischaemia due to the right coronary artery occlusion. The medical registrar should be contacted to urgently
assess the patient. Note right coronary artery occlusions puts the patient at risk of cardiac arrhythmias (due
to blood supply to the sino atrial node).

Thrombolysis or percutaneous intervention in myocardial infarction: Thrombolytic drugs activate


plasminogen to form plasmin. This in turn degrades fibrin and help breaks up thrombi. They in primarily
used in patients who present with a ST elevation myocardial infarction. Other indications include acute
ischaemic stroke and pulmonary embolism, although strict inclusion criteria apply.

Examples: Alteplase. Tenecteplase. Streptokinase

Contraindications to thrombolysis: Active internal bleeding. Recent haemorrhage, trauma or surgery


(including dental extraction). Coagulation and bleeding disorders. Intracranial neoplasm. Stroke < 3 months.
Aortic dissection. Recent head injury. Pregnancy. Severe hypertension

Side-effects: haemorrhage. Hypotension - more common with streptokinase. Allergic reactions may occur
with streptokinase.

66. Which of the following is not typically associated with a degloving injury?
A. Overlying pallor of the skin
B. Abnormal motility of the overlying skin
C. History of friction type injury
D. Improved results when the degloved segment is left in situ as a temporary closure
E. Poor results when primary compression treatment is used in preference to skin grafting
Answer: D
Degloving injuries typically involve extremities and are usually friction injuries eg arm being run over.
There is abnormal motility of the overlying skin, pallor, loss of sensation. Early treatment is key and
should involve skin grafting which may use the degloved segment. This however, should be formally
prepared for the role and simple compression bandaging gives poor results.

404
67. Which of the following statements relating to large volume blood loss in trauma is incorrect?
A. Tranexamic acid reduces the incidence of rebleeding following surgery
B. Hypocalcaemia may complicate resuscitation
C. Colloids are preferred initially as they reduce the incidence of coagulopathy
D. When patients receive over 5 units of whole blood mortality increases when blood products greater than 3
weeks old are utilised
E. In the battlefield setting a ratio of 1 unit fresh blood to 1 unit plasma is often utilized
Answer: C
Fresh blood is the fluid of choice when large volume blood loss complicates trauma. Mortality is doubled
when blood >3 weeks old is used.

68. The following features are typical of superficial dermal burns except:
A. They are usually erythematous
B. Blistering should not occur
C. Spontaneous healing will occur in nearly all cases
D. They do not extend deeper than the proximal dermal papillae
E. Capillary return should be present
Answer: B
Superficial dermal burns are typically erythematous, do not extend beyond the upper part of the dermal
papillae, capillary return and blisters are both usually present.

69. Which of the following is not a feature found on a CXR in traumatic aortic disruption?
A. Widened mediastinum
B. Trachea deviated to the left
C. Depression of the left main stem bronchus
D. Obliteration of the aortic knob
E. Widened paraspinal interfaces
Answer: B
The trachea is normally deviated to the right.

Theme: Head injury management

A. Observation
B. CT head within 1h
C. CT head within 8h
D. Urgent neurosurgical review (even before CT head performed)
E. 3 view c-spine xray
F. 2 view c-spine xray
G. CT c-spine
H. MRI c-spine

What is the best initial management plan for the scenario given? Each option may be used once, more than
once or not at all.

70. A 22 year old male falls of a ladder. He complains of neck pain and cannot feel his legs. His GCS
suddenly deteriorates and a CT head confirms an extradural haematoma. What is the best imaging
for his neck?

The correct answer is CT c-spine. This man needs a CT scan of his c-spine. A CT scan will give the best
resolution of any bony injury.

71. A 25 year old teacher falls down the stairs. She complains of a headache and has vomited 3 times.
She has a GCS of 15/15.

405
CT head within 1h. This lady has a head injury and vomiting > 1, therefore an urgent CT head is indicated.

72. 18 year old student is shot in the back of the head.

The correct answer is Urgent neurosurgical review (even before CT head performed). A penetrating injury
needs urgent neurosurgical review.

Theme: Head injury management

A. Observation
B. CT head within 1h
C. CT head within 8h
D. Urgent neurosurgical review (even before CT head performed)
E. 3 view c-spine xray
F. 2 view c-spine xray
G. CT c-spine
H. MRI c-spine

What is the best initial management plan for the scenario given? Each option may be used once, more than
once or not at all.

73. A 22 year old male falls of a ladder. He complains of neck pain and cannot feel his legs. His GCS
suddenly deteriorates and a CT head confirms an extradural haematoma. What is the best imaging
for his neck?

The correct answer is CT c-spine. This man needs a CT scan of his c-spine. A CT scan will give the best
resolution of any bony injury.

74. A 25 year old teacher falls down the stairs. She complains of a headache and has vomited 3 times.
She has a GCS of 15/15.

CT head within 1h. This lady has a head injury and vomiting > 1, therefore an urgent CT head is indicated.

75. An 18 year old student is shot in the back of the head.

The correct answer is Urgent neurosurgical review (even before CT head performed). A penetrating injury
needs urgent neurosurgical review.

76. A 60-year-old man develops palpitations while on the acute surgical unit. An ECG shows a broad
complex tachycardia at a rate of 150 bpm. His blood pressure is 124/82 mmHg and there is no
evidence of heart failure. The surgical consultant wants to give rate control (the medical team are
not answering their bleeps). Which one of the following is it least appropriate to give?
A. Procainamide
B. Lidocaine
C. Synchronised DC shock
D. Adenosine
E. Verapamil
Answer: E
Ventricular tachycardia - verapamil is contraindicated

Verapamil should never be given to a patient with a broad complex tachycardia as it may precipitate
406
ventricular fibrillation in patients with ventricular tachycardia. Adenosine is sometimes given in this
situation as a 'trial' if there is a strong suspicion the underlying rhythm is a supraventricular tachycardia with
aberrant conduction

Ventricular tachycardia: management: Whilst a broad complex tachycardia may result from a
supraventricular rhythm with aberrant conduction, the European Resuscitation Council advise that in a peri-
arrest situation it is assumed to be ventricular in origin.

If the patient has adverse signs (systolic BP < 90 mmHg, chest pain, heart failure or rate > 150 beats/min)
then immediate cardioversion is indicated. In the absence of such signs antiarrhythmics may be used. If
these fail, then electrical cardioversion may be needed with synchronised DC shocks

Drug therapy: Amiodarone: ideally administered through a central line. Lidocaine: use with caution in
severe left ventricular impairment. Procainamide. Verapamil should NOT be used in VT

If drug therapy fails: electrophysiological study (EPS). Implant able cardioverter-defibrillator (ICD) - this is
particularly indicated in patients with significantly impaired LV function

77. A 24 year old man is admitted to A&E with 35% full thickness burns after being involved in a house
fire. Which fluid is normally avoided during resuscitation in the first 8-12h?
A. Plasmalyte
B. Hartmann's
C. Albumin solution
D. Dextrose saline
E. Dextran 40
Answer: C
Albumin causes increased fluid into the interstitial space, therefore is avoided in the first 8-24h (variable
between different departments).

78. A 62 year old male attends the hernia clinic. He suddenly develops speech problems, left facial
weakness and left sided arm and leg weakness lasting longer than 5 minutes. What is the next line of
management?
A. Aspirin 300mg
B. Aspirin 75 mg
C. Clopidogrel 300mg
D. Urgent referral for thrombolysis
E. Carotid endarterectomy
Answer: D
This patient is within 3h of symptom onset of a stroke. Therefore he should be urgently referred to the
medical team for thrombolysis, before Aspirin is given. This is an example of the type of medical problem
you should be aware of as a surgeon, as ultimately you can make a difference by referring QUICKLY to
the correct specialty for management.

79. A 45-year-old man is seen in the Emergency Department with nausea, pallor and lethargy. He has
no past medical history of note. A cannula is inserted and serum urea and electrolytes show the
following: Na+ 140 mmol/l, K+ 6.7 mmol/l, Bicarbonate 14 mmol/l, Urea 18.2 mmol/l, Creatinine 230
µmol/l. What is the most appropriate initial management?
A. Nebulised salbutamol
B. Intravenous bicarbonate
C. Haemodialysis
D. Insulin/dextrose infusion
E. Intravenous calcium gluconate
Answer: D
407
The first priority in this patient is to stabilise the myocardium with intravenous calcium gluconate.
Management of hyperkalaemia. Untreated hyperkalaemia may cause life-threatening arrhythmias.
Precipitating factors should be addressed (e.g. acute renal failure) and aggravating drugs stopped (e.g. ACE
inhibitors). Management may be categorised by the aims of treatment.
Stabilisation of the cardiac membrane: Intravenous calcium gluconate
Short-term shift in potassium from extracellular to intracellular fluid compartment: Combined
insulin/dextrose infusion and Nebulised salbutamol
Removal of potassium from the body: Calcium resonium (orally or enema), Loop diuretics, Dialysis

Theme: Visceral injury

A. Ruptured spleen
B. Ileum injury
C. Duodenal injury
D. Urethral injury
E. Rectal injury
F. Oesophageal injury
G. Liver laceration

Please select the most likely injury for the scenario given. Each option may be used once, more than once or
not at all.

80. A motorcyclist is involved in a head on road traffic accident with a lorry. He is comatose at scene
and trauma series xrays confirm a pelvic fracture. On rectal examination he has a high riding
prostate.

Urethral injury. This is classical for urethral injury. Features of a urethral injury include; pelvic fracture,
high riding prostate on digital rectal examination and blood at the urethral meatus. Where this is the
suspected diagnosis a suprapubic catheter and urethral contrast studies performed.

81. A cyclist loses control and falls off the side of a road landing on the bicycle handlebars. CT scanning
shows a large amount of retroperitoneal air.

Duodenal injury. Retroperitoneal air is more likely with a duodenal injury. As it is largely retroperitoneal.
A handlebar type injury is the commonest cause and the pancreas should be carefully inspected as it too
may be injured. It would be unusual for the ileum to be injured in this type of scenario as it is mobile.

82. A 23 year old man is shot in the abdomen. He is haemodynamically stable but on ultrasound he has
a large about of intra abdominal free fluid.

Ileum injury. Small bowel injury is the most common type of injury in this scenario. The enteric contents
will tend to result in a large amount of intra abdominal fluid.

83. A 22 year old man has a full thickness burn of his leg after being trapped in a burning car. There
are no fractures of the limb. There burn is well circumscribed. After 2 hours he complains of
tingling of his leg and it appears dusky. What is the best management for this?
A. Fasciotomy
B. Escharotomy
C. Angioplasty
D. Pain control

408
E. Anticoagulation
Answer: B
The full thickness burn has oedema which is affecting the peripheral circulation. Therefore the burn needs
to be divided (not the fascia) to allow normal circulation to return.

84. A 28 year old man is in the surgical intensive care unit. He has suffered a flail chest injury several
hours earlier and he was intubated and ventilated. Over the past few minutes he has become
increasingly hypoxic and is now needing increased ventilation pressures. What is the most common
cause?
A. Pulmonary embolism
B. Cardiac tamponade
C. Fat embolism
D. Tension pneumothorax
E. Adult respiratory distress syndrome
Answer: D
A flail chest segment may lacerate the underlying lung and create a flap valve. A tension pneumothorax
can be created by intubation and ventilation in this situation. Sudden hypoxia and increased ventilation
pressure are clues. Theme from April 2011 Exam

85. A 10 year old boy is playing with a firework which explodes and he sustains a full thickness burn to
his left arm. Which of the following statements is not characteristic of this situation?
A. They have a leathery appearance
B. The burn area is extremely painful until skin grafted
C. They always heal with scarring
D. Blanching does not occur under pressure
E. Absence of,or few, blisters
Answer: B
Full thickness burns involve complete injury to the dermis and sub dermal appendages. They have a
leathery, often white appearance. They are initially insensate although pain often occurs during healing
following skin grafting. They do not blanch under pressure.

Theme: Thoracic injuries

A. Pneumothorax
B. Tension pneumothorax
C. Flail chest
D. Cardiac tamponade
E. Aorta rupture
F. Cardiac contusion
G. Diaphragmatic rupture
H. Acute phrenic nerve injury

For each of the scenarios given, please select the most likely underlying injury. Each option may be used
once, more than once or not at all.

86. An 18 year old student is involved in a car crash, with another car crashing into the side of the car.
A CXR shows an indistinct left hemidiaphragm.

The correct answer is Diaphragmatic rupture. A lateral blunt injury during a road traffic accident is a
common cause of diaphragmatic rupture. Diagnosis is usually evident on chest x-ray. CXR changes
include non visible diaphragm, bowel loops in the hemithorax and displacement of the mediastinum. In
most cases direct surgical repair is the best option.
409
87. A 19 year old motorcyclist is involved in a road traffic accident. His chest movements are irregular.
He is found to have multiple rib fractures, with 2 fractures in the 3rd rib and 3 fractures in the 4th
rib.

Flail chest. Multiple rib fractures with > or = 2 rib fractures in more than 2 ribs is diagnosed as a flail
chest. This is associated with pulmonary contusion.

88. A 19 year old student falls from a 2nd floor window. He is persistently hypotensive. A CXR shows
depression of the left main bronchus and deviation of the trachea to the right.

Aorta rupture. He has a deceleration injury, with persistent hypotension (contained haematoma). This
should indicate aorta rupture. Widened mediastinum may not always be present on a CXR. A CT
angiogram will provide clearer evidence of the extent of injury. The presence of persistent hypotension,
from a early stage is more consistent with haematoma than a tension pneumothorax in which it occurs as a
final periarrest phenomena. CXR findings in diaphragmatic rupture: Hemidiaphragm is not visible. Bowel
loops in the lower half of the hemi-thorax. Mediastinum is displaced

89. A 22 year old man is involved in a motorcycle accident. He suffers from bilateral tibial and fibula
fractures. He is taken to theatre and intramedullary nails are inserted. 6h after surgery he
complains of severe pain in his limb, which increases on passive plantar flexion. What is the best
management plan?
A. Anticoagulation
B. Fasciotomy
C. Intravenous analgesia and clinical review in 8 hours
D. Exploration of tibia for displaced nail
E. Pregabalin
Answer: B
Do not forget to decompress the deep muscle layer during a fasciotomy. This patient has compartment
syndrome and needs a fasciotomy.

90. Which of the features below, following a head injury, is not an indication for an immediate CT head
scan in children?
A. Drowsiness
B. A single, discrete episode of vomiting
C. A 9 month old child with a 6cm haematoma on the head
D. Numb left arm
E. Suspicion of a non accidental head injury
Answer: B
Whilst not an indication for immediate CT there should be a low threshold for admission and observation.

Head injury paediatrics: Criteria for immediate request for CT scan of the head (children)

*Loss of consciousness lasting more than 5 minutes (witnessed). * Amnesia (antegrade or retrograde) lasting
more than 5 minutes. * Abnormal drowsiness. * Three or more discrete episodes of vomiting. * Clinical
suspicion of non-accidental injury. * Post-traumatic seizure but no history of epilepsy. * GCS less than 14,
or for a baby under 1 year GCS (paediatric) less than 15, on assessment in the emergency department. *
Suspicion of open or depressed skull injury or tense fontanelle. * Any sign of basal skull fracture
(haemotympanum, ‘panda' eyes, cerebrospinal fluid leakage from the ear or nose, Battle's sign). * Focal
neurological deficit. * If under 1 year, presence of bruise, swelling or laceration of more than 5 cm on the

410
head. * Dangerous mechanism of injury (high-speed road traffic accident either as pedestrian, cyclist or
vehicle occupant, fall from a height of greater than 3 m, high-speed injury from a projectile or an object)

Theme: Management of osteomyelitis

A. Lautenbach regime
B. Below knee amputation
C. Hindquater amputation
D. Above knee amputation
E. Removal of metalwork and implantation of local antibiotics
F. Removal of metalwork and bone grafting
G. Intravenous antibiotics

Which option is the best management plan? Each option may be used once, more than once or not at all

91. A 65 year old type 2 diabetic with poor glycaemic control is admitted with forefoot cellulitis. X-ray
of the foot shows some evidence of osteomyelitis of the 2nd ray but overlying skin is healthy.

Intravenous antibiotics. It is worth attempting to try and resolve this situation with antibiotics at first
presentation. A primary amputation will not heal well and may result in progressive surgery.

92. A 28 year old infantryman is shot in the leg during combat. Primary debridement and lavage of the
wound is undertaken. Several months post surgery there is ongoing discharge from a sinus
originating in the proximal femur, X-ray and MRI shows evidence of osteomyelitis of the proximal
femur. There are no obvious sequestra.

The correct answer is Lautenbach regime. This involves local administration of antibiotics via
intramedullary lines and is an intensive regime. However, the morbidity of a high above knee or
hindquater amputation makes conservative management an attractive option.

93. A 70 year old man undergoes a revision total hip replacement. 10 days post operatively the hip
dislocates and pus is discharging from the wound. He is systemically unwell with a temperature of
38.5 and WCC 19.

Removal of metalwork and implantation of local antibiotics. Removal of metal work implantation of
gentamicin beads and delayed revision is the mainstay of managing this complication.

Osteomyelitis: Infection of the bone. Causes: S aureus and occasionally Enterobacter or Streptococcus
species. In sickle cell: Salmonella species. Clinical features: Erythema. Pain. Fever. Investigation: X-ray:
lytic centre with a ring of sclerosis. Bone biopsy and culture. Treatment: Prolonged antibiotics. Sequestra
may need surgical removal

94. What is the least likely examination finding in patients with Le Fort II fractures?
A. Excessive mobility of the palate
B. Paraesthesia in the region supplied by the inferior alveolar nerve
C. Malocclusion of the teeth
D. Endopthalmos
E. Parasthesia in the region supplied by the infraorbital nerve
Answer: B
Le Fort II fractures have a pyramidal shape. The fracture line involves the orbit and extends to involve the
bridge of the nose and the ethmoids. In continues to involve the infraorbital rim and usually through the
infraorbital foramen. As a result infraorbital parasthesia, palatal mobility and malocclusion are common

411
findings. Severe fractures may result in endopthalmos. However, the fracture does not, by definition,
involve the inferior alveolar nerve.

95. A Medical F1 phones you as he is concerned his patient has had a major internal bleed. The patient
is 42 years old and is known to have sickle cell anaemia. His blood results are:
Hb 3.7 g /dl, Reticulocyte count 0.4%. His Hb is normally 7g/dl. What is the diagnosis?
A. Psoas haemorrhage
B. Acute sequestration
C. Parvovirus
D. Splenic haemorrhage
E. Acute haemolysis
Answer: C
A sudden anemia and a LOW reticulocute count indicates parvovirus. Acute sequestration and haemolysis
causes a high reticulocyte count. There is no clinical indication to suspect a bleed, therefore you can advise
the F1 not to panic and to speak to the haematologists!

Theme: Management of chest trauma

A. Thoracotomy in operating theatre


B. 36F intercostal chest drain
C. 14F intercostal chest drain
D. Active observation
E. Thoracotomy in the emergency room
F. MRI of aortic arch
G. Bronchoscopy
H. Pericardiocentesis
I. Further transfusion

For each of the following scenarios please select the most appropriate next stage of management. Each
option may be used once, more than once or not at all.

96. A 30 year old male is stabbed outside a nightclub he has a brisk haemoptysis and in casualty has a
chest drain inserted into the left chest. This drained 750ml frank blood. He fails to improve with this
intervention. He has received 4 units of blood. His CVP is now 13.

The correct answer is Pericardiocentesis. This man has cardiac tamponade. The raised CVP in the setting
of haemodynamic compromise is the pointer to this. Whilst he will almost cetainly require surgery, he
requires ungent deompresion of his heart first.

97. A 26 year old male falls from a cliff. He suffers from multiple fractures and has a right sided
pneumothorax that has collapsed a 1/3 of his lung. He has no respiratory compromise.

14F intercostal chest drain. Simple observation is unsafe as he will almost certainly have suffered an
oblique laceration to his lung. These can become tension pneumothoraces. In the absence of blood a 36 F
drain is probably not required

98. An 18 year old male is shot in the left chest he was unstable but his blood pressure has improved
with 1 litre of colloid. His chest x-ray shows a left sided pneumothorax with no lung visible.

36F intercostal chest drain. This man requires wide bore intercostal tube drainage. Smaller intercostal
chest drains can become occluded with blood clot and fail to function adaquetly.

412
Theme: Management of burns

A. Escharotomy
B. Endotracheal intubation
C. Broad spectrum intravenous antibiotics
D. Intravenous fluids calculated according to extent of burned area
E. Discharge with review in outpatients
F. Transfer to regional burn centre once stabilised
G. Split thickness skin graft
H. Full thickness skin graft

What is the best management for the scenario given? Each option may be used once, more than once or not
at all.

99. A 34 year old women trips over and falls into a bonfire whilst intoxicated at a party. She suffers
burns to her arms, torso and face. These are calculated to be 25% body surface area. She is
otherwise stable. The burns to the torso are superficial, her left forearm has a full thickness burn
and the burns to her face are superficial. There is no airway compromise. She has received 1000ml
of intravenous Hartman's solution, with a further 1000ml prescribed to run over 4 hours.

Transfer to regional burn centre once stabilized. This women has been resuscitated and requires transfer
for specialist management

100. A 20 year old man is trapped in a warehouse fire. He has sustained 60% burns to his torso and
limbs. The limb burns are partial thickness but the torso burns are full thickness. He was intubated
by paramedics at the scene and is receiving intravenous fluids. His ventilation pressure
requirements are rising.

Escharotomy. He requires an escharotomy as this will be contributing to impaired ventilation

101. An 18 year old man accidentally pours boiling water onto his left arm. The area is erythematous
and has a blister measuring 5cm. The wound is extremely painful.

Discharge with review in outpatients. This is a superficial burn and should recover with no further input
than simple dressings, an alternative would be deroofing the blister and applying dressings prior to
outpatient review

102.A 56-year-old female is admitted to ITU with a severe pancreatitis. Thyroid function tests show:
TSH = 0.5 Low, Thyroxine = 1.0 Low, T3 = 0.5 Low. What is the most likely cause?
A. Sick euthyroid syndrome
B. Graves disease
C. Hashimotos thyroiditis
D. Levothyroxine
E. None of the above
Answer: A
This patient has sick euthyroid syndrome as all thyroid parameters are reduced. Graves disease and
levothyroxine will cause hyperthyroidism (low TSH and elevated thyroxine/T3). Hashimotos thyroiditis
is associated with hypothyroidism (high TSH and low thyroxine/T3).

Sick euthyroid syndrome: In sick euthyroid syndrome (now referred to as non-thyroidal illness) it is often
said that everything (TSH, thyroxine and T3) is low. In the majority of cases however the TSH level is
within the normal range (inappropriately normal given the low thyroxine and T3). Changes are reversible
upon recovery from the systemic illness.
413
Vascular Surgery
Theme: Amputations

A. Transfemoral amputation
B. Gritti - Stokes amputation
C. Digital amputation
D. Syme's amputation
E. Hindquarter amputation
F. Below knee amputation
G. Trans metatarsal amputation
H. Amputation of digit

Please select the most appropriate procedure for the scenario given. Each option may be used once, more than once or not at all.

1. The operation of choice for a 90 year old lady with infected gangrene of the mid foot secondary to diabetes. She has fixed
flexion deformity of the knee.

The correct answer is Transfemoral amputation. An elderly patient with diabetes and peripheral vascular disease is a high risk
surgical candidate. It is important that the chances of a successful outcome are maximised at the first operation. SInce above
knee amputations usually heal more reliably than below knee amputations this is a preferable option, especially since she has a
fixed deformity.

2. An operation in which Skew flaps are created.

The correct answer is Below knee amputation. This is one variant of a below knee amputation. The Burgess flap is the other
commonly practised approach.

3. An amputation of the lower limb in which the femoral condyles are removed and the patella retained.

The correct answer is Gritti - Stokes amputation. This is a Gritti - Stokes amputation. During a Gritti - Stokes operation the
patella is conserved and swung posteriorly to cover the distal femoral surface. Beware performing amputations in patients with
peripheral vascular disease without optimising inflow first!

Amputations: Amputations are indicated when the affected limb is one of the following: Dead non viable. Deadly where it is
posing a major threat to life. Dead useless where it is viable but a prosthesis would be preferable

Orthopaedic surgery:Amputation is often undertaken as an option of last resort e.g. Limb salvage has failed and the limb is so
non functional that mobility needs would be best met with prosthesis. Chronic fracture non union or significant limb shortening
following trauma would fit into this category. Occasionally following major trauma a primary amputation is preferable. This
would be the case in an open fracture with major distal neurovascular compromise and other more life threatening injuries are
present.

Vascular surgery: The first two categories are the most prevalent. Diabetic foot sepsis is often a major cause of sepsis which can
spread rapidly in the presence of established peripheral vascular disease. As a general rule the main issue in vascular surgery is to
optimise vascular inflow prior to surgery. The more distal the planned amputation is to be, the more important this rule becomes.
In other situations there has been something such as an embolic event that has not been revascularised in time. In this case the
limb shows fixed mottling and an amputation will be needed.

Types of amputations: As the vast majority of commonly performed amputations affect the lower limbs these will be covered
here.

The main categories of amputations are: Pelvic disarticulation (hindquarter). Above knee amputation. Gritti Stokes (through knee
amputation). Below knee amputation (using either Skew or Burgess flaps). Syme's amputation (through ankle). Amputations of
mid foot and digits

Choosing a level of amputation depends on: The disease process being treated. Desired functional outcome. Co-morbidities of the
patient

414
Above knee amputations: Quick to perform. Heal reliably. Patients regain their general health quickly. For this benefit, a
functional price has to be paid and many patients over the age of 70 will never walk on an above knee prosthesis. Above knee
amputations use equal anterior-posterior flaps.

Below knee amputations: Technically more challenging to perform. Heal less reliably than their above knee counterparts.
However, many more patients are able to walk using a below knee prosthesis. In below knee amputations the two main flaps are
Skew flaps or the Burgess
Long posterior flap. There is some evidence that Skew flaps are better vascularised than the long posterior flap and some vascular
surgeons prefer them for this reason.

It is worth remembering that whilst it may be technically feasible to offer a below knee amputation there may be circumstances
where an above knee option is preferable. For example, in fixed flexion deformities of the lower limb, little functional benefit
would be gained from below knee amputation surgery.

Theme: Vascular disorders affecting the upper limb

A. Proximal brachial artery occlusion secondary to atheroma


B. Distal brachial artery occlusion secondary to atheroma
C. Axillary artery embolus
D. Axillary vein thrombosis
E. Cervical rib
F. Raynaud's disease
G. Rheumatoid disease

Please select the most likely cause for the presenting scenario described. Each option may be used once, more than once or not at
all.

4. A 73 year old male presents with a collapse and is brought to the emergency department. On examination he has a cold,
painful left hand and forearm.

The correct answer is Axillary artery embolus. Sudden arterial embolus will affect the axillary artery in up to 30% cases.
Because of the acute nature of the condition there is not time for the development of a collateral circulation so the limb is usually
pale and painful. Emboli occur usually occur as a result of atrial fibrillation. Fast atrial fibrillation can cause syncope and an
acute embolus. Theme from September 2012 Exam
5. A 23 year old man presents with intermittent symptoms of altered sensation in his arm and discomfort when he uses his
hands. He works as an electrician and his symptoms are worst when he is fitting light fixtures.

Cervical rib. Compression of the thoracic outlet by the fibrous band of the "rib" can result in both neurological and circulatory
compromise. When manual tasks are performed in which the hand works overhead the signs and symptoms will be maximal and
this is the basis of Adsons test.
6. A 19 year old lady presents with recurrent episodes of pain in her hands. She notices that her symptoms are worst in cold
weather. When she gets the pain she notices that her hands are very pale, they then become dark blue in colour.

Raynaud's disease. Raynauds disease is characterised by a series of colour changes and discomfort is often present. The young
age at presentation coupled with the absence of a smoking history (in most cases) makes occlusive disease unlikely.

Vascular disorders of the upper limb: Upper limb arterial disease is less common than lesions causing symptoms in the lower
limb. The upper limb circulation may be affected by embolic events, stenotic lesions (both internal and extrinsic), inflammatory
disorders and venous diseases.
The anatomy of the collateral circulation of the arterial inflow may impact on the history and nature of disease presentation. In the
region of the subclavian and axillary arteries the collateral vessels passing around the shoulder joint may provide pathways for
flow if the main vessels are stenotic or occluded. During periods of increased metabolic demand the collateral flow is not
sufficient and the vertebral arteries may have diminished flow. This may result in diminished flow to the brain with neurological
sequelae such as syncope.

Vascular disease of the upper limb:


Axillary/ brachial embolus: 50% of upper limb emboli will lodge in the brachial artery. 30% of upper limb emboli will lodge in
the axillary artery. Sudden onset of symptoms; pain, pallor, paresis, pulselessness, paraesthesia. Sources are left atrium with
cardiac arrhythmia (mainly AF), mural thrombus. Cardiac arrhythmias may cause result in impaired consciousness in addition to
the embolus
415
Arterial occlusions: Those resulting from atheroma are the most common, trauma may result in vascular changes and long term
occlusion but this is rare. Features may include claudication, ulceration and gangrene. Proximally sited lesions may result in
subclavian steal syndrome. The progressive nature of the disease allows development of collaterals, acute ischaemia may occur as
a result of acute thrombosis

Raynaud's disease: Idiopathic condition affecting young females. Usually affects hands > feet. Digits become: white -->blue --
>red. Treatment is with calcium antagonists

Upper limb venous thrombosis: Gradual onset of upper limb swelling and discomfort. Sensation and motor function are normal.
Condition may complicate pre-existing malignancy (especially breast cancer) or arise as a result of repetitive use of the limb in a
task such as painting a ceiling. The condition is diagnosed with duplex ultrasound and treatment is with anticoagulation

Cervical rib: 0.2-0.4% incidence. Consist of an anomalous fibrous band that often originates from C7 and may arc towards, but
rarely reaches the sternum. Congenital cases may present around the third decade, some cases are reported to occur following
trauma. Bilateral in up to 70%. Compression of the subclavian artery may produce absent radial pulse on clinical examination and
in particular may result in a positive Adsons test (lateral flexion of the neck away from symptomatic side and traction of the
symptomatic arm- leads to obliteration of radial pulse). Treatment is most commonly undertaken when there is evidence of
neurovascular compromise. A transaxillary approach is the traditional operative method for excision

Theme: Management of occlusive vascular disease

A. Aorto-bifemoral bypass graft


B. Femoro-femoral cross over graft
C. Femoro-popliteal bypass graft
D. Femoro-distal bypass graft
E. Axillo-bifemoral bypass graft
F. Bilateral above knee amputation

Please select the most appropriate arterial bypass method for the scenario described. Each option may be used once, more than
once or not at all.

7. An 83 year old lady with a significant cardiac history is admitted with rest pain and bilateral leg ulcers. Imaging
demonstrates bilateral occlusion of both common iliac arteries that are unsuitable for stenting.

Axillo-bifemoral bypass graft.. In patients with major cardiac co-morbidities the safest option is to choose an axillo-bifemoral
bypass graft. The long term patency rates are less good than with aorto-bifemoral bypass grafts, however, the operation is less
major. Theme from January 2012 Exam

8. A 54 year old man presents to the vascular clinic with severe rest pain and an ulcer on his right foot that is not healing.
On examination he has bilateral absent femoral pulses. Imaging demonstrates a bilateral occlusion of the common iliac
arteries that is not suitable for stenting.

Aorto-bifemoral bypass graft. In a young patient consideration should be given to aorto-bifemoral bypass grafts as these have
the best long term functional outcome compared with an axillobifemoral bypass graft.

9. A 78 year old man presents with left sided rest pain in his leg and a non healing arterial leg ulcer on the same leg.
Imaging shows normal right leg vessels, on the left side there is a long occlusion of the external iliac artery that is
unsuitable for stenting. He has a significant cardiac history.

Femoro-femoral cross over graft. Femoro-femoral cross over grafts are an option for treatment of iliac occlusions in patients
with significant co-morbidities and healthy contralateral vessels. In reality the idealised situation presented here seldom applies
and the opposite vessels usually have some disease and one must be careful not to damage the "healthy" side.

Peripheral vascular disease: Indications for surgery to revascularise the lower limb: Intermittent claudication. Critical
ischaemia. Ulceration. Gangrene

Intermittent claudication that is not disabling may provide a relative indication, whilst the other complaints are often absolute
indications depending upon the frailty of the patient.

Assessment: Clinical examination. Ankle brachial pressure index measurement. Duplex arterial ultrasound. Angiography
(standard, CT or MRI): usually performed only if intervention being considered.
416
Angioplasty: In order for angioplasty to be undertaken successfully the artery has to be accessible. The lesion relatively short and
reasonable distal vessel runoff. Longer lesions may be amenable to sub-intimal angioplasty.

Surgery: Surgery will be undertaken where attempts at angioplasty have either failed or are unsuitable. Bypass essentially
involves bypassing the affected arterial segment by utilising a graft to run from above the disease to below the disease. As with
angioplasty good runoff improves the outcome.

Some key concepts with bypass surgery: Superficial femoral artery occlusion to the above knee: Angioplasty may be
attempted but otherwise these patients will require a femoro-popliteal bypass graft. Patency rates for Polytetrafluoroethylene
(PTFE) and vein are similar, so PTFE preferred unless co-existing infection makes use of prosthetic material undesirable.

Procedure: Artery dissected out, IV heparin 3,000 units given and then the vessels are cross clamped. Longitudinal arteriotomy.
Graft cut to size and tunneled to arteriotomy sites. Anastomosis to femoral artery usually with 5/0 'double ended' Prolene suture .
Distal anastomosis usually using 6/0 'double ended' Prolene

Distal disease: Femoro-distal bypass surgery takes longer to perform, is more technically challenging and has higher failure rates.
In elderly diabetic patients with poor runoff a primary amputation may well be a safer and more effective option. There is no point
in embarking on this type of surgery in patients who are wheelchair bound. In femorodistal bypasses vein gives superior outcomes
to PTFE.

Rules: Vein mapping 1st to see whether there is suitable vein (the preferred conduit). Sub intimal hyperplasia occurs early when
PTFE is used for the distal anastomosis and will lead to early graft occlusion and failure. Essential operative procedure as for
above knee fem-pop. If there is insufficient vein for the entire conduit then vein can be attached to the end of the PTFE graft and
then used for the distal anastomosis. This type of 'vein boot' is technically referred to as a Miller Cuff and is associated with better
patency rates than PTFE alone. Remember the more distal the arterial anastomosis the lower the success rate.
Theme: Management of occlusive vascular disease

A. Aorto-bifemoral bypass graft


B. Femoro-femoral cross over graft
C. Femoro-popliteal bypass graft
D. Femoro-distal bypass graft
E. Axillo-bifemoral bypass graft
F. Bilateral above knee amputation

Please select the most appropriate arterial bypass method for the scenario described. Each option may be used once, more than
once or not at all.

10. An 83 year old lady with a significant cardiac history is admitted with rest pain and bilateral leg ulcers. Imaging
demonstrates bilateral occlusion of both common iliac arteries that are unsuitable for stenting.

Axillo-bifemoral bypass graft.. In patients with major cardiac co-morbidities the safest option is to choose an axillo-bifemoral
bypass graft. The long term patency rates are less good than with aorto-bifemoral bypass grafts, however, the operation is less
major. Theme from January 2012 Exam

11. A 54 year old man presents to the vascular clinic with severe rest pain and an ulcer on his right foot that is not healing.
On examination he has bilateral absent femoral pulses. Imaging demonstrates a bilateral occlusion of the common iliac
arteries that is not suitable for stenting.

Aorto-bifemoral bypass graft. In a young patient consideration should be given to aorto-bifemoral bypass grafts as these have the
best long term functional outcome compared with an axillobifemoral bypass graft.

12. A 78 year old man presents with left sided rest pain in his leg and a non healing arterial leg ulcer on the same leg.
Imaging shows normal right leg vessels, on the left side there is a long occlusion of the external iliac artery that is
unsuitable for stenting. He has a significant cardiac history.

Femoro-femoral cross over graft. Femoro-femoral cross over grafts are an option for treatment of iliac occlusions in patients
with significant co-morbidities and healthy contralateral vessels. In reality the idealised situation presented here seldom applies
and the opposite vessels usually have some disease and one must be careful not to damage the "healthy" side.

Theme: Ankle brachial pressure index

417
A. >1.2
B. 1.0
C. 0.8
D. 0.5
E. 0.3

Please select the ankle brachial pressure index that is most likely to be present for the scenario given. Each value may be used
once, more than once or not at all.

13. A 73 year old lifelong heavy smoker presents to the vascular clinic with symptoms of foot ulceration and rest pain. On
examination her foot has areas of gangrene and pulses are impalpable.

0.3. This is critical limb ischaemia. Values of 0.3 are typical in this setting and urgent further imaging is needed. Debridement
of necrosis prior to improving arterial inflow carries a high risk of limb loss.

14. A 63 year old man presents with a claudication distance of 15 yards. He is a lifelong heavy smoker. On examination his
foot is hyperaemic and there is a small ulcer at the tip of his great toe.

The correct answer is 0.5. Hyperaemia may occur in association with severe vascular disease and is referred to surgically as a
"sunset foot". ABPI is usually higher than 0.3, but seldom greater than 0.5. Especially when associated with hyperaemic
changes and ulceration. Urgent further imaging and risk factor modification is needed.

15. A 77 year old morbidly obese man with type 2 diabetes presents with leg pain at rest. His symptoms are worst at night
and sometimes improve during the day. He has no areas of ulceration.

The correct answer is >1.2. Type 2 diabetes may have vessel calcification. This will result in abnormally high ABPI readings.
Pain of this nature in diabetics is usually neuropathic and if a duplex scan is normal then treatment with an agent such as
carbamazepine is sometimes helpful. Theme from September 2011 and September 2012 exam

Ankle-Brachial pressure index: Measurement of ankle- brachial pressure index (ABPI) is a commonly performed vascular
investigation. Calculated by dividing lower limb pressure by the highest upper limb pressure.

Results of ABPI: 1.2 or greater: Usually due to vessel calcification. 1.0- 1.2: Normal. 0.8-1.0: Minor stenotic lesion. Initiate risk
factor management. 0.50-0.8: Moderate stenotic lesion. Consider duplex. Risk factor management. If mixed ulcers present then
avoid tight compression bandages. 0.5- 0.3: Likely significant stenosis. Duplex scanning to delineate lesions needed. Compression
bandaging contra indicated. Less than 0.3: Indicative of critical ischaemia. Urgent detailed imaging required

18. A 67 year old patient is due to undergo a femoro-popliteal bypass graft. Which heparin regime should the surgeon ask
for prior to cross clamping the femoral artery?
A. Single therapeutic dose of low molecular weight heparin on the ward prior to coming to theatre
B. Single therapeutic dose of low molecular weight heparin the night before surgery
C. Dose of 10,000 units of unfractionated heparin prior to induction of anaesthesia
D. Dose of 3,000 units of unfractionated heparin, 3 minutes prior to cross clamping
E. Dose of 30,000 units of unfractionated heparin, 3 minutes prior to cross clamping
Answer: D
As a rule most vascular surgeons will administer approximately 3,000 units of systemic heparin 3-5 minutes prior to cross
clamping to help prevent further intra arterial thromboses. A dose of 30,000 units is given prior to going on cardiopulmonary
bypass. Heparin given at induction will cause bleeding during routine dissection.

Heparin: Causes the formation of complexes between antithrombin and activated thrombin/factors 7,9,10,11 & 12

Advantages of low molecular weight heparin: Better bioavailability, Lower risk of bleeding, Longer half life, Little effect on
APTT at prophylactic dosages, Less risk of HIT.

Complications: Bleeding, Osteoporosis, Heparin induced thrombocytopenia (HIT): occurs 5-14 days after 1st exposure.
Anaphylaxis.

418
In surgical patients that may need a rapid return to theatre administration of unfractionated heparin is preferred as low molecular
weight heparins have a longer duration of action and are harder to reverse.

19. An 18 year old lady presents with extensive varicose veins of her left leg. There is associated port wine staining. What is
the most likely diagnosis?
A. Type 1 diabetes
B. Osler syndrome
C. Gardner's syndrome
D. Proteus syndrome
E. Klippel-Trenaunay-Weber syndrome
Answer: E
A less common cause of venous insufficiency is Klippel-Trenaunay-Weber (KTW) syndrome, which involves port-wine stains,
varicose veins, and bony or soft-tissue hypertrophy.
Lower leg ulcers: Venous leg ulcers: Most due to venous hypertension, secondary to chronic venous insufficiency (other causes
include calf pump dysfunction or neuromuscular disorders). Ulcers form due to capillary fibrin cuff or leucocyte sequestration.
Features of venous insufficiency include oedema, brown pigmentation, lipodermatosclerosis, eczema. Location above the ankle,
painless. Deep venous insufficiency is related to previous DVT and superficial venous insufficiency is associated with varicose
veins. Doppler ultrasound looks for presence of reflux and duplex ultrasound looks at the anatomy/ flow of the vein. Management:
4 layer compression banding after exclusion of arterial disease or surgery. If fail to heal after 12 weeks or >10cm2 skin grafting
may be needed

Marjolin's ulcer: Squamous cell carcinoma. Occurring at sites of chronic inflammation e.g; burns, osteomyelitis after 10-20
years. Mainly occur on the lower limb

Arterial ulcers: Occur on the toes and heel. Painful. There may be areas of gangrene. Cold with no palpable pulses. Low ABPI
measurements

Neuropathic ulcers: Commonly over plantar surface of metatarsal head and plantar surface of hallux. The plantar neuropathic
ulcer is the condition that most commonly leads to amputation in diabetic patients. Due to pressure. Management includes
cushioned shoes to reduce callous formation

Pyoderma gangrenosum: Associated with inflammatory bowel disease/RA. Can occur at stoma sites. Erythematous nodules or
pustules which ulcerate

Theme: Management of peripheral arterial disease

A. Primary amputation
B. Angioplasty
C. Arterial bypass surgery using vein
D. Arterial bypass surgery using PTFE
E. Conservative management with medical therapy and exercise
F. Watch and wait
G. Duplex scanning

Please select the most appropriate management for the scenario given. Each option may be used once, more than once or not at all.

20. A 63 year old man is admitted with rest pain and foot ulceration. An angiogram shows a 3 cm area of occlusion of the
distal superficial femoral artery with 3 vessel run off. His ankle - brachial pressure index is 0.4.

The correct answer is Angioplasty. Short segment disease and good run off with tissue loss is a compelling indication for
angioplasty. He should receive aspirin and a statin if not already taking them.

21. A 72 year old man present in the vascular clinic with calf pain present on walking 100 yards. He is an ex-smoker and
lives alone. On examination he has reasonable leg pulses. His right dorsalis pedis pulse gives a monophasic doppler
signal with an ankle brachial pressure index measurement of 0.7. All other pressures are acceptable. There is no
evidence of ulceration or gangrene.

Conservative management with medical therapy and exercise. Structured exercise programmes combined with medical therapy
will improve many patients. Should his symptoms worsen or fail to improve then imaging with duplex scanning would be
required.

22. An 83 year old lady is admitted from a nursing home with infected lower leg ulcers. She underwent an attempted long
superficial femoral artery sub initimal angioplasty 2 weeks previously. This demonstrated poor runoff below the knee.
419
Primary amputation. Poor runoff and sepsis would equate to poor outcome with attempted bypass surgery.

23. A 32 year old woman attends clinic for assessment of varicose veins. She has suffered for varicose veins for many years
and can trace their development back to when she suffered a complex tibial fracture. On examination she has marked
truncal varicosities with a long tortuous long saphenous vein. Which of the following would be the most appropriate next
step?
A. Arrange a venogram
B. Arrange a venous duplex scan
C. List her for a trendelenberg procedure
D. List her for injection foam sclerotherapy
E. List her for multiple avulsion phlebectomies
Answer: B
This lady is likely to have deep venous incompetence as she will have been immobilised for her tibial fracture and may well
have had a DVT. A duplex scan is mandatory prior to any form of surgical intervention. A venogram would provide similar
information but is more invasive.

Chronic venous insufficiency and Varicose veins: Wide spectrum of disease ranging from minor cosmetic problem through to
ulceration and disability. It is commoner in women than men and is worse during pregnancy. Defined as saccular dilatation of
veins (WHO). The veins of the lower limb consist of an interconnected network of superficial and deep venous systems. Varices
occur because of localised weakness in the vein wall resulting in dilatation and reflux of blood due to non union of valve cusps.

Histology: fibrous scar tissue dividing smooth muscle within media in the vessel wall. Tissue damage in chronic venous
insufficiency occurs because of perivascular cytokine leakage resulting in localised tissue damage coupled with impaired
lymphatic flow.

Affected veins: normally long and short saphenous veins

Diagnosis: Typical symptoms of varicose veins include: Cosmetic appearance. Aching. Ankle swelling that worsens as the day
progresses. Episodic thrombophlebitis. Bleeding. Itching

Symptoms of chronic venous insufficiency include: Dependant leg pain. Prominent leg swelling. Oedema extending beyond the
ankle. Venous stasis ulcers

The typical venous stasis ulcer is: Located above the medial malleolus. Indolent appearance with basal granulation tissue.
Variable degree of scarring. Non ischaemic edges. Haemosiderin deposition in the gaiter area (and also lipodermatosclerosis).

Differential diagnosis: Lower limb arterial disease. Marjolins ulcer. Claudication. Spinal stenosis. Swelling due to medical
causes e.g. CCF.

Exclusion of these differentials is by means of physical examination and ankle brachial pressure index measurement.

Examination: Assess for dilated short saphenous vein (popliteal fossa) and palpate for saphena varix medial to the femoral artery.
Brodie-Trendelenburg test: to assess level of incompetence. Perthes' walking test: assess if deep venous system competent

Investigation: Doppler exam: if incompetent a biphasic signal due to retrograde flow is detected. Duplex scanning: to ensure
patent deep venous system (do if DVT or trauma)

All patients should have a Doppler assessment to assess for venous reflux and should be classified as having uncomplicated
varicose veins or varicose veins with associated chronic venous insufficiency. In the history establishing a previous thrombotic
event (DVT/ lower limb fracture) is important and patients with such a history and all who have evidence of chronic venous
insufficiency should have a duplex scan performed.

Owing to litigation patients with saphenopopliteal incompetence should have a duplex scan performed and the site marked by scan
on the day of surgery.

Treatment: Indications for surgery: Cosmetic: majority. Lipodermatosclerosis causing venous ulceration Recurrent superficial
thrombophlebitis. Bleeding from ruptured varix

Condition Therapy

420
Minor varicose veins - no Reassure/ cosmetic therapy
complications
Symptomatic uncomplicated In those without deep venous insufficiency options include foam sclerotherapy, saphenofemoral /
varicose veins popliteal disconnection, stripping and avulsions, compression stockings
Varicose veins with skin Therapy as above (if compression minimum is formal class I stockings)
changes
Chronic venous insufficiency Class 2-3 compression stockings (ensure no arterial disease).
or ulcers

Application of formal compression stockings (usually class II/III). In patients who have suffered ulceration, compression
stockings should be worn long term. Where ulceration is present and established saphenofemoral reflux exists this should be
addressed surgically for durable relief of symptoms, either at the outset or following ulcer healing. Injection sclerotherapy (5%
Ethanolamine oleate), foam is increasingly popular, though transient blindness has been reported. Endo venous laser therapy is
another minimally invasive option. Sapheno-femoral or sapheno-popliteal ligation, in the case of the LSV stripping and multiple
phlebectomies

Trendelenburg procedure (sapheno-femoral junction ligation): Head tilt 15 degrees and legs abducted. Oblique incision 1cm
medial from artery. Tributaries ligated (Superficial circumflex iliac vein, Superficial inferior epigastric vein, Superficial and deep
external pudendal vein). SF junction double ligated. Saphenous vein stripped to level of knee/upper calf. NB increased risk of
saphenous neuralgia if stripped more distally

Theme: Management of abdominal aortic aneurysms


A. Immediate laparotomy
B. Immediate CT
C. Elective AAA repair
D. USS in 6 months
E. CT scan during next 4 weeks
F. Endovascular aortic aneurysm repair
G. Discharge
H. Palliate
I. None of the above

Please select the most appropriate management for the scenario given. Each option may be used once, more than once or not at all.

24. A 66 year old man is referred via the aneurysm screening programme with an abdominal aortic aneurysm measuring 4.4
cm. Apart from well controlled type 2 DM he is otherwise well

USS in 6 months. At this point continue with ultrasound surveillance

25. A 72 year old man has a CT scan for abdominal discomfort and the surgeon suspects AAA. This shows a 6.6cm
aneurysm with a 3.5cm neck and it continues to involve the right common iliac. The left iliac is occluded. He is
hypertensive and has Type 2 DM which is well controlled

Elective AAA repair. Assuming he is fit enough. This would be a typical 'open ' case as the marked iliac disease would make
EVAR difficult

26. An 89 year old man presents with hypotension and collapse and is found by the staff in the toilet of his care home. He is
moribund and unable to give a clear history. He had suffered a cardiac arrest in the ambulance but has since been
resuscitated and now has a Bp of 95 systolic. He has an obviously palpable AAA.

The correct answer is Palliate. He will not survive aortic surgery and whilst some may disagree, I would argue that taking this
case to theatre would be futile

421
Abdominal aorta aneurysm: Abdominal aortic aneurysms are a common problem in vascular surgery. They may occur as either
true or false aneurysm. With the former all 3 layers of the arterial wall are involved, in the latter only a single layer of fibrous
tissue forms the aneurysm wall. True abdominal aortic aneurysms have an approximate incidence of 0.06 per 1000 people. They
are commonest in elderly men and for this reason the UK is now introducing the aneurysm screening program with the aim of
performing an abdominal aortic ultrasound measurement in all men aged 65 years.

Causes: Several different groups of patients suffer from aneurysmal disease. The commonest group is those who suffer from
standard arterial disease, i.e. Those who are hypertensive, have diabetes and have been or are smokers. Other patients such as
those suffering from connective tissue diseases such as Marfan's may also develop aneurysms. In patients with abdominal aortic
aneurysms the extracellular matrix becomes disrupted with a change in the balance of collagen and elastic fibres.

Management: Most abdominal aortic aneurysms are an incidental finding. Symptoms most often relate to rupture or impending
rupture. 20% rupture anteriorly into the peritoneal cavity. Very poor prognosis. 80% rupture posteriorly into the retroperitoneal
space. The risk of rupture is related to aneurysm size, only 2% of aneurysms measuring less than 4cm in diameter will rupture
over a 5 year period. This contrasts with 75% of aneurysms measuring over 7cm in diameter. This is well explained by La Places'
law which relates size to transmural pressure. For this reason most vascular surgeons will subject patients with an aneurysm size
of 5cm or greater to CT scanning of the chest, abdomen and pelvis with the aim of delineating anatomy and planning treatment.
Depending upon co-morbidities, surgery is generally offered once the aneurysm is between 5.5cm and 6cm.

Indications for surgery: Symptomatic aneurysms (80% annual mortality if untreated). Increasing size above 5.5cm if
asymptomatic. Rupture (100% mortality without surgery)

Surgical procedures: Abdominal aortic aneurysm repair: Procedure: GA. Invasive monitoring (A-line, CVP, catheter).
Incision: Midline or transverse. Bowel and distal duodenum mobilised to access aorta. Aneurysm neck and base dissected out and
prepared for cross clamp. Systemic heparinisation. Cross clamp (distal first). Longitudinal aortotomy. Atherectomy. Deal with
back bleeding from lumbar vessels and inferior mesenteric artery
Insert graft either tube or bifurcated depending upon anatomy. Suture using Prolene (3/0 for proximal , distal anastomosis suture
varies according to site). Clamps off: End tidal CO2 will rise owing to effects of reperfusion, at this point major risk of myocardial
events. Haemostasis. Closure of aneurysm sac to minimise risk of aorto-enteric fistula. Closure: Loop 1 PDS or Prolene to
abdominal wall. Skin- surgeons preference

Post operatively: ITU (Almost all). Greatest risk of complications following emergency repair. Complications: Embolic- gut and
foot infarcts. Cardiac - owing to premorbid states, reperfusion injury and effects of cross clamp. Wound problems. Later risks
related to graft- infection and aorto-enteric fistula

Special groups: Supra renal AAA


These patients will require a supra renal clamp and this carries a far higher risk of complications and risk of renal failure.

Ruptured AAA: Preoperatively the management depends upon haemodynamic instability. In patients with symptoms of rupture
(typical pain, haemodynamic compromise and risk factors) then ideally prompt laparotomy. In those with vague symptoms and
haemodynamic stability the ideal test is CT scan to determine whether rupture has occurred or not. Most common rupture site is
retroperitoneal 80%. These patients will tend to develop retroperitoneal haematoma. This can be disrupted if Bp is allowed to rise
too high so aim for Bp 100mmHg.
Operative details are similar to elective repair although surgery should be swift, blind rushing often makes the situation worse.
Plunging vascular clamps blindly into a pool of blood at the aneurysm neck carries the risk of injury the vena cava that these
patients do not withstand. Occasionally a supracoeliac clamp is needed to effect temporary control, although leaving this applied
for more than 20 minutes tends to carry a dismal outcome.

EVAR: Increasingly patients are now being offered endovascular aortic aneurysm repair. This is undertaken by surgeons and
radiologists working jointly. The morphology of the aneurysm is important and not all are suitable. Here is a typical list of those
features favoring a suitable aneurysm: Long neck. Straight iliac vessels. Healthy groin vessels
Clearly few AAA patients possess the above and compromise has to be made. The use of fenestrated grafts can allow supra renal
AAA to be treated.

Procedure: GA. Radiology or theatre. Bilateral groin incisions. Common femoral artery dissected out. Heparinisation
422
Arteriotomy and insertion of guide wire. Dilation of arteriotomy. Insertion of EVAR Device
Once in satisfactory position it is released. Arteriotomy closed once check angiogram shows good position and no endoleak

Complications: Endoleaks depending upon site are either Type I or 2. These may necessitate re-intervention and all EVAR
patients require follow up . Details are not needed for MRCS.

27. During short saphenous vein surgery for varicose veins which of the following nerves is particularly at risk?
A. Sural nerve
B. Popliteal nerve
C. Tibial nerve
D. Femoral nerve
E. Saphenous nerve
Answer: A

Saphenous vein: Long saphenous vein

This vein may be harvested for triple or quadruple bypass surgery. Originates at the 1st digit where the dorsal vein merges with
the dorsal venous arch of the foot. Passes anterior to the medial malleolus and runs up the medial side of the leg. At the knee, it
runs over the posterior border of the medial epicondyle of the femur bone. Then passes laterally to lie on the anterior surface of
the thigh before entering an opening in the fascia lata called the saphenous opening. It joins with the femoral vein in the region of
the femoral triangle at the saphenofemoral junction

Tributaries: Medial marginal. Superficial epigastric. Superficial iliac circumflex. Superficial external pudendal veins

Short saphenous vein : Originates at the 5th digit where the dorsal vein merges with the dorsal venous arch of the foot, which
attaches to the great saphenous vein. It passes around the lateral aspect of the foot (inferior and posterior to the lateral malleolus)
and runs along the posterior aspect of the leg (with the sural nerve). Passes between the heads of the gastrocnemius muscle, and
drains into the popliteal vein, approximately at or above the level of the knee joint.

29. A 21 year old badminton player attends A&E with a painful, swollen right arm. He is right handed. Clinically he has
dusky fingers and his upper limb pulses are present. An axillary vein thrombosis is confirmed. What is the best acute
treatment to achieve vein patency?
A. Intravenous heparin
B. Warfarin
C. Catheter directed tPA
D. Low molecular weight heparin
E. Aspirin
Answer: C
Heparin and warfarin prevent propagation of the clot.

Axillary vein thrombosis: 1-2% of all deep venous thrombosis. Primary cause is associated with trauma, thoracic outlet
obstruction or repeated effort in a dominant arm (young active individuals). Secondary causes include central line insertion,
malignancy, pacemakers

Clinical features: Pain and swelling (non pitting). Numbness. Discolouration: mottling, dusky. Pulses present. Congested veins

Investigations: FBC: viscosity, platelet function. Clotting. Liver function tests. D-dimer. Duplex scan: investigation of choice. CT
scan: thoracic outlet obstruction

Treatment: Local catheter directed TPA. Heparin. Warfarin


28. A 23 year old man presents with a brachial artery embolus. A cervical rib is suspected as being the underlying cause.
From which of the following vertebral levels do they most often arise?
A. C7
B. C5
C. C4
D. C3
E. C2
Answer: A
They usually arise from C7.

423
Cervical ribs: 0.2-0.4% incidence. Consist of an anomalous fibrous band that often originates from C7 and may arc towards, but
rarely reaches the sternum. Congenital cases may present around the third decade, some cases are reported to occur following
trauma. Bilateral in up to 70%. Compression of the subclavian artery may produce absent radial pulse on clinical examination and
in particular may result in a positive Adsons test (lateral flexion of the neck away from symptomatic side and traction of the
symptomatic arm- leads to obliteration of radial pulse). Treatment is most commonly undertaken when there is evidence of
neurovascular compromise. A transaxillary approach is the traditional operative method for excision. 3D reconstruction of a left-
sided cervical rib

29. A 73 year old man with rest pain and ulceration of the foot undergoes a femoro-distal bypass graft with a PTFE graft.
At the end of the procedure there are good distal foot pulses and a warm pink foot. Over the ensuing 6 days the foot
becomes progressively cooler and the pulses diminish. What is the most likely underlying explanation for this process?
A. Embolus
B. Neo-intimal flap
C. Neo-intimal hyperplasia
D. Polyarteritis
E. Steal syndrome
Answer: C
Neo-intimal hyperplasia in distal arterial anastamoses may be reduced by use of a Miller Cuff when PTFE is the bypass conduit.

PTFE may induce neo-intimal hyperplasia with subsequent occlusion of the distal anastomosis. In more proximal arterial bypass
surgery the process of neo-intimal hyperplasia is not sufficient to cause anastomotic occlusion. However, distal bypasses are at
greater risk and if vein cannot be used as a conduit then the distal end of the PTFE should anastomosed to a vein cuff to minimise
the risk of neo-intimal hyperplasia.

Anastomoses: A wide variety of anastomoses are constructed in surgical practice. Essentially the term refers to the restoration of
luminal continuity. As such they are a feature of both abdominal and vascular surgery.

Visceral anastomoses: For an anastomosis to heal three criteria need to be fulfilled: Adequate blood supply. Mucosal apposition.
Minimal tension

When these are compromise the anastomosis may dehisce (leak). Even in the best surgical hands some anastomoses are more
prone to dehiscence than others. Oesophageal and rectal anastomoses are more prone to leakage and reported leak rates following
oesophageal and rectal surgery can be as high as 20%. This figure includes radiological leaks and those with a clinically
significant leak will be of a lower order of magnitude. As a rule small bowel anastomoses heal most reliably.

The decision as to how best to achieve mucosal apposition is one for each surgeon. Some will prefer the use of stapling devices as
they are quicker to use, others will prefer to perform a sutured anastomosis. The attention to surgical technique is more important
than the method chosen and a poorly constructed stapled anastomosis in thickened tissue is far more prone to leakage than a hand
sewn anastomosis in the same circumstances.

If an anastomosis looks unsafe then it may be best not to construct one at all. In colonic surgery this is relatively clear cut and
most surgeons would bring out an end colostomy. In situations such as oesophageal surgery this is far more problematic and
colonic interposition may be required in this situation.

Vascular anastomoses: Most arterial surgery involving bypasses or aneurysm repairs will require construction of an arterial
anastomosis. Technique is important and for small diameter distal arterial surgery the intimal hyperplasia resulting from a badly
constructed anastomosis may render the whole operation futile before the patient leaves hospital.

Some key points about vascular anastomoses: Always use non absorbable monofilament suture (e.g. Polypropylene). Round
bodied needle. Correct size for anastamosis ( i.e. 6/0 prolene for bottom end of a femoro-distal bypass). Suture should be
continuous and from inside to outside of artery to avoid raising an intimal flap.

Theme: Lymphoedema Management

A. Homans operation
B. Charles operation
C. Frusemide at high doses
D. Frusemide at low doses
E. Multilayer compression bandaging
F. Lymphovenous anastomosis

424
Please select the most appropriate management for the lymphoedema scenario given. Each option may be used once, more than
once or not at all.

30. 52 year old lady develops lower leg swelling following redo varicose vein surgery. There is evidence of swelling of the left
leg up to the knee. The overlying skin appears healthy.

The correct answer is Multilayer compression bandaging. Unfortunately lymphoedema may complicate redo varicose vein
surgery (in 0.5% of cases). As the presentation is mild, she should be managed using compression hosiery. Diuretics do not help
in cases of true lymphoedema and a dramatic response suggests an alternative underlying cause.

31. A 57 year old lady has suffered from lymphoedema for many years. The left leg is swollen to the mid thigh. Severe limb
deformity has developed as a result of process and in spite of compression hoisery. Lymphoscintography shows no
patent lymphatics in the proximal leg. The overlying skin is healthy.

The correct answer is Homans operation. Surgery is indicated in less than 10% of cases. However, severe deformity is one of
the indications for surgery. Lymphovenous anastomosis is indicated where the proximal lymphatics are not patent. When the
overlying skin is healthy (and limb deformity a problem), a Homans procedure is a reasonable first line operative option.

32. A 38 year old lady is troubled by lymphoedema that occurred following a block dissection of the groin for malignant
melanoma many years previously. Despite therapy with compression bandages she has persistent lower leg swelling
impairing her activities of daily living. She has no evidence of recurrent malignancy. Lymphoscintography demonstrates
occlusion of the groin lymphatics. However, the distal lymphatic system appears healthy.

Lymphovenous anastomosis. In young patients with proximal disease and healthy distal lymphatics a lymphovenous
anastomosis may be considered. Such cases are rare.

Lymphoedema: Due to impaired lymphatic drainage in the presence of normal capillary function.. Lymphoedema causes the
accumulation of protein rich fluid, subdermal fibrosis and dermal thickening. Characteristically fluid is confined to the epifascial
space (skin and subcutaneous tissues); muscle compartments are free of oedema. It involves the foot, unlike other forms of
oedema. There may be a 'buffalo hump' on the dorsum of the foot and the skin cannot be pinched due to subcutaneous fibrosis.

Causes of lymphoedema

Primary: Congenital < 1 year: sporadic, Milroy's disease. Onset 1-35 years: sporadic, Meige's disease. > 35 years: Tarda

Secondary: Bacterial/fungal/parasitic infection (filariasis). Lymphatic malignancy. Radiotherapy to lymph nodes. Surgical
resection lymph nodes. DVT. Thrombophlebitis

Indications for surgery: Marked disability or deformity from limb swelling. Lymphoedema caused by proximal lymphatic
obstruction with patent distal lymphatics suitable for a lymphatic drainage procedure. Lymphocutaneous fistulae and
megalymphatics

Procedures
Homans operation Reduction procedure with preservation of overlying skin (which must be in good condition). Skin flaps
are raised and the underlying tissue excised. Limb circumference typically reduced by a third.
Charles operation All skin an subcutaneous tissue around the calf is excised down to the deep fascia. Split skin grafts are
placed over the site. May be performed if overlying skin is not in good condition. Larger reduction in size
than with Homans procedure.
Lymphovenous Identifiable lymphatics are anastomosed to sub dermal venules. Usually indicated in 2% of patients with
anastamosis proximal lymphatic obstruction and normal distal lymphatics.

19. Which of the following is not a typical feature of a chronic venous leg ulcer?
A. Heaped raised borders if the ulcer has been present more than 5 years
B. Evidence of surrounding lipodermatosclerosis
C. Irregular shape to the ulcer
D. 20% of cases will have a previous history of deep vein thrombosis
E.Haemosiderin deposits in surrounding skin

425
Answer: A
The borders of the ulcer are often well defined even though they may be irregular. Heaped or raised borders should raise
suspicion of a marjolins ulcer.

20. Which of the following is not a typical feature of an arterial leg ulcer?
A. Well demarcated edges
B. A grey - white base to the ulcer
C. Men are affected more than women
D. Painful
E. Ankle swelling
Answer: E
Ankle swelling is often absent. If present it may be due to mixed arteriovenous disease. With mixed disease the arterial
component is treated first.

Theme: Lower limb ulceration


a. Mixed ulcer
b. Chronic obliterative arterial disease
c. Superficial venous insufficiency
d. Deep venous insufficiency
e. Neuropathic ulcer
f. Basal cell carcinoma
g. Squamous cell carcinoma

Please select the most likely cause of ulceration for the scenario given. Each option may be used once, more than once or not at
all.

21. A 65 year old diabetic female presents with a painless ulcer at the medial malleolus, it has been present for the past 16
years. On examination she has evidence of truncal varicosities and a brownish discolouration of the skin overlying the
affected area.

Superficial venous insufficiency.. Venous ulcers are usually associated with features of venous insufficiency. These include
haemosiderin deposition and varicose veins. Neuropathic ulcers will tend to present at sites of pressure, which is not typically at
the medial malleolus. Theme from September 2012 Exam

22. A 71 year old man presents with a painful lower calf ulcer, mild pitting oedema and an ABPI of 0.3.

The correct answer is Chronic obliterative arterial disease. Painful ulcers associated with a low ABPI are usually arterial in
nature. The question does not indicate that features of chronic venous insufficiency are present. Patients may have mild pitting
oedema as many vascular patients will also have ischaemic heart disease and elevated right heart pressures. The absence of
more compelling signs of venous insufficiency makes a mixed ulcer less likely.

23. A 79 year old retired teacher has had an ulcer for 15 years. It is at the medial malleolus and has associated
lipodermatosclerosis of the lower limb. The ulcer base is heaped up and irregular.

Squamous cell carcinoma. If after many years an ulcer becomes heaped up and irregular, with rolled edges then suspect a .
Squamous cell carcinoma.

24. A 66 year old man is admitted with severe angina. There is a lesion of the proximal left anterior descending coronary
artery. Which of the following would be the most suitable conduit for bypass?
A. Long saphenous vein
B. Short saphenous vein
C. Cephalic vein
D. Internal mammary artery
E. Thoraco-acromial artery
Answer: D

426
The internal mammary artery is an excellent conduit for coronary artery bypass. It has better long term patency rates than
venous grafts. The thoraco-acromial artery is seldom used.

Cardiopulmonary bypass: Indications for surgery: Left main stem stenosis or equivalent (proximal LAD and proximal
circumflex). Triple vessel disease. Diffuse disease unsuitable for PCI

The guidelines state that CABG is the preferred treatment in high-risk patients with severe ventricular dysfunction or diabetes
mellitus.

Technique: General anaesthesia. Central and arterial lines. Midline sternotomy or left sub mammary incision. Aortic root and
pericardium dissected. Heart inspected

Bypass grafting may be performed using a cardiopulmonary bypass circuit with cardiac arrest or using a number of novel 'off
pump' techniques.

Procedure cardiopulmonary bypass: Aortic root cannulated. Right atrial cannula. Circuit primed and patient fully heparinised
(30,000 Units unfractionated heparin) as the circuit is highly thrombogenic. Flow established through circuit. Aortic cross clamp
applied. Cardioplegia solution instilled into the aortic root below cross clamp. Heart now asystolic and ready for surgery.

Off pump techniques are evolving on a constant basis and details are beyond the scope of the MRCS.

Conduits for bypass: > Internal mammary artery is best. Use of both is associated with increased risk of sternal wound
dehiscence. However, many surgeons will use both especially for redo surgery. > Radial artery harvested from forearm. Ensure
ulnar collateral working first! > Reversed long saphenous vein grafts. Typically anastamosed using 7/0-8/0 prolene sutures
(distally) and 6/0 prolene for top end.

Once flow established. Anticoagulation reversed using protamine. Patient is taken off bypass. Inotropes given if needed. Sternum
closed using sternal closure device or stainless steel wire

Complications: Post perfusion syndrome: transient cognitive impairment. Non union of the sternum; due to loss of the internal
thoracic artery. Myocardial infarction. Late graft stenosis. Acute renal failure. Stroke. Gastrointestinal. Perioperative risk is
quantified using the Parsonnet and Euroscores and unit outcomes are audited using this data.

25. Concerning proximal aortic dissection (Debakey types 1 and 2/ Stanford type A) which statement is false?
A. The intimal tear is typically >50% of the aortic circumference.
B. It is usually treated using an endovascular approach.
C. They have a 50% mortality in the first 2 days.
D. Arch reconstructions may require deep hypothermic circulatory arrest.
E. Target systolic pressure of <110mmHg should be maintained.
Answer: B
Usually open surgery is required for these lesions as customised grafts are not usually available for this type of repair yet.

Aortic dissection: More common than rupture of the abdominal aorta. 33% of patients die within the first 24 hours, and 50% die
within 48 hours if no treatment received. Associated with hypertension. Features of aortic dissection: tear in the intimal layer,
followed by formation and propagation of a subintimal hematoma. Cystic medial necrosis (Marfan's). Most common site of
dissection: 90% occurring within 10 centimetres of the aortic valve

Stanford Classification
Type Location Treatment
A Ascending aorta/ aortic root Surgery- aortic root replacement
B Descending aorta Medical therapy with antihypertensives

DeBakey classification
Type Site affected
I Ascending aorta, aortic arch, descending aorta
II Ascending aorta only
III Descending aorta distal to left subclavian artery

Clinical features: Tearing, sudden onset chest pain (painless 10%). Hypertension or Hypotension. A blood pressure difference
greater than 20 mm Hg. Neurologic deficits (20%)

Investigations: CXR: widened mediastinum, abnormal aortic knob, ring sign, deviation trachea/oesophagus. CT (spiral). MRI.
Angiography (95% of patients diagnosed)
427
Management: Beta-blockers: aim HR 60-80 bpm and systolic BP 100-120 mm Hg. Urgent surgical intervention: type A
dissections. This will usually involve aortic root replacement.

26. A 67 year old male is diagnosed as having a 7cm infra renal abdominal aortic aneurysm. What is the likely risk of
rupture over the next 5 years?
A. <10%
B. 20%
C. 25%
D. 75%
E. 35%
Answer: D
Risks of abdominal aortic aneurysm rupture (over 5 years):5-5.9cm = 25%. 6-6.9cm = 35%. 7cm and over = 75%. Aneuryms
greater than 5cm in diameter on USS should be formally assessed using CT scanning with arterial phases to delineate anatomy
and facilitate surgical planning.

Theme: Venous disease

A. No further management needed


B. Injection sclerotherapy 0.5% Sodium tetradecyl sulphate
C. Injection sclerotherapy 5% phenol
D. Long saphenous vein ligation
E. Long saphenous vein stripped to the ankle
F. Long saphenous vein stripped to the knee
G. Doppler scan
H. Duplex scan

Please select the most appropriate management plan for the scenario given. Each option may be used once, more than once or not
at all.

27. A 42 year old teacher presents with an ulcer associated with varicose veins in the long saphenous vein territory. Apart
from a DVT 1 year ago, she has no other past medical history.

Duplex scan. This patient needs a duplex scan to assess the patency of her deep venous system before surgery can be
undertaken. Other indications for duplex scan include recurrent varicose veins or complications.

28. A 42 year old accountant presents with thrombophlebitis of a long standing varicosity of the inner thigh. Doppler and
clinical assessment demonstrate saphenofemoral junction incompetence.

The correct answer is Long saphenous vein stripped to the knee. Long standing varicose veins with complications such as
thrombophlebitis and demonstrated valvular incompetence should receive surgery. Ligation was previously popular but has
higher long term recurrence rates. There is increased risk of saphenous neuralgia if they are stripped to the ankle.

29. A 28 year old lady presents with increasing aching discomfort from a varicosity below the knee. On examination she has
a single large truncal varicosity in the area, doppler assessment shows competent sapheno-femoral and sapheno-
popliteal junctions.

Injection sclerotherapy 0.5% Sodium tetradecyl sulphate. This lady has a single varicosity. Sclerotherapy (probably with foam)
would be the ideal management. A simple avulsion under anaethesia is an alternative. These are best treated with injection
sclerotherapy. 5% phenol is normally used as a sclerosing agent in haemorrhoids!

30. A 24-year-old female is referred to the acute surgical team as she is noted to have an absent left radial pulse. Apart from
some dizziness and lethargy, the patient does not have any features suggestive of an acute ischaemic limb. Blood tests are
as follows: Na+ 136 mmol/lm K+ 4.1 mmol/lm Urea 2.3 mmol/lm Creatinine 77 µmol/lm, ESR 66 mm/hr. What is the
most likely diagnosis?
A. Turner's syndrome
B. Takayasu's arteritis
428
C. Kawasaki disease
D. Coarctation of the aorta
E. Breast carcinoma with local spread
Answer: B

Takayasu's arteritis is a large vessel vasculitis. It typically causes occlusion of the aorta and questions commonly refer to an
absent limb pulse. It is more common in females and Asian people. Associations: renal artery stenosis.Management: steroids

Theme: Leg swelling

A. Milroy's disease
B. Meige's disease
C. Lymphoedema tarda
D. Filariasis
E. Tuberculosis
F. Locally advanced bladder carcinoma
G. Malaria

Which is the most likely diagnosis for the scenario given? Each option may be used once, more than once or not at all.

31. The medical team refer a 72 year old lady with a bilateral swollen legs. Deep vein thrombosis has been excluded and
there is no response to diuretics. On further questioning, the patient reveals that she was born with the swelling in both
of her legs.

Milroy's disease. Milroy's disease is present from birth and is due to failure of the lymphatic vessels to develop. Note that
Meige's disease develops AFTER birth.
32. A 52 year old woman presents with rapid swelling of the left leg. The swelling is greater in the thigh compared to the
calf.

The correct answer is Locally advanced bladder carcinoma. Always consider a malignancy in an older adult with new
lymphoedema in a limb, especially if the swelling is greater proximally than distally. If malignancy is excluded consider the
diagnosis of lymphoedema tarda.

33. A 34 year old African teacher attends A&E with a swollen leg. She has been in England for 2 weeks. She lives in an area
prevalent with mosquitoes and where there is poor sanitation.

Filariasis. Filariasis is caused by the nematode Wucheria bancrofti, which is mainly spread by mosquito. The oedema can be
gross leading to elephantitis. Treatment is with diethylcarbamazine.

34. A 21 year old post man notices leg pain after 5 minutes walking during his round. It improves 3 minutes after stopping.
Clinically he is noted to have reduced hair of the lower limbs and his calf muscles appear atrophied. There is a weak
popliteal pulse, but it is still present when the knee is fully extended. What is the most likely diagnosis?
A. Occlusive arterial disease caused by atherosclerosis
B. Popliteal fossa entrapment
C. Cerebral vascular accident
D. Diabetes mellitus
E. Adductor canal compression syndrome
Answer: E
Adductor canal compression syndrome most commonly presents in young males and is an important differential diagnosis in
men presenting with symptoms of acute limb ischaemia on exertion. It is caused by compression of the femoral artery by the
musculotendinous band from adductor magnus muscle.
The treatment consists of the division of the abnormal band and restoration of the arterial circulation. Popliteal fossa entrapment
is the main differential diagnosis, however the pulse disappears when the knee is fully extended.

Adductor canal: Also called Hunter's or subsartorial canal. Immediately distal to the apex of the femoral triangle, lying in the
middle third of the thigh. Canal terminates at the adductor hiatus.

429
Borders Contents
Laterally Vastus medialis muscle Saphenous nerve
Posteriorly Adductor longus, adductor magnus Superficial femoral artery
Roof Sartorius Superficial femoral vein

In the image below the sartorius muscle is removed to expose the canal contents

Theme: Peripheral arterial disease

A. Femoro-above knee popliteal bypass graft using PTFE


B. Femoro-above knee popliteal bypass graft using long saphenous vein
C. Femoro-distal bypass graft using PTFE
D. Femoro-distal bypass graft using PTFE with Miller Cuff
E. Femoro-distal bypass graft using long saphenous vein
F. Above knee amputation
G. Below knee amputation

Please select the most appropriate management for the vascular scenario given. Each option may be used once, more than once or
not at all.

39. A 49 year old man who smokes 25 cigarettes a day presents with critical limb ischaemia. He has no previous history of
vascular surgery. MRA shows long superficial femoral arterial occlusion with retrograde filling of the above knee
popliteal.

The correct answer is Femoro-above knee popliteal bypass graft using PTFE. This man would be suitable for above knee
femoro-popliteal bypass grafting. At this level PTFE and vein will have similar patency rates. Further arterial surgery in this
man is highly likely given his young age and so vein should be conserved for the time being.

40. A 63 year old man who previously smoked 20 cigarettes a day and has newly diagnosed type II diabetes. He presents
with rest pain. A diagnostic angiogram demonstrates occlusion of the distal superficial femoral artery continuing below
the knee. He has reasonable posterior tibial artery below this level which branches to give good runoff to the foot. He
has varicose veins.

Femoro-distal bypass graft using PTFE with Miller Cuff. This man needs a bypass operation. Using PTFE alone will not give a
good result as sub intimal hyperplasia will give poor outcome early. Using a vein cuff (Miller cuff) at the end of a PTFE graft
will improve the situation.

41. A wheelchair bound 78 year old women with ischaemic heart disease secondary to long smoking history and
longstanding type II diabetes presents with rest pain and a non healing ulcer on the dorsum of her foot. Angiogram
shows reasonable superficial femoral artery and iliacs. At the level of the popliteal there is an occlusion. Below this there
is a short area of patent posterior tibial and this reconstitutes lower down the leg to flow to the foot.

The correct answer is Above knee amputation. A femoro-distal bypass graft would carry a high risk of failure and risk of peri-
operative myocardial infarct. This lady would be well suited to primary amputation.

42. Which of the following is not a feature of a Charcot foot?


B. Bounding foot pulses in the early phases
C. Often occurs in the complete absence of trauma
D. Erythema of the foot in the early phase
E. Autonomic neuropathy
F. Peripheral neuropathy
Answers: B
Do not confuse the early phase of Charcot foot with cellulitis

Trauma (even if only minor) is a prerequisite. Patients cannot usually recall the traumatic event. The associated neuropathy means
that patients continue to walk on the affected foot with subsequent deformity developing over time.

43. A 34 year old man presents with varicose veins and it is suspected that these are part of the Klippel-Trenaunay
syndrome. Which of the following is not a characteristic of this condition?
A. Presence of varicose veins
B. Gigantism of a limb

430
C. Long saphenous vein involvement
D. Port wine stains with clear borders
E. Arteriovenous fistulae
Answer: C
The Klippel-Trenaunay vein is a large, lateral, superficial vein sometimes seen at birth. This vein begins in the foot or the lower
leg and travels proximally until it enters the thigh or the gluteal area. Otherwise, varicosities may not be clinically evident until
the child begins to ambulate.
Varicosities may be extensive, though they often spare the saphenous distribution. They are seen below the knee, laterally
above the knee, and occasionally in the pelvic region. Varicosities may affect the superficial, deep, and perforating venous
systems.
Surgical exploration has demonstrated atresia and agenesis of deep veins, compression due to fibrous bands, aberrant arteries,
abnormal muscles, or venous sheaths.
Rarely, varicosities have been found in the bladder, the colon, and the pulmonary vessels

Klippel-Trenaunay syndrome: generally affects a single extremity, although cases of multiple affected limbs have been
reported. The leg is the most common site followed by the arms, the trunk, and rarely the head and the neck

Signs and symptoms


The birth defect is diagnosed by the presence of a combination of these symptoms: One or more distinctive port-wine stains with
sharp borders. Varicose veins. Hypertrophy of bony and soft tissues, that may lead to local gigantism or shrinking. An improperly
developed lymphatic system
In some cases, port-wine stains (capillary port wine type) may be absent. Such cases are very rare and may be classified as
"atypical Klippel-Trenaunay syndrome".

KTS can either affect blood vessels, lymph vessels, or both. The condition most commonly presents with a mixture of the two.
Those with venous involvement experience increased pain and complications.

Theme: Diabetic foot sepsis

A. IV broad spectrum antibiotics


B. Incision and drainage of pus
C. Ray amputation
D. Below knee amputation
E. Above knee amputation
F. Vacuum Assisted Closure device (VAC)
G. Discharge home
H. Application of 4 layer bandages

Please select the most appropriate management for the scenario given. Each option may be used once, more than once or not at all.

44. A 68 year old man with type II diabetes has a non healing ulcer following a ray amputation 2 weeks ago. An x-ray shows
no osteomyelitis and the ABPI is >1.

The correct answer is Vacuum Assisted Closure device (VAC). A VAC dressing may avoid the need for further surgery.

45. A 48 year old woman is admitted with sepsis secondary to an infected diabetic foot ulcer. She has a necrotic and infected
forefoot with necrosis of the heel. There is a boggy indurated swelling anterior to the ankle joint. Pulses are normal.

The correct answer is Below knee amputation. A below knee amputation is the best option here. The foot is non salvageable.
However, she may ambulate with a prosthesis.

46. An 84 year old lady is admitted with an infected diabetic foot. An x-ray shows osteomyelitis of her calcaneum. She has a
fixed flexion deformity of her knee, but normal pulses.

Above knee amputation. This patient will not be able to walk with a below knee amputation, therefore an above knee
amputation would be preferable, as it guarantees better healing the short term. Consider above knee amputation in patients with
fixed flexion deformity.

47. Which of the following statements related to coronary artery bypass surgery is true?
A. Late graft stenosis is mainly associated with saphenous vein grafts
B. Is indicated if there is stenosis > 70% of the right coronary artery
431
C. The left atrium is cannulated during the procedure
D. The CHADS score is used to assess peri operative risk
E. Cardioplegia is always undertaken at a 37 degrees

Indications are: 1. Left main stem stenosis or equivalent (proximal LAD and proximal circumflex). 2. Triple vessel disease. 3.
Diffuse disease unsuitable for PCI. The right atrium is cannulated. The CHADS score assesses whether a patient should be
warfarinised if they have atrial fibrillation. Cardioplegia can be undertaken at cold or warm temperatures.

43. A 73 year old man develops sudden onset abdominal pain and collapses. On examination he has a tender pulsatile mass
in his upper abdomen. He has a blood pressure of 90/60mmHg and pulse rate of 105 beats per minute. Which of the
following intravenous fluid regimens is most appropriate, whilst waiting for operative repair?
A. 1 Litre of pentastarch over 15 minutes
B. 1 litre of Hartmans solution over 4 hours
C. 1 litre of gelofusin over 30 minutes
D. 1 litre of Hartmans solution over 30 minutes
E. 1 litre of blood over 15 minutes
Answer: B
This man will have a contained haematoma and is awaiting surgery. Rapid, high volume infusions may cause this to dislodge
with disastrous consequences

Theme: Mesenteric vascular disease

A. Abdominal ultrasound
B. Abdominal CT with venous phase contrast
C. CT angiogram
D. Magnetic resonance angiogram
E. Duplex ultrasound
F. Abdominal x-ray
G. Meseneric angiography via brachial artery

Please select the most appropriate investigation for the scenario given. Each option may be used once, more than once or not at
all.

44. A thin 72 year old lady has a 3 week history of postprandial abdominal pain that is centrally located. She has episodic
diarrhoea and occasionally has passed blood PR. She has a history of ischaemic heart disease and marked renal
impairment from ACE inhibitor usage.

The correct answer is Duplex ultrasound. She is likely to have mesenteric vascular disease. Proximal SMA disease would be the
most serious variant. Ideally a CT angiogram would be the best test but with her impaired renal function and low BMI, make a
duplex of the SMA is a reasonable first line investigation. Gut peristalsis may impair acquisition of magnetic resonance images.

45. A 78 year old man develops sudden onset abdominal pain and almost immediately afterwards passes a large amount of
diarrhoea.

CT angiogram. Sudden onset of abdominal pain followed by forceful evacuation are the classical presenting features of acute
mesenteric infarction. This is best investigated by CT angiography, which has a sensitivity of 95% for the diagnosis.

46. A 28 year old female has suffered from abdominal pain for the past 2 weeks since she was started on the contraceptive
pill. The pain has increased significantly over the post 10 hours and has been associated with vomiting.

The correct answer is CT angiogram. Mesenteric venous thrombosis is the likely underlying cause and an angiogram is the
sensible step as it will also facilitate the identification of areas of infarcted bowel , similar to that which may occur in the leg
when massive DVT is present.

Vascular investigations: Venous disease: Venous Doppler: The simplest investigation for assessment of venous junctional
incompetence is a Doppler assessment. This involves the patient standing and manual compression of the limb distal to the
junction of interest. Flow should normally occur in one direction only. Where junctional incompetence is present reverse flow will
occur and is relatively easy to identify.

Venograms and duplex scans: Structural venous information is historically obtained using a venogram. This is an invasive test
and rarely required in modern clinical practice. The most helpful test is a venous duplex scan which will provide information
432
relating to flow and vessel characteristics. Duplex is also useful in providing vein maps for bypass surgery.

Arterial disease: Ankle-brachial pressure: The ankle brachial pressure index measurement is an important investigation as it
will allow classification of the severity of the flow compromise present. False readings may occur in those with calcified vessels
such as diabetics and results in such settings should be interpreted with caution. When auscultating the vessel note should be made
of the character of the signal. Monophasic signals are associated with a proximal stenosis and reduction in flow. Triphasic signals
provide reassurance of a healthy vessel.

Arterial Duplex: As with the vein the duplex scan can provide a substantial amount of information about arterial patency and
flow patterns. In skilled hands they can provide insight as to the state of proximal vessels that are anatomically inaccessible to
duplex (e.g. Iliacs). Through assessment of distal flow patterns. It is an operator dependent test.

Conventional angiogram: Vessel puncture and catheter angiography is the gold standard method of assessing arteries. High
quality information can usually be obtained. Limitations of the technique include the risk of contrast toxicity and risks of vessel
damage. Severely calcified vessels may be difficult to puncture and in this situation a remote access site (e.g. brachial) may be
used. This technique is particularly useful in providing a distal arterial roadmap prior to femoro-distal bypass.

CT angiography: These tests provide a considerable amount of structural and flow information. They require contrast and thus
carry the risks associated with this. They are particularly useful in the setting of GI bleeding as they are rapidly available and can
be performed by a non vascular radiologist. However, they lack the facility for endovascular intervention. In general they do not
provide high enough resolution for distal arterial surgery.

Magnetic resonance angiography: This has the advantage of being non-invasive and not using nephrotoxic contrast. Movement
artifact remains a problem in some sites and distal arterial resolution is imperfect.

43. A 52 year old male attends the stroke unit with dizziness and vertigo while playing tennis. He is known to have
hypertension and a previous myocardial infarct. He now complains of right arm pain. What is the most likely diagnosis?
A. Posterior circulation infarct
B. Vertebrobasilar aneurysm
C. Dissection of thoracic aorta
D. Subclavian steal syndrome
E. Left middle cerebral artery infarct
Answer: D
Subclavian steal syndrome characteristically presents with posterior circulation symptoms, such as dizziness and vertigo, during
exertion of an arm. There is subclavian artery steno-occlusive disease proximal to the origin of the vertebral artery and is associated
with flow reversal in the vertebral artery. Management involves percutaneous transluminal angioplasty or a stent.

Subclavian artery: Path: The left subclavian comes directly off the arch of aorta. The right subclavian arises from the
brachiocephalic artery (trunk) when it bifurcates into the subclavian and the right common carotid artery. From its origin, the
subclavian artery travels laterally, passing between anterior and middle scalene muscles, deep to scalenus anterior and anterior to
scalenus medius. As the subclavian artery crosses the lateral border of the first rib, it becomes the axillary artery. At this point it is
superficial and within the subclavian triangle. Branches: Vertebral artery, Internal thoracic artery, Thyrocervical trunk,
Costocervical trunk, Dorsal scapular artery.

44. A 74 year old lady has a long standing venous leg ulcer overlying her medial malleolus. Which of the following
statements relating to the management of this condition is false?
A. Pentoxifylline may speed ulcer healing
B. Treatment with daily low dose flucloxacillin may speed ulcer healing
C. Multilayer bandages may provide compression equivalent to 40mmHg
D. Large ulcers may be considered for skin grafting
E. They should not be treated with compression stockings if the ankle / brachial pressure index is 0.6
Answer: B
Routine use of antibiotics is not advised as this may predispose to resistant organisms. Pentoxifylline was subjected to a Cochrane
review in 2007 and shown to improve healing rates.

Theme: Investigation of vascular disease

A. Angiography

433
B. Arterial duplex scan
C. Arch aortogram
D. CXR
E. CT scan
F. Venous duplex scan

Please select the most appropriate investigation for the scenario given. Each option may be used once, more than once or not at
all.

45. A 22 year old professional tennis player attends A&E with a swollen painful right arm. His fingers are dusky.

The correct answer is Venous duplex scan. This patient has an axillary vein thrombosis. It classically presents with pain and
swelling of an effort induced limb. Duplex scan is needed to exclude a thombus.

46. A 65 year old man presents with expressive dysphasia and left sided weakness over 4h. His symptoms have now
completely resolved.

The correct answer is Arterial duplex scan. This patient has had a transient ischaemic attack. He urgently needs carotid duplex
scanning to assess if he needs a carotid endarterectomy.

47. A 65 year old man presents, for the first time, with pain at the back of his calves when he mobilises 10 metres. He is
known to have hypertension.

The correct answer is Arterial duplex scan. An arterial duplex should be performed first, before progression to an angiography.

Theme: Ankle Brachial pressure index measurements

A. >1
B. 0.6- 0.8
C. 1
D. 0.4-0.6
E. <0.4

For the scenarios described below, please select the most likely ankle brachial pressure index measurement. Each option may be
used once, more than once or not at all.

43. An 83 year old male with rest pain.

<0.4. Theme from April 2012 Exam. Rest pain is typically associated with low ABPI values.

44. A 45 year old man who develops calf pain after walking 600 yards. It resolves during periods of rest.

0.6- 0.8. Since this is a long claudication distance it may be that only a minor lesion is present. Whilst resting ABPI may be
normal, they are usually abnormal following exercise.

45. A 43 year old lady with long standing diabetes who complains of calf pain. It is worse at night and during minor
exercise.

>1. Diabetes may be complicated by vessel calcification and neuropathic pain. Therefore individuals may present with pain
which is atypical for claudation both in terms of its tempo of onset and location.

Theme: Causes of ulceration

A. Marjolin's ulcer
B. Neuropathic ulcer
C. Arterial ulcer
D. Deep venous dysfunction
E. Superficial venous dysfunction
F. Rheumatoid arthritis

434
G. Pyoderma gangrenosum
H. Pressure ulcer

Please select the most likely cause of ulceration for the scenario given. Each option may be used once, more than once or not at
all.

43. A 62 year old diabetic man presents with long standing plantar ulcer he has clinical evidence of a charcot foot.

The correct answer is Neuropathic ulcer. Plantar ulcers in association with peripheral neuropathy are often neuropathic. They
classically occur at pressure points.

44. A 66 year old female has long standing mixed arteriovenous ulcers of the lower leg. Over the past 6 months one of the
ulcers has become much worse and despite a number of different topical therapies is increasing in size.

Marjolin's ulcer. Marjolin's ulcer is a squamous cell carcinoma occurring at sites of chronic inflammation or previous injury.

45. A 28 year old man undergoes a ileocaecal resection and end ileostomy for Crohn's disease. One year later he presents
with a deep painful ulcer at his stoma site.

Pyoderma gangrenosum. Pyoderma gangrenosum is associated with inflammatory bowel disease (this patient had a stoma for
crohns!). It is commonly found on lower limbs and described as being painful, the size of an insect bite and growing. It looks
like a margarita pizza (with a red base and yellow topping!) Treatment involves steroids.

Breast & Endocrine Surgery


1. A 43 year old lady has recently undergone a wide local excision and sentinel lymph node biopsy for carcinoma of the
breast. Of the factors listed below, which will provide the most important prognostic information?
A. Mitotic number
B. Grade
C. Nodal status
D. Size
E. Oestrogen receptor status
Nodal status is the single most important prognostic factor in breast cancer. Nodal status is important because it serves as a marker
of tumour metastatic potential. This translates to survival advantages of up to 40% at five years. Both grade and size are of
secondary importance as they both less concerning in the absence of nodal involvement. Theme from April 2012 Exam

Breast cancer: Commoner in the older age group. Invasive ductal carcinomas are the most common type. Some may arise as a
result of ductal carcinoma in situ (DCIS). There are associated carcinomas of special type e.g. Tubular that may carry better
prognosis. The pathological assessment involves assessment of the tumour and lymph nodes, sentinel lymph node biopsy is often
used to minimise the morbidity of an axillary dissection. Treatment, typically this is either wide local excision or mastectomy.
There are many sub types of both of these that fall outside of the MRCS. Some key rules to bear in mind. Whatever operation is
contemplated the final cosmetic outcome does have a bearing. A women with small breasts and a large tumour will tend to fare
better with mastectomy, even if clear pathological and clinical margins can be obtained. Conversely a women with larger breasts
may be able to undergo breast conserving surgery even with a relatively large primary lesion (NB tumours >4cm used to attract
recommendation for mastectomy). For screen detected and impalpable tumour image guidance will be necessary. Reconstruction
is always an option following any resectional procedure. However, its exact type must be tailored to age and co-morbidities of the
patient. The main operations in common use include latissimus dorsi myocutaneous flap and sub pectoral implants. Women
wishing to avoid a prosthesis may be offered TRAM or DIEP flaps.

Surgical options: Mastectomy vs Wide local excision

Mastectomy Wide Local Excision


Multifocal tumour Solitary lesion
Central tumour Peripheral tumour
Large lesion in small breast Small lesion in large breast
DCIS >4cm DCIS <4cm
435
Patient Choice Patient choice

Central lesions may be managed using breast conserving surgery where an acceptable cosmetic result may be obtained, this is
rarely the case in small breasts. A compelling indication for mastectomy, a larger tumour that would be unsuitable for breast
conserving surgery. Whatever surgical option is chosen the aim should be to have a local recurrence rate of 5% or less at 5 years
[1].

Nottingham Prognostic Index: The Nottingham Prognostic Index can be used to give an indication of survival. In this system the
tumour size is weighted less heavily than other major prognostic parameters.

Calculation of NPI: Tumour Size x 0.2 + Lymph node score(From table below)+Grade score(From table below).

Lymph nodes involved Grade


Score
1 0 1
2 1-3 2
3 >3 3

Prognosis: Score: Percentage 5 year survival

2.0 to 2.4: 93%, 2.5 to 3.4: 85%, 3.5 to 5.4: 70%, >5.4: 50%.

This data was originally published in 1992. It should be emphasised that other factors such as vascular invasion and receptor
status also impact on survival and are not included in this data and account for varying prognoses often cited in the literature.

Theme: Management of nipple discharge

A. Prescribe danazol
B. Microdochectomy
C. Total duct excision
D. Cytology of duct fluid
E. Core biopsy
F. Prescribe co-amoxiclav
G. Reassure and discharge
H. Mastectomy

What is the best management for each nipple discharge presentation? Each option may be used once, more than once or not at all.

2. A 23 year old women with greenish nipple discharge on one occasion. Clinical examination of the breast is normal.
Ultrasound report is U1.

The correct answer is Reassure and discharge. This is likely to be simple duct ectasia and U1 (normal USS) coupled with
normal examination would favor discharge from clinic. Mammography is generally unhelpful in this age group

3. A 43 year old women has had recurrent episodes of periductal mastitis. She has received multiple courses of antibiotics
and is troubled by persisting green nipple discharge. Clinical examination reveals green nipple discharge, but no discrete
lump. Imaging with mammography and ultrasound is reassuring (U2, M2)

The correct answer is Total duct excision. This woman has troublesome duct ectasia and total duct excision is warranted.

4. A 55 year old women complains of nipple discharge. This was blood stained on one occasion. But not subsequently.
Clinical examination shows clear fluid but no discrete lump. Imaging with ultrasound and mammography is normal.

The correct answer is Microdochectomy. Although this is likely to be benign disease, her age coupled with an episode of blood
stained discharge would attract a recommendation for microdochectomy. She may have an intraductal papilloma. But the
concern would be DCIS.

Nipple discharge: Causes of nipple discharge: Physiological: During breast feeding

436
Galactorrhoea: Commonest cause may be response to emotional events, drugs such as histamine receptor anatagonists are also
implicated

Hyperprolactinaemia: Commonest type of pituitary tumour. Microadenomas <1cm in diameter. Macroadenomas >1cm in
diameter. Pressure on optic chiasm may cause bitemporal hemianopia

Mammary duct ectasia: Dilatation breast ducts. Most common in enopausal women. Discharge typically thick and green in
colour. Most common in smokers

Carcinoma: Often blood stained. May be underlying mass or axillary lymphadenopathy

Intraductal papilloma: Commoner in younger patients. May cause blood stained discharge. There is usually no palpable lump

Assessment of patients: Examine breast and determine whether there is mass lesion present. All mass lesions should undergo
Triple assessment.

Reporting of investigations: Where a mass lesion is suspected or investigations are requested these are prefixed using a system
that denotes the investigation type e.g. M for mammography, followed by a numerical code as shown below: 1: No abnormality;
2: Abnormality with benign features; 3: Indeterminate probably benign; 4: Indeterminate probably malignant; 5: Malignant.

Management of non malignant nipple discharge: Exclude endocrine disease. Nipple cytology unhelpful. Smoking cessation
advice for duct ectasia. For duct ectasia with severe symptoms, total duct excision may be warrented.
Theme: Breast disease

A. Ductal carcinoma in situ


B. Lobular carcinoma in situ
C. Invasive ductal carcinoma
D. Invasive lobular carcinoma
E. Inflammatory carcinoma
F. Phyllodes tumour
G. Paget's disease of the nipple
H. Fibroadenoma
I. Mucinous breast carcinoma

From the list please select the most likely diagnosis for the scenario given. Each diagnosis may be used once, more than once or
not at all.

5. A 32 year old Indian lady presents with breast lump. She has a 4 month old child. Clinically she has jaundice and there
is erythema of the left breast.

The correct answer is Inflammatory carcinomaInflammatory breast cancers have an aggressive nature. Dissemination occurs
early and is more resistant to adjuvent treatments than other types of breast cancer. Often occurs in pregnancy or lactation.

6. A 72 year old female presents with a painless breast lump. Clinically she has a 4cm diameter irregular breast mass, with
no other palpable masses..

The correct answer is Invasive ductal carcinoma. A post menopausal woman is more likely to have a ductal carcinoma and they
tend to occur at a single focus within the breast.

7. A 72 year old woman presents with 2 breast lumps. She has a history of breast cancer in the opposite breast 5 years ago.

Invasive lobular carcinoma. This is likely to be an invasive lobular carcinoma, mainly due to the multifocal lesions and the
history of previous breast cancer in the opposite breast.

8. A 72 year old female is found to have a malignant lesion in her left arm. She had a mastectomy of the left breast 10 years
ago and has chronic lymph oedema of the left arm. What is the most likely cause of the malignancy?
A. Lymphangiosarcoma
B. Lymphoma
C. Myeloma
D. Angiomyolipoma
437
E. Giant cell tumour
Answer: A
Lymphangiosarcoma is a rare condition arising as a result of chronic oedema. It is an aggressive malignancy.
Lymphoedema: Due to impaired lymphatic drainage in the presence of normal capillary function. Lymphoedema causes the
accumulation of protein rich fluid, subdermal fibrosis and dermal thickening. Characteristically fluid is confined to the epifascial
space (skin and subcutaneous tissues); muscle compartments are free of oedema. It involves the foot, unlike other forms of
oedema. There may be a 'buffalo hump' on the dorsum of the foot and the skin cannot be pinched due to subcutaneous fibrosis.

Causes of lymphedema: Primary: Congenital < 1 year: sporadic, Milroy's disease. Onset 1-35 years: sporadic, Meige's disease.
> 35 years: Tarda

Secondary: Bacterial/fungal/parasitic infection (filariasis). Lymphatic malignancy. Radiotherapy to lymph nodes. Surgical
resection lymph nodes. DVT. Thrombophlebitis

Indications for surgery: Marked disability or deformity from limb swelling. Lymphoedema caused by proximal lymphatic
obstruction with patent distal lymphatics suitable for a lymphatic drainage procedure. Lymphocutaneous fistulae and
megalymphatics

Procedures
Homans operation Reduction procedure with preservation of overlying skin (which must be in good condition). Skin flaps
are raised and the underlying tissue excised. Limb circumference typically reduced by a third.
Charles operation All skin an subcutaneous tissue around the calf is excised down to the deep fascia. Split skin grafts are
placed over the site. May be performed if overlying skin is not in good condition. Larger reduction in size
than with Homans procedure.
Lymphovenous Identifiable lymphatics are anastomosed to sub dermal venules. Usually indicated in 2% of patients with
anastamosis proximal lymphatic obstruction and normal distal lymphatics.

9. A 58 year old male is referred to endocrinology clinic for a parathyroidectomy by the F1 in medicine. His corrected
calcium is 2.85 (2.2-2.6), PTH 7.5 (3-7) and 24h urinary calcium is 1.5 (2.5-7.5). What is the diagnosis?
A. Primary hyperparathyroidism
B. Secondary hyperparathyroidism
C. Tertiary hyperparathyroidism
D. Familial hypocalciuric hypercalcaemia
E. Hypercalacemia associated with malignancy
Answer: D
This F1 should have spoken to his senior. This patient has familial hypocalciuric hypercalcaemia, which requires no further
action.
Parathyroid glands and disorders of calcium metabolism: Hyperparathyroidism
Disease type Hormone profile Clinical features Cause
Primary  PTH (Elevated)  May be asymptomatic if Most cases due to solitary adenoma
hyperparathyroidism  Ca2+ (Elevated) mild (80%), multifocal disease occurs in
 Phosphate (Low)  Recurrent abdominal pain 10-15% and parathyroid carcinoma in
 Serum Calcium : (pancreatitis, renal colic) 1% or less
Creatinine clearance  Changes to emotional or
ratio > 0.01 cognitive state

Secondary  PTH (Elevated)  May have few symptoms Parathyroid gland hyperplasia occurs
hyperparathyroidism  Ca2+ (Low or normal)  Eventually may develop as a result of low calcium, almost
 Phosphate (Elevated) bone disease, osteitis fibrosa always in a setting of chronic renal
 Vitamin D levels cystica and soft tissue failure
(Low) calcifications

Tertiary  Ca2+ (Normal or  Metastatic calcification Occurs as a result of ongoing


hyperparathyroidism high)  Bone pain and / or fracture hyperplasia of the parathyroid glands
 PTH (Elevated)  Nephrolithiasis after correction of underlying renal
 Phosphate levels  Pancreatitis disorder, hyperplasia of all 4 glands is
(Decreased or usually the cause

438
Normal)
 Vitamin D (Normal
or decreased)
 Alkaline phosphatase
(Elevated)

Differential diagnoses: It is important to consider the rare but relatively benign condition of benign familial hypocalciuric
hypercalcaemia, caused by an autosomal dominant genetic disorder. Diagnosis is usually made by genetic testing and concordant
biochemistry (Serum Calcium : Creatinine clearance ratio <0.01-distinguished from primary hyperparathyroidism).

Treatment: Primary hyperparathyroidism


Indications for surgery: Elevated serum Calcium > 1mg/dL above normal. Hypercalciuria > 400mg/day. Creatinine clearance <
30% compared with normal. Episode of life threatening hypercalcaemia. Nephrolithiasis. Age < 50 years. Neuromuscular
symptoms. Reduction in bone mineral density of the femoral neck, lumbar spine, or distal radius of more than 2.5 standard
deviations below peak bone mass (T score lower than -2.5)

Secondary hyperparathyroidism: Usually managed with medical therapy. Indications for surgery in secondary (renal)
hyperparathyroidism: Bone pain. Persistent pruritus. Soft tissue calcifications

Tertiary hyperparathyroidism: Usually treatment is surgical: The presence of an autonomously functioning parathyroid gland
may require surgery. If the culprit gland can be identified then it should be excised. Otherwise total parathyroidectomy and re-
implantation of part of the gland may be required.

Theme: Management of breast cancer

A. Simple mastectomy alone


B. Radical mastectomy alone
C. Simple mastectomy and sentinel lymph node biopsy
D. Wide local excision and sentinel lymph node biopsy
E. Simple mastectomy and axillary node clearance
F. Radical mastectomy and axillary node clearance
G. Wide local excision and axillary node clearance
H. Wide local excision alone

Please select the most appropriate treatment for the situation described. Each option may be used once, more than once or not at
all.

10. A 44 year old lady presents with a mass in the upper outer quadrant of her right breast. Imaging, histology and clinical
examination confirm a 1.5cm malignant mass lesion with no clinical evidence of axillary nodal disease.

Wide local excision and sentinel lymph node biopsy. A small peripheral lesion such as this would usually be suitable for breast
conserving surgery. Since imaging and clinical examination is not suspicious for axillary disease, a sentinel lymph node biopsy
should be performed.

11. An infirm 79 year old lady presents with a mass lesion in the middle of her right breast. A core biopsy is performed
which confirms a low grade multifocal ductal carcinoma in situ.

Simple mastectomy alone. Multifocal DCIS is usually treated with a simple mastectomy. A wide local excision is not a sensible
option where multifocal disease is present.

12. A 39 year old lady presents with a mass lesion in her right breast. Clinical examination, biopsy and imaging confirm a 2.5
cm lesion in the upper inner quadrant of her right breast and a 1.5 cm lesion at the central aspect of the same breast. Her
axilla shows lymphadenopathy and a fine needle aspirate from the node shows malignant cells.

The correct answer is Simple mastectomy and axillary node clearance. A combination of established axillary disease and
multifocal invasive lesions attracts an indication for mastectomy and axillary clearance. A radical mastectomy is less frequently
indicated in modern surgical practice, disease that is locally advanced is often best downstaged using medical therapy, rather
than embarking on the operations for breast cancer that were first popularised over 100 years ago.

Breast cancer management: Surgery is performed in most patients suffering from breast cancer. Chemotherapy may be used to
downstage tumours and allow breast conserving surgery. Hormonal therapy may also be used for the same purposes. Radiotherapy

439
is given to all patients who have undergone breast conserving surgery. Patients who have undergone mastectomy may be offered a
reconstructive procedure either in conjunction with their primary resection or as a staged procedure at a later date.

Axillary disease: As a minimum, all patients with invasive breast cancer should have their axilla staged. In those who do not have
overt evidence of axillary nodal involvement this can be undertaken using sentinel lymph node biopsy. Patients with a positive
sentinel lymph node biopsy or who have imaging and cytological or histological evidence of axillary nodal metastasis should
undergo axillary node clearance. Axillary node clearance is associated with the development of lymphoedema, increased risk of
cellulitis and frozen shoulder.

13. A 50 year old lady is commenced on tamoxifen for the treatment of an oestrogen receptor positive breast cancer. Which
of the following malignancies are associated with tamoxifen use?
A. Adenocarcinoma of the colon
B. Hodgkins lymphoma
C. Adenocarcinoma of the lung
D. Ovarian cancer
E. Endometrial cancer
Answer: E
Tamoxifen is an oestrogen receptor antagonist in breast tissues. However, at other sites, such as the endometrium it may act as
an agonist. Hence the reason for increasing risk of endometrial cancer.

Tamoxifen: Synthetic partial oestrogen agonist, acts primarily by binding to the oestrogen receptor. Half life of 7 days, takes 4
weeks for drug to reach plasma steady state. Should usually be considered in patients with oestrogen receptor positive tumours
(alternative agents may be preferred in some groups). Although antagonistic with respects to breast tissue tamoxifen may serve as
an agonist at other sites. Therefore risk of endometrial cancer is increased, preservation of bone density and decreased
cardiovascular risks. Climateric side effects are common, 3% stop taking the drug because of these. Aromatase inhibitors are an
alternative class of drugs, these work by blocking the peripheral aromatization of androgens (post menopausal women produce
oestrogens in this way). They may treat cancers for which tamoxifen is no longer effective.

Theme: Thyroid nodules

A. Toxic adenoma
B. Anaplastic carcinoma of thyroid
C. Follicular carcinoma of thyroid
D. Papillary carcinoma of thyroid
E. Medullary carcinoma of thyroid
F. Thyroid lymphoma
G. Multinodular goitre
H. Parathyroid gland tumour

For each scenario please select the most likely underlying diagnosis. Each option may be used once, more than once or not at all.

14. A 52 year old woman with known Hashimotos thyroiditis presents with a neck swelling. She describes it as rapidly
increasing in size over 3 months and she complains of dysphagia to solids. On examination there is an asymmetrical
swelling of the thyroid gland.

The correct answer is Thyroid lymphoma. Thyroid lymphoma (Non Hodgkin's B cell lymphoma) is rare. It should be considered
in patients with a background of Hashimoto's thyroiditis and a rapid growth in size of the thyroid gland. Diagnosis can be made
with fine-needle aspiration or core needle biopsy; however an incisional biopsy may be needed. Radiotherapy is the main
treatment option.
15. A 52 year old woman presents with a neck swelling. On examination she is noted to have single nodule on the thyroid
gland. A CXR shows two mass lesions.

The correct answer is Follicular carcinoma of thyroid. A solitary nodule with signs of haematogenous spread indicates a
follicular tumour. Note that papillary tumours tend to be multinodular and spread via the lymphatic system.
16. A 52 year old woman presents with a neck swelling. Her GP reports that her TSH value is low at 0.01. A scintigraphy
demonstrates a hot nodule.

Toxic adenoma. This lady has thyrotoxicosis (low TSH) and a hot solitary nodule indicating a toxic adenoma. Thyroid cancer
rarely causes thyrotoxicosis or hot nodules.

440
Thyroid disease: Patients may present with a number of different manifestations of thyroid disease. They can be broadly sub
classified according to whether they are euthyroid or have clinical signs of thyroid dysfunction. In addition it needs to be
established whether they have a mass or not.

Assessment: History. Examination including USS. If a nodule is identified then it should be sampled ideally via an image guided
fine needle aspiration. Radionucleotide scanning is of limited use

Thyroid Tumours: Papillary carcinoma. Follicular carcinoma. Anaplastic carcinoma. Medullary carcinoma. Lymphoma's

Multinodular goiter: One of the most common reasons for presentation. Provided the patient is euthyroid and asymptomatic and
no discrete nodules are seen, they can be reassured. In those with compressive symptoms surgery is required and the best
operation is a total thyroidectomy. Sub total resections were practised in the past and simply result in recurrent disease that
requires a difficult revisional resection.

Endocrine dysfunction: In general these patients are managed by physicians initially. Surgery may be offered alongside radio
iodine for patients with Graves disease that fails with medical management or in patients who would prefer not to be irradiated
(e.g. pregnant women). Patients with hypothyroidism do not generally get offered a thyroidectomy. Sometimes people
inadvertently get offered resections during the early phase of Hashimotos thyroiditis, however, with time the toxic phase passes
and patients can simply be managed with thyroxine.

Complications following surgery: Anatomical such as recurrent laryngeal nerve damage. Bleeding. Owing to the confined space
haematoma's may rapidly lead to respiratory compromise owing to laryngeal oedema. Damage to the parathyroid glands resulting
in hypocalcaemia.

17. Which investigation is best for initial assessment of recurrence of follicular carcinoma of the thyroid?
A. Free T4
B. Thyroid stimulating hormone
C. Scintigraphy
D. Serum thyroglobulin
E. USS thyroid gland
Answer: D
Elevated thyroglobulin levels raises suspicion of recurrence.

Thyroid malignancy: Papillary carcinoma: Commonest sub-type. Accurately diagnosed on fine needle aspiration cytology.
Histologically they may demonstrate psammoma bodies (areas of calcification) and so called 'orphan Annie' nuclei. They typically
metastasise via the lymphatics and thus laterally located apparently ectopic thyroid tissue is usually a metastasis from a well
differentiated papillary carcinoma.

Follicular carcinoma: Are less common than papillary lesions. Like papillary tumours they may present as a discrete nodule.
Although they appear to be well encapsulated macroscopically there invasion on microscopic evaluation. Lymph node metastases
are uncommon and these tumours tend to spread haematogenously. This translates into a higher mortality rate. Follicular lesions
cannot be accurately diagnosed on fine needle aspiration cytology and thus all follicular FNA's will require at least a hemi
thyroidectomy.

Anaplastic carcinoma: Less common and tend to occur in elderly females. Disease is usually advanced at presentation and often
only palliative decompression and radiotherapy can be offered.

Medullary carcinoma: These are tumours of the parafollicular cells ( C Cells) and are of neural crest origin. The serum calcitonin
may be elevated which is of use when monitoring for recurrence. They may be familial and occur as part of the MEN -2A disease
spectrum. Spread may be either lymphatic or haematogenous and as these tumours are not derived primarily from thyroid cells
they are not responsive to radioiodine.

Lymphoma: These respond well to radiotherapy. Radical surgery is unnecessary once the disease has been diagnosed on biopsy
material. Such biopsy material is not generated by an FNA and thus a core biopsy has to be obtained (with care!).

18. A 33 year old lady attends the clinic with a 3 month history of palpitations and irritability. Her thyroid function, PTH
and calcium are measured: Thyroid function: Free T4 40 pmol/L, TSH < 0.1 miu/L, Free T3 25 p mol/L, PTH 10pg/ml.
(Normal values listed in reference range link). What is the most likely diagnosis?
A. Hypothyroidism
B. Hyperthyroidism
C. Hypoparathyroidism

441
D. Hyperparathyroidism
E. Euthyroid
Answer: B

Elevated T4 and suppressed TSH makes this the most likely diagnosis. The PTH level is normal.
Hyperthyroidism: Causes of hyperthyroidism include: Diffuse toxic goitre (Graves Disease), Toxic nodular goiter, Toxic
nodule,Rare causes. Theme from April 2012 Exam

Graves disease is characterised by a diffuse vascular goitre that appears at the same time as the clinical manifestations of
hyperthyroidism. It is commonest in younger females and may be associated with eye signs. Thyrotoxic symptoms will
predominate. Up to 50% of patients will have a familial history of autoimmune disorders. The glandular hypertrophy and
hyperplasia occur as a result of the thyroid stimulating effects of the TSH receptor antibodies.

Toxic nodular goiter: In this disorder the goitre is present for a long period of time prior to the development of clinical
symptoms. In most goitres the nodules are inactive and in some cases it is the internodular tissue that is responsible for the goitre.

Toxic nodule: Overactive, autonomously functioning nodule. It may occur as part of generalised nodularity or be a true toxic
adenoma. The TSH levels are usually low as the autonomously functioning thyroid tissue will exert a negative feedback effect.

Clinical features: Signs: Tachycardia. Agitation. Hot, moist palms. Exopthalmos. Thyroid goitre and bruit. Lid lag/retraction
Symptoms: Lethargy. Emotionally labile. Heat intolerance. Weight loss. Excessive appetite. Palpitations

Diagnosis: The most sensitive test for diagnosing hyperthyroidism is plasma T3 (which is raised). Note in hypothyroidism the
plasma T4 and TSH are the most sensitive tests. A TSH level of <0.5U/L suggests hyperthyroidism. TSH receptor antibodies may
be tested for in the diagnosis of Graves.

Treatment: First line treatment for Graves disease is usually medical and the block and replace regime is the favored option.
Carbimazole is administered at higher doses and thyroxine is administered orally. Patient are maintained on this regime for
between 6 and 12 months. Attempts are then made to wean off medication. Where relapse then occurs the options are between
ongoing medical therapy, radioiodine or surgery.

19. A 23 year old lady has Graves disease that has relapsed on stopping anti thyroid, radioiodine is offered as the next
treatment by the endocrinologists. Which statement is false?
A. Close contact with children is not permitted for up to 4 weeks following treatment.
B. 15% of patients with opthalmopathy will see worsening of eye signs.
C. Symptomatic improvement takes 6-8 weeks
D. Up to 80% of patients will become hypothyroid.
E. None of the above.
Answer: E
Radio-iodine- may worsen opthalmopathy, contraindicated in pregnancy and those wishing to concieve within 6 months.

Radioiodine vs. Surgery


Radioiodine: Symptomatic improvement takes up to 2 months. Eye signs may worsen. No risk of anatomical damage. No contact
with children for 4 weeks

Surgery: Symptomatic improvement within 10 days. No effect on opthalmopathy. Risk of damage to adjacent anatomical
structures. No restrictions on contact

Theme: Thyroid disease

A. Papillary carcinoma
B. Follicular carcinoma
C. Multinodular goitre
D. Parathyroid adenoma
E. Anaplastic thyroid carcinoma
F. Medullary carcinoma
G. Toxic nodule
H. Graves disease

Please select the most likely thyroid lesion for the scenario given. Each option may be used once, more than once or not at all.

20. A 34 year old female presents with a thyroid nodule. She has a family history of thyroid disease and both her sisters have
undergone total thyroidectomies. Her past medical history includes hypertension which has been difficult to manage.
442
Medullary carcinoma. This is a typical scenario for medullary carcinoma in which a phaeochromocytoma may also be present. It
may be inherited in an autosomal dominant fashion and affected family members may be offered prophylactic thyroidectomy.

21. A 46 year old man is admitted to hospital with a femoral shaft fracture that occurred suddenly whilst he was out walking
his dog. On examination there is no neurovascular deficit distal to the fracture site. He has a large firm nodule in the left
lobe of the thyroid, there is no associated lymphadenopathy.

Follicular carcinoma. Follicular carcinomas may metastasise haematogenously (often to bone) where they may give rise to
pathological fractures as in this case.

22. An 18 year old female presents with 3 nodules in the right lobe of the thyroid. Clinically she is euthyroid and there is
associated cervical lymphadenopathy. She has no family history of thyroid disease.

Papillary carcinoma. Papillary thyroid cancers are the most common type of thyroid cancer and are the more common in females
(M:F=1:3). Papillary tumours are more likely to develop lymphatic spread than follicular tumours.

23. A 19 year old male presents with bilateral gynaecomastia, poor vision and nipple discharge. Which of the following blood
tests is most likely to be abnormal?
A. Oestrogen
B. Testosterone
C. β HCG
D. Prolactin
E. Calcitonin
Answer: D
A combination of nipple discharge, gynaecomastia and poor vision may well be associated with a prolactinoma. The poor vision
results from compression of the optic chiasm resulting in bi temporal hemianopia.

Gynaecomastia

Gynaecomastia describes an abnormal amount of breast tissue in males and is usually caused by an increased oestrogen:androgen
ratio. It is important to differentiate the causes of galactorrhoea (due to the actions of prolactin on breast tissue) from those of
gynaecomastia

Causes of gynaecomastia: physiological: normal in puberty. syndromes with androgen deficiency: Kallman's, Klinefelter's.
testicular failure: e.g. Mumps. liver disease. testicular cancer e.g. Seminoma secreting hCG. ectopic tumour secretion.
Hyperthyroidism. Haemodialysis. drugs: see below

Drug causes of gynaecomastia: spironolactone (most common drug cause). Cimetidine. Digoxin. Cannabis. Finasteride.
oestrogens, anabolic steroids.

Very rare drug causes of gynaecomastia: tricyclics. Isoniazid. calcium channel blockers. Heroin. Busulfan. methyldopa

Theme: Management of calcium metabolic disorders

A. No action needed
B. Intravenous fluid (0.9% N.Saline)
C. Risedronate and calcium supplements
D. Calcium supplements
E. Exploration and parathyroidectomy
F. DEXA bone scan
G. Pamidronate IV

For each scenario please select the most appropriate management plan. Each option may be used once, more than once or not at
all.

24. An 80 year old woman has a hip fracture. Her calcium is normal. She has never been given a diagnosis of osteoporosis.

443
The correct answer is Risedronate and calcium supplements. The osteoporosis guidelines state if a postmenopausal woman has a
fracture she should be put on bisphosphonates (there is no need for a DEXA scan).

25. A 60 year old man presents with recurrent renal stones. He is found to have a calcium of 2.72 (elevated) and a PTH of 12
(elevated).

Exploration and parathyroidectomy. This patient has primary hyperparathyroidism and nephrolithiasis, which is an indication for
parathyroidectomy.

26. An 82 year old woman from a nursing home is admitted to the orthopaedic ward with a hip fracture. She is acutely
confused and agitated. Her Calcium is 2.95 (elevated).

Intravenous fluid (0.9% N.Saline). This patient needs rehydration due to hypercalcaemia. An intravenous bisphosphonate is
indicated if the Ca is above 3.

Theme: Management of thyroid disease

A. Total Thyroidectomy
B. Thyroid lobectomy
C. Sub total thyroidectomy
D. Radioactive iodine
E. Carbimazole
F. Tru cut biopsy
G. Further fine needle aspiration
H. Observation

For each scenario please select the most appropriate management option. Each option may be used once, more than once or not at
all.

27. A 59 year old man is referred with symptoms of dysphagia. On examination he has a large goitre and on imaging there is
significant retrosternal extension and features of a multinodular goitre.

The correct answer is Total Thyroidectomy. Sub total thyroidectomy is no longer routinely undertaken in this group.

28. A 48 year old lady with thyrotoxicosis is referred to the clinic, she was poorly controlled on carbimazole and has
received orbital radiotherapy for severe proptosis. This has improved matters but she relapsed on stopping her
carbimazole.

The correct answer is Total Thyroidectomy. Eye signs worsen with radioiodine.

29. A 23 year old lady has re attended the clinic on three occasions with a cyst in her thyroid that refills. Cytology on each
occasion is reassuring.

The correct answer is Thyroid lobectomy. Persist refilling cysts may be associated with a well differentiated tumour and should
be removed by lobectomy.

30. Which of the following are not true of follicular thyroid cancer?
A. They often appear to be encapsulated.
B. Those with a Hurthle cell subtype have an excellent prognosis.
C. Haematogenous metastasis is more common than in Papillary carcinoma.
D. The overall mortality rate is 24%.
E. Vascular invasion is seen in up to 60% of cases.
Answer: B
The Hurthle cell subtype have a worse prognosis.

Theme: Thyroid function tests

A. Thyrotoxicosis

444
B. Sick euthyroid syndrome
C. Hypothyroidism
D. Poor compliance with thyroid medication
E. Hashimotos thyroiditis
F. Multinodular goitre

Please select the most likely diagnosis for the scenario given. Each option may be used once, more than once or not at all.

31. A 52 year old woman presents with an acute ischaemic right arm. She is found to have fast atrial fibrillation. Her blood
results reveal a free T4 level of 20 and a TSH of < 0.01.

Thyrotoxicosis. The diagnosis is thyrotoxicosis. An elevated T4 and a low TSH should indicate this diagnosis. Remember atrial
fibrillation and its complications i.e acute ischaemic limbs can be precipitated by hyperthyroid disorders.

32. A 42 year old woman presents with a goitre. On examination the goitre feels 'lumpy'. The blood results reveal a TSH of
12 and a free T4 of 2. Antithyroid peroxidase antibodies are high.

Hashimotos thyroiditis. Hashimotos usually presents in women aged between 30- 50 years. They are normally associated with a
goitre. To differentiate from hypothyroidism, the antithyroid peroxidase antibodies will be elevated.

33. A 55 year old man is on the intensive care unit for many months after open aortic surgery. He is maintained on total
parenteral nutrition. Clinically he is euthyroid, but his thyroid function tests reveal a low TSH and low T4.

Sick euthyroid syndrome. Sick euthyroid syndrome is most commonly seen in chronically ill patients or those with starvation.
The thyroid function tests are often low and the patient clinically euthyroid.

Thyroid function tests

Theme from January 2012 exam

The interpretation of thyroid function tests is usually straightforward:

Disorder TSH Free T4


Thyrotoxicosis (e.g. Graves' disease) Low High In T3 thyrotoxicosis the free T4 will be normal
Primary hypothyroidism (primary atrophic High Low
hypothyroidism)
Secondary hypothyroidism Low Low Replacement steroid therapy is required prior to
thyroxine
Sick euthyroid syndrome* Low** Low Common in hospital inpatients
Poor compliance with thyroxine High Normal /
high
Steroid therapy Low Normal

*now referred to as non-thyroidal illness. **TSH may be normal in some cases

Theme: Nottingham prognostic index

A. <2.5
B. <3.4
C. 3.4-5.4
D. >5.4
Please match the prognosis of patients who have undergone breast cancer surgery to the most appropriate Nottingham Prognostic
Index score. Each option may be used once, more than once or not at all.

30. Good prognosis

The correct answer is <3.4. The Nottingham prognostic index may be used to stratify patients into various prognostic groups (see
below). An excellent prognosis is seen with a score of <2.4. A good prognosis is usually associated with a score of <3.4.
445
31. Intermediate prognosis

3.4-5.4

32. Poor prognosis

>5.4

34. A 56 year old lady undergoes a mastectomy as treatment for multifocal ductal carcinoma in situ. Two weeks post
operatively she attends the clinic and complains of a diffuse swelling at the surgical site. On examination she has a large,
fluctuant area underlying the mastectomy skin flaps. She is otherwise well. What is the most likely cause?
A. Abscess
B. Seroma
C. Haematoma
D. Disease recurrence
E. Arteriovenous malformation
Answer: B
Seromas are very common after breast surgery. The exposed raw surfaces created during the elevation of the skin flaps are a
common cause. Treatment usually involves percutaneous drainage under aseptic conditions.

Complications of breast surgery: Breast surgery may be associated with the following complications: Long thoracic nerve
injury. This may occur during the axillary dissection and result in winging of the scapula. Intercostobrachial nerve injury. These
nerves traverse the axilla. When they are divided (which they often are) the patient will notice an area of parasthesia in the armpit.
Injury to the thoracodorsal trunk. This nerve and vessels supply latissimus dorsi. If they are damaged the functional effects are not
too serious, the greatest setback is that a latissimus dorsi flap cannot be used for reconstruction purposes. Infections. Cellulitis of
the chest wall and arm may be a major problem if axillary nodal clearance is undertaken. Infections may run a protracted course
and require polytherapy for treatment. Lymphoedema. Usually complicates axillary node clearance or irradiation. Treatment is
with manual lymphatic drainage and compression sleeves. Seroma. This is an accumulation of fluid at the site of surgery. The
fluid is usually straw coloured and may re-accumulate despite drainage. Most will resolve with time.
35. You are the specialist trainee in endocrinology clinic. The medical team have referred a man for a parathyroidectomy
who has a corrected calcium of 2.82 (elevated) and a PTH of 11 (elevated). Which of the following is not an indication
for parathyroidectomy?
A. Nephrolithiasis
B. Reduction in bone mineral density of the femoral neck, lumbar spine, or distal radius of more than 2.5 standard deviations
below peak bone mass
C. Age < 50 years
D. Episode of life threatening hypercalcaemia
E. None of the above
Answer: E

All of the situations listed are indications for parathyroidectomy. See below for more information.

Theme: Breast disease treatment

A. Mastectomy
B. Sentinel lymph node biopsy
C. Axillary node clearance
D. Wide local excision
E. Breast lump excision biopsy
F. Image guided wide local excision
G. Radiotherapy
H. Chemotherapy
I. Excision of margins
J. Discharge to routine follow-up

Please select the most appropriate management option from the list for the scenario given. Each option may be used once, more
than once or not at all.

36. A 35 year old woman has undergone a wide local excision. The histology shows an invasive lobular carcinoma present at
3 of the resection margins. Cavity shavings taken at the original operation are also involved. Sentinel node biopsy was
negative.

446
Mastectomy. This patient has an extensive disease process and lobular cancers are notorious for being multifocal. In this case a
mastectomy is the safest next step.

37. A 56 year old woman has undergone a wide local excision and axillary node sample. The histology shows a 2.5cm invasive
ductal carcinoma grade 1 which is completely excised. None of the axillary lymph nodes show evidence of metastatic
disease.

Radiotherapy. This woman has a good prognosis. However, irradiation of the breast is mandatory after breast cancer has been
treated using breast conserving surgery.

38. A 28 year old female presents with a painless lump in the upper outer quadrant of her left breast. Imaging using
ultrasound is indeterminate (U3). Two core biopsies have now been performed and both show normal breast tissue (B1).

The correct answer is Breast lump excision biopsy. The imaging and biopsy results are not concordant. At this stage an excision
biopsy is the safest option.

Theme: Non operative treatment of breast cancer

A. Radiotherapy
B. Cytotoxic chemotherapy
C. Surveillence alone
D. Endocrine therapy using tamoxifen
E. Endocrine therapy using letrozole
F. Endocrine therapy using medroxyprogesterone acetate

Please select the most appropriate agent for the situation described. Each option may be used once, more than once or not at all.

39. A 55 year old lady has undergone a wide local excision and sentinel lymph node biopsy for breast cancer. The histology
report shows a completely excised 1.3cm grade 1 invasive ductal carcinoma. The sentinel node contained no evidence of
metastatic disease. The tumour is oestrogen receptor negative.

Radiotherapy. Radiotherapy is routine following breast conserving surgery. Without irradiation the local recurrence rates are
approximately 40%.

40. An 88 year old lady presents with a large mass in the upper inner quadrant of her right breast. Investigations confirm an
oestrogen receptor positive, invasive ductal carcinoma. She has declined operative treatment.

The correct answer is Endocrine therapy using letrozole. Elderly patients may be managed using endocrine therapy alone.
Eventually most will escape hormonal control. In post menopausal women oestrogens are produced by the peripheral
aromatization of androgens and aromatase inhibitors are therefore the most popular agent in this age group.

41. A 38 year old lady has undergone a mastectomy and axillary node clearance for invasive ductal carcinoma. The histology
report shows a completely excised 3.5cm lesion which is grade 3. Two of the axillary lymph nodes contain metastatic
disease. The tumour is oestrogen receptor negative.

Cytotoxic chemotherapy. The combination of a grade 3 tumour and axillary nodal metastasis in a young female would attract a
recommendation for chemotherapy. Some may also add herceptin (if they are HER 2 positive).

Breast cancer treatment

 Endocrine therapy: Oestrogen receptor positive tumours. Downstaging primary lesions. Definitive treatment in old,
infirm patients
 Irradiation: Wide local excision. Large lesion, high grade or marked vascular invasion following mastectomy
 Chemotherapy: Downstaging advanced lesions to facilitate breast conserving surgery. Patients with grade 3 lesions or
axillary nodal disease

42. Which of the following statements regarding papillary carcinoma of the thyroid is false?
A. They account for the majority of thyroid carcinomas
B. Spread predominantly via the lymphatics
C. May be diagnosed using fine needle aspiration cytology
D. When viewed microscopically may demonstrate 'orphan Annie 'nuclei

447
E. Have a five year survival of 65% if confined to the thyroid alone
Answer: E
The prognosis for localised papillary carcinomas is excellent. Survival rates at 5 years approach 90%.

Theme: Breast malignancy

A. Eczema of the nipple


B. Ductal carcinoma in situ
C. Mucinous carcinoma
D. Invasive ductal carcinoma
E. Invasive lobular carcinoma
F. Paget's disease of the nipple
G. Lobular carcinoma in situ.

What is the likely diagnosis for the scenario given? Each option may be used once, more than once or not at all.

43. A 74 year old woman presents with a breast lump. On examination is has a soft consistency. The lump is removed and
sliced apart. Macroscopically there is a grey, gelatinous surface.

Mucinous carcinoma. Mucinous carcinomas comprise 2-3% of all breast cancers. They are one of the special type of
carcinomas. These have a better prognosis that tumours of Non Special Type (NST) and axillary nodal disease is rare in this
group.

44. A 74 year old woman presents with an erythematous rash originating in the nipple. It is spreading to the surrounding
areolar area and the associated normal tissue.

The correct answer is Paget's disease of the nipple. Paget's is associated with DCIS or invasive carcinoma.Unlike eczema of the
nipple which predominantly affects the areolar region, Pagets will usually affect the nipple first and then spread to the areolar
area. Diagnosis is made by punch biopsy.

45. 53 year old woman presents with a bloody nipple discharge. On mammography there is calcification behind the nipple
areolar complex. A core biopsy shows background benign change, but cells that show comedo necrosis which have not
breached the basement membrane.

Ductal carcinoma in situ. Comedo necrosis is a feature of high nuclear grade ductal carcinoma in situ. It is has a high risk of
being associated with foci of invasion.

Surgical Techniques
1. An orthopaedic surgeon makes a modification to an operative approach for total knee arthroplasty. After he has
completed 25 cases, he stops and reviews his patient outcomes. He publishes the data. What level of evidence is supplied
by this type of data?
A. II
B. IV
C. III
D. V
E. I
Answer: B
Case series that are non randomised and lack concurrent controls at best supply level IV evidence only. To qualify for level I and II
evidence a prospective randomised controlled trial with appropriate blinding, control matching and power calculations is needed.

Levels of evidence: The level of evidence refers to the study design used by investigators to minimise bias.
Level of Source
evidence
I Evidence obtained from systematic review of all relevant randomised controlled trials
II Evidence derived from at least one properly designed randomised controlled trial
III Evidence derived from well designed pseudo-randomised controlled trials (e.g. alternate allocation) or
historical controls
IV Evidence derived from case series or case reports
V Panel or expert opinion

448
Many of the categories contain sub groups, detailed knowledge of these are not required for MRCS Part A.
Theme: Use of suture materials and closure devices

A. Silk 3/0
B. Polyglactin 3/0
C. Polydioxanone 1/0
D. Stainless steel skin clips
E. Stainless steel wire 1/0
F. 6/0 Polypropylene
G. 3/0 Undyed polyglactin
H. Polypropylene 3/0

Please select the most appropriate suture material for the situation described. Each option may be used once, more than once or
not at all.

2. Mass closure of abdominal wall following elective right hemicolectomy through a midline incision.

Polydioxanone 1/0. PDS or polydioxanone is the ideal suture material. Non absorbable sutures have higher incidence of
incisional herniae.

3. Closure of the sternum following coronary artery bypass grafting.

Stainless steel wire 1/0. Stainless steel wire is typically used.

4. Application of vein patch to femoral artery following endarterectomy.

6/0 Polypropylene. Polypropylene is the suture of choice. Fine sutures are preferred.

Suture material
Agent Classification Durability Uses Special points
Silk Braided Theoretically Anchoring devices, skin closure Knots easily, poor cosmesis
Biological permanent although
strength not
preserved
Catgut Braided 5-7 days Short term wound approximation Poor cosmesis
Biological Degrades rapidly
Not available in UK
Chromic catgut Braided Up to 12 weeks Apposition of deeply sited tissues Unpredictable degradation
Biological pattern
Not in use in UK
Polydiaxonone Synthetic Up to 3 months Widespread surgical applications Used in most surgical
(PDS) Monofilament (longer with thicker including visceral anastomoses, specialties (avoid dyed form
sutures) dermal closure, mass closure of in dermal closure)
abdominal wall
Polyglycolic acid Braided Up to 6 weeks Most tissues can be apposed using It has good handling
(Vicryl, Dexon) Synthetic polyglycolic acid properties, the dyed form of
this suture should not be used
for skin closure
Polypropylene Synthetic Permanent Widely used, agent of choice for Poor handling properties
(Prolene) Monofilament vascular anastomoses
Polyester Synthetic Permanent Its combination of permanency It is more expensive and has
(Ethibond) Braided and braiding makes it useful for considerable tissue drag
laparoscopic surgery

Absorbable vs Non absorbable: Time taken to degrade absorbable materials varies. Usually by macrophages hydrolysing
material. Consider absorbable sutures in situations where long term tissue apposition is not required. In cardiac and vascular
surgery non absorbable sutures are usually used.

Suture size: The higher the index number the smaller the suture i.e. : 6/0 prolene is finer than 1/0 prolene. Finer sutures have less

449
tensile strength. For example 6/0 prolene would not be a suture suitable for abdominal mass closure but would be ideal for small
calibre distal arterial anastomoses.

Braided vs monofilament: Generally speaking braided sutures have better handling characteristics than non braided. However,
they are associated with higher bacterial counts. Braided materials are unsuitable for use in vascular surgery as they are potentially
thrombogenic.

5. Which of the following visceral anastomoses has the lowest risk of anastomotic leak? You may assume that all are
constructed in ideal circumstances.
A. Stapled ileocolic anastomosis
B. Hand sewn anastomosis of the proximal ileum
C. Stapled colorectal anastomosis defunctioned with loop ileostomy
D. Stapled colorectal anastomosis defunctioned with loop colostomy
E. Hand sewn oesophagojejunal anastomosis
Answer: B
Rectal and oesophageal surgery have some the highest risk of anastomotic leakage, rates following anterior resection are quoted to
be up to 10%. Small bowel anastomoses are the most technically forgiving. Factors increasing the risk of anastamotic leakage
include previous irradiation, sepsis, malnutrition, poor blood supply and poor technique.
The defunctioning of rectal anastomoses may reduce the clinical impact of anastomotic leak and make it amenable to percutaneous
drainage, but does not necessarily reduce the incidence of leaks themselves.

Anastomoses: A wide variety of anastomoses are constructed in surgical practice. Essentially the term refers to the restoration of
luminal continuity. As such they are a feature of both abdominal and vascular surgery.

Visceral anastomoses: For an anastomosis to heal three criteria need to be fulfilled: Adequate blood supply. Mucosal apposition.
Minimal tension

When these are compromise the anastomosis may dehisce (leak). Even in the best surgical hands some anastomoses are more
prone to dehiscence than others. Oesophageal and rectal anastomoses are more prone to leakage and reported leak rates following
oesophageal and rectal surgery can be as high as 20%. This figure includes radiological leaks and those with a clinically
significant leak will be of a lower order of magnitude. As a rule small bowel anastomoses heal most reliably.

The decision as to how best to achieve mucosal apposition is one for each surgeon. Some will prefer the use of stapling devices as
they are quicker to use, others will prefer to perform a sutured anastomosis. The attention to surgical technique is more important
than the method chosen and a poorly constructed stapled anastomosis in thickened tissue is far more prone to leakage than a hand
sewn anastomosis in the same circumstances.

If an anastomosis looks unsafe then it may be best not to construct one at all. In colonic surgery this is relatively clear cut and
most surgeons would bring out an end colostomy. In situations such as oesophageal surgery this is far more problematic and
colonic interposition may be required in this situation.

Vascular anastomoses: Most arterial surgery involving bypasses or aneurysm repairs will require construction of an arterial
anastomosis. Technique is important and for small diameter distal arterial surgery the intimal hyperplasia resulting from a badly
constructed anastomosis may render the whole operation futile before the patient leaves hospital.

Some key points about vascular anastomoses: Always use non absorbable monofilament suture (e.g. Polypropylene). Round
bodied needle. Correct size for anastamosis ( i.e. 6/0 prolene for bottom end of a femoro-distal bypass). Suture should be
continuous and from inside to outside of artery to avoid raising an intimal flap.

Theme: Management of skin wounds

A. Immediate split thickness skin graft


B. Delayed split thickness skin graft
C. Primary closure
D. Delayed primary closure
E. Compression bandages
F. Myocutaneous flap
G. Random free flap

For each of the following injury scenarios please select the most appropriate management. Each option may be used once, more
than once or not at all.

450
6. A 63 year old male is gardening when he trips and lands on a scythe. He sustains a deep laceration of his lateral thigh, it
measures 3cm depth by 7cm length, it penetrates down to the bone, but no fracture is evident on imaging or
examination. His co- morbidities include type II diabetes mellitus (diet controlled) and polymyalgia rheumatica (takes
regular low dose prednisolone).

Delayed primary closure.. Wounds which are contaminated or have the potential to become so are unsafe for immediate primary
closure. The combination of diabetes and steroids makes wound complications more likely. Despite his high risk a primary skin
graft or flap is unlikely to be a safer option. Either may be used at a later date in the event that delayed primary closure is
unsuccessful. Theme from September 2012 Exam

7. A 71 year old lady trips over and falls landing on her left skin. She sustains a large pretibial laceration of her leg.

The correct answer is Immediate split thickness skin graft. Pretibial lacerations do not heal well. Simple apposition of skin
edges almost always fails due to poor quality dermal tissues and underlying haematoma. Debridement a primary grafting
usually gives the best results.

8. A 73 year old lady presents with an ulcer overlying her medial malleolus. It is painless and has been present for 4
months. She has oedema of the lower limbs and her ABPI measures 0.9.

The correct answer is Compression bandages. This is likely to be a venous leg ulcer. These are typically managed using
compression bandages. Contra indications to this technique include peripheral vascular disease (not present here).

Methods of wound closure

Primary closure: Clean wound, usually surgically created or following minor trauma. Standard suturing methods will usually
suffice. Wound heals by primary intention

Delayed primary closure: Similar methods of actual closure to primary closure. May be used in situations where primary closure
is either not achievable or not advisable e.g. infection

Vacuum assisted closure: Uses negative pressure therapy to facilitate wound closure. Sponge is inserted into wound cavity and
then negative pressure applied. Advantages include removal of exudate and versatility. Disadvantages include cost and risk of
fistulation if used incorrectly on sites such as bowel

Split thickness skin grafts: Superficial dermis removed with Watson knife or dermatome (commonly from thigh). Remaining
epithelium regenerates from dermal appendages. Coverage may be increased by meshing

Full thickness skin grafts: Whole dermal thickness is removed. Sub dermal fat is then removed and graft placed over donor site.
Better cosmesis and flexibility at recipient site. Donor site "cost"

Flaps: Viable tissue with a blood supply. May be pedicled or free. Pedicled flaps are more reliable, but limited in range. Free flaps
have greater range but carry greater risk of breakdown as they require vascular anastomosis

Theme: Surgical energy devices

A. Monopolar diathermy
B. Bipolar diathermy
C. CUSA device
D. Argon plasma coagulation device
E. Ligasure device
F. Monopolar device in cutting mode
G. Monopolar device in coagulation mode
H. Monopolar device in blend mode

Please select the most appropriate surgical energy device for the procedure described. Each option may be used once, more than
once or not at all.

451
9. Posterior dissection of the thyroid gland during total thyroid lobectomy

The correct answer is Bipolar diathermy. This will minimise thermal trauma to the recurrent laryngeal nerve

10. Undertaking a snare polypectomy for a villous adenoma of the descending colon

The correct answer is Monopolar device in blend mode. Blend applies a mixture of coagulation and cutting modes to achieve
smooth polypectomy

11. Dissection of temporal lobe for tumour

The correct answer is CUSA device. The ultrasonic dissector is the preferred tool for this. It is also extensively used in liver
resections

Diathermy: Diathermy devices are used by surgeons in all branches of surgery. Use electric currents to produce local heat and
thereby facilitate haemostasis or surgical dissection. Consist of a generator unit that is located outside the patient and can be set to
the level of power required by the surgeon. There are two major types of diathermy machine;

Monopolar: The current flows through the diathermy unit into a handheld device that is controlled by the surgeon. Electricity can
flow from the tip of the device into the patient. The earth electrode is located some distance away. The relatively narrow tip of the
diathermy device produces local heat and this can be used to vaporise and fulgurate tissues. The current can be adjusted in terms
of frequency so that different actions can be effected. In cutting mode sufficient power is applied to the tissues to vaporise their
water content. In coagulation mode the power level is reduced so that a coagulum is formed instead. Some diathermy machines
can utilise a setting known as blend that alternates cutting and coagulation functions, these tend to be used during procedures such
as colonoscopic polypectomy.

Bipolar: The electric current flows from one electrode to another however, both electrodes are usually contained within the same
device e.g. a pair of forceps. The result is that heating is localised to the area between the two electrodes and surrounding tissue
damage is minimised.

Ultrasound based devices: These include CUSA and Harmonic scalpel. They generate high frequency oscillations that seal and
coagulate tissues. They have different energy settings that allow them to dissect and simultaneously seal vessels if required. The
CUSA device leaves vessels intact that may then be divided.

Ligasure device: Delivers tailored energy levels to allows simultaneous haemostasis and dissection. The device senses the
impedance of the tissues and tailors energy levels accordingly.

Hazards of diathermy: Inadvertent patient burn. This may result of careless handling of the device or in the case of monopolar
devices forgetting to apply a return electrode plate, In this situation patients may develop a contact burn when electricity flows to
earth. Explosion or fire. This may occur when volatile anaesthetic gases or skin preparation fluid have been used

Theme: Surgical drains

A. Redivac suction drain


B. Corrugated drain
C. Wallace Robinson drain (non suction)
D. Penrose tubing
E. Latex T Tube drain
F. Silastic T Tube drain
G. No drain

Please select the most appropriate surgical drainage system for the indication given. Each option may be used once, more than
once or not at all.

12. A 56 year old lady undergoes and open cholecystectomy and exploration of common bile duct. The bile duct is closed over
a drain.

Latex T Tube drain. This will elicit a fibrotic response and encourage a track to form.

452
13. A 48 year old lady undergoes a mastectomy and axillary node clearance for an invasive ductal cancer of the breast with
lymph node metastasis.

The correct answer is Redivac suction drain. The raw tissue exposed from the mastectomy site will often ooze serous fluid and
may result in seroma formation when the drain is removed.

14. A 75 year old man undergoes a Hartman's procedure for sigmoid diverticular disease with pericolic abscess and
colovesical fistula.

Wallace Robinson drain (non suction). A non suction drain is the preferred option here.

Surgical drains: Drains are inserted in many surgical procedures and are of many types. As a broad rule they can be divided into
those using suction and those which do not. The diameter of the drain will depend upon the substance being drained, for example
smaller lumen drain for pneumothoraces vs haemothorax. Drains can be associated with complications and these begin with
insertion when there may be iatrogenic damage. When in situ they serve as a route for infections. In some specific situations they
may cause other complications, for example suction drains left in contact with bowel for long periods may carry a risk of inducing
fistulation. Drains should be inserted for a defined purpose and removed once the need has passed.

A brief overview of types of drain and sites is given below

CNS: Low suction drain or free drainage systems may be used for situations such as drainage of sub dural haematomas.

CVS: Following cardiothoracic procedures of thoracic trauma underwater seal drains are often placed. These should be carefully
secured. When an air leak is present they may be placed on suction whilst the air leak settles

Orthopaedics and trauma: In this setting drains are usually used to prevent haematoma formation (with associated risk of
infection). Some orthopaedic drains may also be specially adapted to allow the drained blood to be auto transfused.

Gastro-intestinal surgery: Surgeons often place abdominal drains either to prevent or drain abscesses, or to turn an anticipated
complication into one that can be easily controlled such as a bile leak following cholecystectomy. The type of drain used will
depend upon the indication.

Redivac: Suction type of drain. Closed drainage system. High pressure vacuum system

Low pressure drainage systems: Consist of small systems such as the lantern style drain that may be used for short term
drainage of small wounds and cavities. Larger systems are sometimes used following abdominal surgery, they have a lower
pressure than the redivac system, which decreases the risks of fistulation. May be emptied and re-pressurised

Latex tube drains: May be shaped (e.g. T Tube) or straight. Usually used in non pressurised systems and act as sump drains.
Most often used when it is desirable to generate fibrosis along the drain trach (e.g. following exploration of the CBD)

Chest drains: May be large or small diameter (depending on the indication). Connected to underwater seal system to ensure one
way flow of air

Corrugated drain: Thin, wide sheet of plastic, usually soft. Contains corrugations, along which fluids can track

15. Which of the following local anaesthetics is not an amino amide type?
A. Lignocaine
B. Xylocaine
C. Procaine
D. Bupivicaine
E. Prilocaine
Answer: C
All local anaesthetics have a chemical bond linking an amine to either an amide or an ester. Most local anaesthetics are of the
amino- amide types, these have a more favorable side effect profile and are more stable in solution. Procaine and benzocaine have
amino - ester groups, these are metabolised by pseudocholinesterases.

Local anaesthetic agents

Lidocaine: An amide. Local anaesthetic and a less commonly used antiarrhythmic (affects Na channels in the axon). Hepatic
metabolism, protein bound, renally excreted. Toxicity: due to IV or excess administration. Increased risk if liver dysfunction or
453
low protein states. Note acidosis causes lidocaine to detach from protein binding. Drug interactions: Beta blockers, ciprofloxacin,
phenytoin. Features of toxicity: Initial CNS over activity then depression as lidocaine initially blocks inhibitory pathways then
blocks both inhibitory and activating pathways. Cardiac arrhythmias. Increased doses may be used when combined with
adrenaline to limit systemic absorption.

Cocaine: Pure cocaine is a salt, usually cocaine hydrochloride. It is supplied for local anaesthetic purposes as a paste. It is
supplied for clinical use in concentrations of 4 and 10%. It may be applied topically to the nasal mucosa. It has a rapid onset of
action and has the additional advantage of causing marked vasoconstriction. It is lipophillic and will readily cross the blood brain
barrier. Its systemic effects also include cardiac arrhythmias and tachcardia. Apart from its limited use in ENT surgery it is
otherwise used rarely in mainstream surgical practice.

Bupivicaine: Bupivacaine binds to the intracellular portion of sodium channels and blocks sodium influx into nerve cells, which
prevents depolarization. It has a much longer duration of action than lignocaine and this is of use in that it may be used for topical
wound infiltration at the conclusion of surgical procedures with long duration analgesic effect. It is cardiotoxic and is therefore
contra indicated in regional blockage in case the tourniquet fails. The co-administration of adrenaline concentrates it at the site of
action and allows the use of higher doses.

Prilocaine: Similar mechanism of action to other local anaesthetic agents. However, it is far less cardiotoxic and is therefore the
agent of choice for intravenous regional anaesthesia e.g. Biers Block.

All local anaesthetic agents dissociate in tissues and this contributes to their therapeutic effect. The dissociation constant shifts in
tissues that are acidic e.g. where an abscess is present and this reduce the efficacy.

Doses of local anaesthetics


Agent Dose plain Dose with adrenaline
Lignocaine 3mg/Kg 7mg/Kg
Bupivicane 2mg/Kg 2mg/Kg
Prilocaine 6mg/Kg 9mg/Kg
These are a guide only as actual doses depend on site of administration, tissue vascularity and co-morbidities.

Theme: Biological therapies

A. Bevacizumab
B. Infliximab
C. Trastuzumab
D. Basiliximab
E. Imatinib
F. Cetuximab

Please select the most appropriate biological agent for the situation described. Each option may be used once, more than once or
not at all.

16. A 32 year old lady has previously undergone a wide local excision and axillary node clearance (5 nodes positive) for an
invasive ductal carcinoma. It is oestrogen receptor negative, HER 2 positive, vascular invasion is present. She has a
lesion suspicious for metastatic disease in the left lobe of her liver.

The correct answer is Trastuzumab. This ladies young age, coupled with ER negativity and extensive nodal disease with
suspicion of metastatic disease makes her a candidate for treatment with trastuzumab (herceptin).

17. A 22 year old lady has severe peri anal crohns disease with multiple anal fistulae, the acute sepsis has been drained and
setons are in place. She is already receiving standard non biological therapy.

Infliximab. Infliximab is a popular choice in managing complex peri anal crohns. It is absolutely vital that all sepsis is drained
prior to starting therapy.

18.
19. A 63 year old man presents with a locally unresectable gastrointestinal stromal tumour. Biopsies confirm that it is KIT
positive.

The correct answer is Imatinib. Imatinib is licensed for treatment of GIST in the United Kingdom for this situation. The
guidance from the National Institute of Clinical evidence is that patients be reviewed at 12 weeks after initiating therapy.

454
Biological agents
Agents Target Uses
Adalimumab TNF alpha inhibitor Crohns disease
Infliximab Rheumatoid disease
Etanercept
Bevacizumab Anti VEGF (anti angiogenic) Colorectal cancer
Renal
Glioblastoma
Trastuzumab HER receptor Breast cancer
Imatinib Tyrosine kinase inhibitor Gastrointestinal stromal tumours
Chronic myeloid leukaemia
Basiliximab IL2 binding site Renal transplants
Cetuximab Epidermal growth factor inhibitor EGF positive colorectal cancers
Detailed understanding of the actions of biological agents is well beyond the scope of the MRCS syllabus. However, many of these
drugs are being frequently encountered in surgical patients.

20. Which of the following sutures has the largest diameter?


A. 6/0 Polypropylene
B. 5/0 Silk
C. 3/0 Nylon
D. 1 Polypropylene
E. 0 Polydiaxone
Answer: D
The sizes of suture material are not related to the composition of the suture material.

Suture sizes

USP Suture size and corresponding suture diameter


USP Size Diameter in mm
11-0 0.01
10-0 0.02
6-0 0.07
3-0 0.2
0 0.35
1 0.4
Theme: Management of bleeding

A. Ligate vessel
B. Underrun vessel
C. Use of diathermy
D. Application of surgicell
E. Digital pressure

In each of the following scenarios the surgeon has encountered bleeding. Please select the most appropriate immediate
management of the situation from the list below. Each option may be used once, more than once or not at all.

21. A 23 year old man is undergoing an open appendicectomy. The surgeons extend the incision medially and suddenly
encounter troublesome bleeding.

The correct answer is Ligate vessel. Theme from April 2012 Exam. Medial extension of an appendicectomy incision carries the
risk of injury to the inferior epigastric artery. This can bleed briskly and is best managed by ligation.

22. A 45 year old man is undergoing a laparotomy and following incision of the skin multiple bleeding points are identified
in the dermis and sub dermal tissues.

The correct answer is Use of diathermy. Multiple bleeding points are best managed through the use of diathermy.

23. A 38 year old lady is undergoing a laparotomy when the surgeons damage the common iliac vein whilst commencing a
pelvic dissection.

The correct answer is Digital pressure. Major venous bleeding such as this should be controlled with digital pressure in the first
instance. The definitive management will usually consist of suturing the defect closed with prolene sutures. Transection of the
common iliac vein will necessitate a major venous reconstruction.

455
Management of bleeding

Bleeding is a process that is encountered in all branches of surgery. The decision as to how best to manage bleeding depends upon
the site, vessel and circumstances.

Management of superficial dermal bleeding: This will usually cease spontaneously. If it is troublesome then direct use of
monopolar or bipolar cautery devices will usually control the situation. Scalp wounds are a notable exception and the bleeding
from these may be brisk. In this situation the use of mattress sutures as a wound closure method will usually address the problem.

Superficial arterial bleeding: If the vessel can be safely identified then the easiest method is to apply a haemostatic clip and
ligate the vessel.

Major arterial bleeding: If the vessel can be clearly identified and is accessible then it may be possible to apply a clip and ligate
the vessel. If the vessel is located in a pool of blood then blind application of haemostatic clips is highly dangerous and may result
in collateral injury. In this situation evacuating the clot and packing the area is often safer. The pack can then be carefully
removed when the required instruments are available. Some vessels may retract and bleeding may then be controlled by dissection
of surrounding structures or under-running the bleeding point.

Major venous bleeding: The safest initial course of action is to apply digital pressure to the bleeding point. To control the
bleeding the surgeon will need a working suction device. Divided veins may require ligation. Incomplete lacerations of major
veins (e.g. IVC) are best repaired. In order to do this it is safest to apply a Satinsky type vascular clamp and repair the defect with
5/0 prolene.

Bleeding from raw surfaces: This may be mixed bleeding and can be troublesome. Spray diathermy and argon plasma
coagulation are both useful agents. Certain topic haemostatic agents such as surgicell are useful in encouraging clot formation and
may be used in conjunction with, or instead of, the above agents.

24. A 7 year old boy is due to undergo a circumcision for phimosis. Which of the following devices would be the most
appropriate agent to use for achieving haemostasis?
A. Monopolar unit in cutting mode
B. Bipolar unit
C. Monopolar unit in coagulation mode
D. Monopolar unit in blend mode
E. Monopolar unit configured to spray mode
Answer: B

The danger with the use of any source other than bipolar diathermy in this setting is the risk of causing trauma to end vessels. All
the monopolar units, regardless of the setting will carry this risk.

Diathermy: Diathermy devices are used by surgeons in all branches of surgery. Use electric currents to produce local heat and
thereby facilitate haemostasis or surgical dissection. Consist of a generator unit that is located outside the patient and can be set to
the level of power required by the surgeon. There are two major types of diathermy machine;

Monopolar: The current flows through the diathermy unit into a handheld device that is controlled by the surgeon. Electricity can
flow from the tip of the device into the patient. The earth electrode is located some distance away. The relatively narrow tip of the
diathermy device produces local heat and this can be used to vaporise and fulgurate tissues. The current can be adjusted in terms
of frequency so that different actions can be effected. In cutting mode sufficient power is applied to the tissues to vaporise their
water content. In coagulation mode the power level is reduced so that a coagulum is formed instead. Some diathermy machines
can utilise a setting known as blend that alternates cutting and coagulation functions, these tend to be used during procedures such
as colonoscopic polypectomy.

Bipolar: The electric current flows from one electrode to another however, both electrodes are usually contained within the same
device e.g. a pair of forceps. The result is that heating is localised to the area between the two electrodes and surrounding tissue
damage is minimised.

Ultrasound based devices: These include CUSA and Harmonic scalpel. They generate high frequency oscillations that seal and
coagulate tissues. They have different energy settings that allow them to dissect and simultaneously seal vessels if required. The
CUSA device leaves vessels intact that may then be divided.

Ligasure device: Delivers tailored energy levels to allows simultaneous haemostasis and dissection. The device senses the
456
impedance of the tissues and tailors energy levels accordingly.

Hazards of diathermy:Inadvertent patient burn. This may result of careless handling of the device or in the case of monopolar
devices forgetting to apply a return electrode plate, In this situation patients may develop a contact burn when electricity flows to
earth. Explosion or fire. This may occur when volatile anaesthetic gases or skin preparation fluid have been used

25. If a 2 x 2 cm autologus skin graft is placed on an area of healthy granulation tissue, after about a week, a thin bluish -
white margin appears around the graft and spreads at a rate of 1mm per day. What is it?
A. Epidermis alone
B. Epidermis and dermis
C. Dermis alone
D. Inflammatory exudate
E. Fibrin
Answer: A
This is the process of re-epithelialisation.

Wound healing: Surgical wounds are either incisional or excisional and either clean, clean contaminated or dirty. Although the
stages of wound healing are broadly similar their contributions will vary according to the wound type. The main stages of wound
healing include:

Haemostasis: Vasospasm in adjacent vessels, platelet plug formation and generation of fibrin rich clot.

Inflammation: Neutrophils migrate into wound (function impaired in diabetes). Growth factors released, including basic
fibroblast growth factor and vascular endothelial growth factor. Fibroblasts replicate within the adjacent matrix and migrate into
wound. Macrophages and fibroblasts couple matrix regeneration and clot substitution.

Regeneration: Platelet derived growth factor and transformation growth factors stimulate fibroblasts and epithelial cells.
Fibroblasts produce a collagen network. Angiogenesis occurs and wound resembles granulation tissue.

Remodeling: Longest phase of the healing process and may last up to one year (or longer). During this phase fibroblasts become
differentiated (myofibroblasts) and these facilitate wound contraction. Collagen fibres are remodeled. Microvessels regress
leaving a pale scar.

The above description represents an idealised scenario. A number of diseases may distort this process. It is obvious that one of the
key events is the establishing well vascularised tissue. At a local level angiogenesis occurs, but if arterial inflow and venous return
are compromised then healing may be impaired, or simply nor occur at all. The results of vascular compromise are all too
evidence in those with peripheral vascular disease or those poorly constructed bowel anastomoses.

Conditions such as jaundice will impair fibroblast synthetic function and overall immunity with a detrimental effect in most parts
of healing.

Problems with scars: Hypertrophic scars: Excessive amounts of collagen within a scar. Nodules may be present histologically
containing randomly arranged fibrils within and parallel fibres on the surface. The tissue itself is confined to the extent of the
wound itself and is usually the result of a full thickness dermal injury. They may go on to develop contractures
Keloid scars: Excessive amounts of collagen within a scar. Typically a keloid scar will pass beyond the boundaries of the original
injury. They do not contain nodules and may occur following even trivial injury. They do not regress over time and may recur
following removal.

Drugs which impair wound healing: Non steroidal anti inflammatory drugs. Steroids. Immunosupressive agents. Anti neoplastic
drugs

Closure: Delayed primary closure is the anatomically precise closure that is delayed for a few days but before granulation tissue
becomes macroscopically evident. Secondary closure refers to either spontaneous closure or to surgical closure after granulation
tissue has formed.

26. Which of the following methods is most effective at destroying spores of the tubercle bacilli?
A. Immersion in 0.5% chlorhexidine in alcohol
B. Immersion in aqueous iodine
C. Heating in a hot air oven
D. Immersion in 0.1% sodium hypochlorite
E. Autoclaving
Answer: E
457
The tubercle bacilli has a waxy outer membrane that renders it more resistant to sterilisation and cleaning methods. Whilst 0.1%
sodium hypochlorite will destroy may microbes it is less reliable in destroying tubercle bacilli. Hot air ovens provide less reliable
pathogen destruction than autoclaving, but may be indicated in situations where the equipment is sensitive to the autoclaving
process. From the list of options above, autoclaving will most reliably destroy tubercle bacilli.

Sterilisation: Cleaning refers to removal of physical debris. Disinfection refers to reduction in numbers of viable organisms.
Sterilisation is removal of all organisms and spores.

The method chosed depends upon the type of instrument and the procedure for which it will be used. Sterilisation of surgical
instruments typically takes place in an autoclave which uses pressurised steam at a temperature of 134 degrees. This method is
reproducible and safe. However, endoscopy equipment cannot be sterilised by this method as it would damage it. Therefore they
are sterilised using 2% glutaraldehyde solution. Since staff may develop hypersensitivity its use is restricted to those pieces of
equipment that cannot be sterilised by an alternative means. In the industrial setting gamma irradiation is used.

27. Which of the following is a permanent suture material best suited for interrupted mattress dermal closure?
A. 2/0 Polydiaxone
B. 3/0 Polydiaxone
C. 4/0 Polyglycolic acid
D. 1/0 Dexon
E. 3/0 Polypropylene
Answer: E
Of the sutures listed only prolene is a permanent suture material. It is a good agent for skin closure as it does not incite an
inflammatory response and thus provides good cosmesis.

Theme: Surgical site infections

A. Glutaraldehyde 2% applied to the skin


B. Sodium hypochlorite solution applied to the skin
C. Aqueous iodine applied to the skin
D. Perform surgery in a lamninar flow theatre
E. Surgeon to wear exhaust suit
F. Administration of clindamycin
G. Administration of gentamicin
H. Pre operative shaving

Please select the most appropriate modality to reduce the risk of developing a surgical site infection for the scenario given. Each
option may be used once, more than once or not at all.

28. A 42 year old man is due to undergo a Mayo repair of a paraumbilical hernia. He is otherwise well.

The correct answer is Aqueous iodine applied to the skin. The patient will require skin preparation. However, use of
glutaraldehyde or sodium hypochlorite would be an inappropriate choice. As the Mayo repair does not involve implantation of
prosthetic mesh the use of antibiotics is not appropriate.

29. A 63 year old man with end stage oestoarthritis of the hip is due to undergo a total hip replacement. The skin has been
prepared and antibiotics given.

The correct answer is Perform surgery in a lamninar flow theatre. Laminar flow is more important than an exhaust suit although
use of both is ideal.

30. A 22 year old man is undergoing an appendicectomy. At operation there is copious pus around the appendix.

Administration of gentamicin. Gentamicin is the preferred agent. Clindamycin is associated with high rate of clostridium dificile
infection.

Surgical site infection: Surgical site infections may occur following a breach in tissue surfaces and allow normal commensals and
other pathogens to initiate infection. They are a major cause of morbidity and mortality. Surgical site infections (SSI) comprise up
to 20% of all healthcare associated infections and at least 5% of patients undergoing surgery will develop an SSI as a result. In
many cases the organisms are derived from the patient's own body. Measures that may increase the risk of SSI include: Shaving
458
the wound using a razor (disposable clipper preferred). Using a non iodine impregnated incise drape if one is deemed to be
necessary. Tissue hypoxia. Delayed administration of prophylactic antibiotics in tourniquet surgery

Preoperatively: Don't remove body hair routinely. If hair needs removal, use electrical clippers with single use head (razors
increase infection risk). Antibiotic prophylaxis if: placement of prosthesis or valve. Clean-contaminated surgery. Contaminated
surgery. Use local formulary. Aim to give single dose IV antibiotic on anaesthesia. If a tourniquet is to be used, give prophylactic
antibiotics earlier

Intraoperatively: Prepare the skin with alcoholic chlorhexidine (Lowest incidence of SSI). Cover surgical site with dressing. A
recent meta analysis has confirmed that administration of supplementary oxygen does not reduce the risk of wound infection. In
contrast to previous individual RCT's(1)

Post operatively: Tissue viability advice for management of surgical wounds healing by secondary intention

Use of diathermy for skin incisions: In the NICE guidelines the use of diathermy for skin incisions is not advocated(2). Several
randomised controlled trials have been undertaken and demonstrated no increase in risk of SSI when diathermy is used(3).

Theme: Suture materials

A. Silk 3/0
B. Polyglactin 3/0
C. Polydiaxone 1/0
D. Stainless steel skin clips
E. Stainless steel wire 1/0
F. 6/0 Polypropylene
G. 3/0 Undyed polyglactin
H. Polypropylene 3/0

Please select the most appropriate suture for the scenario given. Each option may be used once, more than once or not at all.

Anastomosis of ileum to transverse colon following right hemicolectomy.

Polyglactin 3/0. 3/0 PDS would be an alternative, as would linear stapler but those are not in the list.

Distal anastomosis in a femorodistal bypass using vein.

6/0 Polypropylene. Arterial anastomoses should be constructed using polypropylene. In this case a fine suture material such as 6/0
is indicated.

Closure of skin following thyroidectomy for Graves disease.

The correct answer is Stainless steel skin clips. Although some use sub cuticular stitches skin clips remain the standard of many.
In the event of post operative haematoma causing respiratory obstruction, they are easier to remove.

Theme: Management of skin lesions

A. 5mm punch biopsy


B. Shave biopsy
C. Excisional biopsy
D. Wide excision of 5cm
E. Tru cut biopsy
F. Incisional biopsy

For the skin lesions described please select the most appropriate management option. Each option may be used once, more than
once or not at all.

31. An 83 year old lady presents with multiple patches of pigmented irregular, superficial lesions over the torso. They do not
bleed but have become increasingly itchy.

459
The correct answer is Shave biopsy.This is most likely to be seborrhoeic warts. These are usually superficially sited and are best
managed with shave biopsy and cautery. Theme from April 2012 Exam

32. A 65 year old man presents with a 5cm ulcerated area over his medial malleolus.

The correct answer is 5mm punch biopsy. This is likely to be a venous ulcer and should usually be managed with compression
bandaging if there is no arterial compromise. Long standing lesions may be complicated by the development of malignancy and
for this reason a punch biopsy of long standing or non healing lesions is advisable.

33. A 23 year old lady presents with an itchy, bleeding pigmented lesion on her right thigh.

Excisional biopsy. This may represent a malignant melanoma. Complete excision is required to allow accurate histological
assessment. If the diagnosis is confirmed then re-excision of margins may be required. Clearly if the lesion is benign then no
further action is required.

Treatment of suspicious skin lesions: Skin lesions may be referred to surgeons for treatment or discovered incidentally. The
table below outlines the various therapeutic options:
 Tru-cut biopsy:Most often used for percutaneous sampling of deep seated lesions or used intra operatively for visceral
lesions
 5mm punch biopsy:Used for diagnostic confirmation of lesions that are suspected to be benign or where the definitive
management is unlikely to be surgical. Of limited usefulness in pigmented lesions where they do not include sufficient tissue
for accurate diagnosis. May be used in non melanoma type skin disease to establish diagnosis prior to more extensive
resection.
 Wide excision:Where the complete excision of the lesion (with healthy margins) is the main objective. In cosmetically
sensitive sites, or where the defect is large, this may need to be complemented with plastic surgical techniques
 Incisional biopsy:Used mainly for deep seated or extensive lesions where there is diagnostic doubt (usually following core
or tru-cut biopsy). Used rarely for skin lesions.
 Diagnostic excision:Primarily used for lesions that are suspicious for melanoma, the lesion is excised with a rim of normal
tissue. Excision of margins may be required subsequently.

Theme: Tissue sampling

A. Fine needle aspiration cytology


B. Surgical excision biopsy
C. Smear cytology
D. Core cut biopsy
E. Conventional surgical excision
F. Tru cut biopsy
G. Punch biopsy

Please select the most appropriate sampling method for the situation given. Each option may be used once, more than once or not
at all.

34. A 45 year old patient undergoes a CT scan of the abdomen and is noted to have a 6cm mass in the right adrenal gland.
Urinary catecholamines and other endocrine investigations are negative. CT of the chest and remainder of the abdomen
is otherwise normal.

The correct answer is Conventional surgical excision. Most surgeons would excise a mass of this size rather than attempt biopsy.
Further information relating to adrenal masses is covered under this topic.

35. A 67 year old lady is suspected of having Pagets disease of the nipple

The correct answer is Punch biopsy. This is a relatively clear indication for a punch biopsy. If cellular atypia is present on punch
biopsy then any in situ malignancy should be considered. FNAC would be unsuitable.

36. A 23 year old lady presents with a nodule in the right lobe of the thyroid. Examination of the neck is otherwise
unremarkable and clinically she is euthyroid. Imaging shows a solid nodule at the site.

460
The correct answer is Fine needle aspiration cytology. FNAC is the first line investigation in this setting.Where as FNAC has
declined in popularity recently, it remain a very popular option in the investigation of thyroid masses. It cannot reliably diagnose
a follicular tumour.

Tissue sampling: Tissue sampling is an important surgical process. Biopsy modalities vary according to the site, experience and
subsequent planned therapeutic outcome

The modalities comprise: Fine needle aspiration cytology. Core biopsy. Excision biopsy. Tru cut biopsy. Punch biopsy.
Cytological smears. Endoscopic or laparoscopic biopsy

When the lesion is superficial the decision needs to be taken as to whether complete excision is desirable or whether excision
biopsy is acceptable. In malignant melanoma for example the need for safe margins will mean that a more radical surgical
approach needs to be adopted after diagnostic confirmation from excision biopsy than would be the case in basal cell carcinoma.
Punch biopsies are useful in gaining histological diagnosis of unclear skin lesions where excision biopsy is undesirable such as in
establishing whether a skin lesion is vasculitic or not.

Fine needle aspiration cytology (FNAC) is an operator dependent procedure that may or may not be image guided and essentially
involves passing a needle through a lesion whilst suction is applied to a syringe. The material thus obtained is expressed onto a
slide and sent for cytological assessment. This test can be limited by operator inexperience and also by the lack of histological
architectural information (e.g. Follicular carcinoma of the thyroid). Where a discharge is present a sample may be sent for
cytology although in some sites (e.g. Nipple discharge ) the information gleaned may be meaningless.

Tissue samples may be obtained by both core and tru cut biopsy. A core biopsy is obtained by use of a spring loaded gun with a
needle passing quickly through the lesion of interest. A tru cut biopsy achieves the same objective but the needle moved by hand.
When performing these techniques image guidance may be desirable (e.g. In breast lesions). Consideration needs to be given to
any planned surgical resection as it may be necessary to resect the biopsy tract along with the specimen (e.g. In sarcoma surgery).

Visceral lesions may be accessed percutaneously under image guidance such as ultrasound guided biopsy of liver metastases. Or
under direct vision such as a colonoscopic biopsy.
Theme: Management of wounds

A. Split thickness skin graft


B. Full thickness skin graft
C. Insertion of tissue expander at donor site and delayed split thickness skin graft
D. Myocutaneous flap reconstruction (pedicled)
E. Direct primary closure
F. Delayed primary closure

Please select the most appropriate management for the wound described. Each option may be used once, more than once or not at
all.

37. A 34 year old man has a tissue defect measuring 3 cm by 1 cm following an excision of a lipoma from the scapula.

The correct answer is Direct primary closure>. This wound should be amenable to primary closure. There is minimal associated
tissue loss and the surgery is minor and uncontaminated.

38. A 72 year old lady has a 4cm basal cell carcinoma excised from her right cheek. There is a rhomboid defect measuring
4cm by 4cm.

The correct answer is Full thickness skin graft. Facial wounds that are large and irregularly shaped are best managed with full
thickness skin grafts.

39. A 5 year old suffers 20% burns to the torso. On examination there is fixed pigmentation and the affected area has a
white and dry appearance.

Split thickness skin graft. This is a full thickness burn and will require split thickness skin grafting. Meshing the graft may
increase the donor site yield. However, this is at the expense of cosmesis.

Tissue reconstruction: Skin Grafts and Flaps

Skin flaps or grafts may be required where primary wound closure cannot be achieved or would entail either significant cosmetic
461
defect or considerable functional disturbance as a result of wound contraction.

Reconstructive ladder
Method Types
Direct closure The simplest option where possible
Grafting techniques Split thickness. Full thickness. Skin Substitute. Composite
Flap technique Local: ransposition. Pivot. Alphabetplasty (e.g. Z-Y)
Regional: Myocutaneous. Fasciocutaneous. Neurocutaneous
Distant: Free tissue transfer
Prelamination techniques Allows creation of specialised flaps e.g. buccal mucosa
Tissue expansion Involves placement of tissue expanders to increase amount of tissue at donor
sites

Skin Grafts Vs. Flaps


Skin Grafts Flaps
No size limit (Split)/ Relative size limit (full thickness) Size limited by territory of blood supply
Rely on wound bed for blood supply Tissue has its own blood supply
Take better on clean well vascularised wound beds Will survive independent of the wound bed
Split skin graft donor site typically heals in 12 days Direct closure of donor site or secondary skin graft
Donor site may be reused Donor site cannot be reused

Split thickness skin grafts: Available in range of thicknesses. Thigh is the commonest donor site. Size may be increased by
meshing the graft. However this comes with compromise on cosmesis. Donor sites, especially if thin grafts are taken can be
reused following re-epithelialisation

Full thickness grafts: Most commonly used for facial reconstruction. Include dermal appendages. Provide superior cosmetic
result

Composite grafts: These are grafts containing more than one tissue type, such as skin and fat. They are usually used to cover
small defects in cosmetically important areas.

Flaps: Flaps have their own blood supply and may be pedicled or free. May have multiple components e.g. skin, skin + fat, skin +
fat + muscle. They will have the ability to take regardless of the underlying tissue bed. The type of intrinsic blood supply is
important. For example in breast surgery pedicled latissimus dorsi flaps will be less prone to failure than microsvascular
anastomosed free Diep flaps.

40. You have just completed a laparotomy for peritonitis due to a perforated peptic ulcer. What is the best surgical strategy
for avoidance of a complete abdominal wound dehisence?
A. Use of skin clips to close the skin rather than sub cuticular sutures
B. Careful approximation of the peritonum with non absorbable sutures
C. Mass closure of the midline wound using a 1/0 polydiaxone suture
D. Direct apposition of the rectus muscle rather than linea alba aponeurosis
E. Mass closure of the midline wound using a 3/0 polypropylene suture
Answer: C
The incidence of post operative wound dehisence is minimise by following Jenkins rule which advocates mass closure of the
midline wound. However, the suture strength is an important consideration and 3/0 sutures do not have sufficient tensile strength.
Both polydiaxone (PDS) and polypropylene (Prolene) or nylon (Ethilon) are all equally suitable. Although separate closure of the
peritoneum was practised it has no bearing on the incidence of abdominal wound dehisence.

Abdominal wound dehiscence: This is a significant problem facing all surgeons who undertake abdominal surgery on a regular
basis. Traditionally it is said to occur when all layers of an abdominal mass closure fail and the viscera protrude externally
(associated with 30% mortality). It can be subdivided into superficial, in which the skin wound alone fails and complete,
implying failure of all layers.

Factors which increase the risk are: * Malnutrition* Vitamin deficiencies * Jaundice * Steroid use * Major wound contamination
(e.g. faecal peritonitis)* Poor surgical technique (Mass closure technique is the preferred method-Jenkins Rule)

When sudden full dehiscence occurs the management is as follows:* Analgesia* Intravenous fluids* Intravenous broad spectrum
antibiotics* Coverage of the wound with saline impregnated gauze (on the ward) * Arrangements made for a return to theatre

Surgical strategy: Correct the underlying cause (eg TPN or NG feed if malnourished). Determine the most appropriate strategy
for managing the wound

Options
462
Resuturing of the This may be an option if the wound edges are healthy and there is enough tissue for sufficient coverage.
wound Deep tension sutures are traditionally used for this purpose.
Application of a This is a clear dressing with removable front. Particularly suitable when some granulation tissue is
wound manager present over the viscera or where there is a high output bowel fistula present in the dehisced wound.
Application of a This is a clear plastic bag that is cut and sutured to the wound edges and is only a temporary measure to
'Bogota bag' be adopted when the wound cannot be closed and will necessitate a return to theatre for definitive
management.
Application of a These can be safely used BUT ONLY if the correct layer is interposed between the suction device and the
VAC dressing bowel. Failure to adhere to this absolute rule will almost invariably result in the development of multiple
system bowel fistulae and create an extremely difficult management problem.

Theme: Instrument cleaning

A. Immersion in glutaraldehyde
B. Gamma irradiation
C. Autoclaving
D. Ethylene chloride
E. Phenolic lavage
F. Disposal of instrument

Please select the most appropriate cleaning method of instrument for the situation described. Each option may be used once, more
than once or not at all.

41. A company wishes to sterilise scalpel blades for use.

The correct answer is Gamma irradiation. Industry often uses gamma irradiation. It is not routinely used in hospitals

42. For sterilisation and cleaning of a colonoscope.

Immersion in glutaraldehyde. Washing systems using glutaraldehyde are often used although development of sensitivity in staff
is well known and it is used in closed systems

43. For cleaning instruments following a tonsillectomy in a patient who recieved human growth hormone extract in 1981

The correct answer is Disposal of instrument. High risk of prion disease mandates disposal on instruments which is often
undertaken following all tonsillectomy procedures regardless of level of percieved risk

Sterilisation: Cleaning refers to removal of physical debris. Disinfection refers to reduction in numbers of viable organisms.
Sterilisation is removal of all organisms and spores.

The method chosed depends upon the type of instrument and the procedure for which it will be used. Sterilisation of surgical
instruments typically takes place in an autoclave which uses pressurised steam at a temperature of 134 degrees. This method is
reproducible and safe. However, endoscopy equipment cannot be sterilised by this method as it would damage it. Therefore they
are sterilised using 2% glutaraldehyde solution. Since staff may develop hypersensitivity its use is restricted to those pieces of
equipment that cannot be sterilised by an alternative means. In the industrial setting gamma irradiation is used.

44. Which of the following is not an absorbable suture material?


A. Chromic catgut
B. Nylon
C. Vicryl
D. Dexon
E. Poly diaxone (PDS).
Answer: B
45. A 43 year old man has symptoms of carcinoid syndrome. Which of the following is the most effective therapeutic agent in
controlling the symptoms?
A. Atenolol
B. Octreotide
C. Glucagon
D. Somatostatin
E. Spironolactone
463
1. Answer: B
Octreotide is the usual treatment for carcinoid syndrome. Somatostatin inhibits the release of a number of gut hormones. Octreotide
is the synthetic alternative to somatostatin and thus the most appropriate therapeutic agent. Theme from April 2012 Exam

Carcinoid syndrome: Carcinoid tumours secrete serotonin. Originate in neuroendocrine cells mainly in the intestine (midgut-
distal ileum/appendix). Can occur in the rectum, bronchi. Hormonal symptoms mainly occur when disease spreads outside the
bowel

Clinical features: Onset: years. Flushing face. Palpitations. Tricuspid stenosis causing dyspnea. Asthma. Severe diarrhoea
(secretory, persists despite fasting)

Investigation: 5-HIAA in a 24-hour urine collection. Scintigraphy. CT scan

Treatment: Octreotide. Surgical removal

46. A 43 year old lady is due to undergo a diagnostic laparoscopy. Which of the agents listed below should be used for
inducing pneumoperitoneum?
A. Argon
B. Helium
C. Air
D. Carbon dioxide
E. Nitrogen
Answer: D
Carbon dioxide is the agent of choice. It is rapidly re-absorbed, does not support combustion and is cheap. It is rapidly cleared from
the lungs and so effects on pH are unusual.

Gases for laparoscopic surgery: Laparoscopic surgery may be performed in a number of body cavities. In some areas irrigation
solutions are preferred. In the abdomen insufflation with carbon dioxide gas is commonly used. The amount of gas delivered is
adjusted to maintain a constant intra-abdominal pressure of between 12 and 15 mmHg. Excessive intra-abdominal pressure may
reduce venous return and lead to hypotension. Too little insufflation will risk obscuring the surgical view

47. A surgeon wishes to determine whether different methods of perioperative shaving have an effect on post operative
wound infection rates. Which of the following is the best method for assessing whether one method is better than the
other?
A. Cohort study
B. Retrospective study
C. Case controlled study
D. Cross over study
E. Randomised controlled study
Answer: E
A randomised controlled study is the best method for assessing this relationship. It is important to analyse data from RCT's on
an intention to treat basis.

Randomised controlled trials: Randomised controlled trials are an established method of comparing two variables. These may
consist of comparison of treatments or treatment versus placebo. Ideally the trials should be blinded, usually to the patient and
those treating them.
In most cases a power calculation should be performed to determine the sample size required to detect a difference.

Theme: Electrosurgery

A. Cutting current
B. Coagulation current
C. Blended current
D. Fulguration
E. Desiccation

For each of the following electrosurgical applications please select the most likely modality used. Each option may be used once,
more than once or not at all.

464
48. In this modality the active electrode is placed in direct contact with the tissue and is characterised by low current and
high voltage over a broad area.

The correct answer is Desiccation. In desiccation the device is placed in direct contact with the tissues (unlike fulguration).
Because it is applied over a broad area it tends not to cause protein damage (unlike coagulation).

49. An electrosurgical mode whereby the electrode is held away from the tissue. The current utilises a low amplitude and
high voltage.

Fulguration. Fulguration typically avoids contact between the electrode and the tissue with the current configured to favor arc
formation.

50. A modality in which a sinusoidal, non modulated waveform is produced and vaporises the tissues.

Cutting current. The high energy levels result in tissue vaporisation and cleavage of tissues.

Electrosurgery utilises the heat generated by the passage of high frequency alternating electrical current through living tissues.
The application of a voltage across human tissue results in the formation of an electrical circuit between the voltage source and the
tissue. The tissue acts as a resistor and the level of resistance is determined by the water content of the tissue. It is this resistance
that results in the formation of heat.

An alternating current constantly changes the direction in which the current flows, the speed with which this occurs is measured in
Hertz. Most diathermy units operate at a frequency of between 200,000 kHZ to 5MHz. This means that tissue such as nerves and
muscles will not depolarise (since this seldom occurs at frequencies above 10,000Hz). The current waveform can be adjusted to
deliver three main therapeutic modalities; cutting, coagulation and blend.

Types of current: Cutting: Sinusoidal and non modulated waveform. High average power and current density. Precise cutting
without thermal damage

Coagulation: Modulated current with intermittent dampened sine waves of high peak voltage. Evaporation, rather than
vaporisation of intracellular fluid occurs. Results in formation of coagulum

Desication: Active electrode in direct contact with tissue. Low current and high voltage system. Results in loss of cellular water
but no protein damage

Fulguration: Electrode probe is held away from tissue. Produces spray effect with local, superficial tissue destruction. Low
amplitude and high voltage system

Blend: Alternating cutting and coagulation modes

 Total average power is less than with cutting

Theme: Suture materials

A. Silk 3/0
B. Polyglactin 3/0
C. Polydioxanone 1/0
D. Stainless steel skin clips
E. Stainless steel wire 1/0
F. 6/0 Polypropylene
G. 3/0 Undyed polyglactin
H. 3/0 Polypropylene

Please select the most appropriate suture for the situation described. Each option may be used once, more than once or not at all.

51. Anchoring a RediVac drain to the skin following a mastectomy.

Silk 3/0 ‫ز‬Silk is traditionally used for this purpose because of its reliable knotting.

465
52. A surgeon wishes to closure the linea alba of the abdominal wall following a laparotomy

Polydioxanone 1/0. A large suture such as 1/0 PDS or 1/0 polypropylene is the standard material for this indication. From the
list 1/0 PDS is the most appropriate.

53. Anastomosis of Dacron graft to proximal abdominal aorta during abdominal aortic aneurysm repair.

3/0 Polypropylene. 3/0 polypropylene is the suture of choice in this setting. 6/0 is too fine and will not withstand the tensile
forces.

54. What is the mechanism of action of ciprofloxacin?


A. Inhibition of DNA gyrase
B. Direct injury to the bacterial cell wall
C. Osmotic damage to the cell
D. Inhibition of reverse transcriptase
E. Destruction of bacterial aquaporin proteins
Answer: A

Antibiotics: mechanism of action: The lists below summarise the site of action of the commonly used antibiotics

Inhibit cell wall formation: penicillins. Cephalosporins. Inhibit protein synthesis: aminoglycosides (cause misreading of mRNA).
Chloramphenicol. macrolides (e.g. erythromycin). Tetracyclines. fusidic acid. Inhibit DNA synthesis: quinolones (e.g.
ciprofloxacin). Metronidazole. Sulphonamides. Trimethoprim. Inhibit RNA synthesis: rifampicin

55. A 73 year old lady with gallstones is about the undergo a laparoscopic cholecystectomy. The surgeon inserts a Verress
needle and performs a successful drop test prior to establishing a pneumoperitoneum. A 5 minute delay ensues before a
10mm infraumbilical trocar is inserted. The surgeon performs a diagnostic laparoscopy which shows a thickened
gallbladder but is otherwise normal. The anaesthetist complains that the patient has become hypotensive with a blood
pressure of 80/40 mmHg. Of the options below, which is the most appropriate course of action?
A. Release of pneumoperitoneum
B. Perform a laparotomy
C. Administration of intravenous adrenaline
D. Administration of intravenous amiodarone
E. End the operation
Answer: A
Excessive intra-abdominal pressure may cause decreased venous return and hypotension. Since the preliminary laparoscopy did
not show any major vascular catastrophe an emergency laparotomy would not be indicated. In most cases the release of pressure
is often sufficient. In cases of a vaso-vagal episode (which may be induced by peritoneal stretching) a dose of atropine may be
required.
Pneumoperitoneum- therapeutic: During a laparoscopic procedure a surgeon will need to create a pneumoperitoneum. This
can be achieved by use of a Verress needle (risk of visceral injury). An alternative is the open "Hassan" style technique. Once
access to the abdominal cavity is secured carbon dioxide gas is insufflated to induce a working space. Higher intra-abdominal
pressures may compromise venous return and reduce cardiac output. If the blood pressure is seen to drop in this way then release
of air, will often improve matters. Should this not be the case then a laparotomy may be necessary to exclude a more significant
internal injury.

56. Which of the following is least likely to reduce the risk of post operative wound infection?
A. Electrical clippers to remove body hair
B. Use of poviodone impregnated drapes
C. Antibiotic prophylaxis for prosthesis placement
D. Routine use of mechanical bowel preparation
E. Chlorhexidine to prepare the skin
Answer: D
The routine use of mechanical bowel preparation is not recommended. There is some recent evidence to support the use of
selective gut decontamination. However, this is not in mainstream practice at present.

57. A 67 year old women is undergoing a femoral hernia repair and the surgeon is using a bipolar diathermy unit for
haemostasis. Which of the following is a recognised risk with the use of bipolar diathermy?
A. Patient burns at the site of the contact plate
B. Fires when used near alcoholic skin preparations that have pooled
C. Coupling injuries
466
D. Risk of thermal injury to regional vessels as a result of tissue heating
E. Capacitance injuries
Answer: B
In bipolar units the flow of electricity is from one electrode to the other over a small area. As a result a contact plate is not used
and coupling and capacitance injuries are uncommon. They have a low risk of thermal injury to adjacent structures and are
preferred for this reason. However, they may cause sparks and ignite inflammable solutions.

58. A 34 year old lady is due to undergo a laparoscopic cholecystectomy. Which of the following intrabdominal pressures
should typically be set on the gas insufflation system?
A. 4mm Hg
B. 10mm Hg
C. 20mm Hg
D. 40mm Hg
E. 60mm Hg
Answer: B
Pressures lower than 7mm Hg are not usually compatible with satisfactory views. Pressures >15mm Hg are usually associated
with decreased venous return and hypotension.

59. A 53 year old man undergoes an elective right hemicolectomy. A stapled ileo-colic anastomosis is constructed. Eight
hours later he becomes tachycardic and passes approximately 600ml of dark red blood per rectum. Which of the
following processes is the most likely explanation for this occurrence?
A. Anastomotic leak
B. Discharging mesenteric haematoma
C. Bleeding peptic ulcer
D. Anastomotic staple line bleeding
E. Mesenteric infarct
Answer: D
Safe visceral anastamosis requires: Mucosal to mucosal apposition. Adequate vascularity. Minimal tension

Stapled anastomoses are associated with staple line bleeding and this may typically occur in the early post operative phase. They
should be managed conservatively as most will settle. Stapled anastomoses are quicker to perform. Ironically, although they may
appear easy they can carry considerably more potential pitfalls than their hand sewn equivalent and should be used with caution
by the inexperienced, this is especially true if the bowel is very thick walled.

Pediatric Surgery
1. A 2-day-old baby girl is noted to become cyanotic whilst feeding and crying. A diagnosis of congenital heart disease is
suspected. What is the most likely cause?
A. Transposition of the great arteries
B. Coarctation of the aorta
C. Patent ductus arteriosus
D. Tetralogy of Fallot
E. Ventricular septal defect
Answer: A
Congenital heart disease: cyanotic: TGA most common at birth, Fallot's most common overall. Acyanotic: VSD most common
cause. For Surgeons it is important to be aware of common congenital cardiac abnormalities. The main differentiating factor is
whether the patient is cyanotic or acyanotic. The key point to this question is that whilst tetralogy of Fallot is more common than
transposition of the great arteries (TGA), Fallot's doesn't usually present until 1-2 months following the identification of a murmur
or cyanosis. In the neonate, TGA is the most common presenting cause of cyanotic congenital heart disease. The other 3 options
are causes of acyanotic congenital heart disease Theme from April 2011 exam.

Congenital heart disease: Acyanotic - most common causes: Ventricular septal defects (VSD) - most common, accounts for
30%. Atrial septal defect (ASD). Patent ductus arteriosus (PDA). Coarctation of the aorta. Aortic valve stenosis. VSDs are more
common than ASDs. However, in adult patients ASDs are the more common new diagnosis as they generally presents later

Cyanotic - most common causes: Tetralogy of Fallot. Transposition of the great arteries (TGA). Tricuspid atresia. Pulmonary
valve stenosis. Fallot's is more common than TGA. However, at birth TGA is the more common lesion as patients with Fallot's
generally presenting at around 1-2 months. S

467
2. A 6 month old boy is brought to the clinic by his mother. She is concerned that his testes are not located into the scrotum.
She has noticed them only when he is in the bath, but not at any other time. What is the most likely underlying
diagnosis?
A. Rectractile testis
B. Ectopic testis
C. Undescended testis
D. Testicular agenesis
E. Intersex child
Answer: A

A testis that appears in warm conditions or which can be brought down on clinical examination and does not immediately retract is
usually a retractile testis. Theme from April 2012 Exam

Cryptorchidism: A congenital undescended testis is one that has failed to reach the bottom of the scrotum by 3 months of age. At
birth up to 5% of boys will have an undescended testis, post natal descent occurs in most and by 3 months the incidence of
cryptorchidism falls to 1-2%. In the vast majority of cases the cause of the maldescent is unknown. A proportion may be associated
with other congenital defects including: Patent processus vaginalis. Abnormal epididymis. Cerebral palsy. Mental retardation.
Wilms tumour. Abdominal wall defects (e.g. gastroschisis, prune belly syndrome)

Reasons for correction of cryptorchidism: Reduce risk of infertility. Allows the testes to be examined for testicular cancer. Avoid
testicular torsion. Cosmetic appearance. Males with undescended testis are 40 times as likely to develop testicular cancer
(seminoma) as males without undescended testis. The location of the undescended testis affects the relative risk of testicular cancer
(50% intra-abdominal testes)

Treatment: Orchidopexy at 6- 18 months of age. The operation usually consists of inguinal exploration, mobilisation of the testis
and implantation into a dartos pouch. Intra-abdominal testis should be evaluated laparoscopically and mobilised. Whether this is a
single stage or two stage procedure depends upon the exact location. After the age of 2 years in untreated individuals the Sertoli
cells will degrade and those presenting late in teenage years may be better served by orchidectomy than to try and salvage a non
functioning testis with an increased risk of malignancy.

3. Which of the following statements relating to omphalocele is false?


A. The herniated organs lie outside the peritoneal sac
B. Cardiac abnormalities co-exist in 25%
C. Intestines are almost always malrotated
D. The defects occurs through the umbilicus
E. Mortality may be as high as 15%
Answer: A

Gastroschisis: Isolated abnormality, bowel lies outside abdominal wall through defect located to right of umbilicus.
Exomphalos: Liver and gut remain covered with membranous sac connected to umbilical cord. It is associated with other
developmental defects. They are contained within the peritoneal sac and therefore do not have the fluid losses seen in
gastroschisis. Because the intestines are not located in a intra abdominal location, a degree of intestinal malrotation is almost
inevitable. However, this aspect does not always result in a requirement for surgery.

Paediatric Gastrointestinal disorders

Pyloric stenosis M>F. 5-10% Family history in parents. Projectile non bile stained vomiting at 4-6 weeks of life. USS diagnosis.
Treatment: Ramstedt pyloromyotomy

Acute appendicitis: Uncommon under 3 years. When occurs may present atypically

Mesenteric adenitis: Central abdominal pain and URTI. Conservative management

Intussusception: Telescoping bowel. Proximal to ileocaecal valve. 6-9 months age. Colicky pain, diarrhoea and vomiting,
sausage shaped mass, red jelly stool. Treatment: reduction with air insufflation

Malrotation: High caecum at the midline. Feature in exomphalos, congenital diaphragmatic hernia, intrinsic duodenal atresia

Hirschsprung's disease: Absence of ganglion cells from myenteric and submucosal plexuses. Occurs in 1/5000 births. Full
thickness rectal biopsy for diagnosis. Delayed passage of meconium and abdominal distension

468
Oesophageal atresia: Associated with tracheo-oesophageal fistula and polyhydramnios. PC choking and cyanotic episodes.
VACTERL

Meconium ileus: Majority have cystic fibrosis. X-Rays will not show a fluid level as the meconium is viscid

Biliary atresia: Jaundice > 14 days. Increased conjugated bilirubin. Urgent Kasai procedure

Necrotising enterocolitis: Prematurity is the main risk factor. Early features include abdominal distension and passage of bloody
stools. X-Rays may show pneumatosis intestinalis and evidence of free air. Increased risk when empirical antibiotics are given to
infants beyond 5 days

Theme: Bilious vomiting in neonates

A. Biliary atresia
B. Intestinal malrotation
C. Ileal atresia
D. Necrotising enterocolitis
E. Duodenal atresia
F. Meconium ileus
G. Viral gastroenteritis
H. Pyloric stenosis

Please select the most likely underlying cause of bilious vomiting for the situation described. Each option may be used once, more
than once or not at all.

4. A male infant is born prematurely at 26 weeks gestation by emergency cesarean section. Following the birth he develops
respiratory distress syndrome and is ventilated. He begins to improve twelve days after birth. Then he becomes unwell
and develops abdominal distension and passes bloody stools and vomits a small quantity of bile stained vomit.

The correct answer is Necrotising enterocolitis. Necrotising enterocolitis often has a delayed presentation and affected infants
will typically pass bloody stools. Plain films may show air in the intestinal wall (Pneumatosis).

5. A male infant is born by spontaneous vaginal delivery at 39 weeks gestation. He is well after the birth, established on
bottle feeding and discharged home. His parents are concerned because he subsequently becomes unwell and vomits a
large quantity of bile stained vomit approximately 2 days after discharge home. On examination he looks ill and his
abdomen is soft and non distended.

The correct answer is Intestinal malrotation. Intestinal malrotation with volvulus will typically compromise the vascularisation
and lumenal patency of the gut. This will cause bilious vomiting and the vascular insufficiency will produce a clinical picture of
illness at odds with the lack of overt abdominal signs. Delay in diagnosis and surgery will result in established necrosis,
perforation and peritonitis.

6. A female infant is born by cesarean section at 38 weeks gestation for foetal distress. The attending paediatricians notice
that she has a single palmar crease and an anti mongoloid slant to her eyes. Soon after the birth the mother tries to feed
the child who has a projectile vomit about 10 minutes after feeding. On examination she has a soft, non distended
abdomen.

The correct answer is Duodenal atresia. Proximally sited atresia will produce high volume vomits which may or may not be bile
stained. Abdominal distension is characteristically absent. Whilst under resuscitated children may be a little dehydrated they are
seldom severely ill. The presence of Trisomy 21 (palmar and eye signs) increases the likelihood of duodenal atresia.

Bilious vomiting in neonates

Causes of intestinal obstruction with bilious vomiting in neonates


Disorder Incidence and Age at presentation Diagnosis Treatment
causation
Duodenal 1 in 5000 (higher in Few hours after birth AXR shows "double Duodenoduodenostomy
atresia Downs syndrome) bubble sign, contrast
study may confirm
469
Malrotation Usually cause by Usually 3-7 days after Upper GI contrast Ladd's procedure
with incomplete rotation birth, volvulus with study may show DJ
volvulus during compromised circulation flexure is more
embryogenesis may result in peritoneal medially placed, USS
signs and haemodynamic may show abnormal
instability orientation of SMA
and SMV
Jejunal/ ileal Usually caused by Usually within 24 hours AXR will show air- Laparotomy with primary
atresia vascular of birth fluid levels resection and anastomosis
insufficiency in
utero, usually 1 in
3000
Meconium Occurs in between 15 Typically in first 24-48 Air - fluid levels on Surgical decompression, serosal
ileus and20% of those hours of life with AXR, sweat test to damage may require segmental
babies with cystic abdominal distension and confirm cystic fibrosis resection
fibrosis, otherwise 1 bilious vomiting
in 5000
Necrotising Up to 2.4 per 1000 Usually second week of Dilated bowel loops on Conservative and supportive for
enterocolitis births, risks increased life AXR, pneumatosis and non perforated cases, laparotomy
in prematurity and portal venous air and resection in cases of
inter-current illness perforation of ongoing clinical
deterioration
Theme: Paediatric gastrointestinal disorders

A. Meconium ileus
B. Biliary atresia
C. Oesophageal atresia
D. Pyloric stenosis
E. Intussusception
F. Malrotation
G. Hirschsprung disease
H. Mesenteric adenitis.

What is the most likely diagnosis for each scenario given? Each option may be used once, more than once or not at all.

7. A 3 day old baby presents with recurrent episodes of choking and cyanotic episodes. There is a history of
polyhydramnios.

Oesophageal atresia. Diagnosis is confirmed when an nasogastric tube fails to reach the stomach.

8. 3 day old neonate is developing increasing problems with feeding. On examination she has a pan systolic murmur and
her forearms have not developed properly.

Oesophageal atresia. This child has VACTERL, which is a combination of Vertebral, Ano-rectal, Cardiac, Tracheo-
oesophageal, Renal and Radial limb anomalies. Half of babies with oesophageal atresia will have VACTERL.

9. A 2 year old child has central abdominal pain. He has had a recent upper respiratory tract infection.

Mesenteric adenitis. Mesenteric adenitis may complicate upper respiratory tract infection and clinical exclusion of appendicitis
can be difficult.

Theme: Neonatal gastrointestinal disease

A. Ano-rectal atresia
B. Pyloric stenosis
C. Hirschbrungs disease
D. Duodenal atresia
E. Meconium ileus
F. Intussusception
G. Necrotising enterocolitis
H. Intestinal volvulus
I. Tracheo-oesophageal fistula
470
Please select the most likely diagnosis to account for the case described. Each option may be used once, more than once or not at
all.

10. A newborn baby boy presents with mild abdominal distension and failure to pass meconium after 24 hours. X- Ray
reveals dilated loops of bowel with fluid levels. The anus appears normally located.

Hirschbrungs disease. Hirschsprung's disease is an absence of ganglion cells in the neural plexus of the intestinal wall. It is
more common in boys than girls. The delayed passage of meconium together with distension of abdomen is the usual clinical
presentation. A plain abdominal x ray will demonstrate dilated loops of bowel with fluid levels and a barium enema can be
helpful when it demonstrates a cone with dilated ganglionic proximal colon and the distal aganglionic bowel failing to distend.
11. A premature infant (30-week gestation) presents with distended and tense abdomen. She is passing blood and mucus per
rectum, and she is also manifesting signs of sepsis.

Necrotising enterocolitis. Necrotising enterocolitis is more common in premature infants. Mesenteric ischemia causes bacterial
invasion of the mucosa leading to sepsis. Terminal ileum, caecum and the distal colon are commonly affected. The abdomen is
distended and tense, and the infant passes blood and mucus per rectum. X -Ray of the abdomen shows distended loops of
intestine and gas bubbles may be seen in the bowel wall.
12. A newborn baby boy presents with gross abdominal distension. He is diagnosed with cystic fibrosis and his abdominal x
ray shows distended coils of small bowel, but no fluid levels.

Meconium ileus. One in 15,000 newborns will have a distal small bowel obstruction secondary to abnormal bulky and viscid
meconium. Ninety percent of these infants will have cystic fibrosis and the abnormal meconium is the result of deficient
intestinal secretions. This condition presents during the first days of life with gross abdominal distension and bilious vomiting.
x Ray of the abdomen shows distended coils of bowel and typical mottled ground glass appearance. Fluid levels are scarce as
the meconium is viscid.

13. Which of the following statements relating to biliary atresia is untrue?


A. It most commonly presents as prolonged conjugated jaundice in the neonatal period.
B. Evidence of portal hypertension at diagnosis is seldom present in the UK.
C. It may be confused with Alagille syndrome.
D. The Kasai procedure is best performed in the first 8 weeks of life.
E. Survival following a successful Kasai procedure is approximately 45% at 5 years.
Answer: E
Alagille syndrome autosomal dominant disorder characterised by presence of paucity of bile ducts and cardiac defects. Only the
embryonic form of biliary atresia is associated with cardiac and other embryological defects.
Biliary atresia usually presents with obstructed jaundice. A Kasai procedure is best performed in the first 8 weeks of life. If a
Kasai procedure is successful most patients will not require liver transplantation. 45% of patients post Kasai procedure will
require transplantation. However, overall survival following a successful Kasai procedure is 80%.

Biliary atresia: 1 in 17000 affected. Biliary tree lumen is obliterated by an inflammatory cholangiopathy causing progressive
liver damage

Clinical features: Infant well in 1st few weeks of life. No family history of liver disease. Jaundice in infants > 14 days in term
infants (>21 days in pre term infants). Pale stool, yellow urine (colourless in babies). Associated with cardiac malformations,
polysplenia, situs inversus

Investigation: Conjugated bilirubin (prolonged physiological jaundice or breast milk jaundice will cause a rise in unconjugated
bilirubin, whereas those with obstructive liver disease will have a rise in conjugated bilirubin). TEBIDA radionuclide scan

Management: Early recognition is important to prevent liver transplantation. Nutritional support. Roux-en-Y portojejunostomy
(Kasai procedure). If Kasai procedure fails or late recognition, a liver transplant becomes the only option.

Theme: Administration of intravenous fluids

A. 0.9% Saline
B. 5% Dextrose
C. 20% Glucose
D. 0.18% saline/ 4% glucose
E. 0.45% saline/ 15% glucose
F. 0.45% saline/ 2.5% glucose
G. 4.5% albumin
471
H. 10% Pentastarch
I. 10% Dextrose

For the scenario given please select the most appropriate type of intravenous fluid for the scenario given. Each option may be used
once, more than once or not at all.

14. A 4 year old boy is undergoing an elective orchidopexy.

The correct answer is 0.9% Saline. Isotonic fluids should be used in this setting and 0.9% saline is the safest option.

15. A 2 day old boy is recovering from an inguinal herniotomy he has yet to feed and the nursing staff would like a
prescription for an initial fluid to be given on return to the ward. His potassium is within normal limits.

The correct answer is 10% Dextrose. Neonates require 10% dextrose solutions as they are at risk of developing hypoglycaemia.

16. A 4 year boy with learning difficulties has developed swallowing problems and is awaiting a PEG tube. He required
maintenance IV fluids and the nursing staff require choice of fluid for the next bag. He has just been given 250ml of
0.9% saline.

5% Dextrose. 5% Dextrose would the routine choice for water replacement.

Paediatric fluid management: Since 2000 there have been at least 4 reported deaths from fluid induced hyponatraemia in
children. This led to the National Patient Safety Agency introducing revised guidelines in 2007.
Indications for IV fluids include: Resuscitation and circulatory support. Replacing on-going fluid losses. Maintenance fluids for
children for whom oral fluids are not appropriate. Correction of electrolyte disturbances

Fluids to be avoided: Outside the neonatal period saline / glucose solutions should not be given. The greatest risk is with saline
0.18 / glucose 4% solutions. The report states that 0.45% saline / 5% glucose may be used. But preference should be given to
isotonic solutions and few indications exist for this solution either.

Fluids to be used: 0.9% saline. 5% glucose (though only with saline for maintenance and not to replace losses). Hartmans
solution. Potassium should be added to maintenance fluids according patients plasma potassium levels (which should be
monitored).

Intraoperative fluid management: Neonates should receive glucose during surgery. Other children should receive isotonic
crystalloid.

Maintenance fluids
Weight Water requirement/kg/day Na mmol/kg/day K mmol/kg/day
First 10Kg body weight 100ml 2-4 1.5-2.5
Second 10Kg body weight 50ml 1-2 0.5-1.5
Subsequent Kg 20ml 0.5-1.0 0.2-0.7

Glucose will need to be given to neonates- usually 10% at a rate of 60ml/Kg/day.

Theme: Paediatric gastrointestinal disorders

A. Duodenal atresia
B. Pyloric stenosis
C. Budd Chiari Syndrome
D. Annular pancreas
E. Oesophageal atresia
F. Congenital diaphragmatic hernia
G. Cystic fibrosis

Please select the most likely diagnosis for the scenario given. Each option may be used once, more than once or not at all.

17. A 31 year old women gives birth to a male infant weighing 2.5kg by induction of labour at 38 weeks. During the third
trimester of her pregnancy she has been troubled by polyhydramnios but otherwise her pregnancy proceeded
uneventfully. Instrumental delivery was instituted for foetal distress, but initially the obstetricians were reassured by the
absence of meconium in the liqor. In the hours following birth the baby is struggling to feed and no meconium has been
passed.
472
The correct answer is Oesophageal atresia. Polyhydramnois is a feature of oesophageal atresia. This condition occurs during the
4th foetal week when separation of the trachea and oesophagus occur. It is associated with other birth defects including vertebral
anomalies, imperforate anus (hence the lack of meconium) , cardiac anomalies, tracheal anomalies, renal and limb problems
(VACTERL syndrome)

18. A 6 week old baby is developing well and develops profuse and projectile vomiting after feeds. After assessment by the
paediatricians he is taken for an ultrasound scan which demonstrates an abnormality for which he is due to undergo a
Ramstedts procedure. His blood tests reveal a hypochloraemic metabolic alkalosis.

Pyloric stenosis. This is a typical scenario for pyloric stenosis. A Ramstedts pyloromyotomy is the eponymous name given to the
procedure. Diagnosis is often confirmed by USS.

19. A 25 year old women delivers a Downs syndrome baby of 38 weeks gestation. She has polyhydramnios in the latter stages
of pregnancy and after the birth the infant begins to feed. However, he begins to vomit and the vomit itself is copious and
bile stained. His blood tests reveal a hypochloraemic metabolic acidosis and aciduria.

Duodenal atresia. This is the typical scenario for duodenal atresia. About 1/3 cases occur in patients with Downs Syndrome. Pre
natal USS with show a double bubble deformity. The majority of abnormalities occur distal to the Ampulla and so vomiting is
bile stained although this is not the case with more proximal obstructions.

20. A 3 day old baby develops dyspneoa. A chest x-ray is performed and shows a radio-opaque shadow with an air-fluid
level in the chest. It is located immediately anterior to the 6th hemivertebra. Which of the following is the most likely
underlying diagnosis?
A. Bronchogenic cyst
B. Congenital diaphragmatic hernia
C. Infection with Staphylococcus aureus
D. Oesphageal duplication cyst
E. Hiatus hernia
Answer: A
A midline cystic mass of an infant in this age group is most likely to be a bronchogenic cyst. Hiatus hernia is unusual in the
neonatal period. Oesophageal duplication cysts are very rare and respiratory symptoms are less common than with bronchogenic
cysts.

Bronchogenic cysts: Overview: Bronchogenic cysts most commonly arise as a result of anomalous development of the ventral
foregut. They are most commonly single, although multiple cysts are described. They often lie near the midline and most
frequently occur in the region of the carina. They may be attached to the tracheobronchial tree, although they are seldom in direct
connection with it. Cases may be asymptomatic or present with respiratory symptoms early in the neonatal period. They are the
second most common type of foregut cysts (after enterogenous cysts) in the middle mediastinum. Up to 50% of cases are
diagnosed prior to 15 years of age.

Investigation: Many cases are diagnosed on antenatal ultrasound. Others may be detected on conventional chest radiography as a
midline spherical mass or cystic structure. Once the diagnosis is suspected a CT scan should be performed.

Treatment: Thorascopic resection is the ideal treatment. Very young babies can be operated on once they reach six weeks of age.

Theme: Paediatric umbilical disorders

A. Omphalitis
B. Umbilical hernia
C. Umbilical granuloma
D. Paraumbilical hernia
E. Persistent vitello-intestinal duct
F. Persistent uranchus

Please select the most likely underlying disorder for the umbilical condition described. Each option may be used once, more than
once or not at all.

473
21. A 2 week old baby is referred to the surgical team by the paediatricians. They are concerned because the child has a
painful area of macerated tissue at the site of the umbilicus. On examination a clear- yellowish fluid is seen to be
draining from the umbilicus when the baby cries.

Persistent uranchus. A patent uranchus will present with umbilical urinary discharge. The skin may become macerated if not
properly cared for. The discharge is most likely to be present when intra-abdominal pressure is raised. It is associated with
posterior urethral valves.

22. A premature neonate is born by emergency cesarean section at 29 weeks gestation. He initially seems to be progressing
well. However, the team are concerned because he becomes systemically septic and on examination has a swollen and
erythematous umbilicus.

Omphalitis. Infection from omphalitis may spread rapidly and cause severe sepsis especially in immunologically compromised,
premature neonates.

23. A baby boy is born by elective cesarean section at 39 weeks gestation. He initially seems to progress well and is
discharged from hospital the following day. The parents bring the child to the clinic at 10 days of age and are concerned
at the presence of a profuse and foul smelling discharge at the site of the umbilicus. On examination the umbilicus has
some prominent granulation tissue. When the baby cries a small trickle of brownish fluid is seen to pass from the
umbilicus.

The correct answer is Persistent vitello-intestinal duct. A persistent vitello-intestinal duct may allow the persistent and ongoing
discharge of small bowel content from the umbilicus. This fluid may be very irritant to the surrounding skin.

Paediatric umbilical disorders: Embryology: During development the umbilicus has two umbilical arteries and one umbilical
vein. The arteries are continuous with the internal iliac arteries and the vein is continuous with the falciform ligament (ductus
venosus). After birth the cord dessicates and separates and the umbilical ring closes.

Umbilical hernia: Up to 20% of neonates may have an umbilical hernia, it is more common in premature infants. The majority of
these hernias will close spontaneously (may take between 12 months and three years). Strangulation is rare.

Paraumbilical hernia: These are due to defects in the linea alba that are in close proximity to the umbilicus. The edges of a
paraumbilical hernia are more clearly defined than those of an umbilical hernia. They are less likely to resolve spontaneously than
a paraumbilical hernia.

Omphalitis: This condition consists of infection of the umbilicus. Infection with Staphylococcus aureus is the commonest cause.
The condition is potentially serious as infection may spread rapidly through the umbilical vessels in neonates with a risk of portal
pyaemia, and portal vein thrombosis. Treatment is usually with a combination of topical and systemic antibiotics.

Umbilical granuloma: These consist of cherry red lesions surrounding the umbilicus, they may bleed on contact and be a site of
seropurulent discharge. Infection is unusual and they will often respond favorably to chemical cautery with topically applied silver
nitrate.

Persistent uranchus: This is characterised by urinary discharge from the umbilicus. It is caused by persistence of the uranchus
which attaches to the bladder. They are associated with other urogenital abnormalities.

Persistent vitello-intestinal duct: This will typically present as an umbilical discharge that discharges small bowel content.
Complete persistence of the duct is a rare condition. Much more common is the persistence of part of the duct (Meckels
diverticulum). Persistent vitello-intestinal ducts are best imaged using a contrast study to delineate the anatomy and are managed
by laparotomy and surgical closure.

24. Which one of the following is least associated with Tetralogy of Fallot?
A. Right ventricular outflow tract obstruction
B. Overriding aorta
C. Ejection systolic murmur
D. Left-to-right shunt
E. Right ventricular hypertrophy
Answer: D
Right-to-left shunting is characteristic of Fallot's. It is however known that a small number of asymptomatic infants may initially
have a degree of left-to-right shunting through the ventricular septal defect.
474
Tetralogy of Fallot: Tetralogy of Fallot (TOF) is the most common cause of cyanotic congenital heart disease*. It typically
presents at around 1-2 months, although may not be picked up until the baby is 6 months old. The four characteristic features are:
ventricular septal defect (VSD). Right ventricular hypertrophy. Right ventricular outflow tract obstruction, pulmonary stenosis.
Overriding aorta
The severity of the right ventricular outflow tract obstruction determines the degree of cyanosis and clinical severity

Other features: cyanosis.causes a right-to-left shunt. ejection systolic murmur due to pulmonary stenosis (the VSD doesn't
usually cause a murmur). a right-sided aortic arch is seen in 25% of patients. chest x-ray shows a 'boot-shaped' heart, ECG shows
right ventricular hypertrophy

Management: surgical repair is often undertaken in two parts. Cyanotic episodes may be helped by beta-blockers to reduce
infundibular spasm

*however, at birth transposition of the great arteries is the more common lesion as patients with TOF generally present at around
1-2 months

25. A 3 year old boy is brought to the clinic with symptoms of urinary hesitancy and poor stream. Which of the following is
the most likely underlying diagnosis?
A. Benign prostatic hypertrophy
B. Posterior urethral valves
C. Neurogenic bladder
D. Urethral calculus
E. Hypospadias
Answer: B
Posterior urethral valves are one of the commonest causes of poor urinary stream and hesitancy in children. Prostatic disorders are
rare.
Hypospadias is associated with urine that is difficult to control, but should not produce hesitancy.

Urethral valves: Posterior urethral valves are the commonest cause of infravesical outflow obstruction in males. They may be
diagnosed on ante natal ultrasonography. Because the bladder has to develop high emptying pressures in utero the child may
develop renal parenchymal damage. This translates to renal impairment noted in 70% of boys at presentation. Treatment is with
bladder catheterisation. Endoscopic valvotomy is the definitive treatment of choice with cystoscopic and renal follow up.

26. Which of the following is not a feature of oesphageal atresia in neonates?


A. High incidence of polyhydramnios
B. Risk of recurrence in subsequent pregnancies of 80%
C. Distal tracheoesphageal fistula is the commonest variant
D. High incidence of associated imperforate anus
E. Absence of gastric bubble on antenatal ultrasound
Answer: B

Most are sporadic and risk in subsequent pregnancies is not increased.

27. A 12 day old infant is brought to the emergency department by his anxious mother who notices that he has developed a
right sided groin swelling. On examination the testes are correctly located but it is evident that the child has a right sided
inguinal hernia. It is soft and easily reduced. What is the most appropriate management?
A. Surgery over the next few days
B. Reassure and discharge
C. Surgery at 1 year of age
D. Surgery once the child is 6 months old
E. Application of a hernia truss
Answer: A
Inguinal hernia in infants = Urgent surgery
The high incidence of strangulation necessitates an urgent herniotomy be performed. In infants with a reducible hernia this can be
performed on a daycase list during the same week. Deferring surgery on the basis of age is not justified.

Paediatric inguinal hernia: Inguinal hernias are a common disorder in children. They are commoner in males as the testis
migrates from its location on the posterior abdominal wall, down through the inguinal canal. A patent processus vaginalis may
persist and be the site of subsequent hernia development.
475
Children presenting in the first few months of life are at the highest risk of strangulation and the hernia should be repaired
urgently. Children over 1 year of age are at lower risk and surgery may be performed electively. For paediatric hernias a
herniotomy without implantation of mesh is sufficient. Most cases are performed as day cases.

Theme: Paediatric gastrointestinal disorders

A. Meckel diverticulum
B. Pyloric stenosis
C. Acute appendicitis
D. Mesenteric adenitis
E. Intussusception
F. Malrotation
G. Hirschsprung disease

What is the most likely diagnosis for each scenario given? Each option may be used once, more than once or not at all.

28. A 48 hour old neonate develops increasing abdominal distension. He had a normal delivery but has yet to pass any
meconium. Following digital rectal examination liquid stool is released.

Hirschsprung disease. Hirschsprungs may present either with features of bowel obstruction in the neonatal period or more
insidiously during childhood. After the PR there may be an improvement in symptoms. Diagnosis is by full thickness rectal
biopsy.

29. A 7 month old girl presents with vomiting and diarrhoea. She is crying and drawing her legs up. There is a a sausage
shaped mass in the abdomen.

Intussusception. Sausage shaped mass (colon shaped) is common in intussusception. The other common sign is red jelly stool.

30. A 1 month old baby girl presents with bile stained vomiting. She has an exomphalos and a congenital diaphragmatic
hernia.

Malrotation. Exomphalos and diaphragmatic herniae are commonly associated with malrotation.

Theme: Paediatric gastrointestinal disorders

A. Liver transplant
B. Air insufflation
C. Roux-en-Y portojejunostomy
D. Ramstedt pyloromyotomy
E. Appendicectomy
F. Administration of antibiotics
G. Distal gastrectomy

What is the best management option for these children? Each option may be used once, more than once or not at all.

31. A 2 year old child has central abdominal pain. He has had a recent upper respiratory tract infection. On examination he
is febrile and has a soft abdomen with some paraumbilical tenderness.

Administration of antibiotics. This child has mesenteric adenitis. Symptoms should resolve. Should they fail to do so then
appendicectomy will be required as it can present insidiously in this age group.

32. A 2 month old baby presents with jaundice. He has an elevated conjugated bilirubin level. Diagnosis is confirmed by
cholangiography during surgery.

Roux-en-Y portojejunostomy. This child has biliary atresia. The aim is to avoid liver transplantation (however, most will come
to transplant in time).

33. A 6 week old baby boy presents with non bile stained projectile vomiting. He is otherwise developmentally normal. His
abdomen is soft and non tender.
476
Ramstedt pyloromyotomy. This baby has pyloric stenosis and the treatment of choice is a Ramstedt pyloromyotomy.

34. A 6 year old child develops ballooning of the foreskin on micturition and is brought to the clinic by his anxious mother.
One examination the foreskin is non retractile but otherwise normal. By which age are 95% of all foreskins retractile
A. 2 years
B. 16 years
C. 8 years
D. 5 years
E. 10 years
Answer: B
By 16 years of age almost all foreskins should be retractile and if they are not circumcision should be considered at around this
time.

Paediatric urology- foreskin disorders: Disorders of the foreskin


At birth and in the neonatal period the normal foreskin is non retractile due to the presence of adhesions between the foreskin and
glans. In most cases these will separate spontaneously. By the end of puberty 95% of foreskins can be retracted. In some children
the non-retractile foreskin may balloon during micturition. This is a normal variant and requires no specific treatment.
Balanitis This is inflammation of the glans penis. It may occur in both circumcised and non-circumcised individuals.
Posthitis This is inflammation of the foreskin. It may occur as a result of infections such as gonorrhoea and other
STD's. It may also complicate diabetes. Posthitis may progress to phimosis and as this may make cleaning
of the glans difficult allow progression to balanoposthitis.
Paraphimosis Prolonged retraction of the foreskin proximal to the glans may allow oedema to occur. This may then make
foreskin manipulation difficult. It can usually be managed by compression to reduce the oedema and
replacement of the foreskin. Where this fails a dorsal slit may be required and this followed by delayed
circumcision.
Phimosis This is inability to retract the foreskin and may be partial or complete. It may occur secondary to
balanoposthitis or balanitis xerotica obliterans. Depending upon the severity and symptoms treatment with
circumcision may be required.
Balanitis xerotica This is a dermatological condition in which scarring of the foreskin occurs leading to phimosis. It is rare
obliterans below the age of 5 years. Treatment is usually with circumcision.

35. What is the investigation of choice to look for renal scarring in a child with vesicoureteric reflux?
A. Abdominal x-ray
B. Ultrasound
C. DMSA
D. CT KUB
E. Micturating cystourethrogram
Answer: C

Vesicoureteric reflux (VUR) is the abnormal backflow of urine from the bladder into the ureter and kidney. It is relatively
common abnormality of the urinary tract in children and predisposes to urinary tract infection (UTI), being found in around 30%
of children who present with a UTI. As around 35% of children develop renal scarring it is important to investigate for VUR in
children following a UTI

Pathophysiology of VUR: ureters are displaced laterally, entering the bladder in a more perpendicular fashion than at an angle.
therefore shortened intramural course of ureter. Vesicoureteric junction cannot therefore function adequately

The table below summarises the grading of VUR

Grade
I Reflux into the ureter only, no dilatation
II Reflux into the renal pelvis on micturition, no dilatation
III Mild/moderate dilatation of the ureter, renal pelvis and calyces
IV Dilation of the renal pelvis and calyces with moderate ureteral tortuosity
V Gross dilatation of the ureter, pelvis and calyces with ureteral tortuosity

477
Investigation: VUR is normally diagnosed following a micturating cystourethrogram. A DMSA scan may also be performed to
look for renal scarring

36. Which of the following statements relating to necrotising enterocolitis is false?


1. It has a mortality of 30%
2. Most frequently presents in premature neonates less than 32 weeks gestation.
3. Should be managed by early laparotomy and segmental resections in most cases.
4. Pneumostosis intestinalis may be visible on plain abdominal x-ray.
5. May be minimised by use of breast milk over formula feeds.
Answer: C
Most cases will settle with conservative management with NG decompression and appropriate support. Laparotomy should be
undertaken in patients who progress despite conservative management or in whom compelling indications for surgery exist (eg
free air).

37. A 6 week old baby boy is brought to the clinic by his mother. She is concerned because although the left testis is present
in the scrotum the right testis is absent. She reports that it is sometimes palpable when she bathes the child. on
examination the right testis is palpable at the level of the superficial inguinal ring. What is the most appropriate
management?
A. Discharge
B. Re-assess in 5 years
C. Laparoscopy
D. Re-assess in 6 months
E. Orchidopexy
Answer: D
Undescended testes are not uncommon in young children. They may be present in 4% of term infants, but only in 1.3% children at 3
months of age. In this scenario the testis is retractile and can be managed expectantly.

38. An 8 week old infant is brought to clinic with a history of 18 days of jaundice. The mother is breast feeding. He was a
full term baby. There is no family history of liver disease. What is the most appropriate next step?
A. Liver USS
B. Unconjugated bilirubin measurement
C. Conjugated bilirubin measurement
D. Reassure and discharge
E. ERCP
Answer: C
This baby is a full term and has > 14 days of jaundice, therefore needs an urgent conjugated bilirubin check to rule out biliary
atresia. If physiological jaundice the unconjugated bilirubin levels will be increased. Isotope scanning may be used in diagnosis, but
a definitive diagnosis is normally made during a laparotomy.
39. Which of the following statements relating to Hirschsprungs disease is false?
A. It is more common in males.
B. Is typically associated with a dilated aganglionic segment of bowel.
C. May present with delayed passage of meconium.
D. Mucosal biopsies are inadequate for diagnosis.
E. Disease extending beyond 30cm of colon and rectum is unusual.
Answer: B
There is a transition zone from the contracted aganglionic segment (the abnormal area) to dilated normal bowel on barium enema.
Males are more frequently affected than females. Surgery may involve a pull through procedure. A number of patients will have
ongoing evacuatory disturbance.

40. A 4 year old is admitted with right iliac fossa pain and is due to undergo an appendicectomy. The nursing staff would
like to give the child an infusion of intravenous fluid whilst waiting for theatre. Assuming electrolytes are normal, which
of the following is an appropriate fluid for infusion in this situation?
A. 10% Dextrose solution
B. 0.9% Saline solution
C. 0.45% saline/ 5% glucose solution
D. Gelofusin
E. None of the above
Answer: B
0.45% saline/ 5% glucose solutions carry a risk of hyponatraemia and is contra indicated- see below.

Theme: Paediatric gastrointestinal disorders


478
A. Appendicectomy
B. Active observation
C. Discharge
D. Ultrasound of the abdomen
E. Colonoscopy
F. Hydrostatic reduction under fluoroscopic guidance
G. Laparotomy

Please select the most appropriate form of management from the list above. Each option may be used once, more than once or not
at all.

41. A 5 year old girl has been unwell for 3 days with occasional vomiting and lethargy, she had one episode of diarrhoea. On
examination she has a soft abdomen with tenderness in the region of the right iliac fossa. Her temperature is 38.1. Urine
dipstick shows leucocytes (+) and protein (+).

Appendicectomy. The most likely diagnosis is appendicitis. This can often present with less robust signs in paediatric than adult
practice.

42. A 6 day old baby was born prematurely at 33 weeks. He has been suffering from respiratory distress syndrome and has
been receiving ventilatory support on NICU. He has developed abdominal distension and is increasingly septic.
Ultrasound of the abdomen shows free fluid and evidence of small bowel dilatation. His blood pressure has remained
labile despite inotropic support

Laparotomy. He has necrotising enterocolitis and whilst this is often initially managed medically a laparotomy is required if the
situation deteriorates.

43. A 5 year old child has been unwell with a sore throat and fever for several days. He progresses to develop periumbilical
abdominal discomfort and passes diarrhoea. This becomes blood stained. The paediatricians call you because the
ultrasound has shown a 'target sign'.

Hydrostatic reduction under fluoroscopic guidance. This child has an intussusception. The lymphadenopathy will have initiated
it. A target sign is seen on ultrasound and is the side on view of multiple layers of bowel wall. Reduction using fluoroscopy
with barium or water is the first line option.

44. A 4 year old boy presents with symptoms of dysuria and urinary frequency. A urine dipstick is positive for blood and
nitrites. A UTI is suspected. Which of the following follow up strategies is most appropriate?
A. Watchful waiting
B. Cystoscopy
C. DMSA scan
D. CT scan of pelvis
E. Renal MRI
Answer: A
A first presentation of an uncomplicated UTI (even in male children) may be managed expectantly. More than 1 UTI in a six month
period should prompt further investigation.

Urinary tract infection- paediatric: UTI's may occur in 5% of young girls and 1-2% males. The incidence is higher in premature
infants. E-Coli accounts for 80% cases. In children with UTI it is important to establish whether there is underlying urinary stasis
or vesico-ureteric reflux (or both). Pyelonephritis in children carries the risk of renal scarring 10% and this translates into a 10%
risk of developing end stage renal disease.

Diagnosis: Pyrexia lasting for more than 3 days mandates urine testing. Samples may be taken from mid-stream urine samples or
supra pubic aspiration. Urine collected from nappies usually have faecal contaminants. In samples showing mixed growth
contamination of the sample has usually occurred. As in adults >105 colony forming units of a single organism are usually
indicative of a UTI.

Management: A single isolated UTI (in girls) may be managed expectantly. > 2 UTI's (or 1 in males) in a 6 month period should
prompt further testing. Voiding cystourethrograms show the greatest anatomical detail and is the ideal first line test in males;
isotope cystography has a lower radiation dose and is the first line test in girls. USS should also be performed. Renal cortical
scintigraphy should be performed when renal scarring is suspected.
479
45. An 18 month old boy is brought to the emergency room by his parents. He was found in bed with a nappy filled with
dark red blood. He is haemodynamically unstable and requires a blood transfusion. Prior to this episode he was well
with no prior medical history. What is the most likely cause?
A. Necrotising enterocolitis
B. Anal fissure
C. Oesophageal varices
D. Meckels diverticulum
E. Crohns disease
Answer: D
Meckels diverticulum is the number one cause of painless massive GI bleeding requiring a transfusion in children between the ages
of 1 and 2 years.

Other causes of GI bleeding include:


Site Newborn 1 month to 1 year 1 to 2 years Older than 2 years
Upper GI Haemorrhagic disease, swallowed Oesophagitis/ gastritis Peptic ulcer disease Varices
tract maternal blood
Lower GI Anal fissure/ NEC Anal fissure/ Polyps / Meckels IBD/ Polyps/
tract Intussusception diverticulum Intussusception

Meckel's diverticulum: Congenital abnormality resulting in incomplete obliteration of the vitello-intestinal duct. Normally, in the
foetus, there is an attachment between the vitello-intestinal duct and the yolk sac.This disappears at 6 weeks gestation. The tip is
free in majority of cases. Associated with enterocystomas, umbilical sinuses, and omphaloileal fistulas. Arterial supply:
omphalomesenteric artery. 2% of population, 2 inches long, 2 feet from the ileocaecal valve. Typically lined by ileal mucosa but
ectopic gastric mucosa can occur, with the risk of peptic ulceration. Pancreatic and jejunal mucosa can also occur.

Clinical: Normally asymptomatic and an incidental finding. Complications are the result of obstruction, ectopic tissue, or
inflammation. Removal if narrow neck or symptomatic. Options are between wedge excision or formal small bowel resection and
anastomosis.

Urology
Theme: Urinary tract trauma

A. Urinary tract infection


B. Bladder outlet obstruction
C. Bulbar urethral rupture
D. Membranous urethral rupture
E. Bladder rupture
F. Bladder contusion

For the scenario given please select the most likely injury. Each option may be used once, more than once or not at all.

1. A 56 year old man is involved in a road traffic accident. He is found to have a pelvic fracture. He reports that he has
some lower abdominal pain. He has peritonism in the lower abdomen. The nursing staff report that he has not passed
any urine. A CT scan shows evidence of free fluid.

Bladder rupture. A pelvic fracture and lower abdominal peritonism should raise suspicions of bladder rupture (especially as this
man cannot pass urine).

2. A 52 year old man falls off his bike. He is found to have a pelvic fracture. On examination he is found to have perineal
oedema and on PR the prostate is not palpable. A urine dipstick shows blood.

The correct answer is Membranous urethral rupture. A pelvic fracture and highly displaced prostate should indicate a diagnosis
of membranous urethral rupture.

3. A 52 year old woman falls out of a tree while rescuing a cat. She has a pelvic fracture. She has suprapubic tenderness
and complains of dysuria. Her abdomen is soft and non tender. A urine dipstick shows blood, nitrates and leucocytes.

The correct answer is Urinary tract infection. There is no indication of a more sinister diagnosis here. The patient's abdomen is

480
normal and she is able to pass urine. Her dipstick confirms an infection. Also in women urethral injury is rare. Similar theme
questions in September 2009 and April 2010

Lower genitourinary tract trauma: Most bladder injuries occur due to blunt trauma. 85% associated with pelvic fractures.
Easily overlooked during assessment in trauma. Up to 10% of male pelvic fractures are associated with urethral or bladder injuries

Types of injury

Urethral injury: Mainly in males. Blood at the meatus (50% cases). There are 2 types: i.Bulbar rupture. Most common. Straddle
type injury e.g. bicycles. Triad signs: urinary retention, perineal haematoma, blood at the meatus
ii. Membranous rupture: can be extra or intraperitoneal. Commonly due to pelvic fracture. Penile or perineal oedema/ hematoma.
PR: prostate displaced upwards (beware co-existing retroperitoneal haematomas as they may make examination difficult).
Investigation: ascending urethrogram. Management: suprapubic catheter (surgical placement, not percutaneously)

External genitalia injuries (i.e., the penis and the scrotum):Secondary to injuries caused by penetration, blunt trauma,
continence- or sexual pleasure-enhancing devices, and mutilation

Bladder injury: rupture is intra or extraperitoneal. presents with haematuria or suprapubic pain. history of pelvic fracture and
inability to void: always suspect bladder or urethral injury. inability to retrieve all fluid used to irrigate the bladder through a Foley
catheter indicates bladder injury. investigation- IVU or cystogram. Management: laparotomy if intraperitoneal, conservative if
extraperitoneal

4. Which of the following renal stone types is most radiodense on a plain x-ray?
A. Calcium phosphate
B. Calcium oxalate
C. Uric acid
D. Struvate
E. Cystine
Answer: A
Calcium phosphate stones are the most radiodense stones, calcium oxalate stones slightly less so. Uric acid stones are radiolucent
(unless they have calcium contained within them).

Renal stones: Calcium oxalate: Hypercalciuria is a major risk factor (various causes). Hyperoxaluria may also increase risk
Hypocitraturia increases risk because citrate forms complexes with calcium making it more soluble. Stones are radio-opaque
(though less than calcium phosphate stones). Hyperuricosuria may cause uric acid stones to which calcium oxalate binds
Percentage of all calculi: 85%

Cystine: Inherited recessive disorder of transmembrane cystine transport leading to decreased absorption of cystine from intestine
and renal tubule. Multiple stones may form. Relatively radiodense because they contain sulphur. Percentage of all calculi 1%.

Uric acid: Uric acid is a product of purine metabolism. May precipitate when urinary pH low. May be caused by diseases with
extensive tissue breakdown e.g. malignancy. More common in children Stone type Urine acidity Mean urine
with inborn errors of metabolism. Radiolucent. Percentage of all pH
calculi: 5-10%. Calcium phosphate Normal- alkaline >5.5
Calcium oxalate Variable 6
Calcium phosphate: May occur in renal tubular acidosis, high urinary Uric acid Acid 5.5
pH increases supersaturation of urine with calcium and phosphate. Struvate Alkaline >7.2
Renal tubular acidosis types 1 and 3 increase risk of stone formation Cystine Normal 6.5
(types 2 and 4 do not). Radio-opaque stones (composition similar to
bone). Percentage of all calculi 10%.

Struvite: Stones formed from magnesium, ammonium and phosphate. Occur as a result of urease producing bacteria (and are thus
associated with chronic infections). Under the alkaline conditions produced, the crystals can precipitate. Slightly radio-opaque.
Percentage of all calculi: 2-20%>

Effect of urinary pH on stone formation: Urine pH will show individual variation (from pH 5-7). Post prandially the pH falls as
purine metabolism will produce uric acid. Then the urine becomes more alkaline (alkaline tide). When the stone is not available
for analysis the pH of urine may help to determine which stone was present.

Theme: Scrotal swellings

481
A. Haematocele
B. Epididymal cyst
C. Hydrocele
D. Testicular torsion
E. Orchitis
F. Epididymo-orchitis

For each case please select the most likely underlying diagnosis from the list. Each option may be used once, more than once or
not at all.

5. A 32 year old male presents with a swollen right scrotum which has developed over 3 weeks after being kicked in the
groin area. There is a non tense swelling of the right scrotum and the underlying testis cannot be easily palpated. A
dipstick is positive for nitrates only.

The correct answer is Hydrocele. This is a secondary hydrocele which occurs in patients aged 20-40 years. It develops rapidly
and there may not be a tense swelling. The underlying testis is NOT palpated therefore indicating a hydrocele. Causes include
trauma, infection and tumour.
6. A 40 year old male presents with a non painful, bilateral scrotal swellings over 3 years. The testis is felt separately and
the swelling transilluminates.

The correct answer is Epididymal cyst. The testis is palpated therefore this differentiates it from a hydrocele
7. A 32 year old male presents with a swollen, painful right scrotum after being kicked in the groin area 1 hour ago. There
is a painful swelling of the right scrotum and the underlying testis cannot be easily palpated.

The correct answer is Haematocele. Acute haematocele: tense, tender and non transilluminating mass post trauma. A chronic
haematoma causes a blood clot to surround the testis. The blood clot hardens and contracts causing a hard mass which may be
indistinguishable from a tumour. Therefore the testis will need surgical exploration.

Scrotal swelling: Differential diagnosis: Inguinal hernia: If inguinoscrotal swelling; cannot "get above it" on examination.
Cough impulse may be present. May be reducible
Testicular tumours: Often discrete testicular nodule (may have associated hydrocele). Symptoms of metastatic disease may be
present. USS scrotum and serum AFP and β HCG required

Acute epididymo-orchitis: Often history of dysuria and urethral discharge. Swelling may be tender and eased by elevating testis.
Most cases due to Chlamydia. Infections with other gram negative organisms may be associated with underlying structural
abnormality

Epidiymal cysts: Single or multiple cysts. May contain clear or opalescent fluid (spermatoceles). Usually occur over 40 years of
age. Painless. Lie above and behind testis. It is usually possible to "get above the lump" on examination

Hydrocele: Non painful, soft fluctuant swelling. Often possible to "get above it" on examination. Usually contain clear fluid. Will
often transilluminate. May be presenting feature of testicular cancer in young men

Testicular torsion: Severe, sudden onset testicular pain. Risk factors include abnormal testicular lie. Typically affects adolescents
and young males. On examination testis is tender and pain not eased by elevation . Urgent surgery is indicated, the contra lateral
testis should also be fixed

Varicocele: Varicosities of the pampiniform plexus. Typically occur on left (bacause testicular vein drains into renal vein). May
be presenting feature of renal cell carcinoma. Affected testis may be smaller and bilateral varicoceles may affect fertility

Theme: Management of testicular disorders

A. Antibiotics
B. Aspiration
C. Testicular exploration after 6h
D. Testicular exploration within 6h
E. Orchidectomy via inguinal approach
F. Orchidectomy via scrotal approach
G. No treatment needed
482
Please select the most appropriate management for the scenario given. Each option may be used once, more than once or not at all.

8. A 20 year old male notices a mild painful swelling of his right scrotum. He also complains of abdominal pain. Clinically
the patient is found to have a swollen right testicle. Apart from a supraclavicular node, there is no obvious
lymphadenopathy.

The correct answer is Orchidectomy via inguinal approach. The patient is likely to have a teratoma which has metastasized to
the supraclavicular nodes. There is suspicion of spread to the para-aortic nodes due to the abdominal pain. He will need
orchidectomy and combination chemotherapy. There is no role for orchidectomy via scrotal approach in malignancy.

9. A 40 year old male presents with a non painful, bilateral scrotal swellings over 3 years. The testis is felt separately and
the swelling transilluminates.

No treatment needed. This is an epididymal cyst, the testis is palpated therefore this differentiates it from a hydrocele.

10. A 32 year old male presents with a swollen, painful right scrotum after being kicked in the groin area. There is a painful
swelling of the right scrotum and the underlying testis cannot be easily palpated.

Testicular exploration within 6h. Acute haematocele: tense, tender and non transilluminating mass. The testis will need surgical
exploration to evacuate the blood and repair any damage.

11. A 75 year old man presents with locally advanced carcinoma of the prostate and vertebral body metastasis and
impending spinal cord compression. Which of the following agents (if used in isolation) carries the greatest risk of
worsening his symptoms in the short term?
A. Surgical orchidectomy
B. Cyproterone acetate
C. Luetenising hormone releasing hormone analogues
D. Flutamide
E. None of the above
Answer: C
LHRH analogues may cause flare of metastatic disease and anti androgens should be administered to counter this. Surgical
orchidectomy reduces testosterone levels within 8 hours (but fails to reduce adrenal androgen release). Cyproterone and flutamide
are androgen blockers that may be considered as add on therapy to reduce the risk of tumour flare when commencing treatment
with LH RH analogues.

Prostate Cancer: This is a common condition and up to 30,000 men are diagnosed with the condition each year. Up to 9,000 will
die in in the UK from the condition per year.

Diagnosis: Early prostate cancers have few symptoms. Metastatic disease may present as bone pain. Locally advanced disease
may present as pelvic pain or with urinary symptoms. Prostate specific antigen measurement. Digital rectal examination
Trans rectal USS (+/- biopsy). MRI/ CT and bone scan for staging.

PSA Test: The normal upper limit for PSA is 4ng/ml. However, in this group will lie patients with benign disease and some with
localised prostate cancer. False positives may be due to prostatitis, UTI, BPH, vigorous DRE.
The percentage of free: total PSA may help to distinguish benign disease from cancer. Values of <20% are suggestive of cancer
and biopsy is advised.

Pathology: 95% adenocarcinoma. In situ malignancy is sometimes found in areas adjacent to cancer. Multiple biopsies needed to
call true in situ disease. Often multifocal- 70% lie in the peripheral zone. Graded using the Gleason grading system, two grades
awarded 1 for most dominant grade (on scale of 1-5) and 2 for second most dominant grade (scale 1-5). The two added together
give the Gleason score. Where 2 is best prognosis and 10 the worst. Lymphatic spread occurs first to the obturator nodes and local
extra prostatic spread to the seminal vesicles is associated with distant disease.

Treatment: Watch and wait- Elderly, multiple co-morbidities, low Gleason score. Radiotherapy (External)- Both potentially
curative and palliative therapy possible. Similar survival figures to surgery. However, radiation proctitis and rectal malignancy are
late problems. Brachytherapy is a modification allowing internal radiotherapy. Surgery- Radical prostatectomy. Surgical removal
of the prostate is the standard treatment for localised disease. The robot is being used increasingly for this procedure. As well as
the prostate the obturator nodes are also removed to complement the staging process. Erectile dysfunction is a common side
effect. Hormonal therapy- Testosterone stimulates prostate tissue and prostatic cancers usually show some degree of testosterone
483
dependence. 95% of testosterone is derived from the testis and bilateral orchidectomy may be used for this reason.
Pharmacological alternatives include LHRH analogues and anti androgens (which may be given in combination).

Theme: Haematuria

A. Retroperitoneal liposarcoma
B. Transitional cell carcinoma
C. Retroperitoneal fibrosis
D. Renal squamous cell carcinoma
E. Renal adenocarcinoma
F. Nephroblastoma

Please select the most likely cause of haematuria for the scenarios given. Each option may be used once, more than once or not at
all.

12. A 28 year old man presents with hypertension and haematuria. Haematological investigations show polycythaemia but
otherwise no abnormality. CT scanning shows a left renal mass.

Renal adenocarcinoma. Renal adenocarcinoma is the most common variant and is associated with polycythaemia.

13. A 68 year of man presents with recurrent episodes of left sided ureteric colic and haematuria. Investigations show some
dilatation of the renal pelvis but the outline is irregular.

Transitional cell carcinoma. These arise from urothelium and necessitate a nephroureterectomy.

14. A 4 year old boy presents with haematuria and on examination is found to have a right sided renal mass.

Nephroblastoma. Wilms tumours (nephroblastoma) usually present in the first 4 years of life and may cause lung metastases.

Causes of haematuria.
 Trauma: Injury to renal tract. Renal trauma commonly due to blunt injury (others penetrating injuries). Ureter trauma rare:
iatrogenic. Bladder trauma: due to RTA or pelvic fractures.
 Infection: Remember TB.
 Malignancy: Renal cell carcinoma (remember paraneoplastic syndromes): painful or painless. Urothelial malignancies: 90%
are transitional cell carcinoma, can occur anywhere along the urinary tract. Painless haematuria. Squamous cell carcinoma and
adenocarcinoma: rare bladder tumours. Prostate cancer. Penile cancers: SCC.
 Renal disease: Glomerulonephritis.
 Stones:Microscopic haematuria common.
 Structural abnormalities: Benign prostatic hyperplasia (BPH) causes haematuria due to hypervascularity of the prostate
gland. Cystic renal lesions e.g. polycystic kidney disease. Vascular malformations. Renal vein thrombosis due to renal cell
carcinoma. Coagulopathy: Causes bleeding of underlying lesions.
 Drugs: Cause tubular necrosis or interstitial nephritis: aminoglycosides, chemotherapy. Interstitial nephritis: penicillin,
sulphonamides, and NSAIDs. Anticoagulants.
 Benign: Exercise.
 Gynaecological: Endometriosis: flank pain, dysuria, and haematuria that is cyclical.
 Iatrogenic: Catheterisation. Radiotherapy; cystitis, severe haemorrhage, bladder necrosis.
 Pseudohaematuri

15. A 22 year old man is involved in a road traffic accident. He is found to have a pelvic fracture. While on the ward the
nursing staff report that he is complaining of lower abdominal pain. On examination you find a distended tender
bladder. What is the diagnosis?
A. Bladder rupture
B. Ureter injury
C. Urethral injury
D. Clot retention
E. Prostate rupture
Answer: C

Pelvic fractures may cause laceration of the urethra. Urinary retention, blood at the urethral meatus and a high riding prostate on

484
digital rectal examination are the typical features. Theme from 2009 Exam

Theme: Haematuria

A. Benign prostatic hyperplasia


B. Ureteric calculus
C. Pyelonephritis
D. Prostatitis
E. Cystitis
F. Prostate cancer

Please select the most likely source of haematuria for the scenarios given. Each option may be used once, more than once or not at
all.

16. A 67 year old man presents with recurrent episodes of haematuria, typically at the end of the urinary stream, he has
been suffering from occasional fevers and has noticed pus on the urethral meatus on occasion. On examination the
prostate has no discernable masses but is tender.

The correct answer is Prostatitis. This is most likely prostatitis and the bleeding at the end of micturition suggests a distal
problem. Treatment is usually with prolonged courses of antibiotics.

17. A 23 year old girl is admitted with loin pain and a fever, she has noticed haematuria for the past week accompanied by
dysuria, this was treated empirically with trimethoprim.

Pyelonephritis. This is most likely pyelonephritis and partially treated cystitis is a common cause.

18. A 56 year old man is admitted with severe loin to groin pain associated with haematuria. He was well until 1 week ago
when he was unwell with diarrhoea and vomiting.

Ureteric calculus. Ureteric stones may develop in a background of dehydration.

19. Which of the following would be most consistent with a histologically aggressive form of prostate cancer?
A. FIGO stage 1 disease
B. FIGO stage IV disease
C. EuroQOL score of 5
D. Gleason score of 2
E. Gleason score of 10
Answer: E
Prostate cancer is histologically graded using the Gleason score (see below). A score of 10 is consistent with a histologically
aggressive form of the disease. The FIGO staging system is used to stage gynaecological malignancy. The EuroQOL score is a
quality of life measurement tool.
20. A 13 month old boy is brought to the paediatric clinic by his mother who is concerned that his testis are not palpable. On
examination his testis are not palpable either in the scrotum or inguinal region and cannot be visualised on ultrasound
either. What is the most appropriate next stage in management?
A. Laparoscopy
B. Re-assess at 5 years of age
C. Re-assess at 13 years of age
D. Administration of testosterone
E. Administration of cyproterone acetate
Answer: A
Truly impalpable testis and inability to visualise on ultrasound are indications for a laparoscopy. They may be associated with an
intra-abdominal location. Whilst it is reasonable to defer orchidopexy for retractile testis completely absent testes should be
investigated further.

Cryptorchidism: A congenital undescended testis is one that has failed to reach the bottom of the scrotum by 3 months of age. At
birth up to 5% of boys will have an undescended testis, post natal descent occurs in most and by 3 months the incidence of
cryptorchidism falls to 1-2%. In the vast majority of cases the cause of the maldescent is unknown. A proportion may be
associated with other congenital defects including: Patent processus vaginalis. Abnormal epididymis. Cerebral palsy. Mental
retardation. Wilms tumour. Abdominal wall defects (e.g. gastroschisis, prune belly syndrome)

485
Reasons for correction of cryptorchidism: Reduce risk of infertility. Allows the testes to be examined for testicular cancer.
Avoid testicular torsion, Cosmetic appearance

Males with undescended testis are 40 times as likely to develop testicular cancer (seminoma) as males without undescended testis
The location of the undescended testis affects the relative risk of testicular cancer (50% intra-abdominal testes)

Treatment: Orchidopexy at 6- 18 months of age. The operation usually consists of inguinal exploration, mobilisation of the testis
and implantation into a dartos pouch. Intra-abdominal testis should be evaluated laparoscopically and mobilised. Whether this is a
single stage or two stage procedure depends upon the exact location. After the age of 2 years in untreated individuals the Sertoli
cells will degrade and those presenting late in teenage years may be better served by orchidectomy than to try and salvage a non
functioning testis with an increased risk of malignancy.

21. A 34-year-old man from Zimbabwe is admitted with abdominal pain to the Emergency Department. An abdominal x-ray
reveals urinary bladder calcification. What is the most likely cause?
A. Schistosoma mansoni
B. Sarcoidosis
C. Leishmaniasis
D. Tuberculosis
E. Schistosoma haematobium
Answer: E
Schistosoma haematobium causes haematuria. Schistosomiasis is the most common cause of bladder calcification worldwide

Schistosomiasis, or bilharzia, is a parasitic flatworm infection. The following types of schistosomiasis are recognised:
Schistosoma mansoni and Schistosoma intercalatum: intestinal schistosomiasis. Schistosoma haematobium: urinary
schistosomiasis. Schistosoma haematobium: This typically presents as a 'swimmer's itch' in patients who have recently returned
from Africa. Schistosoma haematobium is a risk factor for squamous cell bladder cancer
Features: Frequency. Haematuria. Bladder calcification
Management: Single oral dose of praziquantel

Theme: Testicular disorders

A. Testicular tumour
B. Torsion of the spermatic cord
C. Acute infective epididymo-orchitis
D. Non infective epididymo- orchitis
E. Torsion of testicular appendage
F. Hydrocele
G. Haematocele

Please select the most likely cause for the testicular disorder described. Each option may be used once, more than once or not at
all.

22. An 28 year old man presents with pain in the testis and scrotum. It began 10 hours previously and has worsened during
that time. On examination he is pyrexial, the testis is swollen and tender and there is an associated hydrocele.

Acute infective epididymo-orchitis. The onset is relatively slow for torsion and the presence of fever favors epididymo-orchitis.
23. A 15 year old boy develops sudden onset of pain in the left hemiscrotum. He has no other urinary symptoms. On
examination the superior pole of the testis is tender and the cremasteric reflex is particularly marked.

The correct answer is Torsion of testicular appendage. The cremasteric reflex is usually preserved when the torsion affects the
appendage only.

24. A 14 year old boy develops sudden onset severe pain in the left testicle radiating to the left groin. He is distressed and
vomits. On examination the testis is very tender and the cremasteric reflex is absent.

The correct answer is Torsion of the spermatic cord.The cremasteric reflex is usually absent in torsion.

Similar theme question in September 2011 exam

Testicular disorders

486
Testicular cancer
Testicular cancer is the most common malignancy in men aged 20-30 years. Around 95% of cases of testicular cancer are germ-
cell tumours. Germ cell tumours may essentially be divided into:

Tumour type Key features Tumour markers Pathology


Seminoma  Commonest subtype  AFP usually normal Sheet like lobular patterns of
(50%)  HCG elevated in 10% cells with substantial fibrous
 Average age at diagnosis seminomas component. Fibrous septa
= 40  Lactate contain lymphocytic
 Even advanced disease dehydrogenase; elevated inclusions and granulomas
associated with 5 year in 10-20% seminomas may be seen.
survival of 73% (but also in many other
conditions)
Non seminomatous germ cell  Younger age at presentation  AFP elevated in up to Heterogenous texture with
tumours (42%) =20-30 years 70% of cases occasional ectopic tissue such
 Advanced disease carries  HCG elevated in up as hair
 Teratoma worse prognosis (48% at 5 years) to 40% of cases
 Yolk sac tumour  Retroperitoneal lymph node  Other markers rarely
 Choriocarcinoma dissection may be needed for helpful
residual disease after
 Mixed germ cell
chemotherapy
tumours (10%)

Risk factors for testicular cancer: Cryptorchidism. Infertility. Family history. Klinefelter's syndrome. Mumps orchitis

Features. A painless lump is the most common presenting symptom. Pain may also be present in a minority of men. Other
possible features include hydrocele, gynaecomastia

Diagnosis: Ultrasound is first-line. CT scanning of the chest/ abdomen and pelvis is used for staging. Tumour markers (see above)
should be measured

Management: Orchidectomy (Inguinal approach). Chemotherapy and radiotherapy may be given depending on staging.
Abdominal lesions >1cm following chemotherapy may require retroperitoneal lymph node dissection.

Prognosis is generally excellent: 5 year survival for seminomas is around 95% if Stage I. 5 year survival for teratomas is around
85% if Stage I

Benign disease

Epididymo-orchitis: Acute epididymitis is an acute inflammation of the epididymis, often involving the testis and usually caused
by bacterial infection. Infection spreads from the urethra or bladder. In men <35 years, gonorrhoea or chlamydia are the usual
infections. Amiodarone is a recognised non infective cause of epididymitis, which resolves on stopping the drug. Tenderness is
usually confined to the epididymis, which may facilitate differentiating it from torsion where pain usually affects the entire testis.

Testicular torsion: Twist of the spermatic cord resulting in testicular ischaemia and necrosis. Most common in males aged
between 10 and 30 (peak incidence 13-15 years). Pain is usually severe and of sudden onset. Cremasteric reflex is lost and
elevation of the testis does not ease the pain. Treatment is with surgical exploration. If a torted testis is identified then both testis
should be fixed as the condition of bell clapper testis is often bilateral.

25. Which of the following statements is false in relation to renal adenocarcinoma?


A. They account for over 75% cases of renal tumours
B. Renal biopsy should be performed in all cases considered for radical nephrectomy
C. They typically spread via the haematogenous route
D. Patients with completely resected T2 disease should not receive adjuvant chemotherapy
E. Partial nephrectomy gives equivalent oncological outcomes in patients with T1 disease
Answer: B
Routine chemotherapy is not effective in patients with renal adenocarcinoma and should not be used following R0 resections.
Routine renal biopsy should not be performed in cases for nephrectomy. Most cases of malignancy can be accurately classified
on imaging.

487
Renal cell carcinoma: Renal cell carcinoma is an adenocarcinoma of the renal cortex and is believed to arise from the proximal
convoluted tubule. They are usually solid lesions, up to 20% may be multifocal, 20% may be calcified and 20% may have either a
cystic component or be wholly cystic. They are often circumscribed by a pseudocapsule of compressed normal renal tissue.
Spread may occur either by direct extension into the adrenal gland, renal vein or surrounding fascia. More distant disease usually
occurs via the haematogenous route to lung, bone or brain.

Renal cell carcinoma comprise up to 85% of all renal malignancies. Males are more commonly affected than females and sporadic
tumours typically affect patients in their sixth decade. Patients may present with a variety of symptoms including; haematuria
(50%), loin pain (40%), mass (30%) and up to 25% may have symptoms of metastasis.Less than 10% have the classic triad of
haematuria, pain and mass.

Investigation: Many cases will present as haematuria and be discovered during diagnostic work up. Benign renal tumours are
rare, so renal masses should be investigated with multislice CT scanning. Some units will add and arterial and venous phase to the
scan to demonstrate vascularity and evidence of caval ingrowth. CT scanning of the chest and abdomen to detect distant disease
should also be undertaken. Routine bone scanning is not indicated in the absence of symptoms. Biopsy should not be performed
when a nephrectomy is planned but is mandatory before any ablative therapies are undertaken. Assessment of the functioning of
the contra lateral kidney.

Management: T1 lesions may be managed by partial nephrectomy and this gives equivalent oncological results to total radical
nephrectomy. Partial nephrectomy may also be performed when there is inadequate reserve in the remaining kidney.

For T2 lesions and above a radical nephrectomy is standard practice and this may be performed via a laparoscopic or open
approach. Preoperative embolisation is not indicated nor is resection of uninvolved adrenal glands. During surgery early venous
control is mandatory to avoid shedding of tumour cells into the circulation. Patients with completely resected disease do not
benefit from adjuvant therapy with either chemotherapy or biological agents. These should not be administered outside the
setting of clinical trials. Patients with transitional cell cancer will require a nephroureterectomy with disconnection of the ureter at
the bladder.

Theme: Haematuria

A. Benign prostatic hyperplasia


B. Transitional cell carcinoma of the bladder
C. Renal cell carcinoma
D. Ureteric calculus
E. Staghorn calculus
F. Uncomplicated urinary tract infection
G. Adenocarcinoma of the bladder

Please select the most likely cause of haematuria from the scenarios given. Each option may be used once, more than once or not
at all.

26. A 58 year old man has an episode of painless frank haematuria whilst undergoing a 24 urine collection for investigation
of hypertension.

Renal cell carcinoma. These tumours may often have paraneoplastic effects such as hypertension.

27. A 73 year old lady has an episode of haematuria whilst receiving a course of intravesical BCG therapy.

Transitional cell carcinoma of the bladder. Transitional cell carcinoma of the bladder may be treated with intravesical BCG
therapy.

28. A 32 year old lady suffers from severe left sided abdominal pain, that radiates to her groin. As part of her evaluation the
nurses identify microscopic haematuria on dipstick.

Ureteric calculus. Ureteric calculi will often present with loin pain radiating to the groin. It is usually severe. There may be
macroscopic or microscopic haematuria. The absence of haematuria on dipstick testing should prompt investigations for
alternative diagnoses. The best investigation is a non contrast CT scan. CT changes consistent with stone or recent stone passage
include evidence of stone, perinephric stranding, ureteric oedema or hydronephrosis.

Theme: Haematuria

A. Benign prostatic hyperplasia


488
B. Transitional cell carcinoma of the bladder
C. Renal cell carcinoma
D. Ureteric calculus
E. Staghorn calculus
F. Uncomplicated urinary tract infection
G. Adenocarcinoma of the bladder

Please select the most likely cause of haematuria from the scenarios given. Each option may be used once, more than once or not
at all.

29. A 58 year old man has an episode of painless frank haematuria whilst undergoing a 24 urine collection for investigation
of hypertension.

Renal cell carcinoma. These tumours may often have paraneoplastic effects such as hypertension.

30. A 73 year old lady has an episode of haematuria whilst receiving a course of intravesical BCG therapy.

Transitional cell carcinoma of the bladder. Transitional cell carcinoma of the bladder may be treated with intravesical BCG
therapy.

31. A 32 year old lady suffers from severe left sided abdominal pain, that radiates to her groin. As part of her evaluation the
nurses identify microscopic haematuria on dipstick.

Ureteric calculus. Ureteric calculi will often present with loin pain radiating to the groin. It is usually severe. There may be
macroscopic or microscopic haematuria. The absence of haematuria on dipstick testing should prompt investigations for
alternative diagnoses. The best investigation is a non contrast CT scan. CT changes consistent with stone or recent stone passage
include evidence of stone, perinephric stranding, ureteric oedema or hydronephrosis.

32. A 24 year old man presents with a persistent and unwanted erection that has been present for the previous 6 hours. On
examination the penis is rigid and tender. Aspiration of blood from the corpus cavernosa shows dark blood. Which of the
following is the most appropriate initial management?
A. Discharge the patient home and review in 12 hours
B. Admit the patient to hospital and review in 12 hours
C. Aspirate further blood from the corpus cavernosa in an attempt to decompress
D. Use a trucut needle to induce an arteriovenous shunt
E. Administer intracavernosal adrenaline 1 in 500 concentration
Answer: C
Low flow priaprism is a urological emergency. Aspiration of bright red blood is more reassuring and may indicate high flow
priaprism that may be actively monitored. Low flow priaprism should be decompressed with aspiration of blood from the corpus
caveronsum.

Physiology of erection
Autonomic  Sympathetic nerves originate from T11-L2 and parasympathetic nerves from S2-4 join to form pelvic
plexus.
 Parasympathetic discharge causes erection, sympathetic discharge causes ejaculation and detumescence.

Somatic Supplied by dorsal penile and pudendal nerves. Efferent signals are relayed from Onufs nucleus (S2-4) to
nerves innervate ischiocavernosus and bulbocavernosus muscles.

Autonomic discharge to the penis will trigger the veno-occlusive mechanism which triggers the flow of arterial blood into the
penile sinusoidal spaces. As the inflow increases the increased volume in this space will secondarily lead to compression of the
subtunical venous plexus with reduced venous return. During the detumesence phase the arteriolar constriction will reduce arterial
inflow and thereby allow venous return to normalise.

Priapism: Prolonged unwanted erection, in the absence of sexual desire, lasting more than 4 hours.

Classification of priaprism: Low flow priaprism: Due to veno-occlusion (high intracavernosal pressures). Most common type.
Often painful. Often low cavernosal flow. If present for >4 hours requires emergency treatment
High flow priaprism: Due to unregulated arterial blood flow. Usually presents as semi rigid painless erection
Recurrent priaprism Typically seen in sickle cell disease, most commonly of high flow type.

489
Causes: Intracavernosal drug therapies (e.g. for erectile dysfunction>. Blood disorders such as leukaemia and sickle cell disease.
Neurogenic disorders such as spinal cord transection. Trauma to penis resulting in arterio-venous malformations

Tests: Exclude sickle cell/ leukaemia. Consider blood sampling from cavernosa to determine whether high or low flow (low flow
is often hypoxic)

Management: Ice packs/ cold showers. If due to low flow then blood may be aspirated from copora or try intracavernosal alpha
adrenergic agonists. Delayed therapy of low flow priaprism may result in erectile dysfunction.
Theme: Management of urinary obstruction

A. Discharge
B. Start oxybutynin
C. Intravenous antibiotics
D. Urethral catheter
E. Emergency nephrostomy
F. Antegrade ureteric stent
G. Retrograde ureteric stent

What is the best management for the scenario given? Each option may be used once, more than once or not at all.

33. A 68 year old man has a TCC of the bladder. He has a right hydronephrosis detected on ultrasound and deteriorating
renal function. A DMSA scan shows a non functioning left kidney.

The correct answer is Antegrade ureteric stent. A TCC occluding the ureteric orifice will obscure its identification during
surgery, so that passage of a retrograde stent is difficult. Therefore passage of a stent from the renal pelvis is preferable.

34. A 52 year old male with hypercalcaemia secondary to primary hyperparathyroidism presents with renal colic. Multiple
attempts at stone extraction are performed. However, the stone could not be removed. He is now septic with a pyrexia of
39.5 oc.

Emergency nephrostomy. The likely scenario is that this man has developed a calculus causing ureteric obstruction. The
stagnant column of urine can become colonised and infected. An infected obstructed system is one of the few true urological
emergencies. A nephrostomy is needed as the stone could not be removed.

35. A 56 year old man is admitted with acute retention of urine. He has had a recent urinary tract infection. An USS shows
bilateral hydronephrosis.

Urethral catheter. Establishing bladder drainage will often correct the situation. These patients often have a significant diuresis
with associated electrolyte disturbance.

Hydronephrosis: Causes of hydronephrosis

Unilateral: PACT: Pelvic-ureteric obstruction (congenital or acquired). Aberrant renal vessels. Calculi. Tumours of renal pelvis

Bilateral: SUPER: Stenosis of the urethra. Urethral valve. Prostatic enlargement. Extensive bladder tumour. Retro-peritoneal
fibrosis

Investigation: USS- identifies presence of hydronephrosis and can assess the kidneys. IVU- assess the position of the obstruction.
Antegrade or retrograde pyelography- allows treatment. if suspect renal colic: CT scan (majority of stones are detected this way)

Management: Remove the obstruction and drainage of urine. Acute upper urinary tract obstruction: Nephrostomy tube. Chronic
upper urinary tract obstruction: Ureteric stent or a pyeloplasty

36. A 45 year old Accountant is involved in a road traffic accident. He complains of pain in his groin. On examination his BP
is 110/60mmHg, HR 80 bpm and saturations are 99%. He has tenderness in his groin area and there is blood at the
entrance of his urethral meatus. A subsequent pelvic x-ray shows a pelvic fracture. What is the next best managment
step?
A. Urgent urethral catheterisation
B. Conservative management
C. Urgent suprapubic catheterisation
D. Retrograde urethrogram
490
E. Cystoscopy
Answer: C
Males with pelvic fracture and blood at the urethral meatus must not have a urethral catheter placed until a retrograde
urethrogram can rule out urethral disruption. The correct management will therefore be a suprapubic catheter insertion pending
more definitive urethral imaging.

Theme: Haematuria

A. Transitional cell carcinoma of the bladder


B. Squamous cell carcinoma of the bladder
C. Renal cell carcinoma
D. Polycystic kidney disease
E. Wilms tumour
F. Neuroblastoma
G. Benign prostatic hyperplasia

Please select the most likely cause of haematuria for the scenarios given. Each option may be used once, more than once or not at
all.

37. A 40 year old women is being investigated for haematuria. She was living with her sister who has just died from a sub
arachnoid haemorrhage. The haematuria is painless and she has mild renal impairment.

The correct answer is Polycystic kidney disease. This is likely to be polycystic kidney disease as she has renal failure and family
history of sub arachnoid haemorrhage.

38. A 75 year old lady is investigated for episodes of painless haematuria. Apart from COPD from long term smoking she is
otherwise well. She has no other urinary symptoms.

Transitional cell carcinoma of the bladder. TCC commonly presents with painless haematuria that may be detected during testing
carried out for other reasons.

39. A 78 year old man has a long history of nocturia, urinary frequency and terminal dribbling. He was admitted with
urinary retention and was catheterised. On removal of the catheter he has noticed some haematuria.

Benign prostatic hyperplasia. The symptoms are typical for prostatic disease.

40. A 65 year old man presents with significant lower urinary tract symptoms and is diagnosed as having benign prostatic
hyperplasia. Which of the following drug treatments will produce the slowest clinical response?
A. Tamsulosin
B. Alfuzosin
C. Doxazosin
D. Finasteride
E. Terazosin
Answer: D
5 alpha reductase inhibitors have a more favorable side effect profile than alpha blockers.

Alpha blockers have a faster onset of action (but lower reduction of complications from BPH) than 5 alpha reductase inhibitors.

Benign Prostatic Hyperplasia: Prostatic enlargement occurs in many elderly men . >90% of men aged over 80 will have at least
microscopic evidence of benign prostatic hyperplasia

Pathology: As part of the hyperplastic process increase in both stromal and glandular components are seen. The changes are most
notable in the central and periurethral region of the gland.

Presentation: The vast majority of men will present with lower urinary tract symptoms. These will typically be: Poor flow.
Nocturia. Hesitancy. Incomplete and double voiding. Terminal dribbling. Urgency. Incontinence

Investigation: Digital rectal examination to assess prostatic size and morphology. Urine dipstick for infections and haematuria.
Uroflowmetry (a flow rate of >15ml/second helps to exclude BOO). Bladder pressure studies may help identify detrusor failure

491
and whilst may not form part of first line investigations should be included in those with atypical symptoms and prior to redo
surgery. Bladder scanning to demonstrate residual volumes. USS if high pressure chronic retention.

Management: Lifestyle changes such as stopping smoking and altering fluid intake may help those with mild symptoms. Medical
therapy includes alpha blockers and 5 alpha reductase inhibitors. The former work quickly on receptor zones located at the bladder
neck. Cardiovascular side effects are well documented. The latter work on testosterone metabolising enzymes. Although they have
a slower onset of action, the 5 alpha reductase inhibitors may prevent acute urinary retention. Surgical therapy includes
transurethral resection of the prostate and is the treatment of choice in those with severe symptoms and those who fail to respond
to medical therapy. More tailored bladder neck incision procedures may be considered in those with small prostates. Retrograde
ejaculation may occur following surgery. The change in the type of irrigation solutions used has helped to minimise the TURP
syndrome of electrolyte disturbances.

Theme: Testicular disorders

A. Haematocele
B. Epididymal cyst
C. Hydrocele
D. Testicular torsion
E. Orchitis
F. Epididymo-orchitis

Please select the most likely diagnosis for the scenario given. Each option may be used once, more than once or not at all.

41. A 20 year old complains of severe pain and swelling of the scrotum after a cystoscopy. He had mumps as a child. The
testis is tender. The urine dipstick is positive for leucocytes.

Epididymo-orchitis. Epididymo-orchitis: acute pain and swelling after urological intervention. To differentiate from testicular
torsion there is usually pyrexia and positive urine dipstick.

42. A 20 year old complains of severe pain in the right scrotal area after jumping onto his moped. He has also noticed
discomfort in this area over the past few months. On examination there is a swollen, painful testis that is drawn up into
the groin.

Testicular torsion. Testicular torsion: Severe pain which can be spontaneous or precipitated by minor trauma. There is usually
severe pain and the patient will often not tolerate the testis being touched. Urgent scrotal exploration is indicated. It is associated
with a high investment of the the tunica vaginalis with horizontal testicular lie, or when the epididymis and testis are separated
by a mesorchium, in which case the twist occurs at that point.

43. An 8 year old presents with scrotal swelling. He has just recovered from an acute viral illness with swelling of the parotid
glands. On examination both testes are tender and slightly swollen.

Orchitis. Orchitis may be associated with mumps viral infections.

Theme: Renal imaging

A. Non contrast abdominal CT scan


B. DMSA scan
C. PET/CT scan
D. MAG 3 Renogram
E. Renal ultrasound scan
F. DTPA Scan
G. Micturating cystourethrogram
H. Intra venous urography

Please select the most appropriate imaging modality for the scenario descrived. Each agent may be used once, more than once or
not at all.

44. A 43 year old female has undergone a renal transplant 12 months previously. Over the past few weeks there have been
concerns about deteriorating renal function.

The correct answer is MAG 3 Renogram. Because it is excreted by renal tubular cells a MAG 3 renogram provides excellent
492
imaging of renal function and is often used in investigating failing transplants. Theme from 2010 Exam
Similar theme September 2012 Exam

45. A 5 year old boy presents with recurrent urinary tract infections and left sided loin pain. On investigation he is found to
have a left sided PUJ obstruction, there are concerns that he may have developed renal scarring.

DMSA scan. Although MAG 3 renograms may provide some information relating to the structural integrity of the kidney, many
still consider a DMSA scan to be the gold standard for the detection of renal scarring (which is the main concern in PUJ
obstruction and infections).

46. A 17 year old man is referred to the urology clinic. As a child he was diagnosed as having a right sided PUJ obstruction.
However, he was lost to follow up. Over the past 7 months he has been complaining of recurrent episodes of right loin
pain. A CT scan shows considerable renal scarring.

The correct answer is MAG 3 Renogram. In patients with long standing PUJ obstruction and renal scarring the main diagnostic
question is whether the individual has sufficient renal function to consider a pyeloplasty or whether a primary nephrectomy is
preferable. Since the CT has demonstrated scarring there is no use in obtaining a DMSA scan. Of the investigations listed both a
DTPA and MAG 3 renogram will allow assessment of renal function. However, MAG 3 is superior in the assessment of renal
function in damaged kidneys (as it is subjected to tubular secretion).

Functional renal imaging: DMSA scan: Dimercaptosuccinic acid (DMSA) scintigraphy. DMSA localises to the renal cortex
with little accumulation in the renal papilla and medulla. It is useful for the identification of cortical defects and ectopic or
abhorrent kidneys. It does not provide useful information on the ureter of collecting system.

Diethylene-triamine-penta-acetic acid (DTPA): This is primarily a glomerular filtration agent. It is most useful for the
assessment of renal function. Because it is filtered at the level of the glomerulus it provides useful information about the GFR.
Image quality may be degraded in patients with chronic renal impairment and derangement of GFR.

MAG 3 renogram: Mercaptoacetyle triglycine is an is extensively protein bound and is primarily secreted by tubular cells rather
than filtered at the glomerulus. This makes it the agent of choice for imaging the kidneys of patients with existing renal
impairment (where GFR is impaired).

Micturating cystourethrogram (MCUG scan): This scan provides information relating to bladder reflux and is obtained by
filling the bladder with contrast media (via a catheter) and asking the child to void. Images are taken during this phase and the
degree of reflux can be calculated

Intra venous urography: This examination is conducted by the administration of intravenous iodinated contrast media. The agent
is filtered by the kidneys and excreted and may provide evidence of renal stones or other structural lesions. A rough
approximation of renal function may be obtained using the technique. But it is not primarily a technique to be used for this
purpose. With the advent of widespread non contrast CT scan protocols for the detection of urinary tract calculi it is now rarely
used.

PET/CT: This may be used to evaluate structurally indeterminate lesions in the staging of malignancy.

47. A 35-year-old female is admitted to hospital with hypovolaemic shock. CT abdomen reveals a haemorrhagic lesion in the
right kidney. Following surgery and biopsy this is shown to be an angiomyolipomata. What is the most likely underlying
diagnosis?
A. Neurofibromatosis
B. Budd-Chiari syndrome
C. Hereditary haemorrhagic telangiectasia
D. Von Hippel-Lindau syndrome
E. Tuberous sclerosis
Answer: E

Tuberous sclerosis: Tuberous sclerosis (TS) is a genetic condition of autosomal dominant inheritance. Like neurofibromatosis,
the majority of features seen in TS are neuro-cutaneous

Cutaneous features: depigmented 'ash-leaf' spots which fluoresce under UV light. roughened patches of skin over lumbar spine
(Shagreen patches). adenoma sebaceum: butterfly distribution over nose. fibromata beneath nails (subungual fibromata). café-au-
lait spots* may be seen
493
Neurological features: developmental delay. epilepsy (infantile spasms or partial). intellectual impairment

Also: retinal hamartomas: dense white areas on retina (phakomata). rhabdomyomas of the heart. gliomatous changes can occur in
the brain lesions. polycystic kidneys, renal angiomyolipomata. *these of course are more commonly associated with
neurofibromatosis. However a 1998 study of 106 children with TS found café-au-lait spots in 28% of patients

48. A 22 year old man is participating in vigorous intercourse and suddenly feels a snap and his penis becomes swollen and
painful. The admitting surgeon suspects a penile fracture. Which of the following is the most appropriate initial
management?
A. MRI scan of the penis
B. Immediate surgical exploration
C. CT scan of the penis
D. USS of the penis
E. Cystogram
Answer: B
Suspected penile fractures should be surgically explored and the injury repaired.

Penile fractures are a rare type of urological trauma that may be encountered. The injury is usually in the proximal part of the
penile shaft and may involve the ureter. A classically history of a snapping sensation followed by immediate pain is usually given
by the patient (usually during vigourous intercourse). On examination there is usually a tense haematoma and blood may be seen
at the meatus if the urethra is injured.
When there is a a strong suspiscion of the diagnosis the correct management is surgical and a circumferential incision made
immediately inferior to the glans. The skin and superficial tissues are stripped back and the penile shaft inspected. Injuries are
usually sutured and the urethra repaired over a catheter.

49. Which of the following does not cause red urine?


A. Rifampicin
B. Phosphaturia
C. Beetroot
D. Rhubarb
E. Blackberries
Answer: B
Phosphaturia causes cloudy urine.

50. From the list below, which drug is known to cause haemorrhagic cystitis?
A. Rifampicin
B. Methotrexate
C. Dexamethasone
D. Leflunomide
E. Cyclophosphamide
Answer: E
Cyclophosphamide is metabolised into a toxic metabolite acrolein. The effects may be attenuated by administration of large
volumes of intravenous fluids and mesna (which neutralises the metabolite). The condition may be managed initially by bladder
catheterisation and irrigation.

Theme: Haematuria

A. Squamous cell carcinoma of the renal pelvis


B. Renal adenocarcinoma
C. Nephroblastoma
D. Retroperitoneal fibrosis
E. Transitional cell carcinoma of the renal pelvis
F. Retroperitoneal sarcoma

Please select the most likely lesion for the scenario given. Each option may be used once, more than once or not at all.

51. A 72 year old man presents with haematuria which is recurrent. On investigation a retrograde pyelogram shows multiple
ureteric filling defects and the renal pelvis is irregular.

Transitional cell carcinoma of the renal pelvis. TCC of the renal pelvis may seed down the ureter.
494
52. An 83 year old man with a long standing staghorn calculus presents with recurrent haematuria and investigation shows a
mass of the left renal pelvis.

The correct answer is Squamous cell carcinoma of the renal pelvis. SCC of the kidney usually arises in an area of chronic
inflammation such as a staghorn calculus.

53. 28 year old man presents to his GP with haematuria and on examination is noted to have a varicocele. He was noted to
have renal colic 8 weeks ago which was secondary to hypercalcaemia.

Renal adenocarcinoma. Renal adenocarcinoma on the left side may invade the gonadal vein and produce varicocele. They also
have paraneoplastic phenomena such as hypercalcaemia.

Theme: Urinary incontinence

A. Bladder diary for 3 days


B. Urodynamic studies
C. Bladder drill training for 6 weeks
D. Pelvic floor exercises 3 months
E. Oxybutynin
F. IV urography
G. Urinary dye studies
H. None of the above

Choose the best management option for each clinical scenario. Each option may be used once, more than once or not at all.

54. A 34 year old woman from Africa presents with continuous dribbling incontinence after having her 2nd child. Apart
from prolonged labour the woman denies any complications related to her pregnancies. She is normally fit and well.

The correct answer is Urinary dye studies. Vesicovaginal fistulae should be suspected in patients with continuous dribbling
incontinence after prolonged labour and from a country with poor obstetric services. A dye stains the urine and hence identifies
the presence of a fistula.

55. A 56 year woman reports incontinence. She has had a difficult recent hysterectomy and has had 2 previous children with
no complications. She is fit and well.

The correct answer is IV urography. Ureter damage should be suspected in patients with incontinence after pelvic surgery.
Ideally ureter damage should be identified intra operatively, however post operatively IVU is the best test to review the ureter.

56. A 56 year old lady reports incontinence mainly when walking the dog. A bladder diary is inconclusive.

Urodynamic studies. Urodynamic studies are indicated when there is diagnostic uncertainty or plans for surgery.

Urinary incontinence: Involuntary passage of urine. Most cases are female (80%). It has a prevalence of 11% in those aged
greater than 65 years. The commonest variants include: Stress urinary incontinence (50%). Urge incontinence (15%). Mixed
(35%)

Males: Males may also suffer from incontinence although it is a much rarer condition in men. A number of anatomical factors
contribute to this. Males have 2 powerful sphincters; one at the bladder neck and the other in the urethra. Damage to the bladder
neck mechanism is a factor in causing retrograde ejaculation following prostatectomy. The short segment of urethra passing
through the urogenital diaphragm consists of striated muscle fibres (the external urethral sphincter) and smooth muscle capable of
more sustained contraction. It is the latter mechanism that maintains continence following prostatectomy.

Females: The sphincter complex at the level of bladder neck is poorly developed in females. As a result the external sphincter
complex is functionally more important, its composition being similar to that of males. Innervation is via the pudendal nerve and
the neuropathy that may accompany obstetric events may compromise this and lead to stress urinary incontinence.
495
Innervation: Somatic innervation to the bladder is via the pudendal, hypogastric and pelvic nerves. Autonomic nerves travel in
these nerve fibres too. Bladder filling leads to detrusor relaxation (sympathetic) coupled with sphincter contraction. The
parasympathetic system causes detrusor contraction and sphincter relaxation. Overall control of micturition is centrally mediated
via centres in the Pons.

Stress urinary incontinence 50% of cases, especially in females. Damage (often obstetric) to the supporting structures
surrounding the bladder may lead to urethral hypermobility. Other cases due to sphincter dysfunction, usually from neurological
disorders (e.g. Pudendal neuropathy, multiple sclerosis).

Urethral mobility: Pressure not transmitted appropriately to the urethra resulting in involuntary passage of urine during episodes
of raised intra-abdominal pressure.

Sphincter dysfunction: Sphincter fails to adapt to compress urethra resulting in involuntary passage of urine. When the sphincter
completely fails there is often to continuous passage of urine.

Urge incontinence: In these patients there is sense of urgency followed by incontinence. The detrusor muscle in these patients is
unstable and urodynamic investigation will demonstrate overactivity of the detrusor muscle at inappropriate times (e.g. Bladder
filling). Urgency may be seen in patients with overt neurological disorders and those without. The pathophysiology is not well
understood but poor central and peripheral co-ordination of the events surrounding bladder filling are the main processes.

Assessment: Careful history and examination including vaginal examination for cystocele. Bladder diary for at least 3 days.
Consider flow cystometry if unclear symptomatology or surgery considered and diagnosis is unclear. Exclusion of other organic
disease (e.g. Stones, UTI, Cancer)

Management: Conservative measures should be tried first; Stress urinary incontinence or mixed symptoms should undergo 3
months of pelvic floor exercise. Over active bladder should have 6 weeks of bladder retraining. Drug therapy for women with
overactive bladder should be offered with oxybutynin if conservative measures fail. In women with detrusor instability who fail
non operative therapy a trial of sacral neuromodulation may be considered, with conversion to permanent implant if good
response. Augmentation cystoplasty is an alternative but will involve long term intermittent self catheterisation. In women with
stress urinary incontinence a urethral sling type procedure may be undertaken. Where cystocele is present in association with
incontinence it should be repaired particularly if it lies at the introitus.

NICE guidelines: Initial assessment urinary incontinence should be classified as stress/urge/mixed. At least 3/7 bladder diary if
unable to classify easily. Start conservative treatment before urodynamic studies if a diagnosis is obvious from the history.
Urodynamic studies if plans for surgery. Stress incontinence: Pelvic floor exercises 3/12, if fails consider surgery. Urge
incontinence: Bladder training >6/52, if fails for oxybutynin (antimuscarinic drugs) then sacral nerve stimulation. Pelvic floor
exercises offered to all women in their 1st pregnancy.

57. A 72 year old man presents with lower urinary tract symptoms. On digital rectal examination benign prostatic
hyperplasia is diagnosed. Which of the following treatments is associated with a reduction in the risk of urinary
retention?
A. Alfuzosin
B. Finasteride
C. Prazosin
D. Tamsulosin
E. Terazosin
Answer: B
5 alpha reductase inhibitors reduce the risk of urinary retention.
In the PLESS study, data show a reduction in the risk of urinary retention although the absolute risk reduction was small.

58. A 22 year old man is involved in a road traffic accident. He is found to have a pelvic fracture. While on the ward the
nursing staff report that he is complaining of lower abdominal pain. On examination you find a distended tender
bladder. What is the best management?
A. 10 Ch foley urethral catheter
B. Suprapubic catheter
C. 16 Ch foley urethral catheter
D. 18 Ch coude tip urethral catheter
E. Pain relief and review in 1 hour
Answer: B
This patient has possible urethral injury based on the history. Urethral catheterisation is contraindicated in this situation.

496
59. Which of the following procedures represents the optimal operative procedure for testicular cancer?
A. Lords procedure
B. Orchidectomy via a scrotal approach
C. Orchidectomy via inguinal approach
D. Orchidectomy via a combined inguino-scrotal approach
E. None of the above
Answer: C
Testicular tumours metastasise to Para aortic nodes and thus an inguinal rather than scrotal approach should be used. There are
two main operations that are termed Lords procedure; one is for fissure in ano and the other is a procedure for hydrocele.

Risk factors for testicular cancer: Cryptorchidism. Infertility. Family history. Klinefelter's syndrome. Mumps orchitis

Features: A painless lump is the most common presenting symptom. Pain may also be present in a minority of men. Other
possible features include hydrocele, gynaecomastia

Diagnosis: Ultrasound is first-line. CT scanning of the chest/ abdomen and pelvis is used for staging. Tumour markers (see above)
should be measured

Management: Orchidectomy (Inguinal approach). Chemotherapy and radiotherapy may be given depending on staging.
Abdominal lesions >1cm following chemotherapy may require retroperitoneal lymph node dissection.

Prognosis is generally excellent: 5 year survival for seminomas is around 95% if Stage I. 5 year survival for teratomas is around
85% if Stage I

Theme: Management of testicular disorders

A. Fine needle aspiration cytology


B. Tru Cut biopsy
C. Orchidectomy via inguinal approach
D. Orchidectomy via scrotal approach
E. Administration of antibiotics
F. Exploration of scrotum via scrotal approach
G. Reassure and discharge

Please select the most appropriate management option for the scenario given. Each option may be used once, more than once or
not at all.

60. A 22 year old man presents with an aching pain and discomfort in his right testicle. He has felt systemically unwell for
the preceding 48 hours. On examination there is tenderness of the right testicle. He has an exaggerated cremasteric
reflex.

Administration of antibiotics. This is likely to represent epididymo-orchitis, this is usually due to infection with gonorrhoea or
chlamydia in this age group. In addition to treatment with antibiotics contact tracing and appropriate swabs should also be
performed.

61. A 25 year old man presents with aching and discomfort of his right testicle. He has felt generally unwell and lethargic
over the past few weeks. On examination there is a small nodule palpable in the testis, on ultrasound this is hypoechoic,
systematic examination demonstrates supraclavicular lymphadenopathy.

Orchidectomy via inguinal approach. Hypoechoic masses within the testicle in the context are most likely to represent
malignancy. He should have a staging CT scan, thereafter an orchidectomy should be performed via an inguinal approach.
Percutaneous biopsy has no role in the management of testicular malignancy.

497
62. A 15 year old boy is woken from sleep by a severe pain in his left testicle. He was previously fit and well. On examination
he has a tender left testicle with an absent cremasteric reflex.

Exploration of scrotum via scrotal approach. This is likely to represent torsion, surgical exploration is warranted.

Theme: Haematuria

A. Interstitial nephritis
B. Membranous glomerulonephritis
C. Endometriosis
D. Placenta percreta
E. Adult polycystic kidney disease
F. Renal vein thrombosis
G. Urinary tract infection

Please select the most likely cause for haematuria for the scenario described. Each option may be used once, more than once or
not at all.

63. A 22 year female who is 24 weeks pregnant presents with frank haematuria. She is sexually active. She has had a
previous pregnancy resulting in caesarean section.

Placenta percreta. Pregnancy and frank haematuria, especially if there is a history of placenta previa or prior caesarean section,
should indicate this diagnosis. There is invasive placental implantation into the myometrium, which can rarely extend into the
bladder causing severe bleeding.

64. A 22 year old woman presents with macroscopic haematuria. She is sexually active. She is known to have renal calculi
and had a berry aneurysm clipped.

The correct answer is Adult polycystic kidney disease. APKD is associated with liver cysts (70%), berry aneurysms (25%) and
pancreatic cysts (10%). Patients may have a renal mass, hypertension, renal calculi and macroscopic haematuria.

65. A 45 year woman presents with haematuria. She has a temperature of 38 oC and is found to have a Hb 17. Her urine
dipstick shows nitrates and 3+ blood. Blood and urine cultures are negative.

The correct answer is Renal vein thrombosis. Renal vein thrombosis is a common feature of renal cell carcinoma as it invades
the renal vein. Other features include PUO, left varicocele and paraneoplastic endocrine effects due to erythropoietin factor,
renin, ACTH and PTH like substance.

Theme: Testicular disorders

A. Antibiotics
B. Aspiration
C. Testicular exploration after 6 hours
D. Testicular exploration within 6 hours
E. Orchidectomy via inguinal approach
F. Orchidectomy via scrotal approach
G. No treatment needed

For each scenario please select the most appropriate management. Each option may be used once, more than once or not at all.

66. 20 year old complains of severe pain and swelling of the scrotum after a cystoscopy. He had mumps as a child. The testis
is tender. The urine dipstick is positive for leucocytes.

Antibiotics. Epididymo-orchitis: acute pain and swelling after urological intervention. To differentiate from testicular torsion
there is usually pyrexia and positive urine dipstick. Chlamydia is common in young males, gonorrhoea and E. coli are commoner
in older patients.

67. A 20 year old complains of severe pain in the right scrotal area after jumping onto his moped. He has noticed discomfort
intermittently in this area over the past few months. Clinically the testis is tender.

498
The correct answer is Testicular exploration within 6 hours. Testicular torsion: Severe pain which can be spontaneous or
precipitated by minor trauma. The patient may have noticed pain previously. Surgical intervention is needed as soon as possible
to prevent the risk of loss of the testis.

68. A 23 year old man presents with a lump of the left testicle. Uss confirms a mass lesion and serum bHCG is raised

Orchidectomy via inguinal approach. This is likely to represent a teratoma

69. A 55 year old man presents with an episode of frank haematuria and on investigation is found to have a T2 transitional
cell carcinoma of the bladder. Hist staging investigations are negative for metastatic disease. What is the most
appropriate treatment?
A. Radical cystectomy
B. Palliative radiotherapy
C. Intravesical BCG
D. Intravesical mitomycin C
E. Intravesical cisplatin
Answer: A
70. T2 lesions in a young fit patient are best managed surgically. Up to 25 % patients may develop perioperative complications.
However, palliative treatments and intravesical chemotherapy (which does NOT include cisplatin) are not used curatively in this
situation.

Bladder cancer is the second most common urological cancer. It most commonly affects males aged between 50 and 80 years of
age. Those who are current, or previous (within 20 years), smokers have a 2-5 fold increased risk of the disease. Occupational
exposure to hydrocarbons such as alanine increases the risk. Although rare in the UK, chronic bladder inflammation arising from
Schistosomiasis infection remains a common cause of squamous cell carcinomas, in those countries where the disease is
endemic.

Benign tumours of the bladder including inverted urothelial papilloma and nephrogenic adenoma are uncommon. Bladder
malignancies: Transitional cell carcinoma (>90% of cases). Squamous cell carcinoma ( 1-7% -except in regions affected by
schistosomiasis). Adenocarcinoma (2%)

Transitional cell carcinomas may arise as solitary lesions, or may be multifocal, owing to the effect of "field change" within the
urothelium. Up to 70% of TCC's will have a papillary growth pattern. These tumours are usually superficial in location and
accordingly have a better prognosis. The remaining tumours show either mixed papillary and solid growth or pure solid growths.
These tumours are typically more prone to local invasion and may be of higher grade, the prognosis is therefore worse. Those with
T3 disease or worse have a 30% (or higher) risk of regional or distant lymph node metastasis.

TNM Staging: Stage: Description


T0 No evidence of tumour
Ta Non invasive papillary carcinoma
T1 Tumour invades sub epithelial connective tissue
T2a Inner half of detrusor invaded
T2b Outer half of detrusor invaded
T3 Tumour extends to perivesical fat
T4a Invasion of uterus, prostate or bowel
T4b Invasion of other abdominal organs
N0 No nodal disease
N1 Single lymph node metastasis (up to 2cm)
N2 Single node >2cm or multiple nodes up to 5cm
N3 Nodes over 5cm
M1 Distant disease

Presentation: Most patients (85%) will present with painless, macroscopic haematuria. In those patients with incidental
microscopic haematuria, up to 10% of females aged over 50 will be found to have a malignancy (once infection excluded).

Staging: Most will undergo a cystoscopy and biopsies or TURBT, this provides histological diagnosis and information relating to
depth of invasion. Locoregional spread is best determined using pelvic MRI and distant disease CT scanning. Nodes of uncertain
significance may be investigated using PET CT.

Treatment: Those with superficial lesions may be managed using TURBT in isolation. Those with recurrences or higher grade/
risk on histology may be offered intravesical chemotherapy. Those with T2 disease are usually offered either surgery (radical
cystectomy and ileal conduit) or radical radiotherapy.
499
Prognosis
T1 90%
T2 60%
T3 35%
T4a 10-25%
Any T, N1-N2 30%

71. A 35-year-old homosexual man is referred to the colorectal clinic with rectal pain and tenesmus. On examination you
note painful inguinal lymphadenopathy and a solitary painless penile ulcer. What is the most likely diagnosis?
A. Herpes simplex infection
B. HIV infection
C. Granuloma inguinale
D. Chancroid
E. Lymphogranuloma venereum
Answer: E

Genital ulcers: painful: herpes >> chancroid. Painless: syphilis > lymphogranuloma venereum + granuloma inguinale

Lymphogranuloma venereum usually involves three stages: 1 - small painless pustule which later forms an ulcer. 2 - painful
inguinal lymphadenopathy. 3 - proctocolitis

Theme: Urinary incontinence

A. Bladder diary for 3 days


B. Urodynamic studies
C. Bladder drill training for 6 weeks
D. Pelvic floor exercises for 3 months
E. Oxybutynin
F. IV urography
G. Dyes studies including phenazopyridine
H. None of the above

Choose the best management option for each clinical scenario. Each option may be used once, more than once or not at all.

72. A 75 year old lady reports urinary incontinence when coughing and sneezing. She has had 2 children with no
complications. She has no significant past medical history and is on no medications. What is the most appropriate initial
management?

Pelvic floor exercises for 3 months. A diagnosis of stress incontinence is obvious from the history, therefore there is no need for
a bladder diary or urodynamic studies.
Pelvic floor exercises would be the first line management.

73. A 26 year old pregnant woman having her 1st child and has never had problems with incontinence.

The correct answer is Pelvic floor exercises for 3 months. Pregnant women should receive instructions as to how to perform
pelvic floor exercises during pregnancy as this may help to decrease subsequent risk of stress urinary incontinence.

74. A 67 year old lady reports urinary incontinence. She describes the sensation of needing to pass urine immediately. She
has had 2 children and is on no medications. What is the most appropriate initial management?

The correct answer is Bladder diary for 3 days. The patient appears to be describing urge incontinence. A bladder diary is
needed to establish the baseline frequency of micturition and amounts of urine passed. Then bladder training can be initiated to
increase the volume of urine passed at reduced frequencies.

75. A 47-year-old woman presents with loin pain and haematuria. Urine dipstick demonstrates: Blood ++++, Nitrites
POS, Leucocytes +++, Protein ++. Urine culture shows a Proteus infection. An x-ray demonstrates a stag-horn
calculus in the left renal pelvis. What is the most likely composition of the renal stone?
A.Xanthine
B.Calcium oxalate
C.Struvite
500
D.Cystine
E.Urate
Answer: C
Stag-horn calculi are composed of struvite and form in alkaline urine (ammonia producing bacteria therefore predispose). Renal
stones on x-ray: cystine stones: semi-opaque. Urate + xanthine stones: radio-lucent
Renal stones: imaging: The table below summarises the appearance of different types of renal stone on x-ray
Type Frequency Radiograph appearance
Calcium oxalate 40% Opaque
Mixed calcium oxalate/phosphate stones 25% Opaque
Triple phosphate stones* 10% Opaque
Calcium phosphate 10% Opaque
Urate stones 5-10% Radio-lucent
Cystine stones 1% Semi-opaque, 'ground-glass' appearance
Xanthine stones <1% Radio-lucent

*stag-horn calculi involve the renal pelvis and extend into at least 2 calyces. They develop in alkaline urine and are composed of
struvite (ammonium magnesium phosphate, triple phosphate). Ureaplasma urealyticum and Proteus infections predispose to their
formation
Theme: Management of prostatic disease

A. Commence alpha blocker


B. Commence 5 alpha reductase inhibitor
C. Transurethral resection of the prostate
D. Commence LnRH analogue
E. Radical prostatectomy
F. Transvesical prostatectomy
G. Unilateral orchidectomy
H. Watch and wait
I. Radical radiotherapy

For the prostatic disorders described please select the most appropriate management option. Each option may be used once, more
than once or not at all.

76. A 49 year old man presents with a single episode of haematuria. Investigations demonstrate adenocarcinoma of the
prostate gland. Imaging shows T2 disease and no evidence of metastasis.

Radical prostatectomy. In a young patient with local disease only a radical prostatectomy is the best chance of cure.
Radiotherapy may be given instead but has long term sequelae. A transvesical prostatectomy is a largely historical operation
performed for BPH before TURP was established.

77. A 72 year old man is admitted with acute urinary retention. On examination he has a small but palpable bladder. Digital
rectal examination identifies a benign feeling enlarged prostate gland. He has been treated with finasteride for the past 9
months.

Transurethral resection of the prostate.Medical therapy has failed and although an alpha blocker may help his symptoms he
would fare better with a TURP.

78. A 73 year old man presents with haematuria. Investigations demonstrate a localised prostatic disease. He suffers from
mild COPD but otherwise has no major co-morbidities. His staging investigations show no evidence of metastatic disease

The correct answer is Radical radiotherapy. Localised disease with minor medical co-morbidities would attract a
recommendation for radical radiotherapy. The long term follow up data does show complications related to radical radiotherapy
and those with more advanced co-morbidities should probably be managed medically.

Surgical Law
1. What is the opposite of relative risk?
A. P value
B. Odds ratio
501
C. Likelihood ratio
D. Absolute risk reduction
E. Number needed to treat
Answer: E

The number needed to treat is the average number of patients who need to be treated to prevent one adverse outcome. It is is the
inverse of relative risk (absolute risk reduction). The high the NNT value is, the less effective the treatment. The ideal NNT is 1.
Theme from September 2012 Exam

Relative risk: Relative risk (RR) is the ratio of risk in the experimental group (experimental event rate, EER) to risk in the
control group (control event rate, CER)
To recap: EER = rate at which events occur in the experimental group. CER = rate at which events occur in the control group. For
example, if we look at a trial comparing the use of paracetamol for back pain compared to placebo we may get the following
results
Total number of patients Experienced significant pain relief
Paracetamol 100 60
Placebo 80 20

Experimental event rate, EER = 60 / 100 = 0.6. Control event rate, CER = 20 / 80 = 0.25. Therefore the relative risk = EER / CER
= 0.6 / 0.25 = 2.4

If the risk ratio is > 1 then the rate of an event (in this case experiencing significant pain relief) is increased compared to controls.
It is therefore appropriate to calculate the relative risk increase if necessary (see below). If the risk ratio is < 1 then the rate of an
event is decreased compared to controls. The relative risk reduction should therefore be calculated (see below).
Relative risk reduction (RRR) or relative risk increase (RRI) is calculated by dividing the absolute risk change by the control
event rate. Using the above data, RRI = (EER - CER) / CER = (0.6 - 0.25) / 0.25 = 1.4 = 140%

2. What is the reciprocal of absolute risk reduction?


A. Odds ratio
B. Number needed to treat
C. False positive
D. False negative
E. None of the above
Answer: B
Similar theme to September 2011 Exam. In epidemiology, the absolute risk reduction, or risk difference is the decrease in risk of
a given activity or treatment in relation to a control activity or treatment. It is the inverse of the number needed to treat.

Absolute risk reduction: The absolute risk reduction is the decrease in risk of a given activity or treatment in relation to a
control activity or treatment. It is the inverse of the number needed to treat.
The absolute risk reduction is usually calculated for two different treatments. For example, consider surgical resection (X)
versus watchful waiting (Y) for prostate cancer. A defined end point, such as 5 year survival is required. If the probabilities pX
and pY of this end point are known then the absolute risk reduction is calculated (pX-pY). The inverse of absolute risk reduction
is the Number Needed to Treat . This is useful in determining the cost Vs benefit of many treatments.

Number needed to treat: Definition: how many patients would be need to receive a treatment to prevent one event. It is the
absolute difference between two treatments.

3. You have been asked to investigate the potential benefit of setting up a service to help patients with stomas in the local
area. What is the most important factor when determining how many resources will be required?
A. Incidence
B. Bayesian factor
C. Prevalence
D. Denominator data
E. P value
Answer: C

Incidence and prevalence: These two terms are used to describe the frequency of a condition in a population. The incidence is
the number of new cases per population in a given time period. For example, if condition X has caused 40 new cases over the
past 12 months per 1,000 of the population the annual incidence is 0.04 or 4%. The prevalence is the total number of cases per
population at a particular point in time. For example, imagine a questionnaire is sent to 2,500 adults asking them how much they
weigh. If from this sample population of 500 of the adults were obese then the prevalence of obesity would be 0.2 or 20%.

Relationship: prevalence = incidence * duration of condition. in chronic diseases the prevalence is much greater than the

502
incidence. in acute diseases the prevalence and incidence are similar. For conditions such as the common cold the incidence may
be greater than the prevalence

4. In medical statistics, which of the following does a p value of 0.04 represent?


A. Risk of type 1 Error
B. Risk of tye 2 Error
C. Size of power of the study
D. Sample size
E. Number of degrees of freedom
Answer: A
P values are related to the significance levels of a statistical test and therefore are in effect measuring the risk of a type 1 error.

Statistical error: Type 1 Error: Test rejects true null hypothesis. Rate of type 1 error and is the given the value of α. It usually
equals the significance level of a test

Type 2 Error: Test fails to reject a false null hypothesis. Rate of type 2 errors is give value of β. It is related to the power of the
test

5. Which of the following statements relating to quantitative data is false?


A. Discrete data cannot be sub divided
B. The median is less susceptible to extreme outliers than the mean
C. The mean is susceptible to extreme outliers
D. Data that fits the standard distribution perfectly will have a mode that is half the value of the mean
E. Values obtained have a numerical scale
Answer: D
Data that fits the standard distribution perfectly will have a mean, median and mode that are all the same value.

Qualitative and quantitative data


Qualitative (categorical) data refers to different descriptions of a characteristic, although it may be possible to allocate a number it
has no scale. Quantitative data is associated with numerical values on a numerical scale. Since quantitative data is based on a
numerical scale it can be organised to create a distribution curve. The central tendency may be estimated using the mode, median
and mean. The standard deviation gives an estimation of the spread of data.

Theme: Consent

A. Consent the patient


B. Consent the parents
C. Proceed without consent
D. Refer the matter to a court
E. Do not proceed with treatment
F. Consent by proxy

For each of the scenarios given please select the most appropriate course of action. Each option may be used once, more than once
or not at all.

6. A 6 year old is hit by a car and is brought to the emergency department. He is haemodynamically unstable with bilateral
femoral shaft fractures and concerns that he may have a ruptured spleen. The parents have refused blood transfusions
on religious grounds.

Proceed without consent. In the UK the GMC and common law advises that emergency life saving treatment can be given to a
child irrespective of the parents views. There is clearly insufficient time here to apply to a court.

7. A 17 year old male motorcyclist is injured in a road traffic accident. He is brought to the emergency department
comatose. He becomes haemodynamically unstable and a ruptured spleen is diagnosed. A laparotomy is proposed.

The correct answer is Consent by proxy. In this situation it is not possible to wait until the patient regains capacity and therefore
consent by proxy is the best way forward. If time permits it is desirable for two separate clinicians (one of whom should be of
consultant status) to agree.

503
8. A 15 year old girl presents with right iliac fossa pain. She becomes progressively tachycardic and a ruptured ectopic
pregnancy is suspected. She is deemed to be mentally competent and will agree to surgery, but not if her parents are
informed.

Consent the patient. Since she is likely to be Gillick competent she can consent for herself.

Consent: There are 3 types of consent: 1. Informed. 2. Expressed. 3. Implied

Consent forms used in UK NHS


Consent Form 1 For competent adults who are able to consent for themselves where consciousness is impaired
Consent Form 2 For an adult consenting on behalf of a child where consciousness is impaired
Consent Form 3 For an adult or child where consciousness is not impaired
Consent Form 4 For adults who lack capacity to provide informed consent

Capacity: Key points include: 1. Understand and retain information. 2. Patient believes the information to be true. 3. Patient is
able to weigh the information to make a decision. All patients must be assumed to have capacity

Bolam test: Considers whether a doctor's decision matches the opinion of a responsible body of doctors skilled in the same
practice.

9. A surgical team wish to conduct a meta analysis of randomised controlled trials of the use of low molecular weight
heparins in the prevention of post operative deep vein thrombosis. How would these results be best displayed
graphically?
A. Forest plot
B. Box Whisker plot
C. Violin plot
D. Kaplan Meier graph
E. None of the above
Answer: A
Data from multiple RCT's are best displayed using Forest plots. Funnel plots may be used to determine the effect of small
studies and their overall effect on the data. Violin plots and Box Whisker plots are often used to graphically display non
parametric data from single studies and are not generally used to display data from meta analyses.

Forest plots: A Forest plot is a graphical display designed to illustrate the relative strength of treatment effects in multiple
quantitative scientific studies, addressing the same question. It is often used to graphically display meta analyses of randomised
controlled trials.

The graph may be plotted on a natural logarithmic scale when using odds ratios or other ratio-based effect measures, so that the
confidence intervals are symmetrical about the means from each study and to ensure undue emphasis is not given to odds ratios
greater than 1 when compared to those less than 1. The area of each square is proportional to the study's weight in the meta-
analysis. The overall meta-analysed measure of effect is often represented on the plot as a vertical line. This meta-analysed
measure of effect is commonly plotted as a diamond, the lateral points of which indicate confidence intervals for this estimate.

A vertical line representing no effect is also plotted. If the confidence intervals for individual studies overlap with this line, it
demonstrates that at the given level of confidence their effect sizes do not differ from no effect for the individual study. The same
applies for the meta-analysed measure of effect: if the points of the diamond overlap the line of no effect the overall meta-
analysed result cannot be said to differ from no effect at the given level of confidence.
A rapid finger-prick blood test to help diagnosis deep vein thrombosis is developed. Comparing the test to current standard
techniques a study is done on 1,000 patients:

DVT present DVT absent


New test positive 200 100
New test negative 20 680

10. What is the specificity of the new test?


A. 680/880
B. 200/220
C. 680/780
D. 680/700
E. 200/300
Answer: C
Specificity = true negatives / (true negatives + false positives) = 680 / (680 + 100)
504
Screening test statistics: It would be unusual for a medical exam not to feature a question based around screening test statistics.
The available data should be used to construct a contingency table as below:

TP = true positive; FP = false positive; TN = true negative; FN = false negative


Disease present Disease absent
Test positive TP FP
Test negative FN TN

The table below lists the main statistical terms used in relation to screening tests:

Sensitivity TP / (TP + FN ) Proportion of patients with the condition who have a positive test
result
Specificity TN / (TN + FP) Proportion of patients without the condition who have a negative
test result
Positive predictive value TP / (TP + FP) The chance that the patient has the condition if the diagnostic test is
positive
Negative predictive value TN / (TN + FN) The chance that the patient does not have the condition if the
diagnostic test is negative
Likelihood ratio for a positive sensitivity / (1 - How much the odds of the disease increase when a test is positive
test result specificity)
Likelihood ratio for a negative (1 - sensitivity) / How much the odds of the disease decrease when a test is negative
test result specificity

Positive and negative predictive values are prevalence dependent. Likelihood ratios are not prevalence dependent

11. A new test to screen for pulmonary embolism (PE) is used in 100 patients who present to the Emergency Department.
The test is positive in 30 of the 40 patients who are proven to have a PE. Of the remaining 60 patients, only 5 have a
positive test. What is the sensitivity of the new test?
A. 8.33%
B. 30%
C. 40%
D. 66.66%
E. 75%
Answer: E
A contingency table can be constructed from the above data, as shown below:
PE diagnosed No PE
Test positive 30 5
Test negative 10 55
The sensitivity is therefore 30 / (30 + 10) = 75%

Screening test statistics: It would be unusual for a medical exam not to feature a question based around screening test statistics.
The available data should be used to construct a contingency table as below:

TP = true positive; FP = false positive; TN = true negative; FN = false negative


Disease present Disease absent
Test positive TP FP
Test negative FN TN

The table below lists the main statistical terms used in relation to screening tests:

Sensitivity TP / (TP + FN ) Proportion of patients with the condition who have a positive test
result
Specificity TN / (TN + FP) Proportion of patients without the condition who have a negative
test result
Positive predictive value TP / (TP + FP) The chance that the patient has the condition if the diagnostic test is
positive
Negative predictive value TN / (TN + FN) The chance that the patient does not have the condition if the
diagnostic test is negative
Likelihood ratio for a positive sensitivity / (1 - How much the odds of the disease increase when a test is positive
test result specificity)
Likelihood ratio for a negative (1 - sensitivity) / How much the odds of the disease decrease when a test is negative
test result specificity

505
Positive and negative predictive values are prevalence dependent. Likelihood ratios are not prevalence dependent
Theme: Consent processes

A. Consent form 1 (competent adult)


B. Consent form 2 (procedure on a child)
C. Consent form 3 (procedure on a child or adult where conciousness not impaired)
D. Consent form 4 (Undertaking procedure where patient lacks capacity)
E. Verbal consent will suffice
F. Research consent form

Please select the most appropriate consent modality for the procedure described. Each option may be used once, more than once or
not at all

12. A 65 year old patient has a wound which is discharging following a laparotomy. The wound is closed with clips and
underlying this is a small superficial collection measuring 2x3cm.

The correct answer is Verbal consent will suffice. This is done on the ward and is a straightforward procedure. Written consent is
not required.

13. An 18 year old male has appendicitis and requires an appendicectomy.

The correct answer is Consent form 1 (competent adult). The patient is 18 so technically an adult where consent is concerned.

14. A 10 year old boy requires removal of a toe nail which has become ingrown and infected. He does not want the operation
performed.

The correct answer is Consent form 3 (procedure on a child or adult where conciousness not impaired). He is a minor.

15. What is the odds ratio of developing appendicitis for people who have had gastroenteritis in the previous year?
A. 0.3
B. 3
C. 5
D. 2.5
E. 3.33
Answer: C
Remember to calculate the odds, rather than risk, initially: Odds of patient with appendicitis having gastroenteritis in the past year
= 30 / 30 = 1.0. Odds of the control group having gastroenteritis in the past year = 10 / 50 = 0.2. The odds ratio therefore = 1 / 0.2
=5
Odds ratio : In studies with binary results (e.g. yes or no) the odds ratio or relative risk is used. OR rate of 1 implies that event
may occur on either group, values of more or less than 1 indicate a skew to one group or the other. The risk ratio is 1 where the
event may occur in either group or less than 1 if it is more likely in one group over the other. Odds are a ratio of the number of
people who incur a particular outcome to the number of people who do not incur the outcome. The odds ratio may be defined as
the ratio of the odds of a particular outcome with experimental treatment and that of control. Odds ratios are the usual reported
measure in case-control studies. It approximates to relative risk if the outcome of interest is rare. For example, if we look at a trial
comparing the use of paracetamol for back pain compared to placebo we may get the following results
Total number of patients Achieved 50% pain relief
Paracetamol 60 40
Placebo 90 30

The odds of achieving significant pain relief with paracetamol = 40 / 20 = 2. The odds of achieving significant pain relief with
placebo = 30 / 60 = 0.5. Therefore the odds ratio = 2 / 0.5 = 4

15. Which one of the following statements best describes a type II statistical error?
A. The p value fails to reach statistical significance
B. The alternative hypothesis is rejected when it is false
C. The null hypothesis is rejected when it is true
D. The null hypothesis is accepted when it is false
E. None of the above
Answer: D

Significance tests: A null hypothesis (H0) states that two treatments are equally effective (and is hence negatively phrased). A
significance test uses the sample data to assess how likely the null hypothesis is to be correct.
506
For example: 'there is no difference in the prevalence of colorectal cancer in patients taking low-dose aspirin compared to those
who are not'. The alternative hypothesis (H1) is the opposite of the null hypothesis, i.e. There is a difference between the two
treatments. The {p value} is the probability of obtaining a result by chance at least as extreme as the one that was actually
observed, assuming that the null hypothesis is true. It is therefore equal to the chance of making a type I error (see below).

Two types of errors may occur when testing the null hypothesis: type I: the null hypothesis is rejected when it is true - i.e.
Showing a difference between two groups when it doesn't exist, a false positive. This is determined against a preset significance
level (termed alpha). As the significance level is determined in advance the chance of making a type I error is not affected by
sample size. It is however increased if the number of end-points are increased. For example if a study has 20 end-points it is likely
one of these will be reached, just by chance.

Type II: the null hypothesis is accepted when it is false - i.e. Failing to spot a difference when one really exists, a false negative.
The probability of making a type II error is termed beta. It is determined by both sample size and alpha

Study accepts H0 Study rejects H0


Reality H0 Type 1 error (alpha)
Reality H1 Type 2 error (beta) Power (1 - beta)

The power of a study is the probability of (correctly) rejecting the null hypothesis when it is false: power = 1 - the probability of a
type II error. Power can be increased by increasing the sample size

16. As part of a research project you are trying to ascertain whether the use of dummies in infants is linked to sudden infant
death syndrome. What is the most appropriate form of study design?
A. Randomised controlled trial
B. Cross-over trial
C. Cross-sectional survey
D. Case-control study
E. Cohort study
Answer: D
As sudden infant death syndrome is relatively rare a case-control design is more appropriate than a cohort study.

Study design: The following highlights the main features of the main types of study:
 Randomised controlled trial: Participants randomly allocated to intervention or control group (e.g. standard treatment or
placebo). Practical or ethical problems may limit use
 Cohort study: Observational and prospective. Two (or more) are selected according to their exposure to a particular agent
(e.g. medicine, toxin) and followed up to see how many develop a disease or other outcome. The usual outcome measure is
the relative risk. Examples include Framingham Heart Study
 Case-control study: Observational and retrospective. Patients with a particular condition (cases) are identified and
matched with controls. Data is then collected on past exposure to a possible causal agent for the condition. The usual
outcome measure is the odds ratio. Inexpensive, produce quick results. Useful for studying rare conditions. Prone to
confounding
 Cross-sectional survey Provide a 'snapshot', sometimes called prevalence studies. Provide weak evidence of cause and
effect

17. Which of the following has the greatest impact on the positive predictive value of a test?
A. Prevalence
B. Subjects who are true negatives
C. Specificity
D. Relative risk
E. None of the above
Answer: A
The positive predictive value (PPV) is the probability that an individual with a positive screening result has the disease. The
sensitivity is the probability that an individual with the disease is screened positive and the specificity is the probability that an
individual without the disease is screened negative. Its value depends upon the prevalence of the condition being tested for and
the sensitivity of the test used. It may be calculated by dividing the number of true positives by the number of true positives and
the number of false positives.

Positive predictive values

507
Screening tests: Sensitivity: proportion of true positives identified by a test. Specificity: proportion of true negatives correctly
identified by a test. Positive predictive value: proportion of those who have a positive test who actually have the disease.
Negative predictive value: proportion of those who test negative who do not have the disease

Predictive values are dependent on the prevalence: Likelihood ratio for a positive test result = sensitivity/(1-specificity).
Likelihood ratio for a negative test result = (1-sensitivity)/specificity

Likelihood ratios are not prevalence dependent

18. Considering cluster randomised trials, which of the following statements is false?
A. They consider interventions targeted at groups
B. They require increased recruitment to achieve the same level of statistical power as individual trials
C. If results are analysed on an individual basis a lower P value may be obtained
D. They are less prone to unit of analyses errors than trials involving individual observations
E. The statistical analyses for these trials is more complex than that required for trials based on individuals
Answer: D
Cluster randomised trials are more prone to unit of analyses errors than individual based trials. Clustering needs to be
considered in trial design and data analysis. One of the commonest errors is where a study is a cluster study but researchers have
failed to recognise this fact. This will then result in the incorrect analysis being pursued. A lower P value will then result and a
false positive error will occur.

Cluster randomised controlled trials: Groups are randomised rather than individuals. Avoids cross contamination amongst
participants. Participants in any one cluster are more likely to respond in a similar fashion. Higher risk of unit of analysis error as
these studies should be analysed as clusters rather than on an individual basis. This leads to a higher false positive rate. It is
possible to adjust for clustering in statistical analyses

Theme: Timing of surgery

A. Immediate surgery
B. Surgery within 2 hours
C. Surgery within 6 hours
D. Surgery within 24 hours
E. Surgery within same hospital admission
F. Urgent elective surgery within 4 weeks
G. True elective surgery

For each procedure please select the most appropriate time interval for surgery. Each option may be selected once, more than once
or not at all.

19. A 43 year old women is admitted with acute cholecystitis, her USS confirms the diagnosis and LFT's are normal. It is
now 10 hours since admission.

The correct answer is Surgery within 24 hours. Ideal case for acute cholecystectomy. This will enable prompt discharge and
facilitate recovery. Whilst expedient surgery is desirable an emergency procedure is not justified.

20. A 5 year old boy is admitted with a suspected acute appendicitis. He has tenderness but no guarding as yet. He requires
appendicectomy.

The correct answer is Surgery within 6 hours. The kind of case that can wait till the following day if presenting out of hours.
Appendicectomy may be deferred where peritoneal signs are absent. Where tenderness and guarding are present a more urgent
approach is warranted.

21. A 72 year old man is admitted with large bowel obstruction. He has been vomiting for 24 hours and his caecum is tender
and measures 11cm.

The correct answer is Surgery within 6 hours. The sun should not rise and set on unrelieved large bowel obstruction! This
patient has a competent ileocaecal valve. As a result lack of surgery would result in caecal perforation leading to faecal
peritonitis with and associated high mortality rate.

508
Preparation for surgery: Elective and emergency patients require different preparation.

Elective cases: Consider pre admission clinic to address medical issues. Blood tests including FBC, U+E, LFTs, Clotting, Group
and Save. Urine analysis. Pregnancy test. Sickle cell test ECG/ Chest x-ray

Exact tests to be performed will depend upon the proposed procedure and patient fitness. Risk factors for development of deep
vein thrombosis should be assessed and a plan for thromboprophylaxis formulated.

Diabetes: Diabetic patients have greater risk of complications. Poorly controlled diabetes carries high risk of wound infections.
Patients with diet or tablet controlled diabetes may be managed using a policy of omitting medication and checking blood glucose
levels regularly. Diabetics who are poorly controlled or who take insulin will require a intravenous sliding scale. Potassium
supplementation should also be given. Diabetic cases should be operated on first.

Emergency cases: Stabilise and resuscitate where needed. Consider whether antibiotics are needed and when and how they
should be administered. Inform blood bank if major procedures planned particularly where coagulopathies are present at the outset
or anticipated (e.g. Ruptured AAA repair). Don't forget to consent and inform relatives.

Special preparation: Some procedures require special preparation: Thyroid surgery; vocal cord check. Parathyroid surgery;
consider methylene blue to identify gland. Sentinel node biopsy; radioactive marker/ patent blue dye. Surgery involving the
thoracic duct; consider administration of cream. Pheochromocytoma surgery; will need alpha and beta blockade. Surgery for
carcinoid tumours; will need covering with octreotide. Colorectal cases; bowel preparation (especially left sided surgery).
Thyrotoxicosis; lugols iodine/ medical therapy.

22. In a randomized study of chemotherapy drugs for bowel cancer, a group receiving treatment A had a recurrence rate of
12.5% and a group receiving treatment B had a recurrence rate of 15%. Both groups are matched for size and length of
follow up. What is the number needed to treat to prevent a recurrence?
A. 2.5
B. 25
C. 4
D. 40
E. 5
Answer: D
There is an absolute risk reduction of 15-12.5%= 2.5% for treatment A
Therefore the NNT = 1/0.025 = 40

23. A new blood test to screen patients for colorectal cancer is trialled on 500 patients. The test was positive in 40 of the 50
patients shown to have colorectal cancer by colonscopy. It was also positive in 20 patients who were shown not to have
colorectal cancer. What is the positive predictive value of the test?
A. 0.8
B. 0.66
C. 0.33
D. 0.1
E. Cannot be calculated
Answer: B
24. Which of the following most closely describes the risk of a type I statistical error?
A. Power calculation
B. P value
C. Odds ratio
D. Relative risk
E. None of the above
Answer: B
Type 1 errors occur when a test rejects a true null hypothesis and is therefore related to the significance level of the test result.
To explain consider the following arbitrary example.

We hypothesise that bowel preparation vs no bowel preparation has no effect on anastomotic leak rates following left
hemicolectomy. If we compare the rates of anastomotic leak and perform a Chi Squared test and obtained a P value of 0.95 we
should conclude that we unable to reject the null hypothesis. Should we choose to do so then we are at risk of committing a type
1 error. In reality the knowledge that a type 1 error was committed is usually some time after the event. When other studies have
been performed that have shown an effect. Power calculations are related to type 2 errors.

Power calculations and statistical error: Statistical error: Type 1 Error: A test rejects a true null hypothesis. Analogus to false
positive. It usually equates to the significance level assigned to a test. Type 2 Error: A test fails to reject a false null hypothesis. It
is related to the power of a test.

509
Statistical power: The power of a test is the probability that the test will reject the null hypothesis when it is false (thereby
avoiding a type 2 error). Increasing the power of a test will reduce the probability of a type 2 error. Usually a value of 0.8 is
selected.

25. Which of the following statements relating to randomised controlled trials is false?
A. Consist of a control group recruited during the same time interval as the treatment group.
B. Are not applicable to retrospectively analysed data even if captured on a prospectively created database.
C. They require concealment of treatment throughout the duration of the study.
D. They require concealment of treatment until after randomisation.
E. They are less susceptible to researcher bias than non-randomised controlled trials.
Answer: C
This statement is true of a blinded study. Concealment of allocation is an important part of RCT's as knowledge of likely allocated
procedure may well affect compliance rates.

Audit and Research: Clinical audit: Quality improvement process that seeks to improve patient care and outcomes through
systematic review of care against explicit criteria and the implementation of change. Aspects of the structure, processes, and
outcomes of care are selected and systematically evaluated against explicit criteria. Where indicated, changes are implemented at
an individual, team, or service level and further monitoring is used to confirm improvement in healthcare delivery. (NICE).

Research:Aims to derive new knowledge which is potentially generalisable or transferable.

26. A cohort study is being designed to look at the relationship between smoking and breast cancer. What is the usual
outcome measure in a cohort study?
A. Odds ratio
B. Experimental event rate
C. Relative risk
D. Absolute risk increase
E. Numbers needed to harm
Answer: C
Cohort studies - relative risk

27. Which of the following statements relating to consenting patients for surgery is false?
A. Cosent should be taken by a person who has sufficient knowledge of the procedure
B. All risks with a frequency of 1 in 500 or greater must be disclosed
C. Patients who have recieved sedating pre medication may no longer be able to provide informed consent
D. Written consent is required for procedures performed under local anaesthesia
E. Where a procedure (or part thereof) consists of research this should be recorded on a separate research consent form
Answer: B
Generally risks with an incidence of 1% or greater are disclosed. Exceptions to this are where a rarer complication is
particularly serious.

28. A group of surgeons conduct a meta analysis of randomised controlled trials comparing the use of analgesic regimes
following laparoscopic cholecystectomy. What level of evidence is provided by such an analysis?
A. V
B. I
C. II
D. III
E. IV
Answer: B

Study design: evidence and recommendations: Levels of evidence: I - evidence from meta-analysis of randomised controlled
trials

 II - evidence from at least one well designed controlled trial which is not randomised
 III - evidence from correlation and comparative studies or use of historical controls
 IV - evidence from case series
 V - Expert opinion or founded on basic principles

510
Knowledge of the sub groups of the levels of evidence are not routinely tested in MRCS Part A.

Grading of recommendation: Grade A - based on evidence from at least one randomised controlled trial (i.e. Ia or Ib)

 Grade B - based on evidence from non-randomised controlled trials (i.e. IIa, IIb or III)
 Grade C - based on evidence from a panel of experts (i.e. IV)

Theme: Audit

A. Standards based audit


B. Departmental review
C. Systems based audit
D. Operational audit
E. Financial audit
F. Peer review

Please select the most appropriate type of audit method for the situation described. Each option may be used once, more than once
or not at all.

29. A surgical department wishes to determine whether it is using types of prosthetic mesh material for incisional hernia
surgery in the most effective manner. Recently there have been cases of non mesh usage and loss of material as a result of
the implants being "out of date".

Systems based audit. This is primarily an issue of stock control. However, the system by which the materials are used within the
theatre will need evaluation. Because it is the usage and stock that are a problem, rather than the sourcing the systems based
audit will be more effective than an operational audit. Theme from April 2012 Exam

30. A group of surgeons wish to determine whether patients are recieving adquete deep vein thrombosis prophylaxis
following surgery.

The correct answer is Standards based audit. This type of audit is widely undertaken in most trusts in the UK.
31. Surgeons are becoming increasing concerned about the adverse results of Mrs X performing a new an innovative
operative procedure not widely practised elsewhere.

Peer review. In the situation where a surgeon performs an unfamiliar procedure a peer review is often the best way to evaluate
the problem. This does not have to be externally based, but often is.

Audit categories: Audits may be used in a variety of clinical settings. These range from standards based audits, which will be
familiar to most clinicians, thorugh to systems based audits which focus more on the processes within an organisation.

Types of audit
Financial audit A historically oriented, independent evaluation performed for the purpose of attesting to the fairness,
accuracy, and reliability of financial data
Operational A future-oriented, systematic, and independent evaluation of organizational activities. Financial data may be
audit used, but the primary sources of evidence are the operational policies and achievements related to
organizational objectives. Internal controls and efficiencies may be evaluated during this type of review.
Departmental A current period analysis of administrative functions, to evaluate the adequacy of controls, safeguarding of
review assets, efficient use of resources, compliance with related laws, regulations and institutional policy and
integrity of financial information.
Standards based Comparison of care or passage of care against set and widely agreed standards or outcomes.
audit
Systems based Evaluation of processes occurring within an institution.
audit

Systems based audits are an integral part of the process of clinical governance.

Theme: Statistics in surgery

A. Mann Whitney U test


B. Analysis of variance
C. LSR post hoc test

511
D. Bonferroni test
E. Kruskall Wallis test
F. T Test

Please select the most appropriate statistical test for the situation described. Each option may be used once, more than once or not
at all.

32. We wish to determine whether there are significantly more patients in a surgical unit presenting with post
appendicectomy wound infections than there were one year previously.

The correct answer is T Test. This will involve the comparison of absolute numbers of patients and therefore this can be
assessed using a T -Test. It does make the assumption that the data is normally distributed. However, the other tests would not
be suitable.

33. We want to make multiple comparisons of different types of side effects of a new drug.

The correct answer is Bonferroni test. As more types of side effects are considered, it becomes more likely that the new drug
will appear to be less safe than existing drugs in terms of at least one side effect. Methods are available to adjust the p value to
reflect the multiple comparisons being made, the aim being to avoid spurious results. A frequently applied correction is the
Bonferroni Method in which the observed p values are multiplied by the number of tests performed, any resulting p value which
is greater than 1 is set to 1 and any which remains at less than 0.05 can be considered significant at the 5% level.

34. 5 surgeons in a colorectal unit wish to determine whether there is a significant difference in their individual leak rates
for anterior resection of the rectum.

The correct answer is Kruskall Wallis test. In this scenario the data is derived from 5 groups of surgeons. If the data were
normally distributed then an ANOVA could be considered. Since these assumptions cannot be met, or satisfied by transforming
the data then the Kruskall-Wallis test provides a non parametric alternative. This is essentially an extension of the Wilcoxon
Rank sum test and detects differences in median values between each group. To compare more accurately differences between
two individual surgeons a Mann Whitney U test may be a more acceptable alternative.

Theme: Governance issues

A. Clinical audit
B. Service evaluation
C. Refer to research ethics committee
D. Implement procedure with no further monitoring
E. Cease activity and refer individual to GMC
F. Cease activity and undertake full service evaluation
G. Obtain written consent from each participant

For the following research/ audit scenarios please select the most appropriate governance modality. Each option may be used
once, more than once or not at all.

35. A general surgical unit has become increasingly concerned about the behaviour of consultant Slasher. Over the past 48
months he has persisted in performing neonatal tracheoesphageal fistula repairs. Unfortunately he has resisted efforts to
prevent him from undertaking these unsupervised. 2 more babies die and the Chief Executive would like your guidance.

The correct answer is Cease activity and refer individual to GMC. While most surgical and departmental problems can be
handled in house is can be seen that this approach has been tried and failed. Given the deaths there is no other option than E

36. As the SpR in general surgery you wish to determine whether your breast cancer unit is complying with the British
Association of Surgical Oncology guidelines for management of high grade ductal carcinoma in situ

The correct answer is Clinical audit. Where there are clear guidelines, an audit is the best measure.

37. A surgeon wishes to undertake some laboratory research into the migratory behaviour patterns of metastatic colorectal
cancer cells. These will be harvested from patients who are undergoing hepatic resection of metastatic colorectal cancer;
apart from diseased tissue no other samples will be taken.

The correct answer is Refer to research ethics committee. Whenever patient tissue is taken for research it is necessary practice to

512
gain ethics approval. Some units may have blanket policies in place for taking tissue for research to tissue banks but as a general
rule most people should seek ethics approval PRIOR to starting research.

38. Which one of the following is equivalent to the pre-test probability?


A. Post test odds / (1 + post-test odds)
B. Pre-test odds x likelihood ratio
C. The prevalence of a condition
D. The incidence of a condition
E. Post-test odds / likelihood ratio
Answer: C
The prevalence is the proportion of a population that have the condition at a point in time whilst the incidence is the rate at which
new cases occur in a population during a specified time period.

Pre- and post- test odds and probability: Pre-test probability: The proportion of people with the target disorder in the
population at risk at a specific time (point prevalence) or time interval (period prevalence). For example, the prevalence of
rheumatoid arthritis in the UK is 1%

Post-test probability: The proportion of patients with that particular test result who have the target disorder. Post-test probability
= post test odds / (1 + post-test odds)

Pre-test odds: The odds that the patient has the target disorder before the test is carried out. Pre-test odds = pre-test probability /
(1 - pre-test probability)

Post-test odds: The odds that the patient has the target disorder after the test is carried out. Post-test odds = pre-test odds x
likelihood ratio. Where the likelihood ratio for a positive test result = sensitivity / (1 - specificity)

39. You are performing a study of weight in patients attending pre operative assessment clinic for elective knee replacement.
Assuming that the results are normally distributed, what percentage of values lie within two standard deviations of the
mean weight?
A. 95.4%
B. 5.3%
C. 98.3%
D. 10%
E. 97.5%
Answer: A
In statistics, the 68-95-99.7 rule or three-sigma rule, or empirical rule states that for a normal distribution, nearly all values lie
within 3 standard deviations of the mean.
About 68.27% of the values lie within 1 standard deviation of the mean. Similarly, about 95.45% of the values lie within 2
standard deviations of the mean. Nearly all (99.73%) of the values lie within 3 standard deviations of the mean
95.4% of values lie within 2 SD of the mean.

Normal distribution:The normal distribution is also known as the Gaussian distribution or 'bell-shaped' distribution. It describes
the spread of many biological and clinical measurements

Properties of the Normal distribution: symmetrical i.e. Mean = mode = median. 68.3% of values lie within 1 SD of the mean.
95.4% of values lie within 2 SD of the mean. 99.7% of values lie within 3 SD of the mean. this is often reversed, so that within
1.96 SD of the mean lie 95% of the sample values. the range of the mean - (1.96 *SD) to the mean + (1.96 * SD) is called the 95%
confidence interval, i.e. If a repeat sample of 100 observations are taken from the same group 95 of them would be expected to lie
in that range

Standard deviation: the standard deviation (SD) represents the average difference each observation in a sample lies from the
sample mean. SD = square root (variance)

40. In Intention to treat analysis, which statement is untrue?


A. It is based on the initial treatment intended.
B. Excludes those who cross over to alternative treatment arms of a trial.
C. Is different from a per protocol analysis.
D. Will affect the statistical power calculation to compare outcomes of treatment.
E. Helps to minimise observer bias.
Answer: B
It includes those who cross over which is how it helps provide additional information relating to those groups.

41. Which of the following is not usually required to make a power calculation?
513
A. The specificity of the intervention being tested
B. The desired significance level
C. The size of the effect being measured
D. The desired power value
E. Sample size used to detect the effect
Answer: A
The specificity of a test is related to type 1 errors.
The components that are nearly always needed for power calculations are : Size of effect, Significance level, Sample size used to
detect the effect., Desired power value

42. A new hernia mesh designed to prevent the risk of infection undergoes clinical trials. One hundred patients are given the
new mesh. During a three month period 10 of the patients have an episode of infection. In the control group there are
300 patients who are given a placebo. In this group 50 people have an infection during the same time period. What is the
relative risk of having an infection when the new mesh is used?
A. 0.8
B. 0.2
C. 1.66
D. 0.6
E. 0.06
Answer: D
Experimental event rate, EER = 10 / 100 = 0.10. Control event rate, CER = 50 / 300 = 0.166.Therefore the relative risk = EER /
CER = 0.1 / 0.166 = 0.6

43.Which of the following statements relating to audit and governance is untrue?


A. An audit standard is a threshold of compliance with an audit criterion
B. Sample size calculations are an important part of audit planning
C. Clinical audit is part of clinical governance
D. Audits should be performed regularly when a novel surgical technique is introduced and where there is little knowledge of
anticipated complications or outcomes
E. An audit criterion is a measurable outcome of care, aspect of practice or capacity
Answer: D
6 pillars of clinical governance: Clinical effectiveness. Research and development. Openess. Risk management. Education and
training. Clinical audit

Audits should compare performance against known standards. Where a novel technique is being introduced standards are unlikely
to exist, sample sizes cannot therefore be accurately calculated. This is an example of research, which is not an audit.

44. Which of the following statements relating to qualitative data is false?


A. The data has no true numerical scale
B. It may comprise multiple data groups
C. May be reported using odds ratios
D. May be reported using frequency histograms
E. It is best analysed statistically using a students T test when multiple factors are present
Answer: E
The students T test should be performed if two sets of data have a normal distribution, the T test cannot be used to analyse
multiple data sets.

45. Which of the following statements is false in relation to consent?


A. All adults by law are assumed to be competent
B. The Bolam test defines if a patient has capacity or not
C. Consent 2 is the form signed by parents on behalf of their children
D. Implied consent is a form of consent
E. Consent 4 is the form signed when a patient is unable to consent for a treatment or investigation
Answer: B
The Bolam test defines if a decision made by a doctor is in agreement with the professional standard of medical practise.

Neurosurgery
1. A 21 year old man is involved in a road traffic accident. After a transient period of concussion he is found to have a GCS
of 15 by the paramedics. On arrival at hospital he is monitored in a side room of the emergency department. When he is
next observed he is noted to have a GCS of 3 and a blown right pupil. Which of the processes below best accounts for this
deterioration?
A. Hydrocephalus
514
B. Intraventricular bleed
C. Sub dural bleed
D. Trans tentorial herniation
E. Sub arachnoid haemorrhage
Answer: D

The presence of a blown right pupil is a sign of a third cranial nerve compression. The most likely cause is an extradural bleed.
However, since this option is not listed the process of trans tentorial herniation would be the most applicable answer.
Intraventricular bleeds are typically more common in premature neonates, deterioration due to hydrocephalus is more chronic.
Theme from April 2012 Exam

Head injuries: Extradural haematoma: Bleeding into the space between the dura mater and the skull. Often results from
acceleration-deceleration trauma or a blow to the side of the head. The majority of extradural haematomas occur in the temporal
region where skull fractures cause a rupture of the middle meningeal artery.
Features: Raised intracranial pressure. Some patients may exhibit a lucid interval

Subdural haematoma: Bleeding into the outermost meningeal layer. Most commonly occur around the frontal and parietal lobes.
May be either acute or chronic. Risk factors include old age and alcoholism. Slower onset of symptoms than a extradural
haematoma.

Subarachnoid haemorrhage: Usually occurs spontaneously in the context of a ruptured cerebral aneurysm but may be seen in
association with other injuries when a patient has sustained a traumatic brain injury

Theme: Cranial nerve lesions

A. Optic nerve
B. Oculomotor nerve
C. Trigeminal nerve
D. Facial nerve
E. Abducens nerve
F. Glossopharyngeal nerve
G. Vestibulocochlear nerve
H. Trochlear nerve
I. Hypoglossal nerve

For each of the scenarios given please give the most likely cranial nerve responsible for the symptom or lesion described. Each
nerve may be used once, more than once or not at all.

2. A 63 year old man is admitted with severe headache, nausea and recent epileptic fit. Fundoscopy shows papilloedema. He
is also noted to have diplopia.

The correct answer is Abducens nerve. The long intracranial course of this nerve makes is susceptible to damage early in the
course of raised ICP.

3. A 32 year old lady is admitted with weakness, visual disturbance and peri orbital pain. On examination she is noted to
have mydriasis and diminished direct and consensual responses to light shone into the affected eye.

The correct answer is Optic nerve. This is likely to be optic neuritis (not really surgical !). This will result in an afferent defect of
the light reflex.

4. An 18 year old boy undergoes an uncomplicated tonsillectomy for recurrent attacks of tonsillitis. Post operatively he
complains of otalgia.

The correct answer is Glossopharyngeal nerve. The glossopharyngeal nerve supplies this area and the ear and otalgia may be the
result of referred pain.

Cranial nerves: Cranial nerve lesions

515
Olfactory nerve: May be injured in basal skull fractures or involved in frontal lobe tumour extension. Loss of olfactory nerve
function in relation to major CNS pathology is seldom an isolated event and thus it is poor localiser of CNS pathology.

Optic nerve: Problems with visual acuity may result from intra ocular disorders. Problems with the blood supply such as
amaurosis fugax may produce temporary visual distortion. More important surgically is the pupillary response to light. The
pupillary size may be altered in a number of disorders. Nerves involved in the resizing of the pupil connect to the pretectal nucleus
of the high midbrain, bypassing the lateral geniculate nucleus and the primary visual cortex. From the pretectal nucleus neurones
pass to the Edinger - Westphal nucleus, motor axons from here pass along with the oculomotor nerve. They synapse with ciliary
ganglion neurones; the parasympathetic axons from this then innervate the iris and produce miosis. The miotic pupil is seen in
disorders such a Horner's syndrome or opiate overdose.

Mydriasis is the dilatation of the pupil in response to disease, trauma, drugs (or the dark!). It is pathological when light fails to
induce miosis. The radial muscle is innervated by the sympathetic nervous system. Because the parasympathetic fibres travel with
the oculomotor nerve they will be damaged by lesions affecting this nerve (e.g. cranial trauma).
The response to light shone in one eye is usually a constriction of both pupils. This indicates intact direct and consensual light
reflexes. When the optic nerve has an afferent defect the light shining on the affected eye will produce a diminished pupillary
response in both eyes. Whereas light shone on the unaffected eye will produce a normal pupillary response in both eyes. This is
referred to as the Marcus Gunn pupil and is seen in conditions such as optic neuritis. In a total CN II lesion shining the light in the
affected eye will produce no response.

Oculomotor nerve: The pupillary effects are described above. In addition it supplies all ocular muscles apart from lateral rectus
and superior oblique. Thus the affected eye will be deviated inferolaterally. Levator palpebrae superioris may also be impaired
resulting in impaired ability to close the eye.

Trochlear nerve: The eye will not be able to look down.

Trigeminal nerve : Largest cranial nerve. Exits the brainstem at the pons. Branches are ophthalmic, maxillary and mandibular.
Only the mandibular branch has both sensory and motor fibres. Branches converge to form the trigeminal ganglion (located in
Meckels cave). It supplies the muscles of mastication and also tensor veli palatine, mylohyoid, anterior belly of digastric and
tensor tympani. The detailed descriptions of the various sensory functions are described in other areas of the website. The corneal
reflex is important and is elicited by applying a small tip of cotton wool to the cornea, a reflex blink should occur if it is intact. It
is mediated by: the naso ciliary branch of the ophthalmic branch of the trigeminal (sensory component) and the facial nerve
producing the motor response. Lesions of the afferent arc will produce bilateral absent blink and lesions of the efferent arc will
result in a unilateral absent blink.

Abducens nerve: The affected eye will have a deficit of abduction. This cranial nerve exits the brainstem between the pons and
medulla. It thus has a relatively long intra cranial course which renders it susceptible to damage in raised intra cranial pressure.

Facial nerve: Emerges from brainstem between pons and medulla. It controls muscles of facial expression and taste from the
anterior 2/3 of the tongue. The nerve passes into the petrous temporal bone and into the internal auditory meatus. It then passes
through the facial canal and exits at the stylomastoid foramen. It passes through the parotid gland and divides at this point. It does
not innervate the parotid gland. Its divisions are considered in other parts of the website. Its motor fibres innervate orbicularis
oculi to produce the efferent arm of the corneal reflex. In surgical practice it may be injured during parotid gland surgery or
invaded by malignancies of the gland and a lower motor neurone on the ipsilateral side will result.

Vestibulo-cochlear nerve Exits from the pons and then passes through the internal auditory meatus. It is implicated in
sensorineural hearing loss. Individuals with sensorineural hearing loss will localise the sound in webers test to the normal ear.
Rinnes test will be reduced on the affected side but should still work. These two tests will distinguish sensorineural hearing loss
from conductive deafness. In the latter condition webers test will localise to the affected ear and Rinnes test will be impaired on
the affected side. Surgical lesions affecting this nerve include CNS tumours and basal skull fractures. It may also be damaged by
the administration of ototoxic drugs (of which gentamicin is the most commonly used in surgical practice).

Glossopharyngeal nerve Exits the pons just above the vagus. Receives sensory fibres from posterior 1/3 tongue, tonsils, pharynx
and middle ear (otalgia may occur following tonsillectomy). It receives visceral afferents from the carotid bodies. It supplies
parasympathetic fibres to the parotid gland via the otic ganglion and motor function to stylopharyngeaus muscle. The sensory
function of the nerve is tested using the gag reflex.

516
Vagus nerve: Leaves the medulla between the olivary nucleus and the inferior cerebellar peduncle. Passes through the jugular
foramen and into the carotid sheath. Details of the functions of the vagus nerve are covered in the website under relevant organ
sub headings.

Accessory nerve: Exists from the caudal aspect of the brainstem (multiple branches) supplies trapezius and sternocleidomastoid
muscles. The distal portion of this nerve is most prone to injury during surgical procedures.

Hypoglossal nerve: Emerges from the medulla at the preolivary sulcus, passes through the hypoglossal canal. It lies on the carotid
sheath and passes deep to the posterior belly of digastric to supply muscles of the tongue (except palatoglossus). Its location of the
carotid sheath makes it vulnerable during carotid endarterectomy surgery and damage will produce ipsilateral defect in muscle
function.

Theme: Head injury- assessment

A. 3
B. 4
C. 6
D. 8
E. 10
F. 15
G. 12
H. 13
I. 5
J. 7

For each of the scenarios given please determine the most likely Glasgow Coma Score. Each option may be used once, more than
once or not at all.

5. A 20 year old man is hit over the head with a mallet. On arrival in the accident and emergency department he opens his
eyes to pain and groans or grunts. On application of a painful stimulus to his hands, he extends his arm at the elbow.

6. E=2, V= 2, M=2. Theme from 2011 Exam


Theme from September 2012 Exam

6. A 20 year old man falls over and bangs his head whilst intoxicated. On arrival in the emergency department he opens his
eyes in response to speech, and is able to speak, although he is disorientated. He obeys motor commands.

13. E=3, V=4, M=6.

7. A 20 year old man is hit over the head with an iron bar. On arrival in the emergency department he opens his eyes in
response to pain, his only verbal responses are in the form of groans and grunts. On application of a painful stimulus to
his hands, he flexes his forearms away from the painful stimuli.

8. E=2, V=2, M=4.


Appropriate flexion to pain carries a higher score than decorticate posturing or inappropriate flexion.

Glasgow coma scale


Eye opening: Spontaneous. To speech. To pain. None
Verbal response: Orientated. Confused. Words. Sounds. None
Motor response: Obeys commands. Localises to pain. Withdraws from pain. Abnormal flexion to pain (decorticate posture).
Extending to pain. None
Responses are taken from each category (marks in descending order) to produce an overall score. Severe brain injuries are
generally associated with GCS <8.
8. A 33 year old lady develops a thunderclap headache and collapses. A CT scan shows that she has developed a
subarachnoid haemorrhage. She currently has no evidence of raised intracranial pressure. Which of the following drugs
should be administered?
A. None
B. Atenotol
C. Labetolol
D. Nimodipine
517
E. Mannitol
Answer: D
Nimodipine is a calcium channel blocker. It reduces cerebral vasospasm and improves outcomes. It is administered to most
cases of sub arachnoid haemorrhage. Theme from 2007 Exam

Sub arachnoid haemorrhage: Spontaneous intracranial haemorrhage: Most commonly sub arachnoid haemorrhage. It is due
to intra cranial aneurysm in 85% cases. Approximately 10% of cases will have normal angiography and the cause will remain
unclear. Patients with inherited connective tissue disorders are at higher risk although most cases are sporadic. >95% cases will
have headache (often thunderclap). >15% will have coma

Investigation: CT scan for all (although as CSF blood clears the sensitivity declines)
Lumbar puncture if CT normal (very unlikely if normal)
CT angiogram to look for aneurysms.

Management: Supportive treatment, optimising BP (not too high if untreated aneurysm) and ventilation if needed.
Nimodipine reduces cerebral vasospasm and reduces poor outcomes. Untreated patients most likely to rebleed in first 2 weeks.
Patients developing hydrocephalus will need a V-P shunt (external ventricular drain acutely). Electrolytes require careful
monitoring and hyponatraemia is common.

Treatment of aneurysm: >80% aneuryms arise from the anterior circulation. Craniotomy and clipping of aneurysm is standard
treatment, alternatively suitable lesions may be coiled using an endovascular approach. Where both options are suitable data
suggests that outcomes are better with coiling than surgery.

9. A 65 year old male with known nasopharyngeal carcinoma presents with double vision over a few weeks. On
examination he is found to have left eye proptosis and it is down and out. He reports pain on attempting to move the eye.
There is an absent corneal reflex. What is the most likely diagnosis?
A. Posterior communicating artery aneurysm
B. Cavernous sinus syndrome
C. Optic nerve tumour
D. Migraine
E. Cerebral metastases
Answer: B
Cavernous sinus syndrome is most commonly caused by cavernous sinus tumours. In this case, the nasopharyngeal malignancy
has locally invaded the left cavernous sinus. Diagnosis is based on signs of pain, opthalmoplegia, proptosis, trigeminal nerve
lesion (opthalmic branch) and Horner's syndrome.

Cavernous sinus: Is a large collection of thin-walled veins creating a cavity. Bordered by the temporal bone of the skull and the
sphenoid bone. Medially: pituitary fossa, sphenoid sinus. Laterally: temporal lobe

Contents: 1. Lateral wall components, from top to bottom: Oculomotor nerve. Trochlear nerve. Ophthalmic nerve. Maxillary
nerve. 2. Components within the sinus, from medial to lateral:

Internal carotid artery (and sympathetic plexus). Abducens nerve

Blood supply: Ophthalmic vein, superficial cortical veins, basilar plexus of veins posteriorly. Drains into the internal jugular vein
via: the superior and inferior petrosal sinuses

Theme: Head injury

A. Subdural haematoma
B. Extradural haematoma
C. Subarachnoid haemorrhage
D. Basal skull fracture
E. Intracerebral haematoma
F. Le fort 1 fracture of maxilla
G. Le fort fracture 3 of maxilla
H. Mandibular fracture

What is the most likely diagnosis for the scenario given? Each option may be used once, more than once or not at all.

10. A 32 year old female hits her head on the steering wheel during a collision with another car. She has periorbital swelling
and a flattened appearance of the face.

518
Le fort fracture 3 of maxilla. The flattened appearance of the face is a classical description of the dish/pan face associated with
Le fort fracture 2 or 3 of the maxilla.

11. A 29 year bouncer is hit on the side of the head with a bat. He now presents to A&E with odd behaviour and complaining
of a headache. Whilst waiting for a CT scan he becomes drowsy and unresponsive.

Extradural haematoma. The middle meningeal artery is prone to damage when the temporal side of the head is hit.
Note that there may NOT be any initial LOC or lucid interval.

12. A 40 year old alcoholic presents with worsening confusion over 2 weeks. He has weakness of the left side of the body.

Subdural haematoma. Subdural haematomas can have a history over weeks/months. It is common in alcoholics due to cerebral
atrophy causing increased stretching of veins.

Head injury: Patients who suffer head injuries should be managed according to ATLS principles and extra cranial injuries should
be managed alongside cranial trauma. Inadequate cardiac output will compromise CNS perfusion irrespective of the nature of the
cranial injury.

Pathophysiology: Primary brain injury may be focal (contusion/ haematoma) or diffuse (diffuse axonal injury). Diffuse axonal
injury occurs as a result of mechanical shearing following deceleration, causing disruption and tearing of axons. Intra-cranial
haematomas can be extradural, subdural or intracerebral, while contusions may occur adjacent to (coup) or contralateral (contre-
coup) to the side of impact. Secondary brain injury occurs when cerebral oedema, ischaemia, infection, tonsillar or tentorial
herniation exacerbates the original injury. The normal cerebral auto regulatory processes are disrupted following trauma rendering
the brain more susceptible to blood flow changes and hypoxia. The Cushings reflex (hypertension and bradycardia) often occurs
late and is usually a pre terminal event

Management: Where there is life threatening rising ICP such as in extra dural haematoma and whilst theatre is prepared or
transfer arranged use of IV mannitol/ frusemide may be required. Diffuse cerebral oedema may require decompressive
craniotomy. Exploratory Burr Holes have little management in modern practice except where scanning may be unavailable and to
thus facilitate creation of formal craniotomy flap. Depressed skull fractures that are open require formal surgical reduction and
debridement, closed injuries may be managed non operatively if there is minimal displacement. ICP monitoring is appropriate in
those who have GCS 3-8 and normal CT scan. ICP monitoring is mandatory in those who have GCS 3-8 and Abnormal CT scan.
Hyponatraemia is most likely to be due to syndrome of inappropriate ADH secretion. Minimum of cerebral perfusion pressure of
70mmHg in adults. Minimum cerebral perfusion pressure of between 40 and 70 mmHg in children.

Interpretation of pupillary findings in head injuries


Pupil size Light response Interpretation
Unilaterally dilated Sluggish or fixed 3rd nerve compression secondary to tentorial herniation
Bilaterally dilated Sluggish or fixed  Poor CNS perfusion
 Bilateral 3rd nerve palsy

Unilaterally dilated or equal Cross reactive (Marcus - Gunn) Optic nerve injury
Bilaterally constricted May be difficult to assess  Opiates
 Pontine lesions
 Metabolic encephalopathy

Unilaterally constricted Preserved Sympathetic pathway disruption

13. A patient is referred due to the development of a third nerve palsy associated with a headache. On examination
meningism is present. Which one of the following diagnoses needs to be urgently excluded?
A. Weber's syndrome
B. Internal carotid artery aneurysm
C. Multiple sclerosis
D. Posterior communicating artery aneurysm
E. Anterior communicating artery aneurysm
Answer: D
Painful third nerve palsy = posterior communicating artery aneurysm

Given the combination of a headache and third nerve palsy it is important to exclude a posterior communicating artery aneurysm
519
Third nerve palsy

Features: eye is deviated 'down and out'. Ptosis. P upil may be dilated (sometimes called a 'surgical' third nerve palsy)

Causes: diabetes mellitus. Vasculitis e.g. temporal arteritis, SLE. False localizing sign* due to uncal herniation through tentorium
if raised ICP. Posterior communicating artery aneurysm (pupil dilated). Cavernous sinus thrombosis. Weber's syndrome:
ipsilateral third nerve palsy with contralateral hemiplegia -caused by midbrain strokes. Other possible causes: amyloid, multiple
sclerosis
*this term is usually associated with sixth nerve palsies but it may be used for a variety of neurological presentations

14. A 23 year old man was driving a car at high speed whilst intoxicated, he was wearing a seat belt. The car collides with a
brick wall at around 140km/h. When he arrives in the emergency department he is comatose. His CT scan appears to be
normal. He remains in a persistent vegetative state. Which of the following is the most likely underlying cause?
A. Extradural haemorrhage
B. Sub dural haemorrhage
C. Sub arachnoid haemorrhage
D. Intracerebral haemorrhage
E. Diffuse axonal injury
Answer: E
Diffuse axonal injury occurs when the head is rapidly accelerated or decelerated. There are 2 components: 1. Multiple
haemorrhages. 2. Diffuse axonal damage in the white matter

Up to 2/3 occur at the junction of grey/white matter due to the different densities of the tissue. The changes are mainly
histological and axonal damage is secondary to biochemical cascades. Often there are no signs of a fracture or contusion.

Interpretation of pupillary findings in head injuries

Theme: Visual defects

A. Right homonymous hemianopia


B. Left homonymous hemianopia
C. Right superior quadranopia
D. Left superior quadranopia
E. Right inferior quadranopia
F. Left inferior quadranopia
G. Upper bitemporal hemianopia
H. Lower bitemporal hemianopia

What is the most likely visual field defect for the scenario given? Each option may be used once, more than once or not at all.

15. A 42 year old woman is admitted to the vascular ward for an endarterectomy. Her CT report confirms a left temporal
lobe infarct.

The correct answer is Right superior quadranopia. Temporal lesions cause a contralateral superior quadranopia. Think temporal
area is at the top of the head i.e. superior quadranopia.

16. A 22 year old man is referred to urology with possible urinary retention. He is passing huge amounts of urine. Post void
bladder ultrasound is normal.

The correct answer is Lower bitemporal hemianopia. This patient has diabetes insipidus due to a craniopharyngioma. This causes
a lower bitemporal hemianopia. Theme from January 2012 exam

17. A 53 year old man is admitted to the vascular ward for a carotid endarterectomy. His CT head report confirms a left
parietal lobe infarct.

The correct answer is Right inferior quadranopia. Parietal lesions cause a contralateral inferior quadranopia. Superior
quadranopia = temporal lobe lesion. Inferior quadranopia = parietal lobe lesion

Visual field defects: left homonymous hemianopia means visual field defect to the left, i.e. Lesion of right optic tract.
homonymous quadrantanopias: PITS (Parietal-Inferior, Temporal-Superior). Incongruous defects = optic tract lesion; congruous
defects = optic radiation lesion or occipital cortex. Theme from January 2012 exam. Theme from April 2012 exam
520
Homonymous hemianopia: Incongruous defects: lesion of optic tract. Congruous defects: lesion of optic radiation or occipital
cortex. Macula sparing: lesion of occipital cortex

Homonymous quadrantanopias: Superior: lesion of temporal lobe. Inferior: lesion of parietal lobe. Mnemonic = PITS (Parietal-
Inferior, Temporal-Superior)

Bitemporal hemianopia: Lesion of optic chiasm. Upper quadrant defect > lower quadrant defect = inferior chiasmal
compression, commonly a pituitary tumour. Lower quadrant defect > upper quadrant defect = superior chiasmal compression,
commonly a craniopharyngioma

Theme: Intra cranial haemorrhage

A. Acute sub dural haematoma


B. Chronic sub dural haematoma
C. Acute extradural haematoma
D. Chronic extradural haematoma
E. Intraventricular haemorrhage
F. Sub arachnoid haemorrhage

Please select the most likely intra cranial bleeding event to account for the scenario described. Each option may be used once,
more than once or not at all.

18. A 28 year old man is playing tennis when he suddenly collapses and has a GCS of 4 when examined.

Sub arachnoid haemorrhage.. A sudden collapse and loss of consciousness is most likely to be due to a sub arachnoid
haemorrhage. The other potential causes in the list usually occur as a sequel to a traumatic event, which has not occurred here.
Theme from April 2012 Exam

19. A 2 day old premature neonate is born by emergency cesarean section for maternal illness. The baby is noted to become
floppy and unresponsive.

Intraventricular haemorrhage. Neonatal deterioration in premature babies is not infrequently due to intra ventricular
haemorrhage. In extreme prematurity the prognosis can be very poor.

20. A 78 year old man is brought to the emergency department by the police. He is found wandering around the town centre
and is confused. His family report that he is usually well apart from a simple mechanical fall 3 weeks previously from
which he sustained no obvious injuries.

Chronic sub dural haematoma. The injuries that are responsible for chronic sub dural haematomas are usually fairly trivial and
forgotten by the patient and their families. The onset of symptoms can be insidious with vague symptomatology and confusion
predominating.

Theme: Management of head injuries

A. Intravenous mannitol
B. Parietotemporal craniotomy
C. Burr Hole decompression
D. Posterior fossa craniotomy
E. Insertion of intracranial bolt monitor
F. Discharge
G. Intravenous frusemide

What is the best immediate management plan for the injury described? Each option may be used once, more than once or not at
all.

21. A 25 year old cyclist is hit by a bus traveling at 30mph. He is not wearing a helmet. He arrives with a GCS of 3/15 and is
intubated. A CT scan shows evidence of cerebral contusion but no localising clinical signs are present

521
The correct answer is Insertion of intracranial bolt monitor. This patient may well develop raised ICP over the next few days and
Intracranial pressure monitoring will help with management
22. A 32 year old rugby player is crushed in a scrum. He is briefly concussed but then regains consciousness. He then
collapses and is brought to A+E. His GCS on arrival is 6/15 and his left pupil is dilated.

Parietotemporal craniotomy.This man needs urgent decompression and extradural haematoma is the most likely event from a
lacerated middle meningeal artery. The debate as to whether Burr Holes or craniotomy is the best option continues. Most
neurosurgeons would perform a craniotomy. However, rural units and those units without neurosurgical kit facing this
emergency may resort to Burr Holes

23. A 30 year old women is injured in a skiing accident. She suffers a blow to the occiput and is concussed for 5 minutes. On
arrival in A+E she is confused with GCS 10/15. A CT scan shows no evidence of acute bleed or fracture but some
evidence of oedema with the beginnings of mass effect

Intravenous mannitol. This women has raised ICP and mannitol will help reduce this in the acute phase

Head injury management- NICE Guidelines: Summary of guidelines :All patients should be assessed within 15 minutes on
arrival to A&E. Document all 3 components of the GCS. If GCS <8 or = to 8, consider stabilising the airway. Treat pain with low
dose IV opiates (if safe). Full spine immobilisation until assessment if:

GCS < 15: neck pain/tenderness. paraesthesia extremities. focal neurological deficit. suspected c-spine injury

If a c-spine injury is suspected a 3 view c-spine x-ray is indicated. CT c-spine is preferred if: Intubated. GCS <13. Normal x-ray
but continued concerns regarding c-spine injury

Immediate CT head (within 1h) if: GCS < 13 on admission. GCS < 15 2h after admission. Suspected open or depressed skull
fracture. Suspected skull base fracture (panda eyes, Battle's sign, CSF from nose/ear, bleeding ear). Focal neurology. Vomiting >
1 episode. Post traumatic seizure. Coagulopathy

Contact neurosurgeon if: Persistent GCS < 8 or = 8. Unexplained confusion > 4h. Reduced GCS after admission. Progressive
neurological signs. Incomplete recovery post seizure. Penetrating injury. Cerebrospinal leak

Observations: 1/2 hourly GCS until 15

24. A 25-year-old female with a history of bilateral vitreous haemorrhage is referred with bilateral lesions in the cerebellar
region. What is the likely diagnosis?
A. Neurofibromatosis type I
B. Neurofibromatosis type II
C. Tuberose sclerosis
D. Von Hippel-Lindau syndrome
E. Sarcoidosis
Answer: D
Retinal and cerebellar haemangiomas are key features of Von Hippel-Lindau syndrome. Retinal haemangiomas are bilateral in
25% of patients and may lead to vitreous haemorrhage

Von Hippel-Lindau syndrome: Von Hippel-Lindau (VHL) syndrome is an autosomal dominant condition predisposing to
neoplasia. It is due to an abnormality in the VHL gene located on short arm of chromosome 3

Features: Cerebellar haemangiomas. Retinal haemangiomas: vitreous haemorrhage. Renal cysts (premalignant).
Phaeochromocytoma. Extra-renal cysts: epididymal, pancreatic, hepatic. Endolymphatic sac tumours

Theme: Head injury

A. Acute sub dural haematoma


B. Intra cerebral haematoma
C. Extra dural haematoma
D. Chronic sub dural haematoma
E. Basal skull fracture
F. Subarachnoid haemorrhage
522
G. Diffuse axonal injury

What is the most likely diagnosis for the scenario given? Each option may be used once, more than once or not at all.

25. A 18 year old boy is involved in a fall from a balcony whilst intoxicated. He has bruising over the mastoid area and is
unconscious.

Basal skull fracture. Bruising over the mastoid process of the temporal bone is battle's sign caused by a basal skull fracture.

26. A 18 year old boy falls off a balcony and hits the right side of the head. He is admitted to the emergency department and
is initially lucid. He is admitted for observation, and over the following twelve hours develops an increasing headache
and confusion. A CT scan shows a hyperdense collection of fluid with associated midline shift.

Acute sub dural haematoma. Sub dural haematomas are the commonest intracranial mass lesions resulting from trauma. They
are classified as acute, sub acute or chronic according to tempo of onset. Acute sub dural haematomas will present within 72
hours of the original injury and have hyperdense appearances on CT scanning.

27. A baby is brought to casualty unconscious and in a vegetative state. She has cigarette burns on her legs.

The correct answer is Diffuse axonal injury. The baby is likely to be a victim of shaken baby syndrome. This may result in
diffuse axonal injury causing extensive lesions in the white matter.

28. A 28 year old man falls and hits his head against a wall. There is a brief loss of consciousness. When assessed in accident
and emergency he is alert and orientated with a GCS of 15, imaging shows no fracture of the skull. What is his risk of
having an intracranial haematoma requiring removal?
A. 1 in 6000
B. 1 in 40
C. 1 in 4
D. 1 in 50,000
E. 1 in 120
Answer: A

Head injury and haematoma: Risk of haematoma (requiring removal) in adults attending accident and emergency units
following head injury.
Injury Conscious level Risk of haematoma requiring removal
Concussion, no skull fracture Orientated 1 in 6000
Concussion, no skull fracture Not orientated 1 in 120
Skull fracture Orientated 1 in 32
Skull fracture Not orientated 1 in 4

29. The term signature fracture is synonymous with which of the following injuries?
A. Depressed skull fracture
B. Le Fort II fracture
C. Orbital blow out
D. Oblique fracture of the tibia
E. Supracondylar fracture
Answer: A
Signature fractures are synonymous with depressed skull fractures, they are usually low velocity injuries where the fracture
impression resembles the injurious source.

Depressed skull fracture: Depressed skull fractures are also referred to as signature fractures. Results from the focal impact of a
moving object on the cranial vault. High velocity objects will not only disrupt bone, but may also drive the fracture fragments into
the brain. Blunt objects moving at low velocity may produce a defect in the skull that is of similar dimensions to the object (c.f.
signature). Injuries may affect the outer table alone or extend to involve the inner table. Open fractures and those associated with
intracranial haematomas may require surgery, uncomplicated fractures without significant cosmetic deformities may be managed
conservatively. CT scanning is the initial imaging modality of choice.

30. Which of the following is not a form of primary brain injury?

523
A. Sub dural haemorrhage after being hit in the head with a hammer
B. Meningitis resulting from infected CSF rhinorrhoea after a basal skull fracture
C. A truck driver is involved in a road traffic accident and suffers an axonal stretch injury
D. A man is hit with a baseball bat and suffers a cerebral contusion
E. A man suffers an intraparenchymal haemorrhage after being hit in head during a car crash
Answer: B
Primary brain damage occurs at the point of injury. It includes contusions and diffuse axonal injury. Non reversible.
Secondary brain damage occurs after the injury. Complications include:1. Haemorrhage 2. Meningitis 3. Herniation 4. Hypoxia
5. Oedema 6. Arterial damage: internal carotid, vertebral artery common

Transplantation
Theme: Renal transplant complications

A. Acute tubular necrosis


B. Renal artery thrombosis
C. Bladder occlusion
D. Ureteric occlusion
E. Acute rejection
F. Acute on chronic rejection
G. Hyperacute rejection

For each of the scenarios given please select the most likely underlying process from the list below. Each option may be used
once, more than once or not at all.

1. A 45 year old man with end stage renal failure undergoes a cadaveric renal transplant. The transplanted organ has a
cold ischaemic time of 26 hours and a warm ischaemic time of 54 minutes. Post operatively the patient receives
immunosuppressive therapy. Ten days later the patient has gained weight, becomes oliguric and feels systemically
unwell. He also complains of swelling over the transplant site that is painful.

The correct answer is Acute rejection.The features described are those of worsening graft function and acute rejection. The fact
that there is a 10 day delay goes against hyperacute rejection. Cold ischaemic times are a major factor for delayed graft function.
However, even 26 hours is not incompatible with graft survival. Theme from April 2012 Exam

2. A 44 year old man with end stage renal failure undergoes a live donor renal transplant. During the immediate post
operative period a good urine output is recorded. However, on return to the ward the nursing staff notice that the
urinary catheter is no longer draining. However, the urostomy is continuing to drain urine.

The correct answer is Bladder occlusion. The most likely explanation for this event is a blocked catheter. This may be the result
of blood clot from the ureteric anastomosis. Bladder irrigation will usually resolve the problem.

3. A 43 year old man undergoes a live donor renal transplant. The donor's right kidney is anastomosed to the recipient. On
removal of the arterial clamps there is good urinary flow noted and the wounds are closed. On return to the ward the
nurses notice that the patient suddenly becomes anuric and irrigation of the bladder does not improve the situation.

The correct answer is Renal artery thrombosis. Right sided live donor transplants are extremely rare. This is because the vena
cava precludes mobilisation of the right renal artery. The short right renal artery that is produced therefore presents a major
challenge. The sudden cessation of urine output in this context is highly suggestive of an acute thrombosis. Delay in
thrombectomy beyond 1 hour almost inevitably results in graft loss.

Complications following renal transplant: Renal transplantation is widely practised. The commonest technical related
complications are related to the ureteric anastomosis. The warm ischaemic time is also of considerable importance and graft
survival is directly related to this. Long warm ischaemic times increase the risk of acute tubular necrosis which may occur in all
types of renal transplanation and provided other insults are minimised, will usually recover. Organ rejection may occur at any
phase following the transplantation process.

Types of organ rejection: Hyperacute. This occurs immediately through presence of pre formed antigens (such as ABO
incompatibility). Acute. Occurs during the first 6 months and is usually T cell mediated. Usually tissue infiltrates and vascular
lesions. Chronic. Occurs after the first 6 months. Vascular changes predominate.

524
Hyperacute: Renal transplants are most susceptible to this process. Risk factors include major HLA mismatch and ABO
incompatibility. The rejection occurs almost immediately and the macroscopic features may become manifest following
completion of the vascular anastomosis and removal of clamps. The kidney becomes mottled, dusky and the vessels will
thrombose. The only treatment is removal of the graft, if left in situ it will result in abscess formation.

Acute: All organs may undergo acute rejection. Mononuclear cell infiltrates predominate. All types of transplanted organ are
susceptible and it may occur in up to 50% cases. Most cases can be managed medically.

Chronic: Again all transplants with HLA mismatch may suffer this fate. Previous acute rejections and other immunosensitising
events all increase the risk. Vascular changes are most prominent with myointimal proliferation leading to organ ischaemia. Organ
specific changes are also seen such as loss of acinar cells in pancreas transplants and rapidly progressive coronary artery disease in
cardiac transplants.
4. A 43 year old man undergoes a cadaveric renal transplant. The operation is uncomplicated. On removal of the vascular
clamps the transplanted kidney immediately turns dusky and over the ensuing hours appears non viable. Which of the
following best explains this event?
A. Chronic rejection
B. Hyper acute rejection
C. Acute rejection
D. Sub chronic rejection
E. Infection of the graft
Answer: B
Immediate rejection is due to the presence of pre-existing antibodies e.g. ABO mismatch. The transplanted organ should be
removed.

Organ Transplant: A number of different organ and tissue transplants are now available. In many cases an allograft is
performed, where an organ is transplanted from one individual to another. Allografts will elicit an immune response and this is
one of the main reasons for organ rejection.

Graft rejection occurs because allografts have allelic differences at genes that code immunohistocompatability complex genes.
The main antigens that give rise to rejection are: ABO blood group. Human leucocyte antigens (HLA). Minor histocompatability
antigens

ABO Matching: ABO incompatibility will result in early organ rejection (hyperacute) because of pre existing antibodies to other
groups. Group O donors can give organs to any type of ABO recipient whereas group AB donor can only donate to AB recipient.

HLA System: The four most important HLA alleles are: HLA A. HLA B. HLA C. HLA DR

An ideal organ match would be one in which all 8 alleles are matched (remember 2 from each parent, four each = 8 alleles).
Modern immunosuppressive regimes help to manage the potential rejection due to HLA mismatching. However, the greater the
number of mismatches the worse the long term outcome will be. T lymphocytes will recognise antigens bound to HLA molecules
and then will then become activated. Clonal expansion then occurs with a response directed against that antigen.

Types of organ rejection: Hyperacute. This occurs immediately through presence of pre formed antigens (such as ABO
incompatibility). Acute. Occurs during the first 6 months and is usually T cell mediated. Usually tissue infiltrates and vascular
lesions. Chronic. Occurs after the first 6 months. Vascular changes predominate.

Hyperacute: Renal transplants at greatest risk and liver transplants at least risk. Although ABO incompatibility and HLA Class I
incompatible transplants will all fare worse in long term.

Acute: All organs may undergo acute rejection. Mononuclear cell infiltrates predominate. All types of transplanted organ are
susceptible and it may occur in up to 50% cases.

Chronic: Again all transplants with HLA mismatch may suffer this fate. Previous acute rejections and other immunosensitising
events all increase the risk. Vascular changes are most prominent with myointimal proliferation leading to organ ischaemia. Organ
specific changes are also seen such as loss of acinar cells in pancreas transplants and rapidly progressive coronary artery disease in
cardiac transplants.

Surgical overview-Renal transplantation: A brief overview of the steps involved in renal transplantation is given.
Patients with end stage renal failure who are dialysis dependent or likely to become so in the immediate future are considered for
transplant. Exclusion criteria include; active malignancy, old age (due to limited organ availability). Patients are medically

525
optimised.

Donor kidneys, these may be taken from live related donors and close family, members may have less HLA mismatch than
members of the general population. Laparoscopic donor nephrectomy further minimises the operative morbidity for the donor.
Other organs are typically taken from brain dead or dying patients who have a cardiac arrest and in whom resuscitation is futile.
The key event is to minimise the warm ischaemic time in the donor phase.

The kidney once removed is usually prepared on the bench in theatre by the transplant surgeron immediately prior to implantation
and factors such as accessory renal arteries and vessel length are assessed and managed.

For first time recipients the operation is performed under general anaesthesia. A Rutherford-Morrison incision is made on the
preferred side. This provides excellent extraperitoneal access to the iliac vessels. The external iliac artery and vein are dissected
out and following systemic heparinisation are cross clamped. The vein and artery are anastamosed to the iliacs and the clamps
removed. The ureter is then implanted into the bladder and a stent is usually placed to maintain patency. The wounds are then
closed and the patient recovered from surgery.

In the immediate phase a common problem encountered in cadaveric kidneys is acute tubular necrosis and this tends to resolve.
Graft survival times from cadaveric donors are typically of the order of 9 years and monozygotic twin transplant (live donor) may
survive as long as 25 years.

5. A 48 year old lady with end stage renal failure receives a cadaveric renal transplant. The organ is ABO group matched
only. On completion of the vascular anastomoses the surgeons remove the clamps. Over the course of the next twelve
minutes the donated kidney becomes dusky and swollen and appears non viable. Which of the following is the most likely
process that has caused this event?
A. IgG anti HLA Class I antibodies in the recipient
B. IgM anti HLA Class I antibodies in the recipient
C. IgG anti HLA Class I antibodies from the donor
D. IgM anti HLA Class I antibodies from the donor
E. IgM anti HLA Class II antibodies from the recipient
Answer: A
Episodes of hyperacute rejection are typically due to preformed antibodies. ABO mismatch is the best example. However, IgG
anti HLA Class I antibodies are another potential cause. These events are now seen less commonly because the cross matching
process generally takes this possibility into account.

6. A 54-year-old man who has end stage diabetic nephropathy is being assessed for a renal transplant. When assessing the
HLA matching between donor and recipient what is the most important HLA antigen to match?
A. DP
B. B
C. DR
D. C
E. A
Answer: C
Renal transplant HLA matching - DR is the most important

Renal transplant:HLA typing and graft failure: The human leucocyte antigen (HLA) system is the name given to the major
histocompatibility complex (MHC) in humans. It is coded for on chromosome 6.

Some basic points on the HLA system: Class 1 antigens include A, B and C. Class 2 antigens include DP,DQ and DR. When HLA
matching for a renal transplant the relative importance of the HLA antigens are as follows DR > B > A

Graft survival: 1 year = 90%, 10 years = 60% for cadaveric transplants. 1 year = 95%, 10 years = 70% for living-donor transplants

Post-op problems: ATN of graft. Vascular thrombosis. Urine leakage. UTI

Hyperacute acute rejection: Due to antibodies against donor HLA type 1 antigens. Rarely seen due to HLA matching

Acute graft failure (< 6 months): Usually due to mismatched HLA. Other causes include cytomegalovirus infection.
Management: give steroids, if resistant use monoclonal antibodies

Causes of chronic graft failure (> 6 months): Chronic allograft nephropathy. Ureteric obstruction. Recurrence of original renal
disease (MCGN > IgA > FSGS)

526
7. Which of the following is not typical of acute renal transplant rejection?
A. Onset 6 hours post operatively.
B. It is mediated by T lymphocytes.
C. On transplant biopsy mononuclear cell infiltrate will typically be seen.
D. It may occur in up to 50% of cases.
E. It may be treated with tacrolimus.
Answer: A
Such an early onset is more typical of hyperacute rejection and would imply pre formed antibodies.

8. Which of the following transplants is most susceptible to donor- recipient HLA mismatches?
A. Skin
B. Renal
C. Liver
D. Corneal
E. Cardiac valves
Answer: B
The kidney is highly susceptible to HLA mismatches and hyperacute rejection may occur in patients with IgG anti HLA Class I
antibodies. The liver is at far lower risk of rejection of this nature. Although the heart is susceptible to HLA mismatch the
cardiac valves are less prone to this effect. Corneal and skin grafts are reasonable interchangeable between donor and recipients.

9. Which of the following is not true of hyper acute solid organ transplant rejection?
A. Onset in the peri operative phase.
B. May occur as a result of blood group A, B or O incompatibility.
C. May be due to pre existing anti HLA antibodies.
D. On biopsy will typically show neo intimal hyperplasia of donor arterioles.
E. Complement system activation is one of the key mediators.
Answer: D
These changes are more often seen in the chronic setting. Thrombosis is more commonly seen in the hyperacute phase.

10. You review a 42-year-old woman six weeks following a renal transplant for focal segmental glomerulosclerosis.
Following the procedure she was discharged on a combination of tacrolimus, mycophenolate, and prednisolone. She has
now presented with a five day history of feeling generally unwell with anorexia, fatigue and arthralgia. On examination
she has a temperature of 37.9 and has widespread lymphadenopathy. What is the most likely diagnosis?
A. Hepatitis C
B. Epstein-Barr virus
C. HIV
D. Hepatitis B
E. Cytomegalovirus
Answer: E
Cytomegalovirus is the most common and important viral infection in solid organ transplant recipients

Primary infection with CMV typically occurs 6 weeks post transplantation in a seronegative individual who receives an organ
from a seropositive donor. Symptoms may occur as early as 20 days but can occur up to 6 months post transplant . Symptoms are
often vague, retinitis can be pathognomonic, but is rarely seen in the transplant population. CMV disease is seen in 8% of renal
transplant patients. Intravenous ganciclovir is the treatment of choice in such patients. Unfortunately, relapses are not uncommon.

Renal transplant:HLA typing and graft failure

The human leucocyte antigen (HLA) system is the name given to the major histocompatibility complex (MHC) in humans. It is
coded for on chromosome 6.

Some basic points on the HLA system: Class 1 antigens include A, B and C. Class 2 antigens include DP,DQ and DR. When HLA
matching for a renal transplant the relative importance of the HLA antigens are as follows DR > B > A
Graft survival: 1 year = 90%, 10 years = 60% for cadaveric transplants. 1 year = 95%, 10 years = 70% for living-donor transplants
Post-op problems: ATN of graft. Vascular thrombosis. Urine leakage. UTI

Hyperacute acute rejection: Due to antibodies against donor HLA type 1 antigens. Rarely seen due to HLA matching
Acute graft failure (< 6 months): Usually due to mismatched HLA. Other causes include cytomegalovirus infection.
Management: give steroids, if resistant use monoclonal antibodies
Causes of chronic graft failure (> 6 months): Chronic allograft nephropathy. Ureteric obstruction. Recurrence of original renal
disease (MCGN > IgA > FSGS)

527
11. A 43 year old lady is recovering following a live donor related renal transplant. She has significant abdominal pain.
Which of the following analgesic drugs should be avoided?
A. Paracetamol
B. Morphine
C. Nefopam
D. Diclofenac
E. Co-codamol
Answer: D
Non steroidal anti inflammatory drugs may be nephrotoxic and therefore are usually avoided in patients who have undergone
renal transplants. Paracetamol and morphine are metabolised predominantly in the liver. There is some renal contribution to
morphine metabolism and excretion and the drug should be administered in reduced doses or avoided if the transplanted kidney
stops functioning.

12. A 52 year old male attends renal transplant clinic for a post operative assessment. You note that he is on ciclosporin and
that a recent blood test shows that the ciclosporin level is elevated. Which of the following is a recognised side effect of
ciclosporin?
A. Hyperthyroidism
B. Diabetes
C. Alopecia
D. Hypothermia
E. Nephrotoxicity
Answer: E
Ciclosporin- nephrotoxicity. This patient is at risk of nephrotoxicity and should be referred to the renal team as soon as possible.
Alopecia is associated with azathioprine and diabetes is associated with tacrolimus.

Organ transplantation: immunosupressants: A number of drugs are available which help to mitigate the processes resulting in
acute rejection. Cyclosporin and tacrolimus are commonly used drugs.
Example regime: Initial: ciclosporin/tacrolimus with a monoclonal antibody. Maintenance: ciclosporin/tacrolimus with MMF or
sirolimus. Add steroids if more than one steroid responsive acute rejection episode
 Ciclosporin: Inhibits calcineurin, a phosphotase involved in T cell activation. Nephrotoxic. Monitor levels
 Azathioprine: Metabolised to form 6 mercaptopurine which inhibits DNA synthesis and cell division. Side effects
include myelosupression, alopecia and nausea
 Tacrolimus: Lower incidence of acute rejection compared to ciclosporin. Also less hypertension and hyperlipidaemia.
However, high incidence of impaired glucose tolerance and diabetes
 Mycophenolate mofetil (MMF): Blocks purine synthesis by inhibition of IMPDH. Therefore inhibits proliferation of B
and T cells. Side-effects: GI and marrow suppression
 Sirolimus (rapamycin): Blocks T cell proliferation by blocking the IL-2 receptor. Can cause hyperlipidaemia
 Monoclonal antibodies: Selective inhibitors of IL-2 receptor. Daclizumab. Basilximab

13. A 48 year old women with end stage renal failure is undergoing a live donor renal transplant. The surgeon decides to
implant the kidney in the left iliac fossa via a Rutherford Morrison incision. To which of the following vessles should the
transplanted kidney be anastomosed?
A. Aorta and inferior vena cava
B. Internal iliac artery and vein
C. Common iliac artery and vein
D. External iliac artery and vein
E. Inferior epigastric artery and vein
Answer:D
First time renal tranplants and typically implanted in the left or right iliac fossae. The vessels are usually joined to the external
iliac artery and vein as these are the most easily accessible. The Rutherford Morrison incision provides access to the external
iliac vessels.

14. A 28-year-old female undergoes a renal transplant for focal segmental glomerulosclerosis. Within hours of the operation
the patient becomes unwell with features consistent with severe systemic inflammatory response syndrome. The patient
is immediately taken back to theatre and the transplanted kidney is removed. What type of immunoglobulins are
responsible for the graft rejection?
A. IgE
B. IgM
C. IgG
D. IgD
E. IgA
Answer: C
Hyperacute graft rejection is due to pre-existent antibodies to HLA antigens and is therefore IgG mediated
528
Renal transplant:HLA typing and graft failure: The human leucocyte antigen (HLA) system is the name given to the major
histocompatibility complex (MHC) in humans. It is coded for on chromosome 6.

15. A 38 year old man is recovering following a live donor related renal transplant. The surgeon prescribes corticosteroids to
reduce the risk of graft rejection. Which of the following will not occur as a result of their administration?
A. Suppression of macrophage activation
B. Reduction of expression of major histocompatability complex antigens on the graft
C. Reduction in the proliferation of lymphocytes
D. Necrosis of activated lymphocytes
E. Reduction of expression of endothelial cell adhesion molecules
Answer: D
Corticosteroids at higher doses are able to induce apoptosis of activated lymphocytes. Necrosis is a different process and not
induced by steroids.

16. A 52 year old female underwent a cadaveric renal transplant and recovers well post operatively. Her immunosupression
regime consists of tacrolimus. Which of the substances listed below should be avoided?
A. Paracetamol
B. Apple juice
C. Penicillin
D. Prune juice
E. Grapefruit juice
Answer: E
Tacrolimus is metabolised by the P450 enzyme system. This is inhibited by a number of naturally occurring substances, these
include grapefruit, watercress and St.Johns Wort. These should all be avoided in immunosupressed patients taking tacrolimus.

Skin Lesions
Theme: Skin disorders

A. Basal cell carcinoma


B. Dermatofibroma
C. Pilar cyst
D. Epidermoid cyst
E. Spitz naevus
F. Seborrhoeic keratosis
G. Atypical naevus
H. Capillary cavernous haemangioma

Please select the most likely underlying nature of the skin lesion described. Each option may be used once, more than once or not
at all.

1. A 70 year old lady presents with a number of skin lesions that she describes as unsightly. On examination she has a
number of raised lesions with a greasy surface located over her trunk. Apart from having a greasy surface the the lesions
also seem to have scattered keratin plugs located within them.

Seborrhoeic keratosis. Seborrhoeic keratosis may have a number of appearances. However, the scaly, think, greasy surface with
scattered keratin plugs makes this the most likely diagnosis. Theme from September 2012 Exam

2. A 28 year old female presents with a small nodule located on the back of her neck. It is excised for cosmetic reasons and
the histology report states that the lesion consists of a sebum filled lesion surrounded by the outer root sheath of a hair
follicle.

The correct answer is Pilar cyst. Pilar cysts may contain foul smelling cheesy material and are surrounded by the outer part of a
hair follicle. Because of their histological appearances they are more correctly termed pilar cysts than sebaceous cysts.

3. A 21 year old lady presents with a nodule on the posterior aspect of her right calf. It has been present at the site for the
past 6 months and occurred at the site of a previous insect bite. Although the nodule appears small, on palpation it
appears to be nearly twice the size it appears on examination. The overlying skin is faintly pigmented.

529
Dermatofibroma. Dermatofibromas may be pigmented and are often larger than they appear. They frequently occur at sites of
previous trauma.

Benign skin diseases: Seborrhoeic keratosis: Most commonly arise in patients over the age of 50 years, often idiopathic. Equal
sex incidence and prevalence. Usually multiple lesions over face and trunk. Flat, raised, filiform and pedunculated subtypes are
recognized. Variable colours and surface may have greasy scale overlying it. Treatment options consist of leaving alone or simple
shave excision

Melanocytic naevi

Congenital melanocytic naevi: Typically appear at, or soon after, birth. Usually greater than 1cm diameter. Increased risk of
malignant transformation (increased risk greatest for large lesions)

Junctional melanocytic naevi: Circular macules. May have heterogeneous colour even within same lesion. Most naevi of the
palms, soles and mucous membranes are of this type

Compound naevi: Domed pigmented nodules up to 1cm in diameter. Arise from junctional naevi, usually have uniform colour
and are smooth

Spitz naevus: Usually develop over a few months in children. May be pink or red in colour, most common on face and legs. May
grow up to 1cm and growth can be rapid, this usually results in excision

Atypical naevus syndrome: Atypical melanocytic naevi that may be autosomally dominantly inherited. Some individuals are at
increased risk of melanoma (usually have mutations of CDKN2A gene. Many people with atypical naevus syndrome AND a
parent sibling with melanoma will develop melanoma

Epidermoid cysts: Common and affect face and trunk. They have a central punctum, they may contain small quantities of sebum.
The cyst lining is either normal epidermis (epidermoid cyst) or outer root sheath of hair follicle (pilar cyst)

Dermatofibroma: Solitary dermal nodules. Usually affect extremities of young adults. Lesions feel larger than they appear
visually. Histologically they consist of proliferating fibroblasts merging with sparsely cellular dermal tissues

Painful skin lesions: Eccrine spiradenoma.Neuroma. Glomus tumour. Leimyoma. Angiolipoma. Neurofibroma (rarely painful)
and dermatofibroma (rarely painful)

Theme: Skin lesion diagnosis

A. Pyogenic granuloma
B. Amelanotic melanoma
C. Dermatitis herpetiformis
D. Scabies
E. Basal cell carcinoma
F. Squamous cell carcinoma
G. Keratoacanthoma

Please select the most likely underlying diagnosis for the scenario given. Each option may be used once, more than once or not at
all.

4. A 72 year old man presents with a large nodule on his face. It is friable. There is no regional lymphadenopathy. He is lost
to follow up and re-attends several months later. On this occasion the lesion has been noted to resolve with scarring.

Keratoacanthoma. Keratoacanthomas may reach a considerable size prior to sloughing off and scarring.

5. A 22 year old girl is troubled by intensely itchy crops of blisters on her arms and legs. On examination she is
malnourished and she has papulovesicular eruptions over her elbows and knees.

Dermatitis herpetiformis.Dermatitis herpetiformis is seen in association with coeliac disease.

530
6. A 30 year old man cuts the corner of his lip whilst shaving. Over the next few days a large purplish lesion appears at the
site which bleeds on contact.

Pyogenic granuloma. Pyogenic granulomas often appear at sites of trauma.

Skin Diseases: Skin lesions may be referred for surgical assessment, but more commonly will come via a dermatologist for
definitive surgical management.

Skin malignancies include basal cell carcinoma, squamous cell carcinoma and malignant melanoma.
Basal Cell Carcinoma: Most common form of skin cancer. Commonly occur on sun exposed sites apart from the ear. Sub types
include nodular, morphoeic, superficial and pigmented. Typically slow growing with low metastatic potential. Standard surgical
excision, topical chemotherapy and radiotherapy are all successful. As a minimum a diagnostic punch biopsy should be taken if
treatment other than standard surgical excision is planned.

Squamous Cell Carcinoma: Again related to sun exposure. May arise in pre - existing solar keratoses. May metastasise if left.
Immunosupression (e.g. Following transplant), increases risk. Wide local excision is the treatment of choice and where a
diagnostic excision biopsy has demonstrated SCC, repeat surgery to gain adequate margins may be required.

Malignant Melanoma
Secondary features (minor criteria):Diameter >6mm. Inflammation. Oozing or bleeding. Altered sensation
The main diagnostic features (major criteria): Change in size. Change in shape. Change in colour

Treatment: Suspicious lesions should undergo excision biopsy. The lesion should be removed in completely as incision biopsy
can make subsequent histopathological assessment difficult. Once the diagnosis is confirmed the pathology report should be
reviewed to determine whether further re-exicision of margins is required (see below):

Margins of excision-Related to Breslow thickness


Lesions 0-1mm thick 1cm
Lesions 1-2mm thick 1- 2cm (Depending upon site and pathological features)
Lesions 2-4mm thick 2-3 cm (Depending upon site and pathological features)
Lesions >4 mm thick 3cm
Marsden J et al Revised UK guidelines for management of Melanoma. Br J Dermatol 2010 163:238-256.

Further treatments such as sentinel lymph node mapping, isolated limb perfusion and block dissection of regional lymph node
groups should be selectively applied.

Kaposi Sarcoma: Tumour of vascular and lymphatic endothelium. Purple cutaneous nodules. Associated with immunosupression.
Classical form affects elderly males and is slow growing. Immunosupression form is much more aggressive and tends to affect
those with HIV related disease.

Non malignant skin disease

Dermatitis Herpetiformis: Chronic itchy clusters of blisters. Linked to underlying gluten enteropathy (coeliac disease).

Dermatofibroma: Benign lesion. Firm elevated nodules. Usually history of trauma. Lesion consists of histiocytes, blood vessels
and fibrotic changes.

Pyogenic granuloma: Overgrowth of blood vessels. Red nodules. Usually follow trauma. May mimic amelanotic melanoma.

Acanthosis nigricans: Brown to black, poorly defined, velvety hyperpigmentation of the skin. Usually found in body folds such
as the posterior and lateral folds of the neck, the axilla, groin, umbilicus, forehead, and other areas. The most common cause of
acanthosis nigricans is insulin resistance, which leads to increased circulating insulin levels. Insulin spillover into the skin results
in its abnormal increase in growth (hyperplasia of the skin). In the context of a malignant disease, acanthosis nigricans is a
paraneoplastic syndrome and is then commonly referred to as acanthosis nigricans maligna. Involvement of mucous membranes is
rare and suggests a coexisting malignant condition

Theme: Management of skin lesions

A. Excision biopsy
B. Excision with 0.5 cm margin
C. Excision with 2 cm margin
531
D. Shave biopsy and cautery
E. Punch biopsy
F. Excision and full thickness skin graft
G. Discharge

For each skin lesion please select the most appropriate management option. Each option may be used once, more than once or not
at all.

7. A 22 year old women presents with a newly pigmented lesion on her right shin, it has regular borders and normal
appearing dermal appendages, however she reports a recent increase in size.

Excision biopsy. Likely to be a benign pigmented naevus, radical excision therefore not warranted.

8. A 58 year old lady presents with changes that are suspicious of lichen sclerosis of the perineum.

Punch biopsy. This will generate sufficient material for histological assessment.

9. A 73 year old man presents with a 1.5cm ulcerated basal cell carcinoma on his back.

Excision with 0.5 cm margin. A small lesion such as this is adequately treated by local excision. The British Association of
Dermatology guidelines suggest that excision of conventional BCC (<2cm) with margins of 3-5mm have locoregional control
rates of 85%. Morpoeic lesions have higher local recurrence rates.

10. A 29 year old man presents with a lump in his scalp. It is located approximately 4cm superior to the external occipital
protuberance. It feels smooth and slightly fluctuant and has a centrally located small epithelial defect. What is the most
likely underlying diagnosis?
A. Cocks peculiar tumour
B. Dermoid cyst
C. Sebaceous cyst
D. Merkel cell tumour
E. Seborrhoeic wart
Answer: C
Sebaceous cysts are most frequently located in the scalp and have an associated central punctum. They may become infected and
develop superficial ulceration in which case they are known as "Cocks Peculiar Tumour". The presence of a punctum is highly
suggestive of a sebaceous cyst and are not typically found in the other lesions described.

Sebaceous cysts: Originate from sebaceous glands and contain sebum. Location: anywhere but most common scalp, ears, back,
face, and upper arm (not palms of the hands and soles of the feet). They will typically contain a punctum. Excision of the cyst wall
needs to be complete to prevent recurrence. A Cock's 'Peculiar' Tumour is a suppurating and ulcerated sebaceous cyst. It may
resemble a squamous cell carcinoma- hence its name.

Theme: Dermatological manifestations of disease

A. Pyoderma gangrenosum
B. Erythroderma
C. Dermatitis herpetiformis
D. Acanthosis nigricans
E. Multiple lipomata
F. Multiple neurofibromata
G. Multiple telangectasia
H. None of the above

Please select the skin disease associated with the condition described. Each option may be used once, more than once or not at all.

11. A 22 year old man is investigated for weight loss. A duodenal biopsy taken as part of his investigations shows total villous
atrophy and lymphocytic infiltrate. He has a skin lesion that has small itchy papules.

The correct answer is Dermatitis herpetiformis. The patient has coeliac disease and this is associated with dermatitis
herpetiformis. Theme from September 2012 Exam

12. A 72 year old man is investigated for weight loss. On examination he is deeply jaundiced and cachectic. He also has a
dark velvety lesion coating his tongue.
532
The correct answer is Acanthosis nigricans.Acanthosis nigricans may be associated with GI malignancies such as gastric and
pancreatic cancer.

13. A lesion that may occur in a 32 year old man with long standing Crohns disease.

Pyoderma gangrenosum. Pyoderma gangrenosum may occur in Crohns disease.

14. Which of the following statements relating to Keloid scars is untrue?


A. They have a predilection for sternal , mandibular and deltiod area wounds
B. They are confined to the margins of the original injury
C. They often recur following excision
D. May occur even after superficial injury
E. They may be treated by injection of triamcinolone
Answer: B
Hypertrophic scars remain confined to the wound edges.

Keloids (by definition) will tend to extend beyond the margins of the wound and in wounds of any depth.

Wound healing: Surgical wounds are either incisional or excisional and either clean, clean contaminated or dirty. Although the
stages of wound healing are broadly similar their contributions will vary according to the wound type.

The main stages of wound healing include:

Haemostasis: Vasospasm in adjacent vessels, platelet plug formation and generation of fibrin rich clot.

Inflammation: Neutrophils migrate into wound (function impaired in diabetes). Growth factors released, including basic
fibroblast growth factor and vascular endothelial growth factor. Fibroblasts replicate within the adjacent matrix and migrate into
wound.Macrophages and fibroblasts couple matrix regeneration and clot substitution.

Regeneration: Platelet derived growth factor and transformation growth factors stimulate fibroblasts and epithelial cells.
Fibroblasts produce a collagen network. Angiogenesis occurs and wound resembles granulation tissue.

Remodeling: Longest phase of the healing process and may last up to one year (or longer). During this phase fibroblasts become
differentiated (myofibroblasts) and these facilitate wound contraction. Collagen fibres are remodeled. Microvessels regress
leaving a pale scar.
The above description represents an idealised scenario. A number of diseases may distort this process. It is obvious that one of the
key events is the establishing well vascularised tissue. At a local level angiogenesis occurs, but if arterial inflow and venous return
are compromised then healing may be impaired, or simply nor occur at all. The results of vascular compromise are all too
evidence in those with peripheral vascular disease or those poorly constructed bowel anastomoses.
Conditions such as jaundice will impair fibroblast synthetic function and overall immunity with a detrimental effect in most parts
of healing.

Problems with scars: Hypertrophic scars: Excessive amounts of collagen within a scar. Nodules may be present histologically
containing randomly arranged fibrils within and parallel fibres on the surface. The tissue itself is confined to the extent of the
wound itself and is usually the result of a full thickness dermal injury. They may go on to develop contractures.

Keloid scars: Excessive amounts of collagen within a scar. Typically a keloid scar will pass beyond the boundaries of the original
injury. They do not contain nodules and may occur following even trivial injury. They do not regress over time and may recur
following removal.

Note the extension beyond the boundaries of the original incision: Drugs which impair wound healing: Non steroidal anti
inflammatory drugs. Steroids. Immunosupressive agents. Anti neoplastic drugs. Closure: Delayed primary closure is the
anatomically precise closure that is delayed for a few days but before granulation tissue becomes macroscopically evident.
Secondary closure refers to either spontaneous closure or to surgical closure after granulation tissue has formed.

15. A 72 year old man presents with a lesion on his back. Its appearances are as shown below.What is the most likely
diagnosis?
A. Amelanotic malignant melanoma
B. Squamous cell carcinoma
C. Merkel Cell tumour
533
D. Basal cell carcinoma
E. None of the above
Answer: D
The lesion has all the characteristic features of a basal cell carcinoma. Including raised surface and overlying telangectasia.
Amelanotic melanomas are rare lesions and usually have a more ulcerated appearance. Since the question is directed towards the
most likely diagnosis the correct answer is basal cell carcinoma.
Clinical images are not currently part of the MRCS Part A

Theme: Management of skin diseases

A. Excision biopsy
B. Excision with 1 cm margin
C. Excision with 5 cm margin
D. Shave biopsy and cautery
E. Punch biopsy
F. Excision and full thickness skin graft
G. Discharge

For each scenario please select the most appropriate management option. Each option may be used once, more than once or not at
all.

16. A 89 year old women presents with long standing seborrhoeic warts of her abdominal wall , they have caused
troublesome itching.

The correct answer is Shave biopsy and cautery. These lesions are often extensive and superficial. Shave excision will suffice,
material must be sent for histology.

17. A 22 year old man has an excision biopsy of a pigmented lesion from his back, histology shows a 1mm depth nodular
melanoma, all resection margins are clear of tumour and the nearest is 0.5cm.

Excision with 1 cm margin. This man will require re-excision of margins so that a 1cm margin around the lesion is achieved.
This can usually be achieved without skin grafting.

18. 18. A 73 year old lady presents to the breast clinic with a weeping crusty skin lesion of the left nipple. There are no
masses to feel in the breast itself and imaging is normal.

The correct answer is Punch biopsy. This is likely to represent Pagets disease of the nipple and is best diagnosed on punch
biopsy.

19. Which of the following statements relating to sebaceous cysts is false?


A. When infected are also known as Cocks peculiar tumour
B. Typically contain pus
C. Are usually associated with a central punctum
D. Most commonly occur on the scalp
E. They will typically have a cyst wall
Answer: B
Sebaceous cysts usually contain sebum, pus is only present in infected sebaceous cysts which should then be treated by surgical
incision and drainage.

Head & Neck Surgery


1. A 46-year-old woman is referred to endocrine surgery for a possible thyroidectomy. She has a tender neck swelling.
Blood results are as follows: TSH is <0.1 mU/l, T4 188 nmol/l, Hb 14.2 g/dl, Plt 377 * 10^9/l, WBC 6.4 * 10^9/l and
the is ESR 65 mm/hr; Technetium thyroid scan shows decreased uptake globally. What is the most likely diagnosis?

534
A.Sick thyroid syndrome
B.Acute bacterial thyroiditis
C.Hashimoto's thyroiditis
D.Subacute thyroiditis
E.Toxic multinodular goitre

Answer: D

This patient does not need surgery! Subacute thyroiditis is suggested by the tender goitre, hyperthyroidism and raised ESR. The
globally reduced uptake on technetium thyroid scan is also typical. This should resolve without any active intervention.

Thyroiditis

Sub acute thyroiditis


Subacute thyroiditis (also known as De Quervain's thyroiditis) is thought to occur following viral infection and typically presents
with hyperthyroidism

Features: Hyperthyroidism; Painful goiter; Raised ESR; Globally reduced uptake on iodine-131 scan

Management: Usually self-limiting - most patients do not require treatment. Thyroid pain may respond to aspirin or other
NSAIDs. In more severe cases steroids are used, particularly if hypothyroidism develops

Hashimotos thyroiditis
Hashimotos thyroiditis is an immunological disorder in which lymphocytes become sensitised to thyroidal antigens. The three
most important antibodies include; thyroglobulin, TPO and TSH-R. During the early phase of Hashimotos the thyroglobulin
antibody is markedly elevated and then declines.

Features: Goitre and either euthyroid or mild hypothyroidism. Progressive hypothyroidism (and associated symptoms)

Management: During the hyperthyroid phase of illness beta blockers may manage symptoms. As hypothyroidism develops
patients may require thyroxine

Theme: Voice disorders

A. Vagus nerve injury


B. Thyroid nerve injury
C. Superior laryngeal nerve injury
D. Unilateral inferior laryngeal nerve injury
E. Bilateral inferior laryngeal nerves injuries
F. Stroke
G. Lacunar infarcts
H. None of the above

Please select the most likely reason for the scenarios given. Each option may be used once, more than once or not at all.

2. A 42 year old singer is admitted for a thyroidectomy. Post operatively she is only able to make a gargling noise. Her
voice sounds breathy.

Answer: Unilateral inferior laryngeal nerve injury


This patient has diplophonia which causes a gargling sound. This is associated with dysphagia. This can also be caused by a
vagus nerve lesion, but the recurrent laryngeal nerve is more at risk of damage.

3. A 42 year old singer is admitted for a thyroidectomy. Post operatively she is unable to sing high pitched notes

Answer: Superior laryngeal nerve injury

SLN lesions cause difficulty in voice pitch.

4. A 42 year old singer is admitted for a thyroidectomy. Post operatively the patient develops dyspnoea and is unable to
speak.

535
Answer: Bilateral inferior laryngeal nerves injuries
This patient has aphonia due to bilateral damage to the recurrent laryngeal nerve.

Voice production
There are 2 main nerves involved:
Superior laryngeal nerve (SLN)
 Innervates the cricothyroid muscle
Since the cricothyroid muscle is involved in adjusting the tension of the vocal fold for high notes during singing, SLN paresis and
paralysis result in:
a. Abnormalities in pitch
b. Inability to sing with smooth change to each higher note (glissando or pitch glide)

Recurrent laryngeal nerve (RLN)/Inferior laryngeal nerve


 Innervates intrinsic larynx muscles
a. Opening vocal folds (as in breathing, coughing)
b. Closing vocal folds for vocal fold vibration during voice use
c. Closing vocal folds during swallowing

Theme: Nasal diseases

A. Ethmoid sinus cancer


B. Maxillary sinus cancer
C. Ethmoid adenoma
D. Maxillary adenoma
E. Ethmoidal fracture
F. Nasal polyps
G. Sphenoid osteoma
H. Ethmoidal sinusitis
I. Maxillary sinusitis

Please select the most likely diagnosis for the scenario given. Each option may be used once, more than once or not at all.

5. A 56 year old man presents with symptoms of nasal pain, anosmia and rhinorrhea. He has been well until recently and
has worked as a wood carver for many years.
Answer: Ethmoid sinus cancer
Nasopharyngeal cancer is strongly associated with wood work. Most cases require an occupational exposure of greater than
10 years and are adenocarcinomas on histology. Most cases are ethmoidal in origin

6. A 32 year old female presents with recurrent episodes of rhinorrhoea, the discharge is watery. She has a medical
history of asthma and intolerance of aspirin. On examination she has multiple soft, semi- transparent polyps within
her nasal cavity.

Answer: Nasal polyps


The combination of nasal polyps and atopy is well described. Some cases will respond favourably to systemic steroids and
avoid surgery.

7. A child is brought to casualty complaining of a headache and a sensation of pressure between the eyes. On
examination she is febrile with a smooth swelling overlying the superomedial aspect of the right eye. The eye is
uncomfortable and there is a purulent discharge from the inner canthus.

Answer: Ethmoidal sinusitis


Ethmoidal sinusitis may spread to the periorbital tissues resulting in periorbital cellulitis. The superomedial distribution
makes a maxillary sinusitis less likely.

Diseases of nose and sinuses


Benign Tumours
 Simple papillomas may be an incidental finding or present with obstructive symptoms. Excision under general
anaesthesia is sufficient management.

536
 Transitional cell papillomas may be more extensive and produce obstructive symptoms. Erosion of local structures is a
recognised complication. These lesions may rarely undergo malignant transformation and therefore careful and complete
excision is required, some cases may require partial or total maxillectomy.
 Pleomorphic adenomas of the maxillary sinuses are reported but are extremely rare, their symptoms typically include
nasal obstruction and pain if the sinus is obstructed. Treatment is by complete surgical excision, the diagnosis is not
infrequently made post operatively.
 Benign osteomas may develop in the paranasal sinuses, the frontal sinus is the most frequent location of such lesions.
Symptoms include; pain, rhinorrhoea and anosmia. Most osteomas may be observed if asymptomatic, sphenoid osteomas
should be resected soon after diagnosis as enlargement may compromise visual fields. Many sinus osteomas can now be
resected endoscopically, complete surgical resection is required.
 Nasal polyps are benign lesions of the ethmoid sinus mucosa. Many patients may also have asthma, cystic fibrosis and a
sensitivity to aspirin. Symptoms include watery rhinorrhoea, infection and anosmia. The polyps are usually a semi
transparent grey mass. They are rare in childhood. Treatment is either with systemic steroids or surgical resection. The
latter should be combined with antral washout. Low dose, nasal, steroid drops may reduce the risk of recurrence.

Malignant disease
 Malignancies encountered in the nose and paranasal sinuses include, adenoid cystic carcinoma, squamous cell carcinoma
and adenocarcinoma.
 Adenocarcinoma of the paranasal sinuses and nasopharynx is strongly linked to exposure to hard wood dust (after >10
years exposure).
 Adenoid cystic carcinoma usually originate in the smaller salivary glands.
 The majority of cancers (50%) arise from the lateral nasal wall, a smaller number (33%) arise from the maxillary antrum,
ethmoid and sphenoid cancers comprise only 7%.
 Signs of malignancy on clinical examination include loose teeth, cranial nerve palsies and lymphadenopathy.
 Nasopharyngeal cancers are most common in individuals presenting from China and Asia and are linked to viral
infection with Epstein Barr Virus. Radiotherapy and chemotherapy are the most commonly used modalities.

Maxillary sinusitis
 Common symptoms include post nasal discharge, pain, headache and toothache.
 Imaging may show a fluid level in the antrum.
 Common organisms include Haemophilus influenzae or Streptococcus pneumoniae.
 Treatment with antral lavage may facilitate diagnosis and relieve symptoms. Antimicrobial therapy has to be continued
for long periods. Antrostomy may be needed.

Frontoethmoidal sinusitis
 Usually presents with frontal headache, nasal obstruction and altered sense of smell.
 Inflammation may progress to involve periorbital tissues. Ocular symptoms may occur and secondary CNS involvement
brought about by infection entering via emissary veins.
 CT scanning is the imaging modality of choice. Early cases may be managed with antibiotics. More severe cases usually
require surgical drainage.

8. Which of the following statements relating to branchial cysts is untrue?


A.The greater auricular nerve may be divided during excision
B.They typically occur in young adults
C.They move upwards on swallowing
D.They are rare over the age of 40 years
E.They are usually located in the anterior triangle of the neck
Answer: C

They do not move on swallowing. They should be diagnosed with caution in those aged >40 years, as lumps in this age group may
in fact be metastatic disease from ENT malignancy.

Neck lumps
The table below gives characteristic exam question features for conditions causing neck lumps:

Reactive By far the most common cause of neck swellings. There may be a history of local infection or a
lymphadenopathy generalised viral illness
Lymphoma Rubbery, painless lymphadenopathy
The phenomenon of pain whilst drinking alcohol is very uncommon
There may be associated night sweats and splenomegaly
Thyroid swelling May be hypo-, eu- or hyperthyroid symptomatically
Moves upwards on swallowing

537
Thyroglossal cyst More common in patients < 20 years old
Usually midline, between the isthmus of the thyroid and the hyoid bone
Moves upwards with protrusion of the tongue
May be painful if infected
Pharyngeal pouch More common in older men
Represents a posteromedial herniation between thyropharyngeus and cricopharyngeus muscles
Usually not seen, but if large then a midline lump in the neck that gurgles on palpation
Typical symptoms are dysphagia, regurgitation, aspiration and chronic cough
Cystic hygroma A congenital lymphatic lesion (lymphangioma) typically found in the neck, classically on the left
side
Most are evident at birth, around 90% present before 2 years of age
Branchial cyst An oval, mobile cystic mass that develops between the sternocleidomastoid muscle and the pharynx
Develop due to failure of obliteration of the second branchial cleft in embryonic development
Usually present in early adulthood
Cervical rib More common in adult females
Around 10% develop thoracic outlet syndrome
Carotid aneurysm Pulsatile lateral neck mass which doesn't move on swallowing

9. A 22 year old female attends clinic after noticing a painless neck lump. On examination she is noted to have bilateral
thyroid masses and multicentric nodes near the base of the thyroid. Her corrected Ca is 2.18. What is the most likely
diagnosis?
A.Sporadic medullary carcinoma of the thyroid
B.Medullary carcinoma of the thyroid associated with multiple endocrine neoplasia
C.Follicular thyroid carcinoma
D.Anaplastic thyroid carcinoma
E.Toxic nodular goitre
Answer: B
Medullary thyroid cancer is a tumour of the parafollicular cells of the thyroid. Less than 10% of thyroid cancers are of this type.
Patients typically present in children or young adults. Diarrhoea occurs in 30% of cases. Toxic nodular goitre are very rare. In
sporadic medullary thyroid cancer, patients typically present with a unilateral solitary nodule and it tends to spread early to the
neck lymph nodes. In association with multiple endocrine neoplasia (MEN) syndromes, medullary thyroid cancers are always
bilateral and multicentric. It may be the presenting feature in MEN 2a and 2b; almost all MEN 2a patients develop medullary
thyroid carcinoma.

Thyroid malignancy

Papillary carcinoma
 Commonest sub-type
 Accurately diagnosed on fine needle aspiration cytology
 Histologically they may demonstrate psammoma bodies (areas of calcification) and so called 'orphan Annie' nuclei
 They typically metastasise via the lymphatics and thus laterally located apparently ectopic thyroid tissue is usually a
metastasis from a well differentiated papillary carcinoma.

Follicular carcinoma
 Are less common than papillary lesions
 Like papillary tumours they may present as a discrete nodule. Although they appear to be well encapsulated
macroscopically there invasion on microscopic evaluation.
 Lymph node metastases are uncommon and these tumours tend to spread haematogenously. This translates into a higher
mortality rate.
 Follicular lesions cannot be accurately diagnosed on fine needle aspiration cytology and thus all follicular FNA's will
require at least a hemi thyroidectomy.

Anaplastic carcinoma
 Less common and tend to occur in elderly females
 Disease is usually advanced at presentation and often only palliative decompression and radiotherapy can be offered.

Medullary carcinoma
 These are tumours of the parafollicular cells ( C Cells) and are of neural crest origin.
 The serum calcitonin may be elevated which is of use when monitoring for recurrence.
 They may be familial and occur as part of the MEN -2A disease spectrum.

538
 Spread may be either lymphatic or haematogenous and as these tumours are not derived primarily from thyroid cells they
are not responsive to radioiodine.

Lymphoma

 These respond well to radiotherapy


 Radical surgery is unnecessary once the disease has been diagnosed on biopsy material. Such biopsy material is not
generated by an FNA and thus a core biopsy has to be obtained (with care!).

Theme: Parotid gland disorders

A. Sialectasis
B. Pleomorphic adenoma
C. Bacterial parotitis
D. Viral parotitis
E. Sjogren's syndrome
F. Adenoid cystic carcinoma
G. Mucoepidermoid carcinoma
H. Warthins tumour

Please select the most likely diagnosis for the scenario given. Each option may be used once, more than once or not at all.

10. A 40 year old lady with longstanding rheumatoid arthritis presents with a dry mouth and parotid gland swelling. Her
symptoms transiently improved following administration of a course of steroids for her rheumatoid disease.

Answer: Sjogren's syndrome


Sjogrens syndrome is linked to other autoimmune conditions such as rhematoid disease. A transient improvement may be
seen with steroid administration. Most patients are treated with artificial saliva.

11. A 77 year old lady presents with facial pain and on examination is found to have clinical evidence of a facial nerve
palsy and a parotid mass. Following surgical excision of the lesion the histological report comments on extensive
perineural invasion.
Answer: Adenoid cystic carcinoma
Extensive perineural invasion is most commonly seen in patients with adenoid cystic carcinoma. Both Warthins tumours and
mucoepidermoid carcinoma rarely show such marked perineural infiltration.

12. An 18 year old boy presents with pancreatitis. He has bilateral painful parotid enlargement.

Answer: correct answer is Viral parotitis


In a young adult with parotid swelling and pancreatitis/orchitis/reduced hearing/meningoencephalitis suspect mumps.

Parotid gland clinical


Causes of bilateral parotid enlargement

 Mumps: Associated with meningoencephalitis,  Myxoedema


pancreatitis, orchitis, or deafness  Cushing's disease
 Parotitis  Diabetes/insulin resistance
 Sialectasis - especially if related to eating  Liver cirrhosis
 Sjogren's syndrome: dry eyes or mouth, connective  Gout
tissue disease  Bulimia nervosa
 Sarcoidosis  Drugs
 Tuberculosis  Severe dehydration
 Alcoholism  Malnutrition

Causes of unilateral parotid enlargement


 Salivary calculus
 Tumour

Parotid gland tumours


 Pleomorphic adenomas are the most common.
539
 Incisional biopsy of parotid masses is not recommended, so superficial parotidectomy is the usual procedure of choice.
 Signs of facial nerve palsy and a parotid mass should raise suspicion of malignancy.
 Warthins tumours are relatively benign lesions that are slow growing and occur most commonly in elderly male smokers.
 Adenoid cystic carcinoma have a tendency for perineural invasion.

Theme: Neck lumps

A. Dermoid cyst
B. Thyroglossal cyst
C. Sjogren's syndrome
D. Mikulicz's syndrome
E. Pleomorphic adenoma of the parotid
F. Carcinoma of the parotid
G. Cystic hygroma
H. Branchial cyst
I. Pharyngeal pouch

Please select the most likely diagnosis for the scenario given. Each option may be used once, more than once or not at all.

13. A dentist treating a women with rheumatoid arthritis for recurrent episodes of dental sepsis notices that both parotid
and submandibular glands are symmetrically enlarged.

Answer: Sjogren's syndrome


Sjogren's is associated with autoimmune disorders. Mikulicz's is similar but there is no sicca or arthritis.

14. A patient presents with a facial nerve palsy. This occurred following repeat excision of a facial lump. The histology
report remarks on the biphasic appearance of the lesion and mucinous connective tissue.

Answer: Pleomorphic adenoma of the parotid


The histological features are as described with a classic biphasic (mixed stromal and epithelial elements), although benign
local recurrence can complicate incomplete excision. As this is a benign lesion direct extension into the facial nerve is
unlikely to occur. Facial nerve injury can happen during repeat parotid surgery.

15. A patient is recovering from a Sistrunk's procedure, what lesion was treated with this operation?

Answer: Thyroglossal cyst


This is the procedure for excision of the cyst and its associated track. Excision must be complete and thus a small segment of
the hyoid bone is removed to gain access to the upper part of the cyst tract.

16. A 44 year old lady presents with epistaxis. From which of the following regions is the bleeding most likely to originate?
A. Posterior nasal space
B. Alar rim
C. Kisselbach's plexus
D. Cribriform plate
E. None of the above
Answer: C

Kisselbachs plexus has an arterial supply derived from both the internal and external carotid arteries and is the commonest area for
bleeding in idiopathic epistaxis.

Epistaxis

540
Usually trivial and insignificant but severe haemorrhage may compromise airway and pose a risk to life.

Anatomy:
Arterial supply

 From internal and external carotid


 An arterial plexus exists at Little's area and is the source of bleeding in 90% cases
 Major arterial supply is from the sphenopalatine and greater palatine arteries (branches of the maxillary artery)
 The facial artery supplies the more anterior aspect of the nose
 Ethmoidal arteries are branches of the ophthalmic artery. They supply the posterosuperior nasal cavity

Venous drainage follows the arterial pattern

Classification

 Primary idiopathic epistaxis accounts for 75% of all cases


 Secondary cases arise as a result of events such as anticoagulants, trauma and coagulopathy
 Classification into anterior and posterior epistaxis may help to locate the source and becomes more important when
invasive treatment is required

Management

 Resuscitate if required
 Subject should sit upright and pinch nose firmly
 Nasal cavity should be examined using a headlight
 Simple anterior epistaxis may be managed using silver nitrate cautery. If difficult to manage then custom manufactured
packs may be inserted
 Posterior packing or tamponade may be achieved by passing a balloon tamponade device and inflating it. This is
indicated where anterior packing along has failed to achieve haemostasis.
 Post nasal pack patients should receive antibiotics
 Failure of these methods will require more invasive therapy. Where a vascular radiology suite is available consideration
may be given to angiographic techniques. Direct ligation of the nasal arterial supply may also be undertaken. Of the
arterial ligation techniques available the endo nasal sphenopalatine arterial ligation procedure is most popular.

17. A 56 year old man presents with a painless swelling in the upper part of the anterior triangle of his neck. On
examination a mass lesion involving the sub mandibular gland is identified. On CT scanning this is shown to be a solid
lesion. There is no regional lymphadenopathy. Two fine needle aspirates have failed to be diagnostic. Which of the
following is the most appropriate management option?
A.Sub mandibular gland excision
B.Incisional biopsy of the mass
C.Manage conservatively and repeat the CT scan in 6 months
D.Sub mandibular gland excision and radical neck dissection
E.Diagnostic excision of the superficial lobe of the submandibular gland

Answer: A

There is a 50% risk that this lesion is malignant (in some series up to 70%). Therefore the gland should be excised entirely. At this
stage a radical neck dissection is not justified.

Submandibular glands- disease

Physiology
The submandibular glands secrete approximately 800- 1000ml saliva per day. They typically produce mixed seromucinous
secretions. When paraympathetic activity is dominant the secretions will be more serous. The parasympathetic fibres are derived
from the chorda tympani nerves and the submandibular ganglion, they travel to the glands via the lingual nerves.

Sialolithiasis

 80% of all salivary gland calculi occur in the submandibular gland


 70% of the these calculi are radio-opaque
 Stones are usually composed of calcium phosphate or calcium carbonate
 Patients typically develop colicky pain and post prandial swelling of the gland
541
 Investigation involves sialography to demonstrate the site of obstruction and associated other stones
 Stones impacted in the distal aspect of Whartons duct may be removed orally, other stones and chronic inflammation will
usually require gland excision

Sialadenitis

 Usually occurs as a result of Staphylococcus aureus infection


 Pus may be seen leaking from the duct, erythema may also be noted
 Development of a sub mandibular abscess is a serious complication as it may spread through the other deep fascial
spaces and occlude the airway

Submandibular tumours

 Only 8% of salivary gland tumours affect the sub mandibular gland


 Of these 50% are malignant (usually adenoid cystic carcinoma)
 Diagnosis usually involves fine needle aspiration cytology
 Imaging is with CT and MRI
 In view of the high prevalence of malignancy, all masses of the submandibular glands should generally be excised.

18. A 36-year-old woman who presented with a goitre is diagnosed with autoimmune thyroiditis. Which one of the
following types of thyroid cancer is she predisposed to developing?
A. Anaplastic
B. Lymphoma
C. Medullary
D. Follicular
E. Papillary
Answer: B
Hashimoto's thyroiditis is associated with thyroid lymphoma

Rarely, a patient with Hashimotos thyroiditis may develop a lymphoma of the thyroid gland. The exact aetiology of thyroid gland
lymphoma is unknown. Hashimotos thyroiditis is a definite risk factor. It is possible that the lymphoma may represent the
exapnsion of a clone of immortalised intrathyroidal lymphocytes. Lymphoma of the gland is characterised by rapid thyroid growth
despite T4 therapy. Whilst adenocarcinoma of the thyroid may occur in association with thyroiditis there are no studies, to date,
showing a correlation between these two conditions. The commonest sequelae of thyroiditis is hypothyroidism.

19. Which of the following are not typical of papillary carcinoma of the thyroid?
A. Haematogenous metastases are common.
B. It has an overall mortality rate of 11%.
C. Histology typically shows 'orphan Annie-eyed' nuclei.
D. They are seldom encapsulated.
E. They are more common in females.

Answer: A

Haematogenous metastasis is rare. Hence the low mortality rate. Lymphatic spread is more common.

20. Which of the following does not cause parotid enlargement?


A. Liver cirrhosis
B. Myxoedema
C. Amiodarone
D. Tuberculosis
E. Sjogrens syndrome

Answer: C

Drugs commonly implicated in parotid gland enlargement include:


Thiouracil, isoprenaline, phenylbutazone, high oestrogen dose contraceptive pills.

21. At which of the following time frames is secondary haemorrhage most likely to occur following tonsillectomy?
A.Between 5 and 10 days after surgery
B.During the first 6 hours after surgery

542
C.Between 6 and 12 hours after surgery
D.Upon resumption of normal feeding
E.Between 2 and 3 days post operatively
Answer: A
Haemorrhage in the first 6 hours after surgery is termed reactionary haemorrhage. Feeding does not increase the risk and may
actually lower the risks of infection developing.

Secondary haemorrhage after tonsillectomy


Haemorrhage is a feared complication following tonsillectomy. Primary, or reactionary haemorrhage most commonly occurs in
the first 6-8 hours following surgery. It is managed by immediate return to theatre.
Secondary haemorrhage occurs between 5 and 10 days after surgery, it is often associated with a wound infection. Treatment is
usually with admission and antibiotics. Severe bleeding may require surgery. Secondary haemorrhage occurs in 3% of all
tonsillectomies.

22. A 8 year old boy with recurrent attacks of otitis media is suspected of developing a glue ear. If his sound conduction is
tested, which of the following is most consistent with a unilateral middle ear effusion?
A. Negative Rinne's test on the ipsilateral side
B. Positive Rinne's test on the ipsilateral side
C. Positive Webers and Rinnes tests on the ipsilateral side
D. Positive Rinne's test on the contralateral side
E. Negative Webers test only on the contralateral side
Answer:A
Rinne's test will localise to the affected side (i.e. it is negative in conductive deafness). In a positive Rinne's test sound heard by
air conduction is better than that conveyed by bone conduction. Reduction of both air and bone conduction in equal measure is a
feature of sensorineural hearing loss.

Hearing Loss
Hearing loss may be conductive or sensorineural. To determine which is present patients will often require a formal assessment
with pure tone audiometry. In the clinical setting Webers and Rinnes tests may be helpful in categorising various types of hearing
loss.

Webers and Rinnes Tests


In a normal patient, the Weber tuning fork sound is heard equally loud in both ears with no one ear hearing the sound louder than
the other. A patient with symmetrical hearing loss will hear the Weber tuning fork sound equally well with diagnostic utility only
in asymmetric (one-sided) hearing losses. In a patient with assymetrical hearing loss, the Weber tuning fork sound is heard louder
in one ear versus the other. This clinical finding should be confirmed by repeating the procedure and having the patient occlude
one ear with a finger; the sound should be heard best in the occluded ear.

Rinne Test Weber without Weber lateralises to left Weber lateralises to right
lateralisation
Both ears Normal Sensorineural loss on right Sensorineural loss on left
Air>Bone
Left Bone > Air Conductive loss on left Combined loss on left
Right Bone> Air Combined loss on right Conductive loss on right
Both Bone > Air Combined loss on right and Combined loss on left and conductive
conductive on left on right

Theme: Head and neck lesions

A. Dermoid cyst
B. Thyroglossal cyst
C. Sjogren's syndrome
D. Mikulicz's syndrome
E. Pleomorphic adenoma of the parotid
F. Carcinoma of the parotid
G. Cystic hygroma
H. Branchial cyst
I. Pharyngeal pouch

Please select the most likely diagnosis for the scenario given. Each option may be used once, more than once or not at all.

543
23. The parents of a 2 year old child are concerned after he develops a lump in his neck. On examination there is a
swelling in the subcutaneous tissue of the posterior triangle which transilluminates.

Answer: Cystic hygroma


This is a classical description. Collection of dilated lymphatic sacs which are fluctuant and brilliantly transilluminable.
Recurrence is common.
24. A 40 year old female presents with a painless neck lump. There is a mass noted beneath the sternocleidomastoid
muscle. There is a long history and somewhat unkindly her husband remarked on her rather noticeable halitosis.

Answer: Pharyngeal pouch


Usual history of regurgitation of undigested food or coughing at night. Associated with halitosis and throat infections.

25. A 32 year old lady is referred to the clinic with recurrent infections and abscesses in the neck. On examination she has
a midline defect with an overlying scab, which moves upwards on tongue protrusion.

Answer: Thyroglossal cyst


Thyroglossal cyst is always located in the midline as it is this route that the thyroid takes during its embryological descent. Its
connection with the foramen caecum means it will move on tongue protrusion.
Similar theme question in September 2011 exam
Orthopedics
Theme: Spinal disorders

A. Osteomyelitis
B. Potts disease of the spine
C. Scheuermannas disease
D. Transverse myelitis
E. Tabes dorsalis
F. Subacute degeneration of the cord
G. Brown-Sequard syndrome
H. Syringomyelia
I. Epidural haematoma

Which is the most likely diagnosis for the scenario given. Each option may be used once, more than once or not at all.

1. A 68 year old man presents to the plastics team with severe burns to his hands. He is not distressed by the burns. He has
bilateral charcot joints. On examination there is loss of pain and temperature sensation of the upper limbs
The correct answer is Syringomyelia
This patient has syringomyelia which selectively affects the spinotholamic tracts.Syringomyelia is a disorder in which a cystic
cavity forms within the spinal cord. The commonest variant is the Arnold- Chiari malformation in which the cavity connects
with a congenital malformation affecting the cerebellum. Acquired forms of the condition may occur as a result of previous
meningitis, surgery or tumours. Many neurological manifestations have been reported, although the classical variety spares the
dorsal columns and medial lemniscus and affecting only the spinothalamic tract with loss of pain and temperature sensation. The
bilateral distribution of this patients symptoms would therefore favor syringomyelia over SCID or Brown Sequard syndrome.
Osteomyelitis would tend to present with back pain and fever in addition to any neurological signs. Epidural haematoma large
enough to produce neurological impairment will usually have motor symptoms in addition to any selective sensory loss, and the
history is usually shorter.

2. A 24 year old man presents with localised spinal pain over 2 months which is worsened on movement. He is known to be
an IVDU. He has no history suggestive of tuberculosis. The pain is now excruciating at rest and not improving with
analgesia. He has a temperature of 39 oC.
Answer: Osteomyelitis
In an IVDU with back pain and pyrexia have a high suspicion for osteomylelitis. The most likely organism is staph aureus and
the cervical spine is the most common region affected. TB tends to affect the thoracic spine and in other causes of osteomyelitis
the lumbar spine is affected.

3. A 22 year man is shot in the back, in the lumbar region. He has increased tone and hyper-reflexia of his right leg. He
cannot feel his left leg.
Answer: Brown-Sequard syndrome
Brown -Sequard syndrome is caused by hemisection of the spinal cord. It may result from stab injuries or lateral vertebral
fractures. It results in ipsilateral paralysis (pyramidal tract) , and also loss of proprioception and fine discrimination (dorsal
columns). Pain and temperature sensation are lost on the contra-lateral side. This is because the fibres of the spinothalamic tract
have decussated below the level of the cord transection.

544
Spinal disorders
 Dorsal column lesion: Loss vibration and proprioception. Tabes dorsalis, SACD
 Spinothalamic tract lesion: Loss of pain, sensation and temperature
 Central cord lesion: Flaccid paralysis of the upper limbs
 Osteomyelitis: Normally progressive. Staph aureus in IVDU, normally cervical region affected. Fungal infections in
immunocompromised. Thoracic region affected in TB
 Infarction spinal cord: Dorsal column signs (loss of proprioception and fine discrimination)
 Cord compression: UMN signs. Malignancy. Haematoma. Fracture
 Brown-sequard syndrome: Hemisection of the spinal cord. Ipsilateral paralysis. Ipsilateral loss of proprioception and
fine discrimination. Contralateral loss of pain and temperature

Dermatomes

 C2 to C4 The C2 dermatome covers the occiput and the top part of the neck. C3 covers the lower part of the neck to the
clavicle. C4 covers the area just below the clavicle.
 C5 to T1 Situated in the arms. C5 covers the lateral arm at and above the elbow. C6 covers the forearm and the radial
(thumb) side of the hand. C7 is the middle finger, C8 is the lateral aspects of the hand, and T1 covers the medial side of
the forearm.
 T2 to T12 The thoracic covers the axillary and chest region. T3 to T12 covers the chest and back to the hip girdle. The
nipples are situated in the middle of T4. T10 is situated at the umbilicus. T12 ends just above the hip girdle.
 L1 to L5 The cutaneous dermatome representing the hip girdle and groin area is innervated by L1 spinal cord. L2 and 3
cover the front part of the thighs. L4 and L5 cover medial and lateral aspects of the lower leg.
 S1 to S5 S1 covers the heel and the middle back of the leg. S2 covers the back of the thighs. S3 cover the medial side of
the buttocks and S4-5 covers the perineal region. S5 is of course the lowest dermatome and represents the skin
immediately at and adjacent to the anus.

Myotomes

Upper limb
Lower limb Elbow flexors/Biceps C5
Hip flexors (psoas) L1 and L2 Wrist extensors C6
Knee extensors (quadriceps) L3 Elbow extensors/Triceps C7
Ankle dorsiflexors (tibialis anterior) L4 and L5 Long finger flexors C8
Toe extensors (hallucis longus) L5 Small finger abductors T1
Ankle plantar flexors (gastrocnemius) S1

The anal sphincter is innervated by S2,3,4

4. A 24 year old man is brought to the emergency department have suffered a crush injury to his forearm. Assessment
demonstrates that the arm is tender, red and swollen. There is clinical evidence of an ulnar fracture and the patient
cannot move their fingers. Which is the most appropriate course of action?
A. Application of an external fixation device
B. Closed reduction
C. Debridement
D. Discharge and review in fracture clinic
E. Fasciotomy
Answer: E
The combination of a crush injury, limb swelling and inability to move digits should raise suspicion of a compartment syndrome
that will require a fasciotomy

Compartment syndrome: This is a particular complication that may occur following fractures (or following ischaemia
reperfusion injury in vascular patients). It is characterised by raised pressure within a closed anatomical space. The raised pressure
within the compartment will eventually compromise tissue perfusion resulting in necrosis. The two main fractures carrying this
complication include supracondylar fractures and tibial shaft injuries.

Symptoms and signs: Pain, especially on movement (even passive). Parasthesiae. Pallor may be present. Arterial pulsation may
still be felt as the necrosis occurs as a result of microvascular compromise. Paralysis of the muscle group may occur

545
Diagnosis: Is made by measurement of intracompartmental pressure measurements. Pressures in excess of 20mmHg are abnormal
and >40mmHg is diagnostic.

Treatment: This is essentially prompt and extensive fasciotomies. In the lower limb the deep muscles may be inadequately
decompressed by the inexperienced operator when smaller incisions are performed. Myoglobinuria may occur following
fasciotomy and result in renal failure and for this reason these patients require aggressive IV fluids. Where muscle groups are
frankly necrotic at fasciotomy they should be debrided and amputation may have to be considered. Death of muscle groups may
occur within 4-6 hours

Theme: Disorders of the knee

A. Chondromalacia patellae
B. Dislocated patella
C. Undisplaced fracture patella
D. Displaced patella fracture
E. Avulsion fracture of the tibial tubercle
F. Quadriceps tendon rupture
G. Osgood Schlatters disease

Please select the most likely explanation for the scenario given. Each option may be used once, more than once or not at all.

5. A 19 year old sportswoman presents with knee pain which is worse on walking down the stairs and when sitting still. On
examination there is wasting of the quadriceps and pseudolocking of the knee.
Answer is Chondromalacia patellae
A teenage girl with knee pain on walking down the stairs is characteristic for chondromalacia patellae(anterior knee pain). Most
cases are managed with physiotherapy.

6. A tall 18 year old male athlete is admitted to the emergency room after being hit in the knee by a hockey stick. On
examination his knee is tense and swollen. X-ray shows no fractures.
Answer is Dislocated patella
A patella dislocation is a common cause of haemarthrosis and many will spontaneously reduce when the leg is straightened. In
the chronic setting physiotherapy is used to strengthen the quadriceps muscles.

7. An athletic 15 year old boy presents with knee pain of 3 weeks duration. It is worst during activity and settles with rest.
On examination there is tenderness overlying the tibial tuberosity and an associated swelling at this site.
Answer is Osgood Schlatters disease
Athletic boys and girls may develop this condition in their teenage years. It is caused by multiple micro fractures at the point of
insertion of the tendon into the tibial tuberosity. Most cases settle with physiotherapy and rest.

Types of injury of knee injury

Ruptured anterior cruciate ligament: Sport injury. Mechanism: high twisting force applied to a bent knee. Typically presents
with: loud crack, pain and RAPID swelling knee (haemoarthrosis). Poor healing. Management: intense physiotherapy or surgery

Ruptured posterior cruciate ligament: Mechanism: hyperextension injuries. Tibia lies back on the femur. Paradoxical anterior
draw test

Rupture of medial collateral ligament: Mechanism: leg forced into valgus via force outside the leg. Knee unstable when put into
valgus position

Menisceal tear:Rotational sporting injuries. Delayed knee swelling. Joint locking (Patient may develop skills to "unlock" the
knee. Recurrent episodes of pain and effusions are common, often following minor trauma

Chondromalacia patellae:Teenage girls, following an injury to knee e.g. Dislocation patella. Typical history of pain on going
downstairs or at rest. Tenderness, quadriceps wasting

Dislocation of the patella:Most commonly occurs as a traumatic primary event, either through direct trauma or through severe
contraction of quadriceps with knee streched in valgus and external rotation. Genu valgum, tibial torsion and high riding patella

546
are risk factors. Skyline x-ray views of patella are required, although displaced patella may be clinically obvious. An
osteochondral fracture is present in 5%. The condition has a 20% recurrence rate

Fractured patella:2 types: i. Direct blow to patella causing undisplaced fragments. ii. Avulsion fracture

Tibial plateau fracture:Occur in the elderly (or following significant trauma in young). Mechanism: knee forced into valgus or
varus, but the knee fractures before the ligaments rupture. Varus injury affects medial plateau and if valgus injury, lateral plateau
depressed fracture occurs. Classified using the Schatzker system (see below)

Schatzker Classification system for tibial plateau fractures


Type Features
1- vertical split of lateral condyle Fracture through dense bone, usually in the young. It may be virtually undisplaced, or the
condylar fragment may be pushed inferiorly and tilted
2- a vertical split of the lateral The wedge fragement (which may be of variable size), is displaced laterally; the joint is
condyle combined with an adjacent widened. Untreated, a valgus deformity may develop
loadbearing part of the condyle
3- depression of the articular surface The split does not extend to the edge of the plateau. Depressed fragments may be firmly
with intact condylar rim embedded in subchondral bone, the joint is stable
4- fragment of the medial tibial Two injuries are seen in this category; (1) a depressed fracture of osteoporotic bone in
condyle the elderly. (2) a high energy fracture resulting in a condylar split that runs from the
intercondylar eminence to the medial cortex. Associated ligamentous injury may be
severe
5-fracture of both condyles Both condyles fractured but the column of the metaphysis remains in continuity with the
tibial shaft
6-combined condylar and subcondylar High energy fracture with marked comminution
fractures
8. A 10 year old boy presents with symptoms of right knee pain. The pain has been present on most occasions for the past
three months and the pain typically lasts for several hours at a time. On examination he walks with an antalgic gait and
has apparent right leg shortening. What is the most likely diagnosis?
A. Perthes Disease
B. Osteosarcoma of the femur
C. Osteoarthritis of the hip
D. Transient synovitis of the hip
E. Torn medial meniscus
Answer: A
There are many causes of the irritable hip in the 10-14 year age group. Many of these may cause both hip pain or knee pain.
Transient synovitis of the hip the commonest disorder but does not typically last for 3 months. An osteosarcoma would not usually
present with apparent limb shortening unless pathological fracture had occurred. A slipped upper femoral epiphysis can cause a
similar presentation although it typically presents later and with different patient characteristics.

Perthes disease: Idiopathic avascular necrosis of the femoral epiphysis of the femoral head. Impaired blood supply to femoral
head, causing bone infarction. New vessels develop and ossification occurs. The bone either heals or a subchondral fracture
occurs. Clinical features: Males 4x's greater than females. Age between 2-12 years (the younger the age of onset, the better the
prognosis). Limp. Hip pain. Bilateral in 20%. Diagnosis:Plain x-ray, Technetium bone scan or magnetic resonance imaging if
normal x-ray and symptoms persist. Catterall staging: Stage 1:Clinical and histological features only. Stage 2:Sclerosis with or
without cystic changes and preservation of the articular surface. Stage 3:Loss of structural integrity of the femoral head. Stage
4:Loss of acetabular integrity. Management: To keep the femoral head within the acetabulum: cast, braces. If less than 6 years:
observation. Older: surgical management with moderate results. Operate on severe deformities. Prognosis: Most cases will
resolve with conservative management. Early diagnosis improves outcomes.

9. Which of the following types of growth plate fractures may have similar radiological appearances?
A. Salter Harris types 1 and 5
B. Salter Harris types 4 and 5
C. Salter Harris types 3 and 5
D. Salter Harris types 1 and 2
E. Salter Harris types 1 and 3
Answer: A
Salter Harris injury types 1 and 5 (transverse fracture through growth plate Vs. Compression fracture) may mimic each other
radiologically. Type 5 injuries have the worst outcomes. Radiological signs of type 5 injuries are subtle and may include
narrowing of the growth plate.

547
Epiphyseal fractures: Fractures involving the growth plate in children are classified using the Salter - Harris system.
There are 5 main types.

Salter Harris Classification: Type 1:Transverse fracture through the growth plate. Type 2:Fracture through the growth plate to
the metaphysis (commonest type). Type 3:Fracture through the growth plate and the epiphysis with metaphysis spared. Type 4:
Fracture involving the growth plate, metaphysis and epiphysis. Type 5: Compression fracture of the growth plate (worst outcome)

Management
Non displaced type 1 injuries can generally be managed conservatively. Unstable or more extensive injuries will usually require
surgical reduction and/ or fixation, as proper alignment is crucial.
Theme: Pathological fractures

A. Osteosarcoma
B. Osteomalacia
C. Osteoporosis
D. Metastatic carcinoma
E. Osteoblastoma
F. Giant cell tumour
G. Ewing's sarcoma

For each pathological fracture please select the most likely aetiology for the scenario given. Each option may be used once, more
than once or not at all.

10. A 30 year old woman presents with pain and swelling of the left shoulder. There is a large radiolucent lesion in the head of
the humerus extending to the subchondral plate.
Answer: Giant cell tumour
Giant cell tumours on x-ray have a 'soap bubble' appearance. They present as pain or pathological fractures. They commonly
metastasize to the lungs.
11. A 72 year old woman has a lumbar vertebral crush fracture. She has hypocalcaemia and a low urinary calcium.
Answer: Osteomalacia
Hypocalcemia and low urinary calcium are biochemical features of osteomalacia. Unfortunately surgeons do need to look at
some blood results!

12. A 16 year old boy presents with severe groin pain after kicking a football. Imaging confirms a pelvic fracture. A
previous pelvic x-ray performed 2 weeks ago shows a lytic lesion with 'onion type' periosteal reaction.
Answer: Ewing's sarcoma
A Ewings sarcoma is most common in males between 10-20 years. It can occur in girls. A lytic lesion with a lamellated or onion
type periosteal reaction is a classical finding on x-rays. Most patients present with metastatic disease with a 5 year prognosis
between 5-10%.

Pathological fractures: A pathological fracture occurs in abnormal bone due to insignificant injury. Causes: Metastatic
tumours:Breast. Lung. Thyroid. Renal. Prostate. Bone disease: Osteogenesis imperfect. Osteoporosis. Metabolic bone disease.
Paget's disease. Local benign conditions: Chronic osteomyelitis. Solitary bone cyst. Primary malignant tumours:
Chondrosarcoma. Osteosarcoma. Ewing's tumour

Theme: Shoulder injuries

A. Glenohumeral dislocation
B. Acromioclavicular dislocation
C. Sternoclavicular dislocation
D. Biceps tendon tear
E. Supraspinatus tear
F. Fracture of the surgical neck of the humerus
G. Infra spinatus tear

For each scenario please select the most likely underlying diagnosis. Each option may be used once, more than once or not at all.

13. A 23 year old rugby player falls directly onto his shoulder. There is pain and swelling of the shoulder joint. The clavicle
is prominent and there appears to be a step deformity.
Answer: Acromioclavicular dislocation
Acromioclavicular joint (ACJ) dislocation normally occurs secondary to direct injury to the superior aspect of the acromion.
Loss of shoulder contour and prominent classical are key features. NB rotator cuff tears rarely occur in the second decade.

548
14. A 22 year old man falls over and presents to casualty. A shoulder x-ray is performed, the radiologist comments that a
Hill-Sachs lesion is present.
Answer: Glenohumeral dislocation
A Hill-Sachs lesion is when the cartilage surface of the humerus is in contact with the rim of the glenoid. About 50% of anterior
glenohumeral dislocations are associated with this lesion.

15. An 82 year old female presents to A&E after tripping on a step. She complains of shoulder pain. On examination there is
pain to 90o on abduction.
Answer: Supraspinatus tear
A supraspinatus tear is the most common of rotator cuff tears. It occurs as a result of degeneration and is rare in younger adults.

Proximal humerus fractures : Very common. Usually through the surgical neck. Number of classification systems though for
practical purposes describing the number of fracture fragments is probably easier. Some key points: It is rare to have fractures
through the anatomical neck. Anatomical neck fractures which are displaced by >1cm carry a risk of avascular necrosis to the
humeral head. In children the commonest injury pattern is a greenstick fracture through the surgical neck. Impacted fractures of
the surgical neck are usually managed with a collar and cuff for 3 weeks followed by physiotherapy. More significant displaced
fractures may require open reduction and fixation or use of an intramedullary device.

Types of shoulder dislocation: Glenohumeral dislocation (commonest): anterior shoulder dislocation most common.
Acromioclavicular dislocation (12%): clavicle loses all attachment with the scapula. Sternoclavicular dislocation (uncommon)

Types of glenohumeral dislocation


Anterior shoulder dislocation: External rotation and abduction. 35-40% recurrent (it is the commonest disorder). Assocociated
with greater tuberosity fracture, Bankart lesion, Hill-Sachs defect. Inferior shoulder dislocation: Luxatio erecta. Posterior
shoulder dislocation:Proportion misdiagnosed. Rim's sign, light bulb sign. Assocociated with Trough sign. Superior shoulder
dislocation:Rare and usually follow major trauma.

Treatment: Prompt reduction is the mainstay of treatment and is usually performed in the emergency department. Neurovascular
status must be checked pre and post reduction and x-rays should be performed again post reduction to ensure no fracture has
occurred. In recurrent anterior dislocation there is usually a Bankart lesion and this may be repaired surgically. Recurrent posterior
dislocations may be repaired in a similar manner to anterior lesions but using a posterior (or arthroscopic) approach.

16. Which of the following statements relating to menisceal tears is false?


A. The medial meniscus is most often affected
B. True locking of the knee joint may occur
C. Most established tears will heal with conservative management
D. In the chronic setting there is typically little to find on examination if the knee is not locked
E. An arthroscopic approach may be used to treat most lesions
Answer: C
Menisci have no nerve or blood supply and thus heal poorly. Established tears with associated symptoms are best managed by
arthroscopic menisectomy.

Theme: Developmental bone disorders

A. Rickets
B. Craniocleidodysostosis
C. Achondroplasia
D. Scurvy
E. Pagets disease
F. Multiple myeloma
G. Osteogenesis imperfecta
H. Osteomalacia
I. Osteopetrosis
J. None of the above

Please select the most likely disease process to account for the clinical scenario. Each option may be used once, more than once or
not at all

17. A 15 year-old boy presents to the out-patient clinic with tiredness, recurrent throat and chest infections, and gradual loss
of vision. Multiple x-rays show brittle bones with no differentiation between the cortex and the medulla.
Answer: Osteopetrosis
549
Osteopetrosis is an autosomal recessive condition. It is commonest in young adults. They may present with symptoms of
anaemia or thrombocytopaenia due to decreased marrow space. Radiology reveals a lack of differentiation between the cortex
and the medulla described as marble bone. These bones are very dense and brittle.

18. 18. A 12 year-old boy who is small for his age presents to the clinic with poor muscular development and hyper-mobile
fingers. His x rays show multiple fractures of the long bones and irregular patches of ossification.
Answer: Osteogenesis imperfecta
Osteogenesis imperfecta is caused by defective osteoid formation due to congenital inability to produce adequate intercellular
substances like osteoid, collagen and dentine. There is a failure of maturation of collagen in all the connective tissues.Radiology
may show translucent bones, multiple fractures, particularly of the long bones, wormian bones (irregular patches of ossification)
and a trefoil pelvis.

19. A 1 year-old is brought to the Emergency Department with a history of failure to thrive. On examination, the child is
small for age and has a large head. X-ray shows a cupped appearance of the epiphysis of the wrist.
Answer: Rickets.
Rickets is the childhood form of osteomalacia. It is due to the failure of the osteoid to ossify due to vitamin D deficiency.
Symptoms start about the age of one. The child is small for age and there is a history of failure to thrive. Bony deformities
include bowing of the femur and tibia, a large head, deformity of the chest wall with thickening of the costochondral junction
(ricketty rosary), and a transverse sulcus in the chest caused by the pull of the diaphragm (Harrison's sulcus). X- Rays show
widening and cupping of the epiphysis of the long bones, most readily apparent in the wrist.

Paediatric fracture types


Complete fracture: Both sides of cortex are breached. Toddlers fracture: Oblique tibial fracture in infants. Plastic deformity:
Stress on bone resulting in deformity without cortical disruption. Greenstick fracture: Unilateral cortical breach only. Buckle
fracture: Incomplete cortical disruption resulting in periosteal haematoma only

Growth plate fractures


In paediatric practice fractures may also involve the growth plate and these injuries are classified according to the Salter- Harris
system: Type I: Fracture through the physis only (x-ray often normal). Type II: Fracture through the physis and metaphysis.
Type III: Fracture through the physis and epiphyisis to include the joint. Type IV: Fracture involving the physis, metaphysis and
epiphysis. Type V: Crush injury involving the physis (x-ray may resemble type I, and appear normal)
As a general rule it is safer to assume that growth plate tenderness is indicative of an underlying fracture even if the x-ray appears
normal. Injuries of Types III, IV and V will usually require surgery. Type V injuries are often associated with disruption to
growth.

Non accidental injury: Delayed presentation. Delay in attaining milestones. Lack of concordance between proposed and actual
mechanism of injury. Multiple injuries. Injuries at sites not commonly exposed to trauma. Children on the at risk register

Pathological fractures: Genetic conditions, such as osteogenesis imperfecta, may cause pathological fractures.

Osteogenesis imperfect: Defective osteoid formation due to congenital inability to produce adequate intercellular substances like
osteoid, collagen and dentine. Failure of maturation of collagen in all the connective tissues. Radiology may show translucent
bones, multiple fractures, particularly of the long bones, wormian bones (irregular patches of ossification) and a trefoil pelvis.
Subtypes: Type I The collagen is normal quality but insufficient quantity. Type II- Poor collagen quantity and quality. Type III-
Collagen poorly formed. Normal quantity. Type IV- Sufficient collagen quantity but poor quality.

Osteopetrosis: Bones become harder and more dense. Autosomal recessive condition. It is commonest in young adults. Radiology
reveals a lack of differentiation between the cortex and the medulla described as marble bone.

Theme: Hip fractures

A. Conservative management
B. Percutaneous pinning
C. Fracture reduction and internal fixation
D. Hemiarthroplasty
E. Total hip replacement
F. Sliding hip screw
G. Intramedullary device

For each scenario please select the most appropriate management option. Each option may be used once, more than once or not at
all.
550
20. A 60 year old male is admitted to A&E with a fall. He lives with his wife and still works as a restaurant manager. He has
a past history of benign prostatic hypertrophy and is currently taking tamsulosin. He is otherwise fit and healthy. On
examination there is right hip tenderness on movement in all directions. A hip x-ray confirms an undisplaced
intracapsular fracture.
Answer: Percutaneous pinning
As this is undisplaced the blood supply to the femoral head may be intact and the fracture may heal. Therefore an attempt at
percutaneous fixation is reasonable.

21. An 86 year old retired pharmacist is admitted to A&E following a fall. She complains of right hip pain. She is known to
have hypertension and is currently on bendrofluazide. She lives alone and mobilises with a Zimmer frame. Her right leg
is shortened and externally rotated. A hip x-ray confirms a displaced intracapsular fracture.
Answer: Hemiarthroplasty
Hemiarthroplasty is offered to older, less mobile individuals compared to fracture reduction and fixation in younger patients.

22. A 74 year old male is admitted to A&E with a fall. He is known to have rheumatoid arthritis and is on methotrexate and
paracetamol. He lives alone in a bungalow and enjoys playing golf. He is independent with his ADLs. He complains of
left groin pain, therefore has a hip x-ray which confirms a displaced intracapsular fracture.
Answer: Total hip replacement
This patient has pre-existing joint disease, good level of activity and a relatively high life expectancy, therefore THR is
preferable to hemiarthroplasty.

The hip is a common site of fracture especially in osteoporotic, elderly females. The blood supply to the femoral head runs up the
neck and thus avascular necrosis is a risk in displaced fractures.

Classification
The Garden system is one classification system in common use. Type I: Stable fracture with impaction in valgus. Type II:
Complete fracture but undisplaced. Type III: Displaced fracture, usually rotated and angulated, but still has bony contact. Type
IV: Complete bony disruption. Blood supply disruption is most common following Types III and IV.

Management of hip fractures in older adults


SIGN Guidelines
Fracture type Patient co-morbidities Management
Undisplaced intracapsular fracture Nil Internal fixation (especially if young)
Undisplaced intracapsular fracture Major illness or advanced organ Hemiarthroplasty
specific disease
Displaced intracapsular fracture Nil If age <70 then internal fixation (if possible), hip
arthroplasty if not
Displaced intracapsular fracture Nil Age >70 total hip arthroplasty
Displaced intracapsular fracture Major/ immobile Hemiarthroplasty
Extracapsular fracture (non special Only major co-morbidities affect Dynamic hip screw
type) management
Extracapsular fracture Only major co-morbidities affect Usually intramedullary device
(reverse oblique, transverse or sub management
trochanteric)

23. Of the list below, which is not a cause of avascular necrosis?


A. Steroids
B. Sickle cell disease
C. Radiotherapy
D. Myeloma
E. Caisson disease
Answer: D
Causes of avascular necrosis: PLASTIC RAGS: P ancreatitis. L upus. A lcohol. S teroids. T rauma. I diopathic, infection. C
aisson disease, collagen vascular disease. R adiation, rheumatoid arthritis. A myloid. G aucher disease. S ickle cell disease

Steroid containing therapy for myeloma may induce avascular necrosis, however the disease itself does not cause it. Caisson
disease as may occur in deep sea divers is a recognised cause.

551
Avascular necrosis: Cellular death of bone components due to interruption of the blood supply, causing bone destruction. Main
joints affected are hip, scaphoid, lunate and the talus. It is not the same as non union. The fracture has usually united. Radiological
evidence is slow to appear.Vascular ingrowth into the affected bone may occur. However, many joints will develop secondary
osteoarthritis. Presentation: Usually pain. Often despite apparent fracture union. Investigation: MRI scanning will show changes
earlier than plain films. Treatment: In fractures at high risk sites anticipation is key. Early prompt and accurate reduction is
essential. Non weight bearing may help to facilitate vascular regeneration. Joint replacement may be necessary, or even the
preferred option (e.g. Hip in the elderly).

24. Which of the following is the first radiological change likely to be apparent in a plain radiograph of a 12 year old
presenting with suspected Perthes disease
A. Multiple bone cysts
B. Sclerosis of the femoral head
C. Loss of bone density
D. Joint space narrowing
E. Collapse of the femoral head
Answer: B
In Catterall stage I disease there may be no radiological abnormality at all. In Stage II disease there may be sclerosis of the
femoral head.
Indication for treatment (aide memoire): Half a dozen, half a head
Those aged greater than 6 years with >50% involvement of the femoral head should almost always be treated.

Theme: Upper limb injuries

A. Pulled elbow
B. Fracture of the coronoid process
C. Scaphoid fracture
D. Moteggia fracture
E. Bennets fracture
F. Fracture of the shaft of the radius and ulnar
G. Galeazzi fracture
H. Fracture of the olecranon
I. Fracture of the radial head

Please select the most likely injury for the scenario given. Each option may be used once, more than once or not at all.

25. A 32 year old man presents with a painful swelling over the volar aspect of his hand after receiving a hard blow to his
palm. On examination, he experiences pain on moving the wrist and on longitudinal compression of the thumb.
The correct answer is Scaphoid fracture
Scaphoid fractures usually occur as a result of direct hard blow to the palm or following a fall on the out-stretched hand. The
main physical signs are swelling and tenderness in the anatomical snuff box, and pain on wrist movements and on longitudinal
compression of the thumb

26. A 26 year old man presents to the emergency department with a swelling over his left elbow after a fall on an
outstretched hand. On examination, he has tenderness over the proximal part of his forearm, and has severely restricted
supination and pronation movements.
The correct answer is Fracture of the radial head
Fracture of the radial head is common in young adults. It is usually caused by a fall on the outstretched hand. On examination,
there is marked local tenderness over the head of the radius, impaired movements at the elbow, and a sharp pain at the lateral
side of the elbow at the extremes of rotation (pronation and supination).

27. A 56 year old lady presents with a painful swelling over the lower end of the forearm following a fall. Imaging reveals a
distal radial fracture with disruption of the distal radio-ulnar joint.
Answer: Galeazzi fracture
Galeazzi fractures occur after a fall on the hand with a rotational force superimposed on it. On examination, there is bruising,
swelling and tenderness over the lower end of the forearm. X- Rays reveal a displaced fracture of the radius and a prominent
ulnar head due to dislocation of the inferior radio-ulnar joint.

Colles' fracture: Fall onto extended outstretched hands. Described as a dinner fork type deformity. Classical Colles' fractures
have the following 3 features: 1. Transverse fracture of the radius 2. 1 inch proximal to the radio-carpal joint. 3. Dorsal
displacement and angulation

552
Smith's fracture (reverse Colles' fracture): Volar angulation of distal radius fragment (Garden spade deformity). Caused by
falling backwards onto the palm of an outstretched hand or falling with wrists flexed

Bennett's fracture: Intra-articular fracture of the first carpometacarpal joint. Impact on flexed metacarpal, caused by fist fights.
X-ray: triangular fragment at ulnar base of metacarpal

Monteggia's fracture: Dislocation of the proximal radioulnar joint in association with an ulna fracture. Fall on outstretched hand
with forced pronation. Needs prompt diagnosis to avoid disability

Galeazzi fracture: Radial shaft fracture with associated dislocation of the distal radioulnar joint. Occur after a fall on the hand
with a rotational force superimposed on it. On examination, there is bruising, swelling and tenderness over the lower end of the
forearm. X Rays reveal the displaced fracture of the radius and a prominent ulnar head due to dislocation of the inferior radio-
ulnar joint.

Barton's fracture: Distal radius fracture (Colles'/Smith's) with associated radiocarpal dislocation. Fall onto extended and
pronated wrist

Scaphoid fractures: Scaphoid fractures are the commonest carpal fractures. Surface of scaphoid is covered by articular cartilage
with small area available for blood vessels (fracture risks blood supply). Forms floor of anatomical snuffbox.. Risk of fracture
associated with fall onto outstretched hand (tubercle, waist, or proximal 1/3). The main physical signs are swelling and tenderness
in the anatomical snuff box, and pain on wrist movements and on longitudinal compression of the thumb. Ulnar deviation AP
needed for visualization of scaphoid. Immobilization of scaphoid fractures difficult

Radial head fracture: Fracture of the radial head is common in young adults. It is usually caused by a fall on the outstretched
hand. On examination, there is marked local tenderness over the head of the radius, impaired movements at the elbow, and a sharp
pain at the lateral side of the elbow at the extremes of rotation (pronation and supination).

Theme: Hand injuries

A. Admission and surgical debridement


B. Application of futura splint and fracture clinic review
C. Application of tubigrip bandage and fracture clinic review
D. Admission for open reduction and fixation
E. Discharge with reassurance
F. Commence oral prednisolone
G. Commence oral diclofenac

Which of the following options is the best management plan? Each option may be used once, more than once or not at all.

28. A 42 year old skier falls and impacts his hand on his ski pole. On examination he is tender in the anatomical snuffbox
and on bimanual palpation. Xrays with scaphoid views show no evidence of fracture.
Answer: Application of futura splint and fracture clinic review
A fracture may still be present and should be immobilised until repeat imaging can be performed.

29. A 43 year old man falls over landing on his left hand. Although there was anatomical snuffbox tenderness no x-rays
either at the time or subsequently have shown evidence of scaphoid fracture. He has been immobilised in a futura splint
for two weeks and is now asymptomatic.
Answer: Discharge with reassurance
This patient is at extremely low risk of having sustained a scaphoid injury and may be discharged.

30. A builder falls from scaffolding and lands on his left hand he suffers a severe laceration to his palm. An x-ray shows
evidence of scaphoid fracture that is minimally displaced.
Answer: Admission and surgical debridement

This is technically an open fracture and should be debrided prior to attempted fixation (which should occur soon after). Scaphoid
fractures: 80% of all carpal fractures. 80% occur in men. 80% occur at the waist of the scaphoid

Scaphoid fractures: Scaphoid fractures are the commonest carpal fractures. Surface of scaphoid is covered by articular cartilage
with small area available for blood vessels (fracture risks blood supply). Forms floor of anatomical snuffbox. Risk of fracture
associated with fall onto outstretched hand (tubercle, waist, or proximal third). Ulnar deviation AP needed for visualization of
scaphoid. Immobilization of scaphoid fractures difficult
553
Management: Non-displaced fractures: Casts or splints. Percutaneous scaphoid fixation. Displaced fracture: Surgical fixation,
usually with a screw

Complications: Non union of scaphoid. Avascular necrosis of the scaphoid. Scapholunate disruption and wrist collapse.
Degenerative changes of the adjacent joint

Theme: Paediatric orthopaedics

A. Musculoskeletal pain
B. Congenital dysplasia of the hip
C. Slipped upper femoral epiphysis
D. Transient synovitis
E. Septic arthritis
F. Perthes disease
G. Tibial fracture

Please select the most likely diagnosis for the scenario given. Each option may be used once, more than once or not at all.

31. A 4 year boy presents with an abnormal gait. He has a history of recent viral illness. His WCC is 11 and ESR is 30.
Answer: Transient synovitis
Viral illnesses can be associated with transient synovitis. The WCC should ideally be > 12 and the ESR > 40 to suggest
septic arthritis.

32. A 6 year old boy presents with an groin pain. He is known to be disruptive in class. He reports that he is bullied for
being short. On examination he has an antalgic gait and pain on internal rotation of the right hip.
Answer: Perthes disease
This child is short, has hyperactivity (disruptive behaviour) and is within the age range for Perthes disease. Hyperactivity
and short stature are associated with Perthes disease.

33. An obese 12 year old boy is referred with pain in the left knee and hip. On examination he has an antaglic gait and
limitation of internal rotation. His knee has normal range of passive and active movement.
Answer: Slipped upper femoral epiphysis
Slipped upper femoral epiphysis is commonest in obese adolescent males. The x-ray will show displacement of the
femoral epiphysis inferolaterally. Treatment is usually with rest and non weight bearing crutches.
Beware of attributing gait disorders to benign processes in young children without careful clinical and radiological assessment.

Diagnosis Mode of presentation Treatment Radiology


Developmental Usually diagnosed in infancy by Splints and harnesses or traction. In Initially no obvious change
dysplasia of the screening tests. May be bilateral, later years osteotomy and hip on plain films and USS gives
hip when disease is unilateral there may realignment procedures may be best resolution until 3
be leg length inequality. As disease needed. In arthritis a joint months of age. On plain
progresses child may limp and then replacement may be needed. films Shentons line should
early onset arthritis. More common However, this is best deferred if form a smooth arc
in extended breech babies. possible as it will almost certainly
require revision
Perthes Disease Hip pain (may be referred to the Remove pressure from joint to allow X-rays will show flattened
knee) usually occurring between 5 normal development. Physiotherapy. femoral head. Eventually in
and 12 years of age. Bilateral disease Usually self-limiting if diagnosed and untreated cases the femoral
in 20%. treated promptly. head will fragment.
Slipped upper Typically seen in obese male Bed rest and non-weight bearing. Aim X-rays will show the femoral
femoral adolescents. Pain is often referred to to avoid avascular necrosis. If severe head displaced and falling
epiphysis the knee. Limitation to internal slippage or risk of it occurring then inferolaterally (like a melting
rotation is usually seen. Knee pain is percutaneous pinning of the hip may ice cream cone) The
usually present 2 months prior to hip be required. Southwick angle gives
slipping. Bilateral in 20%. indication of disease severity

Theme: Eponymous fractures

A. Smith's

554
B. Bennett's
C. Monteggia's
D. Colle's
E. Galeazzi
F. Pott's
G. Barton's

Link the most appropriate eponymously named fracture to the scenario described. Each scenario may be used once, more than
once or not at all.

A 28 year old man falls on the back of his hand. On x-ray the he has a fractured distal radius demonstrating volar
displacement of the fracture.
Answer: Smith's
This is a Smith fracture (reverse Colle's fracture); unlike a Colle's this is a high velocity injury and may require surgical
correction. Note that Colles fractures are usually dorsally displaced

34. A 38 year old window cleaner falls from his ladder. He lands on his left arm and notices an obvious injury. An x-ray and
clinical examination demonstrate that has a fracture of the proximal ulna and associated radial dislocation
Answer: Monteggia's
This constellation of injuries is referred to as a Monteggia's fracture

35. A 32 year old man falls from scaffolding and sustains an injury to his forearm. Clinical examination and x-ray shows
that he has sustained a radial fracture with dislocation of the inferior radio-ulna joint
Answer: Galeazzi
Isolated fracture of the radius alone can occur but is rare. Always check for associated injury

36. A 54-year-old man presents to the Emergency Department with a 2 day history of a swollen, painful left knee. You
aspirate the joint to avoid admission to the orthopaedic wards. Aspirated joint fluid shows calcium pyrophosphate
crystals. Which of the following blood tests is most useful in revealing an underlying cause?
A. Transferrin saturation
B. ACTH
C. ANA
D. Serum ferritin
E. LDH

Answer: A

This is a typical presentation of pseudogout. An elevated transferrin saturation may indicate haemochromatosis, a recognised
cause of pseudogout. A high ferritin level is also seen in haemochromatosis but can be raised in a variety of infective and
inflammatory processes, including pseudogout, as part of an acute phase response.

Pseudogout: a form of microcrystal synovitis caused by the deposition of calcium pyrophosphate dihydrate in the synovium. Risk
factors: Hyperparathyroidism. Hypothyroidism. Haemochromatosis. Acromegaly.Low magnesium, Low phosphate.Wilson's
disease. Features: knee, wrist and shoulders most commonly affected. joint aspiration: weakly-positively birefringent rhomboid
shaped crystals. x-ray: chondrocalcinosis. Management: aspiration of joint fluid, to exclude septic arthritis. NSAIDs or intra-
articular, intra-muscular or oral steroids as for gout

37. A 65-year-old Asian female presents with an extracapsular neck of femur fracture. Investigations show: Calcium: 2.07
mmol/l (2.20-2.60 mmol/l); Phosphate:0.66 mmol/l (0.8-1.40 mmol/l); ALP:256 IU/l (44-147 IU/l). What is the most likely
diagnosis?
A. Bone tuberculosis
B. Hypoparathyroidism
C. Myeloma
D. Osteomalacia
E. Paget's disease
Answer: D
Osteomalacia: low: calcium, phosphate. Raised: alkaline phosphatase
The low calcium and phosphate combined with the raised alkaline phosphatase point towards osteomalacia.
Basics: normal bony tissue but decreased mineral content. Rickets if when growing. Osteomalacia if after epiphysis fusion
Types: vitamin D deficiency e.g. malabsorption, lack of sunlight, diet. Renal failure. Drug induced e.g. anticonvulsants. Vitamin
D resistant (inherited). Liver disease, e.g. cirrhosis. Features: rickets: knock-knee, bow leg, features of hypocalcaemia.
osteomalacia: bone pain, fractures, muscle tenderness, proximal myopathy. Investigation: low calcium, phosphate, 25(OH)

555
vitamin D. raised alkaline phosphatase . X-ray: children - cupped, ragged metaphyseal surfaces; adults - translucent bands
(Looser's zones or pseudofractures). Treatment: calcium with vitamin D tablets.

38. A 78-year-old woman is discharged following a fractured neck of femur. On review she is making good progress but
consideration is given to secondary prevention of further fractures. Unfortunately the orthogeriatricians are all on
annual leave and the consultant has asked you to arrange suitable management. Which is the best option?
A. Alendronate
B. Alendronate, calcium and vitamin D supplementation
C. Strontium
D. Arrange a DEXA scan
E. Hormone replacement therapy
Answer: B
A bisphosphonate, calcium and vitamin D supplementation should be given to all patients aged over 75 years after having a
fracture. A DEXA scan is only needed of the patient is aged below 75 years. Hormone replacement therpay has been shown to
reduce vertebral and non vertebral fractures, however the risks of cardiovascular disease and breast malignancy make this a less
favourable option.

Osteoporosis
secondary prevention: NICE guidelines were updated in 2008 on the secondary prevention of osteoporotic fractures in
postmenopausal women. Key points include. Treatment is indicated following osteoporotic fragility fractures in postmenopausal
women who are confirmed to have osteoporosis (a T-score of - 2.5 SD or below). In women aged 75 years or older, a DEXA scan
may not be required 'if the responsible clinician considers it to be clinically inappropriate or unfeasible'. Vitamin D and calcium
supplementation should be offered to all women unless the clinician is confident they have adequate calcium intake and are
vitamin D replete. Alendronate is first-line. Around 25% of patients cannot tolerate alendronate, usually due to upper
gastrointestinal problems. These patients should be offered risedronate or etidronate (see treatment criteria below). Strontium
ranelate and raloxifene are recommended if patients cannot tolerate bisphosphonates (see treatment criteria below)

Supplementary notes on treatment


Bisphosphonates: Alendronate, risedronate and etidronate are all licensed for the prevention and treatment of post-menopausal
and glucocorticoid-induced osteoporosis .All three have been shown to reduce the risk of both vertebral and non-vertebral
fractures although alendronate, risedronate may be superior to etidronate in preventing hip fractures. Ibandronate is a once-
monthly oral bisphosphonate

Vitamin D and calcium: Poor evidence base to suggest reduced fracture rates in the general population at risk of osteoporotic
fractures - may reduce rates in frail, housebound patients

Raloxifene: selective oestrogen receptor modulator (SERM): Has been shown to prevent bone loss and to reduce the risk of
vertebral fractures, but has not yet been shown to reduce the risk of non-vertebral fractures. Has been shown to increase bone
density in the spine and proximal femur. May worsen menopausal symptoms. Increased risk of thromboembolic events. May
decrease risk of breast cancer

Strontium ranelate: 'Dual action bone agent' - increases deposition of new bone by osteoblasts and reduces the resorption of
bone by osteoclasts. Strong evidence base, may be second-line treatment in near future. Increased risk of thromboembolic events

39. Which of the following statements relating to avascular necrosis is false?


A. When associated with fracture may occur despite the radiological evidence of fracture union.
B. Pain and stiffness will typically precede radiological evidence of the condition.
C. Drilling of affected bony fragments may be used to facilitate angiogenesis where arthroplasty is not warranted.
D. The earliest detectable radiological evidence is a radiolucency of the affected area coupled with subchondral collapse.
E. It is less likely when prompt anatomical alignment of fracture fragments is achieved.
Answer: D
Avascular necrosis- radiological changes occur late.
Radiolucency and subchondral collapse are late changes. The earliest evidence on plain films is the affected area appearing as
being more radio-opaque due to hyperaemia and resorption of the neighboring area. It may be diagnosed earlier using bone scans
and MRI.

Theme: Diseases affecting the spine

A. Spondylolysis
B. Spina bifida occulta
C. Spondylolisthesis
D. Meningomyelocele
E. Meningocele
F. Scoliosis - non structural
556
G. Scoliosis
H. Ankylosing spondylitis
I. Scheuermann's disease

Please select the most likely underlying diagnosis for the condition described. Each condition may be used once, more than once
or not at all.

40. A 19 year old female is involved in an athletics event. She has just completed the high jump when she suddenly develops
severe back pain and weakness affecting both her legs. on examination she has a prominent sacrum and her lower back
is painful.
Answer: Spondylolisthesis
Young athletic females are the group most frequently affected by spondylolythesis who have a background of spondylolysis.
Whilst the latter condition is a risk factor for spondylolythesis the former condition is most likely in a young athletic female who
presents with sudden pain.

41. A 15 year old boy is brought to the clinic by his mother who is concerned that he has a mark overlying his lower spine.
On examination the boy has a patch of hair overlying his lower lumbar spine and a birth mark at the same location.
Lower limb neurological examination is normal.
Answer: Spina bifida occulta
Spina bifida occulta is a common condition and may affect up to 10% of the population. The more severe types of spina bifida
have more characteristic skin changes. Occasionally the unwary surgeon is persuaded to operate on these "cutaneous" changes
and we would advocate performing an MRI scan prior to any such surgical procedure in this region.

42. A 19 year old female presents to the clinic with progressive pain in her neck and back. The condition has been
progressively worsening over the past 6 months. She has not presented previously because she was an inpatient with a
disease flare of ulcerative colitis. On examination she has a stiff back with limited spinal extension on bending forwards.
Answer: Ankylosing spondylitis
Ankylosing spondylitis is associated with HLA B27, there is a strong association with ulcerative colitis in such individuals. The
clinical findings are usually of a kyphosis affecting the cervical and thoracic spine. Considerable symptomatic benefit may be
obtained using non steroidal anti inflammatory drugs. These should be used carefully in patients with inflammatory bowel
disease who may be taking steroids.

Diseases affecting the vertebral column


Ankylosing spondylitis: Chronic inflammatory disorder affecting the axial skeleton. Sacro-ilitis is a usually visible in plain films.
Up to 20% of those who are HLA B27 positive will develop the condition. Affected articulations develop bony or fibrous changes.
Typical spinal features include loss of the lumbar lordosis and progressive kyphosis of the cervico-thoracic spine

Scheuermann's disease: Epiphysitis of the vertebral joints is the main pathological process. Predominantly affects adolescents.
Symptoms include back pain and stiffness. X-ray changes include epiphyseal plate disturbance and anterior wedging. Clinical
features include progressive kyphosis (at least 3 vertebrae must be involved). Minor cases may be managed with physiotherapy
and analgesia, more severe cases may require bracing or surgical stabilisation

Scoliosis: Consists of curvature of the spine in the coronal plane. Divisible into structural and non structural, the latter being
commonest in adolescent females who develop minor postural changes only. Postural scoliosis will typically disappear on
manoeuvres such as bending forwards. Structural scoliosis affects > 1 vertebral body and is divisible into idiopathic, congential
and neuromuscular in origin. It is not correctable by alterations in posture. Within structural scoliosis, idiopathic is the most
common type. Severe, or progressive structural disease is often managed surgically with bilateral rod stabilisation of the spine.

Spina bifida: Non fusion of the vertebral arches during embryonic development. Three categories; myelomeningocele, spina
bifida occulta and meningocele. Myelomeningocele is the most severe type with associated neurological defects that may persist
in spite of anatomical closure of the defect. Up to 10% of the population may have spina bifida occulta, in this condition the skin
and tissues (but not not bones) may develop over the distal cord. The site may be identifiable by a birth mark or hair patch. The
incidence of the condition is reduced by use of folic acid supplements during pregnancy

Spondylolysis: Congenital or acquired deficiency of the pars interarticularis of the neural arch of a particular vertebral body,
usually affects L4/ L5. May be asymptomatic and affects up to 5% of the population. Spondylolysis is the commonest cause of
spondylolisthesis in children. Asymptomatic cases do not require treatment

Spondylolisthesis: This occurs when one vertebra is displaced relative to its immediate inferior vertebral body. May occur as a
result of stress fracture or spondylolysis. Traumatic cases may show the classic "Scotty Dog" appearance on plain films.
Treatment depends upon the extent of deformity and associated neurological symptoms, minor cases may be actively monitored.
Individuals with radicular symptoms or signs will usually require spinal decompression and stabilisation

557
Theme: Management of fractures

A. Discharge home with arm sling and fracture clinic appointment


B. Discharge home with futura splint and fracture clinic appointment
C. Admit for open reduction and fixation
D. Fasciotomy
E. Active observation for progression of neurovascular compromise
F. Reduction of fracture in casualty and application of plaster backslab, followed by discharge home.

Please select the most appropriate immediate management for the fracture scenarios given. Each option may be used once, more
than once or not at all.

43. A 22 year old rugby player falls onto an outstretched hand and sustains a fracture of the distal radius. The x-ray shows a
dorsally angulated comminuted fracture.
The correct answer is Admit for open reduction and fixation
Unlike an osteoporotic fracture in an elderly lady this is a high velocity injury and will require surgical fixation.

44. A 10 year old boy undergoes a delayed open reduction and fixation of a significantly displaced supracondylar fracture.
On the ward he complains of significant forearm pain and paraesthesia of the hand. Radial pulse is normal.
Answer is Fasciotomy
The delay is the significant factor here. These injuries often have neurovascular compromise and inactivity now places him at
risk of developing complications. In compartment syndrome the loss of arterial pulsation occurs late.

45. A 28 year old man falls onto an outstretched hand. On examination there is tenderness of the anatomical snuffbox.
However, forearm and hand x-rays are normal.
Answer: Discharge home with futura splint and fracture clinic appointment
This could well be a scaphoid fracture and should be temporarily immobilised pending further review. A futura splint will
immobilise better than an arm sling for this problem.

Fracture management: Bony injury resulting in a fracture may arise from trauma (excessive forces applied to bone), stress
related (repetitive low velocity injury) or pathological (abnormal bone which fractures during normal use of following minimal
trauma). Diagnosis involves not just evaluating the fracture ; such as site and type of injury but also other associated injuries and
distal neurovascular deficits. This may entail not just clinical examination but radiographs of proximal and distal joints. When
assessing x-rays it is important to assess for changes in length of the bone, the angulation of the distal bone, rotational effects,
presence of material such as glass.

Fracture types
Oblique fracture: Fracture lies obliquely to long axis of bone. Comminuted fracture: >2 fragments. Segmental fracture: More
than one fracture along a bone. Transverse fracture: Perpendicular to long axis of bone. Spiral fracture: Severe oblique fracture
with rotation along long axis of bone

Open Vs Closed
It is also important to distinguish open from closed injuries. The most common classification system for open fractures is
theGustilo and Anderson classification system: Grade 1: Low energy wound <1cm; Grade 2: Greater than 1cm wound with
moderate soft tissue damage; Grade 3 High energy wound > 1cm with extensive soft tissue damage; Grade 3: A (sub group of
3): Adequate soft tissue coverage; Grade 3: B (sub group of 3): Inadequate soft tissue coverage; Grade 3: C (sub group of 3):
Associated arterial injury

Key points in management of fractures: Immobilise the fracture including the proximal and distal joints. Carefully monitor and
document neurovascular status, particularly following reduction and immobilization. Manage infection including tetanus
prophylaxis. IV broad spectrum antibiotics for open injuries. As a general principle all open fractures should be thoroughly
debrided ( and internal fixation devices avoided or used with extreme caution). Open fractures constitute an emergency and should
be debrided and lavaged within 6 hours of injury

46. A 4 year old boy falls and sustains a fracture to the growth plate of his right wrist. Which of the following systems is used
to classify the injury?
A. Salter - Harris system
B. Weber system
C. Gustilo - Anderson system
558
D. Garden system
E. None of the above
Answer: A
The Salter - Harris system is most commonly used. The radiological signs in Type 1 and 5 injuries may be identical. Which is
unfortunate as type 5 injuries do not do well (and may be missed!)

Theme: Bone disease

A. Osteogenesis imperfecta
B. Osteoporosis
C. Rickets
D. Pagets disease
E. Chondrosarcoma
F. Metastatic breast cancer

Please select the most likely diagnosis for the scenario given. Each option may be used once, more than once or not at all.

47. A 66 year old lady presents with pain in her right hip. It has been increasing over the previous three weeks and waking
her from sleep. On examination she is tender on internal rotation. Blood tests reveal a mildly elevated serum calcium and
alkaline phosphatase levels.
Answer: Metastatic breast cancer
Increasing pain at rest, together with increased serum calcium and alkaline phosphatase are most likely to represent metastatic
tumour to bone. Chondrosarcomas do occur in the pelvis but are not associated with increased serum calcium and typically have
a longer history.

48. A 73 year old man presents with pain in the right leg. It is most uncomfortable on walking. On examination he has a
deformity of his right femur, which on x-ray is thickened and sclerotic. His serum alkaline phosphatase is elevated, but
calcium is within normal limits.
Answer is Pagets disease
This is a typical scenario for Pagets disease.

49. A 73 year old lady presents with pain in her left hip. She was walking around the house when she tripped over a rug and
fell over. Apart from temporal arteritis which is well controlled with prednisolone she is otherwise well. On examination
he leg is shorted and externally rotated.Her serum alkaline phosphatase and calcium are normal.
Answer: Osteoporosis
The combination of age, female gender and steroids coupled with hip pain on minor trauma are strongly suggestive of
osteoporosis.

Pagets: Focal bone resorption followed by excessive and chaotic bone deposition. Affects (in order): spine, skull, pelvis and
femur. Serum alkaline phosphatase raised (other parameters normal). Abnormal thickened, sclerotic bone on x-rays. Risk of
cardiac failure with >15% bony involvement. Small risk of sarcomatous change. Treatment: Bisphosphonates

Osteoporosis: Excessive bone resorption resulting in demineralised bone. Commoner in old age. Increased risk of pathological
fracture, otherwise asymptomatic. Alkaline phosphatase normal, calcium normal. Treatment: Bisphosphonates, calcium and
vitamin D.

Secondary bone tumours: Bone destruction and tumour infiltration. Mirel scoring used to predict risk of fracture. Appearances
depend on primary (e.g.sclerotic - prostate, lytic - breast). Elevated serum calcium and alkaline phosphatase may be seen.
Treatment: Radiotherapy, prophylactic fixation and analgesia

Theme: Shoulder pain


A. Impingement syndrome
B. Rotator cuff tear
C. Adhesive capsulitis
D. Calcific tendonitis
E. Biceps tendon rupture
F. Parsonage - Turner syndrome
G. Labral tear

Please select the most likely cause for shoulder pain from the list. Each option may be used once, more than once or not at all.

559
50. A 63 year old lady undergoes an axillary clearance for breast cancer. She makes steady progress. However, 8 weeks
post operatively she still suffers from severe shoulder pain. On examination she has reduced active movements in all
planes and loss of passive external rotation.
The correct answer is Adhesive capsulitis
Frozen shoulder passes through an initial painful stage followed by a period of joint stiffness. With physiotherapy the problem
will usually resolve although it may take up to 2 years to do so.

51. A 78 year old man complains of a long history of shoulder pain and more recently weakness. On examination active
attempts at abduction are impaired. Passive movements are normal.
Answer: Rotator cuff tear
Rotator cuff tears are common in elderly people and may occur following minor trauma or as a result of long standing
impingement. Tears greater than 2cm should generally be repaired surgically.

52. A 28 year old man complains of pain and weakness in the shoulder. He has recently been unwell with glandular fever
from which he is fully recovered. On examination there is some evidence of muscle wasting and a degree of winging of
the scapula. Power during active movements is impaired.
Answer is Parsonage - Turner syndrome
This is a peripheral neuropathy that may complicate viral illnesses and usually resolves spontaneously. Deep seated pain in the
proximal forearm especially during the night and at rest may be due to tumour, especially metastatic lesions.

Theme: Knee injuries

A. Anterior cruciate ligament rupture


B. Posterior cruciate ligament rupture
C. Medial collateral ligament tear
D. Lateral collateral ligament tear
E. Torn meniscus
F. Chondromalacia patellae
G. Dislocated patella
H. Fractured patella
I. Tibial plateau fracture

What is the most likely injury for scenario given? Each option may be used once, more than once or not at all.

53. A 38 year old man is playing football when he slips over during a tackle. His knee is painful immediately following the
fall. Several hours later he notices that the knee has become swollen. Following a course of non steroidal anti
inflammatory drugs and rest the situation improves. However, complains of recurrent pain. On assessment in clinic you
notice that it is impossible to fully extend the knee, although the patient is able to do so when asked.
Answer: Torn meniscus
Twisting sporting injuries followed by delayed onset of knee swelling and locking are strongly suggestive of a menisceal tear.
Arthroscopic menisectomy is the usual treatment.

55. A 34 year old woman is a passenger in a car during an accident. Her knee hits the dashboard. On examination the
tibia looks posterior compared to the non injured knee.
Answer: Posterior cruciate ligament rupture
In ruptured posterior cruciate ligament the tibia lies back on the femur and can be drawn forward during a paradoxical draw test.

54. 28 year old professional footballer is admitted to the emergency department. During a tackle he is twisted with his knee
flexed. He hears a loud crack and his knee rapidly becomes swollen.
Answer: Anterior cruciate ligament rupture
This is common in footballers as the football boot studs stick to the ground and high twisting force is applied to a flexed knee.
Rapid joint swelling also supports the diagnosis.

55. A 10 year old boy is referred to the orthopaedic clinic with symptoms of right knee pain. He has suffered pain for the
past 3 months and the pain typically lasts for several hours. On examination he walks with an antalgic gait and has
apparent right leg shortening. The right knee is normal but the right hip reveals pain on internal and external rotation.
Imaging shows flattening of the femoral head. Which of the following is the most likely underlying diagnosis?
A. Osteogenesis imperfecta
B. Child abuse
C. Osteosarcoma
D. Osteopetrosis

560
E. Perthes disease
Answer: E
This is a typical description of Perthes disease. Management involves keeping the femoral head in the acetabulum by braces, casts
or surgery.

56. Which statement relating to talipes equinovarus is untrue?


A. It has an annual incidence of around 1 in 1000 in the UK.
B. The muscles involved in the disorder are intrinsically abnormal.
C. The cuboid is classically displaced medially.
D. All cases should be treated with an Ilizarov frame initially unless there is minor deformity.
E. The talocalcaneal angle is typically less than 20 degrees in club foot.
Answer: D
In most cases of Club Foot conservative measures should be tried first. The Ponsetti method is a popular approach. Severe cases
may benefit from Ilizarov frame re-aligment.

Congenital talipes equinovarus


Features: Equinus of the hindfoot. Adduction and varus of the midfoot. High arch. Most cases in developing countries. Incidence
in UK is 1 per 1000 live births. It is more common in males and is bilateral in 50% cases. There is a strong familial link(1). It may
also be associated with other developmental disorders such as Down's syndrome.

Key anatomical deformities (2): Adducted and inverted calcaneus. Wedge shaped distal calcaneal articular surface. Severe
Tibio-talar plantar flexion. Medial Talar neck inclination. Displacement of the navicular bone (medially). Wedge shaped head of
talus. Displacement of the cuboid (medially)

Management
Conservative first, the Ponseti method is best described and gives comparable results to surgery. It consists of serial casting to
mold the foot into correct shape. Following casting around 90% will require a Achilles tenotomy. This is then followed by a phase
of walking braces to maintain the correction. Surgical correction is reserved for those cases that fail to respond to conservative
measures. The procedures involve multiple tenotomies and lengthening procedures. In patients who fail to respond surgically an
Ilizarov frame reconstruction may be attempted and gives good results.

57. Which of the following is least likely to impair bone fracture healing?
A. Radiotherapy
B. Osteoporosis
C. Administration of non steroidal anti inflammatory drugs
D. Preservation of periosteum
E. Presence of osteomyelitic sequestra
Answer: D
Periosteal preservation helps fractures to heal.

Fracture healing
Bone fracture: Bleeding vessels in the bone and periosteum. Clot and haematoma formation. The clot organises over a week
(improved structure and collagen). The periosteum contains osteoblasts which produce new bone. Mesenchymal cells produce
cartilage (fibrocartilage and hyaline cartilage) in the soft tissue around the fracture. Connective tissue + hyaline cartilage = callus.
As the new bone approaches the new cartilage, endochondral ossification occurs to bridge the gap. Trabecular bone forms.
Trabecular bone is resorbed by osteoclasts and replaced with compact bone.

Factors Affecting Fracture Healing: Age; Malnutrition; Bone disorders: osteoporosis; Systemic disorders: diabetes, Marfan's
syndrome and Ehlers-Danlos syndrome cause abnormal musculoskeletal healing.; Drugs: steroids, non steroidal anti inflammatory
agents; Type of bone: Cancellous (spongy) bone fractures are usually more stable, involve greater surface areas, and have a better
blood supply than cortical (compact) bone fractures; Degree of Trauma: The more extensive the injury to bone and surrounding
soft tissue, the poorer the outcome; Vascular Injury: Especially the femoral head, talus, and scaphoid bones.; Degree of
Immobilization; Intra-articular Fractures: These fractures communicate with synovial fluid, which contains collagenases that
retard bone healing; Separation of Bone Ends: Normal apposition of fracture fragments is needed for union to occur. Inadequate
reduction, excessive traction, or interposition of soft tissue will prevent healing; Infection

Theme: Disorders of the hip

A. Perthes disease
B. Developmental dysplasia of the hip
C. Osteoarthritis
D. Slipped upper femoral epiphysis
561
E. Septic arthritis
F. Rheumatoid arthritis
G. Intra capsular fracture of the femoral neck
H. Extra capsular fracture of the femoral neck

Please select the most likely diagnosis for the scenario given. Each option may be used once, more than once or not at all.

58. An obese 14 year old boy presents with difficulty running and mild knee and hip pain. There is no antecedent history of
trauma. On examination internal rotation is restricted but the knee is normal with full range of passive movement
possible and no evidence of effusions. Both the C-reactive protein and white cell count are normal.

Answer: Slipped upper femoral epiphysis


Slipped upper femoral epiphysis is the commonest adolescent hip disorder. It occurs most commonly in obese males. It may
often present as knee pain which is usually referred from the ipsilateral hip. The knee itself is normal. The hip often limits
internal rotation. The diagnosis is easily missed. X-rays will show displacement of the femoral epiphysis and the degree of its
displacement may be calculated using the Southwick angle. Treatment is directed at preventing further slippage which may
result in avascular necrosis of the femoral head.

59. A 6 year old boy presents with pain in the hip it is present on activity and has been worsening over the past few weeks.
There is no history of trauma. He was born by normal vaginal delivery at 38 weeks gestation On examination he has an
antalgic gait and limitation of active and passive movement of the hip joint in all directions. C-reactive protein is mildly
elevated at 10 but the white cell count is normal.
Answer: Perthes disease
This is a typical presentation for Perthes disease. X-ray may show flattening of the femoral head or fragmentation in more
advanced cases.

60. A 30 year old man presents with severe pain in the left hip it has been present on and off for many years. He was born at
39 weeks gestation by emergency caesarean section after a long obstructed breech delivery. He was slow to walk and as a
child was noted to have an antalgic gait. He was a frequent attender at the primary care centre and the pains dismissed
as growing pains. X-rays show almost complete destruction of the femoral head and a narrow acetabulum.

Answer: Developmental dysplasia of the hip


Developmental dysplasia of the hip. Usually diagnosed by Barlow and Ortolani tests in early childhood. Most Breech deliveries
are also routinely subjected to USS of the hip joint. At this young age an arthrodesis may be preferable to hip replacement. Early
plain x-ray changes in Perthes Disease:Widening of the joint space and sub chondral linear lucency.

Theme: Paediatric fractures

A. Non accidental injury


B. Accidental fracture
C. Rickets
D. Metabolic bone disease of prematurity
E. Hypophosphataemic rickets
F. Osteopetrosis
G. Osteogenesis imperfecta
H. Hypoparathyroidism
I. Osteoporosis

Please select the most likely explanation for each of the following injury scenarios. Each option may be used once, more than
once or not at all.

61. A toddler aged 3 years presents to the Emergency Department with swelling of his leg and is found to have a spiral
fracture of the tibia. His mother reports that he had tripped and fallen the previous day but she had not noticed any sign
of injury at the time. She is a single parent with little family support. The child is not on the child protection register.
Answer is Non accidental injury
Delayed presentation is unusual and should raise concern. In addition spiral fractures are usually the result of rotational injury
which is not compatible with the mechanism proposed by the parent.

62. A 5 month baby boy presents with swelling of his right arm and is found to have a spiral fracture of the humerus. He had
been in the care of her mother's boyfriend who reported that he had nearly dropped her that day when reaching for his
bottle and had inadvertently pulled on his arm to save him. He was immediately taken to the Emergency Department.

562
Answer: Accidental fracture
The mechanism fits with the fracture pattern and the presentation is not delayed.

63. An infant is admitted with symptoms and signs of respiratory infection and is found to have several posterior rib
fractures on chest radiograph. He was born prematurely at 37 weeks' gestation and was observed overnight on the
special care baby unit for tachypnoea which settled by the following day. On assessment it is also apparent that his head
circumference has increased at an excessive rate and has crossed 3 centiles since birth.
Answer is Non accidental injury
Posterior rib fractures are extremely unusual in neonates. The change in head size may be accounted for by hydrocephalus which
may occur as a sequelae from head injury.

64. In paediatric orthopaedic surgery, which of the following does not fulfill the Kocher criteria for septic arthritis?
A. ESR > 40mm/h
B. Positive blood culture
C. Fever
D. White cell count > 12, 000
E. Non weight bearing on the affected side
Answer B
Kocher criteria: 1. Non weight bearing on affected side; 2. ESR > 40 mm/hr; 3. Fever; 4. WBC count of >12,000 mm3. When 4/4
criteria are met, there is a 99% chance that the child has septic arthritis. The Kocher criteria do not consider blood culture results.
Septic arthritis: Staph aureus commonest organism. Urgent washout and antibiotics otherwise high risk of joint destruction.
Diagnosis: Plain x-rays. Consider aspiration

Theme: Ankle fractures

A. Surgical fixation
B. Below knee amputation
C. Aircast boot
D. Application of full leg plaster cast to include midfoot
E. Application of below knee plaster cast to include the midfoot
F. Application of external fixation device
G. Application of compression bandage and physiotherapy.

Please select the most appropriate management for the injury type described. Each option may be used once, more than once or
not at all.

65. A 24 year old man falls sustaining an inversion injury to his ankle. On examination he is tender over the lateral malleolus
only. On x-ray there is a fibular fracture that is distal to the syndesmosis.

Answer: Application of below knee plaster cast to include the midfoot


These distal injuries are generally managed conservatively. Conservative management will involve a below knee cast, this will
need to extend to the midfoot. It can be substituted for an aircast boot once radiological union is achieved.

66. An 86 year old lady stumbles and falls whilst opening her front door. On examination her ankle is swollen with both
medial and lateral tenderness. X rays demonstrate a fibular fracture at the level of the syndesmosis.
Answer: Application of below knee plaster cast to include the midfoot
Although, this is a potentially unstable injury operative fixation in this age group generally gives poor results owing to poor
quality bone. A below knee cast should be applied in the first instance. If this fails to provide adequate control it can be extended
above the knee.

67. A 25 year old man suffers an injury whilst playing rugby involving a violent twist to his left lower leg. On examination
both malleoli are tender and the ankle joint is very swollen. On x-ray there is a spiral fracture of the fibula and widening
of the ankle mortise.

Answer: Surgical fixation


This is a variant of the Weber C fracture in which disruption of the tibio-fibular syndesmosis occurs leading to joint disruption.
Surgical repair is warranted.

Ankle fractures are a common cause of admission to casualty. Clinical examination is facilitated by the Ottawa ankle rules to try
and minimise the unnecessary use of x-rays. These state that x-rays are only necessary if there is pain in the malleolar zone and:

563
1. Inability to weight bear for 4 steps; 2. Tenderness over the distal tibia; 3. Bone tenderness over the distal fibula. A number of
classification systems exist for describing ankle fractures, these include the Potts, Weber and AO systems. For simplicity the
Weber system is outlined here.

Weber classification
Related to the level of the fibular fracture: Type A is below the syndesmosis. Type B fractures start at the level of the tibial
plafond and may extend proximally to involve the syndesmosis. Type C is above the syndesmosis which may itself be damaged

A subtype known as a Maisonneuve fracture may occur with spiral tibial fracture that leads to disruption of the syndesmosis
with widening of the ankle joint, surgery is required.

Management: Depends upon stability of ankle joint and patient co-morbidites. All ankle fractures should be promptly reduced to
remove pressure on the overlying skin and subsequent necrosis. Young patients, with unstable, high velocity or proximal injuries
will usually require surgical repair. Often using a compression plate. Elderly patients, even with potentially unstable injuries
usually fare better with attempts at conservative management as their thin bone does not hold metalwork well.

Theme: Management of hip fractures

A. Hemiarthroplasty cemented prosthesis


B. Hemiarthroplasty non cemented prosthesis
C. Percutaneous pinning
D. Conservative management
E. Dynamic hip screw
F. Intramedullary device
G. Hip arthrodesis
H. Total hip replacement

For each fracture scenario please select the most appropriate management option from the list. Each option may be used once,
more than once or not at all.

68. A 72 year old retired teacher is admitted to A&E with a fall and hip pain. He is normally fit and well. He lives with his
son in a detached, 2 storey house. A hip x-ray confirms an extracapsular fracture.
Answer is Dynamic hip screw
Extracapsular fractures should be treated surgically. Since the blood supply to the femoral head is not compromised joint
replacement is not usually warranted.

69. A 72 year old retired teacher is admitted to A&E with a fall and hip pain. He is normally fit and well. He lives with his
son in a detached, 2 storey house. A hip x-ray confirms an subtrochanteric fracture.
Answer: Intramedullary device
Intramedullary device is normally recommended for reverse oblique, transverse or subtrochanteric fractures.

70. An 86 year old retired pharmacist is admitted to A&E following a fall. She complains of right hip pain. She is known to
have hypertension and is currently on bendrofluazide. She lives alone and does not mobilise. Her right leg is shortened
and externally rotated. A hip x-ray confirms a displaced intracapsular fracture.

Answer is Hemiarthroplasty non cemented prosthesis


This patient warrants a hemiarthroplasty due to reduced mobility and older age. The anterolateral approach is recommended in
the SIGN guidelines. In this case most surgeons would not use a cemented prosthesis.
Blood supply disruption is most common following Types III and IV.

Theme: Paediatric orthopaedics


A. USS hip
B. Hip x-ray
C. Anteroposterior pelvic x-ray
D. CT scan
E. MRI scan
F. Technetium bone scan
G. USS knee
H. X-ray knee
I. Discharge and reassure

For each of the following scenarios which is the most appropriate investigation? Each option may be used once, more than once or
not at all.
564
71. An obese 12 year old boy presents with knee pain. On examination he has pain on internal rotation of the hip. His knee is
clinically normal.

Answer: Hip x-ray


The main differential diagnosis in a boy over 10 years old is of slipped upper femoral epiphysis. Knee pain is a common
presenting feature. An anteroposterior pelvic x-ray may miss a minor slip, therefore request a hip film.

72. A baby is delivered in the breech position. Barlows and Ortolani tests are normal

Answer: USS hip


This child is at risk of developmental dysplasia of the hip (up to 20% will have DDH), so should have the hip joints scanned to
exclude this.

73. A 5 year old boy presents with a painful limp. The symptoms have been present for 8 weeks. Two hip x-rays have been
performed and appear normal.

Answer is Technetium bone scan


Perthes disease should be suspected in boys over 4 years old presenting with a limp. Early disease can be missed on x-ray,
therefore a bone scan should be performed. MRI is less sensitive than the bone scan.

74. A 5 year old boy is playing in a tree when he falls and lands on his right forearm. He is brought to the emergency
department by his parents. On examination he has bony tenderness and bruising. An X-ray is taken and shows unilateral
cortical disruption is development of periosteal haematoma. Which of the following is the most likely diagnosis?
A. Buckle fracture
B. Greenstick fracture
C. Toddlers fracture
D. Complete fracture
E. None of the above
Answer: B
Greenstick fractures are common childhood injuries. Unilateral cortical disruption is the main radiological feature, since
involvement of both cortices makes the injury a complete fracture. Buckle fractures will show periosteal haematoma formation
only.

Theme: Eponymous fractures

A. Smith's
B. Bennett's
C. Monteggia's
D. Colle's
E. Galeazzi
F. Pott's
G. Barton's

Which is the most likely eponymous fracture for the scenario given. Each option may be used once, more than once or not at all.

75. A 14 year old boy jumps off a 10 foot wall and lands on both feet. An x-ray shows a bimalleolar fracture of the right
ankle.
Answer: Pott's

76. A 22 year old drunk man is involved in a fight. He hurts his thumb when he punches his opponent.
Answer: Bennett's

77. A 63 year nurse falls on an extended and pronated wrist. An x-ray shows a distal radial fracture with radiocarpal
dislocation.
Answer: Barton's

Theme: Fracture management

A. Application of external fixator


B. Open reduction and internal fixation

565
C. Fasciotomy
D. Skeletal traction

For the following upper limb injuries please select the most appropriate initial management. Each option may be used once, more
than once or not at all.

78. A 32 year old man falls from a ladder and sustains a fracture of his proximal radius. On examination he has severe
pain in his forearm and diminished distal sensation. There is a single puncture wound present at the fracture site.

Answer: Fasciotomy
Pain and neurological symptoms in a tight fascial compartment coupled with a high velocity injury carry a high risk of
compartment syndrome and prompt fasciotomy should be performed.

79. A 32 year old man falls a sustains a fracture of his distal humerus. The fracture segment is markedly angulated and
unstable. There is a puncture site overlying the fracture site.
The correct answer is Application of external fixator
Wide exposure to plate the humerus is generally inadvisable owing to its many important anatomical relations. Both
intramedullary nailing and external fixation are reasonable treatments. However, in the presence of an open fracture application of
an external fixator and appropriate tissue debridement would be most appropriate.

80. A 24 year old man sustains a distal radius fracture during a game of rugby. Imaging shows a comminuted fracture
with involvement of the articular surface.
Answer is Open reduction and internal fixation
Meticulous anatomical alignment of the fracture segments is crucial to avoid the development of osteoarthritis and risk of
malunion.

Theme: Fracture management

A. Copious lavage and generous surgical debridement, followed by external fixation


B. Intramedullary nail
C. Open reduction and internal fixation
D. Immobilisation in plaster cast
E. External fixation using a frame device
F. Amputation
G. Application of external fixation device
H. Primary closure of wound and application of plaster cast

Please select the most appropriate management for the fractures described. Each option may be used once, more than once or not
at all.

81. A 55 year old motorcyclist is involved in a road traffic accident and sustained a Gustilo and Anderson IIIc type
fracture to the distal tibia. He was trapped in the wreckage for 7 hours during which time he bled profusely from the
fracture site. He has an established distal neurovascular deficit.

Answer: Amputation
This man is unstable, and at 7 hours after extraction, the limb is not viable. The safest option is primary amputation.

82. A 25 year old ski instructor who falls off a ski lift and sustains a spiral fracture of the mid shaft of the tibia. Attempts
to achieve satisfactory position in plaster have failed. Overlying tissues are healthy.

Answer:Intramedullary nail
This would be a good case for intramedullary nailing. Open reduction and external fixation would strip off otherwise healthy
tissues and hence is unsuitable. In some units the injury may be managed with an Ilizarov frame device but the majority would
treat with IM nailing.

83. A 35 year old mechanic is hit by a fork lift truck. He sustains a Gustilo and Anderson type IIIA fracture of the shaft of
the left femur.
Answer: Copious lavage and generous surgical debridement, followed by external fixation
At the tissues are in better shape than in the first case and as there is no associated vascular injury the patient may be suitable for
debridement of the area and external fixation. If debridement leaves a tissue defect then plastic surgical repair will be needed at a
later stage. Delayed treatment of open fractures with significant vascular injury may be best treated by primary amputation.

566
Preoperative
1. A 56 year old lady with idiopathic thrombocytopenic purpura has a platelet count of 50. She is due to undergo a
splenectomy. What is the optimal timing of a platelet transfusion in this case?
A. 24 hours pre-operatively
B. 2 hours pre-operatively
C. Whilst making the skin incision
D. After ligation of the splenic artery
E. On removal of the spleen
Answer: D
ITP causes splenic sequestration of platelets. Therefore a platelet transfusion should be carefully timed. Too soon and it will be
ineffective. Too late and unnecessary bleeding will occur. The optimal time is after the splenic artery has been ligated.

SplenectomyIndications: Trauma: 1/4 are iatrogenic. Spontaneous rupture: EBV. Hypersplenism: hereditary spherocytosis or
elliptocytosis etc. Malignancy: lymphoma or leukaemia. Splenic cysts, hydatid cysts, splenic abscesses

Post splenectomy changes: Platelets will rise first (therefore in ITP should be given after splenic artery clamped). Blood film will
change over following weeks, Howell Jolly bodies will appear. Other blood film changes include target cells and Pappenheimer
bodies. Increased risk of post splenectomy sepsis, therefore prophylactic antibiotics and pneumococcal vaccine should be given.

Post splenectomy sepsis: Typically occurs with encapsulated organisms. Opsonisation occurs but then not recognized

2. A 19 year old man has a skin lesion excised from his back. He is reviewed clinically at 4 months post procedure and the
surgeon notes that the scar has begun to contract. Which of the following facilitates this process?
A. Myofibroblasts
B. Neutrophils
C. Granuloma formation
D. Macrophages
E. Fibroblasts
Answer: A
As wounds mature the fibroblast population differentiates into myofibroblasts, these have a contractile phenotype and therefore
help in contracting the wound. Immature fibroblasts, though able to adhere to the ECM, do not have this ability.

Wound healing: Surgical wounds are either incisional or excisional and either clean, clean contaminated or dirty. Although the
stages of wound healing are broadly similar their contributions will vary according to the wound type.

The main stages of wound healing include: Haemostasis: Vasospasm in adjacent vessels, platelet plug formation and generation
of fibrin rich clot.

Inflammation: Neutrophils migrate into wound (function impaired in diabetes). Growth factors released, including basic
fibroblast growth factor and vascular endothelial growth factor. Fibroblasts replicate within the adjacent matrix and migrate into
wound. Macrophages and fibroblasts couple matrix regeneration and clot substitution.

Regeneration: Platelet derived growth factor and transformation growth factors stimulate fibroblasts and epithelial cells.
Fibroblasts produce a collagen network. Angiogenesis occurs and wound resembles granulation tissue.

Remodeling: Longest phase of the healing process and may last up to one year (or longer). During this phase fibroblasts become
differentiated (myofibroblasts) and these facilitate wound contraction. Collagen fibres are remodeled. Microvessels regress
leaving a pale scar.

The above description represents an idealised scenario. A number of diseases may distort this process. It is obvious that one of the
key events is the establishing well vascularised tissue. At a local level angiogenesis occurs, but if arterial inflow and venous return
are compromised then healing may be impaired, or simply nor occur at all. The results of vascular compromise are all too
evidence in those with peripheral vascular disease or those poorly constructed bowel anastomoses.

Conditions such as jaundice will impair fibroblast synthetic function and overall immunity with a detrimental effect in most parts
of healing.

Problems with scars: Hypertrophic scars: Excessive amounts of collagen within a scar. Nodules may be present histologically
containing randomly arranged fibrils within and parallel fibres on the surface. The tissue itself is confined to the extent of the
wound itself and is usually the result of a full thickness dermal injury. They may go on to develop contractures.

Image of hypertrophic scarring. Note that it remains confined to the boundaries of the original wound:

567
Keloid scars: Excessive amounts of collagen within a scar. Typically a keloid scar will pass beyond the boundaries of the original
injury. They do not contain nodules and may occur following even trivial injury. They do not regress over time and may recur
following removal.

Image of a keloid scar. Note the extension beyond the boundaries of the original incision:

Drugs which impair wound healing: Non steroidal anti inflammatory drugs. Steroids. Immunosupressive agents. Anti neoplastic
drugs

Closure: Delayed primary closure is the anatomically precise closure that is delayed for a few days but before granulation tissue
becomes macroscopically evident.

Secondary closure refers to either spontaneous closure or to surgical closure after granulation tissue has formed.

3. Which of the following blood products can be administered to a non ABO matched recipient?
A. Whole blood
B. Platelets
C. Packed red cells
D. Fresh frozen plasma
E. Cryoprecipitate
Answer: B

In the UK, platelets either come from pooling of the platelet component from four units of whole donated blood, called random
donor platelets, or by plasmapharesis from a single donor. The platelets are suspended in 200-300 ml of plasma and may be stored
for up to 4 days in the transfusion laboratory where they are continually agitated at 22oC to preserve function. One adult platelet
pool raises the normal platelet count (150 - 450 platelets x 109/litre) by 510 platelets x 109/litre. ABO identical or compatible
platelets are preferred but not necessary in adults; but rhesus compatibility is required in recipients who are children and women
of childbearing age to prevent haemolytic disease of the newborn.

Blood products - cross matching: Whole blood fractions


Fraction Key points
Packed red cells Used for transfusion in chronic anaemia and cases where infusion of large volumes of fluid may result in
cardiovascular compromise. Product obtained by centrifugation of whole blood.
Platelet rich Usually administered to patients who are thrombocytopaenic and are bleeding or require surgery. It is
plasma obtained by low speed centrifugation.
Platelet Prepared by high speed centrifugation and administered to patients with thrombocytopaenia.
concentrate
Fresh frozen  Prepared from single units of blood.
plasma  Contains clotting factors, albumin and immunoglobulin.
 Unit is usually 200 to 250ml.
 Usually used in correcting clotting deficiencies in patients with hepatic synthetic failure who are
due to undergo surgery.
 Usual dose is 12-15ml/Kg-1.
 It should not be used as first line therapy for hypovolaemia.

Cryoprecipitate  Formed from supernatant of FFP.


 Rich source of Factor VIII and fibrinogen.
 Allows large concentration of factor VIII to be administered in small volume.

SAG-Mannitol Removal of all plasma from a blood unit and substitution with:
Blood
 Sodium chloride
 Adenine
 Anhydrous glucose
 Mannitol

Up to 4 units of SAG M Blood may be administered. Thereafter whole blood is preferred. After 8 units,
clotting factors and platelets should be considered.

568
Cross matching
Must be cross matched Can be ABO incompatible in adults
Packed red cells Platelets
Fresh frozen plasma
Cryoprecipitate
Whole blood

4. Which of the following would be the optimal fluid management option for a 45 year old man due to undergo an elective
right hemicolectomy?
A. Remain "nil by mouth" for at least 6 hours preoperatively and avoid intra venous fluids.
B. Remain "nil by mouth" for at least 6 hours pre operatively and receive supplementary intravenous 5% dextrose to replace lost
calories
C. Allow him free access to oral fluids only until 30 minutes prior to surgery
D. Administer a carbohydrate based loading drink 3 hours pre operatively, and avoid intravenous fluids.
E. Administer a carbohydrate based loading drink 6 hours pre operatively and administer 5% dextrose saline thereafter
Answer: D
Patients for elective surgery should not have solids for 6 hours pre operatively. However, clear fluids may be given up to 2 hours
pre operatively. Enhanced recovery programmes are now the standard of care in many countries around the world and involve
administration of carbohydrate loading drinks.
The routine administration of 5% dextrose in the scenarios given above would convey little in the way of benefit and increase the
risks of electrolyte derangement post operatively.

Pre operative fluid management: Fluid management has been described in the British Consensus guidelines on IV fluid
therapy for Adult Surgical patients (GIFTASUP)

The Recommendations include: Use Ringer's lactate or Hartmann's when a crystalloid is needed for resuscitation or replacement
of fluids. Avoid 0.9% N. Saline (due to risk of hyperchloraemic acidosis) unless patient vomiting or has gastric drainage.Use
0.4%/0.18% dextrose saline or 5% dextrose in maintenance fluids. It should not be used in resuscitation or as replacement fluids.

Adult maintenance fluid requirements are: Na 50-100 mmol/day and K 40-80 mmol/day in 1.5-2.5L fluid per day. Patients for
elective surgery should NOT be nil by mouth for >2 hours (unless has disorder of gastric emptying).

Patients for elective surgery should be given carbohydrate rich drinks 2-3h before. Ideally this should form part of a normal pre op
plan to facilitate recovery. Avoid mechanical bowel preparation.

If bowel prep is used, simultaneous administration of Hartmann's or Ringer's lactate should be considered. Excessive fluid losses
from vomiting should be treated with a crystalloid with potassium replacement. 0.9% N. Saline should be given if there is
hypochloraemia. Otherwise Hartmann's or Ringer lactate should be given for diarrhoea/ileostomy/ileus/obstruction. Hartmann's
should also be given in sodium losses secondary to diuretics. High risk patients should receive fluids and inotropes.

An attempt should be made to detect pre or operative hypovolaemia using flow based measurements. If this is not available, then
clinical evaluation is needed i.e. JVP, pulse volume etc.

In Blood loss or infection causing hypovolaemia should be treated with a balanced crystalloid or colloid (or until blood available
in blood loss). A critically ill patient is unable to excrete Na or H20 leading to a 5% risk of interstitial oedema. Therefore 5%
dextrose as well as colloid should be given. Give 200mls of colloid in hypovolaemia, repeat until clinical parameters improve.

Theme: Local anaesthetics

A. 1% xylocaine with 1 in 200,000 adrenaline


B. 1% Lignocaine
C. 0.5% Bupivacaine with 1 in 200,000 adrenaline
D. 0.5% Bupivicaine
E. Prilocaine 1%
F. Procaine 1%
G. Cocaine 25%
H. Cocaine 10%

Please select the local anaesthetic formulation most appropriate to the procedure indicated. Each option may be used once, more
than once or not at all.

569
5. A 25 year old male presents with epistaxis, the ENT SpR plans to cauterise the bleeding point with silver nitrate.

1% xylocaine with 1 in 200,000 adrenaline. Historically cocaine was popular for the management of epistaxis. Some surgeons
will still routinely use cocaine paste for this indication. Its popularity stems from the fact that it causes vasospasm. However,
systemic absorption carries the risk of adverse reactions. Where it is used the correct dose is 4%. Topically applied short acting
local anaesthetic agents with adrenaline may produce similar effects, with lower risks of toxicity.

6. An 18 year old boy requires a Zadeks procedure.

1% Lignocaine. This is excision of the toe nail and a fast acting local anaesthetic is indicated. Adrenaline should be avoided in
this setting as it can cause digital ischaemia

7. A 72 year old women fracture her distal radius. A Biers Block is planned to facilitate reduction of the fracture.

Prilocaine 1%. This is the best local anaesthetic for this. Bupivicaine may cause cardiotoxicity and should be avoided. Local
anaesthetics: avoid use of adrenaline in extremities

Local anaesthetic agents: Lidocaine: An amide. Local anaesthetic and a less commonly used antiarrhythmic (affects Na
channels in the axon). Hepatic metabolism, protein bound, renally excreted. Toxicity: due to IV or excess administration.
Increased risk if liver dysfunction or low protein states. Note acidosis causes lidocaine to detach from protein binding. Drug
interactions: Beta blockers, ciprofloxacin, phenytoin. Features of toxicity: Initial CNS over activity then depression as lidocaine
initially blocks inhibitory pathways then blocks both inhibitory and activating pathways. Cardiac arrhythmias. Increased doses
may be used when combined with adrenaline to limit systemic absorption.

Cocaine: Pure cocaine is a salt, usually cocaine hydrochloride. It is supplied for local anaesthetic purposes as a paste. It is
supplied for clinical use in concentrations of 4 and 10%. It may be applied topically to the nasal mucosa. It has a rapid onset of
action and has the additional advantage of causing marked vasoconstriction. It is lipophillic and will readily cross the blood brain
barrier. Its systemic effects also include cardiac arrhythmias and tachcardia. Apart from its limited use in ENT surgery it is
otherwise used rarely in mainstream surgical practice.

Bupivicaine: Bupivacaine binds to the intracellular portion of sodium channels and blocks sodium influx into nerve cells, which
prevents depolarization. It has a much longer duration of action than lignocaine and this is of use in that it may be used for topical
wound infiltration at the conclusion of surgical procedures with long duration analgesic effect. It is cardiotoxic and is therefore
contra indicated in regional blockage in case the tourniquet fails. The co-administration of adrenaline concentrates it at the site of
action and allows the use of higher doses.

Prilocaine: Similar mechanism of action to other local anaesthetic agents. However, it is far less cardiotoxic and is therefore the
agent of choice for intravenous regional anaesthesia e.g. Biers Block.

All local anaesthetic agents dissociate in tissues and this contributes to their therapeutic effect. The dissociation constant shifts in
tissues that are acidic e.g. where an abscess is present and this reduce the efficacy.

Doses of local anaesthetics


Agent Dose plain Dose with adrenaline
Lignocaine 3mg/Kg 7mg/Kg
Bupivicane 2mg/Kg 2mg/Kg
Prilocaine 6mg/Kg 9mg/Kg
These are a guide only as actual doses depend on site of administration, tissue vascularity and co-morbidities.

8. Which statement relating to the peri operative management of patients with diabetes mellitus is false?
A. They should be placed first on the operating list
B. An intravenous sliding scale should be used in all cases
C. Potassium supplementation is likely to be required in diabetics on a sliding scale
D. Electrolyte abnormalities are more common after major visceral resections
E. Blood glucose monitoring is required during general anaesthesia
Answer: B
This is not the case and some type 2 diabetics may be managed using a watch and wait policy with regular blood glucose
monitoring. The cellular shifts of potassium with sliding scales may cause problems with electrolyte management which should be
anticipated.

Preparation for surgery: Elective and emergency patients require different preparation.

570
Elective cases: Consider pre admission clinic to address medical issues. Blood tests including FBC, U+E, LFTs, Clotting, Group
and Save. Urine analysis. Pregnancy test. Sickle cell test. ECG/ Chest x-ray

Exact tests to be performed will depend upon the proposed procedure and patient fitness. Risk factors for development of deep
vein thrombosis should be assessed and a plan for thromboprophylaxis formulated.

Diabetes: Diabetic patients have greater risk of complications. Poorly controlled diabetes carries high risk of wound infections.
Patients with diet or tablet controlled diabetes may be managed using a policy of omitting medication and checking blood glucose
levels regularly. Diabetics who are poorly controlled or who take insulin will require a intravenous sliding scale. Potassium
supplementation should also be given. Diabetic cases should be operated on first.

Emergency cases: Stabilise and resuscitate where needed. Consider whether antibiotics are needed and when and how they
should be administered. Inform blood bank if major procedures planned particularly where coagulopathies are present at the outset
or anticipated (e.g. Ruptured AAA repair). Don't forget to consent and inform relatives.

Special preparation: Some procedures require special preparation: Thyroid surgery; vocal cord check. Parathyroid surgery;
consider methylene blue to identify gland. Sentinel node biopsy; radioactive marker/ patent blue dye. Surgery involving the
thoracic duct; consider administration of cream. Pheochromocytoma surgery; will need alpha and beta blockade. Surgery for
carcinoid tumours; will need covering with octreotide. Colorectal cases; bowel preparation (especially left sided surgery).
Thyrotoxicosis; lugols iodine/ medical therapy.

Theme: Surgical complications

A. Anastamotic leak
B. Chyle leak
C. Air leak
D. Biliary leak
E. Deep vein thrombosis
F. Portal vein thrombosis
G. Biliary obstruction

Please select the most likely complication for the scenario given. Each option may be used once, more than once or not at all.

9. A 67 year old female undergoes an oesophagogastrectomy for carcinoma of the distal oesophagus. She complains of
chest pain. The following day there is brisk bubbling into the chest drain when suction is applied.

Air leak. Damage to the lung substance may produce an air leak. Air leaks will manifest themselves as a persistent
pneumothorax that fails to settle despite chest drainage. When suction is applied to the chest drainage system, active and
persistent bubbling may be seen. Although an anastomotic leak may produce a small pneumothorax, a large volume air leak is
more indicative of lung injury.

10. A 20 year old man has a protracted stay on ITU following a difficult appendicectomy for perforated appendicitis with
pelvic and sub phrenic abscesses. He has now deteriorated further and developed deranged liver function tests.

The correct answer is Portal vein thrombosis. Such marked intra-abdominal sepsis may well produce coagulopathy and the risk
of portal vein thrombosis.

11. A 63 year old man undergoes an Ivor - Lewis oesophagogastrectomy for carcinoma of the distal oesophagus. The
following day a pale opalescent liquid is noted to be draining from the right chest drain.

Chyle leak. Damage to the lymphatic duct may occur during this procedure and some surgeons administer a lipid rich material
immediately prior to surgery to facilitate its identification in the event of iatrogenic damage.

Surgical complications: Complications occur in all branches of surgery and require vigilance in their detection. In many cases
anticipating the likely complications and appropriate avoidance will minimise their occurrence. For the purposes of the MRCS the
important principles to appreciate are: The anatomical principles that underpin complications. The physiological and biochemical
derangements that occur. The most appropriate diagnostic modalities to utilize. The principles which underpin their management.
This is clearly a very broad area and impossible to cover comprehensively. There is considerable overlap with other topic areas
within the website.

Avoiding complications: Some points to hopefully avert complications: World Health Organisation checklist- now mandatory
571
prior to all operations. Prophylactic antibiotics - right dose, right drug, right time. Assess DVT/ PE risk and ensure adequate
prophylaxis. MARK site of surgery. Use tourniquets with caution and with respect for underlying structures.
Remember the danger of end arteries and in situations where they occur avoid using adrenaline containing solutions and
monopolar diathermy. Handle tissues with care- devitalised tissue serves as a nidus for infection. Be very wary of the potential for
coupling injuries when using diathermy during laparoscopic surgery. The inferior epigastric artery is a favourite target for
laparoscopic ports and surgical drains!

Anatomical principles: Understanding the anatomy of a surgical field will allow appreciation of local and systemic complications
that may occur. For example nerve injuries may occur following surgery in specific regions the table below lists some of the more
important nerves to consider and mechanisms of injury

Nerve Mechanism
Accessory Posterior triangle lymph node biopsy
Sciatic Posterior approach to hip
Common peroneal Legs in Lloyd Davies position
Long thoracic Axillary node clearance
Pelvic autonomic nerves Pelvic cancer surgery
Recurrent laryngeal nerves During thyroid surgery
Hypoglossal nerve During carotid endarterectomy
Ulnar and median nerves During upper limb fracture repairs

These are just a few. The detailed functional sequelae are particularly important and will often be tested. In addition to nerve
injuries certain procedures carry risks of visceral or structural injury. Again some particular favourites are given below:

Structure Mechanism
Thoracic duct During thoracic surgery e.g. Pneumonectomy, oesphagectomy
Parathyroid glands During difficult thyroid surgery
Ureters During colonic resections/ gynaecological surgery
Bowel perforation Use of Verres Needle to establish pneumoperitoneum
Bile duct injury Failure to delineate Calots triangle carefully and careless use of diathermy
Facial nerve Always at risk during Parotidectomy
Tail of pancreas When ligating splenic hilum
Testicular vessels During re-do open hernia surgery
Hepatic veins During liver mobilisation

Again many could be predicted from the anatomy of the procedure.

Physiological derangements: A very common complication is bleeding and this is covered under the section of haemorrhagic
shock. Another variant is infection either superficial or deep seated. The organisms are covered under microbiology and the
features of sepsis covered under shock. Do not forget that immunocompromised and elderly patients may present will atypical
physiological parameters.

Selected physiological and biochemical issues are given below:

Complication Physiological/ Biochemical Problem


Arrhythmias following cardiac Susceptibility to hypokalaemia (K+ <4.0 in cardiac patients)
surgery
Neurosurgical electrolyte SIADH following cranial surgery causing hyponatraemia
disturbance
Ileus following gastrointestinal Fluid sequestration and loss of electrolytes
surgery
Pulmonary oedema following Loss of lung volume makes these patients very sensitive to fluid overload
pneumonectomy
Anastamotic leak Generalised sepsis causing mediastinitis or peritonitis depending on site of leak
Myocardial infarct May follow any type of surgery and in addition to direct cardiac effects the decreased
cardiac output may well compromise grafts etc.

Try making a short list of problems and causes specific to your own clinical area.

Diagnostic modalities: Depends largely on the suspected complication. In the acutely unwell surgical patient the following
baseline investigations are often helpful: Full blood count, urea and electrolytes, C- reactive protein (trend rather than absolute
572
value), serum calcium, liver function tests, clotting (don't forget to repeat if on-going bleeding). Arterial blood gases. ECG
(+cardiac enzymes if MI suspected). Chest x-ray to identify collapse/ consolidation. Urine analysis for UTI
These will often identify the most common complications.

Special tests: CT scanning for identification of intra-abdominal abscesses, air and if luminal contrast is used an anastamotic leak.
Gatrograffin enema- for rectal anastamotic leaks. Doppler USS of leg veins- for identification of DVT. CTPA for PE. Sending
peritoneal fluid for U+E (if ureteric injury suspected) or amylase (if pancreatic injury suspected). Echocardiogram if pericardial
effusion suspected post cardiac surgery and no pleural window made.

Management of complications. The guiding principal should be safe and timely intervention. Patients should be stabilised and if
an operation needs to occur in tandem with resuscitation then generally this should be of a damage limitation type procedure
rather than definitive surgery (which can be more safely undertaken in a stable patient the following day).

Remember that recent surgery is a contra indication to thrombolysis and that in some patients IV heparin may be preferable to a
low molecular weight heparin (easier to reverse).
As a general rule laparotomies for bleeding should follow the core principle of quadrant packing and then subsequent pack
removal rather than plunging large clamps into pools of blood. The latter approach invariable worsens the situation is often
accompanied by significant visceral injury particularly when done by the inexperienced. If packing controls a situation it is
entirely acceptable practice to leak packs in situ and return the patient to ITU for pack removal the subsequent day.

Theme: Wound closure

A. Split thickness skin grafting


B. Full thickness skin graft
C. Local flap
D. Leave wound as it is and apply a simple dressing
E. Primary closure using interrupted 3/0 silk
F. Primary closure using 4/0 interrupted nylon
G. Use of vacuum assisted closure device

Please select the most appropriate wound closure modality for the scenario given. Each option may be used once, more than once
or not at all.

12. A 68 year old man undergoes a wide local excision of a squamous cell carcinoma from the lateral aspect his nose. At the
completion of the operation the alar cartilage is visible.

The correct answer is Local flap. This type of wound should be managed with a local rotational flap.
This question has been used in the EMQ paper several times

13. A 68 year old man has a seborrhoiec wart on his left cheek this is removed by use of curretage leaving a superficial
defect approximately 1cm in diameter

The correct answer is Leave wound as it is and apply a simple dressing. This type of superficial wound will re-epithelialise
satisfactorily without grafting.

14. A 2 year old child accidentally falls onto a hot iron. He sustains a 5cm full thickness burn to dorsum of his hand.

The correct answer is Full thickness skin graft. Grafting is indicated as the wound will invariably contract during the scarring
process.

15. Which of the following statements relating to pre operative fluid management is false?
A. 5% dextrose should be given cautiously in the elderly
B. Patients undergoing elective colonic resections may continue to drink water up to 2 hours prior to surgery
C. Normal saline increases the risk of hyperchloraemic acidosis
D. A 70kg man will need approximately 100mmol of sodium daily
E. Carbohydrate rich beverages and loading drinks can cause ileus therefore should be avoided
Answer: E

Carbohydrate loading is one of the enhanced recovery principles.

573
Pre operative fluid management: Fluid management has been described in the British Consensus guidelines on IV fluid
therapy for Adult Surgical patients (GIFTASUP)

Theme: ASA scoring


A. ASA 1
B. ASA 2
C. ASA 3
D. ASA 4
E. ASA 5

The American society of anaesthesiologists physical status scoring system is a popular method for stratifying patients physical
status. Please select the most appropriate ASA grade for each of the following scenarios. Each option may be used once, more
than once or not at all.

16. A 66 year old man is admitted following a collapse whilst waiting for a bus. Clinical examination confirms a ruptured
abdominal aortic aneurysm. He is moribund and hypotensive

ASA 5. Patients who are moribund and will not survive without surgery are graded as ASA 5. Theme from 2009 Exam

17. A 23 year old man with a 4cm lipoma on his flank is due to have this removed as a daycase. He is otherwise well.

ASA 1. Absence of co-morbidities and small procedure with no systemic compromise will equate to an ASA score of 1.

18. A 72 year old man is due to undergo an inguinal hernia repair. He suffers from COPD and has an exercise tolerance of
10 yards. He also has pitting oedema to the thighs.

The correct answer is ASA 4. Severe systemic disease of this nature is a constant threat to life. Especially as he also has
evidence of cardiac failure.

American Society of anesthesiologists physical status scoring system (ASA)


ASA Description
grade
1 No organic physiological, biochemical or psychiatric disturbance. The surgical pathology is localised and has not
invoked systemic disturbance.
2 Mild or moderate systemic disruption caused either by the surgical disease process or though underlying pre-existing
disease
3 Severe systemic disruption caused either by the surgical pathology or pre-existing disease
4 Patient has severe systemic disease that is a constant threat to life
5 A patient who is moribund and will not survive without surgery

Theme: Wound infections

A. <5%
B. 5-10%
C. 15-25%
D. 25-40%
E. 0%
F. 75-100%

Please select the anticipated risk of surgical site infections for the procedures described. Each option may be used once, more than
once or not at all.

19. A patient is undergoing a Hartmans procedure for perforated sigmoid diverticular disease.

The correct answer is 25-40%. This is a 'dirty' procedure and carries an SSI risk of 25-40 %.

20. A 23 year old male is undergoing an elective inguinal hernia repair.

<5%. This is a clean procedure and carries the lowest risk of SSI.

21. A 43 year old women is undergoing a laparoscopic choelcystectomy for uncomplicated biliary colic.

574
5-10%. This is a clean contaminated procedure as the cystic duct is divided. Inadvertent spill of bile converts the operation to a
contaminated one and the risk of infection rises.

Surgical site infection: Surgical site infections may occur following a breach in tissue surfaces and allow normal commensals and
other pathogens to initiate infection. They are a major cause of morbidity and mortality. Surgical site infections (SSI) comprise up
to 20% of all healthcare associated infections and at least 5% of patients undergoing surgery will develop an SSI as a result. In
many cases the organisms are derived from the patient's own body. Measures that may increase the risk of SSI include: Shaving
the wound using a razor (disposable clipper preferred). Using a non iodine impregnated incise drape if one is deemed to be
necessary. Tissue hypoxia. Delayed administration of prophylactic antibiotics in tourniquet surgery

Preoperatively: Don't remove body hair routinely. If hair needs removal, use electrical clippers with single use head (razors
increase infection risk). Antibiotic prophylaxis if: placement of prosthesis or valve. clean-contaminated surgery. contaminated
surgery. Use local formulary. Aim to give single dose IV antibiotic on anaesthesia. If a tourniquet is to be used, give prophylactic
antibiotics earlier

Intraoperatively: Prepare the skin with alcoholic chlorhexidine (Lowest incidence of SSI). Cover surgical site with dressing. A
recent meta analysis has confirmed that administration of supplementary oxygen does not reduce the risk of wound infection. In
contrast to previous individual RCT's(1)

Post operatively: Tissue viability advice for management of surgical wounds healing by secondary intention

Use of diathermy for skin incisions: In the NICE guidelines the use of diathermy for skin incisions is not advocated(2). Several
randomised controlled trials have been undertaken and demonstrated no increase in risk of SSI when diathermy is used(3).

Theme: Intravenous fluids

A. Gelofusine
B. Dextran 70
C. 0.4%/0.18% dextrose saline
D. 5% dextrose
E. 0.9% Normal saline
F. Hartmann's solution
G. Pentastarch
H. Fluid restriction 500mls
I. Fluid restriction 1L

Which of the following is the best fluid management for the scenario given? Each option may be used once, more than once or not
at all.

22. A 53 year old alcoholic male presents with acute pancreatitis. He is clinically dehydrated. His blood results show normal
renal function and electrolytes.

Hartmann's solution. This patient needs fluid replacement due to large third space losses. Hartmann's solution is recommended.
N. Saline would put this patient at risk of hyperchloraemic acidosis.

23. A 45 year old man with previous laparotomy is admitted with adhesional small bowel obstruction. He is managed with
prolonged nasogastric drainage.

0.9% Normal saline. This man will be hypochloraemic, therefore treatment is with 0.9% sodium chloride.

24. A 19 year old lady is admitted with pyelonephritis. She is in septic shock with a blood pressure of 95/60 and pulse rate of
110. Apart from an allergy to corn she has no other significant medical history.

Gelofusine. In patients with septic shock volume expansion should be considered with an agent that remains in the intra
vascular space for a prolonged period. Dextran 70 has been associated with adverse outcomes when used in this setting.
Pentastarch should not be used in patients with an allergy to corn. Therefore gelofusine is the most appropriate agent.

25. A 48 year old lady has a metallic heart valve and requires a paraumbilical hernia repair. Perioperatively she is receiving
intra venous unfractionated heparin. To perform the surgery safely a normal coagulation state is required. Which of the
following strategies is routine standard practice?
A. Administration of 10 mg of vitamin K the night prior to surgery and stopping the heparin infusion 6 hours pre operatively
575
B. Stopping the heparin infusion 6 hours pre operatively
C. Stop the heparin infusion on induction of anaesthesia
D. Stopping the heparin infusion 6 hours pre operatively and administration of intravenous protamine sulphate on commencing the
operation
E. None of the above
Answer: B
Patients with metallic heart valves will generally stop unfractionated heparin 6 hours pre operatively. Unfractionated heparin is
generally cleared from the circulation within 2 hours so this will allow plenty of time and is the method of choice in the elective
setting. Protamine sulphate will reverse heparin but is associated with risks of anaphylaxis and is thus not generally used unless
immediate reversal of anticoagulation is needed, e.g. coming off bypass.

Heparin: Causes the formation of complexes between antithrombin and activated thrombin/factors 7,9,10,11 & 12

Advantages of low molecular weight heparin: Better bioavailability. Lower risk of bleeding. Longer half life. Little effect on
APTT at prophylactic dosages. Less risk of HIT

Complications: Bleeding. Osteoporosis. Heparin induced thrombocytopenia (HIT): occurs 5-14 days after 1st exposure.
Anaphylaxis

In surgical patients that may need a rapid return to theatre administration of unfractionated heparin is preferred as low molecular
weight heparins have a longer duration of action and are harder to reverse.

Theme: Management of complications

A. Intra venous calcium


B. Intra venous potassium
C. Immediate removal of skin clips on ward
D. Removal of skin clips in theatre
E. Laryngoscopy
F. Intravenous thyroxine

Please select the most appropriate intervention for the scenario given. Each option may be used once, more than once or not at all.

26. A 22 year old lady undergoes a total thyroidectomy for Graves disease. 6 hours post operatively she develops respiratory
stridor and develops a small haematoma in the neck

The correct answer is Immediate removal of skin clips on ward. This is true emergency and evacuation and release of pressure
must be performed immediately, in this case by removal of skin clips on the ward

27. A 44 year old lady undergoes a total thyroidectomy for recurrent multinodular goitre. 3 days post operatively she is still
troubled by a hoarse voice.

Laryngoscopy. Unfortunately one of the laryngeal nerves may have been injured and this will be best demonstrated by
laryngoscopy.

28. A 48 year old lady undergoes a redo thyroidectomy for a multinodular goitre. 24 hours post operatively she develops
oculogyric crises and diffuse muscle spasm.

Intra venous calcium. She has most likely developed hypocalcaemic tetany and will require immediate calcium
supplementation.

Theme: Use of blood products in surgery

A. Wait and see


B. Vitamin K
C. Fresh frozen plasma
D. Cryoprecipitate
E. Platelet cells
F. Packed red cells
G. Human Prothrombin Complex
H. Blood from the cell saver salvaged during surgery
576
I. Human Prothrombin Complex and vitamin K

For each coagulation or bleeding problem please select the most appropriate item. Each item may be used once, more than once or
not at all.

29. A 74 year old male is admitted with a ruptured aortic aneurysm. He is hypotensive and tachycardic. He is urgently
transferred to theatre for a repair.

Blood from the cell saver salvaged during surgery. This is an emergency situation. There will be a huge volume of blood in the
abdomen which would be drained and filtered. This can be reinfused into the patient.

30. A 74 year old male with colon cancer sustains an iatrogenic splenic injury during surgery. He is bleeding profusely.

The correct answer is Packed red cells. The cell saver is inappropriate because the cells will be contaminated with malignant
cells and faecal matter from the open bowel.

31. A 53 year old cleaner is admitted with a fall. She is haemodynamically unstable and a CT has shown a massive
retroperitoneal haematoma. She is on warfarin.

The correct answer is Human Prothrombin Complex and vitamin K. Each hospital has different protocols and would recommend
discussion with a haematologist. However Human Prothrombin Complex with vitamin K is indicated in this situation, as the
condition is life threatening.

Blood products: Whole blood fractions


Packed red cells: Used for transfusion in chronic anaemia and cases where infusion of large volumes of fluid may result in
cardiovascular compromise. Product obtained by centrifugation of whole blood.

Platelet rich plasma: Usually administered to patients who are thrombocytopaenic and are bleeding or require surgery. It is
obtained by low speed centrifugation.

Platelet concentrate: Prepared by high speed centrifugation and administered to patients with thrombocytopaenia.

Fresh frozen plasma: Prepared from single units of blood. Contains clotting factors, albumin and immunoglobulin.Unit is
usually 200 to 250ml. Usually used in correcting clotting deficiencies in patients with hepatic synthetic failure who are due to
undergo surgery. Usual dose is 12-15ml/Kg-1. It should not be used as first line therapy for hypovolaemia.

Cryoprecipitate: Formed from supernatant of FFP. Rich source of Factor VIII and fibrinogen. Allows large concentration of
factor VIII to be administered in small volume.

SAG-Mannitol Blood: Removal of all plasma from a blood unit and substitution with: Sodium chloride, Adenine, Anhydrous
glucose, Mannitol. Up to 4 units of SAG M Blood may be administered. Thereafter whole blood is preferred. After 8 units,
clotting factors and platelets should be considered.

Cell saver devices: These collect patients own blood lost during surgery and then reinfuse it. There are two main types: Those
which wash the blood cells prior to reinfusion. These are more expensive to purchase and more complicated to operate. However,
they reduce the risk of reinfusing contaminated blood back into the patient. Those which do not wash the blood prior to reinfusion.
Their main advantage is that they avoid the use of infusion of blood from donors into patients and this may reduce risk of blood
borne infection. It may be acceptable to Jehovah's witnesses. It is contraindicated in malignant disease for risk of facilitating
disease dissemination.

Blood products used in warfarin reversal: In some surgical patients the use of warfarin can pose specific problems and may
require the use of specialised blood products

Immediate or urgent surgery in patients taking warfarin(1) (2): 1. Stop warfarin.


2. Vitamin K (reversal within 4-24 hours): -IV takes 4-6h to work (at least 5mg). -Oral can take 24 hours to be clinically effective

3. Fresh frozen plasma: Used less commonly now as 1st line warfarin reversal: -30ml/kg-1. -Need to give at least 1L fluid in 70kg
person (therefore not appropriate in fluid overload). -Need blood group. -Only use if human prothrombin complex is not available

4. Human Prothrombin Complex (reversal within 1 hour): -Bereplex 50 u/kg. -Rapid action but factor 6 short half life, therefore
give with vitamin K
577
32. A 22 year old man presents with a peri anal abscess, which is managed by incision and drainage. The perineal wound
measures 3cm by 3cm. Which of the following is best management option?
A. Primary closure with interrupted mattress sutures
B. Delayed primary closure with interrupted mattress sutures
C. Allow the wound to heal by secondary intention
D. Insert a seton through the cavity into the rectum to allow a mature fistula track to develop
E. Perform a V-Y flap 2 weeks later
Answer: C

Peri anal abscess are typically managed by secondary intention healing. Any attempt at early closure is at best futile and at worst
dangerous. Insertion of a seton may be considered by an experienced colorectal surgeon, and only if the tract is clearly identifiable
with minimal probing. There is seldom a need for flaps, ongoing discharge usually indicates a fistula (managed separately).

33. A surgeon is considering using lignocaine to provide local anaesthesia for a minor surgical procedure. Which of the
following may attenuate its action?
A. Hyperkalaemia
B. Administration with adrenaline
C. Administration with bupivicaine
D. Administration with sodium bicarbonate
E. Use in tissues which are infected
Answer: E
Do not use lignocaine with adrenaline in extremity surgery e.g ring blocks- risk of ischaemia.

Most anaesthetic agents are amine bases that become ionised due to the relative alkalinity of tissues. In active infection there may
acidosis of the tissues and therefore local anasthetics may be less effective. Some surgeons mix sodium bicarbonate as it is
reported to reduce the pain experienced by patients during administration.

34. Which of the following statements relating to use of total parenteral nutrition is untrue?
A. It may cause steatosis and derangement of liver function tests
B. Administration via a central line or PICC line is preferable to peripheral administration
C. It is highly irritant to vessel walls
D. It should be administered when a patient has an albumin less than15
E. Administration of TPN for periods of less than 1 week are unlikely to produce noticable benefits
Answer: D
Albumin is a poor indicator of overall nutrition and the decision to start TPN should not be based on this parameter alone.
Patients should ideally be fed enterally where possible and if this is likely to occur within 5-7 days then starting TPN is unlikely
to confer benefit.

Nutrition Monitoring-NICE guidelines: Weight: daily if fluid balance concerns, otherwise weekly reducing to monthly. BMI: at
start of feeding and then monthly. If weight cannot be obtained: monthly mid arm circumference or triceps skin fold thickness.
Daily electrolytes until levels stable. Then once or twice a week. Weekly glucose, phosphate, magnesium, LFTs, Ca, albumin,
FBC, MCV
levels if stable. 2-4 weekly Zn, Folate, B12 and Cu levels if stable. 3-6 monthly iron and ferritin levels, manganese (if on home
parenteral regime). 6 monthly vitamin D. Bone densitometry initially on starting home parenteral nutrition then every 2 years

35. Which of the following statements relating to use of tourniquets in surgery is false?
A. The use of an esmarch bandage tourniquet to exsanguinate the limb reduces the incidence of neuropraxia.
B. Excessive inflation pressures are amongst the commonest causes of nerve injury related to tourniquet use.
C. Tourniquet deflation causes a fall in CVP.
D. Children require lower inflation pressures than adults.
E. In patients developing neuropraxia related to tourniquets the radial nerve is most frequently affected.
Answer: A
The use of esmarch bandage tourniquet increases the risk of nerve injury as it increases pressure in the limb. Limb elevation is
safer.
Tourniquets: These may be applied to reduce blood loss during a procedure or to prevent bleeding obscuring vital structures. As
a rule they should not be used to control traumatic bleeding. Direct pressure is the preferred method.

Side effects/ complications: Skin friction injuries. Neuropraxia (greatest risk in upper limb, usually radial nerve). Direct injury to
underlying muscle. Cardiovascular changes due to limb exsanguination using Esmarch bandage, usually increased circulating
blood volume-may cause problems in patients with pre-existing vascular disease.

578
36. In relation to patients with type 1 diabetes mellitus undergoing surgery, which of the following statements is untrue?
A. They should not receive oral carbohydrate loading drinks as part of enhanced recovery programmes
B. When a variable rate insulin infusion is required 0.45% sodium chloride with either 0.15% or 0.3% potassium are the fluids of
choice
C. Hourly intraoperative blood glucose measurements are required
D. Insulin infusions are only required in patients who will miss more than two meals or who are nil by mouth for greater than 12
hours
E. Blood glucose levels persistently greater than 12 should initiate a change in therapy
Answer: D
Insulin should not be stopped in patients with type 1 diabetes and omission of more than one meal will usually require a variable
rate insulin infusion. Type 1 diabetics who take insulin should have this continued through the perioperative period.

37. A 72 year old man is recovering from an inguinal hernia repair when he suffers from an extensive CVA. He is managed
on the rehabilitation unit. However, he is still not able to feed safely and repeated swallowing assessments have shown
that he tends to aspirate. Which of the following is the best option for long term feeding?
A. PEG tube feeding
B. Feeding jejunostomy
C. Total parenteral nutrition
D. Long term naso gastric tube feeding
E. Withold feeding and palliate
Answer: A
A PEG tube is the best long term option although they are associated with a significant degree of morbidity. A feeding
jejunostomy would require a general anaesthetic. TPN is not a good option. Long term naso gastric feeding is usually
unsatisfactory.

Enteral Feeding: Identify patients as malnourished or at risk (see below). Identify unsafe or inadequate oral intake with
functional GI tract. Consider for enteral feeding. Gastric feeding unless upper GI dysfunction (then for duodenal or jejunal tube).
Check NG placement using aspiration and pH (check post pyloric tubes with AXR). Gastric feeding > 4 weeks consider long-term
gastrostomy.

Consider bolus or continuous feeding into the stomach. ITU patients should have continuous feeding for 16-24h (24h if on
insulin). Consider motility agent in ITU or acute patients for delayed gastric emptying. If this doesn't work then try post pyloric
feeding or parenteral feeding. PEG can be used 4h after insertion, but should not be removed until >2 weeks after insertion.

Surgical patients due to have major abdominal surgery: if malnourished, unsafe swallow/inadequate oral intake and functional GI
tract then consider pre operative enteral feeding.

Patients identified as being malnourished: BMI < 18.5 kg/m2. unintentional weight loss of > 10% over 3-6/12. BMI < 20 kg/m2
and unintentional weight loss of > 5% over 3-6/12

AT RISK of malnutrition: Eaten nothing or little > 5 days, who are likely to eat little for a further 5 days. Poor absorptive
capacity. High nutrient losses. High metabolism

Theme: Anaesthetic agents

A. Halothane
B. Propofol
C. Ketamine
D. Etomidate
E. Sodium thiopentone
F. Flumazenil
G. Naloxone
H. Sevoflurane

Please select the drug which most closely matches the description given. Each option may be used once, more than once or not at
all.

38. An agent which reverses the action of midazolam

The correct answer is Flumazenil. Flumazenil antagonises the effects of benzodiazepines by competition at GABA binding sites.
Since may benzodiazepines have longer half lives than flumazenil patients still require close monitoring after receiving the drug.

579
39. An agent which is associated with hepatotoxicity

Halothane. Halothane is hepatotoxic. Despite this it remains in mainstream use. It should be avoided in patients with hepatic
dysfunction, and scavengers should be used in theatres as accumulation of the drug may be injurious to theatre staff.

40. An anaesthetic agent which has anti emetic properties

Propofol. Propofol is rapidly metabolised and has mild/ moderate anti emetic properties. It is the agent of choice in most day case
operations for this r (rest of text was not found)

41. A 57 year old man is coming off the cardiac bypass circuit following a successful coronary artery bypass procedure.
Which drug should be administered to normalise the patients clotting prior to decannulation and chest closure?
A. Intravenous vitamin K
B. Protamine sulphate
C. Aprotinin
D. Fresh frozen plasma
E. None of the above
Answer: B
Since cardiac bypass circuits are thrombogenic large doses of intravenous heparin are administered. This is reversed with
protamine sulphate. FFP may be effective but would carry a significant risk of fluid overload.

42. A 34 year old man is suffering from septic shock and receives and infusion of Dextran 70. Which of the following
complications may potentially ensue?
A. Anaphylaxis
B. Vomiting
C. Acute hepatic failure
D. Digital necrosis
E. Deep vein thrombosis
Answer: A
Dextran 40 and 70 have higher incidence of anaphylaxis than either gelatins or starches.

Dextrans are branched polysaccharide molecules. Dextran 40 and 70 are available. The higher molecular weight dextran 70 may
persist for up to 8 hours. They inhibit platelet aggregation and leucocyte plugging in the microcirculation. Thereby improving
flow through the microcirculation, primarily of use in sepsis.
Unlike many other intravenous fluids Dextrans are a recognised cause of anaphylaxis.

Post operative fluid management: Composition of commonly used intravenous fluids mmol-1
Na K Cl Bicarbonate Lactate
Plasma 137-147 4-5.5 95-105 22-25 -
0.9% Saline 153 - 153 - -
Dextrose / saline 30.6 - 30.6 - -
Hartmans 130 4 110 - 28

A summary of the recommendations for post operative fluid management: Fluids given should be documented clearly and
easily available. Assess the patient's fluid status when they leave theatre. If a patient is haemodynamically stable and euvolaemic,
aim to restart oral fluid intake as soon as possible. Review patients whose urinary sodium is < 20. If a patient is oedematous,
hypovolaemia if present should be treated first. This should then be followed by a negative balance of sodium and water,
monitored using urine Na excretion levels.Solutions such as Dextran 70 should be used in caution in patients with sepsis as there
is a risk of developing acute renal injury.

43.In relation to operating in the elderly which statement is false?


A. A 30 minute increment in operation length is associated with increase in mortality in patients over the age of 80
B. Hypoalbuminaemia is associated with increased mortality
C. Statins given preoperatively reduce perioperative cardiac events
D. Elevated brain (or B-type) natriuretic peptide (BNP) levels before undergoing non cardiac surgery is associated with high risk of
cardiac mortality and all cause mortality
E. Beta blockers should be stopped acutely prior to surgery due to risk of perioperative hypotension
Answer: E
Beta blockers should not be stopped acutely prior to surgery as there may be a rebound effect associated with increased
complications.
580
Brain natriuretic peptide is a neurohormone synthesized in the cardiac ventricles. Levels have been used to assess prognosis in
heart failure and acute coronary syndromes. Preoperative elevated brain natriuretic peptide levels identify patients undergoing non
cardiac surgery at high risk of cardiac mortality and all cause mortality.

All patients with peripheral vascular disease should take statins prior to vascular surgery as studies have shown a 50% risk
reduction and a reduction in perioperative cardiac events.

Proactive care of older people undergoing surgery (POPS): Comprehensive geriatric assessment. MDT assessment
preoperatively. Main predictors of complications are co-morbidities cardiac disease and reduced functional capacity - preoperative
assessment is the key to preventing adverse postoperative outcomes. Patients screened for risk factors (albumin <30, co
morbidities). Management plan made and disseminated to all involved. Patients education: pain relief, post op exercises, nutrition

Outcomes: Fewer postoperative medical complications. Reduced length of stay by 4.5 days

44. Which statement is true regarding intra operative fluids?


B. Intra operative fluids reduce the length of hospital stay
C. IV dopamine should be given post operatively as part of enhanced recovery programmes
D. Intra operative fluids increase surgical complications
E. Fluid should routinely be given for two hours post operatively
F. Fluid should routinely be given for four hours post operatively
Answer: A
The use of dopexamine is advocated as part of the routine care of selected surgical patients. Dopamine is not part of the enhanced
recover process. The British IV Fluid guidelines references below strongly advocate the use of supplementary fluids during
surgery.

Intra operative fluid management: Composition of commonly used intravenous fluids mmol-1
Na K Cl Bicarbonate Lactate
Plasma 137-147 4-5.5 95-105 22-25 -
0.9% Saline 153 - 153 - -
Dextrose / saline 30.6 - 30.6 - -
Hartmans 130 4 110 - 28

Recommendations for intra operative fluid management: Intra operative fluids are recommended to optimise cardiac stroke
volume. Patients undergoing non elective orthopaedic or abdominal surgery should receive IV fluids for the 1st 8h post
operatively. This may be supplemented by a low dose dopexamine infusion in selected cases.

45. A 43 year old lady with a metallic heart valve has just undergone an elective paraumbilical hernia repair. In view of her
metallic valve she is given unfractionated heparin perioperatively. How should the therapeutic efficacy be monitored,
assuming her renal function is normal?
A. Therapeutic monitoring is not required
B. Measurement of APTT
C. Measurement of INR
D. Measurement of Prothromin time
E. None of the above
Answer: B
Unlike low molecular weight heparins that do not require monitoring unfractionated heparin does require monitoring, this is
done by measuring the APTT.

Theme: Pre operative preparation

A. Methylene Blue intravenously 1 hour pre-operatively


B. Lugol's iodine
C. 100ml single cream given 4 hours prior to surgery
D. Carbohydrate loading drink 2 hours prior to surgery
E. Picolax sachet
F. Fleet enema
G. Intravenous calcium chloride
H. 1mg lorazepam orally 30 minutes pre operatively
I. Patent blue dye intravenously

581
For each procedure please select the most appropriate procedure specific preparation required. Each option may be used once,
more than once or not at all.

46. A 45 year old man is due to undergo an Ivor Lewis oesophagectomy for a carcinoma of the distal oesophagus.

The correct answer is 100ml single cream given 4 hours prior to surgery. This will facilitate identification of the thoracic duct if
it is inadvertently divided during the operation.

47. A 32 year old man is due to undergo a right hemicolectomy for a large caecal sessile polyp.

The correct answer is Carbohydrate loading drink 2 hours prior to surgery. This is now a standard feature of colonic enhanced
recovery programmes.The administration of carbohydrate rich loading drinks results in lower incidence of ileus. The drink is
usually administered 2 hours pre-operatively and is rapidly absorbed from the GI tract.

48. A 67 year old women is due to undergo a parathyroidectomy for a parathyroid adenoma.

The correct answer is Methylene Blue intravenously 1 hour pre-operatively. Though not universally adopted, many endocrine
surgeons will administer methylene blue as it will facilitate identification of the parathyroid glands.

Theme: Local anaesthetics

A. 1% xylocaine with 1 in 200,000 adrenaline


B. 1% Lignocaine
C. 0.5% Bupivicaine with 1 in 200,000 adrenaline
D. 0.5% Bupivicaine
E. Prilocaine 1%
F. Procaine 1%
G. Cocaine 4%
H. Cocaine 10%

Please select the local anaesthetic formulation most appropriate to the procedure indicated. Each option may be used once, more
than once or not at all.

49. A 28 year old man has a sebaceous cyst of the scalp that requires excision.

1% xylocaine with 1 in 200,000 adrenaline. As scalp wounds can bleed profusely an adrenaline containing solution is preferred.
Xylocaine is similar to lignocaine in its onset and duration of action.

50. A 32 year old man has an appendicectomy performed through a Lanz incision, which anaesthetic would you infiltrate
the wound with to provide post operative analgesia.

The correct answer is 0.5% Bupivicaine. A long acting local anaesthetic is preferred.There is little advantage to adding a short
acting local anaesthetic agent since by the time the patient has recovered following surgery the bupivicaine will be active.

51. A 43 year old man is due to undergo a vasectomy.

The correct answer is 1% Lignocaine. Plain lignocaine will suffice. This will give rapid onset of action. Bupivicaine will take
too long to take effect. There would be little additional benefit derived for adding adrenaline. Use of adrenaline with local
anaesthetic agents prolongs duration of actions and allows administration of larger doses.

Theme: Acid - base disorders

A. Respiratory alkalosis
B. Type 1 respiratory failure
C. Type 2 respiratory failure
D. Metabolic alkalosis
E. Metabolic acidosis with normal anion gap
F. Metabolic acidosis with increased anion gap

582
Please match the condition with the blood gas result. Each option may be used once more than once or not at all.

52. pH 7.48, pO2 10.1, Bicarbonate 30, pCO2 4.5, Chloride<10meq

Metabolic alkalosis. This would be typical result of prolonged vomiting.

53. pH 7.49, pO2 7.1, pCO2 2.4, Bicarbonate 22, Chloride 12meq

Respiratory alkalosis. The hyperventilation results in decreased carbon dioxide levels, causing a respiratory alkalosis (non
compensated).

54. pH 7.20, pO2 7.5, Bicarbonate 22, pCO2 8.1, Chloride 10

The correct answer is Type 2 respiratory failure.This is a sign of acute type 2 respiratory failure (non compensated). This is the
result of carbon dioxide retention.

Disorders of acid - base balance are often covered in the MRCS part A, both in the SBA and EMQ sections.
Metabolic acidosis: This is the most common surgical acid - base disorder. Reduction in plasma bicarbonate levels.

Two mechanisms: 1. Gain of strong acid (e.g. diabetic ketoacidosis). 2. Loss of base (e.g. from bowel in diarrhoea)
- Classified according to the anion gap, this can be calculated by: (Na+ + K+) - (Cl- + HCO3-).
- If a question supplies the chloride level then this is often a clue that the anion gap should be calculated. The normal range = 10-
18 mmol/L

Normal anion gap ( = hyperchloraemic metabolic acidosis): Gastrointestinal bicarbonate loss: diarrhoea,
ureterosigmoidostomy, fistula. Renal tubular acidosis. Drugs: e.g. acetazolamide. Ammonium chloride injection. Addison's
disease

Raised anion gap: Lactate: shock, hypoxia. Ketones: diabetic ketoacidosis, alcohol. Urate: renal failure. Acid poisoning:
salicylates, methanol. Metabolic acidosis secondary to high lactate levels may be subdivided into two types: Lactic acidosis type
A: (Perfusion disorders e.g.shock, hypoxia, burns). Lactic acidosis type B: (Metabolic e.g. metformin toxicity)

Metabolic alkalosis: Usually caused by a rise in plasma bicarbonate levels. Rise of bicarbonate above 24 mmol/L will typically
result in renal excretion of excess bicarbonate. Caused by a loss of hydrogen ions or a gain of bicarbonate. It is due mainly to
problems of the kidney or gastrointestinal tract

Causes: Vomiting / aspiration (e.g. Peptic ulcer leading to pyloric stenosis, nasogastric suction). Diuretics. Liquorice,
carbenoxolone. Hypokalaemia. Primary hyperaldosteronism. Cushing's syndrome. Bartter's syndrome. Congenital adrenal
hyperplasia

Mechanism of metabolic alkalosis: Activation of renin-angiotensin II-aldosterone (RAA) system is a key factor. Aldosterone
causes reabsorption of Na+ in exchange for H+ in the distal convoluted tubule. ECF depletion (vomiting, diuretics) --> Na+ and Cl-
loss --> activation of RAA system --> raised aldosterone levels. In hypokalaemia, K+ shift from cells --> ECF, alkalosis is caused
by shift of H+ into cells to maintain neutrality

Respiratory acidosis: Rise in carbon dioxide levels usually as a result of alveolar hypoventilation. Renal compensation may
occur leading to Compensated respiratory acidosis

Causes: COPD. Decompensation in other respiratory conditions e.g. Life-threatening asthma / pulmonary oedema. Sedative
drugs: benzodiazepines, opiate overdose

Respiratory alkalosis: Hyperventilation resulting in excess loss of carbon dioxide. This will result in increasing pH

Causes: Psychogenic: anxiety leading to hyperventilation. Hypoxia causing a subsequent hyperventilation: pulmonary embolism,
high altitude. Early salicylate poisoning*. CNS stimulation: stroke, subarachnoid haemorrhage, encephalitis. Pregnancy

*Salicylate overdose leads to a mixed respiratory alkalosis and metabolic acidosis. Early stimulation of the respiratory centre leads

583
to a respiratory alkalosis whilst later the direct acid effects of salicylates (combined with acute renal failure) may lead to an
acidosis

Theme: Thromboprophylaxis

A. Oral dabigatran alone


B. Oral dabigatran with compression stockings
C. Low molecular weight heparin and compression stockings
D. Warfarin
E. Low molecular weight heparin and pneumatic compression stockings
F. Low molecular weight heparin alone
G. No thromboprophylaxis
H. Unfractionated heparin and compression stockings
I. Unfractionated heparin alone
J. Unfractionated heparin and pneumatic compression stockings

Please select the most appropriate thromboprophylactic regime in the surgical scenarios described below. Each regime may be
used once, more than once or not at all.

55. A 30 year old male is admitted electively for a right inguinal hernia repair under local anaesthesia. He is otherwise well
but his grandfather died from a pulmonary embolism.

No thromboprophylaxis. Inguinal hernia repairs under local anaesthetic have a short operative time and patients are usually
ambulant immediately afterwards. His family history is unlikely to be significant and he is at very low risk.

56. A 63 year old female is admitted for an open cholecystectomy for recurrent biliary colic and cholecystitis. She has
chronic kidney disease (stage 3) but no other co-mobidities.

The correct answer is Unfractionated heparin and compression stockings. Low molecular weight heparin is contra indicated in
chronic renal impairment. Oral dabigatran is not licensed for use following abdominal surgery. Unfractionated heparin is the
safest option (a lower dosing regime is often used).

57. An 83 year old man is admitted for an abdomino-perineal excision of the colon and rectum for a distal rectal tumour.
His co-mobidities include diabetes and intermittent claudication. His renal function is normal.

The correct answer is Low molecular weight heparin and pneumatic compression stockings. Pelvic cancer surgery carries a very
high risk of development of deep vein thrombosis. In a patient with normal renal function the use of a low molecular weight
heparin is standard. However, many surgeons would only use this in the post operative setting. Intermittent compression devices
in claudicants are not without risk, but on balence probably outweight the risk of DVT in this specific case. The perfusion of the
feet should be closely monitored and compression stopped if concerns develop.

Thromboprophylaxis in surgical patients: Deep vein thrombosis may develop insidiously in many surgical patients. Untreated it
may progress to result in pulmonary embolism.
The following surgical patients are at increased risk of deep vein thrombosis:Surgery greater than 90 minutes at any site or greater
than 60 minutes if the procedure involves the lower limbs or pelvis. Acute admissions with inflammatory process involving the
abdominal cavity. Expected significant reduction in mobility. Age over 60 years. Known malignancy. Thrombophilia. Previous
thrombosis. BMI >30. Taking hormone replacement therapy or the contraceptive pill. Varicose veins with phlebitis

Mechanical thromboprophylaxis: Early ambulation after surgery is cheap and is effective. Compression stockings (contra -
indicated in peripheral arterial disease). Intermittent pneumatic compression devices. Foot impulse devices

Therapeutic agents
Agent Mode of action Uses
Low molecular Binds antithrombin In patients with normal renal function, low doses typically given in those with
weight heparin resulting in inhibition of moderate to high risk of thromboembolic events. It is given as once daily
factor Xa subcutaneous injection
Unfractionated Binds antithrombin III Effective anticoagulation, administered intravenously it has a rapid onset and
heparin with affects thrombin and its therapeutic effects decline quickly on stopping and infusion. Its activity is
584
factor Xa measured using the APTT. If need be it can be reversed using protamine
sulphate
Dabigatran Orally administered Used prophylaxis in hip and knee surgery. It does not require therapeutic
direct thrombin inhibitor monitoring. It has no known antidote and should not be used in any patient in
whom there is a risk of active bleeding or imminent likelihood of surgery

Theme: Management of complications

A. Gastrograffin contrast enema


B. Barium enema
C. Oral gastrograffin and CT
D. Barium meal
E. Ultrasound of the thorax
F. Endoanal ultrasound
G. Anorectal physiology studies
H. Biofeedback

Please select the most appropriate intervention from the list given. Each option may be used once, more than once or not at all.

58. A 65 year old male with carcinoma of the oesophagus undergoes endoscopic dilatation. Following which he develops
pleuritic chest pain and sub cutaneous emphysema.

Oral gastrograffin and CT. This is consistent with oesophageal perforation. Gastrograffin and CT will accurately delineate the
site of perforation and guide further therapy. Barium may produce a mediastinitis and should not be used.

59. A 73 year old lady is admitted with large bowel obstruction due to carcinoma of the rectum. She is undergoes a
laparoscopic defunctioning of the rectum with an end colostomy. 24 hours later her stoma has still not worked.

The correct answer is Gastrograffin contrast enema. Occasionally the wrong end of bowel is brought up and fashioned as the end
stoma, effectively leaving the bowel obstructed. A gastrograffin enema will easily demonstrate if this is the case.

60. A 43 year old man develops fast atrial fibrillation 5 days following a low anterior resection of the rectum for cancer. On
examination he has lower abdominal tenderness and a WCC 19.

Gastrograffin contrast enema. An anastomotic leak is the most likely occurrence and will be demonstrated using gastrograffin.
Barium should not be used for this.

61. A 63 year old man is recovering following an open extended right hemicolectomy for carcinoma of the colonic splenic
flexure. Two days post operatively he develops a persistent pyrexia. What is the least likely cause?
A. Ileus
B. Atelectasis
C. Anastomotic leak
D. Wound infection
E. Urinary tract infection
Answer: A
An ileus in itself is seldom a cause of a pyrexia. It may serve as a proxy marker of other complications. In this scenario atelectasis
would be the most likely underlying cause, as open extended right hemicolectomies will necessitate a long midline incision.
Anastomotic leaks are less common after right sided colonic surgery and the timeframe for it is rather short (but are possible).
Both wound infections and UTI's ,may complicate major abdominal surgery at any stage.

Pyrexia- post operative: Many surgical patients will develop a pyrexia post operatively. The cause and investigation depends
upon the nature of the infection.
The following scenarios may account for post operative pyrexia:

Anastomotic leak: Swinging pyrexia. Ileus. Increasing abdominal pain. Raised inflammatory markers

Wound infection: Evidence of superficial erythema, discharge of pus or increasing pain. Usually mild pyrexia (unless major or
deep seated wound infection). May be accompanied by evidence of wound dehiscence. Inflammatory markers raised

Atelectasis: Usually complicates abdominal surgery. Most common after midline laparotomies (pain impairs ventilation). Pyrexia
usually mild and non swinging. Most patients will have chest signs on examination. Inflammatory markers raised

585
Central line sepsis: Patients with complex venous access. May have marked pyrexia. Access site may show evidence of
erythema. Diagnosis is by blood culture from line, line removal and subsequent tip culture. Groin lines and those for TPN have the
highest risk. Inflammatory markers raised

Urinary tract infection: Common in surgical patients. Usually occur in patients with indwelling urinary catheters. Diagnosis is
by dipstick and CSU and signs of raised inflammatory markers. Treatment is with antibiotics (to cover hospital acquired
organisms)

62. A 72 year old man is due to undergo an oesophagectomy for malignancy. His BMI is 17.5. What is the best feeding
regime immediately following surgery?
A. Total parenteral nutrition.
B. Feeding jejunostomy.
C. Feeding duodenostomy.
D. Liquid diet orally.
E. Soft solids orally.
Answer: B
This patient has a condition causing poor absorption, loss of nutrients and high metabolism. Enteral feeds should be used where
possible and many surgeons will site a jejunostomy for this purpose. Oral diet is not permitted following a resection until the
anastamosis has had time to heal. Theme from April 2012 Exam

Theme: Bowel preparation

A. No preparation required
B. Phosphate enema
C. Mechanical bowel preparation with oral laxatives (e.g. picolax)
D. Senokot tablets
E. Oral pergolide
F. Rectal lavage with saline
G. 60 ml oral lactulose

Please select the most appropriate form of bowel preparation for the procedures given. Each agent may be selected once, more
than once or not at all.

63. A 56 year old man with carcinoma of the rectum requires a low anterior resection.

The correct answer is Mechanical bowel preparation with oral laxatives (e.g. picolax). Although some enhanced recovery
programmes may advocate no preparation, most surgeons would undertake a defunctioning ileostomy for a low anterior
resection and thus give full prep. A Cochrane review has failed to generate sufficient evidence to guide bowel preparation for
rectal surgery. Many surgeons do not routinely undertake mechanical bowel preparation for colonic resections above the
peritoneal reflection.

64. A 44 year old man with carcinoma of the hepatic flexure requires a right hemicolectomy.

The correct answer is No preparation required. Formal bowel preparation for right sided colonic resection is unnecessary. The
formal bowel preparation of elective patients for right sided resection results in increased post operative morbidity and delayed
discharge.

65. A 34 year old colitic with acute colitis requires a flexible sigmoidoscopy.

The correct answer is Phosphate enema.For a limited endoscopy a simple enema will suffice. Few acute colitics would tolerate
formal oral prep. A Cochrane review evaluating the role of mechanical bowel preparation in colonic surgery has shown no
increase in adverse events in resections above the peritoneal reflection. At the present time there is insufficient evidence to
guide surgeons in administering bowel preparation for rectal surgery. The concept of omitting mechanical bowel preparation
and then defunctioning a low anterior resection would seem counter intuitive as the role of loop ileostomy is to reduce the
clinical severity of an anastamotic leak. A benefit that is attenuated by not administering bowel preparation.

66. The following are contra indications to the use of lignocaine for local anaesthesia except:
A. Accelerated idioventricular rhythm
B. Current treatment with flecainide
C. 3rd degree heart block without pacemaker
D. Severe sino atrial block

586
E. Protein C deficiency
Answer: E
Lignocaine is widely used as a local anaesthetic. As a class IB antiarrhythmic it should not be used in people with unstable
disorders of cardiac rhythm and ideally should not be co-administered with other anti-arhythmics.

67. A 43 year old lady has undergone a total thyroidectomy for multinodular goitre. You are called to see her because of
respiratory distress. On examination she has a marked stridor, her wound seems healthy but there is a swelling within
the operative site. Which of the following accounts for this problem?
A. Bilateral superior laryngeal nerve injury
B. Hypocalcaemic tetany
C. Anxiety
D. Contained haematoma
E. Unilateral recurrent laryngeal nerve injury
Answer: D
In this setting a contained haematoma is the most likely cause. This will impair venous return resulting in laryngeal oedema and
respiratory compromise.

Thyroid disease: Patients may present with a number of different manifestations of thyroid disease. They can be broadly sub
classified according to whether they are euthyroid or have clinical signs of thyroid dysfunction. In addition it needs to be
established whether they have a mass or not.

Assessment: History. Examination including USS. If a nodule is identified then it should be sampled ideally via an image guided
fine needle aspiration. Radionucleotide scanning is of limited use

Thyroid Tumours: Papillary carcinoma. Follicular carcinoma. Anaplastic carcinoma. Medullary carcinoma. Lymphoma's

Multinodular goiter: One of the most common reasons for presentation. Provided the patient is euthyroid and asymptomatic and
no discrete nodules are seen, they can be reassured. In those with compressive symptoms surgery is required and the best
operation is a total thyroidectomy. Sub total resections were practised in the past and simply result in recurrent disease that
requires a difficult revisional resection.

Endocrine dysfunction: In general these patients are managed by physicians initially. Surgery may be offered alongside radio
iodine for patients with Graves disease that fails with medical management or in patients who would prefer not to be irradiated
(e.g. pregnant women). Patients with hypothyroidism do not generally get offered a thyroidectomy. Sometimes people
inadvertently get offered resections during the early phase of Hashimotos thyroiditis, however, with time the toxic phase passes
and patients can simply be managed with thyroxine.

Complications following surgery: Anatomical such as recurrent laryngeal nerve damage.Bleeding. Owing to the confined space
haematoma's may rapidly lead to respiratory compromise owing to laryngeal oedema. Damage to the parathyroid glands resulting
in hypocalcaemia.

68. Which of the following drugs is not positively inotropic?


A. Dopamine
B. Glucagon
C. Theophylline
D. Sodium thiopentone
E. Dobutamine
Answer: D
Inotropes are a class of drugs that increase the force or cardiac contractility. This may improve cardiac output. Increased blood
pressure may have direct beneficial effects for the heart in that it improves myocardial perfusion pressure. Dopamine and
dobutamine are both commonly used inotropes, they should be administered via a central line and in a monitored setting.
Glucagon and theophylline are also positive inotropes (although not commonly used for this purpose). In contrast sodium
thiopentone causes marked myocardial depression.

Effects of receptor binding Inotropes and cardiovascular receptors


α-1, α-2 vasoconstriction Inotrope Cardiovascular receptor action
β-1 increased cardiac contractility and HR Adrenaline α-1, α-2, β-1, β-2
β-2 vasodilatation Noradrenaline α-1,( α-2), (β-1), (β-2)
D-1 renal and spleen vasodilatation Dobutamine β-1, (β 2)
D-2 inhibits release of noradrenaline Dopamine (α-1), (α-2), (β-1), D-1,D-2
Minor receptor effects in brackets

587
69. A 28 year old man undergoes a laparotomy for perforated duodenal ulcer and broad spectrum antibiotics are
administered. Post operatively he has hearing impairment. Which of the following agents is the most likely underlying
culprit?
A. Gentamicin
B. Ciprofloxacin
C. Metronidazole
D. Ampicillin
E. Co-trimoxazol
Answer: A
Ototoxicity is a recognised adverse reaction with the aminoglycoside antibiotics.

Antibiotics: mechanism of action: The lists below summarise the site of action of the commonly used antibiotics

Inhibit cell wall formation: penicillins. cephalosporins

Inhibit protein synthesis: Aminoglycosides (cause misreading of mrna). Chloramphenicol. Macrolides (e.g. Erythromycin).
Tetracyclines. Fusidic acid

Inhibit DNA synthesis: quinolones (e.g. ciprofloxacin). Metronidazole. Sulphonamides. Trimethoprim

Inhibit RNA synthesis: rifampicin

70. A 62 year old lawyer has a transurethral resection which took 1h to perform. The ST2 contacts you as the patient has
become agitated. He has a HR 105 bpm and his blood pressure is 170/100 mmHg. He is fluid overloaded. His blood
results reveal a Na of 120mmol/l. What is the most likely cause?
A. Over administration of 0.9% Normal Saline
B. Syndrome of inappropriate antidiuretic hormone secretion
C. Congestive cardiac failure
D. TUR syndrome
E. Acute renal failure
Ansewr: D
Complications of Transurethral Resection: TURP: T ur syndrome. U rethral stricture/UTI. R etrograde ejaculation. P erforation of
the prostate

TUR syndrome occurs when irrigation fluid enters the systemic circulation. The triad of features are: 1. Hyponatraemia:
dilutional. 2. Fluid overload. 3. Glycine toxicity
Management involves fluid restriction and the treatment of the complications associated with the hyponatraemia.

Post prostatectomy syndromes: Transurethral prostatectomy is a common and popular treatment for benign prostatic
hyperplasia. The procedure involves insertion of a resectoscope via the penile urethra. The bladder and prostate are irrigated and
strips of prostatic tissue removed using diathermy.

Complications include haemorrhage, urosepsis, retrograde ejaculation and electrolyte disturbances from the irrigation fluids used
during surgery.

Hand Lesions
Theme: Hand disorders

A. de Quervain's tenosynovitis
B. Dupuytren's contracture
C. Bouchard's nodes
D. Ganglion
E. Carpal tunnel syndrome
F. Radial nerve injury
G. Ulnar nerve injury
H. Heberden's nodes
I. Tendon sheath infection

Please select the most likely diagnosis to account for the scenario given. Each option may be used once, more than once or not at
all.
588
1. A 49 -year-old male presents with discomfort in the fingers of his left hand. On examination the ring and little fingers of
his left hand are flexed and unable to extend completely. He is able to make a fist with the hand. Palpation reveals
thickened nodules on the medial half of the palm.

Dupuytren's contracture. Discomfort of the hand is not uncommon in Dupuytrens contracture, true pain is unusual. The disease
most commonly affects the ring and little fingers.

2. A 62 year old man presents after his wife commented on the unusual shape of his fingers. On examination he has a hard
swelling adjacent to the distal interphalangeal joint of his right hand with lateral deviation of the finger tip. There is no
sensory disturbance and the swelling is not tender

Heberden's nodes. These are bony outgrowths that occur in the distal interphalangeal joint in association with osteoarthritis.
They may skew the finger tip sideways. Bouchards nodes are similar but occur in association with the proximal interphalangeal
joint.

3. A 57 year - old lady presents with a three month history of pins and needles in the fingers of the right hand, particularly
at night. On examination, there is some loss of the sensation over the palmar aspect of the lateral three fingers and
wasting of the thenar eminence.

Carpal tunnel syndrome. Carpal tunnel syndrome commonly produces pain at night as the wrists are flexed during sleep.
Compromise of the median nerve may produce wasting of the thenar eminence muscles.

Hand diseases: Dupuytrens contracture: Fixed flexion contracture of the hand where the fingers bend towards the palm and
cannot be fully extended. Caused by underlying contractures of the palmar aponeurosis . The ring finger and little finger are the
fingers most commonly affected. The middle finger may be affected in advanced cases, but the index finger and the thumb are
nearly always spared. Progresses slowly and is usually painless. In patients with this condition, the tissues under the skin on the
palm of the hand thicken and shorten so that the tendons connected to the fingers cannot move freely. The palmar aponeurosis
becomes hyperplastic and undergoes contracture. Commonest in males over 40 years of age. Association with liver cirrhosis and
alcoholism. However, many cases are idiopathic. Treatment is surgical and involves fasciectomy. However, the condition may
recur and many surgical therapies are associated with risk of neurovascular damage to the digital nerves and arteries.

Carpel tunnel syndrome: Idiopathic median neuropathy at the carpal tunnel. Characterised by altered sensation of the lateral 3
fingers. The condition is commoner in females and is associated with other connective tissue disorders such as rheumatoid
disease. It may also occur following trauma to the distal radius. Symptoms occur mainly at night in early stages of the condition.
Examination may demonstrate wasting of the muscles of the thenar eminence and symptoms may be reproduced by Tinels test
(compression of the contents of the carpal tunnel). Formal diagnosis is usually made by electrophysiological studies. Treatment is
by surgical decompression of the carpal tunnel, a procedure achieved by division of the flexor retinaculum. Non - surgical options
include splinting and bracing.

Miscellaneous hand lumps


Osler's Osler's nodes are painful, red, raised lesions found on the hands and feet. They are the result of the deposition of
nodes immune complexes.
Bouchards Hard, bony outgrowths or gelatinous cysts on the proximal interphalangeal joints (the middle joints of fingers or
nodes toes.) They are a sign of osteoarthritis, and are caused by formation of calcific spurs of the articular cartilage.
Heberdens Typically develop in middle age, beginning either with a chronic swelling of the affected joints or the sudden
nodes painful onset of redness, numbness, and loss of manual dexterity. This initial inflammation and pain eventually
subsides, and the patient is left with a permanent bony outgrowth that often skews the fingertip sideways.
Ganglion Swelling in association with a tendon sheath commonly near a joint. They are common lesions in the wrist and
hand. Usually they are asymptomatic and cause little in the way of functional compromise. They are fluid filled
although the fluid is similar to synovial fluid it is slightly more viscous. When the cysts are troublesome they may
be excised.
Theme: Hand disorders

A. Malignant fibrous histiocytoma


B. Oslers nodes
C. Heberdens nodes
D. Bouchards nodes
E. Carpal tunnel syndrome
F. Complex regional pain syndrome
G. Osteoclastoma
H. Osteosarcoma
I. Ganglion
589
Please select the most likely diagnosis for the lesion described. Each option may be used once, more than once or not at all.

4. A 42 year old lady who has systemic lupus erythematosus presents to the clinic with a 5 day history of a painful purple
lesion on her index finger. On examination she has a tender red lesion on the index finger.

Oslers nodes. Osler nodes are normally described as tender, purple/red raised lesions with a pale centre. These lesions occur as a
result of immune complex deposition. These occur most often in association with endocarditis. However, other causes include
SLE, gonorrhoea, typhoid and haemolytic anaemia.

5. A 62 year old lady presents with an non tender lump overlying the distal interphalangeal joint of the index finger. On
examination she has a hard, non tender lump overlying the joint and deviation of the tip of the finger.

Heberdens nodes. Heberdens nodes may produce swelling of the distal interphalangeal joint with deviation of the finger tip.

6. A 17 year old body is brought to the clinic by his mother who is concerned about a lesion that has developed on the
dorsal surface of his left hand. On examination he has a soft fluctuant swelling on the dorsal aspect of the hand, it is most
obvious on making a fist.

Ganglion.Ganglions commonly occur in the hand and are usually associated with tendons. They are typically soft and fluctuant.
They do not require removal unless they are atypical or causing symptoms.

Oncology Principles
1. A 48 year old lady undergoes a laparotomy and a retroperitoneal tumour is identified. The surgeons suspect that the
lesion is a liposarcoma. Which of the following is not typical of liposarcomas?
A. They are the most common variant of sarcoma in adults
B. Core biopsies in low grade liposarcomas may be normal
C. May have a pseudocapsule
D. It is unlikely in a lesion measuring less than 5cm
E. Pulmonary metastasis are common
Answer: A
Malignant fibrous histiocytoma is the commonest variant of sarcoma and liposarcoma the second most common. The presence
of a pseudocapsule should be borne in mind when performing surgery, as incomplete removal will result in local recurrence.

Sarcomas: Malignant tumours of mesenchymal origin

Types May be either bone or soft tissue in origin.


Bone sarcoma include: Osteosarcoma. Ewings sarcoma (although non bony sites recognised). Chrondrosarcoma - originate from
Chondrocytes

Soft tissue sarcoma are a far more heterogeneous group and include: Liposarcoma-adipocytes. Rhabdomyosarcoma-striated
muscle. Leiomyosarcoma-smooth muscle. Synovial sarcomas- close to joints (cell of origin not known but not synovium)

Malignant fibrous histiocytoma is a sarcoma that may arise in both soft tissue and bone.

Features: Certain features of a mass or swelling should raise suspicion for a sarcoma these include: Large >5cm soft tissue mass.
Deep tissue location or intra muscular location. Rapid growth. Painful lump

Assessment: Imaging of suspicious masses should utilise a combination of MRI, CT and USS. Blind biopsy should not be
performed prior to imaging and where required should be done in such a way that the biopsy tract can be subsequently included in
any resection.

Ewings sarcoma: Commoner in males. Incidence of 0.3 / 1, 000, 000. Onset typically between 10 and 20 years of age. Location
by femoral diaphysis is commonest site. Histologically it is a small round tumour. Blood borne metastasis is common and
chemotherapy is often combined with surgery

Osteosarcoma: Mesenchymal cells with osteoblastic differentiation. 20% of all primary bone tumours. Incidence of 5 per
1,000,000. Peak age 15-30, commoner in males. Limb preserving surgery may be possible and many patients will receive
chemotherapy

590
Liposarcoma: Malignancy of adipocytes. Rare approximately 2.5 per 1,000,000. They are the second most common soft tissue
sarcoma. Typically located in deep locations such as retroperitoneum. Affect older age group usually >40 years of age. May be
well differentiated and thus slow growing although may undergo dedifferentiation and disease progression. Many tumours will
have a pseudocapsule that can misleadingly allow surgeons to feel that they can 'shell out' these lesions. In reality tumour may
invade at the edge of the pseudocapsule and result in local recurrence if this strategy is adopted. Usually resistant to radiotherapy
although this is often used in a palliative setting

Malignant Fibrous Histiocytoma: Tumour with large number of histiocytes. Most common sarcoma in adults. Also described as
undifferentiated pleomorphic sarcoma NOS (i.e. Cell of origin is not known). Four major subtypes are recognised: storiform-
pleomorphic (70% cases), myxoid (less aggressive), giant cell and inflammatory. Treatment is usually with surgical resection and
adjuvant radiotherapy as this reduces the likelihood of local recurrence

2. A 63 year old lady with metastatic breast cancer presents with bone pain. Radiological tests show a metastatic lytic
deposit to her femoral shaft. The lesion occupies 75% of the bone diameter. What is the most approprate management?
A. Surgical fixation with a dynamic compression plate
B. Hemi-arthroplasty
C. Fixation with intramedullary nail
D. Radial radiotherapy
E. Chemotherapy
Answer: C
Even with surgical fixation only 30% of pathological fractures unite. The type of fixation should be chosen accordingly.

A lesion of this nature is at high risk of spontaneous fracture. Whilst radiotherapy may palliate her symptoms of pain it will not
reduce the risk of fracture. In fit patients an intramedullary nail should be inserted. Very proximal lesions may be best managed by
a total hip replacement

Secondary malignant tumours of bone: Metastatic lesions affecting bone are more common than primary bone tumours.

The typical tumours that spread to bone include: Breast. Bronchus. Renal. Thyroid. Prostate. 75% cases will affect those over the
age of 50. The commonest bone sites affected are: Vertebrae (usually thoracic). Proximal femur. Ribs. Sternum. Pelvis. Skull.

Pathological fracture: Osteolytic lesions are the greatest risk for pathological fracture
The risk and load required to produce fracture varies according to bone site. Bones with lesions that occupy 50% or less will be
prone to fracture under loading (Harrington). When 75% of the bone is affected the process of torsion about a bony fulcrum may
produce a fracture.

The Mirel scoring[1] system may be used to help determine the risk of fracture and is more systematic than the Harrington system
described above.

Mirel Scoring system

Score points Site Radiographic appearance Width of bone involved Pain


1 Upper extremity Blastic Less than 1/3 Mild
2 Lower extremity Mixed 1/3 to 2/3 Moderate
3 Peritrochanteric Lytic More than 2/3 Aggravated by function

Depending upon the score the treatment should be as follows:

Score Risk of fracture Treatment


9 or greater Impending (33%) Prophylactic fixation
8 Borderline Consider fixation
7 or less Not impending (4%) Non operative management

Where the lesion is an isolated metastatic deposit consideration should be given to excision and reconstruction as the outcome is
better [2].

Non operative treatments: Hypercalcaemia- Treat with re hydration and bisphosphonates.


Pain- Opiate analgesics and radiotherapy.
Some tumours such as breast and prostate will benefit from chemotherapy and or hormonal agents.

591
3. A 50 year old lady presents with pain in her proximal femur. Imaging demonstrates a bone metastasis from an unknown
primary site. CT scanning with arterial phase contrast shows that the lesion is hypervascular. From which of the
following primary sites is the lesion most likely to have originated?
A. Breast
B. Renal
C. Bronchus
D. Thyroid
E. Colon
Answer: B
Renal metastases have a tendency to be hypervascular. This is of considerable importance if surgical fixation is planned.

4. Which of the following group of patients are not screened for colorectal cancer?
A. Peutz Jeghers syndrome
B. Asymptomatic patients aged 55 years
C. Acromegaly
D. Ureterosigmoidostomy
E. Inflammatory bowel disease
Answer: B
Other disorders which are screened for colorectal malignancy include:
Familial adenomatous polyposis, Hereditary non polyposis colorectal cancer. The NHS screening programme starts at 60.

Colorectal cancer screening and diagnosis: Overview: Most cancers develop from adenomatous polyps. Screening for
colorectal cancer has been shown to reduce mortality by 16%. The NHS now has a national screening programme offering
screening every 2 years to all men and women aged 60 to 69 years. Patients aged over 70 years may request screening. Eligible
patients are sent faecal occult blood (FOB) tests through the post. Patients with abnormal results are offered a colonoscopy

At colonoscopy, approximately: 5 out of 10 patients will have a normal exam. 4 out of 10 patients will be found to have polyps
which may be removed due to their premalignant potential. 1 out of 10 patients will be found to have cancer

Diagnosis: Essentially the following patients need referral: Altered bowel habit for more than six weeks. New onset of rectal
bleeding. Symptoms of tenesmus

Colonoscopy is the gold standard, provided it is complete and good mucosal visualisation is achieved. Other options include
double contrast barium enema and CT colonography.

Staging: Once a malignant diagnosis is made patients with colonic cancer will be staged using chest / abdomen and pelvic CT.
Patients with rectal cancer will also undergo evaluation of the mesorectum with pelvic MRI scanning.
For examination purposes the Dukes and TNM systems are preferred.

Tumour markers: Carcinoembryonic antigen (CEA) is the main tumour marker in colorectal cancer. Not all tumours secrete this,
and it may be raised in conditions such as IBD. However, absolute levels do correlate (roughly) with disease burden and whilst
this marker may not be used extensively in follow up, it can be useful for investigation of patients with cancer of unknown
primary.

5. A 56-year-old man with metastatic prostate cancer comes for review. He is known to have spinal metastases but until
now has not had any significant problems with pain control. Unfortunately he is now getting regular back pain despite
taking paracetamol 1g qds. Neurological examination is unremarkable. What is the most appropriate next step?
A. Switch to co-codamol 30/500
B. Refer for radiotherapy
C. Add oral bisphosphonate
D. Add diclofenac
E. Add dexamethasone
Answer: D
Metastatic bone pain may respond to NSAIDs, bisphosphonates or radiotherapy
Bone pain often responds well to NSAIDs. Both radiotherapy and bisphosphonates have a role in managing bony pain but these
are not first-line treatments.

Palliative care prescribing: pain: SIGN issued guidance on the control of pain in adults with cancer in 2008. Selected points: the
breakthrough dose of morphine is one-sixth the daily dose of morphine. All patients who receive opioids should be prescribed a
laxative. Opioids should be used with caution in patients with chronic kidney disease. Alfentanil, buprenorphine and fentanyl are
preferred. Metastatic bone pain may respond to NSAIDs, bisphosphonates or radiotherapy

When increasing the dose of opioids the next dose should be increased by 30-50%.

592
Conversion between opioids
From To
Oral codeine Oral morphine Divide by 10
Oral tramadol Oral morphine Divide by 5

From To
Oral morphine Oral oxycodone Divide by 2

The BNF states that oral morphine sulphate 80-90mg over 24 hours is approximately equivalent to one '25 mcg/hour' fentanyl
patch, therefore product literature should be consulted.

From To
Oral morphine Subcutaneous diamorphine Divide by 3
Oral oxycodone Subcutaneous diamorphine Divide by 1.5

6. A 62 year old male is found to have colorectal cancer. He has Dukes B disease. What is his 5 year prognosis?
A. 100%
B. 90%
C. 80%
D. 70%
E. 60%
Answer: D

Dukes classification: Gives the extent of spread of colorectal cancer


Dukes A Tumour confined to the mucosa (90%)
Dukes B Tumour invading bowel wall (70%)
Dukes C Lymph node metastases (45%)
Dukes D Distant metastases (6%)(20% if resectable)
5 year survival in brackets Theme from September 2011 Exam

7. In examining a biopsy of a primary tumour, the clearest evidence of malignancy is provided by:
A. Absence of a capsule
B. Basophilia of the cytoplasm
C. Invasion of surrounding structures
D. Excess of mitoses
E. Nuclear aberrations
Answer: C
Invasion is the hallmark of malignancy. The others may occur in insitu disease or dysplastic lesions.

Tissue sampling is an important surgical process. Biopsy modalities vary according to the site, experience and subsequent
planned therapeutic outcome: The modalities comprise: Fine needle aspiration cytology. Core biopsy. Excision biopsy. Tru cut
biopsy. Punch biopsy. Cytological smears. Endoscopic or laparoscopic biopsy

When the lesion is superficial the decision needs to be taken as to whether complete excision is desirable or whether excision
biopsy is acceptable. In malignant melanoma for example the need for safe margins will mean that a more radical surgical
approach needs to be adopted after diagnostic confirmation from excision biopsy than would be the case in basal cell carcinoma.
Punch biopsies are useful in gaining histological diagnosis of unclear skin lesions where excision biopsy is undesirable such as in
establishing whether a skin lesion is vasculitic or not.

Fine needle aspiration cytology (FNAC) is an operator dependent procedure that may or may not be image guided and essentially
involves passing a needle through a lesion whilst suction is applied to a syringe. The material thus obtained is expressed onto a
slide and sent for cytological assessment. This test can be limited by operator inexperience and also by the lack of histological
architectural information (e.g. Follicular carcinoma of the thyroid). Where a discharge is present a sample may be sent for
cytology although in some sites (e.g. Nipple discharge ) the information gleaned may be meaningless.

Tissue samples may be obtained by both core and tru cut biopsy. A core biopsy is obtained by use of a spring loaded gun with a
needle passing quickly through the lesion of interest. A tru cut biopsy achieves the same objective but the needle moved by hand.
When performing these techniques image guidance may be desirable (e.g. In breast lesions). Consideration needs to be given to
any planned surgical resection as it may be necessary to resect the biopsy tract along with the specimen (e.g. In sarcoma surgery).

593
Visceral lesions may be accessed percutaneously under image guidance such as ultrasound guided biopsy of liver metastases. Or
under direct vision such as a colonoscopic biopsy.

8. A 45 year old male is referred to clinic for consideration of resection of a lung malignancy. He reports shortness of
breath and haemoptysis. Investigations reveal a corrected calcium of 2.84 mmol/l, an FEV 1.9L and histology of a
squamous cell carcinoma. The patient is noted to have a hoarse voice. Which one of the following is a contraindication to
surgical resection in lung cancer?
A. Haemoptysis
B. FEV 1.9 litres
C. Histology shows squamous cell cancer
D. Vocal cord paralysis
E. Calcium = 2.84 mmol/L
Answer: D
Contraindications to lung cancer surgery include SVC obstruction, FEV < 1.5, MALIGNANT pleural effusion, and vocal cord
paralysis.Paralysis of a vocal cord implies extracapsular spread to mediastinal nodes and is an indication of inoperability.

Lung cancer: non-small cell management: Only 20% suitable for surgery. Mediastinoscopy performed prior to surgery as CT
does not always show mediastinal lymph node involvement. Curative or palliative radiotherapy. Poor response to chemotherapy
Surgery contraindications: Assess general health. Stage IIIb or IV (i.e. metastases present). FEV1 < 1.5 litres is considered a
general cut-off point*. Malignant pleural effusion. Tumour near hilum. Vocal cord paralysis. SVC obstruction
* However if FEV1 < 1.5 for lobectomy or < 2.0 for pneumonectomy then some authorities advocate further lung function tests as
operations may still go ahead based on the results

9. A 43 year old lady is receiving chemotherapy for the treatment of metastatic breast cancer. You are called because it has
become apparent that her doxorubicin infusion has extravasated. What is the most appropriate course of action?
A. Stop the infusion and administer dexamethasone through the infusion device
B. Stop the infusion and administer hyaluronidase through the infusion device
C. Stop the infusion and apply a cold compress to the site
D. Stop the infusion and apply a warm compress to the site
E. Stop the infusion and administer sodium bicarbonate through the infusion device
Answer: C
The application of cold compresses is indicated in doxorubicin extravasation. Warm compresses increase the risk of doxorubicin
ulceration. Hyaluronidase is indicated in the extravasation of contrast media, TPN and vinca alkaloids. However, if administed
following doxorubicin extravasation it will dramatically worsen the situation and is contra indicated. Up to 50% of those
sustaining severe injuries will require delayed surgical reconstruction.

Extravasation injury: Chemotherapy may be complicated by extravasation reactions in up to 6% of cases. The following
chemotherapy agents are recognised causes of extravasation reactions; doxorubicin, vincristine, vinblastine, adriamycin, cisplatin,
mitomycin and mithramycin.
Up to 30% of extravasation reactions may be complicated by the development of ulceration.
When an extravasation reaction is suspected the infusion should be stopped and the infusing device aspirated. The extremity
should be elevated. As a general rule cold compresses have been shown to reduce the incidence of subsequent ulceration with
doxorubicin. Warm compresses have been found to be beneficial in extravasation of vinca alkaloids. Dimethylsulfoxide may be
infused in some cases, ideally within 5 hours of the event occurring. No conclusive evidence exists to support the use of
corticosteroids or sodium bicarbonate for extravasation injuries.

10. A 56 year old lady presents with a pathological fracture of the proximal femur. Which of the following primary sites is
the most likely source of her disease?
A. Thyroid
B. Breast
C. Kidney
D. Endometrium
E. None of the above
Answer: B
The correct answer is breast, because the question asks for the most likely primary site. Breast cancer is the commonest cause of
lytic bone metastasis in women of this age, especially from amongst those options given. Theme from September 2011 exam

11. A 67-year-old man with colorectal cancer is currently taking MST 30mg bd for pain relief. What dose of oral morphine
solution should he be prescribed for breakthrough pain?
A. 5 mg
B. 10 mg
594
C. 15 mg
D. 20 mg
E. 30 mg
Answer: B
Breakthrough dose = 1/6th of daily morphine dose. The total daily morphine dose is 30 * 2 = 60 mg, therefore the breakthrough
dose should be one-sixth of this, 10 mg.

595

Вам также может понравиться